Download as pdf or txt
Download as pdf or txt
You are on page 1of 1027

DAMS QUESTION BANK–II

DQB-II
PARACLINICALS
1. PATHOLOGY
2. PHARMACOLOGY
3. MICROBIOLOGY
4. FORENSIC MEDICINE &
TOXICOLOGY

For PG Medical Entrance


Published by Delhi Academy of Medical Sciences (P)
Ltd.

HEAD OFFICE
Delhi Academy of Medical Sciences (P.) Ltd.
4-B, Grovers Chamber, Pusa Road,
Near Karol Bagh Metro Station,
New Delhi-110 005
Phone : 011-4009 4009
http://www.damsdelhi.com
Email: info@damsdelhi.com

ISBN : 978-93-87503-75-5

First Published 1999, Delhi Academy of Medical Sciences

© 2019 DAMS Publications

All rights reserved. No part of this book may be reproduced


or transmitted in any form or by any means, electronic,
mechanical, including photocopying, recording, or any
information storage and retrieval system without
permission, in writing, from the author and the publishers.
This book contains information obtained from authentic and
highly regarded sources. Reprinted material is quoted with
permission. Reasonable efforts have been made to publish
reliable data and information, but the authors and the
publishers cannot assume responsibility for the validity of all
materials. Neither the authors nor the publishers, nor
anyone else associated with this publication, shall be liable
for any loss, damage or liability directly or indirectly caused
or alleged to be caused by this book.
Neither this book nor any part may be reproduced or
transmitted in any form or by any means, electronic or
mechanical, including photocopying, microfilming and
recording, or by any information storage or retrieval system,
without permission in writing from Delhi Academy of
Medical Sciences. The consent of Delhi Academy of Medical
Sciences does not extend to copying for general distribution,
for promotion, for creating new works, or for resale. Specific
permission must be obtained in writing from Delhi Academy
of Medical Sciences for such copying.
Trademark notice: Product or corporate names may be
trademarks or registered trademarks, and are used only for
identification and explanation, without intent to infringe.
Typeset by Delhi Academy of Medical Sciences Pvt. Ltd., New
Delhi (India).
Contents

1. PATHOLOGY
Q1 Cell as a Unit of Health and Disease – Questions
A1 Cell as a Unit of Health and Disease – Explanations
Q2 Cell Adaptation, Injury and Death – Questions
A2 Cell Adaptation, Injury and Death – Explanations
Q3 Inflammation, Repair and Hemodynamics – Questions
A3 Inflammation, Repair and Hemodynamics – Explanations
Q4 Genetics – Questions
A4 Genetics – Explanations
Q5 Diseases of the Immune System – Questions
A5 Diseases of the Immune System – Explanations
Q6 Neoplasia – Questions
A6 Neoplasia – Explanations
Q7 Red Blood Cells and its Disorders – Questions
A7 Red Blood Cells and its Disorders – Explanations
Q8 WBC Disorders – Questions
A8 WBC Disorders – Explanations
Q9 Platelet Disorders – Questions
A9 Platelet Disorders – Explanations
Q10 Transfusion Medicine – Questions
A10 Transfusion Medicine – Explanations
Q11 Systemic Pathology – Questions
A11 Systemic Pathology – Explanations

2. PHARMACOLOGY
Q1 General Pharmacology — Questions
A1 General Pharmacology — Explanations
Q2 Autonomic Nervous System — Questions
A2 Autonomic Nervous System — Explanations
Q3 Autacoids — Questions
A3 Autacoids — Explanations
Q4 Respiratory System — Questions
A4 Respiratory System — Explanations
Q5 Endocrine System — Questions
A5 Endocrine System — Explanations
Q6 Anaesthesia — Questions
A6 Anaesthesia — Explanations
Q7 Central Nervous System — Questions
A7 Central Nervous System — Explanations
Q8 Cardiovascular System — Questions
A8 Cardiovascular System — Explanations
Q9 Hematology — Questions
A9 Hematology — Explanations
Q10 Gastrointestinal System — Questions
A10 Gastrointestinal System — Explanations
Q11 Antimicrobial Drugs — Questions
A11 Antimicrobial Drugs — Explanations

3. MICROBIOLOGY
Q1 General Microbiology — Questions
A1 General Microbiology — Explanations
Q2 Immunology — Questions
A2 Immunology – Explanations
Q3 Bacteriology — Questions
A3 Bacteriology — Explanations
Q4 Virology — Questions
A4 Virology — Explanations
Q5 Mycology — Questions
A5 Mycology — Explanations
Q6 Parasitology — Questions
A6 Parasitology — Explanations

4. FORENSIC MEDICINE & TOXICOLOGY


Q Questions
A Explanations
Cell as a Unit of Health and Disease –
Questions
1. Which of the following plays a role in gene editing?
a. Gene Xpert
b. CRISPR
c. Heath care apps
d. Big data

2. Cell to cell permeability occurs through:


a. Occludin
b. Zona adherens
c. Connexins
d. Zonulin

3. Which Collagen is typical of basement membrane?


a. Type I
b. Type II
c. Type III
d. Type IV

4. All of the following are Intermediate filament except?


a. Lamin
b. Cadherin
c. Vimentin
d. Desmin

5. Titin protein mutated in?


a. DCM
b. HOCM
c. RCM
d. Non-functional cardiomyopathy

6. Bridging fibrosis in large wounds is due to:


a. Keratinocyte growth factor
b. Epidermal growth factor
c. Platelet derived growth factor
d. Transforming growth factor-β

7. Types of collage playing important role in wound healing


a. I and III
b. II and V
c. III and IV
d. V and IX
8. Oval stem cells are located in-
a. Canal of schlemm
b. Canal of herring
c. Space of disse
d. Basal lamina of myotubules

9. Heritable changes in gene expression not caused by alterations in DNA


sequence refers to?
a. Genetics
b. Epigenetics
c. Mutations
d. Transposons

10. Epigenetic factors refers to?


a. Histone methylation
b. Histone phosphorylation
c. DNA methylation
d. All of the above

11. Collagen present in cornea?


a. Type 1
b. Type 2
c. Type 4
d. Type 7

12. Gene silencing RNA-


a. rRNA
b. tRNA
c. miRNA
d. None

13. The most abundant glycoprotein present in basement membrane is-


a. Laminin
b. Fibronectin
c. Collagen type 4
d. Heparan sulphate

1. b 2. c 3. d 4. b 5. a 6. d 7. a 8. b 9. b 10. d 11. b

12. c 13. a

14. Which of the following statement is not true?


a. Cytokeratin is the marker of muscle cells
b. Vimentin is used to stain fibroblasts
c. Neurofilaments are present in neurons
d. Desmin can be used as a marker for muscle cells
15. Which of the following is both morphogenic and mitogenic?
a. Insulin growth factor
b. Bone morphogenic factor
c. Fibroblast growth factor
d. Epidermal growth factor

16. Which of the following is involved in stem cell self renewal and
pluripotency?
a. Oct 3/4
b. sox 2
c. c-myc
d. FLT3 ligand
e. c-kit

17. All of the following are true about single nucleotide polymorphisms except
a. Almost always biallelic
b. 1% occur in non coding regions
c. Can be neutral variant with no effect on gene function
d. Can serve as a marker for disease

18. Which of the following statements is true?


a. Actin is the most abundant cytosolic protein in the cell
b. Lamin is the most abundant glycoprotein in the basement membrane
c. Fibronectin imparts scaffolding to the cytoplasm
d. FACITs are seen in association with type I collagen

19. Which of the following pairs of cell cycle phase and cyclin/CDK is
incorrectly matched?
a. G1S checkpoint- cyclin D/CDK 6
b. G1S checkpoint- cyclin D/CDK4
c. S phase- cyclin A/CDK 6
d. G2M checkpoint- cyclin B/CDK1

20. Which of the following organ and site of stem cell is incorrectly matched?
a. Liver- oval cells in Space of Disse
b. Skin- bulge of hair follicle
c. Brain- dentate gyrus
d. Cornea- limbus

21. Major cytokine involved in fibrosis


a. Transforming growth factor-α
b. Transforming growth factor-β
c. Fibroblast growth factor
d. Epidermal growth factor

22. Most abundant collagen in the body?


a. Type I
b. Type II
c. Type III
d. Type IV

23. FACIT collagen is?


a. Type I
b. Type III
c. Type IX
d. Type XI

14. a 15. b 16. a, 17. b 18. a 19. c 20. a 21. b 22. a 23. c
b, c, e
Cell as a Unit of Health and Disease –
Explanations
1. B
Reference: Robbins Basic Pathology, 10th Edition
Clustered regularly interspaced short palindromic repeats (CRISPRs) and Cas (or
CRISPRassociated genes) are linked genetic elements that endow prokaryotes with a
form of acquired immunity to phages and plasmids. Bacteria use this system to
sample the DNA of infecting agents, incorporating it into the host genome as CRISPRs.
CRISPRs are transcribed and processed into an RNA sequence that binds and directs
the nuclease Cas9 to a sequences (e.g., a phage), leading to its cleavage and the
destruction of the phage. Gene editing repurposes this process by using artificial
guide RNAs (gRNAs) that bind Cas9 and are complementary to a DNA sequence of
interest. Once directed to the target sequence by the gRNA, Cas9 induces double-
strand DNA breaks.
Option A- The Xpert MTB/RIF is a cartridge-based nucleic acid amplification test for
simultaneous rapid tuberculosis diagnosis and rapid antibiotic sensitivity test. It is an
automated diagnostic test that can identify Mycobacterium tuberculosis DNA and
resistance to rifampicin
Option C and Option D are far removed from gene editing and have no bearing
whatsoever to Genomics

2. C
Reference: Robbins Pathological Basis of Disease, 9th edition
Communicating junctions (gap junctions) mediate the passage of chemical or
electrical signals from one cell to another. The junction consists of a dense planar array
of 1.5- to 2-nm pores (called connexons) formed by hexamers of transmembrane
protein connexins. These pores permit the passage of ions, nucleotides, sugars, amino
acids, vitamins, and other small molecules; the permeability of the junction is rapidly
reduced by lowered intracellular pH or increased intracellular calcium.
Option A and D- Occluding junctions (tight junctions) seal adjacent cells together to
create a continuous barrier that restricts the paracellular (between cells) movement of
ions and other molecules. Viewed en face, occluding junctions form a tight meshlike
network of macromolecular contacts between neighboring cells. The complexes that
mediate these cell–cell interactions are composed of multiple proteins, including
occludin, claudin, zonulin, and catenin
Option B- When the adhesion focus is between cells, and is small and rivet-like, it is
designated a spot desmosome or macula adherens. When such a focus attaches the
cell to the ECM, it is called a hemidesmosome. Similar adhesion domains can also
occur as broad bands between cells, where they are denoted as belt desmosomes or
zona adherens.

3. D
Reference: Robbins Pathological Basis of Disease, 9th edition
Non-fibrillar collagens may contribute to the structures of planar basement
membranes (type IV collagen).

4. B
Intermediate filaments are 10-nm diameter fibrils that comprise a large and
heterogeneous family. Individual types have characteristic tissue-specific patterns of
expression that can be useful for assigning a cell of origin for poorly differentiated
tumors.
Lamin A, B, and C: nuclear lamina of all cells
Vimentin: mesenchymal cells (fibroblasts, endothelium)
Desmin: muscle cells, forming the scaffold on which actin and myosin contract
Neurofilaments: axons of neurons, imparting strength and rigidity
Glial fibrillary acidic protein: glial cells around neurons
Cytokeratins: at least 30 distinct varieties, subdivided into acidic (type I) and
neutral/basic (type II); different types present in different cells, hence can be used as
cell markers
Option A and D- Intermediate filaments
Option B- Cell-cell desmosomal junctions are formed by homotypic association of
transmembrane glycoproteins called cadherins

5. A
Reference: Robbins Pathological Basis of Disease, 9th edition
DCM is familial in at least 30% to 50% of cases, in which it is caused by mutations in a
diverse group of more than 20 genes encoding proteins involved in the cytoskeleton,
sarcolemma, and nuclear envelope (laminin A/C). In particular, mutations in TTN, a
gene that encodes titin (so-called because it is the largest protein expressed in
humans), may account for approximately 20% of all cases of DCM.
Option B- Mutations causing HCM are found most commonly in the gene encoding β-
myosin heavy chain (β-MHC), followed by the genes coding for cardiac TnT, α-
tropomyosin, and myosin-binding protein C (MYBP-C); overall, these account for 70%
to 80% of all cases.
Option C- Restrictive cardiomyopathy can be idiopathic or associated with distinct
diseases or processes that affect the myocardium, principally radiation fibrosis,
amyloidosis, sarcoidosis, metastatic tumors, or the deposition of metabolites that
accumulate due to inborn errors of metabolism.
Option D- It is a non specific terminology.

6. D
Reference: Robbins Pathological Basis of Disease, 9th edition
TGF-β drives scar formation, and applies brakes on the inflammation that accompanies
wound healing.
• TGF-β stimulates the production of collagen, fibronectin, and proteoglycans, and it
inhibits collagen degradation by both decreasing matrix metalloproteinase (MMP)
activity and increasing the activity of tissue inhibitors of proteinases. TGF-β is involved
not only in scar formation after injury, but also drives fibrosis in lung, liver, and
kidneys in the setting of chronic inflammation.
• TGF-β is an antiinflammatory cytokine that serves to limit and terminate
inflammatory responses. It does this by inhibiting lymphocyte proliferation and the
activity of other leukocytes. Animal models lacking TGF-β have widespread and
persistent inflammation
Option A, B and C- Refer to the following table

Growth Factor Sources Functions

Epidermal. growth Activated macrophages, Mitogenic for keratinocytes and fibroblasts; stimulates
factor (EGF) salivary glands, keratinocyte migration; stimulates formation of
keratinocytes, and many granulation tissue
other cells

Transforming growth Activated macrophages, Stimulates proliferation of hepatocytes and many other
factor-α (FGF-α) keratinocytes, many other epithelial cells
cell types

Hepatocyte growth Fibroblasts, stromal cells In Enhances proliferation of hepatocytes and other
factor (HGF) the liver, endothelial cells epithelial cells; Increases cell motility
(scatter factor)

Vascular endothelial Mesenchyme cells Stimulates proliferation of endothelial cells; Increases


growth factor vascular permeability
(VEGF}

Platelet-derived Platelets, macrophages, Chemotactic for neutrophils, macrophages, fibroblasts,


growth factor endothelial cells, smooth and smooth muscle cells; activates and stimulates
IPOGF) muscle cells, keratinocytes proliferation of fibroblasts, endothelial, and other cells;
stimulates ECM protein synthesis

Fibroblast growth Macrophages, mast cells, Chemotactic and mitogenic for fibroblasts; stimulates
factors (FGFs), endothelial cells, many other angiogenesis and ECM protein synthesis
Including acidic (FGF- cell types
1) and basic (FGF-2)

Transforming growth Platelets, T lymphocytes, Chemotactic for leukocytes and fibroblasts; stimulates
factor-β (TGF-β) macrophages, endothelial ECM protein synthesis; suppresses acute Inflammation
cells, keratinocytes, smooth
muscle cells, fibroblasts

Keratinocyte growth fibroblasts Stimulates keratinocyte migration, proliferation, and


factor (KGF) (i.e., differentiation
FGF-7)

ECM Extracellular membrane.

7. A
Reference: Robbins Pathological Basis of Disease, 9th edition
At first a provisional matrix containing fibrin, plasma fibronectin, and type III collagen
is formed, but in about 2 weeks this is replaced by a matrix composed primarily of
type I collagen. Ultimately, the original granulation tissue scaffold is converted into a
pale, avascular scar, composed of spindleshaped fibroblasts, dense collagen,
fragments of elastic tissue, and other ECM components

8. B
Reference: Robbins Pathological Basis of Disease, 9th edition
In situations where the proliferative capacity of hepatocytes is impaired, such as after
chronic liver injury or inflammation, progenitor cells in the liver contribute to
repopulation. In rodents, these progenitor cells have been called oval cells because of
the shape of their nuclei. Some of these progenitor cells reside in specialized niches
called canals of Hering, where bile canaliculi connect with larger bile ducts. The
signals that drive proliferation of progenitor cells and their differentiation into mature
hepatocytes are topics of active investigation.
Option A- circular lymphatic-like vessel in the eye that collects aqueous humor from
the anterior chamber and delivers it into the episcleral blood vessels via aqueous veins
Option C- location in the liver between a hepatocyte and a sinusoid. It contains the
blood plasma. Microvilli of hepatocytes extend into this space, allowing proteins and
other plasma components from the sinusoids to be absorbed by the hepatocytes.
Fenestration and discontinuity of the endothelium, as well as its basement membrane,
facilitates this transport.
Option D- Site of regeneration

9. B
Option A- Genetics is the study of genes, genetic variation, and heredity in living
organism
Option C- A gene mutation is a permanent alteration in the DNA sequence
Option D- DNA sequence that can change its position within a genome, sometimes
creating or reversing mutations and altering the cell’s genetic identity and genome
size. Transposition often results in duplication of the same genetic material.

10. D
Epigenetic factors include-
Histone methylation
Histone phosphorylation
Histone acetylation
DNA methylation
Chromatin organising factors

11. B

COLLAGEN TYPE TISSUE DISTRIBUTION GENETIC DISORDERS

Fibrillar Collagens

I Ubiquitous in hard and soft tissues Osteogenesis Imperfecta

II Cartilage, intervertebral disc, vitreous Achondrogenesis type 2

III Hollow organs, soft tissues Vascular Ehler Danlos Syndrome

V Soft tissues, blood vessels Classical Ehler Danlos Syndrome

IX Cartilage, vitreous Stickler Syndrome

Basement Membrane Collagen

IV Basement membranes Alport Syndrome

12. C
Noncoding RNAs are encoded by genes that are transcribed but not translated.
Although many distinct families of noncoding RNAs exist, we will only discuss two
examples here: small RNA molecules called microRNAs, and long noncoding RNAs
>200 nucleotides in length. Post transcriptional silencing of gene expression by
miRNA is a fundamental and well-conserved mechanism of gene regulation present in
all eukaryotes (plants and animals).
Option A- ribosomal RNA Option B- transfer RNA

13. A
Prototypical adhesive glycoproteins include fibronectin (a major component of the
interstitial ECM) and laminin (a major constituent of basement membrane). Integrins
are representative of the adhesion receptors, also known as cell adhesion molecules
(CAMs); the CAMs also include immunoglobulins, cadherins, and selectins.
Option A- Laminin is the most abundant glycoprotein in basement membrane. It is an
820-kD cross-shaped heterotrimer that connects cells to underlying ECM components
such as type IV collagen and heparan sulfate. Besides mediating attachment to
basement membrane, laminin can also modulate cell proliferation, differentiation, and
motility
Option B- Fibronectin is a large (450 kD) disulfide-linked heterodimer that exists in
tissue and plasma forms; it is synthesized by a variety of cells, including fibroblasts,
monocytes, and endothelium.
Option C- Basement membrane collagen
Option D- Proteoglycans form highly hydrated compressible gels that confer resistance
to compressive forces; in joint cartilage, proteoglycans also provide a layer of
lubrication between adjacent boney surfaces. Proteoglycans consist of long
polysaccharides, called glycosaminoglycans (examples are keratan sulfate and
chondroitin sulfate) attached to a core protein; these are then linked to a long
hyaluronic acid polymer called hyaluronan, in a manner reminiscent of the bristles on
a test tube brush.

14. A
Cytokeratin is a marker for epithelial cells
Option B- Vimentin is a marker for mesenchymal cells and tumors arising from them
Option C- stains axons of neurofilaments
Option D- stains muscle cells and tumors arising from them

15. B
Bone morphogenetic proteins (BMPs) are multi-functional growth factors that belong
to the transforming growth factor beta (TGFbeta) superfamily and are both
morphogenic and mitogenic.

Growth Factor Sources Functions

Epidermal. growth Activated macrophages, Mitogenic for keratinocytes and fibroblasts; stimulates
factor (EGF) salivary glands, keratinocyte migration; stimulates formation of
keratinocytes, and many granulation tissue
other cells

Transforming growth Activated macrophages, Stimulates proliferation of hepatocytes and many other
factor-α (FGF-α) keratinocytes, many other epithelial cells
cell types

Hepatocyte growth Fibroblasts, stromal cells In Enhances proliferation of hepatocytes and other
factor (HGF) the liver, endothelial cells epithelial cells; Increases cell motility
(scatter factor)

Vascular endothelial Mesenchyme cells Stimulates proliferation of endothelial cells; Increases


growth factor vascular permeability
(VEGF}

Platelet-derived Platelets, macrophages, Chemotactic for neutrophils, macrophages, fibroblasts,


growth factor endothelial cells, smooth and smooth muscle cells; activates and stimulates
IPOGF) muscle cells, keratinocytes proliferation of fibroblasts, endothelial, and other cells;
stimulates ECM protein synthesis

Fibroblast growth Macrophages, mast cells, Chemotactic and mitogenic for fibroblasts; stimulates
factors (FGFs), endothelial cells, many other angiogenesis and ECM protein synthesis
cell types
Including acidic (FGF-
1) and basic (FGF-2)

Transforming growth Platelets, T lymphocytes, Chemotactic for leukocytes and fibroblasts; stimulates
factor-β (TGF-β) macrophages, endothelial ECM protein synthesis; suppresses acute Inflammation
cells, keratinocytes, smooth
muscle cells, fibroblasts

Keratinocyte growth fibroblasts Stimulates keratinocyte migration, proliferation, and


factor (KGF) (i.e., differentiation
FGF-7)

ECM Extracellular membrane.

16. A, B, C, E
Pluripotency of embryonic stem cell depends on expression of following transcription
factors-
1. Oct 3/4
2. Sox 2
3. C-Myc
4. Kfl 4
5. ckit
Pluripotency of embryonic stem cell is inhibited by home box protein ‘Nanog’.

17. B
The two most common forms of DNA variation in the human genome are single-
nucleotide polymorphisms (SNPs) and copy number variations (CNVs). SNPs are
variants at single nucleotide positions and are almost always biallelic (i.e., only two
choices exist at a given site within the population, such as A or T). Much effort has
been devoted to mapping common SNPs in human populations. Over 6 million human
SNPs have been identified, many of which show wide variation in frequency in
different populations. SNPs occur across the genome—within exons, introns, intergenic
regions, and coding regions. Overall, about 1% of SNPs occur in coding regions, which
is about what would be expected by chance, since coding regions comprise about
1.5% of the genome. SNPs located in non-coding regions may fall in regulatory
elements in the genome, thereby altering gene expression; in such instances the SNP
may have a direct influence on disease susceptibility. In other instances, the SNP may
be a “neutral” variant that has no effect on gene function or carrier phenotype.
However, even “neutral” SNPs may be useful markers if they happen to be co-inherited
with a disease-associated gene as a result of physical proximity. In other words, the
SNP and the causative genetic factor are in linkage disequilibrium. There is hope that
groups of SNPs may serve as markers of risk for multigenic complex diseases such as
type II diabetes and hypertension. However, the effect of most SNPs on disease
susceptibility is weak, and it remains to be seen if identification of such variants, alone
or in combination, can be used to develop effective strategies for disease prevention

18. A
Laminin is the most abundant glycoprotein. Lamin is an intermediate filament of the
cytosol.
Fibronectin is a component of the extra cellular matrix, not cytoplasm.
FACITs (fibril-associated collagen with interrupted triple helices), such as type IX
collagen are in cartilage.

19. C

20. A

Tissue and Stem Cells Location Extra

Bone marrow= HSCs HSCs can be collected from bone marrow, Bone marrow produces 1.5 x
and MSCs umbilical cord blood and peripheral blood of 106 blood cells per second
individuals receiving CSF’s

Liver- Oval cells Canal of Hering- junction between the biliary ductal Activated only when hepatocyte
(Bipotential- system and parenchymal hepatocytes proliferation is stopped
hepatocytes or biliary
cells)

Brain- Neural Stem Cells Subventricular zone and Dentate gyrus of Capable of generating neurons,
(NSCs) Hippocampus astrocytes and
oligodendrocytes

Skin Hair Follicle Bulge Activated by Wnt pathway and


inhibition of signalling from BMP
Interfollicular area of surface epidermis pathway

Sebaceous glands

Intestine Above paneth cells in SI and base of crypt in case Wnt and BMP pathways
of colon

Skeletal Muscle Beneath myocyte basal lamina Regeneration is through


Satellite cell replication

Cornea At the junction between the epithelium of cornea Limbal Stem Cells (LSCs)
and conjunctiva

21. B
22. A

23. c
Cell Adaptation, Injury and Death –
Questions
24. True about Metaplasia is?
a. Involves only epithelial cells
b. Is irreversible
c. Can occurs at stem cells level
d. Columnar is the most common type

25. All of the following true regarding hypertrophy except:


a. Increased in cell size without increase in number
b. DNA content same as in normal cells
c. Increase in cell size is due to synthesize of more cellular proteins
d. Associated with a switch of contractile proteins from adult to fetal or neonatal
forms

26. Find the false statement about metaplasia:


a. Reversible
b. No loss of polarity
c. Reprogramming of stem cells
d. Pleomorphism present

27. Increase in the number of goblet cells in the non-respiratory terminal


bronchiole is an example of:
a. Anaplasia
b. Dysplasia
c. Metaplasia
d. Hyperplasia

28. In respiratory tract metaplasia occurs from:


a. Squamous to columnar
b. Columnar to cuboidal
c. Columnar to squamous
d. Cuboidal to squamous

29. In Vitamin-A deficiency, cancerous lesions occur due to-


a. Metaplasia
b. Dysplasia
c. Aplasia
d. Hyperplasia

30. All are cellular adaptations except:


a. Hypertrophy
b. Hyperplasia
c. Necrosis
d. Metaplasia

31. Features of irreversible cell injury is/are? (PGI type)


a. Lysosomal rupture
b. Pyknosis
c. Bleb formation on membrane
d. Severe mitochondrial dysfunction

32. In myocardium reperfusion injury is due to?


a. Ca
b. Mg
c. K
d. Mn

33. Which of the following is not a sign of reversible cell injury?


a. ATP depletion
b. Cell shrinkage
c. Fatty acid deposition
d. Reduction of phosphorylation

34. Cells seen in chronic infection of pseudomonas:


a. Neutrophils
b. Eosinophils
c. Lymphocytes
d. Macrophage

35. For programmed cell death and autopagy, which is pro apoptotic genes?
a. BCL2
b. BAX
c. BCL-XL
d. BIM
24. c 25. b 26. d 27. c 28. c 29. a 30. c 31. 32. a 33. b 34. a
a,d

35. b

36. True about p53 gene? (PGI type)


a. Tumor suppression
b. Proapoptotic
c. Antiapoptotic
d. Cell repair
e. Oncogene

37. True about caspases are?


a. They are enzymes starting apoptosis
b. They inhibit apoptosis
c. They are receptors of apoptosis
d. They are proteases which cause cellular death in apoptosis

38. True about necroptosis is all except?


a. Caspase 1 & 11 is involved
b. Caspase independent
c. Failure of activation of caspase 8
d. Lipid peroxidation is seen

39. SMAC/DIAMBLO is a:
a. Anti apoptotic protein
b. Induces necrosis
c. Acts both as anti and pro apoptotic protein
d. Pro-apoptotic protein

40. Which of the following is an execution caspase:


a. Caspase 3
b. Caspase 5
c. Caspase 8
d. Caspase 9

41. Annexin V is a marker of-


a. Necrosis
b. Gangrene
c. Aging
d. Apoptosis

42. The following is not a sensor of apoptosis:


a. Puma
b. Noxa
c. BAX
d. BAD

43. Intrinsic pathway of apoptosis is initiated by all the following except:


a. Growth factor withdrawal
b. DNA damage
c. Protein misfolding
d. Type 1 TNF receptor

44. Defective apoptosis and increased cell survival is seen in:


a. Autoimmune disease
b. Neurodegenerative diseases
c. Viral infections
d. Ischemic injury
45. Receptor associated kinases 1 (RIP1) and 3 (RIP3) are involved in:
a. Necrosis
b. Apoptosis
c. Necroptosis
d. Pyroptosis

46. Inflammasome is formed in:


a. Necrosis
b. Apoptosis
c. Necroptosis
d. Pyroptosis

47. Fibrinoid necrosis is seen in all of the following except:


a. Malignant hypertension
b. Aschoff’s nodule
c. Polyarteritis nodosa
d. Diabetic glomerulosclerosis

48. Dystrophic calcification is/are found in? (PGI type)


a. Monckeberg’s medical sclerosis
b. Papillary carcinoma thyroid
c. Hyperparathyroidism
d. Meningioma
e. Vitamin D intoxication

49. Highest telomerase activity is seen in:


a. Stem cells
b. Somatic cells
c. Germ cells
d. Benign tumors

50. Organelle where H2O2 is produced and destroyed is:


a. Peroxisome
b. Lysosome
c. Golgi body
d. Ribosome

51. Which of these is not responsible for removal of free radicals?


a. Catalase
b. Superoxide Dismutase
c. NADPH oxidase
d. Glutathione peroxidase
36. 37. d 38. a 39. d 40. a 41. d 42. c 43. d 44. a 45. c 46. d
a, b,
d
47. d 48. 49. c 50. a 51. c
a, b,
d

52. Endometrial adenocarcinoma is caused by


a. Hyperplasia
b. Atrophy
c. Both
d. None

53. Metaplasia of respiratory tract is due to


a. Vitamin E deficiency
b. Vitamin K deficiency
c. Vitamin A deficiency
d. Vitamin D deficiency

54. In cell death, myelin figures are derived from-


a. Nucleus
b. Cell membrane
c. Cytoplasm
d. Mitochondria

55. The sign of reversible injury in a case of alcoholic liver disease-


a. Loss of cell membrane
b. Nuclear karyolysis
c. Cytoplasmic vacuole
d. Pyknosis

56. Irreversible cell injury is indicated by-


a. Accumulation of calcium in mitochondria
b. Myelin figures
c. ATP depletion
d. Shifting of ribosomes

57. Type of necrosis occurring in brain-


a. Coagulative
b. Liquefactive
c. Fibrinoid
d. Caseous

58. Fat necrosis is common in-


a. Omentum
b. Breast
c. Retroperitoneal fat
d. All of the above
59. Example of apoptosis is-
a. Councilman bodies
b. Gamma gandy body
c. Russell bodies
d. None

60. Characteristic feature of apoptosis-


a. Cell membrane intact
b. Cytoplasmic eosinophilia
c. Nuclear moulding
d. Cell swelling

61. A 55-years-old male who has a long history of excessive drinking presents
with signs of alcoholic hepatitis. Microscopic Examination of a biopsy of this
patient’s liver reveals irregular eosinophilic hyaline inclusions within the
cytoplasm of the hepatocytes. These eosinophilic inclusions are composed
of:
a. Immunoglobulin
b. Excess plasma proteins
c. Keratin intermediate filaments
d. Lipofuscin

62. Characteristic feature of apoptosis-(PGI)


a. Cell membrane intact
b. Cytoplasmic eosinophilia
c. Nuclear moulding
d. Cell Shrinkage

63. Lipofuscin granules are examples of


a. Apoptotic bodies
b. Residual bodies
c. Psammomatous calcification
d. Hyaline change

64. Squamous metaplasia is seen in all of the following except


a. Vitamin A deficiency
b. Smoking
c. Gastroesophageal reflux disease
d. Stone obstructing a duct

65. All of the following are true about apoptosis except


a. Cell shrinkage
b. Altered orientation of lipids
c. Only pathological
d. No adjacent inflammation

66. All are true about reversible cell injury except


a. Cellular swelling is the earliest manifestation
b. Organ shows increased turgor and has decreased weight
c. Clear vacuoles represent distended and pinched off ER
d. Increased eosinophilia of cytoplasm
52. c 53. c 54. b 55. c 56. a 57. b 58. d 59. a 60. a 61. c 62.
a, d

63. b 64. c 65. c 66. b

67. Which of the following is/are true about the microscopic appearance of
necrotic cells?
a. Increased eosinophilia due to loss of cytoplasmic RNA
b. Increased eosinophilia due to denaturation of cytoplasmic proteins
c. Glassy homogenous appearance due to loss of glycogen particles
d. All of the above

68. Which phenomenon/phenomena characterise irreversibility of cell injury?


a. Mitochondrial dysfunction
b. Profound membrane damage
c. Nuclear disaggregation
d. Both A and B

69. All of the following are pro apoptotic except


a. Smac
b. Diablo
c. BAX
d. BCL- XL

70. All of the following is/are true about extrinsic pathway of apoptosis except
a. Engagement of plasma membrane death receptors
b. Type I TNF receptor is prototype
c. Caspase 9 is classically involved
d. Mechanism of apoptosis of virus infected cells

71. Programmed necrosis is another name for


a. Necroptosis
b. Pyroptosis
c. Atrophy
d. Autophagy

72. Macrophage-specific deletion of which autophagy related gene increases


susceptibility to tuberculosis
a. Atg 3
b. Atg 4
c. Atg 5
d. Atg 6
73. Which of the following is false about lipofuscin?
a. Injurious to cell
b. Perinuclear in location
c. Lipid peroxidation
d. Yellow brown granular pigment

74. All are examples of metastatic calcification except


a. Milk Alkali syndrome
b. Hyperparathyroidism
c. Renal osteodystrophy
d. Atherosclerosis

75. Metaplasia arises from reprograming of-


a. Stem cells
b. Stellate cells
c. Squamous cells
d. Columnar cells

76. “Caspase-independent” programmed cell death-


a. Necrosis
b. Necroptosis
c. Apoptosis
d. None

77. Immune complexes mediated necrosis is of which type?


a. Coagulative necrosis
b. Liquefactive necrosis
c. Caseous necrosis
d. Fibrinoid necrosis

78. Stain for fat all except:


a. Oil red O
b. Sudan black
c. Sudan III
d. Congo red

79. The enzyme that protects brain from free radical injury:
a. Superoxide dismutase
b. Catalase
c. Glutathione peroxidase
d. Monoamine oxidase

80. Physiological hyperplasia and hyper-trophy are seen simultaneously in-


a. Uterus in pregnancy
b. Skeletal muscle in athletes
c. Breast at puberty
d. Thyroid gland during pregnancy

81. All are reversible injury of cell, except-


a. Vacuole
b. Karyorrhexis
c. Fat accumulation
d. Cell wall swelling

67. d 68. d 69. d 70. c 71. a 72. c 73. a 74. d 75. a 76. b 77. d

78. d 79. a 80. 81. b


a,c

82. Fibrinoid necrosis is seen in following except-


a. Pan
b. Diabetes mellitus
c. Malignant hypertension
d. SLE

83. Esoinophilia in necrosed tissue is due to


a. Coagulation of proteins
b. Denaturation of enzymes
c. Lysosomal damage
d. Mitochondrial damage

84. True about apoptosis are all except-


a. Inflammation is present
b. Chromosomal breakage
c. Clumping of chromatin
d. Cell shrinkage

85. CD-95 is a marker of-


a. Death receptor
b. MHC complex
c. T helper cells
d. NK cells

86. True about apoptosis is-


a. Energy dependent
b. Self-initiated
c. Swelling of cytoplasm
d. Loss of membrane integrity

87. CD-95 has a major role in-


a. Apoptosis
b. Cell necrosis
c. Interferon activation
d. Proteolysis

88. Which of the following organelles plays a pivotal role in apoptosis?


a. Mitochondria
b. Endoplasmic reticulum
c. Nucleus
d. Golgi apparatus

89. Cytosolic cytochrome C plays an important function in-


a. Apoptosis
b. Cell necrosis
c. Electron transport chain
d. Cell division

90. CD-95 is a marker of-


a. Intrinsic pathway of apoptosis
b. Extrinsic pathway of apoptosis
c. Monocyte
d. Leucocyte

91. In apoptosis, cytochrome C acts through-


a. Apaf-1
b. BCL-2
c. FADD
d. TNF

92. Which of the following has a direct role in apoptosis-


a. Nitric oxide
b. Adenyl cyclase
c. cAMP
d. Cytochrome C

93. The following is a antiapoptotic gene-


a. Bax
b. Bad
c. BCL-XL
d. Bim

94. SMAC/DIABLO
a. Anti-Apoptotic proteins
b. Induces necrosis most often
c. It can act as both apoptotic and antiapoptotic proteins
d. Pro-apoptotic proteins

95. Which of the following is an antiapoptotic gene?


a. C-myc
b. P53
c. BCL-2
d. Bax

96. Brown atrophy is due to accumulation of-


a. Melanin
b. Hemosiderin
c. Hematin
d. Lipofuscin

97. Which of the following pigments are involved in free radical injury?
a. Lipofuscin
b. Melanin
c. Bilirubin
d. Hematin

98. Dystrophic calcification is seen in-


a. Milk alkali syndrome
b. Atheromatous plaque
c. Hyperparathyroidism
d. Vitamin A intoxication
82. b 83. a 84. a 85. a 86. a 87. a 88. a 89. a 90. b 91. a 92. d

93. c 94. d 95. c 96. d 97. a 98. b

99. True about psammoma bodies are all except-


a. Seen in meningioma
b. Concentric whorled appearance
c. Contains calcium deposits
d. Seen in teratoma

100. Calcification of soft tissues without any disturbance of calcium


metabolism is called-
a. Ionotrophic calcification
b. Monotrophic calcification
c. Dystrophic calcification
d. Calcium induced calcification

101. Metastatic calcification is not seen in-


a. Kidney tubules
b. Fundal glands of stomach
c. Alveoli of lungs
d. Media of Monckeberg degeneration

102. Apoptosis is inhibited by-


a. P53
b. N-myc
c. RAS
d. BCL-2

103. Calcification in necrotic tissue is called-


a. Metastatic calcification
b. Dystrophic calcification
c. Calcinosis
d. Tumoral calcinosis

104. Increase in this patient’s serum enzyme (e.g. AST/ALT) Levels are seen in
hepatitis. Most likely this results from?
a. Autophagy by lysosomes
b. Clumping of nuclear chromatin
c. Defects in the cell membrane
d. Swelling of the mitochondria

105. Which of the following cells is most likely to have the highest telomerase
activity?
a. Endothelial cells
b. Germ cells
c. Neutrophils
d. Erythrocytes

106. Hypertrophy is a type of –


a. Cell injury
b. Cellular adaptation
c. Carcinoma
d. Cell aging

107. Which finding on electron microscopy indicates irreversible cell injury -


a. Dilatation of endoplasmic reticulum
b. Dissociation of ribosomes from rough endoplasmic reticulum
c. Flocculent amorphous densities in the mitochondria
d. Myelin figures

108. Irreversible cell injury is indicated by-


a. Accumulation of calcium in cytosol
b. Myelin figures
c. ATP depletion
d. Shifting of ribosomes

109. In cell death, myelin figures are derived from -


a. Nucleus
b. Cell membrane
c. Cytoplasm
d. Mitochondria

110. MI is a type of-


a. Coagulative necrosis
b. Liquefactive necrosis
c. Caseous necrosis
d. Fat necrosis
99. d 100. c 101. 102. 103. 104. c 105. 106. b 107. 108. a
d d b b c,d

109. 110.
b a
Cell Adaptation, Injury and Death –
Explanations
24. C
Metaplasia is a change in phenotype of differentiated cells, often in response to
chronic irritation, that makes cells better able to withstand the stress; usually
induced by altered differentiation pathway of tissue stem cells; may result in
reduced functions or increased propensity for malignant transformation.
Option A- Both epithelial and mesenchymal metaplasia can be seen
Option B- it is a reversible adaptation
Option D- Squamous metaplasia is the most common type

25. B
Cell size is increased by increase in protein synthesis hence genes unexpressed or
silent before are now expressed which leads to an increase in DNA content.
Other statements are true.

26. D
Pleomorphism is a feature of dysplasia and anaplasia, so is loss of polarity.
Other options are true.
Refer to the answer above for more details.

27. C
Change in cell type is metaplasia.
Option A- histological hallmark of malignancy
Option B- disordered growth
Option D- increase in number of cells (not change in cells)

28. C
The normal pseudostratified ciliated columnar epithelium of respiratory tract is
replaced by stratified squamous epithelium in response to smoking.
Option A- Barret’s esophagus (intestinal metaplasia)

29. A
Squamous metaplasia is a common consequence of vitamin A deficiency. (Lung
cancer has been recently implicated with this mechanism)

30. C
Option A-Increase in cell size
Option B-Increase in cell number
Option C- Cell death
Option D- change in cell type

31. A, D
Reversible to irreversible cell injury is a spectrum and can show similar features.
Some however are more indicative of irreversible cell injury.
32. A
Reference : Robbins and Cotran Pathological Basis of Disease, 9th edition
The factors that contribute to reperfusion injury in the myocardium include the
following:
Mitochondrial dysfunction: Mitochondrial permeability increases → apoptosis
activated
Myocyte hypercontracture due to increased levels of intracellular calcium → After
reperfusion, the contraction of myofibrils is augmented and uncontrolled, causing
cytoskeletal damage and cell death.
Free radicals, including superoxide anion (•O2 − ), hydrogen peroxide (H2O2),
hypochlorous acid (HOCl), nitric oxide–derived peroxynitrite, and hydroxyl radicals
(•OH)
Leukocyte aggregation may occlude the microvasculature and contribute to the “no-
reflow” phenomenon. Further, leukocytes elaborate proteases and elastases that
cause cell death.
Platelet and complement activation also contribute to microvascular injury.
Complement activation is thought to play a role in the no-reflow phenomenon by
injuring the endothelium.

33. B
Option A- ATP depletion is one of the earliest biochemical changes occurring in cell
injury
Option B- Feature of apoptosis (cell death, not reversible cell injury)
Option C- Cellular swelling and fatty change are earliest findings of reversible cell
injury
Option D- Depletion of ATP is a consequence of reduced phosphorylation

34. A
Option A- Although cells of acute inflammation, in some infections they persist even
when chronic infection continues like in Pseuodomonas
Option B- Seen in parasitic infections and allergic reactions
Option C- Seen in chronic inflammation and viral infections
Option D- Seen in chronic inflammation

35. B
Option .12 A and C are anti apoptotic
Option B- pro apoptotic
Option D- sensor protein

36. A, B, D
Options C and E are not the function of p53 which serves as a guardian of the genome.

37. D
Best option here is option D. Caspases are not the first enzymes activated in apoptosis
but they are cysteine proteases involved in protein breakdown leading to apoptosis

38. A
Option A- feature of pyroptosis
Option B- caspase independent programmed cell death
Option C- after death receptor interaction failure to activate caspase 8 can lead to
activation of necroptosis
Option D- free radical damage is a part of the mechanism of cell death by necroptosis

39. D
Smac and Diablo are released from the mitochondria when there is increased
mitochondrial permeability and they inactivate Inhibitors of Apoptosis (IAPs) which
inactivate caspases, thus letting caspases function and promoting apoptosis.

40. A
Caspase 3, 6 and 7 are execution caspases
Caspase 8 is an initiator caspase of extrinsic pathway
Caspase 9 is an initiator caspase of intrinsic pathway

41. D
TUNEL is an acronym for terminal deoxynucleotidyl transferase biotin-dUTP nick end
labeling. Terminal deoxynucleotidyl transferase (TdT) is a template-independent DNA
polymerase that is normally active in primitive lymphoid cells to increase receptor
diversity.
TdT can add random nucleotides to the ends of DNA fragments produced by
endonucleases during apoptosis, since 3′ phosphate groups are the substrate for TdT
(3′ phosphate groups are not produced during cell necrosis). Treating tissue sections
with TdT and labeled nucleotides then provides a convenient assay for apoptosis. In
practice, necrotic debris may nonspecifically trap either fluorescent or chromogenic
signals and result in a false-positive assay. Apoptosis can also be detected by means of
assays for activated caspase-3, Fas ligand, and annexin V.

42. C
Option C- Pro apoptotic
Options A, B, D- Sensor of apoptosis

43. D
Option A, B, C- intrinsic pathway
Option D- death receptor or extrinsic pathway

44. A
Defective apoptosis and increased cell survival of cells which should have died is seen
in autoimmune disease and cancer.
Increased apoptosis and decreased cell survival of cells which should not have died is
seen in the other three options.

45. C
Interaction of TNFR and inactivation of caspase 8 leads to activation of RIP1 and RIP3
to cause necroptosis
Option A- lysosomal enzymes
Option C- Caspases 3, 6, 8, 9, 10
Option D- Caspases 1 and 11

46. D
Microbial products that enter the cytoplasm of infected cells are recognized by
cytoplasmic innate immune receptors and can activate the multiprotein complex
called the inflammasome. The function of the inflammasome is to activate caspase1,
(also known as interleukin-1β converting enzyme) which cleaves a precursor form of
IL-1 and releases its biologically active form. IL-1 is a mediator of many aspects of
inflammation, including leukocyte recruitment and fever. Caspase-1 and, more
importantly, the closely related caspase-11 also induce death of the cells. Unlike
classical apoptosis, this pathway of cell death is characterized by swelling of cells, loss
of plasma membrane integrity, and release of inflammatory mediators. Pyroptosis
results in the death of some microbes that gain access to the cytosol and promotes the
release of inflammasome-generated IL-1.

47. D
Fibrinoid necrosis is a special form of necrosis usually seen in immune reactions
involving blood vessels. This pattern of necrosis typically occurs when complexes of
antigens and antibodies are deposited in the walls of arteries. Deposits of these
“immune complexes,” together with fibrin that has leaked out of vessels, result in a
bright pink and amorphous appearance in H&E stains, called “fibrinoid” (fibrin-like) by
pathologists. This pattern of necrosis is seen classically in options A, B and C as well as
rejection of transplants.

48. A, B, D
Dystrophic calcification seen in dying tissues with normal calcium levels.
Option C and E are examples of metastatic calcification seen in normal tissues with
increased calcium levels.

49. C
Telomere length is maintained by nucleotide addition mediated by an enzyme called
telomerase. Telomerase is a specialized RNA-protein complex that uses its own RNA as
a template for adding nucleotides to the ends of chromosomes. Telomerase activity is
expressed in germ cells and is present at low levels in stem cells, but it is absent in
most somatic tissues

50 A

Properties O2 H2 O2 *Oh

Mechanisms Incomplete reduction of O2 during Generated by SOD from Generated from H2O by
of oxidative phosphorylation; by O2 and by oxidases in hydrolysis, e.g., by
production phagocyte oxidase in leukocytes peroxisomes radiation; from H2 O2 by
Fenton reaction; from O2

Mechanisms Conversion to H2 O2 and O2 by Sod Conversion to H2 O and O2 Conversion to H2 O by


of by catalase (peroxisomes), glutathione peroxidase
inactivation glutathione peroxidase
(cytosol, mitochondria)

Pathologic Stimulates production of degradative Can be converted to *OH Most reactive oxygen-
effects enzymes in leukocytes and other cells; and OCI_ , which destroy derived free radical;
may directly damage lipids, proteins, microbes and cells; can act principal ROS responsible
DNA; acts close to site of production distant from site of for damaging lipids,
production proteins, and DNA

51. C
helps to generate free radicals especially in neutrophils

52. C
Type I endometrial adenocarcinoma arises in a hyperplastic endometrium whereas
Type II may arise in an atrophic endometrium

53. C
In the habitual cigarette smoker, the normal ciliated columnar epithelial cells of the
trachea and bronchi are often replaced by stratified squamous epithelial cells. Stones
in the excretory ducts of the salivary glands, pancreas, or bile ducts, which are
normally lined by secretory columnar epithelium, may also lead to squamous
metaplasia by stratified squamous epithelium. A deficiency of vitamin A (retinoic acid)
induces squamous metaplasia in the respiratory epithelium.

54. B
Phospholipids derived myelin figures are more characteristic for irreversible than
reversible cell injury.

55. C
Cellular swelling and organelle swelling are signs of reversible cell injury

56. A
Influx of calcium into mitochondria increases its permeability thus causing a cascade
of changes responsible for cellular death.
57. B
Enzymatic digestion by both enzymes released from dying cells and inflammatory
cells causes lysis of the dead cells into a liquefied viscous mass- liquefactive necrosis.
Option A- Most common pattern of ischemic necrosis in all organs other than brain
Option C- Usually in vasculitis
Option D- Central necrosis leading to cheese like necrosis on gross

58. D
All sites are fat rich and hence digestion of fatty cells and their membranes followed
by saponification into calcium salts is relatively common in these sites.

59. A
Option B- Calcified bodies seen in congestive splenomegaly
Option C- Intracellular accumulation of Igs in plasma cells

60. A
Intact cell membrane and no inflammation is characteristic of apoptosis. Other
features are more classical for necrosis.

61. C
CK 8/18- Mallory hyaline

62. A, D
Option B- feature of necrosis more than apoptosis

63. B
Autophagy (“self-eating”) is the process in which the starved cell eats its own
components in an attempt to reduce nutrient demand to match the supply. Some of
the cell debris within the autophagic vacuoles may resist digestion and persist in the
cytoplasm as membrane-bound residual bodies. An example of residual bodies is
lipofuscin granules.

64. C
Options A, B, D are examples of change of columnar epithelium to squamous. However
GERD in esophagus causes change of squamous to columnar (called Intestinal
Metaplasia or Barret’s Esophagus)

65. C

Feature Necrosis Apoptosis

Cell size Enlarged (swelling) Reduced (shrinkage)

Nucleus Pyknosis → karyorrhexis → Fragmentation into nucleosome size fragments & condensation
karyolysis of chromatin

Plasma Disrupted Intact; altered structure, especially orientation of lipids


membrane

Cellular Enzymatic digestion; may Intact; may be released in apoptotic bodies


contents leak out of cell
Adjacent Frequent No
inflammation

Physiologic or Invariably pathologic Often physiologic, means of eliminating unwanted cells; may be
pathologic (culmination of irreversible pathologic after some forms of cell injury, especially DNA damage
role cell injury)

66. B
Cellular swelling is the first manifestation of almost all forms of injury to cells.
It is a difficult morphologic change to appreciate with the light microscope; it may be
more apparent at the level of the whole organ.
When it affects many cells, it causes some pallor, increased turgor, and increase in
weight of the organ.
On microscopic examination, small clear vacuoles may be seen within the cytoplasm;
these represent distended and pinched-off segments of the ER → Hydropic change or
vacuolar degeneration.
Swelling of cells is reversible.
Cells may also show increased eosinophilic staining, which becomes much more
pronounced with progression to necrosis

67. D
Necrotic cells show increased eosinophilia in hematoxylin and eosin (H & E) stains,
attributable in part to the loss of cytoplasmic RNA (which binds the blue dye,
hematoxylin) and in part to denatured cytoplasmic proteins (which bind the red dye,
eosin). The necrotic cell may have a more glassy homogeneous appearance than do
normal cells, mainly as a result of the loss of glycogen particles. When enzymes have
digested the cytoplasmic organelles, the cytoplasm becomes vacuolated and appears
moth-eaten.

68. D
Two phenomena consistently characterize irreversibility—the inability to reverse
mitochondrial dysfunction (lack of oxidative phosphorylation and ATP generation) even
after resolution of the original injury, and profound disturbances in membrane
function. Injury to lysosomal membranes results in the enzymatic dissolution of the
injured cell that is characteristic of necrosis.

69. D
Smac/Diablo, enter the cytoplasm, where they bind to and neutralize cytoplasmic
proteins that function as physiologic inhibitors of apoptosis (called IAPs).
The normal function of the IAPs is to block the activation of caspases, including
executioners like caspase-3, and keep cells alive. Thus, the neutralization of these
IAPs permits the initiation of a caspase cascade.
BAX and BAK are the two prototypic members of the BCL2 family.
BCL XL, MCL1 and BCL2 are the anti apoptotic members

70. C
Caspase 8 and 10 (in humans) are involved in the extrinsic pathway

71. A
The following features characterize necroptosis:
Morphologically, and to some extent biochemically, it resembles necrosis, both
characterized by loss of ATP, swelling of the cell and organelles, generation of ROS,
release of lysosomal enzymes and ultimately rupture of the plasma membrane as
discussed earlier.
Mechanistically, it is triggered by genetically programmed signal transduction events
that culminate in cell death. In this respect it resembles programmed cell death,
which is considered the hallmark of apoptosis
Because of the duality of these features, necroptosis is sometimes called programmed
necrosis to distinguish it from the more usual forms of necrosis driven passively by
toxic or anoxic injury to the cell.
In sharp contrast to apoptosis, the genetic program that drives necroptosis does not
result in caspase activation and hence it is also sometimes referred to as “caspase-
independent” programmed cell death.

72. C
Many pathogens are degraded by autophagy; these include mycobacteria, Shigella
spp., and HSV-1. This is one way by which microbial proteins are digested and
delivered to antigen presentation pathways. Macrophage-specific deletion of Atg5
increases susceptibility to tuberculosis.

73. A
Lipofuscin is not injurious to the cell or its functions. Its importance lies in its being a
telltale sign of free radical injury and lipid peroxidation. The term is derived from the
Latin ( fuscus, brown), referring to brown lipid. In tissue sections it appears as a
yellow-brown, finely granular cytoplasmic, often perinuclear, pigment.

74. D
There are four principal causes of hypercalcemia:
(1) increased secretion of parathyroid hormone (PTH) with subsequent bone
resorption, as in hyperparathyroidism due to parathyroid tumors, and ectopic secretion
of PTH-related protein by malignant tumors;
(2) resorption of bone tissue, secondary to primary tumors of bone marrow (e.g.,
multiple myeloma, leukemia) or diffuse skeletal metastasis (e.g., breast cancer),
accelerated bone turnover (e.g., Paget disease), or immobilization;
(3) vitamin D–related disorders, including vitamin D intoxication, sarcoidosis (in which
macrophages activate a vitamin D precursor), and idiopathic hypercalcemia of infancy
(Williams syndrome), characterized by abnormal sensitivity to vitamin D; and
(4) renal failure, which causes retention of phosphate, leading to secondary
hyperparathyroidism.
Less common causes include aluminum intoxication, which occurs in patients on
chronic renal dialysis, and milk-alkali syndrome, which is due to excessive ingestion of
calcium and absorbable antacids such as milk or calcium carbonate

75. A
Refer to answer above for details.

76. B
Refer to explanation above

77. D
Refer to explanation above

78. D
Option D is a stain used for Amyloid

79. A (single best answer)

80. A, C

81. B
Refer to the explanation provided before

82. B
Refer to the explanation provided above

83. A
Refer to the explanation provided above

84. A
Refer to the explanation provided above

85. A
Refer to the explanation provided above

86. A

87. A
Refer to the explanation provided above

88. A
Refer to the explanation provided above

89. A
Refer to the explanation provided above

90. B
Refer to the explanation provided above

91. A
Refer to the explanation provided above

92. D

93. C

94. D

95. C

96. D

97. A
98. B

99. D

100. C

101. D

102. D

103. B

104. C

105. B

106. B

107. C, D

108. A (single best option)

109. B

110. A
Inflammation, Repair and
Hemodynamics – Questions
111. Identify the arrow marked cell in the given condition below?

a. Macrophage
b. Lymphocyte
c. Plasma cell
d. Eosinophil

112. Which of the following cells will increase in case of parasite infection?

a. A
b. B
c. C
d. D

113. Vasodilation in acute inflammation is seen earliest in:


a. Venules
b. Arterioles
c. Capillaries
d. Vein

114. Increased permeability in acute inflammation is due to:


a. Histamine
b. IL-2
c. TGFβ
d. FGF
115. Sequence of events in acute inflammation:
a. Vasodilatation → Stasis → Transient vasoconstriction → Increased permeability
b. Transient vasoconstriction → Increased permeability → Stasis → Vasodilatation
c. Transient vasoconstriction → Vasodilatation → Stasis → Increased permeability
d. Transient vasoconstriction → Vasodilatation → Increased permeability → Stasis

116. The following is not an adhesion molecule?


a. Spectrin
b. Integrins
c. Selectins
d. Cadherin

117. The RBCs with schizonts of P. Falciparum are not visible on peripheral
blood smear due to which of the following reason?
a. Capillary adherence or sequestration of parasitized RBCs
b. ADCC mediated RBC destruction
c. Selective hemolysis of affected RBCs in spleen
d. Cellular lysis due to hemozoin produced by the parasites Erythrocyte changes in
Malaria

118. Amino acid which is useful in Neutrophil extracellular traps (NETs) as to


causes lysis of chromatin is?
a. Arginine
b. Alanine
c. Phenyl alanine
d. Valine
111. d 112. c 113. b 114. a 115. d 116. a 117. a 118. a

119. The following statements are true regarding neutrophil extracellular


trap:
a. Produced by neutrophils in response to infectious pathogens and inflammatory
mediators
b. Provide a high concentration of antimicrobial substances at sites of infection
c. Prevent the spread of the microbes by trapping them in the fibrils
d. All of the above

120. All of the following vascular changes are observed in acute inflammation,
except:
a. Vasodilation
b. Stasis of blood
c. Increased vascular permeability
d. Decreased hydrostatic pressure

121. Correct sequence in extravasation of leukocytes is:


a. Margination – rolling – adhesion – transmigration
b. Transmigration – margination – rolling – adhesion
c. Rolling – adhesion – transmigration – margination
d. Adhesion – transmigration – margination – rolling

122. Transmigration of WBC across the endothelium is called:


a. Margination
b. Rolling
c. Pavementing
d. Diapedesis

123. Which of the following is an alpha chemokine?


a. IL-8
b. MCP-1
c. Eotaxin
d. Lymphotactin

124. Which selectin is stored in weibel palade bodies?


a. Cadherins
b. P-Selectin
c. E-Selectins
d. L-Selectin

125. Which of the following enzymes are responsible for generating ‘oxygen
burst’ within neutrophils for killing intracellular bacteria?
a. Superoxide dismutase
b. Glutathione peroxidase
c. Oxidase
d. Catalase

126. Which of the following is an Opsonin?


a. C3a
b. C3b
c. C5a
d. LTC4

127. Which of the cell derived chemical mediators of inflammation is/are


preformed in secretory granules?(PGI)
a. Histamine
b. Lysosomal enzymes
c. Leukotrienes
d. Prostaglandins
e. Serotonin

128. Which of the following is a negative acute phase reactant?


a. Albumin
b. Haptaglobin
c. Ferritin
d. C reactive protein

129. Th 1 cells produce?


a. IL-1
b. IFN gamma
c. IL-4
d. IL-5

130. Primary granules of neutrophil have?


a. Proteinase 3
b. Alkaline phosphatase
c. Acid protease
d. Lactoferrin

131. Major basic protein is found in?


a. Macrophage
b. Eosinophils
c. Basophil
d. Neutrophil
119. d 120. d 121. a 122. d 123. a 124. b 125. c 126. b 127.
a,b,e

128. a 129. b 130. a 131. b

132. The following are labile cells except:


a. Hepatocytes
b. Bone marrow
c. Intestinal mucosa
d. Epithelium of skin

133. Liver biopsy in chronic RHF shows?


a. Shrinking
b. Greasy fatty liver
c. Hypertrophy
d. Congestion

134. Heart failure cells are:


a. Foam cells
b. Lipid laden macrophages
c. Hemosiderin laden macrophages
d. Type 1 pneumocytes

135. Hypercoagulability due to defective factor V gene is called:


a. Lisbon mutation
b. Leiden mutation
c. Antiphospholipid syndrome
d. Inducible thrombocytopenia syndrome

136. Which of the following is not a recognised function of thrombin?


a. Platelet activation
b. Adhesion of neutrophils to endothelium
c. Activation of monocytes
d. Inactivation of lymphocytes

137. Which of the following glycosaminoglycans is not produced in the golgi


apparatus?
a. Heparan sulphate
b. Dermatan sulphate
c. Keratan sulphate
d. Hyaluronan

138. Healing tissue has which type of collagen


a. I
b. II
c. III
d. IV

139. In mediating acute phase response of inflammation which of the


following does not play a major role?
a. IL-1
b. IL-6
c. IL-13
d. TNF

140. Which of the following is not a major pyrogen in inflammation?


a. IL-1
b. PGE2
c. TNF
d. IFN

141. The following histopathological picture is from a patient with cough,


night sweats and a cheesy white focus in left upper lobe of lung. The
principle cell involved here is
a. Lymphocyte
b. Histiocyte
c. Eosinophil
d. Multinucleated giant cell

142. Eosinophillia is found in?


a. Cryptococcus
b. HPV
c. Stronglyloides
d. Typhoid

143. Leukocyte migration through endothelium is induced by:


a. Selectin
b. N CAM
c. CAM
d. PECAM

144. Role of P-selectin in inflammation-


a. Rolling
b. Adhesion
c. Homing
d. Transmigration

145. What decreases during acute inflammation?


a. Ceruloplasmin
b. Transferrin
c. CRP
d. ESR
e. Albumin

132. a 133. d 134. c 135. b 136. d 137. d 138. c 139. c

140. d 141. b 142. c 143. d 144. a 145. e

146. Which of the following is the chief mediator associated with resetting the
hypothalamic temperature set point at a higher level, resulting in fever?
a. PGF2 alpha
b. PGE1
c. PGE2
d. PGI2

147. Which of the following is a major pyrogenic cytokine: (PGI)


a. IL-12
b. TNF
c. IFN-Y
d. Bradykin
148. Pain during inflammation is mediated by:
a. Nitric oxide
b. Leukotriene B4
c. Bradykinin
d. Chemokines

149. The following is not a chemoattactant in acute inflammation:


a. IL-8
b. C5a
c. LTB4
d. Kinins

150. Most important cytokine for the synthesis and deposition of connective
tissue proteins:
a. TGF-α
b. TGF-β
c. FGF-1
d. FGF-2

151. TNF and IL1 are produced by:


a. Neutrophils
b. Monocytes
c. Lymphocytes
d. Activated Macrophages

152. Stellate Granuloma are seen in:


a. Cat scratch disease
b. Sarcoidosis
c. LGV
d. Histoplasmosis

153. A 9 years old boy is admitted with acute abdominal pain localized in the
right illac fossa. He is pyrexia with localized peritonism in RIF. The causative
cell involved here:
a. Lymphocytes
b. Neutrophil
c. Macrophages
d. Monocytes

154. True statement about alternative macrophage activation (PGI type)


a. Induced by the cytokine IFN-y
b. Kill ingested organisms
c. Secrete cytokines that stimulate inflammation
d. Main function is in tissue repair
e. Anti inflammatory
155. The principle cell in granuloma
a. Fibroblast
b. Epithelioid cell
c. Giant cell
d. Plasma cell

156. Durck granuloma is seen in:


a. Congenital syphilis
b. Cat scratch disease
c. Histoplasmosis
d. Cerebral malaria

157. Hallmark of chronic inflammation:


a. Increased vascular permeability
b. Vasodilation
c. Granulomas
d. Tissue destruction

158. Granulomatous inflammation seen in A/E:


a. Syphilis
b. Tuberculosis
c. Sarcoidosis
d. AIDS

159. Necrotizing epitheloid cell granulomas are seen in all, except:


a. Tuberculosis
b. Wegener’s granulomatosis
c. Cat’s Scratch disease
d. Leprosy

160. Cell involved in the immediate phase of wound healing:


a. Platelets
b. Fibroblasts
c. Basophils
d. Macrophages

146. c 147. b, 148. c 149. d 150. b 151. d 152. a 153. b 154.


c a,d,e

155. b 156. d 157. d 158. d 159. d 160. a

161. Scar contraction is caused by?


a. Fibroblasts
b. Myofibroblasts
c. Epithelial cells
d. None
162. Find the false statement regarding wound healing:
a. Macrophages are the key cellular constituents
b. TGF-β is the most important fibrogenic agent
c. In remodelling type III collagen replaces type I collagen
d. Vitamin C is required for the hydroxylation of procollagen

163. In a sutured incised wound, re-epithelialization is complete by:


a. 24 hours
b. 48 hours
c. 5 days
d. 2 weeks

164. Permanent tissue is?


a. Heart
b. Liver
c. Kidney
d. Skeletal muscle

165. Which of the following DOES NOT present with recurrent episodes of
upper limb thrombosis:
a. Prostatic Cancer
b. Pancreatic Cancer
c. Osteosarcoma
d. Acute promyeloytic leukemia

166. Endothelium activation refers to:


a. Aberration of anatomy of vessel wall
b. Irreversible changes in functional state of vessel wall
c. Smooth muscle proliferation
d. Increased expression of adhesion molecules for leukocyte recruitment

167. Weibel-palade bodies are present in:


a. Vascular endothelial cells
b. Warthin finkeldey cells
c. Leydig cells
d. Dendritic cells

161. b 162. c 163. c 164. a 165. c 166. d 167. a


Inflammation, Repair and
Hemodynamics – Explanations
111. D
Plasma cell is a mononuclear cell with eccentric nucleus and a perinuclear halo.
Option A- mononuclear cell with abundant cytoplasm and pale nucleus
Option B- mononuclear cell with scanty cytoplasm and dense nucleus
Option D- bilobed nucleus with eosinophilic coarse granules

112. C
Option A- Neutrophil- increased in bacterial infections
Option B- Basophil- increased in myeloproliferative neoplasms
Option D- Lymphocyte- increased in viral infections

113. B
Reference: Robbins and Cotran Pathological Basis of Disease, 9th edition and Robbins
Basic Pathology, 10th edition
Vasodilation first involves the arterioles and then leads to opening of new capillary
beds in the area. The result is increased blood flow, which is the cause of heat and
redness (erythema) at the site of inflammation

114. A
Reference: Robbins and Cotran Pathological Basis of Disease, 9th edition and Robbins
Basic Pathology, 10th edition

115. D
Vasodilation (earliest in arterioles) → Increased permeability→ Exudative fluid leaks out
→ Edema → Slower blood flow → Hemoconcentration and Increased viscosity → Stasis
→ Margination

116. A
Spectrin is a part of the RBC cytoskeletal whereas the other options are cellular
adhesion molecules involved in various processes of cell growth, differentiation and
inflammation.

117. A
Reference: Robbins and Cotran Pathological Basis of Disease, 9th edition and Robbins
Basic Pathology, 10th edition
P. falciparum causes infected red cells to clump together (rosette) and to stick to
endothelial cells lining small blood vessels (sequestration), which blocks blood flow.
Several proteins, including P. falciparum erythrocyte membrane protein 1 (PfEMP1),
associate and form knobs on the surface of red cells. PfEMP1 binds to ligands on
endothelial cells, including CD36, thrombospondin, VCAM-1, ICAM-1, and E-selectin.
Red cell sequestration decreases tissue perfusion and leads to ischemia, which is
responsible for the manifestations of cerebral malaria, the major cause of death in
children with malaria

118. A
The process is thought to begin with NADPH oxidase activation of protein-arginine
deiminase 4 (PAD4) via reactive-oxygen species (ROS) intermediaries. PAD4 is
responsible for the citrullination of histones in the neutrophil, resulting in
decondensation of chromatin.

119. D
Neutrophil extracellular traps (NETs) are extracellular fibrillar networks that
concentrate anti-microbial substances at sites of infection and prevent the spread of
the microbes by trapping them in the fibrils.
They are produced by neutrophils in response to infectious pathogens (mainly bacteria
and fungi) and inflammatory mediators (e.g., chemokines, cytokines, and complement
proteins).
The extracellular traps consist of a viscous meshwork of nuclear chromatin that binds
and concentrates granule proteins such as anti-microbial peptides and enzymes.
NETs provide an additional mechanism of killing microbes that does not involve
phagocytosis.
In the process of NET formation, the nuclei of the neutrophils are lost, leading to the
death of the cells, sometimes called NETosis, representing a distinctive form of cell
death affecting neutrophils.
NETs also have been detected in the blood during sepsis.
The nuclear chromatin in the NETs, which includes histones and associated DNA, may
be a source of nuclear antigens in systemic autoimmune diseases, particularly lupus,
in which individuals react against their own DNA and nucleoproteins

120. D
Refer to the answer above (decreased oncotic pressure is seen leading to edema, not
decreased hydrostatic pressure)

121. A
122. D
Option A and C- arrangement of leucocytes towards the margins of the vessel
following increased permeability and loss of fluid
Option B- unstable tethering of the leucocyte over the endothelial surface

123. A
Chemokines are a family of small (8–10 kD) proteins that act primarily as
chemoattractants for specific types of leukocytes. About 40 different chemokines and
20 different receptors for chemokines have been identified. They are classified into
four major groups
C-X-C chemokines (alpha chemokines) have one amino acid residue separating the
first two of the four conserved cysteines. These chemokines act primarily on
neutrophils. IL-8 (now called CXCL8) is typical of this group. It is secreted by activated
macrophages, endothelial cells, and other cell types, and causes activation and
chemotaxis of neutrophils, with limited activity on monocytes and eosinophils. Its
most important inducers are microbial products and cytokines, mainly IL-1 and TNF.
C-C chemokines (beta chemokines) have the first two conserved cysteine residues
adjacent. The C-C chemokines, which include monocyte chemoattractant protein
(MCP-1, CCL2), eotaxin (CCL11), and macrophage inflammatory protein1α (MIP-1α,
CCL3), mainly serve as chemoattractants for monocytes, eosinophils, basophils, and
lymphocytes. Although most of the chemokines in this class have overlapping actions,
eotaxin selectively recruits eosinophils.
C chemokines (gamma chemokines) lack the first and third of the four conserved
cysteines. The C chemokines (e.g., lymphotactin, XCL1) are relatively specific for
lymphocytes.
CX3C chemokines (d chemokines) contain three amino acids between the CL1). This
chemokine exists infirst two cysteines. The only known member of this class is called
fractalkine (CX3) two forms: a cell surface-bound protein induced on endothelial cells
by inflammatory cytokines that promotes strong adhesion of monocytes and T cells,
and a soluble form, derived by proteolysis of the membranebound protein, that has
potent chemoattractant activity for the same cells

124. B

125. C
ROS are produced by the rapid
assembly and activation of a multicomponent enzyme, phagocyte oxidase (also called
NADPH oxidase), which oxidizes NADPH (reduced nicotinamide-adenine dinucleotide
phosphate) and, in the process, reduces oxygen to the superoxide anion (O2• ). In
neutrophils,this oxidative reaction is tightly linked to phagocytosis, and is called the
respiratory burst.

126. B
The major opsonins are immunoglobulin (Ig)G antibodies, the C3b breakdown product
of complement activation, and certain plasma lectins, notably mannose-binding lectin,
all of which are recognized by specific receptors on leukocytes

127. A, B, E
Cell derived mediators are rapidly released from intracellular granules (e.g., amines)
or are synthesized de novo (e.g., prostaglandins, leukotrienes, cytokines) in response
to a stimulus. The major cell types that produce mediators of acute inflammation are
tissue macrophages, dendritic cells, and mast cells, but platelets, neutrophils,
endothelial cells, and most epithelia also can be induced to elaborate some of the
mediators. Therefore, cell-derived mediators are most important for reactions against
offending agents in tissues. Plasma-derived mediators (e.g., complement proteins) are
present in the circulation as inactive precursors that must be activated, usually by a
series of proteolytic cleavages, to acquire their biologic properties. They are produced
mainly in the liver, are effective against circulating microbes, and also can be
recruited into tissues.

128. A
Negative acute-phase proteins
The liver responds by producing a large number of APRs. At the same time, the
production of a number of other proteins is reduced; these are therefore referred to as
“negative” APPs. Negative APPs are albumin, transferrin, transthyretin, transcortin, and
retinol-binding protein.
Positive acute-phase proteins
Positive APPs are CRP, D-dimer protein, mannose-binding protein, alpha 1 antitrpysin,
alpha 1 antichymotrypsin, alpha 2 macroglobulin, fibrinogen, prothrombin, factor VIII,
von-Willebrand factor, plasminogen, complement factors, ferritin, SAP complement,
SAA, ceruloplasmin (Cp), and haptoglobin (Hp).
Positive APPs serve different physiological functions for the immune system. Some act
to destroy or inhibit growth of microbes, e.g., CAA and Hp. Others give negative
feedback on the inflammatory response, e.g., serpins, alpha 2 macroglobulin and
coagulation factors affect coagulation. Positive APPs are produced during the APR
associated with anorexia and changed metabolism.

129. B

Major IFN-γ IL-4, IL-5, IL-13 IL-17, IL-22


cytokines
produced

Cytokines IFN-γ, IL-12 IL-4 TGF-β, IL-6, IL-1, IL-23


that induce
this subset

Immunological Macrophage activation, stimulation of Stimulation of lgE Recruitment of neutrophils,


reactions lgG antibody production production, activation monocytes
triggered of mast cells and
eosinophils

Host defense Intracellular microbes Helminthic parasites Extracellular bacteria, fungi


against

Role in Autoimmune and other chronic Allergies Autoimmune and other


disease inflammatory diseases (such as IBD, chronic inflammatory
psoriasis, granulomatous diseases (such as IBD,
inflammation) psoriasis, MS)

130. A
Azurophilic (primary)granules (formed in promyelocyte stage)
Lysosomal enzymes: acid hydrolases, acid phosphatase, β-glucuronidase, proteinase 3
Myeloperoxidase
Elastase
Arylsulfatase
Cationic antibacterial proteins
Specific (secondary) granules (formed in myelocyte stage)
Lysozyme
Lactoferrin
Collagenase
Plasminogen activator
Aminopeptidase

131. B
Larger granules of eosinophils contain- Major basic protein; Acid hydrolases;
Peroxidase;
Phospholipase; Cathepsin; Eosinophil cationic protein; Eosinophil-derived neurotoxin

132. A
Types Of Tissues In The Body

Labile (continuously dividing) Hematopoietic cells and surface epithelia

Stable (minimal proliferative activity, Parenchyma of most solid tissues (liver, kidney, pancreas), endothelial
quiescent stage G0) cells, fibroblasts, smooth muscle cells

Permanent tissues (terminally Majority of neurons and cardiac muscle cells (skeletal muscle cells too-
differentiated; non proliferative) but harbor adult tissue stem cells as well)

133. D
Congestion of the hepatic and
portal vessels may produce pathologic changes in the liver, the spleen, and the GI
tract. The liver is usually increased in size and weight (congestive hepatomegaly) due
to prominent passive congestion, greatest around the central veins. Grossly, this is
reflected as congested red-brown pericentral zones, with relatively normal-colored tan
periportal regions, producing the characteristic “nutmeg liver” appearance

134. C
Congested tissues take on a dusky reddish-blue color (cyanosis) due to red cell stasis
and the presence of deoxygenated hemoglobin. Microscopically, acute pulmonary
congestion exhibits engorged alveolar capillaries, alveolar septal edema, and focal
intraalveolar hemorrhage. In chronic pulmonary con gestion, which is often caused by
congestive heart failure, the septa are thickened and fibrotic, and the alve oli often
contain numerous hemosiderin-laden macrophages called heart failure cells

135. B
Approximately 2% to 15% of Caucasians carry a singlenucleotide mutation in factor V
that is called the factor V Leiden, after the city in The Netherlands where it was
discovered.

136. D
137. D
4 distinct families of GAGs
Heparan Sulphate
Chondroitin/Dermatan Sulphate
Keratan Sulphate
Hyaluronan (HA)
First three synthesized and assembled in the Golgi apparatus and RER as
proteoglycans
HA is produced at the plasma membrane by enzymes and is not linked to a protein
backbone

138. C
Option A- scar tissue
Option D- basement membrane

139. C
Acute-phase proteins are plasma proteins, mostly synthesized in the liver, whose
plasma concentrations may increase several hundredfold as part of the response to
inflammatory stimuli. Three of the bestknown of these proteins are Creactive protein
(CRP), fibrinogen, and serum amyloid A (SAA) protein. Synthesis of these molecules in
hepatocytes is stimulated by cytokines, especially IL6 (for CRP and fibrinogen) and IL1
or TNF (for SAA).

140. D
Bacterial products, such as LPS (called exogenous pyrogens), stimulate leukocytes to
release cytokines such as IL1 and TNF (called endogenous pyrogens) that increase the
enzymes (cyclooxygenases) that convert AA into prostaglandins. In the hypothalamus,
the prostaglandins, especially PGE2stimulate the production of neurotransmitters that
reset the temperature set point at a higher level.

141. B
The photograph shows central necrosis with lymphocytes, epithelioid histiocytes and
Langhan’s type of giant cell. Principle cell of a granuloma is an epithelioid histiocyte
formed by activation of macrophages.

142. C
Refer to answer above

143. D
Refer to the explanation provided above

144. A
Refer to the explanation provided above

145. E
Refer to the explanation provided above

146. C

147. B, C

148. C

149. D

150. B

151. D

152. A

153. B

154. A, D, E

155. B

156. D

157. D

158. D

159. D

160. A

161. B

162. C

163. C

164. A

165. C

166. D

167. A
Genetics – Questions
168. Frameshift mutation occurs due to?
a. Transition
b. Transversion
c. Insertion
d. Point mutation

169. Chromosome 21 is which type of chromosome?


a. Metacentric
b. Submetacentric
c. Short acrocentric
d. Medium acrocentric

170. Gene silencing RNA-


a. rRNA
b. tRNA
c. miRNA
d. None

171. Y chromosome is?


a. Metacentric
b. Submetacentric
c. Acrocentric
d. Telocentric

172. A mutation in which the base replacement changes the codon for one
amino acid to another is called a
a. Missense mutation
b. Nonsense mutation
c. Silent mutation
d. Frame shift mutation

173. Regarding the centimorgan-


a. The human genome is composed of approximately 6000 centimorgans in
recombination distance
b. It is a measure of genetic distance that reflects the probability of a crossover
between two loci during meiosis
c. Once centimorgan approximates a 5% chance of a crossover during meiosis
d. The average chromosome contains about 500 centimorgans of genetic material

174. Methylation of cytosine (CpG) associated with-


a. Increased expression of protein
b. Decreased expression of protein
c. Point Mutation
d. Change in the function of protein

175. Which of the following disease is X-linked?


a. Thalassemia
b. Color blindness
c. Sickle cell anemia
d. Galactosemia

176. All of the following are autosomal dominant, except?


a. Familial hypercholesterolemia
b. Congenital adrenal hyperplasia
c. Achondroplasia
d. Acute intermittent porphyria

177. Find correct match among the following?


a. Hypophosphatemic rickets-X linked dominant
b. Duchenne muscular dystrophy-X linked recessive
c. Sickle cell: AR
d. Osteogenesis imperfecta-1 : AR
e. Achondroplasia : AR

178. ABO blood group is an example of?


a. Co-dominance
b. AD
c. AR
d. Mitochondrial inheritance

179. Following is transmitted as autosomal dominant disorder-


a. Albinism
b. Sicklecell anemia
c. Hereditary spherocytosis
d. Glycogen storage disease
168. c 169. c 170. c 171. c 172. a 173. b 174. b 175. b 176. b

177. 178. a 179. c


a,b,c

180. Neurofibromatosis I is-


a. AD
b. AR
c. X linked recessive
d. X linked dominant

181. Autosomal dominant inheritance-


a. Albinism
b. Sickle cell anemia
c. Thalassemia
d. Hereditary spherocytosis

182. Inheritance of Huntington’s chorea-


a. AD
b. XR
c. AR
d. XD

183. The inheritance pattern of familial retinoblastoma is-


a. Autosomal recessive
b. Autosomal dominant
c. X-linked dominant
d. X-linked recessive

184. Type of inheritance in tuberous sclerosis-


a. Autosomal dominant
b. Autosomal recessive
c. X-linked dominant
d. X-linked recessive

185. Type of inheritance in Wilson’s disease-


a. Autosomal dominant
b. Autosomal recessive
c. X-linked dominant
d. X-linked recessive

186. Hemophilia is associated with-


a. X chromosome
b. Y chromosome
c. Chromosome 3
d. Chromosome 16

187. Which of the following is not X-linked condition?


a. Duchenne muscular dystrophy
b. Emery-Dreifuss muscular dystrophy
c. Facioscapulohumeral muscular dystrophy
d. Becker muscular dystrophy

188. Duchenne muscular dystrophy inherited as-


a. X-linked
b. Autosomal dominant
c. Autosomal recessive
d. Codominant

189. Which of the following is the gene for Duchenne muscular dystrophy?
a. DMPK gene
b. STK11 gene
c. PTCH gene
d. Dystrophin gene

190. Locations of gene which codes for alpha I anti-trypsin deficiency?


a. 14q
b. 13p
c. 17p
d. 14p

191. Waardenburg syndrome is due to mutation of:


a. PAX2 gene
b. PAX3 gene
c. PAX6 gene
d. PAX9 gene

192. Mutation leading to sickle cell anemia-


a. Crossover mutation
b. Frame shift
c. Deletion
d. Non-disjunction
e. Point mutation

193. The chances of having an unaffected baby, when both parents have
Achondroplasia are-
a. 0%
b. 25%
c. 50%
d. 100%

194. Parents are carrier of an autosomal recessive disorder. Chances of


offspring to get affected-
a. 1/1
b. 1/3
c. 1/2
d. 1/4

195. Chance of having cystic fibrosis if only one parent is affected and the
other is normal-
a. 25%
b. 50%
c. 0%
d. 80%
180. a 181. d 182. a 183. b 184. a 185. b 186. a 187. c 188. a
189. d 190. a 191. b 192. e 193. b 194. 195. c

196. If both parents have sickle cell anemia patients, then the likehood of
offsprings having the disease is-
a. 10%
b. 25%
c. 50%
d. 100%

197. Only Males are affected in which of the following pedigree:


a. Autosomal dominant
b. Autosomal recessive
c. Y-linked disorder
d. X-linked recessive

198. Retinitspigmentosa inherited as-


a. AD
b. AR
c. XR
d. Digenic

199. True statement about inheritances of an X-linked recessive trait is-


a. 50% of boys of carrier mother are affected
b. 50% of girls of diseased father are carrier
c. Father transmits disease to the son
d. Mother transmits disease to the daughter

200. Most severe type of Ehlers danlos syndrome?


a. Type 1
b. Type 4
c. Type 8
d. Type 2

201. Osteogenesisimperfecta defect occurs in-


a. Collagen type I
b. Elastin
c. Collagen type IV
d. Basement membrane

202. Hypermobility of joint and hyperelasticity skin is seen in-


a. Marfansyndrome
b. Ehlers-Danlossyndrome
c. Fragile X-syndrome
d. Angelmansyndrome-

203. In Marfan syndrome, the defect in-


a. Fibrillin 1
b. Collagen
c. Fibrillin II
d. Elastin

204. Gene involved in Rett syndrome-


a. P53
b. MECP2
c. RB
d. BRCA

205. The gene that regulates normal morphogeneis during development is-
a. FMR-l gene
b. Homeoboxgene
c. P-16
d. PTEN

206. HOX gene mutation can cause all except-


a. Syndactyly
b. Polydactyly
c. Fused carpal bones
d. VSD

207. Which one of the following is due to point mutation-


a. Color blindness
b. Sickle cell anemia
c. Diabetes
d. Porphyria

208. The following are single gene disorders except-


a. Hemochromatosis
b. Cystic fibrosis
c. William’s syndrome
d. Huntington’s disease

209. Chromosome involved in friedreisch ataxia:


a. 9q
b. 19q
c. 17p
d. X chromosome

210. Anticipation refers to


a. Waiting for a disease to manifest itself in an individual such as in Huntington’s
chorea
b. Worsening of a disease phenotype over generations within a family
c. Birth of a fetus with positive prenatal tests for genetic abnormalities
d. Expression of a permutation.

211. Anticipation is seen in-


a. Translocation
b. Chromosome breaking
c. Trinucleotide-repeat expansion
d. Mitochondrial mutation
196. d 197. c 198. 199. a 200. b 201. a 202. b 203. a 204. b

205. b 206. d 207. b 208. c 209. a 210. a 211. c

212. True about fragile X syndrome-


a. Triple nucleotide repeat sequence
b. Chromosome breaking
c. Mitochondrial mutation
d. Centromere absent

213. Premutation is seen in:


a. Genomic imprinting
b. Trinucleotide repeat mutation
c. Mitochondrial mutation
d. Gonadal mosaicism

214. All are true about Fragile X syndrome except-


a. Large head
b. Large nose
c. Large ear
d. Large testis

215. One of the following disorders is due to maternal disomy of chromosome


15-
a. Prader-Willisyndrome
b. Angelmansyndrome
c. Hydatidiform mole
d. Klinefelter’s syndrome

216. True about genomic imprinting-


a. Differential expression of gene depending on parent of origin
b. Prader-Willi syndrome is maternal deletion of chromosome 15
c. Angelman syndrome is due to paternal deletion of chromosome 15
d. Uniparentaldisomy is other name of genomic imprinting

217. These syndromes are associated uniparentaldisomy except-


a. Angelman
b. Praderwillisyndrome
c. Russel silver
d. Bloom Syndrome

218. Paternal 15 chromosome deletion is seen in-


a. Angelmansyndrome
b. Prader-Willisyndrome
c. Down syndrome
d. Turner syndrome

219. Angelman syndrome is due to-


a. Digenicinheritance
b. Inversion
c. Uniparentaldisomy
d. Mitochondrialdisorder

220. Mitochondrial chromosomal abnormality leads to-


a. Lebershereditaryopticneuropathy
b. Angelmansyndrome
c. Prader-Willi syndrome
d. Myotonicdystrophy

221. Type of inheritance in Melas-


a. AD
b. AR
c. Mitochondrial
d. X-linked

222. Pedigree analysis-analyze the following pedigree and give the mode of
inheritance-

a. Autosomal recessive
b. Autosomal dominant
c. Mitochondrial inheritance
d. X linked dominant

223. False statements about fragile X syndrome is-


a. Breakage in long arm of X chromosome
b. Common genetic disorder
c. Micro-orchidism
d. Long face

224. Which chromosome is responsible for the production of MIF?


a. Chromosome 16
b. Chromosome 22
c. X Chromosome
d. Y Chromosome
212. a 213. b 214. b 215. a 216. a 217. d 218. b 2191. 220. a
c

221. c 222. c 223. c 224. b

225. Velocardiofacial defect is due to mutation in?


a. Chr 11
b. Chr 13
c. Chr 22
d. Chr 3

226. Cri du chat syndrome is?


a. 22q
b. 5q
c. 22p
d. 5p

227. If a chromosome divides in an axis perpendicular to usual axis of division


it is going to form:
a. Ring chromosome
b. Isochromosome
c. Acrocentric chromosome
d. Subtelocentric chromosome

228. Most common microdeletion syndrome is?


a. WAGR syndrome
b. Velo-cardio-facial syndrome
c. PraderWilli syndrome
d. Angelman syndrome

229. Abnormal chromosomal number which is exact multiple of the 23 is-


a. Euploidy
b. Aneuploidy
c. Mosaicism
d. Trisomy

230. Edward’s syndrome is-


a. Trisomy 21
b. Trisomy 18
c. Trisomy 13
d. 5P

231. Patau’s syndrome is due to-


a. Trisomy 21
b. Trisomy 18
c. 45Xo
d. Trisomy 13

232. Down’s syndrome increases risk of: (PGI)


a. Hirschprung’s disease
b. Leukemia
c. Sensorineural hearing loss
d. Hyperthyroidism
e. Atlanto-occipital dislocation

233. Chromosomal abnormalities in Down syndrome is/are due to: (PGI)


a. Nondisjunctional of maternal chromosome
b. Nondisjunctional of paternal chromosome
c. Translocations between chromosome 21 & 14
d. Disjunction of paternal chromosome
e. Mosaicism

234. True about Lyonization of X chromosome:


a. Inactivation of X chromosome only in somatic cell
b. Inactivation of X chromosome only in germ cell
c. Inactivation of X chromosome in somatic & germ cell both
d. Maximum number of Barr body is equal to X chromosome

235. Which chromosome contains the most known genetic disease of any
human chromosome?
a. 1
b. 9
c. 11
d. 6

236. Down syndrome is most commonly caused by-


a. Maternalnondisjunction
b. Paternalnondisjunction
c. Translocation
d. Mosaicism

237. Down syndrome is due to-


a. Trisomy 21
b. Translocation 13-15/21
c. Translocation 22/21
d. All of the above

238. Which of the following is not associated with Down syndrome?


a. Trisomy 21
b. Mosaic 21
c. Translocation t (15,21), t (22,21)
d. Deletion of 21

239. All of the following are characteristic of Turner syndrome except-


a. Webbing of neck
b. Cubitus valgus
c. Umbilical hernia
d. Coarctation of aorta

240. Blood group antigen A and B are associated with which chromosome-
a. Chromosome 1
b. Chromosome 5
c. Chromosome 9
d. Chromosome 19
225. c 226. d 227. b 228. b 229. b 230. b 231. d 232.
a,bc,e

233. a, 234. c 235. 236. a 237. d 238. d 239. c 240. c


c,e

241. Karyotype in Klinefelter’s syndrome is-


a. 47XXY
b. 46XXY
c. 445XO
d. 45XXX

242. Which of the following is not the karyotype of Turner syndrome?


a. 46, XX, i (Xq)
b. 45, XO
c. 46, XX, I (X p)
d. 46, XX, r (X)

243. A nineteen-year old female with short stature, widespread nipples and
primary amenorrhoea most likely has a karyotype of-
a. 47, XX + 18
b. 46, XXY
c. 47, XXY
d. 45 X

244. Males who are sexually underdeveloped with rudimentary testes and
prostate glands, sparse pubic and facial hair, long arms and legs and large
hands and feet are likely to have the chromosome-
a. 45, XYY
b. 46, XY
c. 47, XXY
d. 46, X

245. Streak gonads are seen in-


a. Turner syndrome
b. Klinefelter’ssyndrome
c. Patau’ssyndrome
d. Down syndrome

246. Barr body is absent in-


a. Klinefelter’s syndrome
b. Turner’s syndrome
c. Super female
d. None

247. Condition not associated with increased risk of cancer are all except?
a. NF-1
b. Turners syndrome
c. Down syndrome
d. Chr 13 deletion

248. A child with a small head, minor anomalies of face including a thin upper
lip, growth delay and development disability can have all of the following
except-
a. A chromosomal syndrome
b. A teratogenic syndrome
c. A mendelian syndrome
d. A polygenic syndrome

249. Which of the following is true regarding ataxia telanglectasia?


a. It is X linked recessive disease
b. It is autosomal recessive disorder
c. It is associated with increased levels of IgM
d. None of the above

250. Xerodermapigmentosa all true except:


a. Autosomal dominant
b. Onset of squamous cell ca by 2nd-3rd decade
c. Develops non melanotic cancer by the age of 9 years
d. Nucleotide excision repair defect

251. An obese women with T2 diabetes and hypertension had endometrial ca.
Which of the following gene is involved?
a. PTEN
b. P53
c. B CATENIN
d. SMAD

252. BRCA1 is the most common gene mutated in familial breast cancer. This
gene is located on which chromosome?
a. 13
b. 17
c. 20
d. 21

253. BRCA1 and BRCA2 is responsible for all these Cancers except:
a. Breast
b. Ovary
c. Intestinal lining cancer
d. Prostate

254. Multifactorial inheritance is most likely to play a significant role in the


appearance of-
a. Achondroplasia
b. Lysosomal storage disease
c. Cleft lip
d. Huntington disease
241. a 242. c 243. d 244. c 245. a 246. b 247. d 248. d

249. b 250. a 251. 252. b 253. c 254. c

255. Which of the following is a DNA repair defect?


a. Bloom syndrome
b. Incontinentiapigmenti
c. Aplastic anemia
d. Tuberous sclerosis

256. Obesity with mental retardation is seen in:


a. Prader-Willisyndrome
b. Turner’ssyndrome
c. Fragile X syndrome
d. Noonamsyndrome

257. Multifactorial inheritance is seen in:


a. Neurofibroma
b. Hemophilia
c. Cardiac septal defects
d. Hypophosphatemic rickets
258. The best suited nucleated cell for chromosomal study-
a. Polymorphs
b. Lymphocytes
c. Epithelial cells
d. Langerhan’s cell

259. Which of the following not used to study epigenetics?


a. Chip on chip
b. Bisulfite sequencing
c. HPLC
d. Methylation-sensitive restriction enzymes

260. Which of the following procedures as routine technique for karyotyping


using light microscopy?
a. C-banding
b. G-banding
c. Q-banding
d. Ethidium bromide staining

261. Karyotyping of the fetus is done in all except-


a. Chorionic villi sampling
b. Cordocentesis
c. Amniocentesis
d. Fetal skin biopsy

262. Microarray is-


a. Study of multiple genes
b. Study of disease
c. Study of organisms
d. Study of blood group

263. Chromosomes are visualised through light microscope with resolution of-
a. 5kb
b. 50mb
c. 5mb
d. 500kb

264. Karyotyping is done in which phase of cell cycle-


a. Anaphase
b. Metaphase
c. Telophase
d. S phase

265. Feulgen reaction is a cytochemical test for-


a. Mrna
b. Trna
c. DNA
d. All of the above
255. a 256. a 257. c 258. 259. b 260. b 261. d 262. d

263. a 264. 265. b


Genetics – Explanations
168. C
A transition is a point mutation that replaces a purine-pyrimidine base pair with a
different purine-pyrimidine base pair. For example, an A-T base pair becomes a G-
C base pair.
A transversion is a point mutation that replaces a purine-pyrimidine base pair with a
pyrimidine-purine base pair. For example, an A-T base pair becomes a T-A or a C-G
base pair.
Frameshift/in-frame: addition or deletion of base(s) - Usually nonfunctional; often
shorter than normal

169. C
Chromosome 21 is Short-Acrocentric. This method of classification is not used now-a-
days.
Translocation between two acrocentric chromosomes leads to Down’s syndrome
Chromosomes 13,14,15 (medium acrocentric), 21,22,Y (Short scrocentric)

170. C
miRNA
The miRNAs do not encode proteins; instead, they function primarily to modulate the
translation of target mRNAs into their corresponding proteins.
Posttranscriptional silencing of gene expression by miRNA is a fundamental and well-
conserved mechanism of gene regulation present in all eukaryotes (plants and
animals).

171. C
Chromosomes.
Acrocentric. If the p (short) arm is so short that it is hard to observe, but still present,
then the chromosome is acrocentric
The human genome includes five acrocentric chromosomes: 13, 14, 15, 21, 22 and Y
X chromosome is sub-metacentric

172. A
Mutation
Silent: new codon specifies same amino acid
Missense: new codon specifies different amino acid
Nonsense: new codon is stop codon

173. B
CentiMorgan (cM)
The recombination frequency provides a measure of genetic distance between any
pair of linked loci. is distance is expressed in centimorgans.
One centimorgan is equal to 1% recombination frequency/ crossing over
1 cM is approximately equal to 1 million base pairs of DNA (1 Mb)
Female genome is approx 4500 cM long and male genome is approx. 3000 cM long

174. B
DNA Methylation
DNA methylation is an epigenetic mechanism that occurs by the addition of a methyl
(CH3) group to DNA, thereby often modifying the function of the genes and affecting
gene expression.
DNA methylation process is the covalent addition of the methyl group at the 5-carbon
of the cytosine ring resulting in 5-methylcytosine (5-mC), also informally known as the
“fifth base” of DNA.
These methyl groups project into the major groove of DNA and inhibit transcription.

175. B

176. B

177. A, B, C
Hypophosphatemic rickets – Has both X-linked dominant (M/C) and X linked recessive
pattern of inheritance.

178. A
Co-Dominant pattern.
Codominance occurs when two versions, or “alleles,” of the same gene are present in a
living thing, and both are expressed.
Instead of one trait being dominant over the other, both traits appear
Eg., ABO blood group and MHC are co-dominant.
Let’s see AB blood group – where A and B are dominant alleles but both of them
manifest in AB blood group

179. C

180. A

181. D

182. A

183. B

184. A

185. B
186. A

187. C
Facioscapulohumeral muscular dystrophy
FSHD1 is inherited in an autosomal dominant pattern
FSHD2 is inherited in a digenic pattern – which involves mutation in two genes

188. A
For all the above questions on inheritance – There are few must know inheritance
patterns below. Try to read them and you will know more inheritance as you read
individual diseases
Autosomal Dominant

System Disorders Disorder

Nervous Huntington disease

Neurofibromatosis

Myotonic dystrophy

Tuberous sclerosis

Urinary Polycystic kidney disease (AD)

Gastrointestinal Familial polyposis coli

Hematopoietic Hereditary spherocytosis

von Willebrand disease

Eye Familial Retinoblastoma

Skeletal Marfan syndrome

Osteogenesisimperfecta

Achondroplasia

Metabolic
Familial hypercholesterolemia

Acute intermittent porphyria

Autosomal Recessive Disorders

System Disorders

Metabolic Cystic fibrosis

Phenylketonuria

Galactosemia

Homocystinuria

α1-Antitrypsin deficiency

Wilson disease

Hemochromatosis
Hematopoietic Sickle cell anemia, Thalassemias

Endocrine Congenital adrenal hyperplasia

Skeletal Ehlers-Danlos syndrome (some variants) Alkaptonuria

Nervous
Neurogenic muscular atrophies

Friedreich ataxia

Spinal muscular atrophy

X-Linked recessive disorders

System Disease

Musculoskeletal Duchenne muscular dystrophy

Blood Hemophilia A and B, Chronic granulomatous disease (membrane NADPH oxidase defect)
Glucose-6-phosphate dehydrogenase deficiency

Eye Colour blindness

Immune Agammaglobulinemia, Wiskott-Aldrich syndrome

Metabolic Diabetes insipidus, Lesch-Nyhan syndrome

Nervous Fragile X syndrome

189. D. Dystrophin gene.


DMD
Duchenne and Becker muscular dystrophy are caused by loss-of-function mutations in
the dystrophin gene on the X chromosome.
Dystrophin is one of the largest human genes, spanning 2.3 million base pairs and
composed of 79 exons

190. A
α1-Antitrypsin deficiency is an autosomal recessive dis- order of protein folding
marked by very low levels of circulating α1-Antitrypsin (α1AT)
α1AT is a small 394–amino acid plasma glycoprotein synthesized predominantly by
hepatocytes
The gene, located on chromosome 14,
The patients develop liver cirrhosis and pan acinar type of emphysema.

191. B
Waardenburg syndrome is a group of genetic conditions that can cause hearing loss
and changes in coloring (pigmentation) of the hair, skin, and eyes
Type III (sometimes called Klein-Waardenburg syndrome) includes abnormalities of the
arms and hands in addition to hearing loss and changes in pigmentation
Klein-Waardenburg syndrome (WS-III) is one such developmental disorder resulting
from a mutation in a PAX 3 gene

192. E
Sicklecellanemia
Mutation in sicklecellanemiais a point mutation
It’s a mutation in the beta globinchain in the 6th position – which changes glutamine
to valine.
The nextfewquestions are on recurrentriskcalculation. Follow the basicrules for solving
the problems.
AD – Oneparentaffected – chance of diseasein offsprings – 50%
AD – Bothparentsaffcted – chance of disease inoffsprings – 75%
AR – Bothparentscarriers - chance of diseasein offsprings – 25%
AR – Oneparentisaffected, otherparentisnormal
chance of disease in offspring – 0%
chance of carrier state in offspring – 100%
XLR – Momcarrier, dadnormal
Chance of disease in sons - 50%
Chance of carrier state in daughters – 50%

193. B
Achodroplasiaisautosomaldominant.
So as per the pointsdiscussedabove, the riskwill be – 25% (unaffectedkid)

194. ¼
Autosomal recessive inheritance the chance of the offspring having the disease is 1/4

195. C
Cystic fibrosis is autosomal recessive disease.
So as per the pointsdiscussedabove, the riskwill be – 0% (unaffectedkid)

196. D
Sickle cell anemia is autosomal recessive.
Both parents are having the disease → Means both parents are having both alleles
mutated.
So the offsprings will get one affected allele from each parent – So all offsprings will
be affected

197. C
The best answer here will be Y linked disorder.
In XLR also males are predominantly affected. Females having the disease in XLR – is
very very rare. (but possible), so the best answer here will be Y linked

198. AR
Retinitis Pigmentosa.
Retinitis pigmentosa isinheritedas AD, AR and X linked. So wehave to go with most
common mode asanswer for the question.
This question has various answers. Standard textbookalsovaries. But american
academy of opthalmologygoes with AR, so we are going with AR as the most common
mode of inheritance

199. A
Just see the rules of inheritance in the beginning of the session.

200. B
Most sever type ofEhlerdanlossyndrome is Type 4
Type 4 is vascular type ofEhlerDanlosSyndrome.
Type 4 – causesaorticdissection and death, thusmaking it the most severe type.

201. A
OsteogenesisImperfecta
Osteogenesisimperfecta (OI), also known as brittle bone disease, is a group of genetic
disorders that mainly affect the bones.
Osteogenesisimperfecta (OI) is a disorder of bone fragility chiefly caused by mutations
in the COL1A1 and COL1A2 genes that encode type I collagen

202. B
Ehlers-Danlos syndrome (EDS) is a group of inherited connective tissue diseases that
have in common a defect in collagen structure or synthesis. Clinically, the disease
causes hyperextensible skin that is easily traumatized and hyperextensible joints
secondary to effects on the joints and adjacent ligaments.
Types of RDS
Kyphoscoliotic EDS: autosomal recessive form
Vascular variant EDS: autosomal dominant form that causes rupture of vessels and
bowel wall
Classical EDS: autosomal dominant form that causes a type V collagen defect; patients
have a normal lifespan

203. A
Marfan syndrome
Marfan syndrome is due to a mutation of the fibrillingene (FBN1) on chromosome
15q21.
Fibrillin is a glycoprotein that functions as a scaffold for the alignment of elastic fibers.

204. B
Rett Syndrome
Rett syndrome is a neurodevelopmental disorder that affects girls almost exclusively.
It is characterized by normal early growth and development followed by a slowing of
development, loss of purposeful use of the hands, distinctive hand movements, slowed
brain and head growth, problems with walking, seizures, and intellectual disability
Rett syndrome are caused by a mutation in the methyl CpG binding protein 2, or
MECP2 gene

205. B
HOX genes.
The regulatory proteins themselves are encoded by genes called homeobox(HOX) or
homeotic genes. Another closely related set of genes is the PAX (paired-box) genes.
Sequential and coordinated gene expression is necessary for proper tissue and cell
differentiation during embryonic life

206. D
Hox genes mutations generally cause abnormalities in the limb orientation and
differentiation
There are more than 35 different syndromes which result in mutation of various HOX
genes, most of them will have limb abnormalities

207. b
Sicklecellanemia
Mutation in sicklecellanemiais a point mutation
It’s a mutation in the beta globinchain in the 6th position – which changes glutamine
to valine

208. C
William syndrome.
It’s not a single gene disorder.
Most cases of Williams syndrome are not inherited but occur as random events during
the formation of reproductive cells (eggs or sperm) in a parent of an affected
individual. These cases occur in people with no history of the disorder in their family.
Williams syndrome is considered an autosomal dominant condition because one copy
of the altered chromosome 7 in each cell is sufficient to cause the disorder.

209. A
Freidriech Ataxia
Mutations in the FXN gene cause Friedreich ataxia. This gene provides instructions for
making a protein called frataxin
The gene is located in chromosome 9q
Patients develop impaired muscle coordination (ataxia) that worsens over time

210. B
Anticipation
The signs and symptoms of some genetic conditions tend to become more severe and
appear at an earlier age as the disorder is passed from one generation to the next.
Occurs with disorders that are caused by an unusual type of mutation called a
trinucleotide repeat expansion
The pathogenesis is dynamic here and it the repeats expand during gametogenesis.

211. C (See above question)

212. A
Fragile X syndrome is a genetic condition that causes a range of developmental
problems including learning disabilities and cognitive impairment
Fragile X is because of CGG repeats in FMR1 gene.
It has XLR inheritance pattern.
Macro-orchidism is one of the salient features

213. B
Premutation
A premutation is a situation in which there are an excess number of repeats in a gene
that is at risk of increasing in length during reproduction but which does not cause
disease in the person with the excess number of repeats
Seen in trinucleotide repeat disorders.

214. B
Features of Fragile X syndrome
Affected males are mentally retarded, with an IQ in the range of 20 to 60.
They express a characteristic physical phenotype that includes a long face with a large
mandible, large everted ears, and large testicles (macro-orchidism).
Hyperextensible joints, a high arched palate, and mitral valve prolapse

215. A
Prader- Willi syndrome
Deletion of band q12 on long arm of paternal chromosome 15 occurs or maternal
disomy of Ch 15
Genes in this region of maternal chromosome 15 are imprinted so there is complete
loss of their functions.
Patients have mental retardation, short stature, hypotonia, hyperphagia, small hands
and feet, and hypogonadism.

216. A
Imprinting
Imprinting involves transcriptional silencing of the paternal or maternal copies of
certain genes during gametogenesis.
For such genes, only one functional copy exists in the individual. Loss of the functional
(not imprinted) allele by deletion gives rise to diseases.

217. D.
Examples for imprinting disorders
Angelmansyndrome
Praderwillisyndrome
Silver Russel syndrome
Beckwithwidemansyndrome
Pseudo-Hypoparathyroidism

218. B
Prader- Willi syndrome
Deletion of band q12 on long arm of paternal chromosome 15 occurs or maternal
disomy of Ch 15

219. C
Inheritance of both chromosomes of a pair from one parent is called
uniparentaldisomy.
Angelman syndrome, can result from uniparentaldisomy of paternal chromosome 15.

220. A
Leber hereditary optic neuropathy is a prototype of mitochondrial disorder.
It is a neurodegenerative disease that manifests as a progressive bilateral loss of
central vision.
Visual impairment is first noted between ages 15 and 35, leading eventually to
blindness.

221. C
Mitochondrial encephalomyopathy, lactic acidosis, and stroke-like episodes (MELAS)
it’s a mitochondrial inherited disorder

222. C
In the above pedigree
When mom is affected all kids are affected
When dad is affected none are affected
This is classical of mitochondrial inheritance.

223. C
Fragile X patients express a characteristic physical phenotype that includes a long face
with a large mandible, large everted ears, and large testicles (macro-orchidism).

224. B
MIF – Macrophage Migration Inhibitory factors is located in Chromosome 22.
Mutation in this is known to be associated with Juvenile Rheumatoid arthritis

225. C. Chromosome 22
Velocardio facial syndrome
Chromosome 22q11.2 deletion syndrome encompasses a spectrum of disorders that
result from a small deletion of band q11.2 on the long arm of chromosome 22.
Clinical features include congenital heart defects, abnormalities of the palate, facial
dysmorphism, developmental delay, and vari- able degrees of T-cell immunodeficiency
and hypocalcemia.
Previously, these clinical features were considered to rep- resent two different
disorders—DiGeorge syndrome and velocardiofacial syndrome.

226. D. 5p
Cri Du chat Syndrome.
Deletion of chromosome 5p
High pitched cry
Congenital heart defects
Mental retardation, microcephaly
Can be diagnosed by FISH and Karyotyping

227. B – Isochromosome
An isochromosome has morphologically identical genetic information in both arms.
The most common isochromosome present in live births involves the long arm of the X
and is designated i(X)(q10)
It is as a result of the plane of separation being horizontal than vertical.

228. B
Microdeletion syndromes.
Usually involves less than 5Mb base pairs
Most common micro-deletion syndromes isDigeorge (Velocardiofacial syndrome)
Microdeletion syndromes are diagnosed by FISH

229. B
Any exact multiple of the haploid number of chromosomes (23) is called euploid.
If an error occurs in meiosis or mitosis and a cell acquires a chromosome complement
that is not an exact multiple of 23, it is referred to as aneuploidy.

230. B
Trisomy 18: Edwards syndrome

231. D
Trisomy 13 – Patau Syndrome
232. A, B, C, E
Thyroid and Down’s
About half of the patients with Down syndrome have been shown to have subclinical
hypothyroidism with elevated TSH and normal thyroxine levels.
Hyperthyroidism is much less frequent in patients with Down syndrome as compared
to hypothyroidism

233. A,C,E
Pathogenesis of Down’s
Most common cause of Down’s is meiotic nondisjunction of chromosome 21 occurs in
the ovum.
4% of cases of Down syndrome, the extra chromosomal material derives from the
presence of a rob- ertsonian translocation of the long arm of chromosome 21 to
another acrocentric chromosome (e.g., 22 or 14)
1% of Down syndrome patients are mosaics, having a mixture of cells with 46 or 47
chromosomes.
234. C
Lyonisation
X inactivation, occurs in the blastocyst (~100 cells) during the development of female
embryos
It’s random inactivation
It’s incomplete
X-chromosome inactivation is permanent in somatic cells and reversible in developing
germ line cells. Both X chromosomes are active during oogenesis.

235. 1
Chromosome 1
Chromosome 1 is the largest human chromosome, containing about 8% of the entire
genome
The chromosome carries an estimated 3,141 genes
More than 350 diseases linked to chromosome 1

236. A
Pathogenesis of Down’s
Most common cause of Down’s is meiotic nondisjunction of chromosome 21 occurs in
the ovum.
4% of cases of Down syndrome, the extra chromosomal material derives from the
presence of a rob- ertsonian translocation of the long arm of chromosome 21 to
another acrocentric chromosome (e.g., 22 or 14)
1% of Down syndrome patients are mosaics, having a mixture of cells with 46 or 47
chromo- somes

237. D
Down’s is trisomy 21.
Down’s can also be as a result of robertsonian translocation. In translocation Down
syndrome, the extra 21 chromosome may be attached to the #14 chromosome, or to
other chromosome numbers like 13, 15, or 22. In some cases, two # 21 chromosomes
can be attached to each other

238. D
Deletion of 21 chromosome will not result in Down’s syndrome.

239. C (Best option in this MCQ)


Turner Syndrome Symptoms - CLOWNS
C - Cardiac anomalies (most common - coarctation of aorta)
L - Lymphoedema, low thyroid
O - Ovaries under developed (streak ovaries)
W - Webbed neck
N - Nipples widely placed
S - Short stature, Sensoneural hearing loss, Short 4th metacarpal
240. C
Chromosome location
ABO blood group – Chromosome 9q
Rh blood group – Chromosome 1

241. A
Klinefelter Syndrome
Klinefelter syndrome is best defined as male hypogonad- ism that occurs when there
are two or more X chromo- somes and one or more Y chromosomes
The incidence of this condition is approximately 1 in 660 live male births
Chromosome – 47 XXY

242. C
TurnersSyndrome – Chromsomalabnormalities (Apartfrom 45XO)
46,X,i(Xq)
46,X,del(Xq)
46,X,del(Xp)
46,Xr(X)
45,X/46,XX
45,X/47,XXX
45,X/46,X,i(X)(q10)
243. D
The features are classical of Turners Syndrome. Genotypeis 45XO

244. C
The features are classical of Klinefelters Syndrome. Genotypeis 47XXY.

245. A
Turners syndrome
The ovaries are reduced to atrophic fibrous strands, devoid of ova and follicles (streak
ovaries)
Fetal ovaries develop nor- mally early in embryogenesis, but the absence of the second
X chromosome leads to an accele

246. B
Barr body – Extra X chromosome is silenced and is visible as the heterochromatin in
the buccal smears
Male – No barr body
Females – Single barr body
Turners - No barr body
Klinefelter – Single barr body

247. D
Chromosome 13 deletion
Features include developmental delay, intellectual disability, behavioral problems and
distinctive facial features. There is no increase in risk of malignancy

248. D
The features given can occur as a part of a chromosomal / teratogenic / mendelian
syndrome. Polygenic inheritance is the answer of exclusion. In a polygenic inheritance,
multiple genes are involved in the phenotypic expression. Some examples of polygenic
inheritance are hypertension and diabetes.

249. B
Ataxia Telangiectasia
It’s an autosomal recessive disorder It affects many parts of the body:
It impairs certain areas of the brain including the cerebellum, causing difficulty with
movement and coordination.
It weakens the immune system, causing a predisposition to infection. (Low amount
immunoglobulins)
It prevents repair of broken DNA, increasing the risk of cancer.

250. A
Xerodermapigmentosum, are at increased risk for the development of cancers of the
skin particularly following exposure to the UV light contained in sun rays.
It’s inherited as AR pattern
UV radiation causes cross-linking of pyrimidine residues, preventing normal DNA
replication. Such DNA damage is repaired by the nucleotide excision repair system.
Several proteins are involved in nucleotide excision repair, and an inherited loss of any
one can give rise to xerodermapigmentosum

251. PTEN
The above history is classical of Type 1 endometrial ca.

Features Type 1 Type 2

Prototypic form Endometrioid carcinoma Uterine (papillary) serous


carcinoma

typical patient Perimenopausal or early postmenopausal Elderly women


women

Background of endometrial hyperplasia Background of atrophic


endometrium

Low-grade High-grade

Estrogen-dependent Not estrogen-dependent

Estrogen receptor usually positive; high grade cases may be negative


negative

MIB1 proliferation index (ki- low high


67)

p53 negative; high grade cases may be positive diffuse positivity

PTEN Positive mutation No mutation

252. B
Few important genes
17q21 : BRCA1 (Breast cancer)
17q12 : Her2/neu (Breast cancer)
17q11: Neurofibromin (NF-1) (Neurofibromatosis)
17p13 : p53 (Li-Fraumeni syndrome)
13q12 : BRCA2
13q14: RB1

253. C (Best answer)


Other cancers predisposed apart from breast cancer.
BRCA 1 - Ovarian, male breast cancer (but lower than BRCA2), prostate, pancreas,
fallopian tube
BRCA 2 - Ovarian, male breast cancer, prostate, pancreas, stomach, melanoma,
gallbladder, bile duct, pharynx

254. C
Cleft lip
Cleft lip with or without cleft palate (CL/P) occurs when the frontal maxillary process
fails to fuse at 35 days gestation.
Cleft lip is very heterogenous. Approximately 75-80% cases are sporadic. Syndromic
cases may be monogenic, due to a chromosome abnormality (trisomy 13), or
environmental factors (maternal rubella). Other cases are familial

255. A
Below are a list of DNA repair defect syndromes
Ataxia telangiectasia
Nijmegen breakage syndrome
Werner syndrome
Bloom syndrome
Fanconi anemia
Xerodermapigmentosum
Cockayne syndrome

256. A
Constitutional obesity and mental retardation cooccur in several multiple congenital
anomaly syndromes, including Prader-Willi syndrome, Bardet-Biedl syndrome, Cohen
syndrome, Albright hereditary osteodystrophy, and Borjeson-Forssman-Lehmann
syndrome

257. C
Multifactorial Inheritance
The term multifactorial inheritance refers to the fact that most common diseases are
caused by multiple genes (i.e., a polygenic component) and expression is often
influenced by environmental factors.
Cardiac septal defects – Only a quite a few have few specific genes associated. Most
are sporadic, there are few teratogenic drugs and environmental toxins proposed as a
cause in some.

258. Lymphocytes
Tissues for karyotyping
Peripheral venous blood - most commonly, the lymphocytes (best source)
Skin (fibroblasts), bone marrow.
For fetal chromosome patterns - amniotic fluid cells, chorionic villi.
5-10 mL of heparinized venous blood is the most commonly used source.

259. C
Epigeneticstudies
Epigenetic research includes chromatin immunoprecipitation(Chip-on-chip and ChIP-
Seq), fluorescent in situ hybridization, methylation-sensitive restriction enzymes, DNA
adenine methyltransferase identification (DamID) and bisulfite sequencing

260. B
Karyotype Staining
A variety of staining methods have been developed that allow identification of
individual chromosomes on the basis of a distinctive and reliable pattern of alternating
light and dark bands.
The one most commonly used involves a Giemsa stain and is hence called G banding.
261. D
Fetal skin biopsy is not used for karyotyping, because the the fibroblasts mature
enough to do karyotyping studies only after 2nd trimester. So the purpose of inutero
diagnosis is lost.
So fetal skin biopsy is not useful for karyotyping.

262. A
Micro Array – Studies all genes known.
In array CGH, the test DNA and a reference (normal) DNA are labeled with two
different fluorescent dyes.
The differentially labeled samples are then co- hybridized to an array spotted with
DNA probes that span the human genome at regularly spaced intervals, and usually
cover all 22 autosomes and the sex chromosomes.

263. 5 Mb
Resolution.
Conventional karyotyping is limited to the detection of rearrangements involving more
than 5 Mb of DNA.
The resolution of the FISH technique, using fluorescent probes, is about 100kb-1Mb in
size.

264. B
Karyotyping
The study of chromosomes— karyotyping
The usual procedure to examine chromosomes is to arrest dividing cells in metaphase
with mitotic spindle inhibitors (e.g., N-diacetyl-N-methylcolchicine [Colcemid]) and
then to stain the chromosomes.

265. DNA
Feulgen Reaction.
Used in histology to identify chromosomal material or DNA in cell specimens.
It is darkly stained. It depends on acid hydrolysis of DNA, therefore fixating agents
using strong acids should be avoided.
Diseases of the Immune System –
Questions
266. Interleukin secreted by Th1 cells include?
a. IL-2
b. IL-4
c. IL-10
d. IL-13

267. Castleman syndrome is associated with?


a. EBV
b. CMV
c. HHV8
d. Herpes virus

268. Which of the following cells have MHC class II?


a. RBC
b. B cells, Dendritic cells, NK cells
c. All nucleated cells
d. Platelets

269. During development, all the antigens of self are introduced to thymic
cells in-order to be removed to prevent autoimmunity. Which of the
following genes is involved in the process?
a. NOTCH1
b. AIRE
c. BB gene
d. CPK gene

270. Antibody in diffuse scleroderma:


a. Anti-DNA topoisomerase
b. Anti-Centromere
c. Anti-RNA polymerase
d. Anti-Smith

271. Class IV Lupus nephritis-


a. Mesangial lupus nephritis
b. Proliferative lupus nephritis
c. Membranous lupus nephritis
d. Diffuse sclerosing lupus nephritis

272. Most severe form of lupus nephritis:


a. Mesangial lupus lomerulonephritis
b. Focal proliferative glomerulonephritis
c. Diffuse proliferative glomerulonephritis
d. Membranous glomerulonephritis

273. Speckled pattern is seen in:


a. Anti Sm antibody
b. Anti ds DNA antibody
c. Anti histoneantibody
d. Anti chromatin antibody

274. Cells involved in GVHD are?


a. Recipient B cells
b. Recipient T cells
c. Donor B cells
d. Donor T cells

275. After renal transplantation, which of the following is responsible for


acute graft rejection?
a. C3a
b. C3b
c. C5a
d. C4d

276. For transplantation which HLA requires minimum matching


a. HLA A
b. HLA B
c. HLA DR
d. HLA DP

277. Which of the following is not a characteristic feature of GVHD?


a. Skin involvement
b. Renal involvement
c. Liver involvement
d. Intestinal involvement

278. Feature(s) of DiGeroge syndrome is/are all except?


a. Results from failure of development of the third and fourth pharyngeal pouches
b. Absent thyroid
c. Absent parathyroid glands
d. B cell defect
e. Enhanced susceptibility of bacterial infection
266. a 267. c 268. b 269. b 270. a 271. d 272. c 273. a 274. d

275. d 276. d 277. b 278. d

279. Chediak Higashi syndrome is characterised by the following except:


a. Neutrophilia
b. Defective degranulation
c. Delayed microbial killing
d. Giant granules

280. Which of the following are AIDS defining cancers? (PGI)


a. Burkitt’s lymphoma
b. Non Hodgkin’s Lymphoma
c. Ca esophagus
d. Primary lymphoma of brain
e. Invasive cancer of uterine cervix

281. Chemical nature of amyloid in hemodialysis associated amyloidosis


a. AA
b. AL
c. Aβ2
d. ATR

282. NK cells activity is enchanced by-


a. IL-1
b. TNF
c. IL-2
d. TGF-β

283. The primary function of toll-like reception is-


a. Vasodilation
b. Activation of immune system
c. Regulation of calcium channel
d. Second messenger

284. HLA-B27 is associated with-


a. Ankylosing spondylitis
b. Rheumatoid arthritis
c. Chronic active hepatitis
d. Diabetes

285. Type II hypersensitivity is mediated by (PGI type)


a. Immune complex
b. IgG
c. IgM
d. Complement
e. IgE

286. Which of the following immune hypersensitivity reaction is responsible


for myasthenia gravis?
a. Type I hypersensitivity
b. Type II hypersensitivity
c. Type III hypersensitivity
d. Type IV hypersensitivity

287. Characteristic feature of SLE is-


a. Uveitis
b. Joint deformity
c. Polyserositis
d. Cavitating lesion in Lung

288. Drug induced lupus antibodies are found in-


a. Anti-Ro
b. Ds-DNA
c. Anti-Sm
d. Anti-histone antibody

289. Hemotoxylin bodies are seen in:


a. SLE
b. PAN
c. RA
d. Wegner’s granulomatosis

290. Which type of amyloidosis is caused by mutation of the transthyretin


protein?
a. Familial Mediterranean fever
b. Familial amyloidotic polyneuropathy
c. Dialysis associated amyloidosis
d. Prion protein associated amyloidosis

291. Major abnormalities in HIV include all of the following except


a. Increased IL-2
b. Loss of memory T cells
c. Decreased HLA II expression
d. Hypergammaglobulinemia

292. Th2 subset of T cells is induced by


a. Interferon gamma
b. IL-4
c. IL-17
d. IL-12

293. All of the following are examples of Type II hypersensitivity reactions


except
a. Autoimmune haemolytic anemia
b. Goodpasture syndrome
c. Acute rheumatic fever
d. Reactive arthritis
279. b 280. 281. c 282. c 283. b 284. a 285. 286. b 287. c
d,e b,c,d

288. d 289. a 290. b 291. a 292. b 293. d

294. Most commonly implicated non MHC gene in autoimmunity is


a. NOD2
b. Gene coding for CD 25
c. PTPN22
d. Gene coding for IL-7 receptor alpha

295. Drug induced SLE may show anti nuclear antibodies with the following
staining pattern on immunofluorescence
a.

b.

c.
d.

296. Onion skin lesions in spleen are characteristically seen in


a. Systemic lupus erythematosus
b. Lymphoma infiltration
c. Hypersplenism
d. Sickle cell anemia

297. The earliest histologic finding in both the major and the minor salivary
glands in Sjogren’s syndrome is
a. Lymphoid follicles with germinal centres
b. Hyperplasia of ductal epithelial cells
c. Perivascular lymphoid infiltrate
d. All of the above

298. HLA associated with Multiple Sclerosis is


a. DRB1*0301
b. DQA1*0501
c. DRB1*1501
d. DQB1*1201

299. HLA matching is mandatory for all of the following organs except
a. Kidney
b. Liver
c. Bone marrow
d. Intestine

300. Leucocyte adhesion deficiency type I is caused by mutations in


a. CD 16
b. CD 17
c. CD 18
d. CD 117
294. c 295. a 296. a 297. c 298. c 299. b 300. c
Diseases of the Immune System –
Explanations
266. A
IL-2 self activation
IFN for macrophages

267. C
Unusual nonmalignant lymphoproliferative disorder
Can be unicentric or multicentric
Most often involves lymph nodes but extranodal sites can also be affected
Three histological subtypes: hyaline vascular type, plasma cell type and mixed hyaline
vascular plasma cell
Unicentric Castleman disease can affect all age groups and shows no predilection for
either gender
Multicentric Castleman disease affects mainly HIV positive individuals; however, HIV
negative individuals can also be affected
Most cases are KSHV / HHV8 associated

268. B
Class I MHC molecules are expressed on all nucleated cells and platelets and class II
MHC molecules are mainly expressed on cells that present ingested antigens and
respond to T-cell help (macrophages, B lymphocytes, and dendritic cells).

269. B
A protein called AIRE (autoimmune regulator) stimulates expression of some
“peripheral tissue-restricted” self antigens in the thymus and is thus critical for
deletion of immature T cells specific for these antigens. Mutations in the AIRE gene are
the cause of an autoimmune polyendocrinopathy

270. A
271. D

Classification For Lupus Nephritis

Class Minimal Normal histology with mesangial deposits


I mesangial

Class Mesangial Mesangial hypercellularity with expansion of the mesangial matrix


II proliferation

Class Focal Focal endocapillary ± extracapillary proliferation with focal subendo-thelial immune
III nephritis deposits and mild mesangial expansion

Class Diffuse Diffuse endocapillary ± extracapillary proliferation with diffuse subendo-thelial immune
IV nephritis deposits and mesangial alterations

Class Membranous Thickened basement membranes with diffuse subepithelial immune deposits; may occur
V nephritis with class III or IV lesions and is sometimes called mixed membranous and proliferative
nephritis

Class Sclerotic Global sclerosis of nearly all glomerular capillaries


VI nephritis

272. C
Diffuse lupus nephritis (class IV) is the most common and severe form of lupus
nephritis (Robbins)

273. A
Homogeneous or diffuse nuclear staining usually reflects antibodies to chromatin,
histones, and, occasionally, double-stranded DNA.
Rim or peripheral staining patterns are most often indicative of antibodies to double-
stranded DNA and sometimes to nuclear envelope proteins.
Speckled pattern refers to the presence of uniform or variable-sized speckles. This is
one of the most commonly observed patterns of fluorescence and therefore the least
specific. It reflects the presence of antibodies to non-DNA nuclear constituents such
as Sm antigen, ribonucleoprotein, and SS-A and SS-B reactive antigens.
Nucleolar pattern refers to the presence of a few discrete spots of fluorescence within
the nucleus and represents antibodies to RNA. This pattern is reported most often in
patients with systemic sclerosis.
Centromeric pattern. Patients with systemic sclerosis often contain antibodies
specific for centromeres, which give rise to this pattern.

274. D
When immune-compromised recipients receive HSC preparations from allogeneic
donors, the immunocompetent T cells present in the donor inoculum recognize the
recipient’s HLA antigens as foreign and react.

275. D
Acute antibody-mediated rejection is manifested mainly by damage to glomeruli and
small blood vessels. Typically, the lesions consist of inflammation of glomeruli and
peritubular capillaries, associated with deposition of the complement breakdown
product C4d, which is produced during activation of the complement system by the
antibody dependent classical pathway

276. D
In kidney transplants, there is substantial benefit if all the polymorphic HLA alleles are
matched (both inherited alleles of HLA-A, -B, and DR - being most important).
However, HLA matching is usually not even done for transplants of liver, heart, and
lungs, because other considerations, such as anatomic compatibility, severity of the
underlying illness, and the need to minimize the time of organ storage, override the
potential benefits of HLA matching.

277. B
Skin, liver and GIT are most commonly involved organs in GVHD

278. D
DiGeorge syndrome is a T-cell deficiency that results from failure of development of
the third and fourth pharyngeal pouches. The latter give rise to the thymus, the
parathyroids, some of the C cells of the thyroid, and the ultimobranchial body. Thus,
individuals with this syndrome have a variable loss of T cell–mediated immunity
(resulting from hypoplasia or lack of the thymus), tetany (resulting from lack of the
parathyroids), and congenital defects of the heart and great vessels. In addition, the
appearance of the mouth, ears, and facies may be abnormal. Absence of cell-mediated
immunity is caused by low numbers of T lymphocytes in the blood and lymphoid
tissues and poor defense against certain fungal and viral infections. The T-cell zones of
lymphoid organs—paracortical areas of the lymph nodes and the periarteriolar sheaths
of the spleen—are depleted. Ig levels may be normal or reduced, depending on the
severity of the T-cell deficiency. In many cases, DiGeorge syndrome is not a familial
disorder. It results from a deletion that maps to chromosome 22q11. This deletion is
seen in more than 50% of patients, and DiGeorge syndrome is now considered a
component of the 22q11 deletion syndrome, discussed in Chapter 5. One gene in the
deleted region is TBX1, which is required for development of the branchial arch and
the great vessels. Notably, TBX1 is involved by loss-of-function mutations in a few
cases of DiGeorge syndrome that lack 22q11 deletions, strongly suggesting that its
loss contributes to the observed phenotype.

279. B
Chédiak-Higashi syndrome is an autosomal recessive condition characterized by
defective fusion of phagosomes and lysosomes, resulting in defective phagocytes
function and susceptibility to infections. The main leukocyte abnormalities are
neutropenia (decreased numbers of neutrophils), defective degranulation, and
delayed microbial killing. Leukocytes contain giant granules, which can be readily
seen in peripheral blood smears and are thought to result from aberrant
phagolysosome fusion. In addition, there are abnormalities in melanocytes (leading to
albinism), cells of the nervous system (associated with nerve defects), and platelets
(causing bleeding disorders). The gene associated with this disorder encodes a large
cytosolic protein called LYST, which is believed to regulate lysosomal trafficking.

280. D, E
Patients with AIDS have a high incidence of certain tumors, especially Kaposi sarcoma
(KS), B-cell lymphoma, cervical cancer in women, and anal cancer in men. It is
estimated that 25% to 40% of untreated HIV-infected individuals will eventually
develop a malignancy. A common feature of these tumors is that they are caused by
oncogenic DNA viruses, specifically Kaposi sarcoma herpesvirus (Kaposi sarcoma), EBV
(B-cell lymphoma), and human papillomavirus (cervical and anal carcinoma). Even in
healthy people, any of these viruses may establish latent infections that are kept in
check by a competent immune system. The increased risk of malignancy in AIDS
patients exists mainly because of failure to contain the infections and reactivation of
the viruses, as well as decreased immunity against the tumors.
NHL- most common malignancy in AIDS

281. C
Patients on longterm hemodialysis for renal failure can develop amyloidosis as a result
of deposition of β2 -microglobulin. This protein is present in high concentrations in the
serum of persons with renal disease and in the past, it was retained in the circulation
because it could not be filtered through dialysis membranes. Patients sometimes
presented with carpal tunnel syndrome because of β2 -microglobulin deposition. With
new dialysis filters, the incidence of this complication has decreased substantially.

282. C
NK cells secrete cytokines such as interferon-γ (IFN-γ), which activates macrophages
to destroy ingested microbes, and thus NK cells provide early defense against
intracellular microbial infections. The activity of NK cells is regulated by many
cytokines, including the interleukins IL-2, IL-15, and IL-12. IL-2 and IL-15 stimulate
proliferation of NK cells, whereas IL-12 activates killing and secretion of IFN-γ.

283. B
The best known of the pattern recognition receptors are the Toll-like receptors (TLRs),
whose founding member, Toll, was discovered in Drosophila. A family of related
proteins was later shown to be essential for host defense against microbes. There are
10 TLRs in mammals, and each recognizes a different set of microbial molecules. The
TLRs are present in the plasma membrane and endosomal vesicles . All these
receptors signal by a common pathway that culminates in the activation of two sets of
transcription factors: (1) NF-κB, which stimulates the synthesis and secretion of
cytokines and the expression of adhesion molecules, both of which are critical for the
recruitment and activation of leukocytes, and (2) interferon regulatory factors (IRFs),
which stimulate the production of the antiviral cytokines, type I interferons

284. A

HLA Association - Robbins 9th

Condition HLA Type

Sarcoidosis Al, B8

Goodpasture's syndrome DRB*1501, *1502

Celiac disease DQ2, DQ8

Ankylosing spondylitis 627*2705, *2702

Reactive arthritis B27

Primary sclerosing cholangitis B8

Primary membranous nephropathy DQA1

Grave's disease DR3

Diabetes mellitusType 1 DR3, DR4, DQ8

Psoriasis Cw*0602

Rheumatoid arthritis DRB1

Psoriatic arthritis B27 and Cw6

21 Hydroxylase deficiency BW47

Hereditary hemochromatosis A

Systemic lupus erythematosis DRB1*0301, *1501

SLE with anti dsDNA, anti Sm and antiphospholipid antibodies DQ

Subacute cutaneous SLE DR3

Hydralazine induced SLE DR4

Procainamide induced SLE DR6

Sjogren's syndrome B8, DR3, DRW52, DQA1/B1

Behcet's disease B51

285. B, C, D

Type Immune Mechanisms Histopathologic Lesions Prototypical Disorders

Immediate Production of IgE antibody → immediate Vascular dilation, edema, Anaphylaxis;


(type I) release of vasoactive amines and other smooth muscle allergies; bronchial
hypersensitivity mediators from mast cells; later contraction, mucus asthma (atopic
recruitment of inflammatory cells production, tissue injury, forms)
inflammation
Antibody- Production of IgG, IgM → binds to antigen Phagocytosis and lysis of Autoimmune
mediated (type on target cell or tissue → phagocytosis or cells; inflammation; in hemolytic anemia;
II) lysis of target cell by activated some diseases, Goodpasture
hypersensitivity complement or Fc receptors; recruitment functional derangements syndrome
of leukocytes without cell or tissue
injury

Immune Deposition of antigen-antibody complexes Inflammation, necrotizing Systemic lupus


complex- → complement activation recruitment → vasculitis (fibrinoid erythematosus;
mediated (type of leukocytes by complement products necrosis) some forms of
III) and Fc receptors → release of enzymes glomerulonephritis;
hypersensitivity and other toxic molecules serum sickness;
Arthus reaction

Cell-mediated Activated T lymphocytes → (1) release of Perivascular cellular Contact dermatitis;


(type IV) cytokines, inflammation and macrophage infiltrates; edema; multiple sclerosis;
hypersensitivity activation; (2) T cell-mediated cytotoxicity granuloma formation; cell type 1 diabetes;
destruction tuberculosis

286. B
Examples of type I hypersensitivity

Clinical Syndrome Clinical and pathologic Manifestations

Anaphylaxis (may be caused by Fall in blood pressure (shock) cause by vascular dilation; airway obstruction
drugs, bee sting, food) due to laryngeal edema

Bronchial asthma Airway obstruction caused by bronchial smooth muscle hyperactivity;


inflammation and tissue injury caused by late-phase reaction

Allergic rhinitis, sinusitis (hay Increased mucus secretion; Inflammation of upper airways, sinuses
fever)

Food allergies Increased peristalsis due to contraction of intestinal muscles

Examples of type II hypersensitivity

Disease Target Antigen Mechanisms of Disease

Autoimmune Red cell membrane proteins (Mi blood group Opsonization and phagocytosis of red
hemolytic anemia antigens. 1 antigen) cells

Autoimmune Platelet membrane proteins (Gplla: illa Opsonization and phagocytosis of


thrombocytopenic integrin) platelets
purpura

Pemphigus vulgaris Proteins In Intercellular Junctions of Antibody-mediated activation of


epidermal cells proteases, disruption of Intercellular
adhesions

Vasculitis caused by Neutrophil granule contents, presumably Neutrophil degeneration and


ANCA released from activated neutrophils Inflammation

Goodpasture Noncollagenous protein In basement Complement- and Fc receptor-mediated


syndrome membranes of Kidney glomeruli and lung Inflammation
alveoli

Acute rheumatic Streptococcal cell wall antigen; antibody Inflammation, macrophage activation
fever cross-reacts with myocardial antigen

Myasthenia gravis Acetylcholine receptor Antibody Inhibits acetylcholine binding,


down-modulates receptors

Graves disease TSH receptor Antibody-mediated stimulation of TSH


(hyperthyroidism) receptors

Insult-resistant Insulin receptor Antibody inhibits binding of insulin


diabetes

Pernicious anemia Intrinsic factor of gastric parietal cells Neutralization of Intrinsic factor,
decreased absorption of vitamin B12

Examples of type III hypersensitivity

Disease Antigen Involved

Systemic lupus erythematosus Nuclear antigens (circulating or “planted” In kidney)

Post streptococcal Streptococcal cell wall antigen(s); maybe “planted” In glomerular basement
glomerulonephritis membrane

Polyarteritis nodosa Hepatitis B virus antigens In some cases

Reactive arthritis Bacterial antigens (e.g., Yersinia)

Serum sickness Various proteins, e.g., foreign serum protein (horse antithymocyte globulin!

Arthus reaction (experimental! Various foreign proteins

Examples of type IV hypersensitivity

Disease Specificity of Pathogenic T cells Principal Mechanisms of tissue Injury

Rheumatoid Collagen? Cirtullinated self-proteins? Inflammation mediated by TH 17 (and TH1?)


arthritis cytokines; role of antibiotics and immune
complexes?

Multiple Protein antigens in myelin (e.g., myelin Inflammation medicated by TH1 and TH17
Sclerosis basic protein) cytokines, myelin destruction by activated
macrophages

Antigens of pancreatic islet ß cells T cell-medicated inflammation, destruction of islet


Type 1 (Insulin, glutamic acid decarboxylase, cells by CTLs
diabetes Other)
Mellitus

Inflammatory Enteric bacteria: self-antigens? Inflammation mediated by TH1 and TH17 cytokines
bowel
diseases

Psoriasis Unknown Inflammation mediated mainly by TH17 cytokines

Contact Various environmental chemicals (e.g., Inflammation mediated by TH1(and TH17?)


sensitivity urushiol from poison ivy or poison oak) cytokines

287. C
SLICC Classification Criteria for Systemic Lupus Erythematosus
Requirements: ≥4 criteria (at least 1 clinical and 1 laboratory criteria) OR biopsy-
proven lupus nephritis with positive ANA or Anti-DNA

Clinical Criteria Immunologic Criteria

1. Acute Cutaneous Lupus* 1. ANA

2. Chronic Cutaneous Lupus* 2. Anti-DNA

3. Oral or nasal ulcers * 3. Anti-Sm

4. Non-scarring alopecia 4. Antiphospholipid Ab*

5. Arthritis * 5. Low complement (C3, C4,


CH50)

6. Serositis * 6. Direct Coombs' test (do not count in the presence of hemolytic
anemia)

7. Renal *

8. Neurologic *

9. Hemolytic anemia

10. Leukopenia *

11.Thrombocytopenia (<100,000/mm3)

288. D
A lupus erythematosus-like syndrome may develop in patients receiving a variety of
drugs, including hydralazine, procainamide, isoniazid, and D-penicillamine, to name
only a few. Somewhat surprisingly, anti-TNF therapy, which is effective in rheumatoid
arthritis and other autoimmune diseases, can also cause drug-induced lupus. Many of
these drugs are associated with the development of ANAs, but most patients do not
have symptoms of lupus erythematosus. For example, 80% of patients receiving
procainamide test positive for ANAs, but only one third of these manifest clinical
symptoms, such as arthralgias, fever, and serositis. Although multiple organs are
affected, renal and central nervous system involvement is distinctly uncommon. There
are serologic and genetic differences from classic SLE, as well. Antibodies specific for
double-stranded DNA are rare, but there is an extremely high frequency of antibodies
specific for histones. Persons with the HLA-DR4 allele are at a greater risk of
developing a lupus erythematosus-like syndrome after administration of hydralazine,
whereas those with HLA-DR6 (but not DR4) are at high risk with procainamide. The
disease remits after withdrawal of the offending drug.

289. A
There is no evidence that ANAs, which are involved in immune complex formation, can
penetrate intact cells. If cell nuclei are exposed, however, the ANAs can bind to them.
In tissues, nuclei of damaged cells react with ANAs, lose their chromatin pattern, and
become homogeneous, to produce so-called LE bodies or hematoxylin bodies. Related
to this phenomenon is the LE cell, which is readily seen when blood is agitated in vitro.
The LE cell is any phagocytic leukocyte (blood neutrophil or macrophage) that has
engulfed the denatured nucleus of an injured cell. The demonstration of LE cells in
vitro was used in the past as a test for SLE. With new techniques for detection of ANAs,
however, this test is now largely of historical interest. Sometimes, LE cells are found in
pericardial or pleural effusions in patients.

290. B
291. A

Lymphopenia

Predominantly caused by selective loss of the CD4+ helper T-cell subset

Decreased T-Cell Function In Vivo

Preferential loss of activated and memory T cells Decreased delayed-type hypersensitivity

Susceptibility to opportunistic infections

Susceptibility to neoplasms

Altered T-Cell Function In Vitro

Decreased proliferative response to mitogens, alloantigens, and soluble antigens

Decreased cytotoxicity

Decreased helper function for B-cell antibody production Decreased IL-2 and IFN-y production

Polyclonal B-Cell Activation

Hypergammaglobulinemia and circulating immune complexes

Inability to mount de novo antibody response to new antigens

Poor responses to normal B-cell activation signals in vitro

Altered Monocyte or Macrophage Functions


Decreased chemotaxis and phagocytosis Decreased class II HLA expression Diminished capacity to present
antigen to T cells

292. B

Major IFN-γ IL-4, IL-5, IL-13 IL-17, IL-22


cytokines
produced

Cytokines IFN-γ, IL-12 IL-4 TGF-β, IL-6, IL-1, IL-23


that induce
this subset

Immunological Macrophage activation, stimulation of Stimulation of lgE Recruitment of neutrophils,


reactions lgG antibody production production, activation monocytes
triggered of mast cells and
eosinophils

Host defense Intracellular microbes Helminthic parasites Extracellular bacteria, fungi


against

Role in Autoimmune and other chronic Allergies Autoimmune and other


disease inflammatory diseases (such as IBD, chronic inflammatory
psoriasis, granulomatous diseases (such as IBD,
inflammation) psoriasis, MS)

293. D

Disease Target Antigen Mechanisms of Disease Clinicopathologic


Manifestations

Autoimmune Red cell membrane proteins (Rh Opsonization and phagocytosis of Hemolysis,
hemolytic anemia blood group antigens, 1 antigen) red cells anemia

Autoimmune Platelet membrane proteins Opsonization and phagocytosis of Bleeding


thrombocytopenic (Gpllb:llla integrin) platelets
purpura

Pemphigus Proteins in intercellular junctions of Antibody-mediated activation of Skin vesicles


vulgaris epidermal cells (epidermal cadherin) proteases, disruption of (bullae)
intercellular adhesions

Vasculitis caused Neutrophil granule proteins, Neutrophil degranulation and Vasculitis


by ANCA presumably released from activated inflammation
neutrophils

Goodpasture Noncollagenous protein in basement Complement- and Fc receptor- Nephritis, lung


syndrome membranes of kidney glomeruli and mediated inflammation hemorrhage
lung alveoli

Acute rheumatic Streptococcal cell wall antigen; Inflammation, macrophage Myocarditis,


fever antibody cross-reacts with activation arthritis
myocardial antigen

Myasthenia gravis Acetylcholine receptor Antibody inhibits acetylcholine Muscle


binding, down-modulates weakness,
receptors paralysis

Graves disease TSH receptor Antibody-mediated stimulation of Hyperthyroidism


(hyperthyroidism) TSH receptors

Insulin-resistant Insulin receptor Antibody inhibits binding of insulin Hyperglycemia,


diabetes ketoacidosis

Pernicious anemia Intrinsic factor of gastric parietal Neutralization of intrinsic factor, Abnormal
cells decreased absorption of vitamin erythropoiesis,
B1? anemia

294. C
Polymorphisms in a gene called PTPN22, which encodes a protein tyrosine
phosphatase, are associated with rheumatoid arthritis, type 1 diabetes, and several
other autoimmune diseases. Because these disorders have a fairly high prevalence
(especially rheumatoid arthritis), PTPN22 is said to be the gene that is most frequently
implicated in autoimmunity. It is postulated that the disease-associated variants
encode a phosphatase that is functionally defective and is thus unable to fully control
the activity of tyrosine kinases, which are involved in many responses of lymphocytes
and other cells. The net result is excessive lymphocyte activation.

295. A
Drug induced SLE shows anti histone antibodies which have a homogenous staining.
Homogeneous or diffuse nuclear staining usually reflects antibodies to chromatin,
histones, and, occasionally, double-stranded DNA.
Speckled pattern refers to the presence of uniform or variable-sized speckles. This is
one of the most commonly observed patterns of fluorescence and therefore the least
specific. It reflects the presence of antibodies to non-DNA nuclear constituents such
as Sm antigen, ribonucleoprotein, and SS-A and SS-B reactive antigens.
Nucleolar pattern refers to the presence of a few discrete spots of fluorescence within
the nucleus and represents antibodies to RNA. This pattern is reported most often in
patients with systemic sclerosis.
Centromeric pattern. Patients with systemic sclerosis often contain antibodies
specific for centromeres, which give rise to this pattern.

296. A
Central penicilliary arteries may show concentric intimal and smooth muscle cell
hyperplasia, producing so-called onion-skin lesions.

297. C
Lacrimal and salivary glands are the
major targets of the disease, although other exocrine glands,
including those lining the respiratory and gastrointestinal tracts
and the vagina, may also be involved. The earliest histologic
finding in both the major and the minor salivary glands is periductal
and perivascular lymphocytic infiltration. Eventually the
lymphocytic infiltrate becomes extensive, and in the larger salivary glands lymphoid
follicles with germinal centers
may be seen. The ductal lining epithelial cells may show hyperplasia,
thus obstructing the ducts. Later there is atrophy of the
acini, fibrosis, and hyalinization; still later in the course atrophy
and replacement of parenchyma with fat are seen.

298. C

Disease HLA allele

Rheumatoid arthritis (anti-CCP Ab positive) DRB1,1 SE allele1

DRB1, 2 SE alleles

Type 1 diabetes
DRB1 *0301 -DQA1*0501 -

DQB1*0201 haplotype

DRB1*0401-DQA1*0301-

DQB1*0302 haplotype

DRB1*0301/0401 haplotype

heterozygotes

Multiple sclerosis DRB1 *1501

Systemic lupus erythematosus


DRB1 *0301

DRB1*1501

Ankylosing spondylitis B*27 (mainly B*2705 and B*2702)

Celiac disease DQA1 *0501 -DQB1*0201 haplotype

299. B
The value of HLA matching between donor and recipient varies in different solid-organ
transplants. In kidney transplants, there is substantial benefit if all the polymorphic
HLA alleles are matched (both inherited alleles of HLA-A, -B, and DR). However, HLA
matching is usually not even done for transplants of liver, heart, and lungs, because
other considerations, such as anatomic compatibility, severity of the underlying
illness, and the need to minimize the time of organ storage, override the potential
benefits of HLA matching.

300. C

Disease Defects

Defects in Leukocyte Function

Leukocyte adhesion Defective leukocyte adhesion because of mutations in β chain of CD11/CD18


deficiency 1 integrins

Leukocyte adhesion Defective leukocyte adhesion because of mutations in fucosyl transferase required
deficiency 2 for synthesis of sialylated oligosaccharide (receptor for selectins)

Chediak-Higashi Decreased leukocyte functions because of mutations affecting protein involved in


syndrome lysosomal membrane traffic
Chronic granulomatous Decreased oxidative burst
disease

X-linked Phagocyte oxidase (membrane component)

Autosomal recessive Phagocyte oxidase (cytoplasmic components)

Myeloperoxidase Decreased microbial killing because of defective MP0-H2O2 system


deficiency

Defects in the Complement System

C2, C4 deficiency Defective classical pathway activation, results in reduced resistance to infection and
reduced clearance of immune complexes

C3 deficiency Defects in all complement functions

Deficiency of Excessive complement activation; clinical syndromes include angioedema,


complement regulatory paroxysmal hemoglobinuria, others
proteins
Neoplasia – Questions
301. Lack of differentiation is called?
a. Anaplasia
b. Metaplasia
c. Dysplasia
d. Carcinoma

302. C-KIT mutations are seen in-


a. Gastrointestinal stromal tumors
b. Ovarian cancers
c. Neuroblastoma
d. Small-cell carcinoma of lung

303. Number of cancer cells present in the smallest clinically detectable mass-
a. 103 cell
b. 106 cell
c. 109 cell
d. 1012 cell

304. Overgrowth of a bile duct at localized region is?


a. Hamartoma
b. Malignant tumor
c. Choriostoma
d. Polyp

305. Which of the following mutation is seen in Cowden syndrome?


a. PTEN mutation
b. STK11 mutation
c. PTCH mutation
d. SMAD4 mutation

306. Which of the following Dyads are correctly matched? (PGI)


a. RB1-retinoblastoma
b. PTEN-melanoma
c. BRCA2-Breast cancer
d. n-MYC-Neuroblastoma
e. WT1=Wilm’s tumor

307. If the RB gene phosphorylation is defective which of the following will


happen?
a. Cell cycle will stop at G2
b. Cell cycle will stop at G1
c. There will be no effect on cell cycle as RB gene phosphorylation is not needed
d. The cell cycle progresses and cell divides

308. E6 protein of high risk human papilloma viruses bind to the following
protein and there by promotes carcinogenesis:
a. RB
b. TP53
c. APC
d. MDM2

309. All are associated with BRCA mutation except:


a. Ovarian carcinoma
b. Prostate carcinoma
c. Endometrial carcinoma
d. Papillary serous cancer of peritoneum

310. BRCA 1 and BRCA 2 genes are located on chromosomes:


a. 13 and 17
b. 17 and 22
c. 17 and 13
d. 13 and 22

311. Gene promoting metastasis:


a. SNAIL
b. TWIST
c. Both
d. None

312. Excessive fibrosis in a tumor in called:


a. Anaplasia
b. Metaplasia
c. Desmoplasia
d. Dysplasia

313. Cancers associated with viruses? (PGI)


a. Hepatocellular cancer
b. Kaposi sarcoma
c. Nasopharyngeal cancer
d. Small cell Ca lung
e. Prostatic Ca
301. a 302. a 303. c 304. a 305. a 306. 307. d 308. a 309. c
a,c,d,e

310. c 311. c 312. c 313.


a,b,e

314. Chromopthysis mechanism of carcinogenesis has been found to be


associated with which of the following tumors?
a. Sarcomas
b. Osteosarcoma
c. RCC
d. All

315. Paraneoplastic syndrome(s) associated with lymphoma?


a. SIADH
b. Hypercalcemia
c. Cushing’s syndrome
d. Acanthosis nigricans

316. In tumor lysis syndrome, all of the following are seen except:
a. Hyperuricemia
b. Hypercalcemia
c. Hyperkalemia
d. Hyperphosphatemia

317. In CA breast, based on Which stage/grade of IHC staining, FISH for gene
amplification will be done?
a. Her2 neu 3+
b. Her2 neu 2+
c. Her2 neu 1+
d. Will be done irrespective of above

318. A surgeon suspecting testicular carcinoma in a patient asks the intern to


send the sample for histopathology, what is the fluid in which the intern
should send the sample to the pathologist?
a. Bouin solution
b. 10% formalin
c. 95% ethanol
d. Alcohol

319. Neuroendocrine cell tumor markers are: (PGI)


a. Chromogranin A
b. CD56
c. Neuron-specific enolase
d. Synaptophysin
e. Cytokeratin 7

320. Tumor markers and tumors-find the wrong match:


a. Desmin-Carcinoma
b. Vimentin-sarcoma
c. Leucocyte specific antigen-lymphoma
d. S100-melanoma

321. All of the following viruses have been implicated in causation of


hematolymphoid neoplasms except
a. EBV
b. CMV
c. HHV8
d. HTLV

322. A child with Down’s syndrome presented with increased TLC and anemia.
On performing differential count, 65% atypical cells were seen. The
diagnosis of AML M7 was rendered. The following morphology in addition to
flow cytometry helped in the identification of the condition
a.

b.

c.

d.

314. b 315. b 316. b 317. b 318. a 319. 320. a 321. b 322. a


a,b,c,d

323. A 45-year man presents a cervical lymphadenopathy, biopsy from the


lesion closely adhesive neoplastic cells forming glands. A suspicion of
metastatic adenocarcinoma was made. Further imaging and investigations
does not reveal any possible primary. A further Immunohistochemistry
shows CK7+, CK20- . which of the following organs is most likely organ for
such a primay
a. Lung
b. Liver
c. Colon
d. Prostate

324. Which of the following cyclins propels cell from G2 --> M phase
a. Cyclin B
b. Cyclin D
c. Cyclin A
d. Cyclin E

325. Philadephia chromosome seen in chronic myeloid leukemia produces a


protein called
a. P170
b. P210
c. P190
d. P230

326. Loss of which of the following protein is implicated in aiding metastasis


of tumor cells?
a. Integrins
b. Cadherins
c. Matrix metalloproteases
d. Cathepsin

327. Ductal carcinoma in situ can be differentiated from invasive ductal


carcinoma by all of the following except
a. Nuclear pleomorphism
b. Basement membrane invasion
c. Loss of myoepithelial cells
d. Central areas of necrosis

328. All of the following points are true about Lynch syndrome except
a. Left colon is involved more
b. Endometrial cancer is the most common extra colonic tumor involved
c. Intra tumoral lymphocytes are more
d. Seen in younger age group

329. Which of the following is the most reliable feature of malignancy in a


paraganglioma?
a. Presence of metastasis
b. Nuclear pleomorphism
c. Vascular or capsular invasion
d. Increased mitotic activity

330. Which of the following is not true regarding genetics of thyroid tumors
a. Driver mutations in KRAS are seen in follicular carcinoma
b. BRAF mutations are the most common mutations seen in papillary carcinoma
c. RET rearrangements are seen in medullary carcinoma
d. PI3K mutations are seen in anaplastic carcinoma

331. All are markers for malignant melanoma except


a. HMB 45
b. MART1
c. SOX 11
d. SOX 10

332. All of the following familial cutaneous cancer syndromes show autosomal
dominant inheritance except
a. Muir Torre syndrome
b. Cowden syndrome
c. Gorlin syndrome
d. Ataxia telangiectasia

333. All are true about aflatoxin except


a. Causes hepatocellular carcinoma
b. Aflatoxin B3 mostly implicated
c. Causes P53 mutation in codon 249
d. Arginine to serine substitution

334. Which of the following Is the most implicated in causation of cutaneous


cancers?
a. UVA
b. UVB
c. UVC
d. All of the above
323. a 324. a 325. b 326. b 327. a 328. a 329. a 330. c 331. c

332. d 333. b 334. b

335. Epigenetic changes in ARID1A is seen in


a. AML
b. Follicular lymphoma
c. Ovarian clear cell carcinoma
d. Renal cell carcinoma

336. Which of the following gene is translocated in Burkitt lymphoma?


a. PML
b. FLI1
c. MYC
d. CCND1

337. Upregulation of which of the following has been noted on tumor cells
which helps them evade immune surveillance
a. PD L1
b. CD 28
c. CTLA4
d. CD 86

338. Which IHC marker will be useful to distinguish between ALL and thymoma
a. TDT
b. CD3
c. CD1a
d. Cytokeratin

339. Which of the following is the most common malignant tumor in adult
males in India?
a. Lung cancer
b. Oropharyngeal carcinoma
c. Gastric carcinoma
d. Colorectal carcinoma

340. Which is not a tumor suppressor gene? (PGI)


a. P53
b. CD95
c. RAS
d. PTEN
e. Stk 7/Stk11

341. On which cell cycle checkpoints BRCA-2 acts?


a. G2-M
b. G1-M
c. M phase
d. G1 phase
e. S phase

342. All are true regarding Retinoblastoma except?


a. Play major role in cell cycle regulation
b. Require deletion of both Rb genes
c. Autosomal Dominant
d. Located on chr 13p 14

343. Most common gene involved in endometrial ca is?


a. PTEN
b. BRAF mutation
c. KRAS
d. Mismatch repair genes

344. The following protein is called “Gatekeeper of colonic neoplasia”:


a. APC
b. STK11
c. SMAD2
d. PTEN

345. Hall mark mechanism of tumor suppressor gene inactivation-


a. Loss of heterozygosity
b. DNA hypermethylation
c. DNA hypomethylation
d. Histone deacetylation

346. CDK4 positive tumors are? (PGI)


a. Melanomas
b. Sarcomas
c. Glioblastomas
d. Lobular Ca breast
e. Prostate Ca

347. Dysplasia is characterized by all except:


a. High nuclear to cytoplasmic ratio
b. Loss of architecture
c. Pleomorphism
d. Invasion

348. Which is the most common site of distant metastasis:


a. Lung
b. Bone
c. Liver
d. Brain

349. All of the following are involve in tumor metastasis cascade except:
a. Fibronectin
b. E-cadherin
c. Type IV collagenase
d. Tyrosine kinase

350. Human papilloma virus is/are associated with all except: (PGI)
a. Oropharyngeal tumors
b. Carcinoma nasopharynx
c. Carcinoma anal canal
d. Carcinoma pancreas
e. Carcinoma cervix
335. c 336. c 337. a 338. d 339. a 340. c 341. d 342. d 343. a

344. a 345. a 346. 347. d 348. a 349. d


a,b,c

351. Malignancy associated with hypercalcemia:


a. Breast cancer
b. Prostate cancer
c. Small cell lung cancer
d. Non small lung cancer

352. Migratory thrombophlebitis is seen in all except?


a. Lung Ca
b. Prostate Ca
c. Colon Ca
d. Pancreatic Ca

353. Which of the following immunohistochemical marker is positive in


neuroendocrine tumor?
a. Cytokeratin
b. Synaptophysin
c. Calretinin
d. GFAP

354. Which of the following neoplasms shows ALK positivity?


a. Ewing sarcoma
b. Inflammatory myofibroblastic tumor
c. Synovial sarcoma
d. Fibromatosis

355. Fixative agent for PAP smear?


a. Norma saline
b. Formalin
c. 95% ethanol
d. Air drying

356. ALK1 mutation seen mc with which hereditary hemorrhagic


telangiectasia?
a. Type 1
b. Type 2
c. Type 3
d. Type 4

357. Carbohydrate marker of recurrent breast cancer is?


a. CA 15.3
b. CA 125
c. CEA
d. CA 19.9

358. For detection of carcinoma lip, stain used is?


a. Giemsa
b. Crystal violet
c. Toulidine blue
d. Hematoxylin and eosin

359. AFP and CEA both rise in:


a. Testicular tumors
b. Hepatocellular carcinoma
c. Renal cell carcinoma
d. Germ cell tumor of ovary

360. Grading of cancers does not depend on:


a. Degree of differentiation
b. Number of mitoses
c. Architectural features
d. Size of primary lesion

361. HMB-45 is positive in:


a. Small cell carcinoma of lung
b. Adenocarcinoma of lung
c. Squamous cell carcinoma of lung
d. Angiomyolipoma of lung

362. An undifferentiated malignant tumor on immuno-histochemical stain


shows cytoplasmic positively of most of the tumor cells for cytokeratin. The
most probable diagnosis of the tumor is:
a. Lymphoma
b. Carcinoma
c. Sarcoma
d. Malignant melanoma

363. Which of the following is not premalignant?


a. Hyperplasia
b. Metaplasia
c. Hypertrophy
d. Dysplasia
350. 351. 352. 353. 354. 355. c 356. 357. a 358. c 359. b
d a a b b b

360. 361. 362. 363. c


d d b
Neoplasia – Explanations
301. A
Tumors composed of undifferentiated cells are said to be anaplastic, a feature that is a
reliable indicator of malignancy. The term anaplasia literally means “backward
formation”—implying dedifferentiation, or loss of the structural and functional
differentiation of normal cells

302. A
303. C
Original untransformed cell- 10 µm
30 population doublings to produce 109 cells- 1 gm- smallest clinically detectable
mass (*)
10 additional doubling cycles to produce 1012 cells- approximately 1 kg – maximal
size compatible with life

304. A
Hamartoma is a mass of disorganized tissue indigenous to the particular site, such as
the lung or the liver.
While traditionally considered developmental malformations, many hamartomas have
clonal chromosomal aberrations that are acquired through somatic mutations and on
this basis are now considered to be neoplastic

305. A
PTEN:
Encodes a lipid phosphatase which is an important negative regulator of PI3K/AKT
pathway
Germline LoF mutations in Cowden syndrome
PTCH1
Encodes membrane receptor that is a negative regulator of Hedgehog signalling
pathway
Germline LoF cause Gorlin syndrome associated with high risk of basal cell carcinoma
and medulloblastoma
STK11
Serine threonine kinase 11
Peutz Jegher syndrome
SMAD4
Signal transduction protein of TGF beta pathway
Pancreatic cancer
Juvenile polyposis

306. A, C, D, E
307. D

308. A
309. C
Although the precise causes are not known, a link with certain variants of BRCA1/2 has
been described. [3] Furthermore, women with BRCA1/2 mutation have a 5% risk of
developing primary peritoneal cancer even after prophylactic oophorectomy.
Primary peritoneal carcinoma shows similar rates of tumor suppressor gene
dysfunction (p53, BRCA, WT1) as ovarian cancer and can also show an increased
expression of HER-2/neu.

310. C
BRCA 1 -17q
BRCA 2- 13 q

311. C
Among candidates for metastasis oncogenes, best recognised are SNAIL andTWIST,
which encode transcription factors whose primary function is to promote epithelial-to-
mesenchymal transition (EMT). In EMT, carcinoma cells downregulate certain
epithelial markers (e.g., E-cadherin) and upregulate certain mesenchymal markers
(e.g., vimentin and smooth muscle actin). These changes are believed to favor the
development of a promigratory phenotype that is essential for metastasis. Loss of E-
cadherin expression seems to be a key event in EMT, and SNAIL and TWIST are
transcriptional repressors that downregulate E-cadherin expression. EMT has been
documented mainly in breast cancers; whether it is a general phe

312. C
In some tumors, connective tissue is scant and so the neoplasm is soft and fleshy. In
other cases, the parenchymal cells stimulate the formation of an abundant
collagenous stroma, referred to as desmoplasia. Some demoplastic tumors—for
example, some cancers of the female breast—are stony hard or scirrhous

313. A, B,C

Microbe Associated cancer

EBV Burkitt lymphoma, Hodgkin lymphoma, nasopharyngeal carcinoma, 1° CNS lymphoma


(in immunocompromised patients)

HBV, HCV Hepatocellular carcinoma

HHV-8 Kaposi sarcoma

HPV Cervical and penile/anal carcinoma (types 16, 18), head and neck cancer

H pylori Gastric adenocarcinoma and MALT lymphoma


HTLV-1 Adult T-cell leukemia/lymphoma

Liver fluke (Clonorchis Cholangiocarcinoma


sinensis)

Schistosoma Bladder cancer (squamous cell)


haematobium

314. B
The true extent of chromosome rearrangements in cancer is only now coming into
view thanks to sequencing of entire cancer cell genomes, which allows for
comprehensive “reconstruction” of chromosomes from DNA sequences. Genomic
sequencing has revealed not only many simple rearrangements (e.g., small deletions,
duplications, or inversions) that were not appreciated by prior methods, but also much
more dramatic chromosome “catastrophes” termed chromothrypsis (literally,
chromosome shattering). Chromothrypsis is observed in 1% to 2% of cancers as a
whole, but is found in up to 25% of osteosarcomas and other bone cancers and at
relatively high frequency in gliomas as well. It appears to result from a single event in
which dozens to hundreds of chromosome breaks occur within part or across the
entirety of a single chromosome or several chromosomes

315. B
Hypercalcemia is the most common paraneoplastic syndrome.
But cancer associated hypercalcemia isn’t always paraneoplastic. When the cancer
produces some calcemic substances, only then you call it paraneoplastic.
Hypercalcemia due to primary or secondary involvement of skeleton isn’t a
paraneoplastic hypercalcemia.
Now cancer associated hypercalcemia can be due to different causes (paraneoplastic
or otherwise). Three major mechanisms of hypercalcemia in tumors are as follows-
1. Osteolysis - either due to a primary tumor like myeloma or a secondary tumor like
breast cancer.
2. Excessive 1,25(OH)2 D production like in lymphomas
3. True paraneoplastic conditions created by secretion of PTH related peptide (PTHrP)
by tumors like squamous cell carcinoma lung
316. B
Cairo-Bishop Definition of Laboratory Tumor Lysis Syndrome

Element Value Change from Baseline

Uric acid ≥ 476 micro mol/L (8 mg/dL) 25% increase

Potassium ≥ 6.0 m mol/L (or 6 mEq/L) 25% increase

Phosphorous 25%increase
≥2.1 mmol/L (6.5 mg/dL) for children

or

≥ 1.45 mmol/L (4-5 mg/dL) for adults

Calcium ≥ l.75 mmol/L (7 mg/dL) 25%decrease

NOTE: Two of more laboratory changes within the period from 3 days before to 7 days
after cytotoxic therapy are required to establish the diagnosis of tumor lysis syndrome.

317. B
Algorithm for evaluation of human epidermal growth factor receptor 2
(HER2) protein expression by immunohistochemistry (IHC) assay of the
invasive component of a breast cancer specimen.

318. A
Fixatives used in pathology
1. Histopathology routine- 10% neutral buffered formalin (most commonly used
fixative)
2. Electron microscopy- glutaraldehyde
3. Testicular biopsies- Bouin’s fluid (can be used for GIT and endocrine too)
4. Lymph node/bone marrow/kidney- Zenker’s (good nuclear details) or B5
5. To dissolve fat for lymph node examination in radical resections- Carnoy’s (also for
karyotyping)
6. Cytopathology - Ethanol (95%)

319. A, B, C, D
Neoplasms which express neuroendocrine markers (synaptophysin, chromogranin A,
NCAM / CD56 or protein gene product (PGP))
Synaptophysin is strongly and diffusely expressed in most tumors
Chromogranin A is focal / patchy
CD56 and PGP are considered less specific
Site specific markers are PDX1 and ISL1

320. A
Desmin- muscle origin

321. B
Three lymphotropic viruses—human T-cell leukemia virus-1 (HTLV-1), Epstein-Barr
virus (EBV), and Kaposi sarcoma herpesvirus/human herpesvirus-8 (KSHV/HHV-8)—
have been implicated as causative agents in particular lymphomas. HTLV-1 is
associated with adult T-cell leukemia/ lymphoma. EBV is found in a subset of Burkitt
lymphoma, 30% to 40% of Hodgkin lymphoma (HL), many B-cell lymphomas arising in
the setting of T-cell immunodeficiency, and rare NK-cell lymphomas. In addition to
Kaposi sarcoma. KSHV is associated with an unusual B-cell lymphoma that presents as
a malignant effusion, often in the pleural cavity

322. 22 A
The megakaryoblasts are usually medium-sized to large blasts (12-18 µm) with a
round, slightly irregular, or indented nucleus with fine reticular chromatin and 1-3
nucleoli. The cytoplasm is basophilic, often agranular, and show distinct blebs or
pseudopod formation.

323. A

324. A

325. B
Due to BCR ABL1 fusion in CML, the new protein formed is 210 kD or p210. P190 is
seen in ALL.

326. B
Integrins are components of ECM which help in binding of cells to ECM. Although
implicated in metastasis, E and N cadherins (part of desmosomes) are more
recognised in the mechanism of invasion of metastasis, the former, when lost, playing
a role in epithelial-mesenchymal transition. Matrix metalloproteases and cathepsin
help breakdown stroma and aid invasion. Loss of E cadherin is also characteristically
noted in Lobular carcinoma breast and diffuse gastric carcinoma.

327. A
DCIS shows nuclear pleomorphism just like an invasive carcinoma. But the presence of
basement membrane invasion, loss of myoepithelial cells favour IDC whereas central
areas of necrosis are more characteristic of DCIS (called comedo type of DCIS). Hence,
nuclear pleomorphism does not help to differentiate between the two entities.

328. A
Warthin Lynch syndrome or Hereditary non polyposis coli cancer is the most common
familial cause of colorectal carcinoma. Patients affected are younger and usually
involve the right colon. Mucinous, signet ring cell features are more common and intra
tumoral lymphocytes are increased. Increased incidence of other cancers like
endometrial (most common), gastric, ovarian and transitional cell is also noted.

329. A
Unfortunately, it is almost impossible to predict the clinical course of a carotid body
tumor—mitoses, pleomorphism, and even vascular invasion are not reliable indicators.
Hence, presence of metastasis is the only reliable indicator of malignancy.

330. C
RET mutations are seen in medullary carcinoma and RET/PTC rearrangements are seen
in papillary carcinoma.

331. C
SOX 11 is a marker for mantle cell lymphoma.

332. D Ataxia telangiectasia

Disease Inheritance Chromosomal Gene/Protein Normal Function/Manifestation of Loss


Location

Ataxia- AR 11 q22.3 ATM/AIM DNA repair after radiation


telangiectasia injury/neurologic and vascular lesions

Nevoid basal cell AD 9q22 PTCtf/PTCH Developmental patterning gene/multiple


carcinoma basal cell carcinomas; medulloblastoma,
syndrome jaw cysts

Cowden AD 10q23 PTENIPTEN Lipid phosphatase/benign follicular


syndrome appendage tumors (trichilemmomas);
internal adenocarcinoma (often breast or
endometrial)

Familial AD 9p21 CDWV2/p16/INK4 Inhibits CDK4/6 phosphorylation of RB,


melanoma CDMV2/p14/ARF promoting cell cycle arrest/ melanoma;
syndrome pancreatic carcinoma Binds MDM2,
promoting p53 function/melanoma;
pancreatic carcinoma

Muir-Torre AD MSH2/ MSH2 Involved in DNA mismatch


syndrome 2p22 MLH1IMLH1 repair/sebaceous neoplasia; internal
malignancy (colon and others)
3p21

Neurofibromatosis AD 17q11 NF1/neurofibromin Negatively regulates RAS


I signaling/neurofibromas

Neurofibromatosis AD 22q12 NF2/merlin Integrates cytoskeletal


II signaling/neurofibromas and acoustic
neuromas

Tuberous sclerosis AD TSC1/hamartin Work together in a complex that


9q34 TSC2/tuberin negatively regulates
mTOR/angiofibromas/ mental retardation
16p13

Xeroderma AR 9q22 and XPA/XPA and Nucleotide excision repair/melanoma and


pigmentosum others others nonmelanoma skin cancers

333. B
Because of their chemical structures, some carcinogens interact preferentially with
particular DNA sequences or bases, and thus produce mutations that are clustered at
“hotspots” or that are enriched for particular base substitutions. One illustrative
example of a chemical carcinogen associated with a mutational “hotspot” is aflatoxin
B1, a naturally occurring agent
produced by some strains of a mold called Aspergillus. Aspergillus grows on
improperly stored grains and nuts, and there is a strong correlation between the
dietary level of this food contaminant and the incidence of hepatocellular carcinoma in
parts of Africa and the Far East. Interestingly, aflatoxin B1
-associated hepatocellular carcinomas tend to have a particular mutation in TP53, a G
: C→T : A transversion in codon 249 that produces an arginine to serine substitution in
the p53 protein. In contrast, TP53 mutations are infrequent in liver tumors from areas
where aflatoxin contamination of food does not occur, and few of these mutations
involve codon 249. Similarly, lung cancers associated with smoking have a 10-fold
higher mutational burden on average than lung cancers in nonsmokers, and these
excess mutations are strongly skewed toward particular base substitutions known to
be caused by carcinogens in cigarette smoke (the proverbial “smoking gun”).

334. B
The UV portion of the solar spectrum can be divided into three wavelength ranges:
UVA (320-400 nm), UVB (280-320 nm), and UVC (200-280 nm). Of these, UVB is
believed to be responsible for the induction of cutaneous cancers. UVC, although a
potent mutagen, is not considered significant because it is filtered out by the ozone
layer surrounding the earth (hence the concern about ozone depletion).

335. C

Gene(s) Function Tumor (Approximate Frequency of Mutation)

DNMT3A DNA methylation Acute myeloid leukemia (20%)

MLL1 Histone methylation Acute leukemia in infants (90%)

MLL2 Histone methylation Follicular lymphoma (90%)

CREBBP/ Histone acetylation Diffuse large B cell lymphoma (40%)


EP300

ARID1A Nucleosome positioning/chromatin Ovarian clear cell carcinoma (60%), endometrial


remodeling carcinoma (30%-40%)

SNF5 Nucleosome positioning/chromatin Malignant rhabdoid tumor (100%)


remodeling

PBRM1 Nucleosome positioning/chromatin Renal carcinoma (30%)


remodeling
336. C

Malignancy Translocation Affected Genes

Chronic myelogenous leukemia (9;22)(q34;q11) ABL 9q34 BCR 22q11


(CML)

Acute myeloid leukemia (AML) (8;21)(q22;q22) (15;17) AML 8q22 ETO 21q22 PML 15q22 RARA
(q22;q21) 17q21

Burkitt lymphoma (8;14)(q24;q32) MKC 8q24 /Gtf 14q32

Mantle cell lymphoma (11;14)(q13;q32) CCND111 q13 IGH14q32

Follicular lymphoma (14;18)(q32;q21) IGH 14q32 BCL2 18q21

Ewing sarcoma (11;22)(q24;q12) FLI1 11 q24 EWSR1 22q12

Prostatic adenocarcinoma (7:21 )(p22;q22) (17:21) TMPRSS2 (21 q22.3) ETV1 (7p21.2) ETV4
(p21;q22) (17q21)

337. A
An important emerging concept is that tumor cells actively inhibit tumor immunity by
engaging normal pathways of immune regulation that serve as “checkpoints” in
immune responses. Through a variety of mechanisms, tumor cells may downregulate
the expression of costimulatory factors on antigen-presenting cells, such as dendritic
cells; as a result, the antigen presenting cells fail to engage the stimulatory receptor
CD28 and instead activate the inhibitory receptor CTLA-4 on effector T cells. This not
only prevents sensitization but also may induce long-lived unresponsiveness in tumor-
specific T cells. Tumor cells also may upregulate the expression of PD-L1 and PD-L2,
cell surface proteins that activate the programmed death-1 (PD-1) receptor on effector
T cells. PD-1, like CTLA-4, may inhibit T cell activation. Antibodies that block the
inhibitory CTLA-4 or PD-1 receptors have produced promising results in clinical trials
conducted in patients with advanced-stage solid tumors. Additional clinical trials are
being planned using both PD-1 and CTLA-4 blocking antibodies in combination with
each other and with conventional or targeted chemotherapy. The success of these
agents has led to a new paradigm in cancer immunotherapy, sometimes called
“checkpoint blockade”. This is centered on the idea that treatments that remove the
“brakes” imposed by tumors on host anti-tumor immune responses can be highly
effective in treating cancer.

338. D

339. A

340. C

341. D

342. D

343. A

344. A

345. A

346. A, B, C
347. D

348. A

349. D

350. D

351. A

352. A

353. B

354. B

355. C

356. B

357. A

358. C

359. B

360. D

361. D

362. B

363. c
Red Blood Cells and its Disorders –
Questions
364. What should be the sequence of events during collection of blood
sample?
a. Ask the patient his name verify from file → collect blood → Label the sample at
bedside
b. Look at the file → Collect sample → Label the sample at bedside
c. Pre label the sample vials → Check the file patient details → Collect sample
d. Collect sample → Confirm name from file → Label the sample vial

365. Which vacutainer is used for electrolyte estimation?


a. Na Citrate
b. EDTA
c. Fluoride
d. Lithium heparin

366. What does the red cell distribution width (RDW) deal with?
a. Hypochromia
b. Anisocystosis
c. Poikilocytosis
d. Anisochromia

367. A 25-years-old female came to OPD 1 year after postpartum. She was
treated for iron deficiency anemia while pregnancy. Now she is pale and her
HB was 5% and reticulocyte count was 9%. Her corrected retic count is?
a. 6
b. 4.5
c. 3
d. 1

368. A 26-years-old patient having RBC count 2 lakhs/mm3, PCV 30% and
haemoglobin 9.3, What is the type of anemia?
a. Iron deficiency anemia
b. Folic acid deficiency anemia
c. Thalassemia
d. Sideroblatic anemia

369. Spur cells are seen in:


a. Hereditary spherocytosis
b. G6PD deficiency
c. Liver disease
d. Sideroblastic anemia

370. Supravital staining is used for?


a. Nucleated RBCs
b. Reticulocytes
c. Basophils
d. Myeloblasts

371. Howel-Jolly bodies may be seen after-


a. Hepatectomy
b. Splenectomy
c. Pancreatectomy
d. Cholecystectomy

372. All are true about polycythemia vera except?


a. Increased ESR
b. Decreased erythropoietin
c. Increased LAP score
d. Increased blood volume

373. A 25-years-old female presented in December month with chronic fatigue


and cyanosis with bluish lips and arthralgia. Peripheral blood film is shown
below. What is the likely cause?

a. Cold AIHA
b. Warm AIHA
c. Hemoglobinopathy
d. G6PD Deficiency
364. 665. 366. 367. c 368. 369. c 370. 371. b 372. a 373. a
a d b a b

374. Which of the following is absent in haemolytic anemia? (PGI)


a. Increased indirect bilirubin
b. Increased direct bilirubin
c. Increased reticulocyte count
d. Jaundice
e. Increased LDH

375. Which one of the following is not associated with a high reticulocyte
count?
a. Acute bleed
b. Haemolytic anemia
c. Megaloblastic anemia
d. Response to treatment in ‘nutrition – deficiency anemia

376. Adult hemoglobin consists of the following chains?


a. 2α + 2β
b. 2α + 2δ
c. 2β + 2ϒ
d. 2α + 2ϒ

377. Direct coomb’s test is positive in haemolytic anemia due to:


a. Paroxysmal cold hemoglobinuria
b. Paroxysmal nocturnal hemoglobinuria
c. Idiopathic thrombocytopenic purpura
d. Hemolytic uremic syndrome

378. Bart hemoglobin is tetramer of:


a. α chain
b. β chain
c. ϒ chain
d. δ chain

379. A 29-years-old female who has hereditary spherocytosis and mild anemia
has developed a low grade fever with malaise. After a week she feels very
tired and appears pale. Her haematocrit drops from the usual value of 36%
to 28%. There is no change in the appearance of red cells morphology.
Reticulocytes are absent from the following events has most likely occurred:
a. Development of anti-red cell antibodies
b. Disseminated intravascular coagulopathy
c. Accelerated extravascular hemolysis in the spleen
d. Reduced erythropoiesis from parvo virus infection

380. CD59 deficiency leads to:


a. PNH
b. Chediak-Higashi disease
c. Hairy cell leukemia
d. Hemolytic uremic syndrome

381. All decrease in iron deficiency anemia except?


a. Ferritin
b. TIBC
c. Iron
d. Transferrin saturation

382. Which of these is not involved in iron metabolism?


a. Hepcidin
b. Ferroportin
c. Transthyretin
d. Ceruloplasmin

383. Which of the following is accurate regarding the internal iron


homeostasis in iron deficiency anemia:
a. Transferrin receptor 1-iron responding elements increases transferrin receptor
mRNA concentration and synthesis
b. Ferritin mRNA concentration – iron response element increases and ferritin
synthesis decreases
c. Ferritin mRNA concentration – iron response element decreases and ferritin
synthesis increases
d. Transferrin receptor 1-iron responding elements decreases transferrin receptor
mRNA concentration and increases synthesis
374. 375. c 376. 377. 378. c 379. 380. 381. b 382. c 383.
b a a d a

384. Match List-I with List-II and select the correct answer using the code
given below the Lists?

385. Criteria for severe aplastic anemia are all except?


a. BM cellularity >25%
b. Reticulocyte < 60,000/ul
c. Platelet < 20,000/ul
d. Absolute neutrophil count < 1500/ul

386. 2 years-old child presents with short stature and café-au-lait spots. Bone
marrow aspiration yields a little material and mostly containing fat. What is
your diagnosis:
a. Fanconi anemia
b. Dyskeratosis congenita
c. Tuberous sclerosis
d. Osteogenesis imperfecta

387. Dry tap is a feature:


a. Anemia of chronic disease
b. Megaloblastic anemia
c. Aplastic anemia
d. Sickle cell anemia

388. Erythropoeitin acts on


a. CFU
b. BFU
c. Pro erythroblast
d. Reticulocyte

389. Intravascular haemolytic anemia is seen in all except-


a. Autoimmune haemolytic anemia
b. TTP
c. Thalassemia
d. Infection

390. Shape of RBC is biconcave due to-


a. Ankyrin
b. Spectrin
c. Band protein
d. Glycophorin-C

391. Not true about hereditary spherocytosis-


a. Defect in ankyrin
b. Decreased MCV
c. Decreased MCHC
d. Reticulocytosis

392. A 23-years-old female presented with jaundice and pallor for 2 months.
Her peripheral blood smear shows the presence of spherocytes. The most
relevant investigation to arrive at a diagnosis is-
a. Reticulocyte count
b. Osmotic fragility test
c. Coombs’ test
d. Tests for PNH

393. What is affected in HBS (Hemoglobin S)


a. Stability
b. Function
c. Affinity
d. Solubility

394. A 6 years old Afro-American boy develops recurrent infections, severe


abdominal pain, and a non-healing ulcer over left foot. RBC has abnormal
shape on microscopy. Which of the following is correct?
a. Trinucleotide repeat mutation
b. Single base substitution
c. Antibodies against RBC membrane
d. Genomic imprinting

395. Autosplenectomy is seen in-


a. Hereditary spherocytosis
b. G6PD deficiency
c. Sickle cell anemia
d. Thalassemia major

396. One of the common variants of sickle cell disease is frequently marked by
lesser degree of haemolytic anaemia and greater propensity for the
development of retinopathy and aseptic necrosis of bones:
a. Sickle cell trait
b. HBSC disease
c. Sickle thalassemia
d. Sickle HbE disease

384. c 385. 386. 387. c 388. 389. c 390. 391. c 392. c 393. d
d a a b

394. 395. c 396.


b b

397. In α-thalassemia- hemolysis is due to


a. Excess α-chain
b. No α-chain
c. Excess β-chain
d. No β-chain

398. Thalassemia gives protection against-


a. Filaria
b. Kala azar
c. Malaria
d. Leptospirosis

399. Paroxysmal nocturnal hemoglobinuria is due to-


a. Acquired red cell defect
b. Congenital red cell defect
c. Autoimmune defect
d. Lead poisoning

400. Which is not a feature of paroxysmal nocturnal hemoglobinuria?


a. Increased LAP score
b. Thrombosis
c. Thrombocytopenia
d. Hemolysis

401. Cold agglutinin is-


a. IgG
b. IgM
c. IgA
d. IgD

402. HbH disease is associated with-


a. Deletion of 3 alpha genes
b. Deletion of 4 alpha genes
c. Deletion of 3 beta genes
d. Deletion of 4 beta genes

403. Red cells containing granules of non-haem iron which gives positive
Prussian blue reaction with Perl’s stain as well as stain with Romanowsky
dyes (referred to as pappenheimer bodies) are known as-
a. Schistocytes
b. Spherocytes
c. Sideroblasts
d. Siderocytes

404. Earliest manifestation of megaloblastic anemia-


a. Macrocytosis
b. Hypersegmented neutrophils
c. Basophilic stippling
d. Cabot ring formation

405. The best suited anticoagulant for osmotic fragility test is-
a. Heparin
b. EDTA
c. Trisodium citrate
d. Potassium oxalate

406. Causes of a dry tap when performing a bone marrow aspiration may be
expected in:
a. Iron deficiency anemia
b. Refractory anemia with ringed sideroblasts
c. Hairy cell leukemia
d. Hereditary spherocytosis

407. In a patient with β thalassemia trait, all of the following features are
typically observed except:
a. Significantly high RDW
b. Mild microcytic hypochromic anemia
c. Target cells on the peripheral smear
d. Disproportionately elevated RBC count

408. A 30-years-old male with long-standing history of diarrhea, has


macrocytosis, Howell-Jolly bodies in RBC, target cells and small shrunken
spleen. The most likely diagnosis is:
a. Sickle cell disease
b. Celiac disease
c. Crohn’s disease
d. Liver disease
409. Iron deficiency may result in the misdiagnosis of:
a. Sickle cell trait
b. Sickle cell disease
c. β thalassemia trait
d. β thalassemia disease
397. c 398. c 399. 400. 401. 402. 403. 404. a 405. a 406. c
a a b a d

407. 408. 409. c


a d

410. A child with a known diagnosis of hereditary spherocytosis develops


“slapped cheek” appearance as well as profound anemia. Reticulocyte
response to the anemia is inadequate. A bone marrow examination is most
likely to demonstrate:
a. Hypocellular marrow with reduction of all cell lines
b. Erythroid hyperplasia as a response to acute hemolysis
c. Reduced erythropoiesis with giant erythroblasts and intranuclear inclusions
d. “Owl eye” appearance, due to CMV infection in erythroid precursors

411. All statements regarding hereditary spherocytosis is true except:


a. May be diagnosed by flow cytometry
b. May be diagnosed by incubated osmotic fragility test
c. When inherited, most cases are autosomal recessive
d. The direct antiglobulin test (DAT) is typically negative

412. All of the following statements regarding paroxysmal nocturnal


hemoglobinuria (PNH) are true except:
a. PNH is transmitted as X-linked recessive
b. PNH may be diagnosed by flow cytometry
c. PNH may be diagnosed by Ham’s acidified serum test
d. Occurs due to mutation of the PIG-A gene

413. Spiculated RBCs with evenly spaced spikes are called:


a. Acanthocytes
b. Stomatocytes
c. Echinocytes
d. Dacryocytes

414. Which of the following indicates intravascular hemolysis?


a. Target cells
b. Schistocytes
c. Acanthocytes
d. Basophilic stippling

415. Pappenheimer bodies are composed of?


a. Copper
b. Zinc
c. Iron
d. Lead

416. Blister cells seen in which condition?


a. G6PD deficiency
b. Thalassemia
c. Sickle cell anemia
d. AIHA

417. Most common mutation in hereditary elliptocytosis:


a. Spectrin
b. Ankyrin
c. Glycophorin A
d. Band 4.2

418. In hereditary spherocytosis, which gene is altered?


a. Spectrin
b. Laminin
c. Desmin
d. Vimentin

419. Which of the following tests is gold standard to diagnose a case of PNH?
a. HAMS test
b. Sucrose lysine test
c. Flow cytometry
d. Bone marrow

420. Which of the following are true regarding a child with iron deficiency
anaemia:
a. Raised MCV
b. Raised transferrin saturation
c. Increased TIBC
d. Increased ferritin

421. Following are causes of megaloblastic anemia except?


a. Defect in DNA synthesis
b. Folic acid deficiency
c. Lead toxicity
d. Vit. B12 deficiency
410. c 411. c 412. 413. c 414. 415. c 416. 417. a 418. a 419. c
a b a

420. c 421. c
422. Iron metabolism and regulation are important for RBC precursor cell.
Which of the following helps in regulation of iron metabolism but is not
specific for iron?
a. Hepcidin
b. DMT-1
c. Ferroportin
d. Ferritin

423. Transfer of Iron from enterocyte to plasma is inhibited by?


a. Hepcidin
b. DMT-1
c. Ferroportin
d. Haphaestin

424. Parvovirus preferentially involves in which of the following cells:


a. Erythroid progenitors
b. Myeloid precursors
c. Megakaryocytes
d. Both a & c

425. Most common tumor associated with pure red cell aplasia:
a. Hepatoma
b. Hodgkins lymphoma
c. Thymoma
d. Bronchogenic carcinoma

426. Anemia with reticulocytosis is seen in-


a. Hemolysis
b. Iron deficiency anemia
c. Vitamin B2 deficiency
d. Aplastic anemia

427. Progenitor hematopoietic stem cells originate in-


a. Bone marrow
b. Thymus
c. Lymph node
d. Spleen

428. Osmotic fragility is increased in


a. Sickle cell anemia
b. Thalassemia
c. Hereditary spherocytosis
d. Chronic lead poisoning

429. Most common cause of spherocytes -


a. Autoimmune haemolytic anemia
b. Vit B12 deficiency
c. Aplastic anemia
d. Heraditary spherocytosis

430. In sickle cell anemia all are seen except-


a. Sickle cells
b. Target cells
c. Howell Jolly bodies
d. Ringed sideroblast

431. Person having heterozygous sickle cell trait is protected from infection
of-
a. Plasmodium falciparum
b. P. vivax
c. Pneumococcus
d. Salmonella

432. HbA2 is raised in-


a. Beta trait thalassemia
b. Sickle cell anemia
c. Hereditary spherocytosis
d. None

433.. In microangiopathic haemolytic anemia, cells seen are-


a. Schistocytes
b. Stomatocytes
c. Spherocytes
d. Anisocytes

434. Response to iron in iron deficiency anemia is denoted by-


a. Restoration of enzymes
b. Reticulocytosis
c. Increase in iron binding capacity
d. Increased in haemoglobin

435. Most common peripheral smear finding in “anemia of chronic disease” is-
a. Microcytic hypochromic
b. Normocytic normochromic
c. Macrocytic
d. None

436. A 40 years old male had undergone splenectomy 20 years ago. Peripheral
blood smear Examination would show the presence of-
a. Dohle bodies
b. Hypersegmented neutrophils
c. Spherocytes
d. Howell-Jolly bodies

422. 423. 424. 425. c 426. 427. 428. c 429. a 430. d 431. a
a a a a a

432. 433. 434. 435. 436.


a a b b d

437. The stain used for the demonstration of reticulocyte is-


a. Wright stain
b. Brilliant cresyl blue
c. Alcian blue
d. Giemsa stain

438. Microspherocytosis in peripheral blood smear are seen in-


a. Congenital spherocytosis
b. Autoimmune acquired hemolytic anemia
c. Thalassemia
d. All of the above

439. Which of the following is an effective marker for erythroid precursors?


a. CD34
b. CD71
c. CD41
d. CD61

440. The following are established causes of bone marrow granulomas:


a. Hodgkin lymphoma
b. Metastatic disease
c. Histoplasmosis
d. All of the above

441. In adults, bone marrow metastasis from the following primary tumors are
common except:
a. Prostatic carcinoma
b. Meningioma
c. Bronchogenic carcinoma
d. Breast cancer

442. Peripheral blood findings most characteristic of metastatic tumor to the


bone marrow is:
a. Leukoerythroblastic blood picture and presence of tear drop cells
b. Microcytic hypochromic red cells
c. Macrocytic anemia
d. Leukocytosis

443. The following are true about reticulocytes except:


a. They are young red cells and contain ribosomal RNA
b. Reticulocyte count is traditionally expressed as a percentage
c. Reticulocyte count is low in bone marrow failure patients
d. Reticulocyte count is low in haemolytic anemia patients

444. The following are all included in the differential diagnosis of microcytic
hypochromic anemia except:
a. HbE disease
b. β thalassemia trait
c. Iron deficiency
d. HbS disease

445. In a patient with iron deficiency, which of the following features are an
expected finding:
a. Minimal anisocytosis
b. Abundant target cells
c. Reduced transferrin iron binding capacity
d. Elevated levels of free erythrocyte protoporphyrin (FEP)

446. Anemia of chronic disease is characterized by all of these except:


a. Low levels of serum iron
b. Reduced transferrin saturation
c. Reduced iron binding capacity
d. Reduced serum ferritin

447. Bone marrow findings in megaloblastic anemia include all of the following
except:
a. Nuclear cytoplasmic asynchrony of developing erythroid cells
b. Giant metamyelocytes
c. Hypersegmented megakaryocytes
d. Absent stainable iron in the bone marrow

448. A patient with known sickle cell disease registers with a new physician.
CBC ordered by the patient’s new physician demonstrates macrocytosis of
the red blood cells. The most likely explanation is:
a. Iron deficiency
b. B12 deficiency
c. Folate deficiency
d. Patient is on hydroxyurea
437. 438. 439. 440. 441. 442. 443. 444. d 445. a 446. d
b b b d b a d

447. 448.
d b
Red Blood Cells and its Disorders –
Explanations
364. A
WHO Guidelines
365. D
366. B
Red cell distribution width (RDW) is a parameter that measures variation in red blood
cell size or red blood cell volume. RDW is elevated in accordance with variation in red
cell size (anisocytosis), ie, when elevated RDW is reported on complete blood count,
marked anisocytosis (increased variation in red cell size) is expected on peripheral
blood smear review.
The reference range for RDW is as follows:
RDW-SD 39-46 fL
RDW-CV 11.6-14.6% in adult (usually followed)
Option A- means low MCHC (central pallor of the cell will be more than 2/3rd)
Option C- means variation in shape of RBCs; needs a peripheral smear examination to
determine
Option D- means variation in chromicity; not a routinely used parameter

367. C
Corrected reticulocyte count = (reticulocyte % × hemoglobin)/15

368. A
Remember these formulae to calculate in questions what’s missing
1. MCV = PCV/RBC x 10
2. MCH = Hb/RBC x10
3. MCHC = Hb/PCV x100
So here MCV= (30/2) x 10 = 150 which is macrocytic anemia and only one of the
options is macrocytic, rest are microcytic anemias.

369. C
370. B
Supravital stains like brilliant cresyl blue, new methylene blue and crystal violet are
used in hematology to stain inclusions like Heinz bodies and HbH inclusions and the
immature RBCs reticulocytes.

371. B

372. A (single best answer)


The ESR is decreased in polycythemia, hyperviscosity, sickle cell anemia, leukemia,
low plasma protein (due to liver or kidney disease) and congestive heart failure.
Although increases in immunoglobulins usually increase the ESR, very high levels can
reduce it again due to hyperviscosity of the plasma.

373. A
Cold agglutinin disease- mediated by IgM (thus causing agglutination as seen in the
peripheral smear)

374. B
Hemolysis causes release of intra cellular enzymes like LDH thus leading to increased
levels. Breakdown of Hb causes increased indirect bilirubin and jaundice.
Compensatory hyperplasia in marrow causes increased reticulocyte counts for most
hemolytic anemias.

375. C > A

376. A
Option B: HbA2
Option D: HbF

Hemoglobin Chain Composition Primary Site Appearance

ζ2 ε2
Gower-1 Yolk sac <5-6 weeks

Gower-2 α2 ε2 Yolk sac 4-13 weeks

Portland ζ2 γ2 Yolk sac 4-13 weeks

Fetal (F) α2 γ2 Liver Early, 53-95% at term

Adult (A] α2 β2 Marrow 9 weeks, 5-45% at term

377. A
PCH is an IgG mediated hemolytic anemia, which can be detected via Coombs test.
Option B- not an antibody mediated hemolytic anemia
Option C- not a hemolytic anemia- antibody mediated platelet destruction
Option D- microangiopathic hemolytic anemia

378. C
Seen in thalassemias

379. D
Parvovirus infection selectively attacks erythroid precursors thus reducing
erythropoiesis and causing a dip in reticulocyte count and haematocrit.

380. A
Also known as Membrane inhibitor of reactive lysis (MIRL)

381. B

Iron deficiency Chronic disease Hemochromatosis

Serum iron ↓ ↓ ↑

Transferrin or TIBC ↑ ↓ ↓

Ferritin ↓ ↑ ↑

% Transferrin saturation (serum iron/TIBC) ↓↓ – ↑↑


382. C
Option A- regulates iron levels by causing internalisation and proteolysis of ferroportin
Option B- channel for iron to leave the enterocyte in ferrous state, on the basolateral
surface
Option D- helps in conversion of ferrous ion to ferric to bind to tran

383.
Transferrin receptor 1 - iron responding elements increases transferrin receptor mRNA
concentration and synthesis.

Decreasing iron Increasing iron

↑ ↓
Iron uptake

Transferrin receptor synthesis ↑ ↓

Transferrin receptor mRNA concentration ↑ ↓

Iron storsge ↓ ↑

Ferritin synthesis ↓ ↑

Ferritin mRNA concentration No change No change

384. C

List-I (Blood picture) List-II (Type of Anemia)

Microcytic, hypochromic red cells Vitamin B12 deficiency anemia

Macrocytic, hypochromic red cells Thalassemia major

Large number of early, intermediate and late erythroblasts Aplastic anemia

Low reticulocyte count Iron-deficiency anemia

385. D

Classification Criteria

Severe BM cellularity < 25% (or < 50% if < 30% of BM is hematopoietic cells) AND ≥ 2 of the
following:

Peripheral blood neutrophil count < 0.5 × 10 9 /L


Peripheral blood platelet count < 20 × 10 9 /L
Peripheral blood reticulocyte count < 20 × 10 9 /L

Very severe As above, but peripheral blood neutrophil count must be <0.2 × 10 9 /L

Nonsevere Hypocellullar BM with peripheral blood values not meeting criteriafor severe aplastic anemia

386. A
Fanconi anemia is the most frequently reported of the rare inherited bone marrow
failure syndromes (IBMFSs), with approximately 2000 cases reported in the medical
literature. In 1927, Guido Fanconi first reported 3 brothers with macrocytosis,
pancytopenia, and physical abnormalities. Subsequent cases were clinically diagnosed
because of the combination of aplastic and various characteristic physical anomalies.
The vast majority (75%) of individuals with Fanconi anemia have at least one physical
anomaly. The most common are short stature and cutaneous, skeletal, craniofacial,
and genitourinary anomalies. Additionally, approximately 5% of patients with Fanconi
anemia have at least 3 of the defining features of VATER, or VACTERL, association
(vertebral anomalies, anal atresia, cardiovascular anomalies, tracheoesophageal
fistula, renal and/or radial anomalies, limb defects). Furthermore, individuals with an
expanded phenotype VACTERC-H (the highest incidence in the FANCD1/BRCA2
genotype), regardless of hematologic status, must be evaluated for Fanconi anemia.

387. C
Dry tap means no yield on marrow aspirate. It is characteristically seen when
hematopoietic cells in marrow are replaced by fat or by fibrosis.

388. A

389. C
Option A- AIHA can be both inside the vessel (intravascular) and in the spleen
(extravascular)
Option B- Intravascular- physical damage
Option D- Intravascular

390. B
Most abundant cytoskeletal protein in the RBC
Responsible for the biconcavity
Deficiency seen in hereditary spherocytosis and elliptocytosis

391. C Dehydration causes increased MCHC along with the shape change.
Option A- Most common cause of HS
Option B- MCV may be normal or decreased
Option D- Hemolytic anemia causes compensatory erythropoiesis thus raised
reticulocyte count

392. C
Adult age group, short history of anemia and spherocytes suggest autoimmune
hemolytic anemia which can be confirmed with the help of Coombs test.

393. D
Deoxygenated HbS is less soluble thus precipitating into polymers and distorting the
RBC shape.

394. B
History, age, race are all suggestive of sickle cell anemia, thus best answer is B
(glutamate to valine at 6th position of beta globin chain)

395. C
Repeated cycles of infarctions replaced by fibrosis due to occlusion of
microvasculature can be seen in sickle cell anemia.

396. B
In HbSC red cells the percentage of HbS is 50%, as compared with only 40% in HbAS
cells. Moreover, HbSC cells tend to lose salt and water and become dehydrated, which
increases the intracellular concentration of HbS. Both of these factors increase the
tendency for HbS to polymerize. As a result, individuals who are compound
heterozygotes for HbS and HbC have a symptomatic sickling disorder (termed HbSC
disease), but it is milder than sickle cell disease

397. C
Deficiency of alpha leads to excess of beta which tetramerises and precipitates, thus
leading to hemolysis.

398. C
Decreased life span of RBCs is responsible for protection against malaria.

399. A
Only hemolytic anemia with acquired genetic defect is PNH

400. A
PNH is a stem cell defect which can lead to aplastic anemia and hence a low LAP score
is seen.

401. 38 B
IgG is warm hemolysin antibody as well as cold hemolysin antibody (Donath
Landsteiner antibody)

402. A

Clinical Genotype Clinical Features Molecular Genetics


Syndromes

β-Thalassemias

β- Homozygous β- Severe; requires blood Mainly point mutations that lead to


Thalassemia thalassemia (β°/β°, transfusions defects in the transcription, splicing,
major β+/β+, β°/β+) or translation of
Severe but does not require
regular blood transfusions β-globin mRNA
β- Variable (β°/β+, β+/ Asymptomatic with mild or
Thalassemia absent anemia; red cell
intermedia β+, β°/β, β+/β) abnormalities seen

β- Heterozygous β-
Thalassemia thalassemia (β°/β,
minor β+/β)

α-Thalassemias

Mainly gene deletions


Silent –/ α/α Asymptomatic; no red cell
carrier abnormality Asymptomatic, like β-
–/ α/α (Asian) thalassemia minor
α-
Thalassemia –/α –/α (black Asymptomatic, Asymptomatic,
trait African, Asian) like β-thalassemia minor

HbH disease –/– –/ α Severe; resembles β-thalassemia


intermedia Lethal in utero without
Hydrops –/– –/– transfusions
fetalis

403. D
Siderocytes are anucleate erythrocytes with iron-containing (siderotic) cytoplasmic
inclusions. The inclusions can be due to aggregates of iron in the cytoplasm or within
mitochondria (the latter are called Pappenheimer bodies). Can be visualised on both
Romanowsky and Perl’s stain.
A nucleated erythoid cell with the same inclusions (usually within mitochondria) is
termed a sideroblast which can be visualised in marrow only on Perl’s stain.

404. A
Option B- most characteristic finding
Option D- remnants of mitotic spindles (microtubules)

405. A
EDTA and sodium citrate remove calcium which is essential for coagulation. Calcium is
either precipitated as insoluble oxalate or bound in non ionized form
Heparin binds to anti thrombin thus inhibiting interaction of several clotting factors
EDTA is used for blood counts
Sodium citrate is used for coagulation testing and ESR
Any anticoagulant can be used for collecting blood for flow cytometry

EDTA
sodium and potassium salts are powerful anticoagulants
dipotassium salt is the most preferable
excess of EDTA can cause shrinkage and degenerative changes of both red cells and
leukocytes
platelets can swell and disintegrate leading to an artificially high platelet count
leucoagglutination can occur ( naturally occurring anti platelet antiplatelet antibody-
if seen, blood collected again in citrate )

Trisodium citrate
9:1 ratio of blood : coagulant for coagulation studies
4:1 ratio of blood : coagulant for ESR

Heparin
lithium or sodium salt
commonly used for chemistry, gas analysis and emergency tests
does not alter size of red cells
best used for OFT
not suitable for blood counts as it induces platelet and leucocyte clumping
not used to make smears as it gives a blue discoloration with Romanowsky dyes
not used in PCR as it inhibit enzyme activity

406. C
Fibrosis is caused by tumor cells

407. A
Anisocytosis is a feature of IDA, not beta thalassemia trait

408. D
Malabsorption → megaloblastic anemia

409. C
Both are .46 microcytic hypochromic

410. C
Parvovirus B19 infection causes aplastic crisis

411. C
Autosomal dominant is most common inheritance

412. A
Acquired, not an inherited genetic trait

413. C

414. B
415. C

416. A

417. A

418. A

419. C

420. C

421. C

422. A

423. A

424. A

425. C

426. A

427. A

428. C

429. A

430. D

431. A

432. A

433. A

434. B

435. B

436. D

437. B

438. B

439. B

440. D

441. B

442. A

443. D
444. D

445. A

446. D

447. D
Not an iron disorder

448. D
WBC Disorders – Questions
449. Dohle bodes with giant platelets are seen in?
a. May heggelin anomaly
b. Pelger huet anomaly
c. Chediak higashi syndrome
d. Bernard Soulier syndrome

450. Marker of T-lymphocyte is?


a. CD8
b. CD20
c. CD19
d. CD45

451. Eosinophillia is found in?


a. Cryptococcus
b. HPV
c. Stronglyloides
d. Typhoid

452. Dilated endoplasmic reticulum is called as?


a. Asteroid bodies
b. Ferruginous bodies
c. Hirano bodies
d. Dohle bodies

453. Which of the following is a B cell marker? (PGI type)


a. CD22
b. CD10
c. CD1a
d. CD19
e. CD20

454. In an ablated animal, myeloid series cells are injected. Which of the
following is seen after incubation period-
a. RBC
b. Fibroblast
c. T lymphocytes
d. Hematopoetic stem cell

455. Bone marrow biopsy is useful in the diagnosis of- (PGI)


a. CML
b. ALL
c. Aleukemic leukemia
d. Hodgkins disease
e. ITP

456. A patient presented with painless b/I proptosis. What is the next
investigation to diagnose it as chloroma?
a. Blood haemoglobin
b. Peripheral smear
c. Platelets
d. Bone marrow (reticulin)

457. All of the following are seen in the development of T lymphocyte at a


point except?
a. Tdt
b. CD34
c. PAX5
d. CD1a

458. Good prognosis for ALL is?


a. Age > 10 years
b. Hyperdiploidy
c. T cell variant
d. TLC > 1,00,000/ul

459. Most common translocation in acute promyelocytic leukemia? (PGI)


a. t(8:14) b. t(15:17)
c. t(9:22) d. t(8:11)
e. t(11:14)

460. Which one is the best prognostic factor for ALL?


a. Hyperploidy
b. Organomegaly
c. TLC more than 50,000/ul
d. Response to treatment

461. Stain used for diagnosis of granulocytic sarcoma:


a. Myeloperoxidase
b. Leukocyte alkaline phosphatase
c. Nonspecific esterase
d. Neuron specific enolase
449. a 450. a 451. c 452. d 453. 454. a 455. b,c 456. b
a,b,d,e

457. c 458. b 459. b 460. 461. a


d,a

462. Most common type of AML in Down’s syndrome:


a. M2
b. M3
c. M6
d. M7

463. A 17-years-old boy presented with TLC of 138x 109/L with 80% blasts on
the peripheral smear. Chest X-ray demonstrated a large mediastinal mass.
Immunophenotyping of this patient’s blasts would most likely demonstrate-
a. No surface antigens (null phenotype)
b. An immature T cell phenotype (Tdt CD34/CD7 positive)
c. Myeloid markers, such as CD13, CD33 and CD15
d. B cell markers, such as CD19, CD20 and CD22

464. A 15-years-old boy presented with one day history of bleeding gums, sub-
conjunctival bleed and purpuric rash. Investigations revealed the following
results: Hb-6.4 gm/dL; TLC-26,500/mm3, Platelet 35,000/mm3; prothrombin
time-20 sec with a control of 13 sec; partial thromboplastin time-50 sec; and
Fibrinogen 10mg/dL. Peripheral smear was suggestive of Acute Myeloblastic
Leukemia. Which of the following is the most likely-
a. Myeloblastic leukemia without maturation
b. Myeloblastic leukemia with maturation
c. Promyelocytic leukemia
d. Myelomonocytic leukemia

465. Lymphohistocytic variant of Reed Sternberg seen in which subtype of


Hodgkin lymphoma?
a. Nodular sclerosis
b. Lymphocyte rich
c. Lymphocyte predominant
d. Lymphocyte depleted

466. A 20-years-old male presented with cervical lymphadenopathy. Histology


of lymph node shows RS cell with vague nodule formation and background T
reactive lymphocytes. The cells were positive for CD20, LCA, EMA and
negative for CD 15 and CD30. Diagnosis is?
a. NLPHL
b. T cell rich B cell lymphoma
c. Nodular sclerosis Hodgkin
d. CLL

467. EBV is not associated with?


a. Lymphocyte predominant HD
b. Plasmablastic lymphoma
c. Nasopharyngeal Ca
d. Mixed cellularity HD

468. Flow cytometry is done on:


a. Polycythemia
b. Thrombocytosis
c. Basophilia
d. Lymphocytosis

469. CLL/SLL ARISES FROM WHICH CELL?


a. Mature B cell
b. Naïve B cell
c. Centrocytes of germinal center
d. Progenitor B-cell

470. Which of the following immunohisto chemistry marker is used in Cyclin


D1 negative Mantle cell lymphoma?
a. SOX11
b. ITRA1
c. MYD88
d. Annexin V

471. A 35-years-old presented with fever. On examination he had enlarged and


ulcerated tonsils. His peripheral blood smear showed lymphocytosis.
Monospot test was negative. Tonsillectomy was done. The biopsy of the
same showed large cells mixed with lymphocytes. The cells were positive for
CD20, EBVLMP1, MUM1, CD79a. Background cells were positive for CD3. The
cells are negative for CD15. Your most probable diagnosis?
a. Infectious mono-nucleosis
b. Hodgkin lymphoma
c. EBV positive – DLBL
d. EBV positive mucocutaneous ulcer

462. 463. 464. c 465. c 466. 467. 468. 469. b 470. a 471. c
d b a a d

472. PAX-5 is a marker for (PGI type)


a. Diffuse large B-celllymphoma
b. Hodgkins lymphoma
c. Anaplastic lymphoma
d. Lymphoblastic lymphoma
e. AML

473. TRAP positivity is seen in?


a. Hairy cell leukemia
b. ALL
c. Burkitts lymphoma
d. T-cell leukemia

474. Cyclin D1 is expressed in-


a. Follicular lymphoma
b. Chronic lymphoid lymphoma
c. Mantle cell lymphoma
d. Splenic marginal zone lymphoma

475. CD20 is positive in all the following lymphomas except:


a. Mantle cell lymphoma
b. Lymphocyte rich HL
c. Follicular lymphoma
d. Butkitt lymphoma

476. A 60-years-old male living in hilly area has Hb of 16 gm%, TC 21000/ul.


DLC showed metamyelocytes and myelocytes 40%, N25% L40%, E5%.
Platelet count 3.25 lakh/u. He presented with hypertension and on
examination, spleen was just palpable below costal margin. What is the next
step?
a. Bone marrow with reticulin stain
b. JAK STAT mutation assessment
c. Philadelphia chromosome
d. Erythropoietin levels

477. 45/m present with leuko-erythroblastic blood picture with dacrocytes.


What is bone marrow finding?
a. Fatty degeneration with erythroid cell hyperplasia with megakaryocytes
b. Abundant fibrosis
c. Focal cellular marrow with hypocellular areas and atypical megakaryocytes.
d. Hypercellular marrow with prominent blasts

478. Mutation seen in systemic mastocytosis:


a. FGFR1 fusion genes
b. BCR-ABL fusion gene
c. JAK 2 point mutation
d. c-kit point mutation

479. A 60-years-old man presented with fatigue, weight loss and heaviness in
left hypochondrium for 6 months. The hemo-gram showed Hb=10 gm/dL, TLC
5 lakhs/mm3, platelet count 4 lakhs/mm3, DLC neutrophil 55% lymphocytes
4% monocytes 2% basophils 6% metamyelocytes 10%, myelocytes 18%,
promyelocytes 2% and blasts 3%. The most likely cytogenetic abnormality in
this case is:
a. t(1;21) b. t(9;22)
c. t(15;17) d. Trisomy 21

480. A 50-years-old man presents with fatigue and easy bruising. A complete
blood count and peripheral blood smear show leucocytosis with 56%
circulating blasts. The patient has no previous history of myeloid neoplasm
or treatment with chemotherapy. Which of the following findings would
definitively establish a diagnosis of acute myeloid leukemia?
a. Presence of blasts with deeply basophilic and vacuolated cytoplasm
b. Presence of Auer rods in the blasts
c. Presence of small blasts with round nuclei, variably dispersed chromatin,
occasional nucleoli and scant amounts of basophilic cytoplasm
d. Presence of large blasts with round nuclei, dispersed chromatin, prominent
nucleoli and abundant basophilic cytoplasm

481. Detection of which of the following molecular genetic abnormalities is


equivalent to a diagnosis of acute myeloid leukemia, regardless of blast
count?
a. t(8;21)(q22;q22)
b. t(9;11)(q22;q23)
c. t(6;9)(q23;q34)
d. t(3;3)(q21;q26.2)
472. 473. a 474. c 475. b 476. b 477. b 478. d 479. b 480. b
a,b,d

481. a

482. Which of the following molecular genetic abnormalities results in the


disruption of core binding factor-α?
a. Inv(16)(p13.1q22)
b. t(16;16)(p13;1;q22)
c. t(6;9)(p23;q34)
d. t(8;21)(q22;q22)

483. Which of the following statements regarding acute promyelocytic


leukemia with PML-RARA is the most correct?
a. The disease shows a poor response to treatment
b. The disease is almost always preceded by chronic myelogenous leukemia
c. Patients are at high risk for the development of disseminated intravascular
coagulation
d. Most patients present in the seventh or eight decade of life

484. Which of the following findings would not be expected in the blast
population of acute myeloid leukemia with monocytic differentiation?
a. Expression of CD14
b. Expression of CD34
c. Expression of CD64
d. Expression of CD4

485. Which of the following molecular genetic abnormalities would be most


likely to impart a poor prognosis in a patient with acute myeloid leukemia?
a. T(8;21)(q22;q22); RUNX1-RUNX1T1
b. Normal karyotype
c. Complex karyotype
d. T(15;17)(q22;q12); PML-RARA

486. A 70-years-old man presents with pancytopenia for six months. A bone
marrow is performed and shows dysplasia in the granulocyte, erythrocyte
and megakaryocyte lineages. The blast count in the peripheral blood is less
than 1% and the blast count in the bone marrow is less than 5%. The best
diagnosis is:
a. Myelodysplastic syndrome with excess blasts-1
b. Myelodysplastic syndrome with excess blasts-2
c. Myelodysplastic syndrome with multilineage dysplasia
d. Acute myeloid leukemia with myelodysplasia-related changes

487. All of the following are features of accelerated phase of chronic myeloid
leukemia, BCR-ABL1 positive, except:
a. Presence of a second Philadelphia chromosome
b. Presence of 20% blasts in the peripheral blood
c. Presence of 20% basophils in the peripheral blood
d. Persistent thrombocytosis (>1000 x 109/L), unresponsive to therapy

488. Follicular lymphoma is characterized by the following cytogenetic


abnormality:
a. t(8;14) b. t(14;18)
c. t(15;17) d. t(11;14)

489. A characteristic feature of lymph node involved by CLL/SLL is:


a. Effacement of lymph node architecture with presence of large and small cells
b. Presence of pseudofollicles
c. Marked increase in vascularity
d. Large cells resembling Reed-Sternberg cells

490. Thesaurocytes are:


a. Plasma cells with flame shaped cytoplasm
b. Plasma cells with ground glass cytoplasm
c. Plasma cell with intra nuclear inclusion
d. Binucleated plasma cells
482. d 483. c 484. d 485. c 486. a 487. b 488. b 489. b 490. a

491. Which of the following malignancy is associated with underlying


progression and spreads characteristically in a stepwise fashion and hence
staging the disease is an important prognostic factor?
a. Hodgkin’s lymphoma
b. Multiple myeloma
c. Mature T cell NHL
d. Mature B cell NHL

492. International Prognostic index for lymphomas includes the following


prognostic factors, except
a. Stage of disease
b. Number of extralymphatic sites involved
c. LDH
d. Hemoglobin and albumin

493. A 48-years-old woman was admitted with a history of weakness for two
months. On Examination, cervical lymph nodes were found enlarged and
spleen was palpable 2 cm below the costal margin. Her haemoglobin was
10.5 g/dL, platelet count 2.7 x 109/L and total leukocyte count 40 x 109/L,
which included 80% mature lymphoid cells with coarse clumped chromatin.
Bone marrow revealed a nodular lymphoid infiltrate. The peripheral blood
lymphoid cells were positive for CD19, CD5, CD20 and CD23 and were
negative for CD79 B and FMC-7. The Histopathological Examination of the
lymph node in this patient will most likely exhibit effacement of lymph node
architecture by-
a. A pseudofollicular pattern with proliferation centers
b. A monomorphic lymphoid proliferation with a nodular pattern
c. A predominantly follicular pattern
d. A diffuse proliferation of medium to large lymphoid cells with high mitotic rate

494. Burkitt’s lymphoma is positive for-


a. CD5
b. CD15
c. CD20
d. CD25

495. Most common extranodal site of lymphoma in HIV is?


a. CNS
b. GIT
c. Retroperitoneum
d. Mediastinum

496. Post-transplant lymphoma occurs due to proliferation of which of the


following cells-
a. T cell
b. B cell
c. NK cell
d. Monocyte

497. DIC is common in which AML-


a. Monocytic (M5)
b. Promyelocystic (M3)
c. Erythroblastic (M6)
d. Megakaryocytic (M7)

498. A 10 years old child presents with pallor and history of blood transfusion
2 months back. On investigation, Hb 4.5 gm, total count 60000, platelet
count 2 lakh and CD10 (+) ve, CD19 (+) ve, CD117 (+) ve, MPO (+) ve&
CD33(-) ve. What is the most likely diagnosis?
a. ALL
b. AML
c. Undifferentiated leukemia
d. Mixed phenotypic acute leukemia

499. Compared to the other leukemias, hairy cell leukemia is associated with
which of the following infections-
a. Parvovirus B19
b. Mycoplasma
c. Atypical mycobacteria
d. Salmonella

500. Mycosis fungoides is-


a. T cell ymphoma
b. B cell ymphoma
c. Mixed
d. Plasma cell tumour

501. Which of the following M protein immunoglobulin is not synthesized in


multiple myeloma?
a. IgD
b. IgM
c. IgA
d. IgG

491. 492. 493. 494. c 495. 496. 497. 498. d 499. c 500. a
a d a a b b

501.
b

502. Russel bodies are seen in-


a. Lymphocytes
b. Neutrophils
c. Macrophage
d. Plasma cells

503. Not true regarding Waldenstorm’s macroglobulinemia is-


a. Lymphadenopathy is usually present
b. Blood viscosity increased
c. IgM immunoglobulin increased
d. Hypercalcemia

504. Lethal midline granuloma arises from-


a. T cells
b. B cells
c. NK cells
d. Macrophages
505. A patient with increased TLC was investigated for leukemia. On flow
cytometry the following profile was obtained.
CD 34 positive
CD 11c positive
CD 14 positive
CD 64 positive
CD 22 positive
CD 19 positive
CD 10 positive
The diagnosis is?
a. Acute lymphoblastic leukemia with aberrant myeloid expression
b. Acute monoblastic leukemia with aberrant lymphoid expression
c. Mixed phenotypic acute leukemia
d. Acute myeloid leukemia

506. The labelled area in the lymph node can be highlighted by which of the
following markers on immunohistochemistry

a. CD 3
b. CD 20
c. CD 56
d. CD 68

507. A child presents with protrusion of eyes, increased thirst and diuresis
and bony defects in the calvarium. A biopsy was performed and electron
microscopy showed the following. Which marker would confirm the diagnosis
of this condition?
a. S-100
b. MHC II
c. CD 207
d. All of the above

508. All of the following stem cell populations are found within the bone
marrow, except-
a. Endothelial Progenitor cells
b. Myoblast Progenitor cells
c. Mesenchymal stem cells
d. Hematopoietic stem cells

509. Leukocyte common antigen is:


a. CD45
b. CD20
c. CD19
d. CD41

510. CD30 is/are marker for (PGI Type)


a. Anaplastic large cell lymphoma
b. Embryonal carcinoma
c. Squamous cell carcinoma
d. Seminoma
e. Hodgkin’s lymphoma

511. Bimodal distribution is seen in?


a. Hodgkins lymphoma
b. DLBCL
c. ALL
d. CML

502. 506. 504. c 505. c 506. 507. 508. 509. a 510. 511. a
d d b d b a,b,e

512. Primary extranodal neoplasms is/are (PGI type)


a. Burkitt lymphoma
b. Waldeyer’s ring lymphoma
c. Gastric lymphoma
d. Thyroid lymphoma
e. Mycosis fungoides

513. Which of the following is not a characteristic feature of Myelodysplastic


syndrome?
a. Leucoerythroblastic blood picture
b. Pawn ball megakaryocytes
c. Pseudo pelger heut cells
d. Transformation to AML

514. The following is true regarding polycythemiavera?


a. Raised ESR
b. Decreased LAP score
c. Thrombocytopenia
d. Leukocytosis

515. Triad of leukoerythroblastosis, tear drop erythrocytes and large platelets


is seen in:
a. Essential thrombocytosis
b. Primary myelofibrosis
c. Myelodysplastic syndrome
d. Langerhan cell histiocytosis

516. Which of the following conditions is most likely to lead to a dry tap during
bone marrow aspiration?
a. Acute lymphoblastic leukemia
b. Multiple myeloma
c. Megaloblastic anemia
d. Idiopathic myelofibrosis

517. CD marker for Langerhans cellhistiocytosis is?


a. CD 17
b. CD 23
c. CD 1a
d. CD117

518. Which of the following findings would be most useful for establishing
myeloid lineage in acute leukemia?
a. Expression of CD13 by the blasts
b. Expression of CD33 by the blasts
c. Expression of myeloperoxidase by the blasts
d. Expression of cytoplasmic CD3 by the blasts

519. Which of the following antigens is not typically expressed in acute


promyelocytic leukemia with PML-RARA?
a. CD13
b. CD33
c. MPO
d. HLA-DR

520. On which chromosome is the retinoic acid receptor α gene located?


a. 15q22
b. 17q12
c. 18q11.2
d. 11q13
521. Which of the following is the strongest predictor of response to therapy
in acute myeloid leukemia?
a. Percentage of blasts in the peripheral blood
b. Immunophenotype of the leukemic blasts
c. Percentage of blasts in the bone marrow
d. Karyotype of the leukemic blasts

522. All of the following are causes of blasts in the peripheral blood except:
a. Myelodysplastic syndrome
b. Chronic myeloid leukemia, BCR-ABL1+
c. Chronic lymphocytic leukemia
d. Sepsis

523. Which of the following is an unfavourable factor for a child with B-


lymphoblastic leukemia/lymphoma?
a. Being six years of age
b. Presence of t(12;21)(p13.2;q22.1);ETV6-RNX1
c. Presence of MLL (KMT2A) gene rearrangement
d. Lack of central nervous system involvement at presentation

524. Which of the following antigens is specific for B-lymphoblastic


leukemia/lymphoma by flow cytometry?
a. CD19
b. CD10
c. TdT
d. None of the above

512. 513. a 514. d 515. b 516. d 517. c 518. c 519. d 520. b


a,b,c,d,e

521. d 522. c 523. c 524. a

525. Which of the following is not a feature of myelodysplastic syndrome with


ringed sideroblasts?
a. >15% ring sideroblasts (as a percentage of all erythroid cells) in the bone marrow
b. Anemia
c. 2% blasts with Auer rods in the peripheral blood
d. Presence of SF3B1 mutation

526. Which of the following statements regarding t(9;22)(q34;q11.2); BCR-


ABL1 and its resultant fusion proteins is false?
a. The fusion protein p210 is most frequently seen in chronic myeloid leukemia,
BCR-ABL1 positive
b. The fusion protein p190 is most frequently seen in Philadelphia chromosome-
positive B-lymphoblastic leukemia/lymphoma
c. The fusion protein p230 is associated with morphologic features that resemble
chronic myelomonocytic leukemia
d. The fusion protein p230 is associated with increased neutrophilic maturation in
chronic myeloid leukemia

527. The highest incidence of JAK2 mutation is seen in:


a. Polycythemiavera
b. Primary myelofibrosis
c. Essential thrombocythemia
d. Chronic myeloid leukemia, BCR-ABL1 positive

528. All of the following abnormalities may be seen in primary myelofibrosis


except:
a. JAK2 mutation
b. BCR-ABL1 fusion
c. CALR mutation
d. MPL mutation

529. Neoplastic mast cells are positive for;


a. CD20
b. CD3
c. CD25
d. CD117

530. Detection of which of the following antigens by flow cytometry is


diagnostic of T-cell differentiation in acute lymphoblastic leukemia?
a. CD1a
b. Cytoplasmic CD22
c. Cytoplasmic CD3
d. Surface CD4

531. The follicles in a case of follicular lymphoma are expected to be positive


for:
a. CD10
b. Bcl 6
c. Bcl 2
d. All of the above

532. All of the following regarding chronic lymphocytic leukemia/small


lymphocytic lymphoma (CLL/SLL) are true except:
a. CLL/SLL is the most common leukemia in Western countries
b. Radiation exposure does not increase the risk of developing CLL/SLL
c. Most patients at presentation have fever and weight loss
d. CLL/SLL patients are typically over 50 years of age

533. Which of the following imparts a better prognosis for patients with
CLL/SLL?
a. CD38 negativity
b. ZAP-70 positivity
c. Presence of Trisomy 12
d. Lymphocyte doubling time of 8 months

534. Mantle cell lymphoma should be positive for all of the following markers
except:
a. Cyclin D1
b. CD 20
c. CD 10
d. CD 5

535. Which of the following is not a risk factor for the development of
extranodal marginal zone lymphoma (MALT lymphoma)?
a. Sjogren’s syndrome
b. Pernicious anemia
c. Helicobacter pylori infection
d. Hashimoto’s thyroiditis

525. c 526. c 527. 528. 529. c 530. c 531. 532. b 533. a 534. c
a b d

535.
b

536. The following are all features of nodal marginal zone lymphoma except:
a. Tumor cells are CD79a positive
b. Tumor cells are Bcl2 positive
c. Tumor cells are CD10 negative
d. Tumor cells are Bcl6 negative

537. All of the following are characteristic histologic features of Burkitt


lymphoma except:
a. Diffuse growth pattern
b. Starry sky appearance
c. Large cells with inconspicuous nucleoli
d. High mitotic rate

538. Which of the following positive immune histochemical stains should make
one reconsider a diagnosis of Burkitt lymphoma?
a. CD10 positivity
b. Bcl2 positivity
c. Bcl 6 positivity
d. CD20 positivity

539. Which of the following translocations may be seen in Burkitt lymphoma?


a. t(8;14)
b. t(2;8)
c. t(8;22)
d. All of the above

540. Which of the following is a true statement regarding lymphoplasmacytic


lymphoma (LPL)?
a. It is synonymous with Waldenstrom macroglobulinemia
b. This is a high-grade lymphoma
c. It is associated with hepatitis C infection
d. It is not known to transform to a high-grade lymphoma

541. Which of the following variants of diffuse large B-cell lymphoma (DLBCL)
is not associated with EBV infection?
a. DLBCL associated with chronic inflammation
b. Intravascular large B-cell lymphoma
c. Primary effusion lymphoma
d. Lymphomatoid granulomatosis

542. Primary effusion lymphoma patients may be infected with:


a. HIV
b. EBV
c. HHV-8
d. All of the above

543. All of the following are features of hairy cell leukemia except:
a. The disease involves the red pulp of the spleen
b. There is leucocytosis
c. Monocytopenia occurs
d. Bone marrow aspiration may result in a dry tap

544. All of the following markers are expected to be positive in hairy cell
leukemia except:
a. Annexin 1
b. CD25
c. CD103
d. CD3

545. Which of the following neoplastic cell types is not expected to be seen in
a case of classical Hodgkin lymphoma?
a. Reed-Sternberg cell
b. L&H cell
c. Lacunar cell
d. Mummified cell

546. Which of the following is not a subtype of classical Hodgkin lymphoma?


a. Nodular sclerosis
b. Eosinophil rich
c. Lymphocyte depleted
d. Lymphocyte rich
536. b 537. c 538. b 539. d 540. a 541. a 542. d 543. d 544. d

545. b 546. b

547. A 55-years-old Caucasian woman presents for evaluation of fatigue. The


patient states that her symptoms have been present for approximately
eight months and reports two episodes of cellulitis over the last year.
Physical examination reveals mild splenomegaly; no skin lesion,
lymphadenopathy, or other abnormalities are identified. Review of a
complete blood count (CBC) and peripheral blood smear shows absolute
neutropenia, normocytic, normochromic anemia, and mild absolute
lymphocytosis composed of numerous large lymphocytes with abundant
cytoplasm and prominent red-pink granules. Which of the following is the
most likely diagnosis?
a. T-cell prolymphocytic leukemia
b. T-cell large granularlymphocyticleukemia
c. Adult T-cell leukemia/lymphoma
d. Aggressive NK-cell leukemia

548. Classical markers for Hodgkin’s disease


a. CD15 and CD30
b. CD15 and CD22
c. CD15 and CD20
d. CD20 and CD30

549. Lacunar cells are seen in which type of Hodgkin’s lymphoma-


a. Lymphocyte predominant
b. Lymphocyte depletion
c. Nodular sclerosing
d. Mixed cellularity

550. ‘Popcorn-cells’ are seen in which variety of Hodgkin’s disease-


a. Nodular sclerosis
b. Mixed cellularity
c. Lymphocyte predominant
d. Lymphocyte depletion

551. All of the following are true about nodular sclerosis of Hodgkin’s disease
except-
a. Well-formed collagen bands
b. CD15+
c. CD20+
d. Infiltration by plasma cells

552. Which cell is/are in not seen in Hodgkin’s lymphoma?


a. Reed-Sternberg cell
b. Lacunar cell
c. L & H cell
d. Langerhans cell
e. Hodgkin’s cell

553. A 63 years-old man presented with massive splenomegaly,


lymphadenopathy and a total leucocyte count of 17000 per mm1. The flow
cytometry showed CD23 negative and CD5 positive monoclonal B cells with
bright kappa positively comprising 80% of the peripheral blood lymphoid
cells. The most likely diagnosis is-
a. Mantle cell lymphoma
b. Splenic lymphoma with villous lymphocytes
c. Follicular lymphoma
d. Hairy cell leukemia

554. Starry sky appearance is seen in-


a. Burkitt’s lymphoma
b. CLL
c. Diffuse large B cell lymphoma
d. ALCL

555. Maltoma is positive for-


a. CD3
b. CD10
c. CD43
d. CD5

556. Poor prognostic indicator of All is-


a. Female sex
b. Leukocyte count < 50,000
c. Age greater than 1 year
d. Hypodiploidy

557. t (2;8) is characteristically seen with-


a. Pre B cell lymphoma
b. Pre T cell lymphoma
c. Burkitt’s lymphoma
d. Mantlecelllymphoma

558. Most common extranodal site for Non-Hodgkin’s lymphoma is-


a. Stomach
b. Brain
c. Intestines
d. Tonsils

547. c 548. 549. c 550. c 551. c 552. 553. 554. a 555. d 556. d
a d a

557. c 558.
a

559. B ALL is due to-


a. T cells
b. Immature B cells
c. Immature T cells
d. Both T and B cells

560. CD10 is seen in-


a. ALL
b. CLL
c. HCL
d. CML

561. Good prognosis of ALL


a. Hyperdiploidy
b. Hypodiploidy
c. T cell lineage
d. Philadelphia chromosome

562. All of the following are good prognostic factors for ALL except-
a. Age of onset between 2-8 years
b. Initial WBC count less than 50000
c. Hyperdiploidy
d. t (9:22), t (8:14), t (4:11)

563. Nonspecific esterase in present in-


a. Megakaryocytic leukemia
b. Lymphocytic leukemia
c. Erythroleukemia
d. AML M4 and M5

564. What is the chromosomal translocation in AML M3?


a. T (18,21) b. T (15,17)
c. T (8,21) d. T (9,11)

565. AML is characterized by-


a. Philadelphia chromosome
b. Auer rods
c. Hemolyticanemia
d. Dohle bodies

566. A 50 years old patient comes with pancytopenia, splenomegaly and skin
lesion (erythema nodosum). Most probable diagnosis is:
a. Mycosis fungoides
b. Myledysplastic syndrome
c. Hairy cell leukemia
d. Sezary syndrome

567. Marker for hairy cell leukemia is-


a. CD30
b. CD103
c. CD1a
d. CD4

568. Hyperviscosity syndrome occurs with which of the following types of M


protein of Multiple myeloma?
a. IgG1
b. IgG2
c. IgG3
d. IgG4

569. Most common cytogenetic abnormality in multiple myeloma?


a. Deletion 13q
b. t (11;14)
c. t (8;14)
d. t (8;22)

570. A 2 years old child presents with scattered lesions in the skull. Biopsy
revealed Langerhans cells. The most commonly associated is marker with
this condition will be-
a. CD1a
b. CD57
c. CD3
d. CD68

571. Birbeck granules in the cytoplasm are seen in-


a. Mast cells
b. Langerhans cells
c. Thrombocytes
d. Myelocytes

572. CD marker of histiocytosis is-


a. CD1a
b. CD1b
c. CD1c
d. CD1d

573. Dohle bodies-


a. Dilated rough endoplasmic reticulum in neutrophils
b. Mitochondria
c. Golgi apparatus
d. Lysosomes

574. Auer rods are seen maximally in-


a. M2-AML
b. M3-AML
c. M6-AML
d. ALL

575. BCL2 in lymphoma the translocation involving-


a. t (8:14)
b. t (8:12)
c. t (14:18)
d. t (14:22)

559. 560. 561. 562. 563. 564. 565. 566. c 567. b 568. c
b a a d d b b

569. 570. 571. 572. 573. 574. 575. c


b b b a a b
WBC Disorders – Explanations
449. A
May-Hegglin anomaly (MHA) is an autosomal dominant disorder characterized by
various degrees of thrombocytopenia that may be associated with purpura and
bleeding; giant platelets containing few granules; and large, well-defined, basophilic,
cytoplasmic inclusion bodies in granulocytes that resemble Döhle bodies.
Option B- bilobed neutrophils
Option C- large coarse granules with albinism and neuropathies
Option D- giant platelets

450. A
Option B and C- B cell markers
Option D- Leucocyte commonantigen (LCA)

451. C
Helminthic infections induce eosinophilia.

452. D
In sepsis or severe inflammatory disorders (e.g., Kawasaki disease), leukocytosis is
often accompanied by morphologic changes in the neutrophils, such as toxic
granulations, Döhle bodies, and cytoplasmic vacuoles. Toxic granules, which are
coarser and darker than the normal neutrophilic granules, represent abnormal
azurophilic (primary) granules. Döhle bodies are patches of dilated endoplasmic
reticulum that appear as sky-blue cytoplasmic "puddles."
Option A- seen in sarcoidosis
Option B- iron rich proteinaceous material covered particles seen in pneumoconiosis
Option C-

453. A, B, D, E
Option C- thymocytes and Langerhans cells

454. A
Myeloid series cells include RBCs, platelets, monocytes and granulocytes.

455. B, C
Option A- Aspirate is mandatory for diagnosis as counting of cells needed
Option C- absence of tumor cells in blood mandate an investigation of the marrow to
determine blast count
Option E- due to antibodies against platelet antigens, hence marrow will show
megakaryocyte hyperplasia but not specific for diagnosis of ITP

456. B
Chloroma or myeloblastoma or granulocytic sarcoma is extramedullary blast
proliferation which may coexist with acute myeloid leukemia and hence a primary first
line investigation of the peripheral blood is needed.
Option A- Establishing anemia will not contribute to diagnosis
Option C- No history of bleeding- hence not needed here
Option D- to determine fibrosis in marrow- not needed here

457. C
PAX5 is a B cell transcription factor and has no role in T cell development

458. B

Prognostic Factors in Acute Lymphoblastic Leukemia

Determinants Favorable Unfavorable

White blood cell counts <10 × 109/L >200 × 109/L

Age 3–7 y <1 y, >10 y

Gender Female Male

Ethnicity White Black

Node, liver, spleen enlargement Absent Massive

Testicular enlargement Absent Present

Central nervous system leukemia Absent Overt (blasts + pleocytosis)

FAB morphologic features L1 L2

Ploidy Hyperdiploidy Hypodiploidy <45

Cytogenetic markers Trisomies 4, 10, and/or 17 t(9;22) (BCR-ABL)

t(12;21) (TEL-AML1) t(4;11) (MLL-AF4)

Time to remission <14 d >28 d

Minimal residual disease <10-4 >10-3

Prognostic Factors For Remission Duration in Adults with Acute Lymphoblastic Leukemia (All)

Patient Features Prognostic Factor

Age (y)

<30 Favorable

30 Unfavorable

White blood cell count ×(106/ml)

<30,000 Favorable

30,000 (>100,000 for T cell) Unfavorable

Immunophenotype

T-cell ALL Favorable

Mature B-cell ALL; early T-cell ALL Unfavorable

Cytogenetics

12p abnormality; t(10;14)(q24;q11) Favorable

Normal; hyperdiploid Intermediate

t(9;22), t(4;11), t(1;19), hypodiploid, –7, +8 Unfavorable


Response to therapy

Complete remission within 4 wk Favorable

Persistent minimal residual disease Unfavorable

459. B
Option A- Burkitt lymphoma
Option C- CML, ALL AML
Option E- Mantle cell lymphoma, Plasma cell Myeloma

460. D, A
In order of preference, response of treatment, cytogenetics and clinical features are
considered to be important prognostic factors for ALL.

461. A
The MPO reaction is positive in cells of the granulocytic series and may be positive in
monocytes. The basis of the stain is breakdown of hydrogen peroxide by the enzyme
MPO, releasing an oxygen radical that reacts with a soluble substrate (e.g.,
diaminobenzidine or o-tolidine) to form a colored precipitate. MPO is located in the
peroxisomes of neutrophils and monocytes and specific granules of eosinophils.
Lymphoid cells and erythroid precursors are negative.
SBB is a direct stain of phospholipid in granule membranes. The pattern of staining
closely parallels that of MPO, but the SBB reaction is usually stronger. Monoblasts may
contain sudanophilic granules. Erythroid precursors are negative.
Auer rods are MPO- and SBB-positive.
Nonspecific esterase reactivity is found in monocytes. The basis for the esterase
cytochemical stains is enzymatic release of a side chain from a naphthol ring, with
subsequent reaction of the free ring with a soluble color developer to generate a
colored precipitate. The most commonly used substrates for NSE are α-naphthyl
butyrate (ANB) and α-naphthyl acetate (ANA).
Megakaryoblasts are negative with ANB staining. These features may be used to
identify megakaryoblasts
The PAS stain reacts primarily with glycogen, generating a fuchsia-colored precipitate.
Lymphoblasts in ALL often have prominent coarse granular or block PAS staining, but
the stain is of limited utility for leukemia diagnosis, as it is not specific for
lymphoblasts.

462. D
AML M7, Acute megakaryoblasticleukemia is the most common AML in DS. However,
overall most common leukemias are lymphoid in origin.

463. B
History is suggestive of T-ALL (mediastinal mass is a common presentation, especially
in adoloscents), IPT of option B is most appropriate
Option A- undifferentiated leukemia/lymphoma
Option C- AML
Option D- B cell lymphoma/leukemia

464. C
History is suggestive of DIC (clinical features, low platelets, raised PT and APTT and
low fibrinogen) and this is seen most commonly with AML M3 or APML or AML with
t(15;17)

465. C
L and H variant, or LP type of cell is seen in NLPHL type.

466. A
HODGKIN LYMPHOMA- classical vs nodular lymphocyte predominant

Marker Classical RS cells NLPHL – L and H cells

CD 20 Negative Positive

CD 30 Positive Negative

CD 15 Positive Negative

CD 45 Negative Positive

CD 10 Negative Positive

BCL6 Negative Positive

Pax 5 Weak positive Strong positive

Oct2/BOB1 Negative Positive

467. A
Least association of EBV is seen with NLPHL

468. D
To differentiate between benign and malignant lymphocytosis

469. B

Postulated Cells of Origin of Lymphomas Reference- WHO 2017

Lymphoplasma Postulated normal counterpart (cell of Origin)

Mantle cell lymphoma B cell of inner mantle zone (Naive B cell) (CD 5 positive)

CLL/SLL An antigen experienced CD 5 positive cell (either a transformed Naive B cell or


memory B cell)

Follicular lymphoma Germinal centre B cell

Burkitt lymphoma Germinal centre B cell

DLBCL Germinal centre B cell or Post germinal centre activated B cell (GCB vs ABC)

Marginal zone lymphoma Post germinal centre marginal zone (memory B cell)

Lymphoplasmacytic Post germinal centre cell differentiating into plasma cell


lymphoma
Plasma cell myeloma Plasma cells in which Ig genes have undergone class switching and somatic
hypermutation

B-ALL Hematopoetic stem cell or B cell progenitor

T-ALL T cell progenitor

470. A
Option B- Marginal zone lymphoma
Option C- Lymphoplasmacyticlymphoma
Option D- Hairy cell leukemia

471. C

472. A, B, D
B cell transcription factor so absent in option C- T cell condition and in option E-
myeloid condition

473. A
Tumor cells stain positive for acid phosphatase which does not go away on washing
with tartarate.

474. C
Due to t(11;14)

475. B
Others are lymphomas with retained B cell signature

476. D
To differentiate between benign and malignant polycythemia

477. B
LEBR plus dacrocytes= fibrosis in marrow
478. D
Also in seminoma, GIST and AML
CD 117 positive

479. B
Suggestive for CML

480. B
Auer rods (fused granules) are characteristic of myeloblasts

481. A
Regardless of blast count, AML is diagnosed if one of the following cytogenetic
abnormalities are present- t(8;21), t(15;17) and t(16;16) or inv(16)

482. D
CBF alpha or RUNX1T1- option D
CBF beta- options A and B

483. C
Procoagulant molecules made by the tumor cells

484. D
T cell marker
Options A and C- Monocytic differentiation
Option B- stem cell marker

485. C
Prognostic Factors in Acute Myeloid Leukemia Favorable
486. A

Blood Cytopenia Bone Marrow Dysplasia Ringed Sideroblasts Blasts in marrow


(c) (> 10%) (Marrow)

MDS – SLD (Single) UN1/Bicytopenia One cell line < 15% < 5% (No Aner
Rods)
MDS – MLD (Multi) UNI To More than one cell line < 15% < 5% (No Aner
Pancytopenia Rods)

MDS – RS – SLD UNI Cytopenia One or two (Maybe) cell >15% < 5% (No Aner
(Single) lines Rods)

MDS – RS – SLD UNI To More than one cell line > 15% < 5% (No Aner
(Multi) Pancytopenia Rods)

MDS – EB1 (Excess UNI To One or three cell lines None or Any 5 -9% (No Aner
Blasts) Pancytopenia Rods)

MDS – EB2 UNI To One or three cell lines None or Any 10 -19% or Aner
Pancytopenia Rods

487. B
CML-AP is defined by the presence of £ 1 of the following haematological/cytogenetic
criteria or provisional criteria concerning response to tyrosine kinase inhibitor (TKI)
therapy
Haematologica! /cytogenetic criteria8
Persistent or increasing high white blood cell count (> 10 × 109/L), unresponsive to
therapy
Persistent or increasing splenomegaly, unresponsive to therapy
Persistent thrombocytosis (>1000 × 109/L), unresponsive to therapy
Persistent thrombocytopenia (< 100 × 109/L), unrelated to therapy
£20% basophils in the peripheral blood
10—19% blasts in the peripheral blood and/or bone marrow b,c
Additional clonal chromosomal abnormalities in Philadelphia (Ph) chromosome-
positive (Ph+) ceils at diagnosis, including so-called major route abnormalities (a
second Ph chromosome, trisomy 8, isochromosome 17q, trisomy 19), complex
karyotype, and abnormalities of 3q26,2
Any new clonal chromosomal abnormality in Ph+ ceils that occurs during therapy
Provisional response-to-TKl criteria
Haematological resistance (or failure to achieve a complete haematologica!
responsed) to the first TKI
Any haematological, cytogenetic, or molecular indications of resistance to two
sequential TKIs
Occurrence of two or more mutations in the BCR-ABL1 fusion gene during TKI therapy

488. B
Option A- Burkitt lymphoma
Option C- APML
Option D- Mantle cell lymphoma and Multiple myeloma

489. B
Option A- Follicular lymphoma
Option C- T cell lymphomas
Option D- HL, PTCL and ALCL

490. A
Option C- Dutcher bodies

491. A
Ann Arbor staging system is used

492. D

Clinical Risk Factor Point per Risk Factor

Age >60 1

serum LDH>normal 1

performance status 2-4 1

stage 3 or 4 1

extra nodal involvement >1 1

Risk for CNS involvement based on total points: 0 - 1 points is low risk, 2 points is low
to intermediate risk, 3 points is intermediate to high risk, and 4 or 5 points is high risk.

493. A
Diagnosis is CLL
Option A- pathognomonic for CLL

494. C
B cell lymphoma
Option A- positive in MCL and CLL
Option B- HL
Option D- Mastocytosis

495. A
Option B- most common site in immunocompetent for extranodal NHL

496. B
Usually EBV induced (not CMV)

497. B
Option A- gum hypertrophy
Option D- most common in Down’s syndrome and causes myelofibrosis

498. D
Myeloid lineage
MPO (by flow cytometry, immunohistochemistry, or cytochemistry) or
Monocytic differentiation (>2 of the following: non-specific esterase, CD11c, CD14,
CD64, lysozyme)
T-cell lineage
Cytoplasmic CD3 (by flow cytometry with antibodies to CD3 epsilon chain.
Immunohistochemistry using polyclonal anti-CD3 antibody may detect CD3 zeta chain,
which is not T-cell-specific)
or
Surface CD3 (rare in mixed-phenotype acute leukaemias)
B-cell lineage (multiple antigens required)
Strong CD19
with > 1 of the following strongly expressed: CD79a, cytoplasmic CD22, CD10
or
WeakCD19
with >2 of the following strongly expressed: CD79a, cytoplasmic CD22, CD10

499. C
Due to unexplained monocytopenia

500. A
Cutaneous T cell lymphoma

501. B (single best option)


An M protein is found in the serum or urine in about 97% of cases: lgG in 50% of these
cases; lgA in 20%; light chain in 20%; and lgD, lgE, lgM, or biclonal in < 10%. About
3% of cases are non-secretory. The serum M protein is usually > 30 g/L of lgG and > 20
g/L of lgA. In 90% of patients, there is a decrease in polyclonal lg(< 50% of normal).
Other laboratory findings include hypercalcaemia (found in up to 10% of cases),
elevated creatinine (in 20- 30%), hyperuricaemia (in > 50%), and hypoalbuminaemia
(in - 15%).

502. D
Intracytoplasmic Ig Inclusions

503. D
Lymphoplasmacytic lymphoma is a B-cell neoplasm of older adults that usually
presents in the sixth or seventh decade of life. Although bearing a superficial
resemblance to CLL/SLL, it differs in that a substantial fraction of the tumor cells
undergo terminal differentiation to plasma cells. Most commonly, the plasma cell
component secretes monoclonal IgM, often in amounts sufficient to cause a
hyperviscosity syndrome known as Waldenström macroglobulinemia. Unlike multiple
myeloma, complications stemming from the secretion of free light chains (e.g., renal
failure and amyloidosis) are relatively rare and bone destruction does not occur.
Recent deep sequencing studies have shown that virtually all cases of
lymphoplasmacytic lymphoma are associated with acquired mutations in MYD88. The
MYD88 gene encodes an adaptor protein that participates in signaling events that
activate NF-κB and also augment signals downstream of the B-cell receptor (Ig)
complex, both of which may promote the growth and survival of the tumor cells

504. C
Extranodal NK/T celllymphoma

505. C
Reference WHO 2017
Myeloid lineage
MPO (by flow cytometry, immunohistochemistry, or cytochemistry) or
Monocytic differentiation (>2 of the following: non-specific esterase, CD11c, CD14,
CD64, lysozyme)
T-cell lineage
Cytoplasmic CD3 (by flow cytometry with antibodies to CD3 epsilon chain.
Immunohistochemistry using polyclonal anti-CD3 antibody may detect CD3 zeta chain,
which is not T-cell-specific)
or
Surface CD3 (rare in mixed-phenotype acute leukaemias)
B-cell lineage (multiple antigens required)
Strong CD19
with > 1 of the following strongly expressed: CD79a, cytoplasmic CD22, CD10
or
WeakCD19
with >2 of the following strongly expressed: CD79a, cytoplasmic CD22, CD10

506. B
The marked areas show lymphoid follicles which are rich in B cells, highlighted by CD
20. CD 3 is a T cell marker. CD 56 is a marker for NK cells. CD 68 is a marker for
histiocytic cells.

507. D
The clinical features and electron microscopy picture are consistent with Hand
Schuller Christian triad- which is a multifocal unisystem occurrence of Langerhan’s cell
histiocytosis. The markers for LCH are all mentioned (CD 207 is Langerin)

508. B

509. A

510. A, B, E

511. A

512. A, B, C, D, E

513. A

514. D

515. B

516. D

517. C

518. C

519. D

520. B

521. D

522. C

523. C
524. A

525. C

526. C

527. A

528. B

529. C

530. C

531. D

532. B

533. A

534. C

535. B

536. B

537. C

538. B

539. D

540. A

541. A

542. D

543. D

544. D

545. B

546. B

547. B

548. A

549. C

550. C

551. C

552. D
553. A

554. A, C

555. D

556. D

557. C

558. A

559. B

560. A

561. A

562. D

563. D

564. B

565. B

566. C

567. B

568. C

569. B

570. A

571. B

572. A

573. A

574. B

575. C
Platelet Disorders – Questions
576. Bleeding time increased in which of the following conditions?
a. Von Willebrand disease
b. Hemophilia A
c. DIC
d. Both a & c

577. Which of the following is true regarding Bernard-Soulier syndrome?


a. It is due to defect in platelet adhesion
b. It is due to defect in platelet aggregation
c. It is due to defect in platelet receptor GpIb-IX
d. Both a and c

578. Which of the following is/are released from dense granules of platelets?
(PGI)
a. Serotonin
b. Histamine
c. PDGF
d. ATP
e. Lysosome

579. In PT test, the addition of Ca2+ & tissue thromboplastin activates which
pathway?
a. Extrinsic
b. Intrinsic
c. Fibrinolytic
d. Common

580. Both PT and aPTT will be increased in deficiency of? (PGI)


a. Factor 2
b. Factor 5
c. Factor 8
d. Factor 10
e. Factor 12

581. Which of the following is a vitamin K dependent clotting factor? (PGI)


a. Factor XII
b. Factor II
c. Factor VII
d. Factor X
e. Factor IX

582. Von Willebrand disease, all are true except-


a. Factor VIII C deficiency
b. BT prolonged
c. Normal ristocetin test
d. Defective aggregation

583. In Glanzmann thromboasthenia there is a defect in-


a. Platelet aggregation
b. Platelet adhesion
c. Decreased ADP release
d. Disordered platelet secretion

584. A newborn baby presented with profuse bleeding from the umbilical
stump after birth. Rest of the Examination and PT, APTT are within normal
limits. Most of probable diagnosis is-
a. Factor X deficiency
b. Glanzmann thrombasthenia
c. Von Willebrand disease
d. Bernard-Soulier disease

585. What is not associated with DIC-


a. Thrombocytopenia
b. Increased PT
c. Hyperfibrinogenemia
d. Increased FDP

586. Earliest event of vascular trauma is?


a. Vasoconstriction
b. Platelet adhesion
c. Platelet aggregation
d. Vasodilatation

587. PDGF is present in which granules of platelets?


a. Alpha
b. Beta
c. Delta
d. None

588. Which of the following is true regarding Von willebrand disease?


a. Type 1&3 are associated with quantitative defects in vWF
b. Normal platelet count
c. Desmopressin stimulates release of VWF
d. All of the above
576. d 577. d 578. 579. a 580. 581. 582. c 583. a
a,b a,b,d b,c,d,e

584. b 585. c 586. a 587. a 588. d


589. Which of the following is the most common manifestation of
haemophilia?
a. Hemoptysis
b. Hemarthrosis
c. Hematemesis
d. Mucosal bleeding

590. Which coagulation factor is not in circulating form in blood?


a. F XI
b. F X
c. F III
d. F XIII

591. Which of the following is not associated with prolonged prothrombin


time? (PGI)
a. Haemophilia A
b. Von-Willebrand disease
c. Factor VII deficiency
d. Disseminated intravascular coagulation (DIC)

592. Feature(s) of XIII factor deficiency is/are: (PGI)


a. Delayed wound closure
b. Clot solubility tests are abnormal
c. ↑aPTT
d. ↑PT
e. ↑BT

593. Endotoxin first damages?


a. Endothelium
b. RBC
c. Platelets
d. WBC

594. Which of the following measures the intrinsic pathway?


a. CT
b. aPTT
c. TT
d. Prothrombin time

595. An 11 years old boy has elevated prothrombin and activated partial
prothrombin time. What is the most likely defect?
a. Defect in extrinsic pathway
b. Defect in intrinsic pathway
c. Defect in common pathway
d. Defect in platelet function
596. Which of the following is true regarding Factor V Leiden mutation?
a. Increased risk of deep vein thrombosis
b. Factor V becomes resistant to cleavage by protein C
c. Glutamine to Arginine substitution at 506
d. All of the above

597. Where can Von Willebrand factor (VWF) be found in the body?
a. Delta granules of platelets
b. Weibel-Palade bodies
c. Kupffer cells
d. Merkel cells

598. What is the most common inherited bleeding disorder?


a. Hemophilia A
b. Hemophilia B
c. Von Willebrand disease
d. Factor XI deficiency

599. Which factors are part of the intrinsic system of the coagulation
cascade?
a. II, VII, IX, X
b. VII, TF
c. X, V, II, I
d. VIII, IX, XI, XII
e. II, V, VIII, X

600. Which coagulopathy is associated with amyloidosis?


a. Factor V inhibitor
b. Factor X deficiency
c. Factor VIII inhibitor
d. Factor V deficiency
e. Plasminogen activator inhibitor-1

601. Which factor has the shortest plasma half-life?


a. Factor IX
b. Factor XI
c. Factor VII
d. Factor I
e. Factor X

602. Which of the following conditions has the highest incidence of


disseminated intravascular coagulation (DIC)?
a. Solid tumors
b. Aortic aneurysm
c. Advanced lung cancer
d. Sepsis/severe infection
589. b 590. c 591. 592. 593. a 594. b 595. c 596. d 597. b
a,b a,b

598. c 599. d 600. b 601. c 602. d

603. Activated protein C resistance due to Factor V Leiden is caused by which


of the following?
a. A point mutation in Factor V
b. An autoantibody to activated protein C
c. A deletion in protein C
d. A point mutation in protein C
e. Abnormal thrombin binding to Factor V

604. Major source of von Willebrand factor-


a. Erythrocytes
b. Neutrophils
c. Endothelial cells
d. Monocytes

605. Hemophilia A has following except:


a. VIII factor ↓
b. PTT ↑
c. PT ↓
d. Normal BT

606. The presence of small sized platelets on the peripheral smear is


characteristic of-
a. Idiopathic thrombocytopenic purpura
b. Bernard-Soulier syndrome
c. Disseminated intravascular coagulation
d. Wiskott-Aldrich syndrome

607. Which is not true regarding Bernard – Soulier syndrome?


a. Ristocetin aggregation is normal
b. Aggregation with collagen and ADP is normal
c. Large platelets
d. Thrombocytopenia

608. Which of the following statements about platelet function defects is


true?
a. Normal platelet count with prolonged bleeding time
b. Thrombocytopenia with prolonged bleeding time
c. Thrombocytosis with prolonged bleeding time
d. Normal platelet count with normal bleeding time

609. In DIC, following are seen except-


a. Fibrinogen decreased
b. Thrombocytopenia
c. Normal aPTT
d. PT elevation

610. Cryoprecipitate is useful in-


a. Hemophilia B
b. Thrombasthenia
c. Afibrinogenemia
d. Warfarin reversal

611. All of the following are surface bound zymogens except


a. Hageman factor
b. Christmas factor
c. Fletcher factor
d. Factor XI

612. All of the following are features of primary hemostasis except


a. Delayed onset post trauma
b. Sites affected are skin and mucous membranes
c. Thrombocytopenia is a clinical example
d. Deep tissues are usually not involved

613. PT and aPTT are normal in all of the following except


a. Von Willebrand disease
b. Factor XIII deficiency
c. Lupus anticoagulant
d. Vascular disorder of hemostasis

614. Most common cause of bleeding caused by platelet dysfunction in a


myeloproliferative neoplasm is
a. Polycythemia vera
b. Idiopathic myelofibrosis
c. Chronic myeloid leukemia
d. Essential thrombocytosis

615. A male patient with history of bleeding was investigated and was found
to have increased APTT but normal PT. His maternal uncle had a similar
presentation. The next step of investigation should be
a. Factor VIII assay
b. Dilute Russell viper venom test
c. Mixing studies
d. Factor IX assay

603. 604. c 605. c 606. c 607. 608. 609. c 610. c 611. b 612. a
a a a
613. c 614. 615. c
a
Platelet Disorders – Explanations
576. D
Option C- coagulation factor defect
Bleeding time is a laboratory test to assess platelet function and the body’s ability to
form a clot. The test involves making a puncture wound in a superficial area of the
skin and monitoring the time needed for bleeding to stop (ie, bleeding site turns
“glassy”).

577. D
Bernard-Soulier syndrome illustrates the consequences of defective adhesion of
platelets to subendothelial matrix. Bernard-Soulier syndrome is caused by an inherited
deficiency of the platelet membrane glyco protein complex Ib-IX. This glycoprotein is a
receptor for vWF and is essential for normal platelet adhesion to the subendothelial
extracellular matrix. Affected patients have a variable, often severe, bleeding tendency

578. A, B
α-Granules have the adhesion molecule Pselectin on their membranes (Chapter 3) and
contain proteins involved in coagulation, such as fibrinogen, coagulation factor V, and
vWF, as well as protein factors that may be involved in wound healing, such as
fibronectin, platelet factor 4 (a heparinbinding chemokine), plateletderived growth
factor (PDGF), and transforming growth factorβ. Dense (or δ) granules contain
adenosine diphosphate (ADP) and adenosine triphosphate, ionized calcium, serotonin,
and epinephrine.

579. A

Coagulation factor PT APTT

Fibrinogen (Factor I) Increased Increased

Prothrombin (Factor II Increased Increased

Labile factor/ Proaccelerin (Factor V) Increased Increased


Stable factor/Proconvertin (Factor VII) Increased Normal

Anti hemophilic factor (Factor VIII) Normal Increased

Christmas factor (Factor IX) Normal Increased

Stuart Power factor (Factor X) Increased Increased

Plasma thromboplastin antecedent (Factor XI) Normal Increased

Hageman factor (Factor XII) Normal Increased

Fibrin stabilising factor (Factor XIII) Normal Normal

580. A, B, D

581. B, C, D, E
Surface bound zymogens
Factor XII (Hageman factor)
Prekallikrein (Fletcher factor)
Factor XI
Vitamin K dependent zymogens (phospholipid bound)
Factor X
Factor IX (Christmas factor)
Factor VII
Factor II (Prothrombin)
Protein C (acts as an inhibitor when activated)
Cofactors or substrates
HMWK (Fitzgerald factor or Williams factor)
Factor VIII (antihemophilic factor)
Factor V
Fibrinogen
Protein S (Vitamin K dependent cofactor for activated Protein C)

582. C
Patients with von Willebrand disease have defects in platelet function despite a
normal platelet count. The plasma level of active vWF, measured as the ristocetin
cofactor activity, is reduced. Because vWF stabilizes factor VIII, a deficiency of vWF
gives rise to a secondary decrease in factor VIII levels. This may be reflected by a
prolongation of the PTT in von Willebrand disease types 1 and 3. However, except in
rare type 3 patients, adverse complications typical of severe factor VIII deficiency, such
as bleeding into the joints, are not seen. Even within families in which a single
defective vWF
allele is segregating, wide variability in clinical expression is common. This is due in
part to modifying genes that influence circulating levels of vWF, which show a wide
range in normal populations. Persons with von Willebrand disease facing hemostatic
challenges (dental work, surgery) can be treated with desmopressin, which stimulates
vWF release, or with infusions of plasma concentrates containing factor VIII and vWF.

583. A
Bleeding due to defective platelet aggregation is exemplified by Glanzmann
thrombasthenia, which is also transmitted as an autosomal recessive trait.
Thrombasthenic platelets fail to aggregate in response to adenosine diphosphate
(ADP), collagen, epinephrine, or thrombin because of deficiency or dysfunction of
glycoprotein IIbIIIa, an integrin that participates in “bridge formation” between
platelets by binding fibrinogen. The associated bleeding tendency is often severe

584. B
Option A- increased PT and APTT

585. C

586. A

587. A

588. D

589. B

590. C

591. A, B

592. A, B

593. A

594. B

595. C
596. D

597. B

598. C

599. D

600. B

601. C

602. D

603. A

604. C

605. C

606. D

607. A

608. A

609. C

610. C

611. B

612. A

613. C

614. A

615. C
Transfusion Medicine – Questions
616. Blood group AB is denoted by which colour label
a. Pink
b. Red
c. White
d. Yellow

617. All are true about FFP except


a. Frozen within 8 hours of collection of whole blood
b. 240 ml plasma in one unit
c. Can be stored for 5 years
d. Minimal loss of labile coagulation factors

618. Interval between 2 blood donations should be at least


a. 48 hours
b. 6 weeks
c. 12 weeks
d. 20 weeks

619. Platelets can be stored at 22-25 deg C with agitation for how many days?
a. 5 days
b. 7 days
c. 21 days
d. 35 days

620. Patient comes to ER with severe bleeding. Blood group is not known.
Which group FFP will you give to the patient
a. O +ve
b. O –ve
c. Ab +ve
d. A +ve

621. Patient with severe bleeding comes to emergency dept. As per your
demand you get 4 platelet and 2 PRC units from blood bank. What should be
the next step
a. Transfuse PC and store PRC at room temp
b. Transfuse PC and store PRC between 2-4 degrees
c. Transfuse PRC and store PC at room temp
d. Transfuse PRC and store PC in 2-4 degrees

622. All are true about cryoprecipitate except


a. Thawed at -4 deg C
b. 120 units of factor VIII
c. 150 mg of Fibrinogen
d. Can be used for Hemophilia A

623. TRALI is seen within __ hours of transfusion


a. 4
b. 6
c. 8
d. 10

624. Most common complication of blood transfusion is


a. Hypothermia
b. Hypokalemia
c. Hyperkalemia
d. Hypocalcemia

625. Function of glucose in stored blood is


a. Isotonicity
b. Nutrition
c. Preventing hemolysis
d. Maintain ATP

626. Any transfusion should be completed within __ hours of initiation


a. 4 hours
b. 6 hours
c. 8 hours
d. 10 hours

627. All of the following components should be mandatorily tested for


compatibility except
a. RBCs
b. Cryoprecipitate
c. FFP
d. Platelets

616. c 617. c 618. c 619. 620. c 621. c 622. 623. b 624. a 625. c
a

626. 627.
a b

628. In which of the following scenarios is blood radiated


a. BMT patient
b. Blood relative is a donor
c. Following intra uterine transfusion
d. All of the above

629. Most common reaction associated with blood transfusion is


a. Febrile non hemolytic transfusion reaction
b. TRALI
c. GVHD
d. Delayed hemolytic reactions

630. All are true about post transfusion purpura except


a. >5 days after platelet transfusion
b. More in women
c. Platelet specific antibodies against HPA-1a
d. Additional platelet transfusion is not helpful

631. All are true about transfusion in a patient with IgA deficiency except
a. More risk of anaphylaxis with plasma transfusion
b. IgA rich plasma given
c. <1% population affected
d. Washed cellular components given

632. Genes for A and B antigens are on


a. 9p
b. 9q
c. 19p
d. 19q

633. Interval between 2 plasma pharesis should be


a. 48 hours
b. 6 weeks
c. 12 weeks
d. 20 weeks

634. Shelf life of RBCs stored in CPDA is


a. 21 days
b. 28 days
c. 35 days
d. 42 days

635. Platelet transfusion should be completed within


a. 60 minutes
b. 4 hours
c. 6 hours
d. 8 hours

628. d 629. a 630. a 631. b 632. a 633. a 634. c 635. a


Transfusion Medicine – Explanations
616. C
The following colour code is used to differentiate the ABO group label
Blood group O - Blue
Blood group B - Pink
Blood group A - Yellow
Blood group AB – White

617. C
Fresh frozen plasma (FFP) is prepared by separating citrated plasma from whole
blood and freezing it within 8 hours of collection or by freezing citrated apheresis
plasma within 6 hours of collection.
Each unit of FFP prepared from whole blood contains approximately 200 ml of
plasma.
Apheresis plasma may be packaged into 200- or 400-ml bags.
FFP may be stored at -18°C or below for up to 1 year.
Under these conditions, there is minimal loss of activity of the labile coagulation
factors V and VIII.
One milliliter of FFP contains approximately one unit of coagulation factor activity.
After thawing,
FFP may be stored in the refrigerator for up to 24 hours before use

618. C
The interval between two blood donations should be at least 12 weeks.
At least 48 hours must elapse after plasma pheresis or Cytapheresis before whole
blood is collected from a donor
Apheresis should be done only after 90 days of whole blood collection or in an event
when red cells are not returned at the end of pheresis

619. A
Platelets should be stored at 22-24° C (controlled temperature) with continuous
gentle agitation in platelet incubator and agitator.
Can be stored for upto 5 days
ABO compatibility needed whenever possible

620. C. AB +ve
Fresh Frozen Plasma
FFP is plasma prepared from whole blood, either from the primary centrifugation of
whole blood into red cells and plasma or from a secondary centrifugation of platelet
rich plasma.
The plasma is rapidly frozen to –25°C or colder within 8 hours of collection and
contains normal plasma levels of stable clotting factors, albumin, immunoglobulin
and
Factor VIII at a level of at least 70% of normal fresh plasma.
Should be ABO compatible.
Universal Plasma Donor
Plasma components should be compatible with the ABO group of the recipient to
avoid potential haemolysis caused by donor anti-A or anti-B.
Group AB individuals have neither anti-A nor anti-B antibodies in their plasma. Group
AB plasma can therefore be given to patients of any ABO blood group and is often
referred to as the universal plasma donor
Choice of plasma transfusion

Component ABO Group

Recipient ABO Group 1st Choice 2nd Choice 3rd Choice 4th Choice

AB AB (A) (Bj (0)

A A AB (B) (0)

B 8 AB (A) (0)

0 0 A B AB

621. C
This is a straight forward practical question. In a severely bleeding patient first we
have to transfuse PRBC.
Now we will look at the platelets.
If you have a place to store platelets, platelets can be transfused without agitation in
room temperature upto 24 hours.
Seeing the options, the it seems there is a facility to store platelets.
So PC will be the second to be transfused.
In a setting where you don’t have storing option (like hospital ward), platelets should
be transfused first for concern of bacterial contamination (since two options in the
above questions are about storing platelets we presume there is a facility to store
platelets)

622. A. Cryoprecipitated AHF, OR Cryoprecipitate


An extract of FFP that is enriched in high-molecular-weight plasma proteins.
It is prepared by thawing one unit of FFP at 1°C to 6°C
Each unit of this cryoprecipitate contains approximately 80 to 120 units of Factor VIII
and at least 150 mg of fibrinogen.
It also contains factor XIII, fibronectin, and the high-molecular-weight multimers of
vWF

623. B
TRALI
Most common cause of death following blood transfusion
Occurs within 6 hours of transfusion
Most associated with plasma products
Anti HLA class 1 antibodies and anti granulocyte antibodies

624. A
Complications of Massive Blood Transfusion-
Hypothermia (most common complication) (cold fluid infusion and no thermal
regulation)
Hypokalemia (re entry into transfused RBCs, and stress hormone release) >
Hyperkalemia (stored blood has more potassium and there is potassium retention
due to loss of hydrogen ions in response to acidosis)
Metabolic alkalosis (citrate metabolism to bicarbonate) > acidosis
Hypocalcemia (rapid transfusion can cause citrate overload)
Hypomagnesemia (magnesium poor fluid)
Dilutional coagulopathy (most common cause of death following massive blood
transfusion)

625. C
Red Cells
Additives
Sodium chloride: Isotonicity
Mannitol: reduces cell lysis
Dextrose: nutrition
Glucose: osmotic agent to reduce hemolysis
Phosphate: buffer solution
Adenine: for ATP maintenance, increases shelf life
Citrate: anti coagulant
One unit of red cells is often given in 1 to 4 hours, depending on the amount to be
transfused and on the patient’s cardiovascular status.
Infusion of a unit for longer than 4 hours is not recommended, as there is a risk of
bacterial proliferation because the opened unit is at room temperature.
Storage times
CPD : 21 days
CPDA : 35 days
SAGM : 42 days

626. A
In general, it is desirable to complete a red cell transfusion within 2 hours (not
exceeding 4 hours) and a platelet or plasma transfusion within 30–60 minutes.
Any transfusion should be completed within 4 hours of initiation.

627. B
Recipient should receive ABO type specific compatible whole blood or red blood cell
components.
In the absence of ABO type specific blood, group O packed red cells should be
transfused.
Platelet concentrate should be ABO and Rh(D) type specific with the recipient blood
as far as possible
Leucocyte concentrate should be ABO and Rh(D) type specific or compatible with the
recipient blood
ABO compatibility not mandatory for cryoprecipitate but mandatory for fresh frozen
plasma

628. D
Irradiation
Cellular components should be irradiated in order to reduce the risk of post
transfusion GVHD when a patient is identified as being at risk for GVHD
For all immunosuppressed patients including bone marrow transplant (BMT) patients
When blood from a blood relative is used.
In case of exchange transfusion following intra uterine transfusion.
The minimum dose delivered to the blood bag should be 25 Gy + 2
629. A
The most frequent reaction associated with the transfusion of cellular blood
components is a febrile nonhemolytic transfusion reaction (FNHTR). These reactions
are characterized by chills and rigors and a more than 1 deg C rise in temperature.
FNHTR is diagnosed when other causes of fever in the transfused patient are ruled out.
Antibodies directed against donor leukocyte and HLA antigens may mediate these
reactions; thus multiply transfused patients and multiparous women are felt to be at
increased risk

630. A
This reaction presents as thrombocytopenia 7 – 10 days after platelet transfusion and
occurs predominantly in women. Platelet-specific antibodies are found in the
recipient’s serum, and the most frequently recognized antigen is HPA-la found on the
platelet glycoprotein IIIa receptor. The delayed thrombocytopenia is due to the
production of antibodies that react to both donor and recipient platelets. Additional
platelet transfusions can worsen the thrombocytopenia and should be avoided.
Treatment with intravenous immunoglobulin may neutralize the effector antibodies, or
plasmapheresis can be used to remove the antibodies.

631. B
Patients who are IgA-deficient, < 1 % of the population, may be sensitized to this Ig
class and are at risk for anaphylactic reactions associated with plasma transfusion.
Individuals with severe IgA deficiency should therefore receive only IgA-deficient
plasma and washed cellular blood components. Patients who have anaphylactic or
repeated allergic reactions to blood components should be tested for IgA deficiency.

632. A
The first blood group antigen systemrecognized in 1900 was ABO, the most important
in transfusion medicine. The major blood groups of this system are A, B, AB, and O. 0
type RBCs lack A or B antigens. These antigens are carbohydrates attached to a
precursor backbone, may be found on the cellular membrane either as
glycosphingolipids or glycoproteins, and are secreted into plasma and body fluids as
glycoproteins. H substance is the immediate precursor on which the A and B antigens
are added. This H substance is formed by the addition of fucose to the glycolipid or
glycoprotein backbone. The subsequent addition of N-acetylgalactosamine creates the
A antigen, whereas the addition of galactose produces the B antigen. The genes that
determine the A and B phenotypes are found on chromosome 9p and are expressed in
a Mendelian codominant manner. The gene products are glycosyl transferases, which
confer the enzymatic capability of attaching the specific antigenic carbohydrate
Individuals who lack the “A” and “B” transferases are phenotypically type “0,” whereas
those who inherit both transferases are type “AB.” Rare individuals lack the H gene,
which codes for fucose transferase, and cannot form H substance. These individuals
are homozygous for the silent h allele (hh) and have Bombay phenotype (Oh)’

633. A

634. C

635. A
Systemic Pathology – Questions
636. Loss of function mutations in which of the following leads to
Hirschsprung disease
a. MET
b. RAS
c. Rb
d. RET

637. A 56-years-old chronic smoker, mass in bronchus resected. What is the


possible marker to confirm diagnosis?
a. Cytokeratin
b. Vimentin
c. Epithelial membrane antigen
d. Leukocyte common antigen

638. Patient with h/o long standing depressive illness come to emergency with
acute breathlessness. The X-ray shows diffuse infiltrates with predominance
in right middle lobe and right lower lobe. The patient did not survive and the
following picture in the lungs was seen on autopsy. What is the diagnosis?

a. Severe necrosis with fungal hyphae


b. Caseous necrosis
c. Vegetable matter- Aspiration pneumonia
d. Severe necrotizing pneumonia

639. A patient underwent lung transplantation. The resected lung from the
patient showed followig features. What could be your possible diagnosis?

a. Bronchiectasis
b. Lung abscess
c. Lung carcinoma
d. Miliary tuberculosis

640. Endotoxin shock is initiated by-


a. Endothelial injury
b. Peripheral vasodilation
c. Increased vascular permeability
d. Cytokines action

641. Silicosis biopsy which is true about features and radiological correlation?
(PGI)
a. Lower lobe involved
b. Dense collagen and calcifications in the lymph nodes seen
c. Progressive massive fibrosis can be seen as late complication
d. Immune granuloma can be seen
e. Macules may be seen

642. WHO 2015 new inclusion in lung squamous cell carcinoma is/are?
a. Basaloid type
b. Lymphoephtheliod type
c. Papillary type
d. Clear cell variety
e. Small cell variety
636. d 637. a 638. c 639. a 640. d 641. b 642. a

643. Which one of the following markers is/are useful in differentiating


between mesothelioma and adenocarcinoma- (PGI)
a. CEA
b. Cytokeratin
c. Calretinin
d. TTF
e. Vimentin

644. Lung Ca metastasize early is?


a. Adeno ca
b. Sq cell Ca
c. Small cell Ca
d. Large cell ca

645. Which of these is used as a marker in mesothelioma? (PGI)


a. Calretinin
b. TTF-1
c. CK-5/6
d. Glypican
e. D240
646. The highest malignant potential is seen in-
a. a.Crohn’s disease
b. Ulcerative colitis
c. FAP
d. Peutz Jegher syndrome

647. Macrophages containing PAS positive granules and rod-shaped bacilli in


small intestinal mucosa are found in-
a. Agammaglobulinemia
b. Tropical sprue
c. Whipple’s disease
d. Coeliac spure

648. Ectopic ACTH production is seen in:


a. Small cell carcinoma of lung
b. Anaplastic carcinoma of lung
c. Squamous cell carcinoma of lung
d. Adenocarcinoma of lung

649. 1-antitrypsin deficiency is associated with-


a. Centriacinar emphysema
b. Panacinar emphysema
c. Paraseptal emphysema
d. Irregular emphysema

650. Ferruginous bodies are seen in-


a. Silicosis
b. Bysinosis
c. Asbestosis
d. Bagassosis

651. Bilateral hilar lymphadenopathy with non caseating granuloma is seen in-
a. TB
b. Lymphoma
c. Sarcoidosis
d. All of the above

652. A 40-year-old alcoholic man is admitted to the hospital in severe


respiratory distress. The temperature is 38.7°C (103°F), respirations are 32
per minute, and blood pressure is 130/90 mm Hg. He coughs constantly and
expectorates "currant-jelly" sputum. A chest X-ray reveals bilateral diffuse
pulmonary consolidation. Physical examination shows bilateralcrackles,
dullness to percussion over both pulmonary fields, and use of accessory
muscles. The patient subsequently dies from complications of bacterial
sepsis. The lung at autopsy is shown below. What is the appropriate
diagnosis?
a. Atypical pneumonia
b. Bronchopneumonia
c. Lobar pneumonia
d. Pulmonary abscess
643. 644. c 645. 646. c 647. c 648. a 649. b
a,c,d,e a,c,e

650. c 651. c 652. c

653. A 36-year-old man with AIDS presents with fever, dry cough,and dyspnea.
A chest X-ray shows bilateral and diffuse infiltrates.Laboratory studies
reveal a CD4+ cell count of less than 50/L. A lung biopsy discloses a chronic
interstitial pneumonitis and an intra-alveolar foamy exudate. A silver stain
ofa bronchoalveolar lavage is shown in the image. Which of the following
organisms is the most likely pathogen responsible for these pulmonary
findings?

a. Cryptococcus
b. Histoplasma
c. Pneumocystis
d. Mycobacterium

654. A 35-year-old woman with a long history of dyspnea, chroniccough,


sputum production, and wheezing dies of respiratoryfailure following a bout
of lobar pneumonia. She was a non-smoker and did not drink alcoholic
beverages. The lung atautopsy is shown in the image. Which of the following
underlyingconditions was most likely associated with the pathologic changes
shown here

a. Alpha 1 antitrypsin deficiency


b. Cystic fibrosis
c. Goodpasture syndrome
d. Wegener granulomatosis

655. All of the following findings can be seen in the histopathological picture
of a bronchial biopsy of a patient with respiratory insufficiency due to
Asthma except

a. Smooth muscle hypertrophy


b. Basement membrane thickening
c. Eosinophilia
d. Squamous metaplasia

656. A 60-year-old mason complains of shortness of breath, which has become


progressively worse during the past year. A chest X-ray shows small nodular
shadows in both lungs. Pulmonary function studies reveal a pattern
consistent with restrictivelung disease. The patient subsequently develops
congestive heart failure and expires. Autopsy discloses numerous small,
fibrotic nodules in both lungs. Histologic examination of thesenodules is
shown in the image. Which of the following is the most likely diagnosis?
a. Anthracosis
b. Asbestosis
c. Sarcoidosis
d. Silicosis
653. c 654. a 655. d 656. d

657. What is true about Succinate dehydrogenase deficient GIST? (PGI)


a. Negative for C-KIT
b. Younger age
c. Most common site is stomach
d. Sensitive to Imatinib
e. Aggressive clinical course

658. 40-years-old immunocompromized patient presented with complaints of


dysphagia. UGI scopy showed multiple ulcers in the distal esophagus. Biopsy
from the esophagus showed the following. What is the diagnosis?

a. Candida
b. Cytomegalovirus
c. Herpes viral infection
d. Eosinophilic esophagitis

659. A 40-years-old patient presented with heart burn and increased


salivation. UGE scopy was done and biopsy was taken and is as shown below.
Which of the following is the diagnosis?

a. Barret esophagus
b. Adenocarcinoma
c. Esophagitis
d. Squamous cell carcinoma

660. Which of the following is associated with PAS positive macrophages?


a. Whipple disease
b. Abetalipoproteinemia
c. Crohn disease
d. Ulcerative colitis

661. Pathologist examines biopsy from a patient presenting with bleeding per
rectum with a past history of intussuption for the past 6 months.
Histopathology obtained has been shown below.
Identify the Pathology?

a. Tubulo villous adenoma


b. Adenocarcinoma
c. Hamartomatous polyp
d. Juvenile polyposis syndrome

662. The following is a histopathological picture of a lung biopsy from a


patient. Which category would you classify this condition under?

a. Atypicaladenomatoushyperplasia
b. Adenocarcinoma in situ
c. Minimally invasive adenocarcinoma
d. Invasive adenocarcinoma

657. a,b,c 658. b 659. a 660. a 661. c 662. b

663. Which of the following is a feature of Crohn’s disease?


a. Pseudopolyps can be seen
b. Non-caseating granulomas are present
c. Backwash ileitis may be associated with Crohn’s disease
d. Both b & c
664. Which of the following is the earliest change in intestine which occurs in
Crohn’s disease?
a. Cobblestone appearance
b. Aphthous ulcer
c. Perforation
d. Stricture

665. Which gene involved in colonic adenoma to colonic carcinoma?


a. P53
b. Rb
c. K RAS
d. EGFR

666. All of the following conditions are characterized by neoplastic polyps


except:
a. Peutz Jeghers syndrome
b. Gardner syndrome
c. Turcot syndrome
d. Lynch syndrome

667. Mallory bodies contain-


a. Vimentin
b. Cytokeratin
c. Desmin
d. Collagen

668. All of the following drug induced liver injuries are correctly matched with
their causative agents except
a. Valproate: Fatty liver disease
b. Amiodarone: Vascular lesions
c. Allopurinol: Fibrin ring granuloma
d. Angiosarcoma: Vinyl chloride

669. The pathway activated in inflammatory type of hepatocellular adenoma


is
a. HNF1 alpha
b. WNT/Beta catenin
c. JAK/STAT
d. MAPK

670. Menetrier disease is caused by excessive secretion of


a. Transforming growth factor alpha
b. Tumor necrosis factor alpha
c. Transforming growth factor beta
d. IL-1
671. Which of the following gastric polyps are seen associated with familial
adenomatosis polyposis?
a. Inflammatory
b. Hyperplastic
c. Fundic gland
d. Adenomatous

672. Most specific marker for GIST


a. ckit
b. CD 117
c. DOG1
d. Desmin

673. Microdeletion leading to instability of DNA, causing the cell to undergo


degeneration is seen as a marker in which cancer?
a. Colon cancer
b. Breast cancer
c. Small cell lung cancer
d. Melanoma

674. Michaelis Gutmann bodies are seen in-


a. Xanthogranulomatous pyelonephritis
b. Malakoplakia
c. Nail patella syndrome
d. Chronic Pyelonephritis

675. Perivenular fibrosis is caused by:


a. Methotrexate
b. Alcohol
c. OCPs
d. Amiodarone

676. Hallmark of chronic hepatitis:


a. Interface hepatitis
b. Ballooning degeneration of hepatocytes
c. Cholestasis
d. Periportal fibrosis and bridging fibrosis

663. b 664. b 665. a 666. a 667. b 668. b 669. b 670. a 671. d

672. c 673. a 674. b 675. b 676. d

677. A 1 month old child with conjugated billirubinemia and intrahepatic


cholestasis. On Liver biopsy and staining with PAS red coloured granules
were seen inside the hepatocytes. Probable diagnosis is?
a. Alpha 1 Antitrypsin deficiency
b. Congenital hepatic fibrosis
c. Wilson disease
d. Hereditary hemochromatosis

678. Most common gene responsible for hereditary hemochromatosis is?


a. HIV gene
b. HAMP gene
c. TfR2 gene
d. HFE gene

679. Most common gene associated with pancreatic cancer:


a. KRAS
b. SMAD
c. P53
d. Rb

680. Which of the following dyads are correctly matched regarding urinary
casts & associated condition? (PGI)
a. Hyaline casts – may be normally present in healthy person
b. Muddy brown casts-acute tubular necrosis
c. WBC cast – pyelonephritis
d. Epithelial cast – acute glomerulonephritis
e. Myoglobin cast – Rhabdomyolysis

681. Seen in post streptococcal glomerulonephritis (PSGN)?


a. Nephrotic range proteinuria
b. Neutrophilic infiltration of tubules
c. Subepithelial deposits
d. Linear deposits along glomerular basement membrane

682. Most common renal lesions in HIV:


a. MPGN
b. RPGN
c. FSGS
d. Membranous nephropathy

683. Which is seen in Electron microscopy in PSGN?


a. Subpithelial humps
b. Spike and dome appearance
c. Mesangial deposits
d. Subendothelial deposits

684. PSGN is associated with-(PGI)


a. Subepithelial deposits
b. Nephritis along with acute Renal failure rarely
c. Transiently low complement levels
d. HTN and proteinuria
e. Normal complements

685. Increased levels of C3NeF are associated with?


a. Type I MPGN
b. Type II MPGN
c. FSGS
d. Berger Disease

686. A 60-years-old male with complaints for frothy urine and facial puffiness.
The DIF and electron microscopic picture of the patient is given below.
Diagnosis is?

a. Membranoproliferative nephritis
b. Membranous glomerulopathy
c. Minimal change disease
d. FSGS

677. a 678. d 679. a 680. 681. c 682. c 683. a


a,b,c,e

684. 685. b 686. b


a,b,c,d

687. Collapsing variant of FSGS is seen in:


a. NSAIDs
b. Heroin abuse
c. HIV
d. CMV

688. A 50-years-old male presented with blurring of vision. Urine examination


showed proteinuria. Fundus examination showed dot and blot
haemorrhages, microaneurysm and cotton wool spots. Histopathology pic of
kidney given below. Your diagnosis?

a. Kimmelstiel Wilson nodules


b. Crescents
c. Amyloid
d. Segmental sclerosis

689. The intra cytoplasmic vacuoles seen in the Armanni Epstein cell are rich
in-
a. Na and K
b. Glycogen
c. Lipids
d. Ketones

690. A 40-years-old hypertensive male was admitted to the hospital with


sudden onset of headache and altered sensorium. On Examination his Blood
Pressure was observed to be 220/110mm Hg and the patient died four later.
What is likely pathological finding in his kidneys-
a. Small kidney with granular surface
b. Small kidney with petechial hemorrhages
c. Large kidney with waxy appearance
d. Large kidney with granular surface

691. Most common gene associated with renal cell carcinoma is?
a. WT-1
b. BRCA-1
c. VHL
d. PATCH

692. Chromophobe variant of renal cell carcinoma is associated with:


a. VHL gene mutations
b. Trisomy 7 and 17
c. 3 p deletions
d. Monosomy of 1 and Y

693. Cylindrical dilations of renal tubules is seen in-


a. Autosomal Recessive polycystic kidney disease
b. Medullary cystic disease
c. Autosomal dominant polycystic kidney diasese
d. Lipoid nephrosis

694. A 50-year-old woman complains of severe headaches anddizziness. The


patient has a history of repeated urinary tractinfections. The blood pressure
is 180/110 mm Hg. Laboratorystudies show elevated levels of BUN (38
mg/dL) and creatinine(2.8 mg/dL). A CT scan of the lower abdomen reveals
small,irregularly shaped kidneys with deep coarse scars. A
percutaneousrenal biopsy is shown. Which of the following is theappropriate
diagnosis?

a. Acute pyelonephritis
b. Chronic pyelonephritis
c. Nephrosclerosis
d. Renal amyloidosis

695. Cleft like space in atheromatous plaque mainly contains?


a. Smooth muscle cell
b. Fibrous tissue
c. Cholesterol
d. Macrophages

687. c 688. a 689. b 690. b 691. c 692. d 693. a 694. b 695. c

696. Not true about monkeberg medial sclerosis:


a. Calcification of the walls of muscular arteries
b. Typically involving the internal elastic membrane
c. Persons older than age 50 are most commonly affected
d. Calcifications cause significant narrowing of vessel lumen

697. Characteristic histological finding in benign hypertension:


a. Proliferative end arteritis
b. Necrotizing arteriolitis
c. Hyaline arteriosclerosis
d. Cystic medial necrosis

698. Which of the following is seen in kidney in malignant hypertension?


a. Hyaline necrosis
b. Fibrinoid necrosis
c. Medial wall hyperplasia
d. Microaneurysm

699. Small vessel vasculitis is/are? (PGI)


a. Anti GBM ab
b. Takayasu arteritis
c. Kawasaki disease
d. Carcinoma induced vasculitis
e. IgA vasculitis

700. Which is not a characteristic of wegeners granulomatosis:


a. Granuloma in vessel wall
b. Focal necrotising glomerulonephritis
c. Positive for cANCA
d. Involves large vessels

701. Kaposi sarcoma all are true except?


a. Causative agent HHV 8
b. Can be Seen in depressed cell mediated immunity
c. Always associated with HIV
d. Vascular tumor

702. CD marker of angiosarcoma is?


a. CD 10
b. CD 19
c. CD 25
d. CD 31

703. Characteristic pathological finding in carcinoid of heart:


a. Fibrous endocardial thickening of right ventricle and tricuspid valve
b. Collagen deposition in wall of right ventricle
c. Interstitial fibrous thickening of right ventricles
d. Mononuclear inflammatory infiltrate in the wall

704. The following is not an aging change in heart


a. Increased LA cavity size
b. Increased LV cavity size
c. Sigmoid-shaped ventricular septum
d. Lambl excrescences

705. Autopsy specimen of the heart of a patient who diet due to myocardial
infraction was stained with triphenyltetrazolium chloride dye. Colour of
normal part of the heart will be:-
a. Blue
b. White
c. Red
d. Dark brown

706. In Rheumatic carditis, Mac Callum patch is seen in subendothelium of?


a. Right atrium
b. Right Ventricle
c. Left atrium
d. Left ventricle

707. Tigered effect in myocardium is due to:


a. Malignant change
b. Fatty change in heart
c. Seen in rheumatic fever
d. Associated with myocarditis

708. Which of the following breast cancer is bilateral usually?


a. Medullary
b. Lobular
c. Mucinous
d. Apocrine

709. Most common histological subtype associated with BRCA1 mutations is


a. Inflammatory
b. Lobular
c. Papillary
d. Medullary
696. d 697. c 698. b 699. 700. d 701. c 702. d 703. a
a,d,e

704. b 705. c 706. c 707. b 708. b 709. d

710. Highest susceptibility of breast cancer is seen with which gene mutation?
a. P53
b. BRCA1
c. BRCA2
d. Her 2 neu

711. BRCA2 mutated breast cancers show the following molecular profile
a. ER positive Her 2 neu positive
b. ER positive Her 2 neu negative
c. ER negative Her 2 neu positive
d. ER negative Her 2 neu negative

712. All are true about the basal like molecular group of breast cancer except
a. BRCA 1 mutation associated
b. Mainly grade I
c. Early relapse
d. Most common site of metastasis is bone

713. Which of the following is false about Her 2 neu positive breast cancers?
a. Usually low grade
b. Short timing of relapse
c. Can be apocrine or micropapillary in histology
d. P53 germline mutations associated

714. Most important prognostic factor for breast cancer without metastasis?
a. Tumor size
b. Lymph node status
c. ER/PR status
d. Her 2 neu status

715. Single file pattern of tumor cells with minimal desmoplasia is


characteristic of which breast cancer?
a. Medullary
b. Lobular
c. Inflammatory
d. Papillary

716. Involvement of dermal lymphatics is seen in which subtype of breast


cancer
a. Paget’s disease
b. Inflammatory
c. Medullary
d. Lobular

717. Which of the following benign breast lesions has the most risk of
malignant transformation?
a. Epithelial hyperplasia
b. Fibrocystic disease
c. Atypical ductal hyperplasia
d. Sclerosing adenosis

718. Migratory thrombophlebitis seen with all except


a. Carcinoma Breast
b. Carcinoma Colon
c. Carcinoma Prostate
d. Carcinoma Lung

719. Spongiform neurons are seen in-


a. Creutzfeldt-Jakob disease
b. Subacute sclerosing panencephalitis
c. Fatal familial insomnia
d. Cerebral toxoplasmosis
720. Verocay bodies are seen in-
a. Meningioma
b. Glioma
c. Medulloblastoma
d. Schwannoma

721. Which of the following is a DNA repair defect?


a. Bloom syndrome
b. Incontinentiapigmenti
c. Aplastic Anemia
d. Tuberous sclerosis

722. Childhood polycystic disease is -


a. Autosomal dominant
b. Autosomal recessive
c. X-linked
d. Multifactorial genetic disease

723. Marker for Squamous cell carcinoma


a. Vimentin
b. Cytokeratin
c. Desmin
d. Myogenin

724. SYT-SSX1 gene is associated with-


a. Liposarcoma
b. Rhabdomyosarcoma
c. Synovial sarcoma
d. Ewings sarcoma

710. 711. 712. 713. 714. 715. 716. 717. c 718. c 719. a
b b b a b b b

720. 721. 722. 723. 724. c


d a b b

725. A glomerulus-like structure composed of central blood vessel enveloped


by tumor cells within a space lined by tumor cells, is seen in-
a. Sertoli-Leydig cell tumor
b. Granulosa cell tumor
c. Endodermal sinus tumor
d. Sex cord tumor with annular tubules

726. Small cell cancer commonly metastasizes to-


a. Brain
b. Liver
c. Bone
d. Kidney

727. Most common cause of nephrotic syndrome in elderly


a. Membranous glomerulopathy
b. Minimal change disease
c. Acute GN
d. Focal GN

728. Perivascular lymphocytes and microglial nodules are seen in-


a. Multiple sclerosis
b. HIV encephalitis
c. CMV meningitis
d. Bacterial meningitis

729. Most common cerebellar tumor in children-


a. Astrocytoma
b. Medulloblastoma
c. Ependymoma
d. PNET

730. Tadpole cells or comma shaped cells on histopathology are seen in-
a. Trichoepithelioma
b. Seminoma
c. Rhabdomyosarcoma
d. Histiocytoma

731. Maximum malignant potential is of which of the following conditions?


a. Familial adenomatous polyposis
b. Juvenile polyp
c. PeutzJegher polyp
d. Ulcerative colitis

732. A patient with history of smoking presents with a small <1 cm


endobronchial nodule which was biopsied and studied. Electron microscopy
showed the following. Which marker on immunohistochemistry would help
reach a diagnosis?

a. TTF1
b. Napsin A
c. Synaptophysin
d. Calretinin

733. A 58 year old woman with breast cancer is being evaluated for further
management. On IHC the following findings were obtained. Which of the
following is true about the course of this patient’s condition?

a. Trastuzumab is a treatment option


b. Hormonal therapy is not a treatment option
c. Patient has favourable prognosis
d. Patient falls into Basal subgroup of breast cancers

725. c 726. a 727. a 728. b 729. a 730. c 731. a 732. c 733. c

734. An 8 year old male patient with history of hematuria 3 days after
contracting a respiratory infection is being evaluated. Which of the following
is most likely to be the electron microscopy finding?
a. Basket weave pattern of the basement membrane
b. Subepithelial deposits
c. Mesangial deposits
d. Subendothelial deposits

735. All of the following genes are mutated in lung carcinoma except
a. EGFR
b. KRAS
c. P53
d. ALK

736. Lymphoid aggregates in the form of follicles are seen in which of the
following?
a. Hepatitis A
b. Hepatitis B
c. Hepatitis C
d. Hepatitis D

737. Following is a picture of a lung tissue biopsy from an elderly patient with
smoking history and history of working in a shipyard. Which is the most
common tumor that can occur in this patient?

a. Mesothelioma
b. Lung carcinoma
c. Laryngeal carcinoma
d. Nasopharyngeal carcinoma

738. All of the following are characteristic features of malignancy except


a. Atypical mitosis
b. Hyperchromatic nuclei
c. Loss of polarity
d. Chromosomal diploidy

739. All of the following features can help differentiate Crohn’s disease from
Ulcerative colitis except
a. Granuloma formation
b. Cryptab scesses
c. Transmural inflammation
d. Pseudopoylps

740. Lardaceous spleen occurs due to


a. Deposition of amyloid in white pulp
b. Deposition of amyloid in red pulp
c. Deposition of hemosiderin
d. Deposition of calcium

741. Tumor marker(s) associated with both colon cancer and pancreatic cancer
is
a. CEA
b. CA-125
c. CA 15-3
d. All of the above

742. Which of the following has the least chance of metastasising to the
brain?
a. Lung carcinoma
b. Melanoma
c. Breast carcinoma
d. Prostate carcinoma

743. Coffee bean nuclei are seen in all except


a. Brenner tumor
b. Langerhan’s cell histiocytosis
c. Follicular carcinoma thyroid
d. Chondroblastoma

744. All of the following are true about the following tumor except
a. Can express progesterone receptor
b. Rhabdoid morphology is grade III
c. Bone invasion upgrades the tumor
d. EMA positivity is seen

734. c 735. d 736. c 737. b 738. d 739. b 740. b 741. a 742. d

743. c 744. c

745. The following is triphenyl tetrazolium chloride stain of a post-mortem


heart, the corresponding labelling to 1,2 3 are

a. Viable myocardium, recent infarct, old infarct


b. Old infarct, recent infarct, viable myocardium
c. Recent infarct, old infarct, viable myocardium
d. Pericardial fat, recent infarct, viable myocardium

746. Not ture about microscopic polyangitis is


a. Lungs are rarely involved
b. Glomerulonephritis is frequent
c. Anti MPO anti bodies
d. Leucocytoclastic vasculitis

747. A 35-year old smoker presents with severe retro sternal pain which is
radiating to neck and worsens after heavy meals occasionally it makes him
wake up at night. Upper GI Endoscopic examination does not reveal any
mass but a velvety patch in the lower esophagus. Biopsy from the lesion is
likely to show
a. Squamous pearls with malignant cells
b. Malignant looking glands infiltrating the muscle
c. Benign looking glands which are Alcian blue positive
d. Viral inclusions in the epithelial cells

748. A 2-year old is having a long standing diarrhea and steatorrhea. The child
is also short in stature and has not gained weight. The peripheral smear and
an intestinal biopsy was done and is as shown in the figure. What is the
likely diagnosis?

a. celiac disease
b. Whipples Disease
c. Abetalipoproteinemia
d. Lactase deficiency

749. A 7-year child has a mass in the front of neck. Notable findings are BP of
160/90 mm of Hg. His father also had history of hypertension and a neck
surgery was performed on him Biopsy from the mass is as follows. Likely
diagnosis is

a. Papillary carcinoma of thyroid


b. Parathyroid hyperplasia
c. Medullary carcinoma of thyroid
d. Carotid body tumor
745. c 746. a 747. c 748. c 749. c

750. The following gross picture and histopathological picture is of which


condition

a. Capillary hemangioma
b. Cavernous hemangioma
c. Lymphangiosarcoma
d. Kaposi’s sarcoma

751. A portion of large bowel of a young patient is shown below. The patient
most likely harbors a germline mutation in which of the following genes?

a. KRAS
b. p53
c. MYC
d. APC

752. All of the following are true about the following cerebellopontine angle
tumor except

a. Well circumscribed mass


b. S-100 is negative
c. Associated with neurofibromatosis type 2
d. Areas of nuclear palisading are seen

753. All are features of Crohn’s disease except


a. Earliest feature is aphthous ulcer
b. Elongated, serpentine ulcers oriented along the axis of the bowel
c. Intestinal wall is thinned out
d. Cobblestone appearance

754. Mallory Denk bodies are composed of


a. Cytokeratin 8/18
b. Cytokeratin 5/6
c. Vimentin
d. Cytokeratin 20

755. All of the following are true about familial breast cancers with BRCA1
gene mutations except
a. Luminal A subtype
b. Poorly differentiated
c. Medullary features
d. Seen in younger women

756. All of the following are features of a stable plaque except


a. Large lipid core
b. Less neutrophils
c. Thick fibrous cap
d. Smooth muscle cells

757. The rule of “10’s” of phaeochromocytoma stands for all of the following
except
a. Extra adrenal
b. Bilateral
c. Malignant
d. Familial

758. Most common pituitary adenoma in MEN-1 is


a. Lactotroph
b. Somatotroph
c. Corticotroph
d. Gonadotroph

759. All of the following are epithelial tumors of stomach except


a. Signet ring cell carcinoma
b. Neuroendocrine tumor
c. Granular cell tumor
d. Undifferentiated carcinoma

750. 751. 752. 753. c 754. 755. 756. 757. d 758. a 759. c
a d b a a a

760. Which of the following is false about the most common primary cardiac
tumor in adults?
a. Most common site is left atrium
b. Associated with Carney triad
c. Activating mutations in GNAS
d. Wrecking ball effect

761. A patient with diffuse enlargement of thyroid gland showed the following
histopathological picture. Which of the following statements is false about
this condition?

a. CD4 mediated killing of epithelial cells


b. Female preponderance
c. Thyrotoxicosis may occur
d. Most common cancer arising in this condition is papillary carcinoma

762. The following gross image of liver shows

a. Metastasis
b. Focal nodular hyperplasia
c. Leiomyoma
d. Hepatic adenoma

763. Mutations associated with nonsmall cell Lung ca are? (PGI)


a. p53
b. EGFR
c. Rb
d. Myc
e. RAS

764. Hyaline membrane in the lung is seen in-


a. Respiratory distress syndrome
b. Pulmonary edema
c. Pneumococcal pneumonia
d. Acute viral hepatitis

765. A 22-year-old man who is being treated for leukemia complains of


shortness of breath on exertion, pleuritic chest pain, and a low-grade fever.
Physical examination reveals crackles in both lung bases and clubbing of the
figers. Bronchoalveolar lavage demonstrates PAS-positive material and
elevated levels of surfactant proteins. An open-lung biopsy is shown in the
image. Which of the following is the most likely diagnosis?

a. Alveolar proteinosis
b. Eosinophilic pneumonia
c. Goodpasture syndrome
d. Hyaline membrane disease

766. Morphologic features of celiac disease include all, except:


a. Increase in intra-epithelial lymphocytes
b. Increase crypt : villous ratio
c. Distended macrophages with PAS positive granules in lamina propria
d. Elongated, hyperplastic and tortuous crypts

760. b 761. a 762. a 763. a, b 764. a 765. a 766. b


,e

767. In carcinoma of unknown primary, if the tissue marker CDX-2 is positive,


it indicates:
a. Bladder cancer
b. Gastrointestinal cancer
c. Lung cancer
d. Thyroid cancer

768. Councilman bodies are seen in-


a. Alcoholic cirrhosis
b. Wilson’s disease
c. Acute viral hepatitis
d. Autoimmune hepatitis

769. Piecemeal necrosis on liver biopsy is a feature of-


a. Alcoholic hepatitis
b. Indian childhood cirrhosis
c. Chronic active hepatitis
d. Primary alcoholic cirrhosis

770. Chromosome involved in Wilson’s disease


a. 11
b. 12
c. 13
d. 14

771. Giant hypertrophy of gastric mucosa similar to brain surface is seen in-
a. Peptic cell metaplasia
b. Carcinoma stomach
c. Menetrier’s disease
d. Leiomysarcoma

772. Gastric carcinoma is associated with-(PGI)


a. Inactivation of p53
b. Over expression of C-erb-B2
c. Over expression of C-MET
d. Activation of RAS

773. Skip lesions of colon with epithelioid granuloma are usually seen with-
a. Crohn’s disease
b. Ulcerative colitis
c. Intestinal TB
d. Sarcoidosis

774. Gluten sensitive enteropathy is most strongly associated with-


a. HLA-DQ2
b. HLA-DR4
c. HLA-DQ3
d. HLA DR2

775. Antigliadin antibodies are detectable in-


a. Tropical sprue
b. Whipple’s disease
c. Celiac disease
d. Intestinal lymphoma

776. Massive hepatocellular necrosis is seen with:


a. Tetracycline
b. Macrolides
c. Methyldopa
d. Acetaminophen

777. Fibrin ring granuloma in liver is caused by:


a. Suphonamides
b. Amlodarone
c. Isoniazid
d. Allopurinol

778. Peliosis hepatis is caused by:


a. Contraceptives
b. Anabolic steroids
c. Erythromycin
d. Ezetemibe

779. Macrovesicular fatty change in hepatocytes is seen due to infection with:


a. Hepatitis A
b. Hepatitis B
c. Hepatitis C
d. Hepatitis D

780. Microvesicular fatty liver is caused by:


a. DM
b. Valproate
c. Starvation
d. IBD

781. On kidney biopsy, PAS positive structures are:


Glomerular basement membrane
a. Tubule
b. Neutrophils
c. Interstitium
d. Mesangial matrix

767. 768. c 769. c 770. c 771. c 772. 773. 774. a 775. c 776. d
b a a

777. 778. 779. c 780. 781.


d b b c,d

782. Rapid advanced renal parameters is clinically termed as Rapidly


progressive glomerulonephritis. What exactly is the finding on microscopy?
a. Membrane thickening
b. Segmental sclerosis
c. Crescents
d. Mesangial expansion

783. True about membranoproliferative glomerulonephritis is? (PGI)


a. Low C3 level
b. Double basement membrane appearance on light microscopy
c. Dense deposit along basement membrane on electron microscopy
d. Mesangial hypocellularity
e. GBM thickening

784. Which of the following disease has deafness as well as hematuria?


a. Alport Syndrome
b. Good pasture syndrome
c. IgA nephropathy
d. Cryoglobinemia

785. In renal biopsy of a 14 years old boy with nephritic syndrome, glomeruli
are showing proliferation of mesangial cells with GBM thickening and
mesangial cell interposition. What is the most likely diagnosis in this case?
a. Membranous nephropathy
b. Diffuse proliferative glomerulonephritis
c. Focal segmental glomerulosclerosis
d. Mesangiocapillary glomerulonepthritis

786. Dysmorphic RBC with ARF is seen in?


a. Glomerular disease
b. Renal carcinoma
c. Proximal tubule disease
d. Distal tubule disease

787. Mesangial cells of IgA Nephropathy overexpresses:


a. CD51
b. CD61
c. CD71
d. CD81

788. A 35-year-old man with a history of smoking presents with hematuria and
bloody sputum. Over the next 2 days, he develops oliguria and renal failure,
after which he is placed on dialysis. A renal biopsy is stained with
fluorescein-conjugated goat antihuman IgG, and the results are shown.
Which of the following best describes the pattern of direct
immunofluorescence observed on this photomicrograph?

a. Discontinuous and peripheral


b. Finely granular along the perimesangial reflections
c. Linear along the glomerular basement membrane
d. Mesangial with a stalk predominance
789. Berger nephropathy is due to mesangial deposition of-
a. Fibrin and C3
b. IgD and C3
c. IgE and C3
d. IgA and C3

790. In IgA nephropathy (Berger’s disease) there are-


a. Subepithelial deposits
b. Subendothelial deposits
c. Mesangial deposits
d. Basement membrane deposits

791. Kimmelstiel-Wilson lesions in kidney consists-


a. Splitting of glomerular basement membrane
b. Nodular sclerosis of the glomeruli
c. Hyaline sclerosis
d. Hyperplastic arteriosclerosis

782. c 783. 784. 785. 786. 787. c 788. c 789. d 790. c 791. b
a,b,c,e a d a

792. Pauci immune glomerulonephritis is seen in:


a. After transplant in Alports
b. Microscopic polyangitis
c. Henoch-schonlein nephritis
d. Lupus nephritis

793. Mutation in NPHS1 gene causes which disease?


a. Alport syndrome
b. Congenital Finnish type nephrotic syndrome
c. Focal segmental glomerulosclerosis
d. Nail patella syndrome

794. Broad cast is characteristic of-


a. CRF
b. Acute glomerulonephritis
c. ARF
d. Renal TB

795. Cast is produced due to damage to-


a. Nephron
b. Tubule
c. PCT
d. DCT

796. Maximum endocapillary proliferation is a feature of-


a. Membranous GN
b. Mesangioproliferative
c. Focal glomerulosclerosis
d. Post-streptococcal GN

797. Rapidly progressive glomerulonephritis is histologically characterised by


the presence of numerous-
a. Intramembranous dense depositis
b. Atrophic proximal convoluted tubules
c. Hyalinized small arterioles
d. Epithelial cell crescents

798. Crescent are derived from-


a. Parietal epithelial cells + fibrin + macrophage
b. Mesangium + fibrin + macrophage
c. Tubule + mesangium + fibrin
d. Mesangium + fibrin

799. Malignancies are most commonly associated with which type


ofGlomerulonephritis?
a. MGN
b. MPGN
c. Minimal change disease
d. FSGS

800. Most common cause of death in APCKD (Adult Polycystic Kidney Disease)-
a. Chronic renal failure
b. Cardiovascular disease
c. Ruptured berry aneurysm
d. Sepsis

801. The mother of a 2-month-old child palpates a mass on the left side of the
child’s abdomen. Vital signs are normal. A CT-guided renal biopsy shows
undifferentiated tubules surrounded by undifferentiated mesenchyme,
smooth muscle, and islands of cartilage. The mass is removed (shown in the
image) and displays variably sized cysts. Which of the following is the most
likely diagnosis for this child’s flank mass
a. Autosomal recessive polycystic kidney disease
b. Autosomal dominant polycystic kidney disease
c. Medullary sponge kidney
d. Renal dysplasia

792. 793. 794. 795. 796. 797. 798. 799. b 800. b 801. d
b b a b d d a

802. An 8-year-old boy presents with headaches, dizziness, and malaise


approximately 2 weeks after a severe sore throat. His mother describes
puffiness of his face and darkening of his urine. She also notes that her son
is passing less urine and that he is becoming increasingly short of breath.
On physical examination, there is anasarca, hypertension (190/130 mm Hg),
and tachycardia. The urine is scanty and brownish red. Urinalysis shows 3+
proteinuria. Microscopic examination of the urine discloses numerous RBCs,
as well as occasional granular and red cell casts. A renal biopsy is stained by
direct immunofluorescence microscopy for complement C3, and the results
are shown. Which of the following best describes the pattern of immune-
fluorescence observed in this renal biopsy?

a. Deposits limited to the mesangium


b. Granular deposits along the perimesangial reflections
c. Linear staining along the glomerular basement membranes
d. Subepithelial and subendothelial deposits

803. In Wegener’s granulomatosis cytoplasmic anti neutrophilic antibodies are


directed against:
a. Proteinase 1
b. Proteinase 2
c. Proteinase 3
d. Proteinase 4

804. Type of renal cell carcinoma seen in sickle cell trait is


a. Chromophobe
b. Clear cell
c. Papillary
d. Medullary

805. Which one is found in HSP?


a. IgA
b. IgM
c. IgG
d. IgE

806. Arterial biopsy of elderly male show fragmentation of elastic lamina,


lymphocyte infiltration and giant cells-
a. Temporal arteritis
b. Takayasu disease
c. Polyarteritisnodosa
d. Kawasaki disease

807. Silk Road disease is:


a. Behcet’s syndrome
b. Giant cell arteritis
c. Henoch schonlein purpura
d. Wegener’s granulomatosis

808. Necrotizing arterioritis with fibrinoid necrosis is:


a. Immediate hypersensitivity
b. Cell mediated immunity
c. Antigen-antibody complex mediated
d. Cytotoxic cell mediated

809. A 55-years-old male presents with severe chest pain radiating to the left
arm. ECG shows ST segment elevation in the V4, V5 and V6 leads. CK-MB and
troponin levels are found to be increased. The most likely cause for the
increase in enzyme in serum is:
a. Clumping of nuclear chromatin
b. Lysosomal Autophagy
c. Mitochondrial swelling
d. Cell membrane defects

810. Which of the following salivary gland tumors have a variable mixture of
three types of cells?
a. Pleomorphic adenoma
b. Mucoepidermoid carcinoma
c. Warthin’s tumor
d. Adenoid cystic carcinoma

802. b 803. c 804. d 805. a 806. a 807. b 808. c 809. d 810. d

811. Irreversible change in ischemia of heart occurs in?


a. 10 min
b. 30 min
c. 60 min
d. 90 min

812. Post mortem finding in a case of death due to myocardial infarction is?
a. Fat necrosis
b. Caseous necrosis
c. Liquefactive necrosis
d. Coagulative necrosis

813. Which of the following is not true about medullary carcinoma of breast?
a. Relatively good prognosis as compared to other poorly differentiated tumors
b. Rich lymphocytic infiltrates
c. Associated with BRCA2 mutations
d. Maximum desmoplasia

814. Which of the following benign breast lesions is non proliferative


a. Ductal papilloma
b. DCIS
c. Sclerosing adenosis
d. Apocrine metaplasia

815. Patients with Hashimoto’s thyroiditis are at increased risk of developing-


a. Papillary carcinoma
b. Follicular carcinoma
c. T-cell lymphoma
d. B-cell lymphoma

816. Paraneoplastic syndromes are classically associated with (PGI type)


a. Renal ca
b. Oesophagealca
c. Breast ca
d. Testicular ca
e. Small cell carcinoma of lung

817. Which of the following soft tissue tumors is not caused by fusion of EWS
with another gene?
a. Ewing sarcoma family tumors
b. Extraskeletal myxoid chondrosarcoma
c. Desmoplastic small round cell tumor
d. Alveolar soft part sarcoma

818. Ectopic ADH production is seen in-


a. Small cell carcinoma is lung
b. Anaplastic carcinoma of lung
c. Squamous cell carcinoma of lung
d. Adenocarcinoma of cerebellum

819. Retinoblastoma is associated with-


a. Osteosarcoma
b. Hepatocellular carcinoma
c. Squamouscell carcinoma
d. Osteoclastoma

820. Most common cancer arising in a patient with chronic osteomyelitis is


a. Squamous cell carcinoma
b. Osteosarcoma
c. Fibrosarcoma
d. Osteoid osteoma

821. High risk of malignancy is seen in?


a. Simple hyperplasia with atypia
b. Simple hyperplasia without atypia
c. Complex hyperplasia with atypia
d. Complex hyperplasia without atypia

822. S 100 is a marker of?


a. Melanoma
b. Schwannoma
c. Histiocytoma
d. All of the above

823. N-MYC amplification is associated with which tumor?


a. Burkitt lymphoma
b. Squamous cell carcinoma lung
c. Astrocytoma
d. Neuroblastoma
824. Pseudomyxoma peritonei is seen with -
a. Mucinous cystadenocarcinoma ovary
b. Carcinoidappedix
c. Endometrial carcinoma
d. Ileal carcinoid

811. 812. 813. 814. 815. 816. 817. 818. a 819. a 820. a
b d d d d a,e d

821. c 822. 823. 824.


d d a

825. A child is born with a single functional copy of a tumor suppressor gene.
At the age of 5 years, the remaining normal allele is lost through mutation.
As a result, the ability to control the transition from G1 to the S phase of the
cell cycle is lost. Which of the following neoplasms is most likely to arise by
means of this mechanism-
a. Retinoblastoma
b. Breast carcinoma
c. Adenocarcinoma of colon
d. Cerebral astrocytoma

826. Keratin is a tumour marker for-


a. Carcinoma cervix
b. Neurofibroma
c. Rhabdoyosarcoma
d. Choriocarcinoma

827. Content of Aschoff’s nodules are A/E


a. Histiocytes
b. Langhan’s giant cell
c. Fibrinoid deposition
d. Aschoff’s cells

828. PTH like substance is produced by which type of lung malignacy-


a. Squamouscell carcinoma
b. Oatcell carcinoma
c. Adeno carcinoma
d. Large cell carcinoma

829. Nephrotic syndrome is characterised by all except


a. Proteinuria
b. Hyperlipidemia
c. Oedema
d. Haematuria

830. Hypercoagulation in nephrotic syndrome is caused by-


a. Loss of antithrombin III
b. Decreased fibrinogen
c. Decreased metabolism of Vitamin K
d. Increase in protein C

831. Skip granulomatous lesions are seen in-


a. Ulcerative colitis
b. Crohn’s disease
c. Whipple’s disease
d. Reiter’s disease

832. True about Barrett’s esophagus is?


a. Squamous metaplasia
b. Columnar metaplasia
c. Irreversible
d. Not premalignant

833. The highest malignant potential is seen in-


a. Crohn’s disease
b. Ulcerative colitis
c. FAP
d. PeutzJegher syndrome

834. Macrophages containing PAS positive granules and rod-shaped bacilli in


small intestinal mucosa are found in-
a. Agammaglobulinemia
b. Tropical sprue
c. Whipple’s disease
d. Coeliac spure

835. Psammoma bodies can be seen in the following except:


a. Follicular carcinoma of thyroid
b. Papillary carcinoma of thyroid
c. Meningioma
d. Serous cystadenoma of ovary

836. Myotonic dystrophy is inherited in chromosomes-


a. 21
b. 20
c. 19
d. 24

837. All are true about silicosis except?


a. Bifringent crystals seen
b. Pleural plaque may be seen
c. Lower lobe is usually involved
d. Most common pneumoconiosis

838. Asbestosis causes?


a. Lymphoma
b. Leukaemia
c. Renal cell carcinoma
d. Mesothelioma

839. In thyroglossal cyst, carcinoma which arises is-


a. Follicular
b. Medullary
c. Papillary
d. Anaplastic

825. 826. 827. 828. 829. 830. 831. 832. b 833. c 834. c
a a b a d a b

835. 836. c 837. c 838. 839. c


a d

840. MEN-2 syndrome is associated with increase in-


a. Papillary Ca of thyroid
b. Medullary
c. Follicular
d. Anaplastic

841. Most common primary tumor of heart?


a. Myxoma
b. Rhabdomyosarcoma
c. Fibroma
d. Leiomyosarcoma

842. Which one of these thyroid cancers cannot be diagnosed by FNAC?


a. Papillary
b. Medullary
c. Follicular
d. Anaplastic

843. Pseudopolyps are features of-


a. Crohn’s disease
b. Ulcerative colitis
c. Celiac sprue
d. Whipple’s disease

844. Tumor marker for lung adenocarcinoma? (PGI)


a. CK 7
b. CK 20
c. TTF1
d. CK 5/6
e. BerEP- 4

845. Reid’s index is-


a. Increased in chronic bronchitis
b. Decreased in chronic bronchitis
c. Increased in bronchial asthma
d. Decreased in bronchial asthma

846. True about light microscopy in minimal change disease is:


a. Loss of foot process seen
b. Anti GBM Abs seen
c. IgA deposits seen
d. No change seen

847. Chromosome associated with familial polyposis colon?


a. Chromosome 5
b. Chromosome 6
c. Chromosome 11
d. Chromosome 13

848. Buerger disease is associated with:


a. Alcoholism
b. Smoking
c. Trauma
d. Cold environment

849. Which of the following soft tissue tumors is not caused by fusion of EWS
with another gene?
a. Ewing sarcoma family tumors
b. Extraskeletal myxoid chondrosarcoma
c. Desmoplastic small round cell tumor
d. Alveolar soft part sarcoma

850. Ectopic ADH production is seen in-


a. Small cell carcinoma is lung
b. Anaplastic carcinoma of lung
c. Squamous cell carcinoma of lung
d. Adenocarcinoma of cerebellum

851. Retinoblastoma is associated with-


a. Osteosarcoma
b. Hepatocellular carcinoma
c. Squamouscell carcinoma
d. Osteoclastoma
852. Most common cancer arising in a patient with chronic osteomyelitis is
a. Squamous cell carcinoma
b. Osteosarcoma
c. Fibrosarcoma
d. Osteoid osteoma

853. High risk of malignancy is seen in?


a. Simple hyperplasia with atypia
b. Simple hyperplasia without atypia
c. Complex hyperplasia with atypia
d. Complex hyperplasia without atypia

854. Gene of Wilson’s disease is-


a. ATP 7ª
b. ATP 7B
c. ADP 7ª
d. ADP 7B

855. Most common benign tumor of liver is


a. Adenoma
b. Hemangioma
c. Focal nodular hyperplasia
d. Hepatoblastoma

840. 841. 842. c 843. 844. 845. 846. 847. a 848. b 849. d
b a b a,c,e a a

850. 851. 852. 853. c 854. 855.


a a a b b

856. Which of the following special stains specifically help to identify goblet
cells in a case of suspected Barret’s esophagus?
a. Alcian blue at pH 2.5
b. Mucicarcmine
c. PAS
d. Masson trichrome

857. Ulcerative colitis is characterised by all except


a. Mucosal and submucosal involvement
b. Absent granulomas
c. Brisk Lymphoid aggregates
d. Minimal edema

858. Early to late adenoma transformation in colorectal carcinoma is


mediated by
a. Loss of APC
b. Activating mutations of KRAS
c. Loss of p53
d. Loss of SMAD4

859. Exfoliative cytology is useful in -


a. Carcinoma stomach
b. Carcinoma bronchus
c. Carcinoma cervix
d. All of the above

860. Atheromatous plaque do not contain:


a. Platelet
b. Neutrophils
c. Smooth muscle fibres
d. Monocytes

861. Autopsy finding after 12hrs in a case of death due to M.I.-


a. Caseous necrosis
b. Coagulative necrosis
c. Fat necrosis
d. Liquefactive necrosis

862. Nottingham system of grading breast cancers of IDC subtype does not
include which of the following parameters
a. Necrosis
b. Mitotic activity
c. Nuclear pleomorphism
d. Tubular differentiation

863. True about Takayasu is all except?


a. Causes transmural granuloma
b. Granulomatous vasculitis
c. Large to medium vessel
d. Called giant cell aortoarteritis

864. 30-years-old mal presented with severe dyspnoea and fatigue. X-ray
showed left atrial enlargement. Physician suspects the patient of having
mitral stenosis and gets a histopath examination done, the image of which
is shown, it shows?
a. Sarcoidosis
b. Tuberculosis
c. Aschoff bodies
d. Fungal granuloma

865. Anti transglutaminase antibody is seen in?


a. Celiac disease
b. SLE
c. Collagenous colitis
d. Tropical sprue
856. 857. c 858. 859. 860. 861. 862. 863. d 864. c 865. a
a b d b b a
Systemic Pathology – Explanations
636. D
Gain of function mutations in RET leads to MEN 2 and Medullary carcinoma thyroid.

637. A
Option A >C - epithelial malignancy marker
Option B- mesenchymal malignancy marker
Option D- marker for lymphomas

638. C
Multinucleated giant cells with refractile matter can be seen in the picture suggestive
of aspiration pneumonia.

639. A
The gross specimen shows dilated bronchioles close to the pleura.
Option B- localised mass with necrosis
Option C- infiltrate grey white mass
Option D- tiny grey while millet like areas spread across the entire lung

640. D

641. B
Features of silicosis
Associated with sandblasting, foundries, mines.
Macrophages respond to silica and release fibrogenic factors, leading to fibrosis.
It is thought that silica may disrupt phagolysosomes and impair macrophages,
increasing susceptibility to TB.
Risk of cancer, cor pulmonale, and Caplan syndrome
Affects upper lobes
Egg shell calcification seen in hilar lymph nodes om CXR
Collagenous silicotic nodules in the lung
Option CE are features of coal workers pneumoconiosis

642. A
In the new WHO classification, the number of subtypes has been reduced to three,
which makes the diagnosis easier and avoids rare subtypes with confusing names
Keratinizing
Non-keratinizing
Basaloid squamous cell carcinoma
PreinvasivelesionSquamouscell carcinoma in situ

643. A, C, D, E
Option B- positive in both
Options AD- positive in adenocarcinoma
Option CE- positive in mesothelioma

644. C
Most malignant and aggressive form of lung cancer is small cell carcinoma

645. A, C, E
Option B- marker for primary lung carcinoma
Option D- marker for hepatocellular carcinoma

646. C

647. C

648. A
Most common paraneoplastic syndrome associated with small cell carcinoma is
ectopic hormone production, ACTH and ADH being most common.

649. B
Option A- most common type of emphysema

650. C
Asbestos (ferruginous) bodies are golden-brown fusiform rods resembling dumbbells,
found in alveolar sputum sample or histology, visualized using Prussian blue stain.

651. C
Tuberculosis can present with both caseating and non caseating granulomas. Bilateral
hilar lymphadenopathy is characteristic of sarcoidosis.

652. C
An entire lobe shows consolidation hence C, Klebsiella being the probable cause.

653. C
Cup and saucer appearance on special silver stain with history of immunocompromise-
Pneumocystis

654. A
Panacinar emphysema is seen with option A
Option B will show bronchiectasis
Option D may show cavitations

655. D
Option ABC are features of bronchial asthma
Option D is seen in chronic bronchitis.

656. D
Picture shows collagenous silicotic nodules suggestive of silicosis.

657. A, B, C
SDH deficient GIST is ckit negative and hence non responsive to Imatinib and they
have an indolent course. It is a part of Carney Stratakis Syndrome.

658. B
Biopsy shows owl eye inclusions and the history is suggestive of CMV inclusions
Option A- pseudohyphae can be seen
Option C- multinucleation, margination and nuclear molding can be seen
Option D- increased number of eosinophils

659. A
The history and histology of transition from squamous to columnar shows Barret’s
metaplasia with goblet cells. Adenocarcinoma would show infiltrating glands of
varying shapes and sizes.

660. A
Tropheryma whippeli, a gram positive Actinomycetes engulfed by macrophages give
them a foamy appearance and they stain positive with PAS.
Option B- lipid laden epithelial cells
Option C- architectural distortion, cryptitis, aphthous ulcer, non caseating granulomas,
transmural inflammation
Option D- cryptitis, superficial inflammation, pseudopolyps

661. C
Histology shows arborizing smooth muscle bundles with glands arising from them
suggestive of PeutzJegherpolp which is a hamartomatous polyp.

662. B Adenocarcinoma in situ


Adenocarcinoma in situ is a pre invasive lesion depicted in this picture. Thecells may
be columnar and mucus producing or cuboidal andsimilar to type II pneumocytes.
They tend to grow along thealveolar septa, as depicted in the photomicrograph

663. B
A and C are features characteristic of ulcerative colitis.

664. B
The earliest lesion, the aphthous ulcer, may progress, and multiple lesions often
coalesce into elongated, serpentine ulcers oriented along the axis of the bowel. Edema
and loss of the normal mucosal texture are common. Sparing of interspersed mucosa,
a result of the patchy distribution of Crohn disease, results in a coarsely textured,
cobblestone appearance in which diseased tissue is depressed below the level of
normal mucosa

665. A
KRAS mutation is involved in transformation of early adenoma to late adenoma.

666. A (single best option)


Although D is a non polyposis condition, it is neoplastic as it B and C. A is an example
of hamartomatous polyps and hence is the best option here.

667. B
CK 8/18 skeins left after damage are called Mallory Hyaline bodies

Pattern of Morphologic Findings Examples of Associated Agents


Injury

Cholestasis Bland hepatocellular cholestasis, without Contraceptive and anabolic steroids,


inflammation antibiotics, HAART

Cholestatic Cholestasis with lobular necro inflammatory activity; Antibiotics, phenothiazine, statins
hepatitis may show bile duct destruction

Hepatocellular Spotty hepatocyte necrosis Massive necrosis Chronic Methyldopa, phenytoin Acetaminophen,
necrosis hepatitis halothane Isoniazid

Fatty liver
disease Large and small droplet fat Ethanol, corticosteroids, methotrexate,
total parenteral nutrition
“Micro vesicular steatosis” (diffuse small droplet fat)
Valproate, tetracycline, aspirin (Reye
Steatohepatitis with Mallory-Denk bodies syndrome), HAART

Ethanol, amiodarone

Fibrosis and Periportal and pericellular fibrosis Alcohol, methotrexate, enalapril,


cirrhosis vitamin A and other retinoids

Granulomas
Noncaseating epithelioid Sulfonamides,

granulomas Fibrin ring granulomas amiodarone, isoniazid Allopurinol

Vascular
lesions Sinusoidal obstruction syndrome (veno-occlusive High-dose chemotherapy, bush teas
disease): obliteration of central veins
Oral contraceptives Anabolic steroids,
Budd-Chiari syndrome Peliosis hepatis: blood-filled tamoxifen
cavities, not lined by endothelial cells

Neoplasms
Hepatocellular adenoma Oral contraceptives, anabolic steroids
Alcohol, thorotrast
Hepatocellular carcinoma
Thorotrast
Cholangiocarcinoma
Thorotrast, vinyl chloride
Angiosarcoma

668. B

669. B
Hepatocellular adenomas are benign tumors of neoplastic hepatocytes. Most can be
subclassified on the basis of molecular changes:
HNF1-α inactivated adenomas, with virtually no risk of malignant transformation,
often associated with oral contraceptive pill use or in individuals with MODY-3
β-Catenin activated adenoma, with mutations in the β-catenin gene leading to
marked atypia and associated with a very high risk for malignant transformation
Inflammatory adenomas, the hallmark of which is up-regulation of C-reactive protein
and serum amyloid A (often derived from gp130 mutations); 10% of these have
concomitant β-catenin activating mutations. Risk for malignant transformation is
intermediate.

670. A
Ménétrier disease is a rare disorder associated with excessive secretion of
transforming growth factor α (TGF-α). It is characterized by diffuse hyperplasia of the
foveolar epithelium of the body and fundus and hypoproteinemia due to protein-losing
enteropathy. Secondary symptoms, such as weight loss, diarrhea, and peripheral
edema, are commonly present. Symptoms and pathologic features of Ménétrier
disease in children are similar to those in adults, but pediatric disease is usually self-
limited and often follows a respiratory infection. Risk of gastric adenocarcinoma is
increased in adults with Ménétrier disease

671. D
Neoplastic tubular, villous and tubulovillous adenomatous polyps are seen in FAP,
tubular most commonly.

672. C
A is the name of a gene, CD 117 is the marker used on IHC and it is the most sensitive
marker of GIST.

673. A
Lynch syndrome (Hereditary Non Polyposis Colon Cancer) due to defects in DNA
Mismatch repair genes like MSH2 and MLH1

674. B
Malakoplakia is a distinctive chronic inflammatory reaction that appears to stem from
acquired defects in phagocyte function, malakoplakia arises in the setting of chronic
bacterial infection, mostly by E. coli or occasionally Proteus species. It occurs with
increased frequency in immunosuppressed transplant recipients.In the bladder,
malakoplakia takes the form of soft, yellow, slightly raised mucosal plaques, 3 to 4 cm
in diameter, that are filled with large, foamy macrophages mixed with occasional
multinucleate giant cells and lymphocytes. The macrophages have an abundant
granular cytoplasm due to phagosomes stuffed with particulate and membranous
debris of bacterial origin. In addition, laminated mineralized concretions resulting
from deposition of calcium in enlarged lysosomes, known as Michaelis-Gutmann
bodies, are

675. B
Also called chicken wire fibrosis

676. D

677. A
α1-Antitrypsin deficiency is an autosomal recessive dis-Antitrypsin (α1order of protein
folding marked by very low levels of circulating α1AT). The major function ofthis
protein is the inhibition of proteases, particularly neutrophil elastase, cathepsin G, and
proteinase 3, which are normally released from neutrophils at sites of inflammation.
α1AT deficiency leads to the development of pulmonary emphysema, because the
activity of destructive proteases is not inhibited. It also causes liver disease as a
consequence of hepatocellular accumulation of the misfolded protein. Cutaneous
necrotizing panniculits also occurs in a minor subset of patients. α1AT is a small 394–
amino acid plasma glycoproteinsynthesized predominantly by hepatocytes. It is a
member of the serine protease inhibitor (serpin) family. The gene, located on
chromosome 14, is very polymorphic.

678. D
HFE gene located on chromosome 6p

679. A
As per latest update in the new edition of Harrison’s Principles of Internal Medicine,
KRAS is seen in almost 100% cases of pancreatic cancer.

680. A, B, C, E

RBC casts A Glomerulonephritis, hypertensive emergency.

WBC casts B Tubulointerstitial inflammation, acute pyelonephritis, transplant rejection.

Fatty casts ("oval fat bodies") Nephrotic syndrome. Associated with "Maltese cross" sign.

Granular ("muddy brown") casts Acute tubular necrosis (ATN).


C

Waxy casts D End-stage renal disease/chronic renal failure.

Hyaline casts E Nonspecific, can be a normal finding, often seen in concentrated urine
samples.

681. C
Option A- PSGN is a nephritic syndrome with non nephrotic range proteinuria
Option B- seen in acute pyelonephritis
Option D- seen in anti GBM antibody mediated disorder

682. C
Collapsing type of FSGS with visceral cell hypertrophy and retraction of glomerulus is
classical of HIV.

683. A
Option B – seen in membranous nephropathy
Option C- seen in IgA nephropathy
Option D- seen in MPGN type I

684. A, B, C, D
Salient Features of Different Glomerulopathies

Disease Clinically Light Microscopy (H And E, Immunofluorescence Electron


Silver/Pas Special Stain) Microscopy

Minimal change Nephrotic No change Negative Effacement of


disease foot processes

Focal segmental Predominantly Focal and segmental Usually negative Effacement of


glomerulosclerosis nephrotic hyalinosis and sclerosis foot
processes,
epithelial
denudation

Membranous Nephrotic Capillary thickening, “spike Granular IgG and C3 Subepithelial


glomerulopathy and dome” appearance deposits

Membranoproliferative Nephrotic/ Mesangial and Granular IgG, C3, Subendothelial


glomerulonephritis Nephritic endocapillary proliferation, Clq, C4 deposits
type 1 “tram track” appearance

Acute infectious Nephritic Endocapillary proliferation, Granular IgG, C3 Primarily


glomerulonephritis mesangial proliferation, subepithelial
leucocytic infiltration (early-
subendothelial)

IgA nephropathy Recurrent Mesangial proliferation IgA +/- IgG, IgM, C3 Mesangial and
hematuria in the mesangium paramesangial
deposits

Pauci immune Extra capillary Fibrin in crescents No deposits,


glomerulonephritis Rapidly proliferation, crescent fragmented
(ANCA mediated) formation GBM
proliferative

glomerulonephritis

685. B
Also called Dense Deposit Disease
Most patients with dense-deposit disease (formerly called type II MPGN) have
abnormalities resulting in excessive activation of the alternative complement pathway.
These patients have a consistently decreased serum C3 but normal C1 and C4, the
early components of complement. They also have diminished serum levels of Factor B
and properdin, components of the alternative complement pathway. In the glomeruli,
C3 and properdin are deposited, but IgG is not. Recall that in the alternative
complement pathway, C3 is directly cleaved to C3b. The reaction depends on the
initial activation of C3 by such substances as bacterial polysaccharides, endotoxin, and
aggregates of IgA via a pathway involving Factors B and D. This leads to the
generation of C3bBb, the alternative pathway C3 convertase. Normally, this C3
convertase is labile, but more than 70% of patients with dense-deposit disease have a
circulating autoantibody termed C3 nephritic factor (C3NeF) that binds the alternative
pathway C3 convertase and protects it from inactivation. This favors persistent C3
activation and hypocomplementemia.

686. B
IF shows granular pattern and EM shows subepithelial deposits hence suggestive of B.

687. C

688. A
History is suggestive of diabetic retinopathy and other related complications. The
histopathology shows nodular glomerulosclerosis also known as Kimmelstein Wilson
nodules.

689. B
Armanni-Ebstein lesions were first described by Luciano Armanni, a pathologist at the
University of Naples, during autopsy studies undertaken in 1872, as a unique vacuolar
nephropathy associated with poorly controlled diabetes that involves selective renal
epithelial cell glycogen accumulation. However, within the last two decades, a broader
range of vacuolar changes, including lipid deposition, have also been termed Armanni-
Ebstein (AE) lesions, creating some confusion on possible etiology. The term AE
phenomenon is best reserved for the original clear cell change associated with
glycogen deposition, and that this should be clearly distinguished from subnuclear
lipid vacuolization («basal vacuolization»).

690. B
The history is suggestive of malignant hypertension. The kidney size varies depending
on the duration and severity of the hypertensive disease. Small, pinpoint petechial
hemorrhages may appear on the cortical surface from rupture of arterioles or
glomerular capillaries, giving the kidney a peculiar “flea-bitten” appearance.

691. C
Most common type of RCC is clear cell type in which VHL gene mutations are most
common.

692. D
Chromophobe carcinoma represents 5% of renal cell cancers and is composed of cells
with prominent cell membranes and pale eosinophilic cytoplasm, usually with a halo
around the nucleus. On cytogenetic examination these tumors show multiple
chromosome losses and extreme hypodiploidy. Like the benign oncocytoma, they are
thought to grow from intercalated cells of collecting ducts and have an excellent
prognosis compared with that of the clear cell and papillary cancers. Histologic
distinction from oncocytoma can be difficult.
typically present within the macrophages.

693. A
694. B
Thyroidisation of tubules is seen in chronic pyelonephritis. The microscopic changes
involve predominantly tubules and interstitium. The tubules show atrophy in some
areas and hypertrophy or dilation in others. Dilated tubules with flattened epithelium
may be filled with casts resembling thyroid colloid (thyroidization). There are varying
degrees of chronic interstitial inflammation and fibrosis in the cortex and medulla.

695. C
Atherosclerotic plaques have three principal components: (1) smooth muscle cells,
macrophages, and T cells; (2) extracellular matrix, including collagen, elastic fibers,
and proteoglycans; and (3) intracellular and extracellular lipid. These components
occur in varying proportions and configurations in different lesions. Typically, there is a
superficial fibrous cap composed of smooth muscle cells and relatively dense collagen.
Beneath and to the side of the cap (the “shoulder”) is a more cellular area containing
macrophages, T cells, and smooth muscle cells. Deep to the fibrous cap is a necrotic
core, containing lipid (primarily cholesterol and cholesterol esters), debris from dead
cells, foam cells (lipidladen macrophages and smooth muscle cells), fibrin, variably
organized thrombus, and other plasma proteins; the cholesterol content is frequently
present as crystalline aggregates that are washed out during routine tissue processing
and leave behind only empty “clefts.”

696. D
Mönckeberg medial sclerosis is characterized by calcification of the walls of muscular
arteries, typically involving the internal elastic membrane. Persons older than age 50
are most commonly affected. The calcificationsdo not encroach on the vessel lumen
and are usually not clinically significant.

697. C
Hyaline arteriolosclerosis- arterioles show homogeneous, pink hyaline thickening with
associated luminal narrowing. These changes reflect both plasma protein leakage
across injured endothelial cells, as well as increased smooth muscle cell matrix
synthesis in response to the chronic hemodynamic stresses of hyper tension. Although
the vessels of older patients (either nor motensive or hypertensive) also frequently
exhibit hyaline arteriosclerosis, it is more generalized and severe in patients with
hypertension. The same lesions are also a common feature of diabetic
microangiography; in that case, the underlying etiology is hyperglycemia-induced
endothelial cell dysfunction. In nephrosclerosis due to chronic hypertension, the
arteriolar narrowing of hyaline arteriosclerosis causes diffuse impairment of renal
blood supply and glomerular scarring

698. B
Hyperplastic Arteriolosclerosis -this lesion occurs insevere hypertension; vessels
exhibit concentric, laminated (“onion-skin”) thickening of the walls with luminal
narrowing.The laminations consist of smooth muscle cells with thickened,
reduplicated basement membrane; in malignant hypertension, they are accompanied
by fibrinoid deposits and vessel wall necrosis (necrotizing arteriolitis), particularly in
the kidney.

699. A, D, E

700. D
*The granulomas of giant cell arteritis are found within the vessel wall as part of the
inflammation comprising the vasculitis, but need not be present to render the
diagnosis. The granulomas of granulomatosis with polyangiitis are larger,
spanning between vessels, and associated with areas of tissue necrosis.

701. C

Type Population Clinical Course

Classic Older men (50-80 years) Usually confined to lower Usually indolent, survival 10-15
extremity years

Endemic Young black males, 15-40 Localized nodular lesions or large Nodules indolent; aggressive
(African) yrs, and children exophytic, aggressive lesions lesion survival 3-5 years

Iatrogenic Immune suppressed (e.g., Localized or widespread May regress when immune
renal transplant) involvement suppressants discontinued

Epidemic Primarily gay men in U.S., Head, face, neck, Gl and lung Fulminant, survival I-3 years
(AIDS- Europe; adults in Africa most common without effective HIV therapy
relatcd)

AIDS, Amitendu Bhatt acquired immunodeficiency syndrome; GI, gastrointestinal system;


HIV, human immunodeficiency virus.

702. D
Endothelial cell marker is PECAM/CD 31 seen in tumors of endothelial cell origin.

703. A
Classically, endocardium and valves of the right heart are primarily affected since they
are the first cardiac tissues bathed by the mediators released by gastrointestinal
carcinoid tumors. The cardiovascular lesions associated with the carcinoid syndrome
are distinctive, glistening white intimal plaquelike thickenings of the endocardial
surfaces of the cardiac chambers and valve leaflets

704. B

Chambers

Increased left atrial cavity size

Decreased left ventricular cavity size

Sigmoid-shaped ventricular septum

Valves

Aortic valve calcific deposits

Mitral valve annular calcific deposits

Fibrous thickening of leaflets

Buckling of mitral leaflets toward the left atrium

Lambl excrescences

Epicardial Coronary Arteries

Tortuosity

Diminished compliance

Calcific deposits

Atherosclerotic plaque

Myocardium

Decreased mass

Increased subepicardial fat

Brown atrophy

Lipofuscin deposition

Basophilic degeneration

Amyloid deposits

Aorta

Dilated ascending aorta with rightward shift

Elongated (tortuous) thoracic aorta

Sinotubular junction calcific deposits

Elastic fragmentation and collagen accumulation

Atherosclerotic plaque

705. C
This gross histochemical stain imparts a brick-red color to intact, noninfarcted
myocardium where lactate dehydrogenase activity is preserved. Because
dehydrogenases leak out through the damaged membranes of dead cells, an infarct
appears as an unstained pale zone.

706. C
Regurgitant jets due to mitral valve dysfunction leading to plaque formation in the
posterior wall in the sub endothelium of left atrium are called Mac Callum patches
seen in rheumatic heart disease.

707. B
Fatty degenerated heart in which the fat is disposed in the form of broken stripes in
the subendocardial myocardium is called tigered effect.

708. B
Lobular breast carcinoma has a tendency to be multifocal and bilateral, hence
contralateral breast should always be examined

709. D
Medullary carcinoma is of great interest due to the findingthat many tumors of this
type have features that are characteristic of BRCA1associated carcinomas. Among
cancers arising in BRCA1 carriers, 13% are of medullary type, and up to 60% have a
subset of medullary features. Although the majority of medullary carcinomas are not
associated with germline BRCA1 mutations, hypermethylation of the BRCA1 promoter
leading to downregulation of BRCA1 expression is observed in 67% of these tumors.
The basis for the relatively good prognosis of this subtype compared to other poorly
differentiated carcinomas is not known, but it has been noted that the presence of
lymphocytic infiltrates within the tumors is associated with higher survival rates and a
greater response to chemotherapy, suggesting that improved outcomes may be
related to a host immune response to tumor antigens. Medullary carcinoma is softer
than other carcinomas (medulla is Latin for "marrow") due to minimal desmoplasia,
and often presents as a well-circumscribed mass. It is characterized by (1) solid,
syncytium-like sheets of large cells with pleomorphic nuclei, and prominent nucleoli,
which compose more than 75% of the tumor mass; (2) frequent mitotic figures; (3) a
moderate to marked lymphoplasmacytic infiltrate surrounding and within the tumor;
and (4) a pushing (noninfiltrative) border. DCIS is minimal or absent

710. B
Approximately 12% of breast cancers occur due to inheritance of an identifiable
susceptibility gene or genes. The probability of a hereditary etiology increases when
there are multiple affected firstdegree relatives, early onset cancers, multiple cancers,
or family members with other specific cancers.Mutations in BRCA1 and BRCA2 are
responsible for80% to 90% of "single gene" familial breast cancers and about 3% of all
breast cancers. Penetrance (the percentage of carriers who develop breast cancer)
varies from 30% to 90% depending on the specific mutation present. Mutations in
BRCA1 also markedly increase the risk of developing ovarian carcinoma, which occurs
in as many as 20% to 40% of carriers. BRCA2 confers a smaller risk for ovarian
carcinoma (10% to 20%) but is associated more frequently with male breast cancer.
BRCA1 and BRCA2 carriers are also athigher risk for other epithelial cancers, such as
prostatic and pancreatic carcinomas.

711. B
712. B

713. A

714. B
Option C- Most important prognostic factor for metastatic breast cancer

715. B
Lobular carcinoma most commonly forms hard irregular masses similar to other breast
cancers, but may also have a diffuse infiltrative pattern with minimal desmoplasia.
Such cancers can be difficult to palpate or detect by imaging. This is the most
common type of breast carcinoma to present as an occult primary. The histologic
hallmark is the presence of discohesive infiltrating tumor cells, often including signet-
ring cells containing intracytoplasmic mucin droplets. Tubule formation is absent.

716. B
Inflammatory breast cancers show extensive invasion and proliferation within
lymphatic channels, causing swelling that mimics non-neoplastic inflammatory
lesions. These tumors are usually of high grade, but do not belong to any particular
molecular subtype.

717. C
Pathologic Lesion Relative Risk (Absolute Lifetime Risk)*

Nonproliferative Breast Changes (Fibrocystic changes) 1 (3%)

Duct ectasia

Cysts

Apocrine change

Mild hyperplasia

Adenosis

Fibroadenoma without complex features

Proliferative Disease Without Atypia 1.5 to 2 (5%-7%)

Moderate or florid hyperplasia

Sclerosing adenosis

Papilloma

Complex sclerosing lesion (radial scar)

Fibroadenoma with complex features

Proliferative Disease with Atypia 4 to 5 (13%-17%)

Atypical ductal hyperplasia (ADH)

Atypical lobular hyperplasia (ALH)

Carcinoma in Situ 8 to 10 (25%-30%)

Lobular carcinoma in situ (LCIS)

Ductal carcinoma in situ (DCIS)

Relative risk is the risk compared to women without any risk factors. Absolute lifetime risk
is the percentage of patients expected to develop invasive carcinoma if untreated.

718. C
Adenocarcinomas of GIT, Lung, Breast and Pancreas are associated with migratory
thrombophlebitis.

719. A
The progression of the dementia in CJD is usually so rapid that there is little if any
grossly evident brain atrophy. The pathognomonic finding is a spongiform
transformation of the cerebral cortex and, often, deep gray matter structures (caudate,
putamen); this multifocal process results in the uneven formation of small, apparently
empty, microscopic vacuoles of varying sizes within the neuropil and sometimes in the
perikaryon of neurons

720. D
Schwannomas are well-circumscribed, encapsulated masses that abut the associated
nerve without invading it, a feature that simplifies surgical excision. Grossly, these
tumors form firm, gray masses. Microscopically, they are comprised of an admixture of
dense and loose areas referred to as Antoni A and Antoni B areas, respectively. The
dense eosinophilic Antoni A areas often contain spindle cells arranged into cellular
intersecting fascicles. Palisading of nuclei is common and “nuclear-free zones” that lie
between the regions of nuclear palisading are termed Verocay bodies. In the loose,
hypocellular Antoni B areas the spindle cells are spread apart by a prominent myxoid
extracellular matrix that may be associated with microcyst formation.

721. A
Homologous recombination repair gene defect – Bloom syndrome, Fanconi Anemia,
and Ataxia telangiectasia

722. B

723. B
Option A- mesenchymal tumor origin
Option C and D- muscle tumor origin

724. C

Tumor Cytogenetic Abnormality Gene fusion

Ewing sarcoma family tumors t (11 ;22) (q24;q12) EWS-FLI1

Extraskeletal myxoid chondrosarcoma t (21 ;22) (q22;q12 EWS-ERG

Desmoplastic small round-cell tumor t (9;22) (q22;q12) EWS-CHN

Clear-cell sarcoma t (11 ;22) (p13;q12) EWS-WT1

t (12;22) (q13;q12) EWS-ATF1

Liposarcoma–myxoid and round-cell type t (12;16) (q13;p11) FUS-DDIT3

Synovial sarcoma t (x;18) (p11;q11)


SS18-SSX1

SS18-SSX2

SS18-SSX4

Rhabdomyosarcoma–alveolar type
t (2;13) (q35;q14) PAX3-F0X01

t (1;13) (p36;q14) PAX7-F0X01

Dermatofibrosarcoma protuberans t (17;22) (q22;q15) COLA1-PDGFB

Alveolar soft-part sarcoma t (X;17) (p11.2;q25) TFE3-ASPL

Infantile fibrosarcoma t (12;15) (p13;q23) ETV6-NTRK3

Nodular fasciitis t (22;17) MYH9-USP6

725. C
Description is of Schiller Duval bodies in Yolk sac or endodermal sinus tumor

726. A

727. A
Option B- most common cause of nephrotic syndrome in children
728. B
HIV encephalitis is a chronic inflammatory reaction associated with widely distributed
microglial nodules, often containing macrophage-derived multinucleated giant cells;
foci of tissue necrosis and reactive gliosis are sometimes seen together with these
lesions

729. A
Option B- most common primary CNS malignancy in children

730. C
Elongated cells are referred to as strap cells or tadpole cells. Rhabdomyoblasts contain
an eosinophilic granular cytoplasm rich in thick and thin filaments. The presence of
desminand myogenin can be detected using immunohistochemistry.

731. A
Familial adenomatous polyposis is an inherited condition with neoplastic polyps,
whereas JPS and PJP are classified under non neoplastic hamartomatous polyps.
Ulcerative colitis and Crohn’s disease comprise the spectrum of inflammatory bowel
disease and therefore also carry a risk of colorectal cancer (equal incidence).

732. C
The location, size and electron microscopy picture of this lesion goes in favour of a
carcinoid tumor or a neuroendocrine tumor. The best marker to identify such lesions is
synaptophysin and chromogranin.

733. C
The IHC pictures show nuclear positivity for ER and no membranous positivity for Her 2
Neu. This falls under the Luminal A group of molecular classification and has the best
prognosis among all the subtypes. Since Her 2 Neu is negative, Trastuzumab is not an
option for treatment but since ER is positive, hormonal therapy can be used. Basal
subtype is triple negative, having one of the worst prognosis of all the tumors.

734. C
Since the history is short, the possibility of acquiring IgA nephropathy is higher. It is
characterised by mesangial deposits.

735. D
ALK is rearranged, not mutated in lung cancer. It is translocated leading to fusion of
EML4-ALK.

736. C
Lymphoid follicles along with macrovesicular steatosis are seen in chronic hepatitis C

737. B
Most common carcinoma occurring in a patient with asbestos exposure is lung
carcinoma and most specific is mesothelioma

738. D
Chromosomal aneuploidy is seen in malignancies

739. B
Cryptitis and crypt abscesses maybe be seen in both. Granuloma formation and
transmural inflammation are characteristic of CD while pseudopolyposis is
characteristic of UC.

740. B
Red pulp involving sinuses are the site of amyloid deposition in lardaceous spleen

741. A
CEA or carcinoembryonic antigen is seen in both colon and pancreatic cancers and
other adenocarcinomas as well. CA 125 is a marker for ovarian carcinoma and CA 15-3
is a marker for breast carcinoma.

742. D
Most common primary tumors metastasising to brain are lung, breast, melanoma,
kidney and GIT. Prostate carcinoma has a tendency to primarily involve the bones, not
brain.

743. C
A longitudinal groove in the nucleus is seen in tumor cells of Brenner tumor,
Granulosa cell tumor, LCH, Chondroblastoma and papillary carcinoma thyroid.

744. C
Bone invasion does not upgrade meningioma. Chordoid and clear cell morphology is
grade II and rhabdoid and papillary morphology is grade III. EMA and PR are expressed.

745. C
Triphenyl tetrazolium chloride is a gross histochemical stain which impartsa brick-red
color to intact, noninfarcted myocardium where lactate dehydrogenase activity is
preserved. Because dehydrogenases leak out through the damaged membranes of
dead cells, an infarct appears as an unstained pale zone By 12 to 24 hours after
infarction, an MI can usually be identified grossly as a reddish-blue area of
discoloration caused by stagnated, trapped blood. progressively more sharply defined,
yellow-tan, and soft. By 10 days to 2 weeks, it is rimmed by a hyperemic zone of highly
vascularized granulation tissue. Over the succeeding weeks, the injured region evolves
to a fibrous scar.

746. A
The skin, mucous membranes, lungs, brain, heart, gastrointestinal tract, kidneys, and
muscle can all be involved; necrotizing glomerulonephritis (90% of patients) and
pulmonary capillaritis are particularly common. Microscopic angiitis can be a feature
of a number of immune disorders, such as Henoch-Schönlein purpura, essential mixed
cryoglobulinemia, and vasculitis associated with connective tissue disorders

747. C
A velvety patch in lower esophagus with the clinical history indicative of reflux
suggests Barret’s esophagus. Option B is the histopathological picture best describing
the condition. Option A is the description for squamous cell carcinoma. Option B is the
description for adenocarcinoma.

748. C
The peripheral smear shows acanthocytes and the intestinal biopsy shows vacuolated
epithelial cells suggestive of abetalipoproteinemia.

749. C
Medullary carcinomas of the thyroid are neuroendocrine neoplasms derived from the
parafollicular cells, or C cells, of the thyroid, and account for approximately 5%
ofthyroid neoplasms. Medullary carcinomas, similar tonormal C cells, secrete
calcitonin, the measurement of whichplays an important role in the diagnosis and
postoperativefollow-up of patients. In some instances the tumor cellselaborate other
polypeptide hormones, such as serotonin, ACTH, and vasoactive intestinal peptide
(VIP). About 70%of tumors arise sporadically. The remainder occurs in the setting of
MEN syndrome 2A or 2B or as familial tumorswithout an associated MEN syndrome.

750. A Capillary hemangioma


Capillary hemangiomas are the most common type; these occur in the skin,
subcutaneous tissues, and mucous membranes of the oral cavities and lips, as well as
in the liver, spleen, and kidneys. Histologically, they are composed of thin-walled
capillaries with scant stroma

751. D. APC
The photograph shows numerous adenomas of the colon, consistent with familial
adenomatous polyposis (FAP), also termed adenomatous polyposis coli (APC). This
autosomal dominant inherited disease accounts for about 1% of colorectal cancers. It
is characterized by the progressive development of innumerable adenomatous polyps
of the colorectum, particularly in the rectosigmoid region.
Germline mutations in the APC gene, a putative tumor suppressor gene, are
responsible for FAP. Carcinoma of the colon and rectum is inevitable in these patients,
and the mean age of onset is 40 years

752. B. S-100 is negative


S-100 is positive in nerve sheath tumors like schwannomas. Rest of the statements are
true.

753. C. Intestinal wall is thinned out

Feature Crohn Disease Ulcerative Colitis

Macroscopic

Bowel region affected Ileum ± colon Colon only

Rectal involvement Sometimes Always

Distribution Skip lesions Diffuse

Stricture Yes Rare

Bowel wall appearance Thick Thin

Inflammation Transmural Limited to mucosa and submucosa

Pseudopolyps Moderate Marked

Ulcers Deep, knifelike Superficial, broad-based

Lymphoid reaction Marked Moderate

Fibrosis Marked Mild to none

Serositis Marked No
Granulomas Yes (-35%) No

Fistulas/sinuses Yes No

Clinical

Perianal fistula Yes (in colonic disease) No

Fat/vitamin malabsorption Yes No

Malignant potential With colonic involvement Yes

Recurrence after surgery Common No

Toxic megacolon No Yes

754. A. Cytokeratin 8/18


MalloryDenkbodiesconsist of tangledskeins of intermediatefilaments (including
ubiquitinylatedkeratins 8 and 18) and arevisible as eosinophiliccytoplasmicinclusions
in degeneratinghepatocytes

755. A. Luminal A subtype


756. A
Stable plaques have densely collagenized and thickened fibrous caps with minimal
inflammation and negligible underlying atheromatous cores, whereas vulnerable
plaques have thin fibrous caps, large lipid cores, and increased inflammation.

757. D
One “traditional” 10% rule that has since been modified pertains to familial cases. It is
now recognized that as many as 25% of individuals with pheochromocytomas and
paragangliomas harbor a germ line mutation in one of at least six known genes,
including RET, which causes type 2 MEN syndromes; NF1, which causes type 1
neurofibromatosis ; VHL, which causes von Hippel-Lindau disease; and three genes
encoding subunits within the succinate dehydrogenase complex (SDHB, SDHC, and
SDHD), which is involved in mitochondrial electron transport and oxygen sensing. It is
postulated that loss of function in one or more of these subunits leads to stabilization
of the transcription factor hypoxia-inducible factor 1α (HIF-1α), promoting
tumorigenesis

758. A
The most frequent pituitary tumor in patients with MEN-1 syndrome is a prolactin-
secreting macroadenoma.

759. C
Who histological classification of gastric tumours1

Epithelial tumours Non-epithelial tumours

Intraepithelial neoplasia - Adenoma 8140/0 Leiomyoma 8890/0

Carcinoma Schwannoma 9560/0

Adenocarcinoma 8140/3 Granular cell tumour 9580/0

diffuse type 8144/3 Glomus tumour 8711/0

Papillary adenocarcinoma 8145/3 Leiomyosarcoma 8890/0

Tubular adenocarcinoma 8211/3 GI Stromal tumour 8936/1

Mucinous adenocarcinoma 8480/3 benign 8936/0

Signet-ring cell carcinoma 8490/3 uncertain malignant potential 8936/1

Adenosquamous carcinoma 8560/3 malignant 8936/3

Squamous cell carcinoma 8070/3 Kaposi sarcoma 9140/3

Small cell carcinoma 8041/3 Others

Undifferentiated carcinoma 8020/3 Malignant lymphomas

Others Marginal zone B-cell lymphoma of MALT- 9699/3


type
Carcinoid (well differentiated endocrine 8240/3 Mantle cell lymphoma 9673/3
neoplasm)

Diffuse large B-cell lymphoma 9680/3

Others

Secondary tumours

760. B
Myxomas are the most common primary tumor of the adult heart. These are benign
neoplasms thought to arise from primitive multipotent mesenchymal cells. Although
sporadic myxomas do not show consistent genetic alterations, familial syndromes
associated with myxomas have activating mutations in the GNAS1 gene, encoding a
subunit of G protein (Gsα) (in association with McCune-Albright syndrome) or null
mutations in PRKAR1A, encoding a regulatory subunit of a cyclic-AMPdependent
protein kinase (Carney complex). About 90% of myxomas arise in the atria, with a left-
to-right ratio of approximately 4 : 1.

761. A
Multiple immunologic mechanisms that may contribute to thyroid cell damagehave
been identified including the following:
CD8+ cytotoxic T-cell–mediated killing of thyroid epithelial cells.
Cytokine-mediated cell death. T-cell activation leads to the production of
inflammatory cytokines such as interferon-γ in the thyroid gland, with resultant
recruitment and activation of macrophages and damage to follicles.
Binding of anti-thyroid antibodies (anti-thyroglobulin, and anti-thyroid peroxidase
antibodies), followed by antibody-dependent cell–mediated cytotoxicity
In the usual clinical course, hypothyroidism develops gradually. In some cases,
however, it may be preceded by transient thyrotoxicosis caused by disruption of
thyroid follicles, with secondary release of thyroid hormones (hashitoxicosis). During
this phase, free T4 and T3 concentrations are elevated, TSH is and T3 diminished, and
radioactive iodine uptake is decreased. As hypothyroidism supervenes, T4 levels
progressively fall, accompanied by a compensatory increase in TSH.

762. A
Involvement of the liver by metastatic malignancy is far more common than primary
hepatic neoplasia. Although the most common primary sources are the colon, breast,
lung, and pancreas, any cancer in any site of the body may spread to the liver.
Typically, multiple nodular metastases are found that often cause striking
hepatomegaly and replace much of the normal liver parenchyma. The liver weight can
exceed several kilograms.

763. A, B, E

764. A

765. A

766. B
767. B

768. C (single best answer)

769. C

770. C

771. C

772. A All are associated.

773. A

774. A (single best option)

775. C

776. D

777. D

778. B

779. C

780. B

781. CD

782. C

783. A, B, C, E

784. A

785. D

786. A

787. C

788. C

789. D

790. C

791. B

792. B

793. B

794. A

795. B
796. D

797. D

798. A

799. B

800. B

801. D

802. B

803. C

804. D

805. A

806. B

807. A

808. C

809. D

810. B. Mucoepidermoid carcinoma


Mucoepidermoid carcinomas are composed of variable mixtures of squamous cells,
mucus-secreting cells, and intermediate cells. These neoplasms represent about 15%
of all salivary gland tumors, and while they occur mainly (60%–70%) in the parotids,
they account for a large fraction of salivary gland neoplasms in the other glands,
particularly the minor salivary glands. Overall, mucoepidermoid carcinoma is the most
common form of primary malignant tumor of the salivary glands

811. B

812. D

813. D

814. D

815. D

816. A, E

817. D

818. A

819. A

820. A

821. C
822. D

823. D

824. A

825. A

826. A

827. B

828. A

829. D

830. A

831. B

832. B

833. C

834. C

835. A

836. C

837. C

838. D

839. C

840. B

841. A
Option B- most common primary cardiac malignancy in children

842. C

843. B

844. A, C, E
Lung adenocarcinoma is negative for CK 20 and CK 5/6
CK 5/6 is positive in mesothelioma

845. A
Increase in the thickness of mucus gland layer can cause increase in Reid’s index.

846. A
Option B- Goodpasture syndrome
Option C- IgA nephropathy
847. A
FAP is due to APC gene mutations, present on chromosome 5.

848. B
Thrombangitis obliterans is caused due to smoking in young males in peripheral
vessels.

849. D

850. A

851. A

852. A

853. C

854. B
ATP7B on chromosome 13q

855. B

856. A

857. C

858. B

859. D

860. B

861. B

862. A
All types of invasive carcinoma are graded using the Nottingham Histologic Score.
Carcinomas are scored for tubule formation, nuclear pleomorphism, and mitotic rate
and the points added to divide carcinomas into grade I (well differentiated), grade II
(moderately differentiated), and grade III (poorly differentiated) types.

863. D
Giant cell arteritis is a medical emergency requiring prompt recognition and
treatment. Lesions also occur in other arteries, including the aorta (giant cell
aortoarteritis)

864. C

865. A
General Pharmacology — Questions
1. Which of the following drug undergo high first pass metabolism so given at
high dose orally is
A. Isoprenaline
B. Lignocaine
C. Testosterone
D. Verapamil

2. Inhibition of Pgp-mediated efflux in the BBB will lead to


A. decrease in the concentration of loperamide in the CNS
B. Increase in the concentration of loperamide in the CNS
C. Both are false
D. Both are true

3. Following drug is an inhibitor and inducer of P Glycoprotein


A. Ritonavir
B. Digoxin
C. RifampicinD. Morphine

4. Following are true about oral route of drug administration EXCEPT


A. Unpalatable drugs can’t be given
B. Absorption of drugs may be variable and erratic
C. Not useful in emergencies
D. Drugs with high first pass metabolism can be given

5. Hepatic extraction ratio is given by


A. Hepatic blood flow multiplied by Vd
B. Hepatic clearance divided by hepatic blood flow
C. Hepatic blood flow multiplied by hepatic clearance
D. Hepatic blood flow multiplied by bioavailability

6. After starting continuous IV infusion of a drug X, 75% of the drug steady-


state concentration is reached after
A. 1 half-life of drug X
B. 2 half-life of drug X
C. 4 half-life of drug X
D. 3 half-life of drug X

7. Following are examples of competitive enzyme inhibition except


A. Acetazolamide: Carbonic anhydrase
B. Carbidopa: Dopa decarboxylase
C. Moclobemide: MAO-A
D. Captopril: Angiotensin converting enzyme (ACE)
8. Parallel rightward shift of agonist DRC is seen in
A. Noncompetitive antagonism
B. Competitive agonism
C. Competitive antagonism
D. Noncompetitive agonism

9. EC50 is
A. Drug dose that produces 50% of maximal effect
B. Drug concentration that produces 50% of toxic effect
C. Drug concentration that produces 50% of lethal effect
D. Drug concentration that produces 50% of maximal effect

10. Upto 500 participants are involved in which phase of clinical trail
A. Phase I
B. Phase IV
C. Phase III
D. Phase II

11. About partial agonistic property of Buprenorphine, which of the following


is true
A. Less respiratory depression even when given at high dose
B. Decreased affinity to mu receptor
C. Can be given in opioid dependent patients
D. None of the above

1. d 2. b 3. a 4. d 5. b 6. b 7. a 8. c 9. d 10. d

11. a

12. Elevation of glucose by corticosteroids is counteracted by insulin is an


example
A. Physiological antagonism
B. CHEMICAL antagonism
C. Competitive antagonism
D. Noncompetitive antagonism

13. Binding of B-arrestin to the Gs receptor leads to


A. Increases response to agonist
B. Increased response to antagonist
C. Reduced response to agonist
D. Reduced response to agonist

14. Gentamicin volume of distribution is increased in following condition


A. Edema
B. Ascites
C. Pleural effusion
D. All of the above
15. Which of the following phase II conjugation Enzymes are located at
mitochondria
A. Glucuronidation
B. Glycine conjugation
C. Glutathione conjugation
D. Acetylation

16. Which of the following CYP inducers are less known or unknown till now
A. CYP 1A2
B. CYP 2D9
C. CYP 3A4
D. CYP 2C9

17. Charcoal-broiled foods and smoking induces following enzyme


A. CYP 3A4
B. CYP 2D9
C. CYP 2C9
D. CYP 1A2

18. Desulfuration is an example for


A. Phase 1 hydrolysis
B. Phase 1 oxidation
C. Phase 1 reduction
D. Phase 1 decyclization

19. Following are Cytochrome P450-Independent oxidations except


A. Flavin monooxygenase
B. Deamination
C. Amine oxidases
D. Dehydrogenations

20. Grapefruit juice is an


A. Enzyme inducer
B. Enzyme modulator
C. None
D. Enzyme inhibitor

21. Flucloxacillin-induced liver injury is associated with


A. HLA-B *58:01
B. DRB5 *01:01
C. HLA-DRB1 *15:01
D. HLA-B *57:01

22. Following are “rights” of safe medication except


A. Right drug
B. Right doctor
C. Right patient
D. Right route

23. Margin of safety is given by


A. LD50/ ED50
B. LD25/ ED75
C. LD5/ ED95
D. LD1/ ED99

24. Half-life is directly proportional to


A. Clearance
B. Plasma concentrations
C. Rate of elimination
D. All of the above

25. When we give gentamicin, its half life is 2-3 h but it accumulates in kidney
and its half life is prolonged for 53 hour. This half life is called as
A. Secondary half life
B. Zero order half life
C. First order half life
D. Terminal half life

26. At fetal plasma, drugs showing ion trapping are


A. Weakly acidic drugs
B. Both
C. Weakly basic drugs
D. None of the above

27. Apparent volume of distribution is


A. Total amount of drug in body divided by Drug clearance
B. Total amount of drug in body divided by Drug half life
C. Total amount of drug in body divided by elimination rate constant
D. Total amount of drug in body divided by Drug Plasma Concentration
12. a 13. d 14. d 15. b 16. b 17. d 18. b 19. b 20. d 21. d

22. b 23. d 24. b 25. d 26. c 27. d

28. The following are true about apparent volume of distribution except
A. Drugs with high molecular weight has less aVd
B. aVd of >20L/kg means the drug has a large aVd
C. Drugs with High aVd are difficult to remove by hemodialysis
D. Drugs with aVd and high protein bound are known to cause less toxicity during
displacement reactions

29. Which of the following is incorrect statement


A. In celiac disease absorption of cephalexin is increased
B. In celiac disease absorption of cotrimoxazole is increased
C. Achlorhydria decreases aspirin absorption by favoring its ionization
D. In celiac disease absorption of amoxicillin is increased

30. Which of the following drugs should be given in sustained release oral
damage form
A. An antihypertensive with a plasma half-life of 3 hours
B. An anti-arrhythmic drug with a plasma half-life of 10 seconds used for acute
treatment of PSVT
C. An anti-inflammatory drug with a plasma half-life of 24 hour
D. A hypnotic drug with a plasma half-life of 2 hours

31. Which of the following statements best describes an ‘orphan drug’


A. It is a drug which acts on orphanin receptors
B. It is a very cheap drug
C. It is a drug required for treatment or prevention of a rare disease
D. It is a drug which has no therapeutic use

32. Which of the following is true of ‘placebo’


A. Placebo is the inert material added to the drug for making tablets
B. Placebo do not produce any effect
C. All patients respond to placebo
D. Placebo is a dummy medication

33. Therapeutic drug monitoring of plasma concentrations of antihypertensive


drugs are not practiced because
A. Determination of plasma level of these drugs is quite tedious and long process
B. Antihypertensive effect does not increase linearly with the dose
C. All antihypertensive drugs are prodrugs
D. It is easier to measure the effect of these drugs

34. A 56-year-old man, Surender with heart failure is to be treated with a


diuretic drug. Drugs A and B have same mechanism of action. Drug A in dose
of 50 mg produces the same magnitude of diuresis as 500 mg of drug B. This
suggests that
A. Drug B is less efficacious than drug A
B. Drug A is safer drug than drug B
C. Drug A will have shorter duration of action than drug
D. Drug A is more potent than drug B

35. Which of the following does not act as second messenger


A. G protein
B. Cyclic AMP
C. Inositol trisphosphate
D. Diacylglycerol
36. Which of the following drugs act through heptahelical (serpentine)
receptors
A. Insulin
B. Salbutamol
C. Estrogen
D. Local anesthetics

37. Fastest acting receptor/transduction mechanism is


A. Intrinsic ion channel operation
B. Adenylyl cyclase- cyclic AMP pathway
C. Phospholipase C-IP3: DAG pathway
D. Nuclear receptor
28. d 29. d 30. a 31. c 32. d 33. d 34. d 35. a 36. b 37. a

38. Dose response curves of salbutamol for bronchodilation and tachycardia


are widely separated in the dose axis. This information suggests that
salbutamol is
A. Highly selective drug
B. Highly potent cardiac stimulant
C. Highly efficacious bronchodilator
D. Highly toxic drug

39. Which of the following is most likely due to a pharmacogenetics condition


A. Primaquine induced hemolytic anemia
B. Hypoglycemia by insulin
C. Tachycardia by albuterol
D. Metoclopramide induced muscle dystonia

40. Which of the following most accurately describes the transmembrane


signaling process involved in the steroid hormone action
A. Action on a membrane spanning tyrosine kinase
B. Diffusion across the membrane and binding to an intracellular receptor
C. Action of a G protein, which activates or inhibits adenylyl cyclase
D. Opening of transmembrane ion channels

41. Which of the following terms best describes the antagonism of


leukotrienes’ Broncho constrictor effect (mediated at the leukotriene
receptors) by terbutaline (acting at the adrenoceptor) in a patient with
asthma
A. Pharmacologic antagonist
B. Physiologic antagonist
C. Partial agonist
D. Chemical antagonist

42. This drug has to be administered to a 65 years old patient Uttam Singh,
with a GFR of 60 ml/min. (assuming normal GFR is 120 ml/min). Liver and
biliary functions are normal in this patient. What should be the dose rate of
drug X in this patient
A. 50 mg/hour
B. 25 mg/hour
C. 100 mg/hour
D. 30 mg/hour

43. A volunteer Ram will receive a new drug in phase I clinical trial. The
clearance and the volume of distribution of the drug in Ram are 1.386 L/hr.
and 80 L respectively. The half-life of the drug in him would be
approximately
A. 40 hr. B. 80 hr.
C. 77 hr. D. 0.02 hr.

44. A patient requires an infusion of procainamide. It half-life is 2 hrs. The


infusion is begun at 9 AM at 1 PM on the same day, the blood concentration
is found to be 3 mg/L. What is the probably steady state concentration after
2 days of infusion
A. 3 mg/L
B. 6 mg/L
C. 15 mg/L
D. 4 mg/L

45. A young male Kallu is brought to the hospital with severe asthma. The
pharmacokinetics of theophylline include the following parameters: V4= 35
L; CL= 48 ml/min; half-life is 8 hrs. If an intravenous infusion of theophylline
is started at the rate of 0.48 mg/min, how will it take to reach 93.75% of the
final steady state
A. Approximately 32 hrs
B. Approximately 48 min
C. Approximately 5.8 hrs
D. Approximately 8 hrs

46. An old man enters the hospital with myocardial infraction and a severe
ventricular arrhythmia. The antiarrhythmic drug chosen has a narrow
therapeutic window. The minimum toxic plasma concentration is 1. 5 times
the minimum therapeutic plasma concentration. The half-life is 6 hrs. It is
essential to maintain the plasma concentration above the minimum
therapeutic level to prevent a possible lethal arrhythmia. Of the following,
the most appropriate dosing regimen would be
A. Constant intravenous infusion
B. Once a day
C. Twice a day
D. Four times a day

38. a 39. a 40. b 41. b 42. d 43. a 44. d 45. a 46. a

47. A 30-year-old patient o11 digoxin therapy has developed d digitalis


toxicity. The plasma digoxin level is 4 n 9lml. Renal function is normal and
the plasma t1/2 for digoxin in this patient is 1. 6 days. How long should you
withhold digoxin in order to reach a safer yet probably therapeutic level of 1
ng/ml
A. 1.6 Days
B. 3.2 Days
C. 2.4 Days
D. 4.8 Days

48. A drug following first order kinetics is being administered by constant i.v.
infusion at a rate of 10 mg/min. Its steady state plasma concentration is 2
mg/min. If the dose rate is increased to 20 mg/dl, what will be the new
steady state plasma concentration?
A. 6 mg/ dl
B. 3 mg/ dl
C. 1 mg/ dl
D. 4 mg/ dl

49. Rate of elimination of a new drug is 20 mg/hr at a steady state plasma


concentration of 10 mg/L, then its renal clearance will be
A. 0.5 L/hr
B. 5.0 L/hr
C. 20 L/hr
D. 2.0 L/hr

50. Maintenance dose rate of a drug depends primarily on


A. Total body clearance
B. Volume of distribution
C. Half life
D. Lipid soluble

51. Amount of a drug X administered to a patient is 4.0 g and its plasma


concentration is found to be 50 ug/ml, what will be the volume of
distribution of drug X?
A. 100 L
B. 60 L
C. 50 L
D. 80 L

52. A patient, Rajesh with a history of wheezing, coughing and shortness of


breath is being evaluated in the asthma clinic. Several drug treatments with
different routes are under consideration. Which of the following statements
about routes of administration is most correct?
A. Administration of a bronchodilator drug by inhaled aerosol is usually associated
with more adverse effects than administration of this drug by mouth.
B. Bioavailability of most drugs is greater with rectal administration than with
sublingual administration
C. Administration of a drug by transdermal patch is often faster but is associated with
more first pass metabolism than oral administration
D. The first pass effect is the result of elimination of a drug after administration and
before it enters systemic circulation.

53. A three-year-old child is brought to the emergency department having just


ingested a large overdose of an antihistaminic drug. This drug is a weak
base capable of entering most tissues including the brain. On physical
examination the heart rate is 1007 minute, blood pressure is 110/60 mm Hg
and the respiratory rate is 20/minute. In this case of poisoning
A. Urinary excretion would be accelerated by administration of NaHCO3 an
alkalinizing agent
B. More of the drug would be ionized at blood pH than at stomach pH
C. Absorption of the drug would be faster from the stomach than from the small
intestine.
D. Urinary excretion would be accelerated by administration of NH4 C1, an acidifying
agent

54. A factor that is likely to increase the duration of action of a drug D that is
partially metabolized by CYP3A4 in the liver is:
A. Chronic administration of phenobarbital with the drug
B. Displacement from tissues binding sites by another drug
C. Chronic administration of rifampicin
D. Chronic administration of cimetidine with the drug
47. b 48. d 49. d 50. a 51. d 52. d 53. d 54. d

55. Which of the following factors has maximum effect on filtration of a drug
by the glomerulus
A. Lipid solubility
B. Degree of ionization
C. Rate of tubular secretion
D. Plasma protein binding

56. All of the following reactions are catalyzed by microsomal enzymes


EXCEPT:
A. Glucuronidation
B. Oxidation
C. Reduction
D. Acetylation

57. Which of the following statements about a drug having high plasma
protein binding is true?
A. High plasma protein binding decreases the volume of distribution
B. Volume of distribution of the drug is very high
C. This drug will be filtered quickly by glomerulus
D. This drug is likely to have minimum chances of drug interactions

58. All of the following are advantages of transdermal drug delivery systems
EXCEPT:
A. They produce smooth and no fluctuating plasma concentration of the drug
B. They minimize interindividual variations in the achieved plasma drug concentration
C. They produce high peak plasma concentration of the drug
D. They avoid hepatic first pass metabolism of the drug

59. A new drug is found to be highly lipid soluble, it is metabolized at a slower


rate of 10% per hour. On intravenous injection it produces general
anesthesia that lasts only for 15 min. This short duration of anesthesia is
due to:
A. Metabolism of the drug in liver
B. Redistribution
C. High plasma protein binding of the drug
D. Excretion of drug by kidney

60. Metabolism of a drug primarily results in


A. Conversion of lipid soluble drugs to water soluble metabolites
B. Activation of the active drug
C. Conversion of prodrug to active metabolite
D. Conversion of water soluble drug to lipid soluble metabolites

61. A drug X is secreted through renal tubules, tubular secretion of this drug
can be confirmed if renal clearance of drug X is
A. Equal to the GFR
B. Less than the GFR
C. More than the GFR
D. More than volume of distribution

62. All of the following factors tend to increase the volume of distribution of a
drug EXCEPT
A. Low ionization at physiological pH values
B. High lipid solubility
C. High tissue binding
D. High plasma protein binding

63. Which of the following drugs has maximum chances of absorption from
gastric mucosa?
A. Morphine sulfate
B. Hyoscine hydrobromide
C. Quinine dihydrochloride
D. Diclofenac sodium

64. Urinary alkalinizing agents are administered in case of poisoning due to


drugs which are
A. Weak acids
B. Strong bases
C. Strong acids
D. Weak bases
65. Titration of the dose of a drug with the response can be done with which
of the following routes of administration?
A. Inhalational
B. Sublingual
C. Transdermal
D. Subcutaneous
55. d 56. d 57. a 58. c 59. b 60. a 61. c 62. d 63. d 64. a

65. a

66. The most general term for the process by which the amount of active drug
in the body is reduced after absorption into the systemic circulation is
A. Excretion
B. First pass metabolism
C. Distribution
D. Elimination

67. The process by which the amount of a drug in the body decreases after
administration but before entering the systemic circulation is called
A. Excretion
B. First order elimination
C. Metabolism
D. First pass effect

68. Regarding termination of drug action


A. Hepatic metabolism and renal excretion are the two most important mechanisms
Drugstore
B. Drugs must be excreted from the body to terminate their action
C. Metabolism of drugs always abolishes their pharmacologic activity
D. Distribution of a drug out of blood stream terminates the drug’s effects

69. Drug which works by psychodynamic rather than Pharmacodynamic means


is
A. Orphan Drug
B. Nocebo
C. Psychotropic Drug
D. Placebo

70. All of the following are predicted from the pharmacological profile of a
drug except
A. Secondary Effect
B. Adverse drug event
C. Toxic Effect
D. Side Effect

71. All of the following are predicted from the pharmacological profile of a
drug except
A. Secondary Effect
B. Toxic Effect
C. Side Effect
D. Adverse drug event

72. Which of the following acts through membrane Soluble guanylyl cyclase
A. Insulin
B. NO
C. Herceptin
D. Nesiritide

73. A drug Z has affinity and intrinsic activity of 0.8, then it is a


A. Agonist
B. Inverse Agonist
C. Competitive Antagonist
D. Partial Agonist

74. Which of the following studies in clinical trial is done mainly for
pharmacokinetic profile of drug using < 100 microgram of drug
A. Human therapeutic exploratory study
B. Human safety pharmacology study
C. Human therapeutic confirmatory study
D. Human micro dosing study

75. Choose the incorrect statement regarding drug absorption


A. First class metabolism reduces bioavailability
B. Heparin not absorbed due to its large size
C. Drugs which dissolve in water are rapidly absorbed from GIT
D. Highly ionized molecule are absorbed from stomach

76. Evaluation of safety and tolerability in humans using maximum tolerated


dose (MTD) as end point is done in
A. Phase II
B. Phase III
C. Phase IV
D. Phase I

77. Rebound hypertension after clonidine withdrawal is following type of ADR


A. Type A
B. Type E
C. Type B
D. Type D

78. Essentials Medicines are those that


A. Are essential for lifestyle modification
B. Treat only epidemic diseases
C. Are newer in the market
D. Satisfy the priority health care needs
66. d 67. d 68. a 69. d 70. b 71. d 72. b 73. d 74. d 75. d

76. d 77. b 78. d

79. Conversion of Diazepam to Oxazepam is an example of


A. Active drug to inactive metabolite
B. Inactive drug to active metabolite
C. Active drug to active metabolite
D. Active drug toxic metabolite

80. Which of the following drug undergo high first pass metabolism so given at
high dose orally is
A. Isoprenaline
B. Lignocaine
C. Testosterone
D. Verapamil
79. c 80. d
General Pharmacology — Explanations
1. D. Verapamil
Option A, B and C - Isoprenaline, Lignocaine and Testosterone respectively undergo
high first pass metabolism so not given orally. Drug undergoing high first pass
metabolism so given at high dose orally is Verapamil. [Ref: KDT, Topic first pass
metabolism]

2. B. Increase in the concentration of loperamide in the CNS


Loperamide:
Substrate of Pgp
Pgp prevents accumulation of loperamide in the CNS
Inhibition of Pgp-mediated efflux in the BBB would cause an increase in the
concentration of loperamide in the CNS and potentiate adverse effects.
Clinical Implication:
Coadministration of Loperamide and the potent Pgp inhibitor quinidine results in
significant respiratory depression [Ref: GG]

3. A. Ritonavir
Ritonavir is an inhibitor and inducer of P Glycoprotein
Digoxin = Is a substrate of P Glycoprotein
Rifampicin = is an inducer of P Glycoprotein and also induces CYP enzymes
[Ref: Richard B. Kim (2002) Drugs as P-glycoprotein substrates, inhibitors and
inducers, Drug Metabolism Reviews, 34:1-2, 47-54.]

4. D. Drugs with high first pass metabolism can be given


Note: Ref: KDT
Oral route of drug administration
Slower onset not useful in emergencies
Unpalatable drugs can’t be given
Cannot be used for uncooperative/unconscious/vomiting patient
Drugs destroyed by digestive juices like penicillin G, insulin
Drugs with high first pass metabolism can’t give like testosterone, Lidocaine

5. B. Hepatic clearance divided by hepatic blood flow


Ref: KAT
Hepatic extraction ratio is given by hepatic clearance divided by hepatic blood flow.

6. B. 2 half-life of drug X
Note: Ref: KAT
Fifty percent of the steady-state concentration is reached after one half life
75% of the steady-state concentration is reached after two half life
Over 90% of the steady-state concentration is reached after four half lives

7. A. Acetazolamide: Carbonic anhydrase


Note: Ref: KDT
Competitive enzyme inhibition
Carbidopa: Dopa decarboxylase
Moclobemide: MAO-A
Captopril: Angiotensin converting enzyme (ACE)
Noncompetitive enzyme inhibition
Acetazolamide: Carbonic anhydrase

8. C. Competitive antagonism
Note: Ref: KAT
Parallel rightward shift of agonist DRC is seen in competitive (reversible) antagonism
Flattening of agonist DRC is seen in noncompetitive antagonism
Irreversible competitive antagonism agonist DRC is shifted to the right and the
maximal response is lowered.

9. D. Drug concentration that produces 50% of maximal effect


Note: Ref: KAT
EC50 is the Drug concentration that produces 50% of maximal effect
ED 50 Drug dose that produces 50% of maximal effect. Lower the ED50, higher the
potency

10. D. Phase II
Note: Ref: GG
50-500 participants are involved in Phase II of clinical trail
10-100 participants are involved in Phase I of clinical trail
A few hundred to a few thousand participants are involved in Phase III of clinical trail

11. A. Less respiratory depression even when given at high dose


Note: Ref: KAT
About partial agonistic property of Buprenorphine:
Affinity to mu receptor is normal
Less respiratory depression even when given at high dose, because it is a partial
agonist
Cannot be given in opioid dependent patients, because partial agonist acts as
antagonist in presence of agonist at precipitates withdrawal

12. A. Physiological antagonism


Note: Ref: KAT
Physiological antagonism:
It is an antagonism between endogenous regulatory pathways mediated by different
receptors
Glucocortisteriods and insulin act on quite distinct receptor effector systems ad
regulate glucose
Histamine and Adrenaline act at different receptor and cause bronchoconstriction
and bronchodilation respectively

13. D. Reduced response to agonist


Note: Ref: KAT
In receptor desensitization
Binding of B-arrestin to the receptor further diminishes the receptor’s ability to
interact with G s and leads to reduced cellular response for binding of agonist
Example beta 2 agonist induced bronchodilation

14. D. All of the above


Note: Ref: KAT
Gentamicin:
Hydrophilic
Has small volumes of distribution
But in Abnormal accumulation of fluid in edema, ascites, pleural effusion can
markedly increase the volume of distribution of it

15. B. Glycine conjugation


Note: Ref: KAT
Glycine conjuction: Acyl-CoA glucinetranferase (mitochondria)
Glucuronidation: UDP glucuronosyltransferase (microsomes)
Acetylation: N-Acetyltransferase (cytosol)
Glutathione conjuction: GSH-S-transferase (cytosol, microsomes)

16. B. CYP 2D9


Note: Ref: KAT
for CYP 2D6, inducers are less known or unknown till now

17. D. CYP 1A2


Note: Ref: KAT
Charcoal-broiled foods, cruciferous vegetables, grilled meat and smoking induces CYP
1A2
Clinical implication:
Theophylline metabolized by CYP 1A2 smoking induces CYP 1A2 and reduce levels of
theophylline

18. B. Phase 1 oxidation


Note: Ref: KAT
Desulfuration is an example for phase 1 oxidation Thiopentone undergoes
Desulfuration

19. B. Deamination
Note: Ref: KAT
Cytochrome P450-independet oxidations except:
Flavin monooxygenase
Amine oxidases
Dehydrogenations
Deamination IS a cytochrome P450 dependent oxidation

20. D. Enzyme inhibitor


Note: Ref: Kat
Grapefruit juice is an Enzyme Inhibitor
Increases levels of Alprazolam, Atorvastatin, Cisapride, Cyclosporine
Midazolam, Triazolam
Remember: St. John’s worst is an Enzyme inducer
21. D. HLA-B *57:01
Note: Ref: KAT
HLA-B *57:01 Flucloxacillin-induced liver injury
HLA-B *57:01 Abacavir-induced skin toxicity
HLA-B *58:01 Allopurinol-induced skin toxicity
HLA-DRB1 *15:01 Amoxicillin-clavulanate-induced liver injury
HLA-B *15:02 Carbamazepine-induced skin toxicity

22. B. Right doctor


Note: Ref: GG
Following are “five rights” of safe medication administration can help practitioners
avoid medication errors
Right drug
Right patient
Right dose
Right route
Right time

23. D. LD1/ ED99


Note: Ref: GG
The ED99 for the therapeutic effect can be compared to LD1 for lethality (toxic effect),
to yield a margin of safety.
Therapeutic Index = LD50/ ED50

24. B. Plasma concentrations


Note: Ref: GG
If you know the formula, very easy to answer
T ½ = 0.693 Vd /CL
Cl= rate of elimination / PLASMA concentrations
So from the above formula inference is
Half Life
Is directly proportional to plasma concentrations and Vd
Half life
Is indirectly proportional to elimination rate and clearance

25. D. Terminal half life


Note: Ref: GG
Terminal half life:
With prolonged dosing (or with high drug concentrations), a drug may penetrate
beyond the central compartment into “deep” or secondary body compartments that
equilibrate only slowly with the plasma
When the infusion or dosing stops, the drug will be initially cleared from plasma as
expected but will eventually drop to a point at which net diffusion from the secondary
compartments begins, and this slow equilibration will produce a prolongation of the
half-life of the drug, referred to as the terminal half-life.

26. C. Weakly basic drugs


Note: Ref: GG
At fetal plasma is slightly more acidic than that of the mother (pH 7.0-7.2 vs 7.4)
So that ion trapping of weakly basic drugs occurs.

27. D. Total amount of drug in body divided by Drug Plasma Concentration


Note: Ref: KDT
Apparent volume of distribution is total amount of drug in body divided by Drug
Plasma Concentration

28. D. Drugs with aVd and high protein bound are known to cause less toxicity
during displacement reactions
Note: Ref: KDT
Drugs with high molecular weight has less aVd
EXAMPLE herapin
Drugs with low aVd and high protein bound are known to cause more toxicity during
displacement reactions
Salicylates displacing Tolbutamide from protein binding leading to hypoglycaemia
aVd of >20L/kg means the drug has a large aVd
a greater the value, greater the aVd
Drugs with High aVd are difficult to remove by hemodialysis
These are present in tissue stores, so not available for removal by dialysis
Ex: Digoxin

29. D. In celiac disease absorption of amoxicillin is increased


Note: Ref: KDT
In celiac disease absorption of amoxicillin is decreased
In celiac disease absorption of cephalexin and cotrimoxazole is increased
Achlorhydria decreases aspirin absorption by favoring its ionization

30. A. An antihypertensive with a plasma half-life of 3 hours


Sustained release formulations of a drug are used for the drugs having short duration
of action requiring prolonged administration.
PSVT is m acute condition and requires a short and fast acting drug. Adenosine is
therefore, the drug of choice for this condition.
Option (b) describes a drug having long half-life (24 hours). This drug need not be
administered as sustained release preparation,
For a hypnotic drug & action required is brief I. e. To induce sleep. Therefore, it should
not be given as a sustained release preparation.
Treatment of hypertension is lifelong. Drug with hall life of 3 hours, needs to be
administered several times a day, therefore sustained release oral dosage form is
best utilized for this drug.

31. C. It is a drug required for treatment or prevention of a rare disease


Orphan drugs are used for rare diseases e. g. Erythropoietin for the treatment of
anemia in patients with chronic renal failure
Ref KDT

32. D. Placebo is a dummy medication


Placebo plays a very important role m the clinical trials. To know, whether the effect is
produced by a drug or it is just by chance, a dummy medication known as placebo is
given to the control group, placebo may or may not produce an effect in a subject.
Ref KDT
33. D. It is easier to measure the effect of these drugs
TDM is done for drugs (with narrow therapeutic index) whose response cannot be
monitored by clinical examination Antihypertensive effect can be easily measured,
therefore TDH · 1 is not required
Ref KDT

34. D. Drug A is more potent than drug B


The drug producing the same response at lower dose is more potent. In this question,
Drug A is 10 times more potent than drug B. This data does not indicate anything
about efficacy, safety or duration of action.
Ref Katzung

35. A. G protein
Cyclic AMP, IP3, and DAG are second messengers whereas G Proteins are the first
messengers. IP3 and DAG increase the release of Ca2+ that acts as a third messenger.
Ref KDT

36. B. Salbutamol
GPCRs are heptahelical or serpentine receptors. Salbutamol acts through 02 receptors
which arc GPCRs.
Ref KDT

37. A. Intrinsic ion channel operation


Drugs acting via ionotropic receptors are fastest acting whereas those acting though
nuclear receptors at a slowest in action
Ref Katzung; KDT

38. A. Highly selective drug


Wide separation of two curves on DRC suggests that the dose required to produce one
action is much higher than the other.
It is seen that bronchodilation (13, action) appears at low doses whereas tachycardia
(P, action) is present only at high doses. Thus, it shows that the drug is highly selective
for P/receptors.
Ref Katzung

39. A. Primaquine induced hemolytic anemia


Primaquine is an oxidant drug. It can cause hemolytic anemia in subjects having
deficiency of G-6-PD enzyme. This condition is genetically determined.
Ref KDT

40. B. Diffusion across the membrane and binding to an intracellular receptor


Steroid hormones are lipid soluble and act on cytoplasmic receptors after crossing the
plasma membrane.
Ref KDT

41. B. Physiologic antagonist


Drugs producing opposite action by acting on different receptors are called
physiological antagonists.
Ref Katzung
42. D. 30 mg/hour
In this question 80 percent of drug is eliminated by renal route and 20 percent by
non-renal routes (10 percent by hepatic metabolism and 10 percent by biliary
secretion).
This patient has 50 percent renal fiction (60 m1/min of GFR instead of 120 ml/min).
Thus, the drug that can be eliminated in this person is 20 percent (Non-renal route) +
40 percent (Rena] route 50 percent of 80 percent)
=60 percent
Thus, the dose rate should be 60 percent of the original. i.e. 50 mg/hr. x 60 percent =
30 mg/hr.
Ref Goodman and Gilman 12e/ p 36, 37

43. A. 40 hr
T1/2 = 0.693 * vd/CL
Ref Katzung

44. D. 4 mg/L
Half-life of this drug is 2 hours and its plasma concentration is 3 mg/L after 4 hours (9
AM to 1 PM)
This means, after 2 half-lives (4 hours) plasma concentrations is 3 mg /L. We know, by
constant i.v. infusion, plasma concentration attained is 75% of the steady state in 2
half-lives, so if 3 mg/L is 75% of steady state it will be amount to 4 mg/L
Ref Katzung

45. A. Approximately 32 hrs


For a drug following first order kinetics, rise in plasma concentration as well as fall in
plasma concentration is similar. when the steady state Is attained and the drug
administration is stopped, it will be eliminated from the body. 50% will be eliminated
from the body. 50% will be eliminated in one half-life, 75% in 2t1/2 87.5% (50 + 25 +
12. 5%) in 3t1/2 and 93 75% (50 + 25 + 12. 51 6. 25%) in four hail lives.
When constant i.v. infusion is administered, plasma concentration increases in the
same manner. In one half-life, it is 50% of the steady state and to reach 93.75% of
steady state, 4 half-lives will be required.
As half-life of this drug is 8 hours, approximately 32 hours (4 × 8) will be taken.

46. A. Constant intravenous infusion


When a drug is administered less frequently, it produces marked variation in the
plasma concentrations. In this question the drug has a half-Life of 6 hours. If we
repeat the dose at 6 hourly intervals, there will be 100% variation in the plasma
concentration.
More frequent dosing will minimize the variation between maximum and minimum
plasma concentrations. As the margin of safety of this drug will (give in the question)
is very low (maximum tolerable concentration is 1.5 times the effective
concentration), constant i.v. infusion is the best route
Ref Katzung

47. B. 3.2 Days


We want to decrease the plasma concentration of digoxin from 4 ng/ml to 1 ng/ml. It
will take two half-lives. Thus time required will be 2 × t1/2 i. e. 2 × 1.6 = 3. 2 days.
48. D. 4 mg/ dl
Dose Rate = Clearance * Steady state plasma concentration
This means plasma concentration at steady state is a direct function of the dose rate,
if clearance is constant. In first order kinetics (clearance is constant), plasma
concentration attained is directly proportional to the dose rate. Thus, doubling of dose
rate from 10 to 20 mg/min, will double the steady state plasma concentration (from 2
to 4 mg/dl).
Ref Katzung

49. D. 2.0 L/hr


Clearance = Rate of Elimination / Plasma Concentration
= 20 mg /hr / 10 mg/L = 2L / hr
Ref Katzung

50. A. Total body clearance


Maintenance dose is determined by clearance.
Maintenance dose = CL × Plasma concentration required
Ref KDT

51. D. 80 L
Vd = Amount Administered / Plasma Concentration
= 4 g / 50 ug / ml = 80L
Ref Katzung

52. D. The first pass effect is the result of elimination of a drug after
administration and before it enters systemic circulation.
Inhalational route provides localized delivery to respiratory system and thus is
associated with lesser adverse effects than the systemic routes like oral
Option (a) is thus false.
Option (b) is the definition of first pass metabolism as given in the text.
When a drug is administered by rectal route, first pass metabolism is less than oral
route. But sublingual administration completely avoids first pass metabolism.
Therefore, option c is also wrong.
Transdermal route is associated with slower absorption of a drug because the pore
size is smaller. However, first pass metabolism is avoided because the drug directly
enters the systemic circulation.
Ref Katzung

53. D. Urinary excretion would be accelerated by administration of NH4 C1, an


acidifying agent
This question can be solved by the knowledge that basic drugs are ionized in the
acidic medium and vice-a-versa. This antihistamine drugs a weak base and will be
highly ionized in the acidic urine. As ionized drugs cannot be reabsorbed in the
nephron, urinary acidifying agents like NH, C1 will accelerate excretion of this agent.
On the other hand, NaHCO3. will decrease its excretion by increasing the unionized
form. Blood pH is slightly alkaline, whereas gastric pH is highly acidic. Basic drugs are
ionized more in the acidic pH, therefore option (c) is false. Only ionized molecules can
cross the membranes, therefore more drug will be absorbed by the small intestine
(alkaline pH) than by the stomach.
Ref Katzung

54. D. Chronic administration of cimetidine with the drug


Cimetidine is a microsomal enzyme inhibiting drug. It increases the duration of
action of the drugs metabolized by these enzymes On the other hand rifampicin and
phenobarbitone are enzyme inducers and will decrease the duration of action of such
drugs.
Displacement from binding sites increases the free drug that can be quickly
metabolized.
Ref KDT

55. D. Plasma protein binding


Glomerular filtration is dependent on renal blood flow and plasma protein binding. It
does not depend on the lipid solubility
Ref KDT

56. D. Acetylation
Most of the phase 1 reactions and glucuronide conjugation (phase II reaction) are
catalyzed by microsomal enzymes. These enzymes can be induced or inhibited by
drugs. Acetylation is carried out by N-acetyl transferase, a non-microsomal enzyme.
Ref KDT

57. A. High plasma protein binding decreases the volume of distribution


When a drug is highly bound to plasma proteins, it is more likely to stay in blood and
thus Vd is less. Glomerular filtration depends on renal blood flow and plasma by
protein binding. Highly protein bound drugs are less likely to be filtered by the
glomerulus. Due to non-specific binding sites on plasma proteins these drugs are
subjected to several drug interactions.
Ref Katzung

58. C. They produce high peak plasma concentration of the drug


Transdermal route is employed for highly lipid soluble drugs that can traverse intact
skin, the size of the pores in transdermal patch is adjusted to produce a uniform and
smooth absorption of the drug. This will thus, Produce a delayed and smaller peak in
the plasma concentration. As the drug is going directly in the blood stream, first pass
metabolism is avoided.
Ref KDT

59. B. Redistribution
Highly lipid soluble drugs like Thiopentone are quickly distributed to the tissue having
high blood supply (like brain). If the target organ is also having high blood supply,
drug action will be very quick. This is the case with general anesthetics like
thiopentone. Now, the drug will be distributed to less vascular tissues like fat and
muscle Movement of the drug outside Me brain results in the termination of its action.
This is called redistribution.
Ref KDT

60. A. Conversion of lipid soluble drugs to water soluble metabolites


After metabolism most of the drugs become inactive and are excreted through the
kidney. Lipid soluble drugs will be reabsorbed whereas water soluble drugs are easily
excreted. Thus, metabolism of drugs helps in the conversion of lipid soluble drugs to
water soluble metabolites.
Ref Katzung

61. C. More than the GFR


After filtration from glomerulus, a drug may undergo two processes (tubular
reabsorption and tubular secretion) before going out from the body the body i.e. renal
clearance.
Suppose 100 mg of a drug is filtered by glomerulus and the renal clearance is 150
mg, it means 50 mg is coming from somewhere else, Le. Tubular secretion must be
present. However, we cannot say that reabsorption is not occurring because if 20 mg
is reabsorbed and 70 mg is secreted, same timing can happen.
Suppose, 100 mg of a drug is filtered but renal clearance is 50 mg. Therefore, 50 mg
must have gone somewhere I. e. Tubular reabsorption must be occurring, Again, we
cannot say that tubular secretion is not present.
Ref KDT

62. D. High plasma protein binding


If a drug is highly bound to plasma proteins, it is more likely to stay in the blood.
Thus, its vd will be less.
Low ionization favors the distribution of a drug because unionized molecules can
cross the membranes of blood vessels and the tissues.
More lipid soluble drugs can easily cross the membranes and are more likely to be
highly distributed.
Ref: Katzung

63. D. Diclofenac sodium


Diclofenac is an acidic drug and is non-ionized in the acidic medium of stomach.
Therefore, it has the maximum chances of absorption from the stomach. Other drugs
given in the options are basic drugs that are ionized at gastric pH.
Ref: Katzung; KDT

64. A. Weak acids


Strong electrolytes (strong acid and strong base) are ionized in all media, whether it is
acidic or basic. Weak acids are ionized in the alkaline medium and are easily excreted.
Ref: Katzung

65. A. Inhalational
Inhalational anesthetic agents like halothane are used in the clinical practice by
titration of dose with response.
Ref: KDT

66. D. Elimination
Ref: Katzung

67. D. First pass effect


Reduction in the amount of drug before it enters the systemic circulation is called first
pass metabolism (also known a6 first pass effect) whereas if the amount of drug
decreases after entry into the systemic circulation, it is called elimination. Latter
includes excretion and metabolism.
Ref Katzung

68. A. Hepatic metabolism and renal excretion are the two most important
mechanisms Drugstore
Action of a drug can be terminated either by hepatic metabolism or by renal
excretion. Most of the drugs are inactivated by metabolism. However, some drugs
may be activated from inactive form (pro-drugs) and others may produce active
metabolites.
Some drugs may act away from blood e. g. Digoxin leaves blood stream and enters
the heart to produce its action
Ref: KDT

69. D. Placebo
Note: Ref: KDT
Placebo:
An inert substance which is given in the garb of a medicine works by Psychodynamic
rather than Pharmacodynamic means.
Uses:
Control device in clinical trial of drugs
To treat a patient who are placebo reactors
Placebos can release endorphins in brain causing analgesia
Naloxone blocks placebo analgesia.
Placebo effects are highly variable even in the same individual
Example:
Placebo are lactose tablets/capsules and distilled water injection
Sham surgeries
Ref: KDT
Placebo:
An inert substance which is given in the garb of Pharmacodynamic means
Uses:
Control device in clinical trial of drugs
To treat a patient who are placebo reactors
Placebos can release endorphins in brain causing analgesia
Naloxone blocks placebo analgesia.
Placebo effects are highly variable even in the same individual
Example:
Placebo are lactose tablets/capsules and distilled water injection
Sham surgeries

70. B. Adverse drug event


Note: Ref: KDT
Adverse drug event:
Any untoward medical occurrence that may present during treatment with a
medicine, but which does not necessarily have a causal relationship with the
treatment
Cannot be predicted from the pharmacological profile of a drug
NOTE:
Side effect, toxic effect and Secondary effects can be predicted from the
Pharmacological profile of a drug

71. D. Adverse drug event


Note: Ref: KDT
Adverse drug event:
Any untoward medical occurrence that may present during treatment with a
medicine, but which does not necessarily have a causal relationship with the
treatment
Cannot be predicted from the pharmacological profile of a drug
NOTE:
Side effect, toxic effect and Secondary effects can be predicted from the
Pharmacological profile of a drug

72. B. NO
Note: Ref: GG
Soluble guanylyl cyclase:
NO activites Soluble guanylyl cyclase
Produces a 200- to 400-fold increase in the Vmax of guanylyl cyclase, leading to an
elevation to cellular Cgmp
This is how nitrates act
Membrane bound guanylyl cyclase Tyrosine phosphatases
Natriuretic peptides act through this
They increase c GMP
EXAMPLE: Nesiritide

73. D. Partial Agonist


Note: Ref: KDT
Agonist: has affinity and intrinsic activity 1
Partial Agonist: has affinity and intrinsic activity (0 to 1)
Inverse Agonist: has affinity and intrinsic activity (0 to -1)
Competitive Antagonist: has affinity and intrinsic activity (0)

74. D. Human micro dosing study


Note: Ref: KDT
Micro dosing:
Conducted prior to phase I human clinical trail
Involved limited human exposure
No therapeutic/diagnostic intent in this phase
Done for Pharmacokinetic profile of drug
1/100th or < 100 microgram dose tested
Helps in choosing drug with more success rate
Helps in reduction and replacement of animal testing

75. D. Highly ionized molecule are absorbed from stomach


Note: Ref: KDT
Factors affecting drug absorption
First class metabolism reduces bioavailability
Highly ionized molecule are not at all absorbed from stomach or intestine
or from any place
Heparin not absorbed due to its large size
Drugs which dissolve in water are rapidly absorbed from GIT

76. D. Phase I
Note: Ref: KDT
Phase I: Human pharmacology and safety
Goals:
Evaluation of safety and tolerability in humans using maximum tolerated dose (MTD)
as end point
Determination of qualitative and quantitative aspects of toxicity
Determination of dose limiting toxicity
Characterization of pharmacokinetic parameters (ADME)

77. B. Type E
Type E or end of treatment effects
Withdrawal effects like rebound hypertension after clonidine withdrawal
Type A or Augmented (Predictable) reaction
Mechanism based adverse reaction
Side effects, Secondary effects, toxic effects, Drug withdrawal reactions
Type B or Bizarre (Unpredictable) reactions:
Based on peculiarities of patient
Allergy
Idiosyncrasy
Pharmacogenetics
Type C or Chronic effects
Antipsychotics induced Tardive dyskinesia
Cushing syndrome due to chronic use of Prednisolone
Type D or delayed effects
Adverse effects occurring years after treating with a drug
Secondary cancers induced by anti-cancer drugs
Teratogenicity

78. D. Satisfy the priority health care needs


Note: Ref: National List of Essential Medicines (NLEM)
NLEM is one of the key instruments in balanced healthcare delivery system of a
country which includes accessible, affordable quality medicine at all the primary,
secondary and tertiary level of healthcare.
First NLEM of India was release in 1996 and subsequently revised in 2003, 2011 and
2015
DEFINITION:
As per the WHO, “Essential Medicines are those that satisfy the priority health care
needs of the population”.
The list is made with consideration to disease prevalence, efficacy, safety and
comparative cost-effectiveness of the medicines. Such medicines are intended to be
available in adequate amounts, in appropriate dosage forms and strengths with
assured quality. They should be available in such a way that an individual or
community can afford
79. C. Active drug to active metabolite
Note: Ref: KDT 7E/P 22
Active drug to active metabolite:
Examples-
Codeine to Morphine
Diazepam to Oxazepam
Imipramine to Des-Imipramine

80. D. Verapamil
Note: Ref: KDT
Isoprenaline, Lignocaine and Testosterone undergo high first pass metabolism so not
given orally
Drug undergo high first pass metabolism so given at high dose orally is Verapamil.
Autonomic Nervous System — Questions
81. Atropine reduces following secretions except
A. HCI secretion
B. Bile secretion
C. Intestinal secretions
D. Pancreatic secretions

82. Topical instillation atropine causes


A. Mydriasis
B. Abolition of light-reflux
C. Corneal anesthesia
D. All of the above

83. Anionic site of acetylcholine esterase enzyme can be occupied by all


except
A. Edrophonium
B. Pralidoxime
C. Tacrine
D. Parathion

84. Edrophonium binds to


A. Anionic site of acetylcholine esterase enzyme
B. Esteratic site of acetylcholine esterase enzyme
C. Both A and B
D. None of the above

85. Rate limiting step in AC h synthesis is


A. Formation of acetate inside Presynaptic membrane
B. Synthesis of acetyl cholinesterase enzyme
C. Acetyl Co A synthesis
D. Choline uptake inside Presynaptic membrane

86. Which of the following cranial nerves doesn’t have parasympathetic


nerves
A. X
B. IX
C. III
D. VI

87. When there is ganglionic blockade by ganglionic blockers, the effect seen
on heart rate is
A. Bradycardia
B. No effect is seen
C. Tachycardia
D. None of the above

88. Scopolamine hydrochloride is used for


A. Treating motion sickness
B. Preventing motion sickness
C. Both A and B
D. None of the above

89. Pralidoxime is ineffective as an antidote to carbaryl due to


A. The Esteric site of the enzyme is not free to provide attachment
B. Has weak anti-ChE activity
C. The anionic site of the enzyme is not free to provide attachment
D. Both A and C

90. Prazosin has a relatively high affinity at


A. A2A and a2B subtypes compared with a2C receptors
B. A2B and a2C subtypes compared with a2A receptors
C. A2A and a2C subtypes compared with a2B receptors
D. A2C and a2A subtypes compared with a2B receptors

91. Clonidine is used in to prevent diarrhea in diabetic patients with


autonomic neuropathy because
A. Stimulation of a1 receptors in the G1 tract may increase absorption of sodium
chloride and fluid and increase secretion of bicarbonate
B. Stimulation of a1 receptors in the G1 tract may increase absorption of sodium
chloride and fluid and inhibit secretion of bicarbonate
C. Stimulation of a2 receptors in the G1 tract may decrease absorption of sodium
chloride and fluid and increase secretion of bicarbonate
D. Inhibition of Imidazoline receptors in the GI tract may increase absorption of
sodium chloride and fluid and inhibit secretion of bicarbonate
81. b 82. d 83. d 84. a 85. d 86. d 87. c 88. b 89. b 90. b

91. a

92. Beta blockers with local anesthetic property is


A. Atenolol
B. Bisoprolol
C. Cartenolol
D. Acebutolol

93. The following drugs have alpha blocking property except


A. Chlorpromazine
B. Fluoxetine
C. Amitriptyline
D. Trazodone

94. The following are true about Dipivefrine except


A. Prodrug of adrenaline
B. Decreases hydraulic conductivity of trabecular filtering cells
C. Lowers intraocular tension by augmentation uveoscleral outflow
D. Reduces aqueous humor formation

95. Which of the following is not a use of Cholinomimetic Drugs


A. Glaucoma
B. Post-operative shivering
C. Myasthenia gravis
D. Post-operative atony

96. Following directly acting cholinergic drugs are natural alkaloids except
A. MUSCURAINE
B. Methacholine
C. Pilocarpine
D. Arecoline

97. Stimulation of M1 receptor leads to following action except


A. Increased cognitive function
B. Increased seizure activity
C. Increased in dopamine release and locomotion
D. Increase in depolarization of autonomic ganglia

98. Which of the following enzyme is more sensitive to organophosphates


A. Acetylcholine esterase
B. Angiotensin converting enzyme
C. Butyrylcholinesterase
D. All of the above

99. Which of the following anti-glaucoma drug cause bitter taste on topical
application on eyes
A. Latanosprost
B. Betaxolol
C. Apraclonidine
D. Dorzolamide

100. Which of the following has more retinoprotective effects


A. Timolol
B. Levobunolol
C. Betaxolol
D. None

101. Which of the following acts by decreasing aqueous humor


A. Latanoprost
B. Dipivefrine
C. Trovaprost
D. Betaxolol

102. Growth of eyelashes is caused by


A. Dipivefrine
B. Timolol
C. Ohysiostigime
D. Bimatoprost

103. Beta-blockers WHICH can achieve longest half-life among the following is
A. Nodalol
B. Bisoprolol
C. Betaxolol
D. Nebivolol

104. Drug found to mutagenic Ames test in animals is


A. Phenatolamine
B. Prozosin
C. Bunazosin
D. Phenoxybenzamine

105. Which of the following is false about Yohimbine


A. Found in the bark of the tree Pausinystalia Yohimbine and in Rouwolfia root
B. Yohimbine also antagonizes effect of 5-HT
C. In the past, it was used extensively to treat male sexual dysfunction
D. Yohimbine is a noncompetitive antagonist selective for a2 receptor

106. Induction of apoptosis smooth muscle cells in BPH is shown by


A. Doxazosin
B. Terazosin
C. Prozosin
D. Both A and B
92. d 93. b 94. b 95. b 96. b 97. c 98. c 99. d 100. c 101. d

102. 103. 104. 105. 106.


d d d d d

107. An old patient Ram Kishore having asthma and glaucoma is to receive a β
blocker. Regarding β blocking drugs, true is?
A. Timolol lacks the local anesthetic effects of propanolol
B. Metoprolol block β2 receptor selectively
C. Esmolol’s pharmacokinetics are compatible with chronic topical use
D. Nadolol lacks β2 blocking action

108. Adverse effects that limit the use of adrenoceptor blockers are all except
A. Bronchoconstriction from a blocking agents
B. Impaired blood sugar response with a blocker
C. Increased intraocular pressure with β blockers
D. Heart failure exacerbation from β blockers

109. Propanolol is useful in the treatment of all of the following EXCEPT


A. Partial atrioventricular block
B. Angina
C. Idiopathic hypertrophic subaortic cardiomyopathy
D. Familial tremor

110. Which of the following effects of adrenaline would be blocked by


phentolamine but NOT by propranolol
A. Contraction of the radial smooth muscle in the iris
B. Cardiac stimulation
C. Relaxation of bronchial smooth muscle
D. Relaxation of the uterus

111. All of the following are features of metoprolol in comparison to


propranolol EXCEPT
A. It is less likely to cause bradycardia
B. It is ineffective in suppressing muscle tremor
C. It is safer in diabetics
D. It is less likely to worsen Raynaud’s disease

112. Propranolol a non-selective beta blocker can be prescribed to decrease


anxiety associated with
A. Short term stressful situations
B. Chronic neurotic disorder
C. Schizophrenia
D. Endogenous depression

113. An old man Baba comes to you and is diagnosed to be having benign
hypertrophy of prostate. The drug which provides faster and greater
symptomatic relief to this patient will be
A. Desmopressin
B. Finasteride
C. Terazosin
D. Sildenafil

114. The property which makes betaxolol different from timolol is that
betaxolol
A. Is more efficacious in glaucoma
B. Produces less ocular side effects
C. Is shorter acting
D. Is a β1 selective blocker

115. A patient Ram kali presents with the symptoms suggestive of


pheochromocytoma. Her urine metanephrine and vinylmandellic acid levels
are above normal but normetanephrine level is less than normal. She later
presents to you in emergency with chest pain and severe headache. An ECG
indicates MI. Her blood pressure is 220/160 mm Hg and the heart rate is 160/
min. On examination she appears to be dehydrated also. The doctor
attending her gives her phentolamine i.v. Within 8-10 min, she goes in a
state of shock with her blood pressure being 36/0 mm Hg. Vasoconstrictors
are ineffective and she dies within 4 hours. Which of the following best
explains the exaggerated response to phentolamine in this patient?
A. Patient’s humor secreting almost pure adrenaline and no noradrenaline
B. Escape of autonomic nervous system control over blood pressure
C. Metastasis of the tumor to the vasometer center in medulla
D. Overdose of phentolamine because of the rarity of such cases in the emergency
107. a 108. c 109. a 110. a 111. a 112. a 113. c 114. d 115. a

116. A drug that blocks the uptake of dopamine and norepinephrine into
presynaptic nerve terminals and also blocks sodium channels in the axonal
membrane is
A. Ephedrine
B. Imipramine
C. Fluoxetine
D. Cocaine

117. Which of the following drugs will decrease heart rate in a patient with a
normal heart rate but will have little effect on heart rate in a cardiac
transplant recipient?
A. Phenylephrine
B. Adrenaline
C. Noradrenaline
D. Isoproterenol

118. B2 selective agonists are often effective in


A. Angina due to coronary insufficiency
B. Delayed labour
C. Asthma
D. All the above

119. The neurotransmitter agent that is normally released in the SA node of


the heart in response to increased blood pressure is
A. Dopamine
B. Adrenaline
C. Noradrenaline
D. Acetylcholine

120. A child, Ramu has swallowed the contents of 2 bottles of a nasal


decongestant whose primary ingredient is a adrenoceptor agonist drug. The
signs of an activation that may occur in this patient include
A. Tachycardia
B. Vasodilation
C. Dilatation of pupil
D. All of the above

121. The correct statement regarding adrenergic neuron blocking drugs is


A. Do not block any effect of injected adrenaline
B. Block the action of adrenaline on neuronal a2 adrenoceptor
C. Block both a and P adrenoceptor mediated effects of injected adrenaline
D. Do not block the effects of sympathetic nerve stimulation

122. A patient in shock comes to you in trauma ward. You examine him and
decide not to give Mm vasoconstrictors. Which is the type of shock your
patient is having?
A. Neurogenic shock
B. Hemorrhagic shock
C. Hypotension due to spinal anesthesia
D. Secondary shock

123. A drug is shown to activate dopaminergic D1 and D2 and adrenergic a and


b1, but not b2 receptors. Which of the following can be the drug being talked
about?
A. Dobutamine
B. Methoxamine
C. Phenylephrine
D. Dopamine

124. Which of the following drugs shows the phenomenon of vasomotor


reversal of Dale after administration of an adrenergic blocker?
A. Noradrenaline
B. Isoprenaline
C. Adrenaline
D. All of the above

125. The only non-catecholamine sympathomimetic drug out of the following


is
A. Adrenaline
B. Dopamine
C. Isoprenaline
D. Ephedrine

126. What is the most dangerous effect of belladonna in very young children?
A. Hyperthermia
B. Dehydration
C. Hallucination
D. Hypertension
116. 117. 118. c 119. 120. c 121. 122. 123. d 124. c 125. d
d a d a d

126.
a

127. You are being asked to give your expert opinion as a toxicologist
regarding an effective antidote for belladonna poisoning. Which of the
following agents would you suggest?
A. Neostigmine
B. Pilocarpine
C. Methacholine
D. Physostigmine

128. A drug ‘X’ belongs to the anticholinergic drug group. It is primarily used
in pre anesthetic medication and also during surgery. Which of the following
can be ‘X’?
A. Pipenzolate methyl bromide
B. Isopropamide
C. Dicyclomine
D. Glycopyrrolate

129. The difference between hyoscine and atropine is that hyoscine


A. Exerts more potent effects on the heart than on the eye
B. Is longer acting
C. Has weaker antimotion sickness activity
D. Exerts depressant effects on the CNS at relatively low doses

130. Intramuscular injection of atropine causes initial bradycardia. The reason


for this effect being seen is
A. Blockade of muscarinic auto receptor on vagal nerve endings
B. Stimulation of medullary vagal center
C. Stimulation of vagal ganglia
D. Blockade of M2 receptors of SA nodal cells

131. Mr. James has just been diagnosed with myasthenia gravis. You are his
physician and are considering different therapies for his disease.
Neostigmine and pyridostigmine may cause which one of the following?
A. Bronchodilation
B. Cycloplegia
C. Irreversible inhibition of acetylcholinesterase
D. Diarrhea

132. A 28-year-old woman has been treated with several autonomic drugs for
about a month. Which of the following signs would distinguish between an
overdose of a muscarinic blocker and a ganglionic blocker?
A. Postural hypotension
B. Blurred vision
C. Dry mouth, constipation
D. Mydriasis

133. Lallu, a farmer comes to you in the emergency in comatose state. Patient
had profuse sweating and lacrimation. Diarrhea and urination were
apparent. On examination pupil was constricted and BP of the farmer was
80/60 mm Hg. You make a diagnosis of anticholinesterase poisoning. You
decide to administer him atropine. All of the following actions will be
reversed by atropine EXCEPT
A. Hypotension
B. Central excitation
C. Bronchoconstriction
D. Muscle paralysis

134. Sunder Lal, 28-year-old farmer is found convulsing in the farm. Heartrate
is 100 min and blood pressure is 180/110 mm Hg. Diarrhea, sweating and
urination are apparent. Pupils are pin point. Drug poisoning is suspected.
Most probable cause is
A. Organophosphate poisoning
B. Acetaminophen overdose
C. Amphetamine toxicity
D. Atropine poisoning

135. A direct acting cholinomimetic that is lipid soluble and has been used in
the treatment of glaucoma is
A. Pilocarpine
B. Acetylcholine
C. Physostigmine
D. Neostigmine
127. d 128. d 129. d 130. a 131. d 132. a 133. d 134. a 135. a

136. Several children at a summer camp were hospitalized with symptoms


thought to be due to ingestion of food containing botulinum toxin. The
effects of botulinum toxin are likely to include
A. Bronchospasm
B. Diarrhea
C. Cycloplegia
D. Skeletal muscle spasms

137. Which of the following is the longest acting ocular beta blocker?
A. Timolol
B. Cartiolol
C. Metoprolol
D. Betaxolol

138. You are in the eye OPD and wish to use topical beta blocker in a patient.
The chosen drug by you should a have all the following properties EXCEPT
A. High lipophilicity
B. High ocular capture
C. Strong local anesthetic activity
D. Low systemic activity

139. Agonistic action at which of the following adrenergic receptors results in


the reduction of aqueous secretion?
A. B1 receptor
B. A2 receptor
C. B2 receptor
D. M2 receptor

140. Which of the following properties make pyridostigmine different from


neostigmine?
A. It is 1LTore potent
B. It produces less muscarinic side effects
C. It does not have any direct action on NM receptors
D. It is longer acting

141. Which of the following provides the best explanation for neostigmine
being preferred over physostigmine for treating myasthenia gravis
A. It has additional direct agonistic action on nicotinic receptors at the muscle end
plate
B. It is better absorbed orally
C. It has longer duration of action
D. It penetrates blood brain barrier

142. A patient Raj Kishore was given pilocarpine. All of the following can be
the features seen in him except
A. Cycloplegia
B. Sweating
C. Salivation
D. Miosis

143. All of the following effects are seen with cholinergic muscarinic receptor
stimulation except
A. Sweating
B. Brady cardia
C. Urination
D. Rise in blood pressure
136. c 137. d 138. c 139. b 140. d 141. a 142. a 143. d
Autonomic Nervous System —
Explanations
81. B. Bile secretion
Atropine decreases:
Secretion of acid, pepsin and mucus in the stomach
Bicarbonate secretion is also reduced
Intestinal and pancreatic secretions are not significantly reduced
Bile production is not under cholinergic control, so not affected
Thus, option B is correct. Ref: KDT, Topic: anticholinergics

82. D. All of the above


Topical instillation atropine causes:
Mydriasis
Abolition of light reflux
Cycloplegia lasting 7-10 days
Has a mild anesthetic action on the cornea
Thus, option D is correct. Ref: KDT, Topic: anticholinergics

83. D. Parathion
Parathion is an organophosphate and it binds to esteric site of acetylcholine esterase
enzyme
Edriphonium is a carbamate and tacrine is a Acridine derivative
They both attach only to the anionic site and do not form covalent bonds with the
enzyme
Pralidoxime:
Cholinesterase reactivators
Has a positively charged quaternary nitrogen: attaches to the anionic site of the
enzyme which remains unoccupied in the presence of organophosphate inhibitors
Organophosphates attach only to the esteratic site forming covalent bonds.
Thus, option D is correct. Ref: KDT, Topic: Cholinergics

84. A. Anionic site of acetylcholine esterase enzyme


Edrophenium and tacrine attach only to the anionic site and do not form convalent
bonds with the enzyme
Reactivation of edrophonium-inhibited enzyme occurs in < 10 min, and does not
involve hydrolysis of the inhibitor, but only its diffusion, so the action is brief
Organophosphates attach only to the esteratic site forming covalent bonds
Thus, option A is correct. Ref: KDT, Topic: cholinergics

85. D. Choline uptake inside Presynaptic membrane


Rate limiting step in AC h synthesis is Choline uptake inside Presynaptic membrane
Hemicholinium (HC3) blocks choline uptake and depletes Ach
Thus, option D is correct. Ref: KDT, Topic: cholinergics

86. D. VI
Parasympathetic
Cranial nerves: III, VII, IX, X
Sacral: S2, S3 and S4
Thus, option D is correct. Ref: KDT, Topic: cholinergics

87. C. Tachycardia
When ganglionic blockers, the query is whether it increases or decreases heart rate,
this depends on predominant tone on particular organ or tissue.
The predominant tone on heart is Parasympathomimmetic, so it will have decreased
heart rate.
But when ganglion is blocked, there will be increased heart rate (tachycardia)
Thus, option C is correct. Ref: KDT, Topic: ganglionic blockers

88. B. Preventing motion sickness


Scopalamine:
Effective in preventing motion sickness, probably by blocking neural pathways from
the vestibular apparatus in the inner ear to the emeric center in the brainstem
Drug should be given prophylactically because administration after symptoms have
set in is less effective, so not given in treatment of motion sickness
Thus, option B is correct. Ref: GG Topic: Anti-Cholinergics

89. B. Has weak anti-ChE activity


Pralidoxime:
The anionic site of the enzyme is not free to provide attachment to it
Contraindicated in carbamate poisoning as it has weak anti-ChE activity of its own
Thus, option B is correct. Ref: KDT, Topic: Cholinergics

90. B. A2B and a2C subtypes compared with a2A receptors


Prozosin:
Has a relatively high affinity at a2B and a2C subtypes compared with a2A receptors
Is much more portent in blocking a1 than a2 receptors
Has similar potencies at a1A, a1B and a1D subtypes
Is a relatively potent inhibitor of cyclic nucleotide phosphodiesterases
Have fourable effects on serum lipids in humans decreasing low-density lipoproteins
(LDL) and triglycerides while increasing concentrations of high-density lipoproteins
(HDL)
Thus, option B is correct. Ref: GG Topic : Alpha blockers

91. A. Stimulation of a1 receptors in the G1 tract may increase absorption of


sodium chloride and fluid and increase secretion of bicarbonate
Clonidine is used in to prevent diarrhea in diabetic patients with autonomic
neuropathy because
Stimulation of a2 receptors in the G1 tract may increase absorption of sodium
chloride and fluid and increase secretion of bicarbonate
Clonidine:
It is an Imidazoline group of drug
More potent agonists at a2 than a1 receptors
Action of clonidine are from its a2 receptors agonism and also by Imidazoline
receptors.
Intravenous infusion of clonidine causes an acute rise in blood pressure, apparently
because of activation of postsynaptic a2 receptors in vascular smooth muscle. This is
not seen when clonidine is given orally
A2 and imidazoline receptors mediate the hypotensive actions of centrally acting
drugs
Clonidine is used in postural hypotension
This is due to activation of postsynaptica 2 receptors in vascular smooth muscle
In severe autonomic failure, reflux sympathetic responses on standing are absent, so
postural hypotension is present
Since the central effect of clonidine on blood pressure is unimportant in these
patients, the drug can elevate blood pressure and improve the symptoms of postural
hypotension
Thus, option A is correct. Ref: GG Topic Sympathomimetics

92. D. Acebutolol
Beta blockers with Local anesthetic or membrane stabilizing activity
Pripranolol
Acebutolol
Carvedilol
Slight membrane-stabilizing effects
Pinodolol
Metoprolol
Betaxolol
Labetalol
Thus, option D is correct. Ref: GG Topic Sympatholytics

93. B. Fluoxetine
Drugs having alpha blocking property
Chlopromazine:
Is an antipsychotic
With blockade if alpha 1, Muscuranic, 5HT 2a and histamine 1 receptors
Amitriptyline
Is a Tricycli antidepressant
With blockade of alpha 1, muscuranic and histamine 1 receptors
Trazodone is atypical antidepressant
First atypical antidepressant
Less efficiently blocks 5-HT uptake
Less a prominent a 1 adrenergic and weak 5-HT2 antagonistic actions
Cause priapism and postural hypotension (due to alpha blockside)
Fluoxeteine
First SSRI to be introduced
And the longest acting having plasma t ½ of 2 days
Its active demethylated metabolite has half-life of 7-10 days
With no blockade of alpha, muscuranica and histamine receptors
Thus, option B is correct. Ref: KDT, Topic Sympathomimetics

94. B. Decreases hydraulic conductivity of trabecular filtering cells


Dipiverfrine:
It is a prodrug of Adrenaline
Penetrates cornea and is hydrolysed by the esterases into Adrenaline
The active adrenaline lower i.o.t. BY following mechanisms
By increasing uveoscleral outflow
B2 receptor mediated increase in hydraulic conductivity of trabecular filtering cells
By reducing aqueous formation (a1 + a2receptor mediated)
Adverse effects
Ocular burning
Thus, option B is correct. Ref: KDT, Topic: Glaucoma

95. B. Post-operative shivering


Uses of Cholinomimetic Drugs:
Glaucoma
Myasthenia gravis
Post-operative atony
Postoperative urinary retension
Postoperative decurarization
Alzheimer’s disease
Cobra bite
Belladonna poisoning
Thus, option B is correct. Ref: KDT, Topic: Cholinergics

96. B. Methacholine
Directly acting cholinergic drugs which are natural alkaloids:
MUSCURAINE
Pilocarpine
Methacholine
Arecoline
Directly acting cholinergic drugs which are synthetic choline esters (MCB)
Methacholine
Carbachol
Bethanecol
Thus, option B is correct. Topic : cholinergics

97. C. Increased in dopamine release and locomotion


Stimulation of M1 receptor leads to following action except:
Increased cognitive function, so used in Alzheimer’s disease
Increase seizure activity
Decrease in dopamine release and locomotion (remember the balance between
cholinergic and dopamine in Parkinson’ s disease)
Increase in depolarization of autonomic ganglia
Thus, option C is correct. Ref: GG Topic : cholinergics

98. C. Butyrylcholinesterase
Butyrylcholinesterase is more sensitive to organophosphates
Acetylcholine esterase is more sensitive to Physostigmine (carbamates)
Thus, option C is correct. Ref: KDT, Topic: cholinergics

99. D. Dorzolamide
Dorzolamide:
Topically useful carbonic anhydrase inhibitor side effects
Ocular stinging, burning, itching, corneal edema and bitter taste
Thus, option D is correct. Ref: KDT, Topic: Glaucoma

100. C. Betaxolol
Betaxolol lowers aqueous humor formation
Has retinoprotective effects
Blocks some Ca2+ channels and reduces Na+/Ca2+ influx in retina
Even timolol has retinoprotective effects but less compared to Betaxolol
Thus, option C is correct. Ref: KDT, Topic: Glaucoma

101. D. Betaxolol
Betaxolol:
Acts on beta receptors and decreases aqueous humor formation
Thus, option D is correct. Ref: KDT, Topic: Glaucoma

102. D. Bimatoprost
Prostaglandin analogues cause:
Iris pigmentation growth of eyelashes, macular edema (latanoprost)
Bimatoprost:
Cause growth of eyelashes
Can be used in hypotrichosis
Thus, option D is correct. Ref: GG Topic Glaucoma

103. D. Nebivolol
Shortest half-life beta-blockers is Esmolol (0.15 hours)
Longest half-life:
Nodalol (20-24 hours)
Nebivolol (11-30 hours)
Bisoprolol (9-12 hours)
Betaxolol 15 hours
Thus, option D is correct. Ref: GG Topic Glaucoma

104. D. Phenoxybenzamine
Phenoxybenzamine:
Is mutagenic in the Ames test
Repeated administration of this drug to experimental animals causes peritoneal
sarcomas and lung tremors
The clinical significance of these findings is not known
Thus, option D is correct. Ref: GG Topic Sympatholytics

105. D. Yohimbine is a noncompetitive antagonist selective for a2 receptor


Yohimbine:
Is a competitive antagonist that is selective for a2 receptors
Is a indolealkylamine alkaloid and is found in the bark of the tree Pausinystalia
Yohimbine and in Rouwolfia root
Structure resembles that of Reserpine
Readily enters the CNS, where it acts to increase blood pressure and heart rate
Also enhances motor activity and produces tremors
All these actions are opposite to those of clonidine, an a2 agonist
Yohimbine also antagonizes effect of 5-HT
In the past, it was used extensively to treat male sexual dysfunction
Thus, option D is correct. Ref: GG Topic Sympatholytics

106. D. Both A and B


Induction of apoptosis in prostate smooth muscle cells in BPH is shown by terazosin
and Doxazosin
This apoptosis may lessen the symptoms associated with chronic BPH by limiting cell
proliferation
The apoptotic effect of Terazosin and Doxazosin appears to be related to the
quinazoline moiety rather than a1 receptor antagonism
Tamsulosin, a non-quinazoline a1 receptor antagonist, does not produce apoptosis
Thus, option D is correct. Ref: GG Topic : Sympatholytics

107. A. Timolol lacks the local anesthetic effects of propanolol


Metoprolol is a cardio selective (β, selective) blocker and not P. selective.
Esmolol’s is due shortest acting B-blocker and is useful for acute treatment.
Nadolol is a non-selective B-blocker. It blocks both β1 as well as β2 receptors.
Propanolol possesses maximum local anesthetic activity whereas timolol lacks this
property.
Thus, option A is correct. Ref: KDT, Topic: Sympatholytics

108. C. Increased intraocular pressure with β blockers


Beta blockers are contraindicated in acute CHF because these can exacerbate the
heart failure. Bronchoconstriction and impairment of blood glucose response is seen
with P-blockers. These agents also decrease intraocular pressure and are used for the
treatment of glaucoma.
Thus, option D is correct. Ref: KDT, Topic: Sympatholytics

109. A. Partial atrioventricular block


Beta blockers are contra-indicated in heart block because these can convert partial AV
block to complete AV block. These agents are the drugs of choice for HOCM
Thus, option A is correct. Ref: KDT, Topic: Sympatholytics

110. A. Contraction of the radial smooth muscle in the iris


Smooth muscle of iris contains a receptor whereas heart, bronchus and uterus possess
P-adrenergic receptors.
Thus, option A is correct. Ref: KDT, Topic: Sympatholytics

111. A. It is less likely to cause bradycardia


Metoprolol is n cardio-selective P-blocker and thus is safer than non-selective blockers
(like Propanolol]) in diabetes, asthmatics and the patients of peripheral vascular
disease (e.g. Raynaud’s Disease). Due to lack of B2-blocking action, it cannot suppress
muscle tremors Potential to cause bradycardia is similar to non-selective B-blockers.
Drugs possessing ISA (like pindolol) are less likely to ca use bradycardia.
Thus, option A is correct. Ref: KDT Topic: Sympatholytics

112. A. Short term stressful situations


Propanolol is Used for controlling the performance anxiety, major manifestations of
which are due to increased sympathetic, activity (tachycardia, palpitations, etc.)
Thus, option A is correct. Ref: KDT, Topic: Sympatholytics

113. C. Terazosin
Selective a1 blockers provide faster and greater Symptomatic relief Io the patients of
BHP, but do not affect the disease progression. 5a reductase inhibitors like finasteride
slow the disease progression but the beneficial effects are delayed (tasks about 6
months)
Thus, option C is correct. Ref: KDT, Topic: Sympatholytics

114. D. Is a β1 selective blocker


Betaxolol is less likely to induce bronchospasm than timolol.
Thus, option D is correct. Ref: KDT, Topic: Sympatholytics

115. A. Patient’s humor secreting almost pure adrenaline and no


noradrenaline
Urinary excretion of metanephrine in excess suggests the secretion of mainly
adrenaline by the tumor (metanephrine is a metabolite of adrenaline and nor-
metanephrine is a degradation product of nor-adrenaline). Adrenaline acts on a-
receptors to cause increase in blood pressure (220/160 mmHg in this case) and on B1
receptor to cause tachycardia (160 min in this patient). However, it also possesses B2-
action that is responsible for vasodilation. This action is suppressed in the presence of
strong a-action. On giving a-blockers like phentolamine. This B2 action is unmasked
leading to profound hypotension (36/0 mm Hg in this patient) This phenomenon is
known as vasometer reversal of Dale.
Thus, option A is correct. Ref: KDT, Topic: Sympatholytics

116. D. Cocaine
It is a local anesthetic agent (acts by inhibiting Na+ channel in the axonal membrane)
that also possesses indirect sympathomimetic activity.
Thus, option D is correct. Ref: KDT, Topic: Sympathomimetics

117. A. Phenylephrine
Nor-adrenaline decreases the heart rate due to reflex stimulation of baroreceptors.
These reflexes are lost in a transplanted heart. However, by its B action, it produces a
tachycardia.
Phenylephrine is a selective al agonist, it has no direct effect on the heart but
produces bradycardia due to reflex stimulation of baro-receptors. These reflexes are
lost in a translated heart. Therefore, it produces no effect on heart.
Thus, option A is correct. Ref: KDT, Topic: Sympatholytics

118. C. Asthma
Beta-2 agonists are useful in premature uterine contractions to delay labour and not
for delayed labor. Beta blockers are used in the treatment of angina.
Thus, option C is correct. Ref: KDT, Topic: Sympathomimetics

119. D. Acetylcholine
Whenever blood pressure increases there is reflex stimulation of baroreceptors. These
release ACH and depresses the heart.
Thus, option D is correct. Ref: KDT, Topic: Cholinergics

120. C. Dilatation of pupil


Stimulation of a-receptors cause Mydriasis whereas tachycardia and vasodilation are
due to activation of p-adrenergic receptors.
Thus, option C is correct. Ref: KDT, Topic: Sympathomimetics

121. A. Do not block any effect of injected adrenaline


Adrenergic neuron blocking drugs like Guanethidine inhibits the release of nor-
adrenaline from the neuron. These drugs have no effect on a or preceptors Therefore,
these may inhibit the effect of sympathetic nerve stimulation but will have no effect
on exogamous adrenaline action.
Thus, option A is correct. Ref: KDT, Topic: Sympathomimetics

122. D. Secondary shock


Sympathomimetic drugs are indicated in all types of shock except secondary shock. In
this condition, there is reflex vasoconstriction. Alpha blockers are useful in this type of
shock.
Thus, option D is correct. Ref: KDT, Topic: Sympathomimetics

123. D. Dopamine
Dopamine has concentration dependent effects on various receptors. When infused at
a rate of less than 2 ug kg/min., it stimulates only dopamine receptors (resulting in
renal vasodilation). At 2-10 ug/kg/min. Infusion rate, it stimulates B2 receptors also
and at a rate greater than 1o ug/kg/min. It causes vasoconstriction due to stimulation
of a-receptors
Thus, option D is correct. Ref: KDT, Topic: Sympathomimetics

124. C. Adrenaline
Vasomotor reversal of dale is seen with adrenaline. when this drug is infused quickly,
initially there is rise in blood pressure (due to stimulation of a receptors) followed by
prolonged fall (B2, action). a-blocking drugs inhibit the initial rise and only fall in blood
pressure is recorded. This is known a vasomotor reversal. Nor-adrenaline has 11o P,
activity and Isoprenaline lacks a activity. Therefore, cannot demonstrate this
phenomenon.
Thus, option C is correct. Ref: KDT, Topic: Sympathomimetics

125. D. Ephedrine
Catecholamines are the drugs having dihydroxybenzene nucleus in its structure.
Adrenaline, Isoprenaline and dopamine contain this structure.
Thus, option D is correct. Ref: KDT, Topic: Sympathomimetics

126. A. Hyperthermia
Atropa belladonna contains anticholinergic principles like atropine and hyoscine.
Atropine is contra-indicated in children due to the risk of hyperthermia.
Thus, option A is correct. Ref: KDT, Topic: parasympatholytics

127. D. Physostigmine
Being tertiary amine, physostigmine can reverse CNS manifestations of belladonna
(source of atropine) poisoning. It is therefore, preferred as an antidote for this type of
poisoning.
Thus, option D is correct. Ref: KDT, Topic: cholinergics

128. D. Glycopyrrolate
Glycopyrrolate is used to reduce the secretions (to prevent reflex bronchospasm)
during anesthesia. It is mainly used as pre-anesthetic medication.
Thus, option D is correct. Ref: KDT, Topic: parasympatholytics

129. D. Exerts depressant effects on the CNS at relatively low doses


Hyoscine is a CNS depressant and call be used as truth scrum (to induce twilight
sleep). Atropine at low doses stimulates the brain whereas inhibits it at very high
concentration.
Thus, option D is correct. Ref: KDT, Topic: parasympatholytics

130. A. Blockade of muscarinic auto receptor on vagal nerve endings


Atropine is n non-selective antagonist of M1 M2 and M3, muscarinic receptors. M2,
cholinergic receptors are responsible for bradycardia and blockade of these receptors
can result in tachycardia. Atropine initially acts on presynaptic M, receptors (normally
decrease the release of ACH) and result in greater release of ACH which is responsible
for bradycardia. Later on, lockable of M2 receptors will lead to tachycardia.
Thus, option A is correct. Ref: KDT, Topic: parasympatholytics

131. D. Diarrhea
Neostigmine and pyridostigmine are reversible cholinesterase inhibitors that can
cause cholinergic adverse effects like diarrhea and increased secretions ·67. Ans. (d)
Blockade of muscarinic auto-receptors on vagal nerve endings.
Thus, option D is correct. Ref: KDT, Topic: Cholinergics

132. A. Postural hypotension


Postural hypotension is due to blockade of sympathetic system. Ganglion blockers
inhibit the transmission through both sympathetic as well as parasympathetic ganglia
whereas muscarinic blockers inhibit only parasympathetic activity.
Thus, option A is correct. Ref: KDT, Topic: parasympatholytics

133. D. Muscle paralysis


Atropine is a non-selective antagonist at muscarinic receptors. It can penetrate blood
brain barrier and reverse the muscarinic action in the CNS. It can also reverse
hypertension and bronchoconstriction caused due to stimulation of muscarinic
receptors. However, muscle paralysis is due to Nicotinic W.) action on which it has no
activity.
Thus, option D is correct. Ref: KDT, Topic: parasympatholytics

134. A. Organophosphate poisoning


All the symptoms are of anticholinesterase poisoning except raised blood pressure and
heart rate. At high doses, ACH can stimulate N. and NM receptors ·These symptoms
may occur due to the nicotinic actions of ACH.
Thus, option A is correct. Ref: KDT, Topic: cholinergics
135. A. Pilocarpine
Pilocarpine is a directly acting and physostigmine is an indirectly acting
cholinomimetic useful for glaucoma.
Thus, option A is correct. Ref: KDT, Topic: cholinergics

136. C. Cycloplegia
Botulinum toxin interferes with the release of ACH and thus acts as a Para
sympatholytic agent. Bronchospasm and diarrhea are the symptoms of muscarinic
stimulation whereas muscle spasms may be seen on nicotinic stimulation. Cholinergic
drugs cause cyclospasm whereas anticholinergics result in Cycloplegia.
Thus, option C is correct. Ref: KDT, Topic: anticholinergics

137. D. Betaxolol
Betaxolol is a cardio selective p-block useful in glaucoma. It is longer acting than
Timolol. Another non-selective P-blocker used topically for the treatment of glaucoma
is levobunolol.
Thus, option D is correct. Ref: KDT, Topic: sympatholytics

138. C. Strong local anesthetic activity


Timolol and betaxolol are the preferred P-blockers for the treatment of glaucoma
because they lack local anesthetic activity. Drugs possessing this property increase the
risk of corneal ulcers.
Thus, option C is correct. Ref: KDT, Topic: sympatholytics

139. B. A2 receptor
Stimulation of a2 receptors located on ciliary epithelium reduces secretion of aqueous
humor.
Thus, option B is correct. Ref: KDT, Topic: glaucoma

140. D. It is longer acting


Pyridostigmine acts for 3-6 hours as compared to 0. 5-2 hours’ duration of action of
neostigmine.
It is less potent than neostigmine.
Rest of the properties are similar to neostigmine.
Thus, option D is correct. Ref: KDT, Topic: glaucoma

141. A. It has additional direct agonistic action on nicotinic receptors at the


muscle end plate
Neostigmine is a quaternary ammonium derivative and is lipid insoluble. Its absorption
from GIT and penetration in the brain and cornea is much less than physostigmine. It
produces additional action on NM receptors.
Thus, option A is correct. Ref: KDT, Topic: cholinergics

142. A. Cycloplegia
Pilocarpine is, directly acting cholinergic drug. It causes miosis and cyclospasm (not
Cycloplegia). It can increase all secretions of the body.
Thus, option A is correct. Ref: KDT, Topic: cholinergics

143. D. Rise in blood pressure


Stimulation of muscarinic (M3) receptors decreases BP whereas parasympathetic
system stimulation has no effect on BP because blood vessels contain M3 receptors but
no parasympathetic supply.
Thus, option D is correct. Ref: KDT, Topic: Cholinergics
Autacoids — Questions
144. The primary endogenous substrate for nitric oxide synthase (NOS) is
A. Citrulline
B. Heme
C. Methionine
D. Arginine

145. Which of the following is used to treat severe pulmonary hypertension?


A. Angiotensin 1
B. Bosentan
C. Omapatrilat
D. Endothelin

146. A drug X is useful in treatment of rheumatoid arthritis. It is available only


in parential formulation and its mechanism of action is antagonism of tumor
necrosis factor. Which of the following can be X?
A. Etanercept
B. Cyclosporine
C. Penicillamine
D. Phenylbutazone

147. A drug that is effective for rheumatoid arthritis but is not appropriate for
osteoarthritis is
A. Acetaminophen
B. Keterolac
C. Rofecoxib
D. Infliximab

148. Rasburicase is a newer drug used in gout. It acts by


A. Decreasing urate synthesis
B. Decreasing intestinal absorption of uric acid
C. Increasing renal excretion of uric acid
D. Increasing urate oxidation

149. Allopurinol is useful in all of the following conditions EXCEPT


A. Acute gouty arthritis
B. Cancer chemotherapy induced hyperuricemia
C. Hydrochlorothiazide induced by hyperuricemia
D. Kala-azar

150. Drug of choice for acute gout is


A. Colchicine
B. Allopurinol
C. Dexamethasone
D. Indomethacin

151. ll of the following drugs can produce hyperuricemia EXCEPT


A. Ethambutol
B. Sulfinpyrazone
C. Pyrazinamide
D. Hydrochlorothiazide

152. A new born was diagnosed as having a congenital abnormality that


resulted in transposition of great vessels. While preparing the infant for
surgery, the medical team needed to keep the ducts arteriosus open. They
did this by infusing
A. Alprostadil
B. Cortisol
C. Indomethacin
D. Tacrolimus

153. Individuals with alcoholic cirrhosis of liver may develop severe


hepatoxicity after doses of acetaminophen that are not toxic to individuals
with normal liver function. This increased sensitivity to acetaminophen’s
toxicity is due to
A. Decrease availability of acetaldehyde dehydrogenase
B. Decreased activity of Cytochrome P450 enzymes
C. Increased liver blood flow
D. Decrease hepatocellular store of glutathione
144. 145. 146. 147. 148. 149. 150. 151. b 152. a 153. d
d b a d d a d

154. A college student is brought to emergency after taking an overdose of a


non-prescription drug. The patient is confused and lethargic. He has been
hyperventilating and dehydrated. Arterial blood gas analysis demonstrates
metabolic acidosis. In the management of this patient, which is NOT likely to
be of any therapeutic value
A. Treatment with acetyl cysteine
B. Alkalization of urine
C. Correction of metabolic acidosis
D. Gastric lavage

155. A 3-year-old child presented to OPD with symptoms of influenza. Aspirin


is contraindicated in this patient because of increased risk of
A. Reye’s syndrome
B. Gastric bleeding
C. Thrombocytopenia
D. Fanconi syndrome

156. A patient comes to you complaining that whenever he takes aspirin for
headache, he develops severe shortness of breath. Which of the following
may be partly responsible for this effect?
A. Prostaglandin E
B. Thromboxane A2
C. Prostacyclin
D. Leukotrienes

157. Mechanism of action of aspirin in MI is


A. Inhibition of cyclooxygenase
B. Decreased serum lipids
C. Thrombolytic action
D. Inhibition of thromboxane synthesis

158. Aspirin is used in the prophylaxis of myocardial infarction because it


results in
A. Inhibition of thromboxane synthetase
B. Decreased serum lipids
C. Coronary steal phenomenon
D. Inhibition of cyclooxygenase

159. Rofecoxib as compared to indomethacin is


A. Likely to be more effective in rheumatoid arthritis
B. Not likely to produce renal complications
C. Less likely to cause gastric ulcer and their complications
D. All of the above

160. A truck driver presented to the hospital with a minor soft tissue injury.
Which of the following NSAID should not be prescribed to him?
A. Celecoxib
B. Naproxen
C. Diclofenac sodium
D. Indomethacin

161. The plasma half-life is aspirin


A. Is independent of dose
B. Is shorter for anti-inflammatory doses compared to that for analgesic dose
C. Can be increased by alkalinizing the urine
D. Is longer for anti-inflammatory doses compared to that for analgesic dose

162. All of the following actions of aspirin are mediated by inhibition of


prostaglandin synthesis EXCEPT
A. Hyperventilation
B. Analgesia
C. Closure of patent ductus arteriosus
D. Bleeding tendency

163. All of the following effects are produced by inhibitors of prostaglandin


synthesis EXCEPT
A. Prolongation of bleeding time
B. Prolongation of labour
C. Gastric mucosal damage
D. Prolongation of prothrombin time

164. True statement about cyclooxygenase-2 is


A. It is not inhibited by indomethacin
B. It generates cytoprotective prostaglandins in gastric mucosa
C. It is found only in fetal tissues
D. It is inducible
154. 155. 156. 157. 158. 159. c 160. 161. d 162. a 163. d
a a d d d d

164.
d

165. Aspirin in low doses produces long lasting inhibition of platelet


cyclooxygenase because
A. Platelet contain low quantity of COX
B. Platelets bind aspirin with high affinity
C. Platelet COX is inducible
D. Platelets cannot synthesize fresh COX molecules

166. Prostaglandin E2 analogs can be used for all of the following conditions
EXCEPT
A. Treatment of bronchial asthma
B. Cervical priming
C. Treatment of patent ductus arteriosus
D. Treatment of NSA1D induced peptic ulcer

167. Which of the following patient characteristics is a possible reason for the
use of celecoxib in the treatment of arthritis?
A. History of peptic ulcer disease
B. History of severe rash after treatment with a sulfonamide antibiotic
C. History of gout
D. History of type 2 DM

168. The chief advantage of ketorolac over aspirin is that the former
A. Is available in a parenteral formulation that can be used intramuscularly or
intravenously.
B. Can be combined more safely with an opioid such as codeine,
C. Does not prolong bleeding time.
D. Is less likely to cause acute renal failure in patients with pre-existing

169. Which of the following drugs reduces the activity of phospholipase A2


A. Alprostadil
B. Prednisolone
C. Aspirin
D. Ibuprofen

170. Which of the following is a component of slow reacting substance of


anaphylaxis (SRS-A)?
A. LTB4
B. Misoprostol
C. Prostacyclin
D. LTC4

171. Which of the following drugs inhibit platelet cyclooxygenase reversibly


A. Ibuprofen
B. Alprostadil
C. Aspirin
D. Prednisolone

172. Choose the CORRECT statement about sumatriptan


A. It activates 5-HT1A receptors
B. It does not carry risk of precipitating coronary vasospasm
C. It is combined with ergotamine for the treatment of severe migraine
D. It tends to suppress both pain and vomiting in migraine

173. Ergot alkaloid commonly used to prevent post-partum hemorrhage is


A. Ergotamine
B. Dihydroergotamine
C. Dihydroergotoxine
D. Methyl Ergometrine

174. Dihydroergotamine differs from ergotamine in the following respect


A. It is more potent an adrenergic blocker and less potent vasoconstrictor
B. It is a more potent oxytocic
C. It has antipeptic property
D. It has high oral bioavailability

175. Which of the following is most useful for reversing severe ergot induced
vasospasm?
A. Ergotamine
B. Nitroprusside
C. Methysergide
D. Phenoxybenzamine

176. Two antihistaminics Terfenadine and astemizole were withdrawn from the
market following the occurrence of cardiac arrhythmias when they were
present in high levels in the blood. These effects were explained by the fact.
A. Treatment of these patients with erythromycin, a macrolide antibiotic
B. Use of these drugs by addicts
C. Genetic predisposition to metabolize succinylcholine slowly
D. Concurrent treatment with phenobarbital
165. 166. c 167. 168. 169. 170. 171. 172. a 173. d 174. a
d a a b d a

175. 176.
b a

177. H, antihistaminics afford benefit in a number of conditions. All of the


following conditions are benefited by antagonism of histamine EXCEPT
A. Common cold
B. Dermographism
C. Insect bite
D. Seasonal hay fever

178. Which of the following can reverse one or more smooth muscle effects of
circulating histamine in humans
A. Granisetron
B. Ranitidine
C. Sumatriptan
D. Adrenaline

179. Erenumab, a CGRP receptor antagonist has been approved for


A. ITP
B. Migraine
C. Atopic dermatitis
D. Plaque Psoriasis

180. Dose of the following anticancer need not be reduced while on Allopurinol
therapy
A. Thioguanine
B. 6-Mercaptopurine
C. Azathioprine
D. All of the above

181. Following is/are orphan uses for Allopurinol


A. Lesch-Nyhan syndrome
B. Chagas disease
C. Ex vivo preservation of cadaveric kidneys prior to transplantation
D. All of the above

182. Tocilizumab is used in


A. Multicentric Castleman’s disease
B. Systemic Sclerosis
C. Chronic gout
D. Rheumatoid Arthritis
183. A patient was on Rasburicase for hyperuricaemia, he had laboratory
values of uric acis inspite of having symptoms of hyperuricaemia. The
reason for this is
A. May be because of excess allantoin in the body
B. Rasburicase causes enzymatic degradation of the uric acid in blood samples
C. Both are correct
D. Both are false

184. Which of the following NSAID has property of inhibiting of


Metalloproteinase activity
A. Naproxen
B. Nabumetone
C. Diclofenac
D. Nimesulide

185. Which of the following is incorrect about mechanism of action of


Colchicine in acute gout
A. Inhibits the synthesis of uric acid
B. Binds to tubulin and causes its depolymerisation
C. Colchicine blocks cell division by binding to mitotic spindles
D. Colchicine inhibits the synthesis and release of the leukotrienes

186. Drug that can be used to prevent platelet aggregation and damage
during hemodialysis or cardiopulmonary bypass is
A. Alprostadil
B. Epoprostenol
C. Latanoprost
D. Misoprostol

187. Lorcarserin is a
A. 5HT1B/1D Receptor Agonists
B. 5HT2B Receptor Agonists
C. 5HT1A Receptor Agonists
D. 5HT2C Receptor Agonists

188. Which of the following antihistamine has both H1 and H2 antagonist


activity
A. Fexofenadine
B. Epinastine
C. Azelastine
D. Emedastine

189. Certolizuman is a
A. Fab fragment toward IL-1
B. Fab fragment toward IL-6
C. Fab fragment toward TNF-a
D. Fab fragment toward Nerve growth factor
177. 178. 179. 180. 181. 182. 183. 184. d 185. a 186.b
a d b a d d b

187. 188. 189. c


d b

190. Which of the following stabilizes lysosomes and inhibit LOX


A. Diclodenac
B. Ketoprofen
C. Naproxen
D. Puroxicam

191. The most common side effect of sumatriptan nasal spray is


A. Bitter taste
B. Nasal irritation
C. Rhinitis
D. Excessive sneezing

192. Which of the following is a non-selective COX INHIBITOR


A. Celecoxib
B. Parecoxib
C. Meloxicam
D. Piroxicam

193. Which of the following statement is false about leflunomide


A. Used in treatment of RA
B. Reduced purine synthesis in actively dividing cells
C. Act by inhibiting dihydroorotate dehydrogenase
D. Inhibits proliferation of T and B cells

194. The following are true about PAF except


A. It is potent peptic ulcerogen
B. Released only by platelets during inflammation
C. Produced by fetus in third trimester
D. It increases vascular permeability and has chemotactic property

195. Highest COX 2 Selective inhibitor among the following is


A. Etoricixib
B. Valdecoxib
C. Rofecoxib
D. Celecoxib

196. Aprotinin is a
A. CGRP receptor antagonist
B. Kallikrein antagonist
C. Serotonin 1F agonist
D. Bradykinin receptor agonist

197. Following are Preferential COX-2 inhibitors except


A. Etodolac
B. Nimesulide
C. Meloxicam
D. Ketorolac

198. Which of the first generation antihistamine has mild sedative action
A. Triprolidine
B. Diphenhydramine
C. Dimenhydrinate
D. Meclizine

199. Following Prostaglandin is given by inhalational route


A. Epoprostenol
B. Misoprostol
C. Alprostadil
D. Treprostonil

200. Drug which acts by inhibiting influx of Ca2+ from endolymph into the
vestobular sensory cells is
A. Doxylamine
B. Cinnarizine
C. Meclizine
D. Promethazine

201. Prokinetic drug which is mainly metabolized by Flavin monooxygenases is


A. Cisapride
B. Domperidone
C. Itopride
D. Metoclopramide

202. True statement about Prokinetic action of Domperidone and


Metacloprapimde is
A. Metacloprapimde prokinetic action is not attenuated by atropine
B. Domperidone prokinetic action is attenuated by atropine
C. Domperidone prokinetic action is not attenuated by atropine
D. None of the above

203. The prokinetic activity of metoclopramide is mainly because of


A. Central D2 antagonism
B. Peripheral D2 antagonism
C. 5-HT4 agonism
D. Both B and C
204. All 5-HT receptors are heptahelical serpentine receptors EXCEPT
A. 5-HT1
b. 5-HT3
C. 5-HT2
d. 5-HT4

190. 191. 192. 193. 194. 195. 196. 197. d 198. a 199. d
b a d b b a b

200. 201. c 202. c 203. 204.


b d b

205. True statement about fexofenadine is


A. It does not block cardiac K+ channels
B. It undergoes high first pass metabolism in liver
C. Terfenadine is an active metabolite of this drug
D. It has high affinity for central receptors

206. Ketoconazole should not be given to a patient being treated with


astemizole because
A. Ketoconazole induces the metabolism of astemizole
B. Astemizole inhibits the metabolism of ketoconazole
C. Astemizole antagonizes the antifungal action of ketoconazole
D. Dangerous ventricular arrhythmias can occur

207. All of the following statements are TRUE about second generation
antihistaminic agents EXCEPT
A. These do not impair psychomotor performance
B. The lack anticholinergic actions
C. These may process additional antiallergic mechanisms
D. These possess high anti-motion sickness activity

208. Mr. Surya Kant was prescribed a first generation antihistaminic drug. He
should be advised to avoid
A. Consuming processed cheese
B. Strenuous physical exertion
C. Driving motor vehicles
D. All of the above

209. All of the following antihistaminic agents lack anticholinergic property


except
A. Astemizole
B. Levo-cetrizine
C. Loratidine
D. Promethazine
210. Which of the following drugs can cause hypotension by release of
histamine from mast cells?
A. Aspirin
B. Morphine
C. Procaine
D. Sulfadiazine

211. All of the following actions of histamine are mediated through H1


receptors EXCEPT
A. Release of EDRP from vascular endothelium resulting in vasodilation
B. Bronchoconstriction
C. Release of catecholamines from adrenal medulla
D. Direct action on vascular smooth muscle causing vasodilation

212. A drug ‘X’ is an adrenergic blocker but paradoxically produces


vasoconstriction ‘X’ is
A. Phenoxybenzamine
B. Prazosin
C. Tolazoline
D. Ergotamine
205. a 206. d 207. d 208. c 209. d 210. b 211. d 212. d
Autacoids — Explanations
144. D. Arginine
NOS normally metabolizes arginine to NO and Citrulline.
Arginine → Citrulline + NO
NO is a potent vasodilator that acts by increasing cGMP.
Thus, option D is correct. Ref: KDT, Topic: Autacoids

145. B. Bosentan
Bosentan is a non-specific antagonist of endothelin receptors (both ETA and ETB). It is
useful for the treatment of primary pulmonary hypertension
Omapatrilat is a vasopeptidase inhibitor used for the treatment of CHF.
Thus, option B is correct. Ref: Katzung 10E/P 287.

146. A. Etanercept
Infliximab and Etanercept are TNF-antagonists useful for the treatment of rheumatoid
arthritis. These are administered by parenteral route. TNF-a antagonists can cause
reactivation of latent tuberculosis.
Thus, option A is correct. Ref: KDT, Topic: rheumatoid arthritis

147. D. Infliximab
It is a TNF-a antagonist that plays no role in osteoarthritis.
Thus, option D is correct. Ref: KDT, Topic: of rheumatoid arthritis

148. D. Increasing urate oxidation


Rasburicase and pegloticase are recombinant urate oxidase enzymes that convert
insoluble uric acid to soluble allantoin.
Thus, option D is correct. Ref: Goodman Gilman 11E/P 710.

149. A. Acute gouty arthritis


Allopurinol is contra-indicated in acute gout. Rarely it can be used in kala agar
patients.
Thus, option A is correct. Ref: KDT, Topic: Gout

150. D. Indomethacin
NSAIDs except aspirin are the agents of choice for the treatment of acute gout.
Thus, option D is correct. Ref: KDT, Topic: Gout

151. B. Sulfinpyrazone
This drug is uricosuric agent and is used in the treatment of hyperuricemia.
Thus, option B is correct. Ref: KDT, Topic: Gout

152. A. Alprostadil
Prostaglandins (like alprostadil) are used to keep ductus arteriosus patent whereas
aspirin or indomethacin are used for the treatment (closure) of FDA.
Thus, option A is correct. Ref: KDT, Topic: Prostaglandins

153. D. Decrease hepatocellular store of glutathione


Chronic alcohol intake results in liver dysfunction and decreased glutathione stores.
Deficiency of this compound increases the toxicity of acetaminophen cause NAPQ, a
metabolite of this compound, can now react easily with sulfhydryl groups of the
biomolecules. Alcohol also induces the enzymes responsible for conversion of
acetaminophen to NAPA.
Thus, option D is correct. Ref: KDT, Topic: NSAIDS

154. A. Treatment with acetyl cysteine


This patient is a case of salicylate poisoning. Gastric lavage is done to remove
unabsorbed poison. Metabolic acidosis must be corrected by sodium bicarbonate.
Alkalization of urine will increase the excretion of this weakly acidic drug. Treatment
with acetyl cysteine has no role in salicylate poisoning. It is the antidote of choice for
paracetamol poisoning.
Thus, option A is correct. Ref: KDT, Topic: NSAIDS

155. A. Reye’s syndrome


Aspirin can increase the risk of Reye’s syndrome, if used in children with viral diseases.
Thus, option A is correct. Ref: KDT, Topic: NSAIDS

156. D. Leukotrienes
Aspirin inhibits COX enzyme and results in the diversion of AA pathway towards LT
synthesis. As LTs are powerful bronchoconstrictor agents, these may result in the
shortness of breath in patients who are susceptible. Aspirin acetylated COX starts
producing lipoxins (known as aspirin triggered lipoxins) that also have
bronchoconstrictor properties.
Thus, option D is correct. Ref: KDT, Topic: NSAIDS

157. D. Inhibition of thromboxane synthesis


See the difference from the above question. Here in the question, it is written
thromboxane synthesis and not synthetase. Aspirin inhibits TX synthesis via inhibiting
cox and thus this is more specific explanation of its antiplatelet action.
Thus, option D is correct. Ref: KDT, Topic: NSAIDS

158. D. Inhibition of cyclooxygenase


Aspirin acts as an antiplatelet drug in MI. It inhibits cox enzyme and thus reduces the
synthesis of TXA2. However, it has no effect on the enzyme, thromboxane synthetase.
Therefore, answer here is the inhibition of cox.
Thus, option D is correct. Ref: KDT, Topic: NSAIDS

159. C. Less likely to cause gastric ulcer and their complications


Selective COX-2 inhibitors are less likely to cause GI complications like PUD. However,
as COX-2 is also present constitutively in the kidney, chances of renal complications
are similar. These drugs have similar or less efficacy than non-selective COX inhibitors.
Thus, option C is correct. Ref: KDT, Topic: NSAIDS

160. D. Indomethacin
Sedation caused by indomethacin can interfere with driving.
Thus, option D is correct. Ref: KDT, Topic: NSAIDS
161. D. Is longer for anti-inflammatory doses compared to that for analgesic
dose
Aspirin and other salicylates show saturation kinetics (zero order) and thus, their half-
life is not constant. Clearance at high doses (anti-inflammatory) is lesser than at low
(analgesic) doses. Thus t1/2 is more for anti-inflammatory dose than for analgesic dose.
Thus, option D is correct. Ref: KDT, Topic: NSAIDS

162. A. Hyperventilation
All other actions are mediated via inhibition of PG synthesis whereas hyperventilation
caused by aspirin is due to respiratory stimulation.
Thus, option A is correct. Ref: KDT, Topic: NSAIDS

163. D. Prolongation of prothrombin time


Prothrombin time is increased by the drugs interfering with the coagulation pathway
(e.g. warfarin). As cox has no role in coagulation, inhibitors of PG synthesis do not
prolong PT. However, bleeding time (BT) is prolonged by the drugs interfering with
platelet function. Aspirin increases BT by acting as an antiplatelet agent.
Thus, option D is correct. Ref: KDT, Topic: NSAIDS

164. D. It is inducible
COX-1 is a house-keeping enzyme that is responsible for the generation of gastro
protective PGs. COX-2 is an inducible enzyme whose production is markedly increased
at inflammatory sites. Indomethacin is a non-selective inhibitor of both isoforms of
COX.
Thus, option D is correct. Ref: KDT, Topic: NSAIDS

165. D. Platelets cannot synthesize fresh COX molecules


Aspirin in low doses irreversibly inhibits COX enzyme in platelets and the endothelium.
Platelets lack nucleus and thus cannot regenerate COX whereas endothelium can
synthesize fresh enzyme. Net effect of this process is the inhibition of platelet cox and
thus, TXA2 synthesis. Low TXA2 level results in the anti-aggregation of platelets.
Thus, option D is correct. Ref: KDT, Topic: NSAIDS

166. C. Treatment of patent ductus arteriosus


PGE2 analogues are used to maintain the patency of ductus arteriosus whereas
aspirin or indomethacin are used for the treatment of PDA.
PGE2 is a bronchodilator and can be used to treat bronchial asthma via inhalational
route.
Cervical priming and NSAID induced PUD are other indications of this agent.
Thus, option C is correct. Ref: KDT, Topic: NSAIDS

167. A. History of peptic ulcer disease


Non-selective COX inhibitors on long term use are associated with peptic ulcer
disease. These should be avoided in the patients having history of PUD. Selective COX-
2 inhibitors like celecoxib are safe in this regard.
Thus, option A is correct. Ref: KDT, Topic: NSAIDS

168. A. Is available in a parenteral formulation that can be used


intramuscularly or intravenously.
Ketorolac is the only NSAID that can be administered intravenously.
Thus, option A is correct. Ref: KDT, Topic: NSAIDS

169. B. Prednisolone
It is a powerful anti-inflammatory agent. It acts by inhibiting the synthesis of PG, TX
and LT via the inhibition of phospholipase A, enzyme. However, its most important
mechanism of anti-inflammatory action is the inhibition of chemotaxis.
Thus, option B is correct. Ref: KDT, Topic: NSAIDS

170. D. LTC4
LTC4, D4 and B4 are known as slow reacting substance of anaphylaxis as these can
cause bronchoconstriction.
Thus, option D is correct. Ref: KDT, Topic: NSAIDS

171. A. Ibuprofen
Alprostadil (PGE,) and prednisolone do not inhibit cox enzyme whereas aspirin is an
irreversible inhibitor of this enzyme.
Thus, option A is correct. Ref: KDT, Topic: NSAIDS

172. D. It tends to suppress both pain and vomiting in migraine


Sumatriptan decreases whereas ergotamine aggravates the vomiting of migraine.
These two agents should not be administered concomitantly due to the risk of severe
coronary spasm. Triptans act as agonists at 5HT18/1D receptors.
Thus, option D is correct. Ref: KDT, Topic: migraine

173. D. Methyl Ergometrine


Methylergometrine (methergin) is administered during delivery of anterior shoulder to
prevent postpartum hemorrhage.
Thus, option D is correct. Ref: KDT, Topic: migraine

174. A. It is more potent an adrenergic blocker and less potent


vasoconstrictor
Hydrogenation of ergot alkaloids decrease their vasoconstrictor action and increase
the a-blocking activity.
Thus, option A is correct. Ref: KDT, Topic: migraine

175. B. Nitroprusside
This agent is a mixed dilator and is highly effective agent against ergot induced
vasospasm. It is also the drug of choice for most of the hypertensive emergencies.
Thus, option B is correct. Ref: Goodman & Gilman 11/e p311. Topic: migraine

176. A. Treatment of these patients with erythromycin, a macrolide antibiotic


Astemizole and Terfenadine are metabolized by microsomal enzymes. Erythromycin
inhibits these enzymes and raises the plasma concentration of these drugs. At high
levels, these agents block cardiac K+ channels leading to arrhythmias.
Thus, option A is correct. Ref: KDT, Topic: Antihistaminic

177. A. Common cold


In this condition, excessive secretions (like rhinorrhea) are due to cholinergic over
activity. First generation antihistaminics possess anticholinergic property and therefore
are effective for common cold. On the other hand, second generation agents (lacking
anticholinergic actions) are ineffective.
Thus, option A is correct. Ref: KDT, Topic: Antihistaminic

178. D. Adrenaline
Adrenaline is a physiological antagonist of histamine. It reverses bronchoconstriction
caused by histamine. Ranitidine is an n antagonist that decreases gastric acid
secretion but has no effect on smooth muscles.
Thus, option D is correct. Ref: KDT, Topic: Antihistaminic p129

179. B. Migraine
Erenumab:
MOA:
Mab against calcitonin gene-related peptide
USE:
For the preventive treatment of migraine in adults
Available as SC injection

180. A. Thioguanine
6-mercaptopurine and azathioprine
Allopurinol inhibits the degradation of 6-mercaptopurine and azathioprine
so the doses of these drugs need to be reduced
Thioguanine:
Dose need not to be reduced
Because it follows a different metabolic path and undergoes S-methylation
Thus, option A is correct. Ref: KDT, Topic: Gout

181. D. All of the above


Orphan uses for Allopurinol
Lesch-Nyhan syndrome
Lowering the high plasma concentrations of uric acid
No effect on disease progression
Chagas Disease
Ex vivo presentation of cadaveric kidneys prior to transplantation
Thus, option D is correct. Topic: Gout; Ref: GG

182. D. Rheumatoid Arthritis


Tocilizumab:
Mab against IL-64
Is FDA-approved for treatment of rheumatoid arthritis and systemic juvenile
idiopathic arthritis
Thus, option B is correct. Topic: Rheumatoid Arthritis; Ref: GG

183. B. Rasburicase causes enzymatic degradation of the uric acid in blood


samples
Rasburicase causes enzymatic degradation of the uric acid in blood samples
So when analyzing uric acid levels, special handling is required to prevent spuriously
low values for plasma uric acid in patients receiving the drug.
Thus, option B is correct. Ref: GG, Topic : Gout
184. D. Nimesulide
Nimesulide:
Weak PG synthesis inhibitor
Relative COX-2 inhibitor
Other mechanism like:
Inhibition of neutrophil activation
Reduced generation of superoxide
Inhibition of PAF synthesis & TNF a release,
Free radical scavenging
Inhibition of metalloproteinase activity
Possibly activation of glucocorticoids receptors
Thus, option D is correct. Ref: KDT, Topic: NSAIDS

185. A. Inhibits the synthesis of uric acid


Colchicine:
Mechanism of Action:
Binds to tubulin, a microtubular protein, causing its depolymerisation
This disrupts cellular functions, such as the mobility of granulocytes, thus
decreasing their migration into the affected area.
Colchicine blocks cell division by binding to mitotic spindles
Colchicine inhibits the synthesis and release of the leukotrienes
Do not Inhibits the synthesis or promote the excretion of uric acid
Thus, option A is correct. Ref: KDT, Topic: Gout

186. B. Epoprostenol
PGI2 (Epoprostenol):
Can be used to prevent platelet aggregation and damage during hemodialysis or
cardiopulmonary bypass
It also improves harvest of platelets for transfusion
Used in primary pulmonary hypertension
Thus, option B is correct. Ref: KDT, Topic: Prostaglandins

187. D. 5HT2C Receptor Agonists


Lorcarserin
Is a 5HT2C Receptor Agonists
Promotes weight loss through decreased food consumption and increased satiety
Thus, option D is correct. Ref: GG; topic serotonin

188. B. Epinastine
Epinatine has both H1 and H2 antagonist activity, which may help reduce eyelid
edema
Epinastine and azelastine exhibit mast cell-stabilizing and anti-inflammatory
Properties
Emesdastine is a highly selective H1 antagonist without these additional actions
Thus, option B is correct. Ref: GG Topic : antihistaminic

189. C. Fab fragment toward TNF-a


Certolizumab:
Humanized anti-TNF-a monoclonal antibody
Fab fragment toward TNF-a
Induction or maintenance of remission in moderate-to-severe Crohn disease in
patients who have not responded well to other therapies
Thus, option C is correct. Ref: GG, Topic Rheumatoid Arthritis

190. B. Ketoprofen
Ketoprofen:
Propionic acid derivative
Has additional action to stabilize lysosomes and inhibit LOX has been demonstrating.
Thus, option B is correct. Ref: KDT, Topic: NSAIDS

191. A. Bitter taste


The most common side effect of sumatriptan nasal spray is Bitter Taste
Subcutaneous injection of sumatriptan, patients often experience irritation at the
site of injection.
Thus, option A is correct. Ref: GG Topic : Migraine

192. D. Piroxicam
Non-selective COX inhibitor:
Enolic acid derivatives: Piroxican, Tenoxicam
Preferential COX-2 inhibitor:
Nimesulide, Diclofenac, Aceclofenac Meloxicam, Etodolac
Selective COX-2 inhibitors:
Celecoxib, Etoricoxib, Parecoxib
Thus, option D is correct. Ref: KDT, Topic: NSAIDS

193. B. Reduced purine synthesis in actively dividing cells


Leflunomide:
MOA:
Leflunomide is rapidly converted in the body to an active metabolite which inhibits
dihydroorotate dehydrogenase and pyrimidine synthesis in actively dividing cells.
Antibody production by B-cells may be depressed
The active metabolite has a long t1/2of 2-3 weeks
Inhibits proliferation of T and B cells
Note:
Leflunomide is an alternative to Mtx or can be added to it, but the combination is
more hepatotoxic
Combination with sulfasalazine improves benefit.
Thus, option B is correct. Ref: KDT, Topic: Rheumatoid Arthritis

194. B. Released only by platelets during inflammation


Platelet Activating Factor:
It is potent peptic ulcerogen
Produced by fetus in third trimester
Released by mainly WBC, Platelets, Vascular endothelium and kidney cells during
inflammation but not only by platelets
It increases vascular permeability and has chemotactic property
Rupatadine:
Antihistamine has additional PAF antagonistic property
Indicated in allergic rhinitis
Thus, option B is correct. Ref: KDT, Topic: Antihistaminic

195. A. Etoricixib
The relative degree of selectively of COX 2 inhibition is
Lumiracoxib = Etoricoxib Valdecoxib = Rofecoxib > Celecoxib
Thus, option A is correct. Ref: GG Topic NSAIDS

196. b. Kallikrein antagonist


CGRP receptor antagonist
Olcegepant & Telcagepant (used in migraine)
Serotonin 1F agonist
Lasmiditan (used in migraine)
Bradykinin receptor antagonist
Icatibant
Kallikrein antagonist
Aprotinin (used in coronary bypass surgery)
Thus, option B is correct. Topic Migraine

197. D. Ketorolac
Preferential COX-2 inhibitors:
Nimesulide, Diclofenac, Aceclofenac
Meloxicam, Etodolac
Selective COX-2 inhibitors:
Celecoxib, Etoricoxib, Parecoxib
Non Selective COX inhibitors:
Acetic acid derivatives
Ketorolac, Indomethacin and Nabumetone
Thus, option D is correct. Ref: KDT, Topic: NSAIDS

198. A. Triprolidine
Thus, option A is correct. Ref: KDT, Topic: Antihistaminic

199. D. Treprostonil
In pulmonary HTN
Treprostonil is given by continuous subcutaneous infusion or as an inhalation,
consisting for four daily treatment sessions with none breaths per session
IIoprost is a stable analog that is given by inhalation, but it needs to be given by
nebulizer six to nine times daily.
Misoprostol:
Is a PGE1 analog
Misoprostol in combination with mifepristone or methotrexate is highly effective in
the termination of early pregnancy.
Approved in treatment of NSAID induced peptic ulcer
Not given by inhalational route
Intravaginal, oral tablet
Alprostadil (PGE1):
To maintain patency of Ductus Arteriosus
Injected into the penis causes erection lasting 1-2 hours
Thus, option D is correct. Prostaglandins Ref: GG

200. B. Cinnarizine
Cinnarizine:
Antihistamine drug
Antivertigo drug
Has antimotion sickness property
Probably acts by inhibiting influx of Ca2+ from endolymph into the vestobular
sensory cells which mediates labyrinthine reflexes
Other H1 antihistaminics:
Promethazine
Diphenhydramine
Dimenhydrinate
Doxylamine
Meclozine (Meclizine),
Cinnarizine
Thus, option B is correct. Prostaglandins Ref: KDT, Topic:

201. C. Itopride
Itopride:
Itopride is metabolized mainly by Flavin monooxygenases and not by CYP450
isoenzymes
So Minimal drug interactions other options
Cisapride, Domperidone and Metoclopramide metabolized mainly by CYP450
isoenzymes
Thus, option C is correct. Ref: KDT, Topic: serotonin

202. C. Domperidone prokinetic action is not attenuated by atropine


Domperidone:
Its prokinetic action is not attenuated by atropine and is based only on D2 receptor
blockade in upper g.i.t.
Metacloprapimde:
The peripheral action can augment Ach release in the gut, but appears to be minor
Its prokinetic action is attenuated by atropine.
Thus, option C is correct. Ref: KDT, Topic: serotonin

203. D. Both B and C


Metoclopramide:
This is the primary mechanism
Metoclopramide acts in the g.i.t. to enhance Ach release from myenteric motor
neurons
This results from 5-HT4 receptor activation on primary afferent neurons (PAN) of the
ENS via excitatory interneurons
The gastric hurrying and LES tonic effects are mainly due to this action which is
synergized by Bethanecol and attenuated by atropine
Peripheral D 2 antagonism:
Hastening gastric emptying
Enhancing LES tone by augmenting Ach release
This is the secondary mechanism
Thus, option D is correct. Ref: KDT, Topic: serotonin
204. B. 5-HT3
All serotonin receptors are G-protein coupled receptors except 5-FIT. which is an
inotropic receptor.
Thus, option B is correct. Ref: KDT, Topic: serotonin

205. A. It does not block cardiac K+ channels


Fexofenadine is an active metabolite of Terfenadine. It lacks K+ channel blocking
property, therefore is devoid of arrhythmogenic potential It is available as separate
second generation H] blocking agent.
Thus, option A is correct. Ref: KDT, Topic: antihistaminic

206. D. Dangerous ventricular arrhythmias can occur


Ketoconazole, erythromycin and clarithromycin are potent inhibitors of microsomal
enzymes. Astemizole, Terfenadine and Cisapride are the drugs that car block cardiac
K+ channels at high concentration. If administered along with microsomal ewe
inhibitors, these drugs can result in torsades de pointes, a serious polymorphic
ventricular tachycardia.
Thus, option D is correct. Ref: KDT, Topic: antihistaminic

207. D. These possess high anti-motion sickness activity


Anti-motion sickness activity is related to anticholinergic property. This property is
exhibited by first generation compounds whereas second generation antihistaminics
lack this potential.
Thus, option D is correct. Ref: KDT, Topic: antihistaminic

208. C. Driving motor vehicles


First generation antihistaminic agents cause sedation and impairment of psychomotor
function.
Thus, option C is correct. Ref: KDT, Topic: antihistaminic

209. D. Promethazine
All the drugs listed in the question are second generation anti-histaminic agents
except promethazine.
Thus, option D is correct. Ref: KDT, Topic: antihistaminic

210. B. Morphine
Basic drugs like morphine, d-TC and amphotericin B etc. Act as histamine liberators
and can cause acute reaction leading to itching and hypotension
Thus, option B is correct. Ref: KDT, Topic: antihistaminic

211. D. Direct action on vascular smooth muscle causing vasodilation


Both H1 and H2 receptors cause vasodilation. H1 receptors are present in the
endothelium and cause vasodilation by releasing NO. However, H2 receptors present
on the smooth muscles and cause direct vasodilation.
Thus, option D is correct. Ref: KDT, Topic: antihistaminic

212. D. Ergotamine
Ergot alkaloids are the a-blockers that can cause vasoconstriction.
Thus, option D is correct. Ref: KDT, Topic: Migraine
Respiratory System — Questions
213. All are true statements about Omalizumab except
A. Humanized monoclonal antibody of the IgGI subclass
B. It doesn’t block binding of IgE to low-affinity IgE receptors (FceRIL, CD23) on
other inflammatory cells)
C. Delivered as a single SC injection every 2 to 4 weeks
D. Binds tightly to free IgE in the circulation to form Omalizumab-IgE

214. The following statements are true about clicesonide except


A. Activated by the esterases in the lung
B. It has high pulmonary deposition, prolonged pulmonary residence time
C. Administered through a hydrofluoroalkaline propellant metered dose inhaler
D. Ciclesonide and des-Ciclesonide are least bound to plasma proteins

215. Which of the following is incorrect about inhaled corticosteroids


A. Optimal respirable particle range is between 1-5 microns
B. Chlorofluorocarbons (CFC) have higher output of aerosol particles compared to
hydrofluoroalkane (HFA)
C. Higher particle size results in deposition in upper airways
D. Smaller particles size will result in expelling out of drug during exhalation

216. Preferred ICS in patients of asthma requiring high doses (>1000 ug) of
ICS is
A. Beclomethasone dipropionate
B. Triamcinolone
C. Budesonide
D. All of the above

217. The following are mechanism of Methylxanthines in bronchial asthma and


COPD except
A. Deactivation histone deacetylase
B. PDE 4 inhibition
C. Blockade of adenosine receptors
D. Interleukin-10 release

218. Beta 2 agonists used in asthma have anti-inflammatory effect. Which of


the following Incorrect about their anti-inflammatory effect?
A. They inhibit mast cell mediator release
B. They have significant inhibitory effect on the chronic inflammation of asthmatic
airways
C. They inhibit microvascular leakage
D. All of the above

219. Formoterol is FDA approved for treatment of following conditions except


A. Aspirin induced asthma
B. Treatment of asthma
C. Prophylaxis of exercise-induced bronchospasm,
D. COPD

220. Following statements are true about bambuterol except


A. Prodrug of terbutaline
B. Hydrolyzed by pseudocholinestrase in lungs
C. Used in chronic asthma
D. It is an amide drug

221. Which of the following inhaled corticosteroids (ICS) has long duration of
action
A. Beclomethasone dipropionate
B. Fluticasone hemihydrate
C. Fluticasone propionate
D. Fluticasone furoate
213. b 214. d 215. b 216. c 217. a 218. b 219. a 220. d 221. d

222. Zileuton acts by


A. Inhibiting production of IgE
B. Inhibiting Cyclooxygenase
C. Inhibiting Lipoxygenase
D. Inhibiting mast cell degranulation

223. Proposed mechanism for cardiac arrhythmia cause by theophylline is due


to
A. A 1 receptor antagonism
B. Both A 1 receptor antagonism and PDE 3 inhabitation
C. PDE 3 inhabitation
D. PDE 4 inhabitation

224. Following statements are true about Salmeterol except


A. Slow onset of action and longer acting due to more lipophilicity
B. Can be used alone or in combination with inhaled Glucocorticoids while treating
asthma
C. More B2 selective than salbutamol and partial agonist at B2
D. Used to prevent Nocturnal Asthmatic Attacks and exercise induced asthma

225. Which of the following is incorrect about Arformoterol


A. It an enantiomer of Formoterol
B. It cannot be used with a nebulizer
C. It has twice the potency of racemic formoterol
D. It is FDA-approved for the long-term treatment of bronchoconstriction in patients
with COPD

226. Which of the following is incorrect about Salmeterol


A. Not suitable monotherapy for acute attacks of bronchospasm
B. Salmeterol should not be sued more than twice daily
C. Salmeterol is metabolized by CYP3A4 to a-hydroxy-salmeterol
D. Not suitable for nocturnal asthma

227. Which of the following is a full agonist at beta 2 receptor


A. Formoterol
B. Salmeterol
C. Both A and B
D. None of the above

228. A 55-year-old female who is taking Propanolol for the management of a


cardiovascular disease experiences an acute asthmatic attack. Which of the
following drugs would you prescribe to attenuate this asthmatic attack?
A. Cromolyn sodium
B. Ipratropium bromide
C. Beclomethasone
D. Salbutamol

229. Which of the following B2 agonists is given by inhalation, and is suitable


for both terminating acute asthma attacks as well as for twice daily
prophylaxis?
A. Formoterol
B. Bambuterol
C. Salmeterol
D. Terbutaline

230. Omalizumab is a monoclonal antibody used for treatment of


A. Digitalis Toxicity
B. Breast carcinoma
C. Rheumatoid arthritis
D. Bronchial asthma

231. One of the most common side effects of inhaled beclomethasone


dipropionate is
A. Pneumonia
B. Pituitary adrenal suppression
C. Atrophic rhinitis
D. Oropharyngeal candidiasis

232. In comparison to inhaled adrenergic agonists, the inhaled


anticholinergics?
A. Produce slower response-in bronchial asthma
B. Are better suited for control of an acute attack of asthma
C. Are more effective in bronchial asthma
D. Produce little benefit in chronic-obstructive lung disease
222. c 223. 224. 225. 226. 227. 228. 229. a 230. d 231. d
b b b d a b

232.
a

233. Which of the following drugs CANNOT be administered by inhalation?


A. Budesonide
B. Ipratropium bromide
C. Theophylline
D. Terbutaline

234. Relatively higher dose of theophylline is required to attain therapeutic


plasma concentration in
A. Those receiving cimetidine
B. Congestive heart failure patients
C. Those receiving erythromycin
D. Smokers

235. Which of the following statements about theophylline is-TRUE?


A. Its plasma half-life is longer in children as compared to that in adults
B. Its dose needs to be reduced in the smokers
C. It acts by increasing the formation of cAMP
D. Its use in asthma has declined because of narrow safety margin

236. Which of the following actions is NOT exhibited by methylxanthines?


A. Inhibit the acetylation of core histones
B. Induction of histone deacetylase
C. Anti-inflammatory action
D. Restore steroid responsiveness in COPD patients

237. Which of the following is delivered as an aerosol formulation through


adry powder inhaler.
A. Ipratropium bromide
B. Formoterol
C. Salmeterol
D. Indacaterol
233. c 234. d 235. d 236. d 237. d
Respiratory System — Explanations
213. B. It doesn’t block binding of IgE to low-affinity IgE receptors (FceRIL,
CD23) on other inflammatory cells)
Omalizumab:
Humanized monoclonal antibody of the IgGI subclass
Delivered as a single subcutaneous injection every 2 to 4 weeks
Binds tightly to free IgE in the circulation to form Omalizumab-IgE
It also blocks binding on IgE to low-affinity IgE receptors (FceRIL, CD23) on other
inflammatory cells, including T and B lymphocytes, macrophages and possibly
eosinophils to inhibit chronic inflammation
Omalizumab also reduces levels of circulating IgE
In severe extrinsic asthma it reduces exacerbation and steroid requirement
The most frequent adverse effect was injection-site reactions (e.g. redness, stinging,
bruising and induration)
The major side effect of Omalizumab is an anaphylactic response, which is
uncommon.
Thus, B is correct. Ref: GG, Topic : Asthma

214. D. Ciclesonide and des-Ciclesonide are least bound to plasma proteins


Cilcesonide
Prodrug which is administered through a hydrofluoroalkane propellant metered dose
inhaler
Once it reaches the lungs, the parent compound is metabolized by esterases to
desisobutyryl Ciclesonide (des-CIC), an active metabolite with a 100-fold greater
affinity to the glucocorticoid receptor
It has high pulmonary deposition, prolonged pulmonary residence time and on-site
activation minimizes the risk for local side effects
It’s low oral bioavailability high hepatic clearance, and extensive plasma protein
binding, among other factors, decrease the risk for systemic side effects
Ciclesonide and des-Ciclesonide are highly bound to plasma proteins minimizing
their systemic side effects
It is a soft steroid
Thus, D is correct. Ref: Emma D. Deeks and Caroline M. Perry Ciclesonide. A review of
its Use in the Management of Asthma. Drugs 2008;68 (12): 1741-1770

215. B. Chlorofluorocarbons (CFC) have higher output of aerosol particles


compared to hydrofluoroalkane (HFA)
Inhaled corticosteroids
Optimal respirable particle range is between 1-5 microns
Higher particle size results in deposition in upper airways
Smaller particles size will result in expelling out of drug during exhalation
Hydrofluoroalkane (HFA) have higher output of aerosol particles compared to
chlorofluorocarbons (CFC)
Thus, B is correct. Ref: KDT, Topic: Asthma

216. C. Budesonide
Preferred ICS in patients of asthma requiring high doses (>1000 ug) of ICS is:
Budesonide
Flutocasone propionate
Because they are preferred in patients who need high doses of ICSs and in children.
Thus, C is correct. Ref: GG, Topic : Asthma

217. A. Deactivation histone deacetylase


Mechanism of Actions-Theophylline:
PDE inhibition: 3 & 4
Blockade of adenosine receptors
Interleukin-10 release
Has a broad spectrum of anti-inflammatory effects, and there is evidence that its
secretion is reduced in asthma. IL-10 release is increased by theophylline
Effects on gene transcription
Inhibits transcription of inflammatory genes
Effects on apoptosis: (increased apoptosis of eosinophils and neutrophils)
Activation of Histone deacetylase
Recruitment of histone deacetylase-2 (HDAC2) by glucocorticoid receptors switches
off inflammatory genes.
Therapeutic concentrations of theophylline activate HDAC, thereby enhancing the
anti-inflammatory effect of corticosteroids.
Thus, A is correct. Ref: GG, Topic : Asthma

218. B. They have significant inhibitory effect on the chronic inflammation of


asthmatic airways.
Beta 2 agonists used in asthma have anti-inflammatory effect.
Inhibitory effects of B2 agonists on mast cell mediatory release
Inhibitory effects of B2 agonists on microvascular leakage
B2 agonists may modify acute inflammation
They DONOT have significant inhibitory effect on the chronic inflammation of
asthmatic airways.
Thus, B is correct. Ref: GG; Topic: Asthma

219. A. Aspirin induced asthma


Formoterol:
FDA-approved for
Treatment of asthma and bronchospasm
Prophylaxis of exercise-induced bronchospasm
COPD
Thus, A is correct. Ref: GG; Topic: Asthma

220. D. It is an amide drug


Bambuterol
Prodrug of terbutaline
It is an ester drug
Hydrolyzed by pseudocholinestrase in lungs to active Terbutaline
Used in chronic asthma
Orally, single evening dose of 10-20 mg
Thus, D is correct. Ref: KDT, Topic: Asthma

221. d. Fluticasone furoate


Fluticasone furoate has the longest duration of action and is suitable for once-daily
dosing.
Thus, D is correct. Ref: GG; Topic: Asthma

222. C. Inhibiting Lipoxygenase


Zileuton:
5-LOX inhibitor
So, blocks LTC4/D4 as well as LTB4 synthesis
Duration of action of zileuton is short and it has hepatotoxic potential.
Thus, C is correct. Ref: KDT, Topic: Asthma

223. B. Both A 1 receptor antagonism and PDE 3 inhabitation


Theophylline:
Belongs to methylxanthines group of drugs
It has bronchodilator action and nonbronchodilator effects
It has various mechanisms of action
Inhabitation of Phosphodiesterase’s
Adenosine receptor antagonism
Interleukin-10 release
Prevents the translocation of the pro-inflammatory transcription factor NF-kB into the
nucleus
Promotes apoptosis in eosinophils and neutrophils in vitro
Histone deacetylase activation
Side Effects:
The most common side effects are headache, nausea and vomiting, abdominal
discomfort and restlessness and other side effects are described in the below table

Side Effects of Theophylline and Mechanisms

Side Effect Proposed Mechanism

Nausea and vomiting PDE4 Inhibitation

Headches PDE 4 Inhibitation

Gastric discomfort PDE 4 Inhibitation

Diuresis A1 receptor antagonism

Behavioural disturbance (?) ?

Cardiac arrhythmias PDE 3 Inhibitation

Epileptic seizures A1 receptor antagonism

A1 receptor antagonism

PDE, Phosphodiesterase: A, Adenosine

Thus, B is correct. Ref: GG, Topic: Asthma

224. B. Can be used alone or in combination with inhaled Glucocorticoids while


treating asthma
Salmeterol
Slow onset action
More B2 selective than salbutamol
More lipophilic so longer acting
Used to prevent Nocturnal Asthmatic Attacks and exercise induced asthma
Always used in combination with inhaled Glucocorticoids in Bronchial asthma but not
alone
IN COPD
LABA are effective bronchodilators that may be used alone or in combination with
anticholinergics or ICS. LABA improve symptoms and exercise tolerance by reducing
both air trapping and exacerbations.
In asthma patients
LABA should never be used alone because they do not treat the underlying chronic
inflammation; rather, LABA should always be used in combination with ICS
(preferably in a fixed-dose combination inhaler). LABA are an effective add-on-
therapy to ICS and are more effective than increasing the dose of ICS when asthma is
not controlled at low doses
Formoterol is full agonist so is more rapid acting, whereas Salmeterol is a partial
agonist with a slower onset of action.
Thus, B is correct. Ref: GG; Topic: Asthma

225. B. It cannot be used with a nebulizer


Arformoterol:
It an enantiomer of formoterol
It has twice the potency of racemic formoterol
It is FDA-approved for the long-term treatment of bronchoconstriction in patients with
COPD, chronic bronchitis and emphysema
It was the first LABA developed as inhalational therapy for use with a nebulizer.
Thus, B is correct. Ref: GG; Topic: Asthma

226. D. Not suitable for nocturnal asthma


Salmeterol:
The onset of action of inhaled salmeterol is relatively slow, so it is not suitable
monotherapy for acute attacks of bronchospasm
Salmeterol and Formoterol are the agent of choice for nocturnal asthma in patients
who remain symptomatic despite anti-inflammatory agents and other standard
management.
It is metabolized by CYP3A4 to a-hydroxy-Salmeterol, which is eliminated primarily in
the feces
Salmeterol should not be used more than twice daily
Has the potential to increase heart rate and plasma glucose concentration, to
produce tremors, and to decrease plasma K+ concentration through effects on extra
pulmonary B2 receptors.
Thus, D is correct. Ref: GG 13E/P 201; Topic: Asthma

227. A. Formoterol
Formoterol is full agonist so is more rapid acting whereas salmeterol is a partial
agonist with a slower onset of action.
Thus, A is correct. Ref: GG; Topic: Asthma

228. B. Ipratropium bromide


Inhaled β2 agonists are the agents of choice for termination of acute attack of
bronchial asthma. However, as the patient is receiving β-blockers, treatment with β2
agonists will be ineffective (receptors are already blocked). Therefore, other
bronchodilators like anticholinergic agents (ipratropium) or methybcanthines
(theophylline) will be useful in such a case.
Thus, B is correct. Ref: KDT, Topic: Asthma

229. A. Formoterol
Terbutaline is a fast-acting bronchodilator useful for terminating the acute attack of
bronchial asthma. Due to short duration of action, it is not suitable for chronic
prophylaxis.
Bambuterol, salmeterol and formoterol are long acting 132 agonists useful for
chronic prophylaxis.
Bambuterol and salmeterol are delayed acting, therefore are not suitable for acute
attacks.
Formoterol is fast acting also, therefore can be used for the treatment of acute attack
of asthma.
Thus, A is correct. Ref: KDT, Topic: Asthma

230. D. Bronchial asthma


Omalizumab is a monoclonal antibody against IgE. It is useful for the management of
bronchial asthma.
Thus, D is correct. Ref: KDT, Topic: Asthma

231. D. Oropharyngeal candidiasis


Most common adverse effect of inhaled corticosteroids is oropharyngeal candidiasis.
Pituitary adrenal suppression is less likely with inhalational route of corticosteroids
than with oral route.
Thus, D is correct. Ref: KDT, Topic: Asthma

232. A. Produce slower response-in bronchial asthma


Anticholinergic drugs like ipratropium and tiotropium produce slower response in
bronchial asthma.
Anticholinergic drugs are more effective in COPD than bronchial asthma.
Thus, A is correct. Ref: KDT, Topic: Asthma

233. C. Theophylline
Budesonide nebules can be mixed with terbutaline, salbutamol and ipratropium
Thus, C is correct. Ref: KDT, Topic: Asthma

234. D. Smokers
Smokers taking theophylline generally tend to require higher doses than non-smokers
as tobacco smoke contains polycyclic hydrocarbons, which induce CYP1A2. Smoking
cessation will therefore result in an increase in serum theophylline concentrations, and
possibly toxicity, if the dose is not reduced.
Thus, D is correct. Ref: KDT, Topic: Asthma

235. D. Its use in asthma has declined because of narrow safety margin
Children are fast metabolizers of theophylline as compared to neonates and elderly,
assuming adult are normal metabolizers. Theophylline dose needs to be increased in
smokers. It acts by non selective PDE enzyme inhibition
Thus, D is correct. Ref: KDT, Topic: Asthma
236. D. Restore steroid responsiveness in COPD patients
All are correct molecular mechanisms of anti-inflammatory action of Theophylline.
Thus, D is correct. Ref: KDT, Topic: Asthma

237. D. Indacaterol
It needs to be taken only once a day, unlike the related drugs formoterol and
salmeterol. It is licensed only for the treatment of chronic obstructive pulmonary
disease (COPD) (long-term data in patients with asthma are thus far lacking). It is
delivered as an aerosol formulation through a dry powder inhaler.
Thus, D is correct. Ref: KDT, Topic: Asthma
Endocrine System — Questions
238. Raju, a 30-year-old male presents to the OPD with sudden onset pain,
swelling and redness of the left first metatarsophalangeal joint. A needle
aspirate of the joint shows needle-shaped, negatively birefringent crystals.
The physician prescribed a drug for the patient, but he came back next day
with nausea, vomiting and diarrhea after taking the medication. Which of
the following is the most likely drug that was prescribed to this patient?
A. Allopurinol
B. Steroids
C. Indomethacin
D. Colchicine

239. Desmopressin can be given by following routes except


A. Transdermal
B. Intranasal
C. Sublingual
D. Inhalation through metered-dose nasal inhaler

240. All are true except


A. Terlipressin is better than desmopressin for esophageal variceal bleeding
B. Desmopressin used for both pituitary and renal diabetes insipidus
C. Desmopressin not much effective against Nephrogenic diabetes insipidus
D. Amiloride used for lithium induced diabetes insipidus

241. One of the health benefit of the use of combined oral contraceptives in
pre-menopausal women is that the Be contraceptives reduce the risk of
A. Ovarian cancer
B. Migraine
C. Deep vein thrombosis
D. Ischemic stroke

242. The combined estrogen-progestin oral


A. Production of cervical mucus ho
B. Making endometrium unsuitable for implantation
C. Suppression of FSH and LH release
D. Enhancing uterine contraction to dislodge the fertilized ovum

243. Clomiphene citrate is not known to produce which of the following effects
in a young female of 30 years of age (child bearing aw group)?
A. Decreased FSH and LH secretion
B. Ovulation
C. Hot flushes
D. Polycystic ovaries
244. Dr. Neelam decides to give estrogen therapy in a postmenopausal
woman. The risk of which of the following will not be increased?
A. Gall stones
B. Breast cancer
C. Endometrial carcinoma
D. Osteoporosis

245. An endocrinologist decided to give a 7-year-old boy testosterone therapy


and continued it till puberty. The therapy is likely to
A. Increase adult stature
B. Cause hypertrophy of penis
C. Have no effect on adult stature
D. Reduce adult statue

246. The enzyme 5a reductase mediated conversion of testosterone to


dihydrotestosterone is NOT required for which of the following actions?
A. Spermatogenesis
B. Prostatic hypertrophy in elderly males
C. Pubertal changes in the male adolescent
D. Formation of male external genitalia in the fetus
238. d 239. a 240. b 241. a 242. c 243. a 244. d 245. d 246. a

247. A patient Geeta began taking alendronate and she was advised to large
amount of water and remain in the standing position for at least half an
hour till she had the first meal of the day. These instructions were given to
reduce the risk of
A. Erosive esophagitis
B. Constipation
C. Cholelithiasis
D. Osteonecrosis

248. All of these drugs can be used in the treatment of post-menopausal


osteoporosis EXCEPT
A. Thyroxine
B. Teriparatide
C. Calcium
D. Alendronate

249. The unique property of SERMs is that they


A. Have both estrogenic and progestational agonistic activity
B. Act as agonist in some tissues and antagonist in other tissues
C. Produces estrogenic effect without binding to estrogen receptors
D. Inhibits the aromatase enzyme that is required for estrogen synthesis

250. A child Mahesh has been diagnosed to be having vitamin D dependent


rickets. The most appropriate vitamin D preparation for Mm is
A. Calcitriol
B. Cholecalciferol
C. Calcifediol
D. Calciferol

251. Chronic use of which of the following medications is most likely to cause
osteoporosis?
A. Prednisone
B. Propanolol
C. Warfarin
D. Lovastatin

252. Long term steroid therapy can lead to suppression of hypothalamic


pituitary-adrenal axis. It can be overcome by using alternate day therapy
with corticosteroids. Which of the following steroids are unsuitable for
alternate day therapy for such purpose?
A. Betamethasone
B. Prednisolone
C. Cortisol
D. Hydrocortisone

253. A 50-year-old man with advanced tuberculosis has developed signs of


severe acute adrenal insufficiency. The patient should be treated
immediately with a combination of
A. Cortisol and fludrocortisone
B. Triamcinolone and dexamethasone
C. Aldosterone and Fludrocortisone
D. Dexamethasone and metyrapone

254. Shanti has been diagnosed to have brain tumor. You would prefer to give
her betamethasone/dexamethasone over hydrocortisone as steroids is to
decrease her cerebral edema because
A. They can be administered intravenously
B. They are more potent
C. They do not cause Na+ and water retention
D. They inhibit brain tumors

255. A patient Dharampal has been diagnosed to have bronchial asthma and is
maintained on oral prednisolone 20 mg daily and inhaled salbutamol as
required. The patient develops chest infection. Which of the following
measures would you like to take?
A. Stop prednisolone
B. Reduce prednisolone dose to 5 mg/day
C. Substitute prednisolone with inhaled budesonide
D. No change/increase in prednisolone dose

256. Toxic effects of long term administration of a glucocorticoid include


A. Hepatotoxicity
B. Lupus like syndrome
C. Precocious puberty
D. Osteoporosis
247. 248. 249. 250. 251. 252. 253. 254. c 255. d 256. d
a a b a a a a

257. In the treatment of congenital adrenal hyperplasia due to lack of 21 B-


hydroxylase, the purpose of administration of a synthetic glucocorticoid is
A. Inhibition of aldosterone synthesis
B. Suppression of ACTH secretion
C. Recovery of normal immune function
D. Prevention of hypoglycemia

258. Corticosteroids are used in which of the following disorders is therapy?


A. Peptic ulcer
B. Diabetes mellitus
C. Hyper tension
D. Congenital adrenal hyperplasia

259. Select the regime of corticosteroids which has the maximum adverse
effect potential?
A. Methyl-prednisolone 1000 mg Intravenous twice single dose
B. Prednisolone 60 mg/day oral for 7 days
C. Dexamethasone 4 mg intravenous
D. Prednisolone 20 mg/day oral for one year

260. Hydrocortisone acts as an anti-inflammatory agent because of induction


of the synthesis of which of the following protein?
A. Heat shock protein 90
B. Lipocortin
C. Transcortin
D. Inhibin

261. Aldosterone is known to cause sodium retention. Its Na+ retaining action
is exerted on which part of the nephron?
A. Collecting ducts
B. Ascending limb of loop of Henle
C. Proximal convoluted tubule
D. Early distal convoluted tubule

262. A 54-year-old obese patient with type 2 diabetes mellitus and a history of
alcohol is in probably should not receive metformin because it can increase
the risk of
A. Lactic acidosis
B. Hypoglycemia
C. Disulfiram like reaction
D. Severe hepatic toxicity

263. A 15-year-old girl with type l diabetes is brought to emergency


complaining of dizziness. Laboratory findings include severe hyperglycemia,
ketoacidosis and blood pH of 7. 15. To achieve rapid control of severe
ketoacidosis, appropriate drug is
A. Tolbutamide
B. NPH insulin
C. Crystalline zinc insulin
D. Ultra Lente insulin

264. Which of the following drugs is most likely to cause hypoglycemia when
used as a monotherapy in the treatment of type 2 diabetes?
A. Acarbose
B. Rosiglitazone
C. Metformin
D. Glipizide

265. The correct statement regarding the present status of oral hypoglycemic
in diabetes mellitus is
A. They are used first in most cases of uncomplicated mild to moderate type 2
diabetes
B. They should be prescribed only if the patient refuse insulin injections
C. They are used in type 1 diabetes mellitus
D. They are the first choice drug in all cases

266. Which of the Following statements about Nateglinide is TRUE?


A. It is a long acting oral hypoglycemic drug
B. It acts by opening K · channels in myocytes and adipocytes
C. It lowers blood glucose in both type 1 and type 2 diabetes mellitus
D. Taken just before a meal, it limits post prandial hyperglycemia in type 2 diabetes
mellitus
257. 258. 259. 260. 261. 262. 263. c 264. d 265. a 266. d
b d d b a a

267. Metformin is NOT effective in lowering of blood sugar level in which of


the following patients?
A. Type 2 Diabetics
B. Obese diabetic
C. Non diabetics
D. Diabetics not responding to sulfonylureas

268. Glibenclamide reduces blood glucose in a11 of the following EXCEPT


A. Non diabetics
B. Obese diabetics
C. Type 2 diabetics
D. Type 1 diabetics

269. Which of the following characteristics make metformin a preferred


biguanide than phenomenon?
A. It is more potent
B. It does not interfere with vitamin B, 2 absorption
C. It is less liable to cause lactic acidosis
D. It is not contraindicated in patients with kidney disease

270. Glipizide differs from Chlorpropamide in that it.


A. Does not lower blood sugar in non-diabetic subjects
B. Is longer acting
C. Is more potent
D. Is less prone to cause hypoglycemic reaction

271. Human insulin as compared to pork/beef insulin is


A. More potent
B. More antigenic
C. Longer acting
D. Rapidly absorbed

272. Inulin acts by stimulation of


A. Enzymatic receptor
B. Ionotropic receptor
C. Metabotropic receptor
D. Nuclear receptor

273. Which of the following patients is most likely to be treated with


intravenous glucagon?
A. An old man with severe bradycardia and hypotension resulting from ingestion of
overdose of atenolol
B. A middle aged man with type II diabetes who has not taken his regular dose of
glipizide for last 4 days
C. A young man who took cocaine and has a blood pressure of 190/110 mm Hg
D. An old woman with lactic acidosis as a complication of severe infection and shock

274. Which of the following drugs is taken during the first part of the meal for
the purpose of delaying absorption of dietary carbohydrates?
A. Nateglinide
B. Glipizide
C. Acarbose
D. Pioglitazone

275. Manish, a patient of hypothyroidism was prescribed thyroxine. Which the


following is the most reliable guide for adjustment of thyroxine dose in him?
A. Pulse rale
B. Serum TSH level
C. Scrum thyroxine level
D. Body weight

276. A pregnant female Salma with thyrotoxicosis is planned for Surgery.


Before surgery can be done, her gland should be reduced in size and
vascularity by administering
A. Iodine ion
B. Propylthiouracil
C. Propanolol
D. Radioactive iodine

277. In the treatment of hypothyroidism, thyroxine is preferred over


levothyroxine because thyroxine
A. Has longer Half-life
B. Has higher affinity for thyroid hormone receptors
C. Is faster acting
D. Can be made more easily by recombinant DNA technology
267. c 268. 269. c 270. c 271. 272. 273. 274. c 275. b 276. b
d d a a

277.
a

278. I 31 is the preferred treatment for


A. Elderly patients with ischemic heart disease
B. Young adult with recent onset Graves’ disease
C. Children
D. Pregnant women

279. Which of the following drugs inhibit 5’- Deiodinase?


A. Lugol’s Iodine
B. Methimazole
C. Propylthiouracil
D. Radioactive iodine

280. Mechanism of action of Propylthiouracil in hyperthyroidism is


A. Inhibition of organification of iodine
B. Inhibition of oxidation of iodine
C. Inhibition of coupling of two DIT residues
D. All of the above

281. Triiodothyronine (T3) as compared to T4


A. ls more plasma protein bound
B. Has delayed action
C. Is less potent
D. Is shorter acting
282. The anti-thyroid drug with the most rapid onset of anti-thyroid action is
A. I 31
B. Propylthiouracil
C. Methimazole
D. Sodium iodide

283. A 7-year old boy, Manoj underwent successful chemotherapy and cranial
radiation for the treatment of acute lymphocytic leukemia. One month after
the completion of therapy, the patient presented with excessive thirst and
urination plus hypernatremia. Laboratory testing revealed pituitary diabetes
insipidus. To correct these problem, thin patient is likely to be treated with
A. Corticotropin
B. hCG
C. Desmopressin
D. Menotropins

284. A 47-year old male, Kishore exhibited signs and symptoms of acromegaly.
Radiologic studies showed the presence of a large pituitary tumor. Surgical
treatment of the tumor was on partially effective in controlling the disease.
At this point, which of the following drugs is most Likely to be used a8
pharmacological therapy
A. Octreotide
B. Leuprolide
C. Desmopressin
D. Somatropin

285. A young female, Rama with amenorrhea, infertility and galactorrhoea


was treated with a drug that successfully restored ovulation and
menstruation. Before being given the drug, the women was carefully
questioned about previous mental health problems, which she did not have.
She was advised to take the drug orally, the drug used to treat this patient
was probably
A. Leuprolide
B. Desmopressin
C. Human gonadotropin hormone
D. Bromocriptine

286. Which of the following drugs Do Not cause hyper-prolactinemia?


A. Reserpine
B. Haloperidol
C. Bromocriptine
D. Chlorpromazine

287. Important difference between leuprolide and ganirelix is that ganirelix


A. Can be given orally
B. Initially stimulates release of LH and FSFI
C. Must be given in a pulsatile fashion
D. Immediately reduces gonadotropin secretion
278. 279. c 280. 281. 282. 283. c 284. 285. d 286. c 287. d
a d d d a

288. All of the following agents are useful in acromegaly EXCEPT


A. Bromocriptine
B. Nafarelin
C. Octreotide
D. Somatostatin

289. The oral diabetic dispensed with black box warning of bladder cancer is
A. Glimepiride
B. Pioglitazone
C. Sitagliptin
D. Acarbose

290. All of the following is supposed to increase the risk of lactic acidosis in a
patient taking metformin except
A. Advanced age
B. Liver dysfunction
C. Renal dysfunction
D. Smoking

291. A specific antidote for sulfonylurea-induced hypoglycemia


A. Glucagon
B. Hydrocortisone
C. Octreotide
D. Adrenaline

292. A patient was diagnosed with diabetic ketoacidosis, but the treating
intern gave only insulin rapidly to lower blood glucose. What complication is
expected by this
A. Hypernatremia
B. Hypokalemia
C. Hypocalcaemia
D. Hyperuricaemia

293. A DMT2 patient was in a foreign country for a business trip. His doctor
had given him insulin U 40 Vial with U-40 insulin syringe. He forgets to take
the syringe in a hurry. The country, he visited has only U-100 insulin syringe.
His doctor had advised taking 12 units of U-40 insulin in the morning. Now
how much divisions in the U-100 Syringe he need to get 12 units
A. 25
B. 30
C. 28
D. 34
294. A small amount of Zinc is added in insulin preparations to improve
A. Affinity
B. Duration of Action
C. Efficacy
D. Rapid Onset

295. Following statements about Lente insulin are true except


A. An intermediate-acting insulin
B. 7:3 mixtures of Ultralite and semilente
C. Large particles in Lente insulin is crystalline and practically soluble in water
D. Also known as Insulin-Zinc suspension

296. False statement about insulin receptor is


A. Consists of two covalently linked heterodimers, each containing a & b subunit
B. A subunit, entirely extracellular recognition site & b subunit that spans the
membrane, contains a tyrosine kinase
C. Binding of insulin to a subunit activates the receptor
D. The first proteins to be phosphorylated by the activated receptor tyrosine kinases
are the MAP kinase & protein phosphatases

297. Which of the following drug is approved as FDC with empagliflozin


recently
A. Sitagliptin
B. Linagliptin
C. Saxagliptin
D. Alogliptin

298. A patient developed a sore throat on treatment with Methimazole after 3


months. What immediate investigation do you recommend?
A. Renal function test
B. Liver function test
C. Throat swab culture
D. Complete blood count
288. 289. 290. 291. c 292. 293. 294. 295. c 296. d 297. b
b b d b b b

298.
d

299. Both iodothyronine deiodinase type 1 and type 2 is inhibited by


A. Propylthiouracil
B. Amiodarone
C. Propranolol
D. Corticosteroids

300. Drug of choice for Graves’ disease is Methimazole for following reasons
except
A. Effective when given as a single daily dose
B. Improved patient adherence
C. Has a relatively short plasma half-life, so toxicity is less
D. Less toxic than propylthiouracil

301. In case of hypothyroidism, patient becomes pregnant, a dose of


thyroxine needs to
A. Decreased
B. Increased
C. Continue the same dose
D. Doubles

302. Which of the following is incorrect regarding I-thyroxine


A. Should be administered in empty stomach
B. Proton pump inhibitors also reduce I-thyroxine absorption
C. Rifampin INCREASES metabolism of T4
D. Oral bioavailability of I-thyroxine is – 25%Note: Ref: KDT 7E/P 250

303. Thyrotropin alpha is used for


A. Stimulation of radioiodine uptake and thyroglobulin release in patients with
thyroid cancer after thyroidectomy
B. Prepare patients for radioiodine ablation of thyroid remnants after thyroidectomy
for thyroid cancer
C. Both A & B
D. NONE

304. A drug used in osteoporosis which doesn’t act by decreasing bone


resorption is
A. Raloxifene
B. Teriparatide
C. Denosumab
D. Alendronate

305. Subcutaneous Calcitonin is used in


A. Osteomalacia
B. Osteoporosis
C. Hypercalcemia
D. Paget’s disease

306. Strontium ranelate


A. Increases bone formation
B. Decreases bone resorption
C. Both A & B
D. NONE

307. A drug which is known to cause osteonecrosis on IV administration is


A. Tamoxifen
B. Zolendronate
C. Calcitriol
D. Strontium ranelate

308. Most potent Bisphosphonate among the following is


A. Etidronate
B. Pamidronate
C. Alendronate
D. Risedronate

309. Following drugs increase bone resorption except


A. Corticosteroids
B. Furosemide
C. Hydrochlorothiazide
D. Hypervitaminosis D

310. Following adverse effects are seen on use of topical corticosteroids


except
A. Easy bruising
B. Increased growth of hairs
C. Hypopigmentation
D. Delayed wound healing

311. Following are measures that minimize HPA axis suppression except
A. Use shorter acting steroids
B. Use steroids for the shortest period of time
C. Switch to alternate-day therapy if possible
D. Give the dose divided into three times daily so that peak steroid concentration is
reduced
299. 300. c 301. 302. 303. c 304. 305. 306. c 307. b 308. d
b b d b d

309. c 310. 311.


b d

312. All of the following statements regarding corticosteroids are correct


except
A. Inhibit production of surfactant
B. Increase the concentration of neutrophils in circulation
C. Inhibit function of macrophages
D. Reduce synthesis of prostaglandins

313. Which of the following glucocorticoid has half-life between 12-36 hours
A. Hydrocortisone
B. Betamethasone
C. Prednisolone
D. Cortisol

314. Following adverse effects are seen with the use of mineralocorticoids
except
A. Sodium and water retention
B. Oedema
C. The progressive rise in BP
D. Hyperkalemic alkalosis

315. Which of the following GnRH antagonists used for the management of
moderate to severe pain associated with endometriosis
A. Cetrorelix
B. Degarelix
C. Elagolix
D. Abarelix

316. MALA D contains


A. Norgestrel 3 mg Ethinylestradiol 30 ug
B. Norgestrel 0.3 mg Ethinylestradiol 50 ug
C. Norgestrel 5 mg Ethinylestradiol 5 0 ug
D. Norgestrel 0.5 mg Ethinylestradiol 30 ug

317. All are emergency contraceptive except


A. Levonorgestrel
B. Ulipristal
C. Desogestrel
D. IUCD

318. Onapriston is
A. SERM
B. Pure progesterone antagonist
C. Anti-estrogen drug
D. Anti-androgen drug

319. Which of the following drugs is used in metastatic breast carcinoma in


premenopausal women
A. Fulvestrant
B. Raloxifene
C. Tamoxifen
D. Clomiphene

320. Which of the following is correct regarding the treatment of infertility


with clomiphene
A. To be taken once daily for 5 days starting from the 5th day of the cycle
B. Daily dose needs to be doubled. If 2-3 months of treatment does not result in
conception
C. It can be tried up to one year and not more than that
D. All of the above

321. Women receiving estrogen therapy have an increased risk of developing


all the following except
A. Hepatoma
B. Endometrial cancer
C. Breast Cancer
D. Carcinoma of gallbladder

322. Identify the wrong match


A. Sildenafil: used in pulmonary hypertension
B. Finasteride: approved in male pattern baldness
C. Goserelin also approved for ovarian cancer
D. Flutamide: used in Hirsutism in females

323. Following are true about 5 alpha-Reductase inhibitors except


A. Block the conversion of testosterone to dihydrotestosterone
B. Impotence is not seen with the use of these drugs
C. Finasteride also is approved for use in the treatment of male pattern baldness
D. Finasteride is an effective as flutamide and the combination of estrogen and
cyproterone in the treatment of hirsutism
312. 313. c 314. 315. c 316. 317. c 318. 319. c 320. d 321. d
a d d b

322. c 323.
b

324. Which of the following is a steroidal antiandrogen


A. Flutamide
B. Megestrol
C. Bicalutamide
D. None of the above

325. A female was given a higher dose of Methylergometrine for post-partum


hemorrhage and continued after that for several days. Mother had very little
milk secretion for 1 month. The reason for this is
A. the partial agonistic action of Methylergometrine on 5-HT2 receptors
B. The agonistic action of Methylergometrine on dopamine receptors
C. The partial agonistic action of Methylergometrine on a-adrenergic receptors
D. None of the above

326. All the given drugs are GnRH agonists except


A. Buserelin
B. Leuprolide
C. Tesamorelin
D. Goserelin
327. The following drug is/are associated with valvar heart disease except
A. Cabergoline
B. Pergolide
C. Both a and b
D. None of the above

328. Which of the following is not a therapeutic indication of Octreotide


A. Acute Stomatitis
B. Carcinoid Syndrome
C. Acromegaly
D. Acute Pancreatitis

329. Octreoscan is used in


A. Diagnostic imaging of gliomas
B. Diagnostic imaging of osteosarcoma
C. Diagnostic imaging of osteoid osteoma
D. Diagnostic imaging of pituitary adenomas

330. Tesamorelin is used in


A. Prolactinoma
B. Pituitary dwarfism
C. Gigantism
D. HIV-associated lipodystrophy

331. Following statements are true about Exenatide except


A. Can cause hypoglycemia
B. It is a GLP-1 agonist
C. Do not promote weight gain
D. Suppress glucagon

332. Which of the following is incorrect ABOUT PRAMLINTIDE


A. The most common adverse effect is nausea
B. Administered as a subcutaneous injection prior to meals
C. Approved for treatment of types 1 and 2 diabetes
D. Pramlintide can be given in patients with gastroparesis

333. The oral diabetic dispensed with black box warning of bladder cancer is
A. Glimepiride
B. Pioglitazone
C. Sitagliptin
D. Acarbose

334. A 62-year-old female, Phoolwati presents to the emergency with acute


severe low back pain after too quickly sitting down onto a chair. She has a
history of rheumatoid arthritis and bronchial asthma. She reports that the
was on many medications for several years. X-ray shows a fracture of the
fifth lumbar vertebra. Which of the following drugs are likely responsible for
the patient’s complaints?
A. Methotrexate
B. Salbutamol
C. Indomethacin
D. Prednisolone

335. All of the following agents act through nuclear receptors EXCEPT
A. Prednisolone
B. Rosiglitazone
C. Thyroxine
D. Estrogen
324. 325. 326. c 327. c 328. 329. 330. 331. b 332. d 333. b
b b a b d

334. 335.
d a

336. Teriparatide can be used for the treatment of


A. Polycystic ovarian disease
B. Hormone responsive breast carcinoma
C. Osteoporosis
D. Hyperparathyroidism

337. Exenatide is a newer drug proposed to be used in the treatment of


A. Osteoporosis
B. Hyperparathyroidism
C. Diabetes mellitus
D. Anovulatory infertility

338. Which of the following is an adrenergic drug preferred for arresting


labour?
A. Salbutamol
B. Isoprenaline
C. Ritodrine
D. Terbutaline

339. A patient parul gives you the history that the has missed a single dose of
her combined oral contraceptive pill. Which of the following will you advise
her?
A. Continue with the course without regard to the missed dose
B. Discontinue the course and use alternative method of contraception
C. Take 2 pills every day for the remaining part of the course
D. Take 2 pills the next day and continue with the course

340. A young female Shagun comes to you in the gynaecology OPD and gives
the history that she had intercourse with her boyfriend 5 hours back Select
the drug that can act as a single dose postcoital contraceptive for her
A. Clomiphene citrate
B. Danazol
C. Mifepristone
D. Medroxyprogesterone acetate

341. Dr. Shweta decided to add a progestin for 10-12 days each month to
estrogen replacement therapy in menopausal women in the gynecology
OPD. Addition of progestin is recommended because the progestin
A. Blocks the increased risk of myocardial infarction due to estrogen
B. Enhances the metabolic benefits of estrogen treatment
C. Reverses vulval atrophy occurring in post-menopausal women
D. Blocks the increased risk of endometrial carcinoma due to estrogen

342. A drug ‘X’ primarily reduces the static component of urinary obstruction
in benign hypertrophy of prostate and takes more than 3 months to exert its
beneficial effect. Which of the following is ‘X’?
A. Finasteride
B. Terazosin
C. Tamsulosin
D. Amphetamine

336. c 337. c 338. c 339. d 340. c 341. d 342. a


Endocrine System — Explanations
238. D. Colchicine
This patient’s acute onset pain and swelling of the right first metatarsophalangeal
joint and joint aspirate showing needle-shaped negatively birefringent crystals
indicate that he most likely has acute gouty arthritis. Colchicine is an effective anti-
inflammatory agent in acute gouty arthritis but commonly result in nausea, vomiting,
diarrhea and myopathy as adverse effects.
Thus, option D is correct. Topic Gout

239. A. Transdermal
Desmopressin acetate routes of administration:
Intranasally every 12-24 hours as needed for thirst and polyuria and central DI
Can be administered via metered-dose nasal inhaler containing 0.1 mL (10
mcg/spray) or via a calibrated rhinal tube
Oral desmopressin, 0.1- and 0.2 mg tablets, is given in a starting dose of 0.005 mg
twice daily
Sublingual desmopressin
Can be given also via intravenously, intramuscularly, or subcutaneously
Thus, option A is correct. Ref: CMDT: Nocturnal enuresis

240. B. Desmopressin used for both pituitary and renal diabetes insipidus
Desmopressin:
DI of pituitary (neurogenic) is the most important indication for vasopressin. It is
ineffective in renal (nephrogenic) DI. Since kidney is unresponsive to ADH.
Desmopressin is a drug of choice for central diabetes insipidus and not much
effective against nephrogenic diabetes insipidus
It can be used by intranasal route and oral route
Used in nocturnal enuresis in children and also in hemophilia and von Willebrand’s
Disease
In case of esophageal variceal bleeding:
Terlipressin is a synthetic analogue of vasopressin with longer activity and fewer side
effects. It reduces portal pressure and its effects are still significant 4 hours after
administration. So used in esophageal variceal bleeding
Desmopressin (DDAVP), a drug that significantly decrease bleeding time in cirrhosis,
has shown no clinical benefits in the setting of variceal bleeding.
In case of lithium induced diabetes insipidus:
Amiloride is the drug of choice: It blocks entry of Li+ through Na+ channels in the CD
cells and mitigates diabetes insipidus induced by lithium
So among, the options: Desmopressin used for both pituitary and renal diabetes
insipidus is a false statement, so it is the option you need to choose.
Thus, option B is correct. Ref: KD Tripathi, Topic: Diabetes insipidus

241. A. Ovarian cancer


Oral contraceptives contain estrogens and these can lead to more chances of migraine
and thromboembolism but because of progesterone component the chances of
ovarian and endometrial carcinoma decrease.
Thus, option A is correct. Ref: KDT, Topic: OCPs
242. C. Suppression of FSH and LH release
Oral contraceptive pills act by several mechanisms, most important of which is the
inhibition of ovulation due to suppression of LH and FSH.
Other mechanisms include disruption of proliferative and secretory phases of
endometrium and increase in viscosity of cervical mucus.
Thus, option C is correct. Ref: KDT, Topic: OCPs

243. A. Decreased FSH and LH secretion


Clomiphene citrate is an antagonist of estrogen in the peripheral tissues. Due to this
property, it can cause hot flushes.
Due to decreased action of estrogen, feedback pituitary inhibition is blunted resulting
in increased secretion of LH and FSH.
It can cause polycystic ovaries and multiple pregnancy (hyper stimulation syndrome).
Thus, option A is correct. Ref: KDT, Topic: SERMs

244. D. Osteoporosis
Estrogen is used for the treatment of osteoporosis in post-menopausal female.
Thus, option D is correct. Ref: KDT, Topic: Hormone replacement therapy

245. D. Reduce adult statue


Testosterone causes fusion of epiphyses and can result in reduction of adult stature
Thus, option D is correct. Ref: KDT, Topic: Sex hormones

246. A. Spermatogenesis
Testosterone [not DHT] is required for
F—Feedback inhabitation of LH secretion
I—Internal genitilia development
S—Spermatogenesis
H—Hematopoiesis
Thus, option A is correct. Ref: KDT, Topic: Sex hormones

247. A. Erosive esophagitis


Bisphosphonates (alendronate, risedronate etc.) are used for the prophylaxis and
treatment of osteoporosis. Esophageal toxicity is their distinct adverse effect.
Thus, option A is correct. Ref: KDT, Topic: osteoporosis

248. A. Thyroxine
Thyroid hormones and glucocorticoids increase the risk of osteoporosis whereas other
drugs mentioned in the options are used to treat osteoporosis.
Thus, option A is correct. Ref: KDT, Topic: drugs acting on Thyroid gland

249. B. Act as agonist in some tissues and antagonist in other tissues


SERMs are drugs with agonistic action on estrogen receptors in some tissues and
antagonistic action in other tissues, e.g. Tamoxifen, raloxifene
Thus, option B is correct. Ref: KDT, Topic: SERMs

250. A. Calcitriol
More active vitamin D preparation is Calcitriol
Thus, option A is correct. Ref: KDT, Topic: osteoporosis

251. A. Prednisone
Steroids result in osteoporosis on long term use
Thus, option A is correct. Ref: KDT, Topic: Steroids

252. A. Betamethasone
Steroids with long half-life like Betamethasone and dexamethasone cannot be used for
alternate day therapy because even in alternate day therapy there will be sufficient
blood levels of these steroids to cause suppression of HPA axis.
Thus, option A is correct. Ref: KDT, Topic: Steroids

253. A. Cortisol and fludrocortisone


Patient should be given both mineralocorticoids as well as glucocorticoid
Max. mineralocorticoid activity – Aldosterone
Max. glucocorticoid activity – Dexamethasone
Thus, option A is correct. Ref: KDT, Topic: Steroids

254. C. They do not cause Na+ and water retention


Triamcinolone, Betamethasone, Dexamethasone and Paramethasone are selective
glucocorticoids (have zero mineralocorticoid action).
These are preferred for cerebral edema because they lack salt and water retaining
potential mineralocorticoid action.
Thus, option C is correct. Ref: KDT, Topic: Steroids

255. D. No change/increase in prednisolone dose


During conditions of stress or injection, dose of steroids should not be decreased.
Rather, increase in dose may be required
Thus, option D is correct. Ref: KDT, Topic: Steroids

256. D. Osteoporosis
Glucocorticoids have lot of adverse effects on long term use. These can lead to
Cushing syndrome, hyperglycemia, osteoporosis, delayed wound healing, increased
susceptibility to infections, cataract, glaucoma and many other adverse effects.
Thus, option D is correct. Ref: KDT, Topic: Steroids

257. B. Suppression of ACTH secretion


In congenital adrenal hyperplasia, due to decreased formation of steroids, there is
decreased feedback inhibition of ACTH. Thus more ACTH is formed which leads to
adrenal hyperplasia. Thus exogenous steroids are given to suppress ACTH secretion.
Thus, option B is correct. Ref: KDT, Topic: Steroids

258. D. Congenital adrenal hyperplasia


Corticosteroids are used for the management of congenital adrenal hyperplasia.
Corticosteroids can result in hyperglycemia and thus may vitigate the control of blood
glucose in diabetics.
Due to retention of Na+ and water, corticosteroids can worsen the hypertension.
By inhibiting the production of gastro protective prostaglandins, corticosteroids
increase the risk of peptic ulcer disease.
Thus, option D is correct. Ref: KDT, Topic: Steroids

259. D. Prednisolone 20 mg/day oral for one year


If used for more than two weeks, corticosteroids can lead to HPA-axis suppression. If
discontinued abruptly, precipitation of acute adrenal insufficiency can result. This is
the most serious adverse effect seen with the use of corticosteroids that can cause
death of the patient.
Thus, option D is correct. Ref: KDT, Topic: Steroids

260. B. Lipocortin
Corticosteroids induce the synthesis of lipocortins that inhibit the enzyme
phospholipase A2.
Thus, option B is correct. Ref: KDT, Topic: Steroids

261. A. Collecting ducts


Aldosterone is the principal mineralocorticoid. It stimulates the reabsorption of Na+
and excretion of K+ and H+ by its action on late distal tubules and collecting ducts.
Thus, option A is correct. Ref: Katzung 14E/P 240 Topic : Steroids

262. A. Lactic acidosis


Biguanides like metformin and phenformin increase the risk of lactic acidosis
particularly in the patients with hepatic or renal disease. Both these drugs can cause
lactic acidosis although phenformin has more potential to cause this adverse effect
than metformin. Metformin is more likely to cause megaloblastic anemia than
phenformin.
Thus, option A is correct. Ref: KDT, Topic: Diabetes

263. C. Crystalline zinc insulin


Diabetic ketoacidosis must be managed by fast acting insulin preparations like
Regular insulin [crystalline zinc insulin]
Insulin lispro
Insulin aspart
Thus, option C is correct. Ref: KDT, Topic: Diabetes

264. D. Glipizide
Hypoglycemia is caused by the drugs that cause release of insulin. Two group of such
drugs are sulfonylureas (like chlorpropamide, glipizide etc.) and meglitinides (e.g.
Repaglinide)
Thus, option D is correct. Ref: KDT, Topic: Diabetes

265. A. They are used first in most cases of uncomplicated mild to moderate
type 2 diabetes
Thus, option A is correct. Ref: KDT, Topic: Diabetes

266. D. Taken just before a meal, it limits post prandial hyperglycemia in type
2 diabetes mellitus
Nateglinide and repaglinide are short acting oral hypoglycemic agents.
These are used to limit the post-prandial hyperglycemia.
Like sulfonylureas, these drugs also act by blocking K4 channels in the 0-cells of
pancreas that lead to depolarization and release of insulin.
Thus, option D is correct. Ref: KDT, Topic: Diabetes

267. C. Non diabetics


Metformin is the drug of choice for the treatment of obese diabetic patients, as it
causes weight loss.
It does not cause release of insulin, therefore less chances of hypoglycemia.
Thus, option C is correct. Ref: KDT, Topic: Diabetes

268. D. Type 1 diabetics


Sulfonylureas decrease blood glucose in diabetics as well as non-diabetics.
It requires at least 30% of functional beta cells for their action.
Insulin is the only treatment for type 1 diabetes.
Thus, option D is correct. Ref: KDT, Topic: Diabetes

269. C. It is less liable to cause lactic acidosis


Incidence of lactic acidosis is more with phenformin and that of megaloblastic anemia
(due to interference with vitamin B12 absorption) is more with metformin.
Thus, option C is correct. Ref: KDT, Topic: Diabetes

270. C. Is more potent


Second generation (like glipizide) sulfonylureas are more potent than first generation
agents (like chlorpropamide).
Chlorpropamide is the longest acting sulfonylurea.
Sulfonylureas can cause hypoglycemia (even in non-diabetics) due to release of
insulin.
Thus, option C is correct. Ref: KDT, Topic: Diabetes

271. D. Rapidly absorbed


Homan insulin has rapid absorption and shorter duration of action than pork or beef
insulin.
Thus, option D is correct. Ref: KDT, Topic: Diabetes

272. A. Enzymatic receptor


Insulin acts by stimulation of tyrosine kinase receptors.
Thus, option A is correct. Ref: KDT, Topic: Diabetes

273. A. An old man with severe bradycardia and hypotension resulting from
ingestion of overdose of atenolol
Glucagon is the antidote of 0-blocker poisoning. It acts by increasing cAMP in the
heart via stimulation of glucagon receptors. Cyclic AMP stimulatm the heart
Calcium gluconate can also be used for the treatment of B-blocker poisoning
Thus, option A is correct. Ref: KDT, Topic: Diabetes

274. C. Acarbose
It is an alpha-glucosidase inhibitor.
It inhibits the breakdown of complex carbohydrates to simple carbohydrates and thus
decreases their absorption.
Thus, option C is correct. Ref: KDT, Topic: Diabetes

275. B. Serum TSH level


TSH levels are elevated in a hypothyroid patient (because feedback inhibition by
thyroid hormones is not present).
Administration of thyroid hormones (T3 and T4) decreases TSH by contributing to
feedback inhibition.
If TSH levels are less than normal, it signifies overtreatment whereas high TSH level
suggests inadequate treatment.
Thus, option B is correct. Topic : Drugs acting on thyroid gland

276. B. Propylthiouracil
Iodides are commonly used to decrease the size and vascularity of thyroid gland
before surgery.
As this patient is pregnant, iodides and radioactive iodine is contra-indicated
Propylthiouracil is safe in pregnancy and is used for this purpose.
Thus, option B is correct. Ref: KDT, Topic: Drugs acting on thyroid gland

277. A. Has longer Half-life


Liothyronine (T3) as compared to thyroxine (T4) is:
Fast acting
More potent
Short half life
Indication of Liothyronine includes myxedema coma and for most of the indications
thyroxine is used because it has a long half-life thus can be used less frequently than
T3.
Thus, option A is correct. Ref: KDT, Topic: Drugs acting on thyroid gland

278. A. Elderly patients with ischemic heart disease


1131 is contra-indicated in children and young adults because if hypothyroidism
develops in response to radioactive iodine, it is permanent.
Use of radioactive iodine is contra-indicated in pregnancy.
In ischemic heart disease patients, surgery should be avoided. 113’ therapy is a good
alternative in such patients.
Thus, option A is correct. Ref: KDT, Topic: Drugs acting on thyroid gland

279. C. Propylthiouracil
For the peripheral conversion of T4 to T3, the enzyme needed is 5’-deiodinase. It is
inhibited by
Propylthiouracil
Propanolol
Amiodarone
Thus, option C is correct. Ref: KDT, Topic: Drugs acting on thyroid gland

280. D. All of the above


Propylthiouracil, Carbimazole and Methimazole act by inhibiting the enzyme, thyroid
peroxidase. It catalysis:
Oxidation of iodine
Organification
Coupling
Thus, option D is correct. Ref: KDT, Topic: Drugs acting on thyroid gland

281. D. Is shorter acting


T3 is the main active thyroid hormone. It is more potent (less plasma protein bound)
and faster acting than T4. However, short duration of action limits its use for the
treatment of hypothyroidism (requires lifelong treatment).
T3 is indicated for the treatment of myxedema coma.
Thus, option D is correct. Ref: KDT, Topic: Drugs acting on thyroid gland

282. D. Sodium iodide


Iodides inhibit the secretion of thyroid hormones in the circulation and therefore are
the fastest acting ant thyroid drugs.
Thyroid peroxidase inhibitors are delayed acting because their action manifests only
when already stored pool of thyroid hormones is utilized.
I131 takes more than 3 weeks to manifest their action.
Thus, option D is correct. Ref: KDT, Topic: Drugs acting on thyroid gland

283. C. Desmopressin
Drugs used for the treatment of central (pituitary) diabetes insipidus are:
Desmopressin (selective V2 agonist)
Thiazides
Chlorpropamide
Carbamazepine
Drugs used for the treatment of nephrogenic (renal) diabetes insipidus are:
Thiazides
Amiloride (for lithium induced)
Thiazides are useful for the treatment of both central as well as nephrogenic diabetes
insipidus.
Desmopressin is not effective in nephrogenic diabetes insipidus.
Thus, option C is correct. Ref: KDT, Topic: diabetes insipidus

284. A. Octreotide
Drugs useful in acromegaly are:
Bromocriptine and cabergoline
Somatostatin
Octreotide (long acting somatostatin analogue)
Pegvisomant (growth hormone receptor antagonist)
Thus, option A is correct. Ref: KDT, Topic: Pituitary hormones

285. D. Bromocriptine
Symptoms of the patient (amenorrhea, galactorrhoea and infertility) points towards
the diagnosis of hyperprolactinemia.
Bromocriptine is a D2 receptor agonist that can be used in the treatment of
hyperprolactinemia (dopamine acts as prolactin release inhibiting hormone in the
hypothalamus).
Psychosis occurs due to excessive stimulation of D2 receptors (D, blockers are used as
typical antipsychotic drugs) and bromocriptine can precipitate the symptoms in
predisposed patients.
Thus, option D is correct. Ref: KDT, Topic: Pituitary hormones

286. C. Bromocriptine
Hyperprolactinemia is caused by D2 blockers. All the drugs listed are D2 blockers
except bromocriptine which is a D2 receptor agonist. It is used in the treatment of
hyperprolactinemia.
Thus, option C is correct. Ref: KDT, Topic: Pituitary hormones

287. D. Immediately reduces gonadotropin secretion


GnRH agonists like leuprolide, goserelin and Nafarelin etc. are used by parenteral
route. Continuous administration of these agents decreases gonadotropin secretion
whereas pulsatile administration increases the secretion. When used continuously
even then first few doses cause increased secretion of gonadotropins (LH and FSH)
leading to flare up reaction in prostatic carcinoma.
GnRH antagonists like cetrorelix and ganirelix are also used by parenteral route but
these drugs immediately reduce gonadotropin secretion.
Thus, option D is correct. Ref: KDT, Topic: Pituitary hormones

288. B. Nafarelin
Nafarelin is a GnRH agonist. It has no role in acromegaly.
Thus, option B is correct. Ref: KDT, Topic: Pituitary hormones

289. B. Pioglitazone
Pioglitazone
Preclinical and clinical trial data, and results from an observational study suggest an
increased risk of bladder cancer in Pioglitazone users. The observational data further
suggest that the risk increases with duration of the age
Do not use in patients with active bladder cancer
Use caution when using in patients with a prior history of bladder cancer.
Thus, option B is correct.

290. D. Smoking
The risk of lactic acidosis in a patient taking metformin:
Advanced age (because of age-related decline in renal function and increased risk for
acute renal failure and other catastrophic medical conditions
Impaired lactate removal (in liver damage, which inhibits gluconeogenesis)
Increase the risk of lactic acidosis include severe dehydration, shock, alcohol use,
hypoxic states, sepsis and advanced age.
Metformin plasma levels > 5 ug/ml are generally found when metformin is implicated
as the cause of lactic acidosis
Such sustained very high elevations in plasma metformin concentrations usually are
observed in individuals with
Poor renal function (i.e., reduced metformin clearance)
Impaired hepatic metabolism (i.e., reduced lactate clearance)
And/or in the presence of increased production (i.e., sepsis, CHF, reduced
Thus, option D is correct. Ref: GG, Topic : Diabetes

291. C. Octreotide
A specific antidote for sulfonylurea-induced hypoglycemia
Octreotide
Suppress insulin release from beta islet cells, can act as a specific sulfonylurea
antidote
Can be administered either subcutaneously or intravenously to patients exhibiting
hypoglycemia following sulfonylurea poisoning
Thus, option C is correct. Ref: GG, Topic : Diabetes
292. B. Hypokalemia
In diabetic ketoacidosis
Up to 400 mEq of K+ may be lost in urine during ketoacidosis,
Serum K+ is usually normal due to exchange with intercellular stores
When insulin therapy is instituted ketosis subsides
K+ is driven back intracellularly leading to dangerous hypokalemia
After 4 hours it is appropriate to add 10-20 mEq/hr KCI to the i.v. fluid
The further rate of infusion is guided by serum K+ measurements and ECG
Thus, option B is correct. Ref: KDT, Topic: Diabetes

293. B. 30
U-40 VIAL contains 40 units of insulin
U-100 vials contain 100 units of insulin
1ml contains 40 units of U40 syringe
So, 40 units in U 40 syringe = 100 units in U 100 syringe
You need to give 12 units of U 40 in U 100 syringe. So, 12 divisions in U 40=
100/40x12=
30 divisions in U 100 syringe
Simply, remember when you want to give U 40 insulin from U 100 syringe multiple
by 2.5
When you want to give U 40 insulin from U 40 syringe multiply by 0.4
Thus, option B is correct. Ref: KDT, Topic: Diabetes

294. B. Duration of Action


Insulin:
Insulin is a short-acting, to make it long-acting
Insulin molecules are made to form self-aggregate hexamers around zinc ions.
After injection, insulin monomers are released gradually by dilution, so that
absorption occurs slowly
Peak action is produced only after 2-3 hours and action continues up to 6-8 hours
Thus, option B is correct. Ref: KDT, Topic: Diabetes

295. C. Large particles in Lente insulin is crystalline and practically soluble in


water
Lente Insulin
Intermediate-acting insulin
7:3 mixtures of ultralente (crystalline) and semilente (amorphous)
Large particles in Lente insulin are crystalline and practically insoluble in water
Also known as Insulin-Zinc suspension
Thus, option C is correct. Ref: KDT, Topic: Diabetes

296. D. The first proteins to be phosphorylated by the activated receptor


tyrosine kinases are the MAP kinase & protein phosphatases
Insulin Receptor:
Are tyrosine kinase receptors
Consists of two covalently linked heterodimers each containing a & b subunit
A subunit entirely extracellular, recognition site & b subunit that spans the
membrane, contains a tyrosine kinase
Binding of insulin to a subunit activates the receptor
The first proteins to be phosphorylated receptor activated receptor tyrosine kinases
are the docking proteins, insulin receptor substrate-1
Thus, option D is correct. Ref: KAT, Topic: Diabetes

297. B. Linagliptin
Linagliptin is approved as FDC with empagliflozin recently.
Thus, option B is correct.

298. D. Complete blood count


Methimazole:
Agranulocytosis usually occurs during the first few weeks or months of therapy but
may occur later.
Because agranulocytosis can develop rapidly periodic white cell counts usually are of
little help, although a baseline white blood cell count and differential should be
obtained before antithyroid drug treatment is initiated
Patients should be instructed to immediately report the development of a sore throat
or fever, which are often signs of the presence of leucopenia
If these signs or symptoms occur, patients should discontinue their anti-thyroid drug
and obtain a granulocyte count
Agranulocytosis is reversible upon discontinuation of the offending drug
Administration of recombinant human granulocyte colony-stimulating factor may
hasten recovery

299. B. Amiodarone
Iodothyronine deiodinase (D) which exists in 3 forms
D1
D2
D3
These forms differ in their organ and cellular localization, as well as product, formed.
(type 1) D1 form generates both T3 and Rt4
Type 2 deiodinase (D2) generates T3
D3 generates reverse T3
Propylthiouracil inhibits Type 1 deiodinase
Amiodarone inhibits both D1 and D2 forms
Propranolol (high dose) and glucocorticosteroids also inhibit peripheral conversion of
T4 and T3 (except in brain and in pituitary)
Thus, option B is correct. Ref: KDT, Topic: thyroid

300. C. Has a relatively short plasma half-life, so toxicity is less


Drug of choice for Graves’ disease is Methimazole because
Methimazole
Effective when given as a single daily dose
Improved patient adherence
less toxic than propylthiouracil
has a relatively long plasma and intrathyroidal t1/2
the long duration of action
the usual starting dose for Methimazole is 15-40 mg per day
Thus, option C is correct. Ref: GG, Topic: thyroid

301. B. Increased
Hypothyroidism and Pregnancy
Due to the increased serum concentration of TBG induced by estrogen, the
expression of D3 by the placenta
There will be a small amount of transplacental passage of L-T4 from mother to fetus
Overt hypothyroidism during pregnancy is associated with an increased risk of
miscarriage, fetal distress, preterm delivery and impaired psychoneural and motor
development in the progeny
Clinical implication:
As a part of prepregnancy planning, the dose of levothyroxine should be adjusted to
maintain the TSH in the lower portion of the reference range. Women should increase
their levothyroxine dose by sour 30% as soon as pregnancy is confirmed, thus
anticipating the increased need.
Thus, option B is correct. Ref: GG, Topic: thyroid

302. D. Oral bioavailability of I-thyroxine is – 25%Note: Ref: KDT 7E/P 250


I-thyroxine:
Oral bioavailability of I-thyroxine is – 75
To be taken in empty stomach to avoid food interaction
Sucralfate, iron, calcium and proton pump inhibitors also reduce I-thyroxine
absorption
CYP3A3 inducers; ole rifampin, phenytoin and carbamazepine increase metabolism of
T4.
Thus, option D is correct. Ref: GG, Topic: thyroid

303. C. Both A & B


Thyrotropin alpha is used for
Stimulation of radioiodine uptake and thyroglobulin release in patients with thyroid
cancer after thyroidectomy
Prepare patients for radioiodine ablation of thyroid remnants after thyroidectomy for
thyroid cancer.
Thus, option C is correct. Ref: GG, Topic: thyroid

304. B. Teriparatide
Teriparatide
Is a recombinant PTH 1-34
PTH, when given in low and pulsatile dose, stimulates bone formation whereas
Continuous administration causes resorption of bones
By intermittent s.c. administration Teriparatide is used in the treatment of
osteoporosis
So it acts by increasing bone formation not by decreasing bone resorption
Thus, option B is correct. Ref: GG, Topic : Osteoprorosis

305. D. Paget’s disease


Calcitonin:
Is effective in disorders of increased skeletal remodelling, such as Paget disease, and
in some patients with osteoporosis
For Paget disease, calcitonin generally is administered by subcutaneous injection
because intranasal delivery is relatively ineffective owing to limited bioavailability
Intranasal route is used in osteoporosis
Thus, option D is correct. Ref: GG, Topic : Osteoporosis
306. C. Both A & B
Oral strontium ranelate:
Is an alternative oral treatment, belonging to a class of drug called ‘dual action bone
agents’ (DABAs)
Proven efficacy, especially in the prevention of vertebral fracture.
Mechanism of action: collagen & noncollagen protein, synthesis, enhances
preosteoblast differentiation, osteoclast function
Dosage: 2 g oral suspension daily
Adverse effects: Thromboembolism
Thus, option C is correct. Ref: KDT, Topic: Osteoporosis

307. B. Zolendronate
Zolendronate
Osteonecrosis of the jaw is a rare complication of i.v. high dose Bisphosphonate
therapy.
Thus, option B is correct. Ref: KDT, Topic: Osteoporosis

308. D. Risedronate
Bisphosphonates classification and their relative potency
First generation BPNs
Etidronate 1
Tiludronate 10
Second generation BPNs
Pamidronate 100
Alendronate 100-500
Ibandronate 500-1000
Third generation BPNs
Risedronate 1000
Zoledronate 5000 (Overall highest potency is zoledronate)
Thus, option D is correct. Ref: KDT, Topic: Osteoporosis

309. C. Hydrochlorothiazide
List of drugs increasing and decreasing bone resorption

Resorpation Resorpation

Corticosteroids Androgens/Estrogens

Parathormone Calcitonin

Thyroxine (excess) Growth Harmone

Hypervitaminosis D Bisphosphonates

Prostaglandin E2 Fluoride

Interleukin 1 & 6 Gallium Nitrate

Alcoholism Mithramycin

Loop diuretics Thiazide diuretics

Thus, option C is correct. Ref: KDT, Topic: Osteoporosis

310. B. Increased growth of hairs


Adverse effects were seen on use of topical corticosteroids
Thinning of epidermis
Dermal changes
Atrophy Telangiectasia, Striae Easy Bruising Hypopigmentation
Delayed wound healing
Fungal and bacterial infections
Thus, option B is correct. Ref: KDT, Topic: steroids

311. D. Give the dose divided into three times daily so that peak steroid
concentration is reduced
Measures that minimize HPA axis suppression are:
Use shorter acting steroids at the lowest possible dose
Use steroids for the shortest periods of time possible
Give the entire daily dose at one time in the morning
A moderate dose of a short-acting steroid (e.g. prednisolone) given a t 48 hr interval
did not cause HPA suppression, whereas the same total amount given in 4 divided 12
hourly doses produced marked HPA suppression
Switch to alternate-day therapy if possible
Use topical or local steroid preparations wherever possible
Ex: dermal, inhaled, ocular, nasal, rectal intrasynovial route.
Thus, option D is correct. Ref: KDT, Topic: steroids

312. A. Inhibit production of surfactant


Corticosteroids:
Increase the number of RBCs, platelets and neutrophils in circulation but decrease
the concentration of lymphocytes in circulation
Inhibit function of macrophages
Increase production of surfactant, so used in premature infants for lung maturity
By inhibiting PLA 2 they reduce the synthesis of prostaglandins
Thus, option A is correct. Ref: KDT, Topic: steroids

313. C. Prednisolone

Thus, option C is correct. Ref: KDT, Topic: steroids

314. D. Hyperkalemic alkalosis


Adverse effects are seen with use mineralocorticoids
Sodium and water retention
Oedema
Hypokalemic alkalosis
The progressive rise in BP
The gradual rise in BP occurs due to excess glucocorticoid as well
Thus, option D is correct. Ref: KDT, Topic: steroids

315. C. Elagolix
Elagolix:
GnRH receptor antagonist
Approved for the management of moderate to severe pain associated with
endometriosis
Thus, option C is correct.

316. D. Norgestrel 0.5 mg Ethinylestradiol 30 µg


MALA D contains
Norgestrel 0.3mg Ethinylestradiol 30 µg
Thus, option D is correct. Ref: KDT, Topic: contraceptives

317. C. Desogestrel
Emergency contraceptives:
Levonorgestrel
Emergency (post-coital) contraceptive
OCPs (estrogen + Progesterone)
Mifepristone single dose
Within 72 hours of intercourse
Ulipristal
Progesterone agonist/antagonist
Emergency contraceptive
Long-acting contraceptive methods
Norplant and DMPA
Thus, option C is correct. Ref: KDT, Topic: contraceptives

318. B. Pure progesterone antagonist


Onapriston:
Is a pure progesterone antagonist
Mifepristone:
Is a selective progesterone receptor modulator
Is not a pure antagonist
In the presence of progesterone, it acts as a competitive receptor antagonist of
progesterone, but in the absence of progesterone it acts as a partial agonist
Ineffective against mineralocorticoids
Has no estrogenic side effect
Has glucocorticoid and androgen receptor blocking action
Thus, option B is correct. Ref: KDT, Topic: contraceptives

319. C. Tamoxifen
Tamoxifen:
Is the only drug approved for primary as well as metastatic breast carcinoma in
premenopausal women
It is also effective in surgically treated cancer in the male breast
Thus, option C is correct. Ref: KDT, Topic: SERMs

320. D. All of the above


Clomiphene is for infertility due to the failure of ovulation:
50 mg once daily for 5 days starting from the 5th day of the cycle
Treatment is given monthly
If 2-3 months’ treatment does not result in conception, then the daily dose may be
doubled for 2-3 months
No more than 6 treatment cycles should be tried.
Thus, option D is correct. Ref: KDT, Topic: SERMs

321. D. Carcinoma of gallbladder


Women receiving estrogen therapy have an increased risk of developing
Benign Hepatoma
Carcinoma of vagina & cervix
Postmenopausal
Endometrial cancer
Premenopausal
Ca. Breast
Gall Stones
Coronary & thrombo-embolic disorders
Decrease DL & Increase HDL & TG
Hyperglycemia
Thus, option D is correct. Ref: KDT, Topic: contraceptives

322. C. Goserelin also approved for ovarian cancer


Sildenafil:
Used in pulmonary hypertension
Male erectile dysfunction
Finasteride:
Benign prostatic hyperplasia
Approved in male pattern baldness
Flutamide:
Used in Hirsutism in females
Adjuvant treatment of metastatic prostate
Cancer
Goserelin
Approved for breast cancer, dysfunction uterine bleeding and endometriosis
Thus, option C is correct. Ref: GG, Topic : Pituitary hormones

323. B. Impotence is not seen with the use of these drugs


5alpha-Reductase inhibitors:
5a-Reductase converts testosterone to dihydrotestosterone
Finasteride antagonist of 5a-reductase, especially the type II
Both agents like the conversion of testosterone to dihydrotestosterone, especially in
the male external genitalia.
Use:
Benign prostatic hyperplasia
Serum and prostatic concentrations of dihydrotestosterone decrease prostatic volume
decreases, and urine flow rate increases
Finasteride also is approved for use in the treatment of male pattern baldness and
Hirsutism
Finasteride is an effective flutamide and the combination of estrogen and
cyproterone in the treatment of Hirsutism
Adverse Effects:
Impotence is a well-documented, albeit infrequent side effect of this use
Mechanism unknown
Gynecomastia is an even less common side effect
Thus, option B is correct. Ref: GG, Topic 5 alpha reductase inhibitors

324. B. Megestrol
Anti-androgen is classified
Steroidal:
Cyproterone
Megestrol
Nonsteroidal,
Flutamide
Bicalutamide
Nilutamide
Mechanism of Action of Nonsteroidal Anti-Androgens.
The nonsteroidal anti-androgens are taken orally and inhibit ligand binding and
consequent Androgen Receptor translocation from the cytoplasm to the nucleus.
Thus, option B is correct. Ref: GG, topic: Antiandrogens

325. B. The agonistic action of Methylergometrine on dopamine receptors


Methylergometrine and ergometrine:
Can decrease milk secretion if higher doses are used for many days postpartum
This is due to inhabitation of prolactin release (dopamine action)
Thus, option B is correct. Ref: KDT, Topic: Oxytocics

326. C. Tesamorelin
GnRH agonists:
Buserelin (nasal)
Goserelin
Leuprolide
Nafarelin (nasal route)
TriptoreliDeslorelin
Tesamorelin:
A modified form of human GnRH
Resistant to degradation by dipeptidyl peptidase 4
Has a prolonged duration of action
Able to increase the levels of GH and IGF-1
FDA-approved for treatment of HIV-associated lipodystrophy but not for GH deficiency
Thus, option C is correct. Ref: GG, Topic: pituitary hormones

327. C. Both a and b


Cabergoline
Has been linked to valvar heart disease
An effect proposed to reflect agonist activity at the serotonin 5-HT2B receptor
Pergolide:
Has been linked to valvar heart disease
In part due to concerns of valvar heart disease, pergolide has been withdrawn from
the market.
Echocardiography assessment seems appropriate for patients receiving chronic
therapy with carbolic and pergolide, particularly those on higher doses.
Thus, option C is correct. Ref: GG, Topic : pituitary hormones

328. A. Acute Stomatitis


Octreotide:
Long-acting somatostatin analogue
Uses:
Acute bleeding due to oesophagal varices
Acromegaly
Islet cell tumours
Secretory diarrhoea in carcinoid syndrome and VIPoma
Diarrhoea due to vagotomy, short bowel syndrome and AIDS.
Treatment and prophylaxis of acute pancreatitis
Thus, option A is correct. Ref: GG, Topic: pituitary hormones

329. B. Diagnostic imaging of osteosarcoma


Octreoscan:
Are modified forms of octreotide labelled within indium or technetium
It has been used for diagnostic imaging of neuroendocrine tumors such as pituitary
adenomas and carcinoids.
Thus, option B is correct. Ref: GG, Topic : pituitary hormones

330. D. HIV-associated lipodystrophy


Tesamorelin:
A modified form of human GHRH
Resistant to degradation by dipeptidyl peptidase 4
Has a prolonged duration of action
Able to increase the levels of GH and IGF-1
FDA approved for treatment of HIV-associated lipodystrophy but not for GH deficiency
Thus, option D is correct. Ref: GG, Topic : pituitary hormones

331. B. It is a GLP-1 agonist


GLP-1 agonists:
Incretins increase glucose-stimulated insulin secretion
Drugs which are GLP-1 receptor agonist or enhance endogenous GLP-1 activity are
approved for the treatment of type 2 DM
Agents in this class do not cause hypoglycemia because of the glucose-dependent
nature of incretin stimulated insulin secretion
Exenatide, a synthetic version of a peptide initially identified in the saliva of the Gila
moinster (exendin-4) is an analogue of GLP-1
Unlike native GLP-1, which has a half-life of >5 min, differences in the exenatide
amino acid sequence render it resistant to the enzyme that degrades GLP-1
(dipeptidyl peptidase IV [DPP-IV])
Thus, exantide has prolonged GLP-1 like action and binds to GLP-1 receptors found in
islets, the gastrointestinal tract, and the brain
Liraglutide, another GLP-1 receptor agonist is almost identical to native GLP-1 except
for an amino acid substitution and the addition of a fatty acyl group (coupled with a
y-glutamic acid spacer) that promote binding to albumin and plasma proteins and
prolong its half-life.
GLP-1 receptor agonists increase
Glucose-stimulated insulin secretion, suppress glucagon and slow gastric emptying.
These agents do not promote weight gain: in fact, most patients experience modest
weight loss and appetite suppression
Treatment with these agents should start at a low dose to minimize initial side effects
(nausea being the limiting one), GLP-1 receptor agonists, available in twice.
Thus, option B is correct. Ref: Harrison, Topic Diabetes

332. D. Pramlintide can be given in patients with gastroparesis


Pramlindte:
Acts through the amylin receptor in specific regions of the hindbrain
Amylin receptor reduces glucagon secretion, delays gastric emptying, and
Fosters a feeling of safety
Administered as a subcutaneous injection prior to meals
Adverse Effects:
Nausea and Hypoglycemia (MC)
Pramlindte alone does not lower blood glucose, addition to insulin at mealtimes can
cause increased rates of hypoglycemia, occasionally severe.
Because of its effects on GI motality
Pramlindte is contradicted in patients with gastroparesis or other disorders of
motality
Pregnancy category C drug
Can be used in persons with moderate renal dieseas (creatinine clearance >
20ml/min)
Thertapeutic Uses:
Approved for treatment of types 1 and 2 diabetes as an adjunct in patients who
take insulin with meals.
Thus, option D is correct. Ref: GG

333. B. Pioglitazone
Pioglitazone
Preclinical and clinical trial data, and results from an observational study suggest an
increased risk of bladder cancer in Pioglitazone users. The observational data further
suggest that the risk increases with duration of the age
Do not use in patients with active bladder cancer
Use caution when using in patients with a prior history of bladder cancer.
Thus, option B is correct.

334. D. Prednisolone
Osteoporosis is a common cause of pathological vertebral fractures. Chronic systemic
use of corticosteroids like prednisolone promotes osteoporosis and therefore may
cause such fractures.
Thus, option D is correct. Ref: KK Sharma 2E/P 573 Topic : Steroids
335. A. Prednisolone
Prednisolone (corticosteroids) act through cytoplasmic receptors whereas thyroid
hormones (Ty T4), retinoids (vitamin A) and thiazolidinedione (like rosiglitazone) have
nuclear receptors.
Thus, option A is correct. Ref: KDT, Topic:

336. C. Osteoporosis
Teriparatide is a recombinant PTH having first 34 amino acids. It can be used for the
treatment of osteoporosis.
Thus, option C is correct. Ref: KDT, Topic: Steroids

337. C. Diabetes mellitus


Exenatide is a glucagon like peptide analogue which is proposed to be used in the
treatment of post prandial hyperglycemia.
Thus, option C is correct. Ref: Katzung 14E/P 701 Topic Diabetes

338. C. Ritodrine
Isoxsuprine and ritodrine are selective 132 agonists useful as tocolytic agents for
arresting labour. Pulmonary edema is an important adverse effect of these agents.
Thus, option C is correct. Ref: KDT, Topic: tocolytics

339. D. Take 2 pills the next day and continue with the course
If a woman misses one pill of OCP, she should take 2 pills next day and then continue
one pill a day as usual.
If the pills have been missed for 2-3 days, then the course should be stopped,
mechanical barriers (like condom) should be used and the next course should start
from the 5th day of menses as usual.
Thus, option D is correct. Ref: KDT, Topic: contraceptives

340. C. Mifepristone
Single 600 mg dose of mifepristone taken within 72 hours of unprotected intercourse
is an effective method to prevent pregnancy.
Two tablets of OCP within 72 hours of unprotected intercourse followed by 2 tablets
after 12 hours can also be used as a post-coital contraceptive method.
Two tablets of levonorgestrel (0 75 mg each) taken 24 hours apart can also be used
for emergency contraception.
Thus, option C is correct. Ref: KDT, Topic: contraceptives

341. D. Blocks the increased risk of endometrial carcinoma due to estrogen


Use of estrogen for the treatment or prevention of post-menopausal osteoporosis
places a patient at the risk of breast and endometrial carcinoma. Progesterone is used
to decrease the risk of endometrial carcinoma.
Thus, option D is correct. Ref: KDT, Topic: Hormone replacement therapy

342. A. Finasteride
5-a-reductase inhibitors are used to reduce the static component of urinary
obstruction in BHP. It is delayed acting and takes more than 3 months to exert its
beneficial effect.
Selective a, blockers are used to relieve the dynamic obstruction in BHP. These
provide rapid symptomatic relief.
Thus, option A is correct. Ref: KDT, Topic: 5 alpha reductase inhibitors
Anaesthesia — Questions
343. Ortho-toluidine is an oxidizing agent and it is a metabolite of
A. Procaine
B. Prilocaine
C. Lignocaine
D. Tetracaine

344. Drug used for ‘narcoanalysis’ of criminals is


A. Methohexitone
B. Propofol
C. Thiopentone
D. Ketamine

345. A patient, Tina was anesthetized with halothane and nitrous oxide and
tubocurarine was used for skeletal muscle relaxation. She became
hypertensive along with marked muscle rigidity and hyperthermia. Lab
reports showed that she has developed hyperkalemia and acidosis. This
complication was caused by
A. Block of autonomic ganglia by tubocurarine
B. Excessive release of calcium from the sarcoplasmic reticulum
C. Activation of brain dopamine receptors by halothane
D. Pheochromocytoma

346. A young man having pheochromocytoma has BP of 188/92 mm Hg and a


hematocrit of around 50%. Pulmonary function tests and renal functions are
normal. His catecholamines are elevated. Which of the following drugs
should not be included in the anesthesia protocol?
A. Desflurane
B. Fentanyl
C. Midazolam
D. Halothane

347. Postoperative vomiting is uncommon with this intravenous anesthetic


agent and patients are able to ambulate sooner than those who receive
other anesthetic agents
A. Propofol
B. Enflurane
C. Ketamine
D. Remifentanil

348. If ketamine is the only agent used in reducing a dislocated shoulder. Its
actions will include
A. Hypotension
B. Bradycardia
C. Analgesia
D. Respiratory depression

349. Ketamine is the preferred anesthetic for the following Except


A. Short operations on asthmatics
B. Trauma cases that have bled significantly
C. Burn dressing
D. Hypertensive

350. Dissociative anesthesia is produced by


A. Propofol
B. Fentanyl
C. Ketamine
D. Both (a) and (b) are correct

351. Malignant hyperthermia is a rare complication of the use of the following


anesthetic
A. Halothane
B. Thiopentone sodium
C. Ketamine
D. Ether

352. As a general anesthetic, halothane has the following advantages EXCEPT


A. Reasonably rapid induction of anesthesia
B. Non-inflammable and non-explosive
C. Very good analgesic action
D. Pleasant and non-irritating

343. 344. c 345. 346. 347. 348. c 349. 350. c 351. a 352. c
b b d a d

353. Ether is still used as a general anesthetic in India. Specially in peripheral


hospitals because:
A. It is nonexplosive
B. It is cheap and can be administered without anesthetic machine
C. It induces anesthesia rapidly
D. It is pleasant smelling and non-irritating

354. Which of the following general anesthetics has poor muscle relaxant
action?
A. Ether
B. Isoflurane
C. Halothane
D. Nitrous oxide

355. Second gas effect is exerted by which of the following gas when
coadministered with halothane
A. Helium
B. Cyclopropane
C. Nitrogen
D. Nitrous oxide

356. Which general anesthetic selectively inhibits excitatory NMDA receptors


A. Ketamine
B. Halothane
C. Desflurane
D. Thiopentone

357. The minimal alveolar concentration of an inhalational anesthetic is a


measure of its
A. Diffusibility
B. Therapeutic Index
C. Potency
D. Ketamine

358. While performing a rapid sequence intubation in the operation theatre, a


patient was given a standard intravenous dose of muscle relaxant “A”. To
maintain the muscle relaxation during surgery, another muscle relaxant
Vecuronium was given. At the end of the surgery, neostigmine was used to
reverse the residual muscle relaxation. However, the patient did not
respond and continued to display too much muscle paralysis to permit safe
extubation. Drug A was most likely to be?
A. Pancuronium
B. Tubocurarine
C. Midazolam
D. Succinylcholine

359. Which of the following drugs is hydrolyzed by a plasma esterase that is a


normally low in activity in about 1 in every 2500 humans?
A. Succinylcholine
B. Rifampicin
C. Cimetidine
D. Ethanol

360. Patient undergoing surgery was given a muscle relaxant. It produced


marked fall in B. P. and increase in airway resistance which were reversed
with diphenhydramine. The muscle relaxant was most probably
A. Tubocurarine
B. Diazepam
C. Atracurium
D. Vecuronium

361. 0ne of the following statements about succinylcholine is true


A. It is the drug of choice in non-traumatic rhabdomyolysis
B. It has a long duration of action
C. It may induce life threatening hyperkalemia
D. It is useful in patients with spinal cord injuries with paraplegia

362. Which of the following is a skeletal muscle relaxant that acts as a central
a2 adrenergic agonist.
A. Chlormezanone
B. Brimonidine
C. Tizanidine
D. Quinine

363. Dantrolene sodium reduces skeletal muscle tone by


A. Reducing Ca2+ release from sarcoplasmic reticulum in the muscle fibre
B. Suppressing spinal polysynaptic reflexes
C. Inhibiting the generation of muscle action potential
D. Reducing acetylcholine release from motor nerve endings
353. 354. 355. 356. 357. c 358. 359. 360. a 361. c 362. c
b d d a d d

363.
a

364. The following antibiotic accentuates the neuromuscular blockade


produced by Pancuronium
A. Penicillin G
B. Erythromycin
C. Streptomycin
D. Chloramphenicol

365. The most rapidly acting nondepolarizing neuromuscular blocking agent


which can be used as an alternative to succinylcholine for tracheal
intubation is
A. Pipecuronium
B. Pancuronium
C. Doxacurium
D. Rocuronium

366. The neuromuscular blocker that does not need reversal of action by
neostigmine at the end of the operation is
A. d-Tubocurarine
B. Mivacurium
C. Pipecuronium
D. Doxacurium

367. Pancuronium differs from tubocurarine in that


A. It can cause rise in BP on rapid I. V. injection
B. Its action is not reversed by neostigmine
C. It is a depolarizing blocker
D. It causes marked histamine release

368. The fall in blood pressure caused by d-tubocurarine is due to


A. Reduced venous return
B. Ganglionic blockade
C. Histamine release
D. All of the above

369. Which of the following drugs have a high surface activity and
vasoconstrictor actions that reduce bleeding in mucus membranes?
A. Bupivacaine
B. Procaine
C. Lidocaine
D. Cocaine

370. Ram has a 4 ml lignocaine vial of 2% solution. How much lignocaine is


present in 1 ml?
A. 20 mg
B. 8mg
C. 2mg
D. 200 mg

371. AU of the following are properties of local anesthetics EXCEPT


A. Blockade of voltage dependent Na+ channels
B. Increase in the membrane refractory period
C. Slowing of axonal impulse conduction
D. Preferential binding to resting channels

372. Epinephrine added to a solution of lignocaine for a peripheral nerve block


will
A. Increase risk of convulsions
B. None of these
C. Both (a) and (b)
D. Increase the duration of action of the local anesthetic

373. An agent added to local anesthetic to speed the onset of action is


A. Methylparapben
B. Adrenaline
C. Fentanyl
D. Bicarbonate

374. A patient receives a toxic dose of lignocaine I. v.; the patient is likely to
exhibit.
A. Seizures and coma
B. Mydriasis and diarrhea
C. Respiratory paralysis
D. Excessive salivation
375. Intravenous regional anesthesia is suitable for
A. Head and neck surgery
B. Vascular surgery on the lower limb
C. Orthopedic manipulation on the upper limb
D. Caesarian section

364. c 365. 366. 367. 368. 369. 370. 371. d 372. d 373. d
d b a d d a

374. 375. c
a

376. The duration of spinal anesthesia depends on all of the following EXCEPT
A. Local anesthetic that is used
B. Concentration of the local anesthetic used
C. Whether adrenaline has been added to the local anesthetic
D. Posture of the patient

377. In spinal anesthesia the segmental level of


A. Sympathetic block is lower than the sensory block
B. Sympathetic, motor and sensory block has the same level
C. Motor block is higher than the sensory block
D. Sympathetic block is higher than the sensory block

378. The segmental level of spinal anesthesia depends on


A. Volume of the local anesthetic injected
B. Specific gravity of the local anesthetic solution
C. Posture of the patient
D. All of the above

379. Eutectic lignocaine-prilocaine has the following unique property


A. It causes motor blockade without sensory block
B. It has strong vasoconstrictor action
C. It is not absorbed after surface application
D. By surface application, it can anaesthetize unbroken skin

380. The local anesthetic with the longest duration of action is


A. Dibucaine
B. Chlorprocaine
C. Lignocaine
D. Procaine

381. The following local anesthetic raises BP instead of tending to cause a fall
A. Lignocaine
B. Dibucaine
C. Cocaine
D. Procaine
382. Which of the following statements is not true of local anesthetics?
A. The local anesthetic binds to its receptor mainly when the Na+ channel is in the
resting state
B. me local anesthetic approaches its receptor only from the in neuronal face of the
Na+ channel
C. The local anesthetic is required in the unionized form for penetrating the
neuronal membrane
D. The local anesthetic combines with its receptor in the ionized cationic form

383. A patient is having a malignancy and has been suffering from severe
pain. Which of the following opioid analgesics can be used as transdermal
patch for alleviation of pain in him?
A. Fentanyl
B. Pentazocine
C. Morphine
D. Tramadol

384. Which of the following drugs is a full agonist at opioid receptors, has
excellent oral bioavailability, analgesic equipotency to morphine and a
longer duration of action with milder withdrawal symptoms on abrupt
discontinuation?
A. Methadone
B. Hydromorphone
C. Fentanyl
D. Nalbuphine

385. Which of the following drugs does not activate opioid receptors, has been
proposed as a drug in the management of opioid addiction and with just a
single dose blocks the action of injected heroin for up to 48 hours?
A. Amphetamine
B. Naltrexone
C. Methadone
D. Buspirone

386. Drug used for ‘narcoanalysis’ of criminals is


A. Methohexitone
B. Propofol
C. Thiopentone
D. Ketamine
376. 377. 378. 379. 380. 381. c 382. 383. a 384. a 385. b
d d d d a a

386. c

387. Which of the following centrally acting skeletal muscle relaxant is GABA
mimetic
A. Mephenesin
B. Thiocolchocoside
C. Chlorzoxazone
D. Methocarbamol

388. Sugamadex is developer for the reversal of


A. Pancuronium
B. Vecuronium
C. Pipecuronium
D. Succinyl choline

389. Which of the following is longest acting non-depolarizing skeletal muscle


relaxant
A. Metocurine
B. Pipecuronium
C. Pancuronium
D. Doxacurium

390. Which of the following is shortest acting depolarizing skeletal muscle


relaxant
A. Mivacurium
B. Ganatacurium
C. Succinylcholine
D. Rocuronium

391. Following drugs are metabolized by pseudocholinestrase except


A. Succinylcholine
B. Amlodipine
C. Mivacurium
D. Clevidipine

392. Following concentrations of adrenaline are used alone with Local


Anesthetic drug during Local anesthesia except
A. 1:50,000
B. 1:2,50,000
C. 1:1,00,000
D. 1:2,00,000

393. Eutectic Mixture of Local Anesthetics contain


A. 2.5% lidocaine and 2.5% procaine
B. 2.5% lidocaine and 2.5% procaine
C. 5% lidocaine and 2.5% prilocaine
D. 2.5% lidocaine and 5% procaine

394. Highest rate of liver metabolism is seen with following amide local
anesthetic
A. Prilocaine
B. Bupivacaine
C. Levobupivacaine
D. Procaine

395. Which of the following local anesthetic is more water soluble among the
following
A. Tetracaine
B. Rupivacaine
C. Lidocaine
D. Rupovacaine

396. Which of the following is an ester local anesthetic


A. Lidocaine
B. Mepivacaine
C. Benzocaine
D. Bupivacaine

397. Which nerve fiber type is least susceptible to Local Anesthetic


A. A Alpha
B. C type
C. A delta
D. B type

398. A 32-year-old male is known hypertensive and is being planned for


cholecystectomy. Which of the following anesthetic agents is
contraindicated in this person?
A. Propofol
B. Midazolam
C. Etomidate
D. Ketamine

399. Drug contraindicated in head injury with raised ICP is


A. Ketamine
B. Thiopentone
C. Methohexitone
D. Propofol

400. Pain on injection due to Propofol can be minimized by


A. Use of Lidocaine
B. Use of larger arm
C. Use of antecubital veins
D. All of the above

401. Which of the following inhaled anesthetics has highest extent of hepatic
metabolism
A. Sevoflurane
B. Isoflurane
C. Enflurane
D. Desflurane
387. 388. 389. 390. c 391. 392. 393. 394. a 395. c 396. c
b b d b b a

397. 398. 399. 400. 401. c


b d a d

402. Which of the following inhalational anesthetic is not a volatile liquid


A. Ether
B. Halothane
C. Isoflurane
D. Nitrous oxide

403. Intravenous anesthetic shown to be effective in depression is


A. Diazepam
B. Ketamine
C. Thiopentone
D. Etomidate

404. Drug having agonist action at the TREK channel is


A. CNO
B. Xenon
C. Halothane
D. Sevoflurane

405. Caffeine-halothane contracture test is used for


A. Detection of susceptibility to halothane induced hepatitis
B. Detection of halothane induced uterine relaxation
C. Detection of generic susceptibility to halothane induced malignant hyperthermia
D. None of the above

406. Which of the following is incorrect about Remifentanil


A. Useful for short painful procedures
B. Administered by intravenous bolus dose
C. Metabolized in plasma esterase
D. Long acting anesthetic

407. Xenon anesthesia all are true except


A. Non-Explosive
B. Minimal cardiovascular side-effects
C. Low blood solubility
D. Slow induction and recovery

408. Nitrous oxide is


A. Good muscle relaxant
B. Lighter than air
C. Good analgesic
D. Highly soluble in blood

409. False about ketamine is


A. Phencyclidine derivative
B. Causes “dissociative anesthesia”
C. S form is more potent than the R isomer
D. Produces least analgesia among intravenous anesthetics

410. Least postoperative vomiting is seen with


A. Fentanyl
B. Etomidate
C. Propofol
D. Ketamine

411. A patient with mitral stenosis had to undergo surgery. Pre-anesthetic


checkup revealed the increased liver enzymes. Which of the following
inhalational agent should be preferred in this patient?
A. Enflurane
B. Halothane
C. Xenon
D. Sevoflurane

412. Most potent inhalational anesthetics exerting stabilizing effect on the


post junctional membrane at NMJ is
A. Sevoflurane
B. Isoflurane
C. Halothane
D. Desflurane

413. Anesthesia intraindicated in a intraocular air bubble is


A. Halothane
B. Nitrous oxide
C. Ketamine
D. Desflurane
402. 403. 404. 405. c 406. 407. 408. c 409. d 410. c 411. c
d b b d d

412. 413.
d b
Anaesthesia — Explanations
343. B. Prilocaine
Ortho-toluidine is an oxidizing agent and it is a metabolite of prilocaine
Prilocaine cause Methemoglobinemia due to accumulation of ortho-toluidine, WHICH is
consequence of the metabolism of the aromatic ring.
Thus, option B is correct. Ref: KAT, Topic Local anaesthetics

344. C. Thiopentone
Thiopentone:
Gradual i.v. infusion of subanaesthetic does can be used to facilitate verbal
communication with psychiatric patients and for ‘narcoanalysis’ of criminals; acts by
knocking off guarding.
Thus, option C is correct. Ref: KDT, Topic: General Anaesthetics

345. B. Excessive release of calcium from the sarcoplasmic reticulum


The symptoms of the patient (muscle rigidity, hypertension, hyperthermia,
hyperkalemia and acidosis) suggests the diagnosis of malignant hyperthermia.
Halothane can precipitate malignant hyperthermia in susceptible individuals. SCh
increases this incidence.
Malignant hyperthermia is due to excessive release of Ca2* from the sarcoplasmic
reticulum.
Drug of choice for malignant hyperthermia is Dantrolene that inhibits the release of
Ca2+ (by blocking ryanodine receptors in the sarcoplasmic reticulum
Thus, option B is correct. Thus, option A is correct. Ref: KDT, Topic: General
anaesthetics

346. D. Halothane
Halothane sensitizes the heart to arrhythmogenic action of catecholamines.
In pheochromocytoma, there are elevated levels of catecholamines.
Therefore, halothane should not be used in patients with pheochromocytoma.
Thus, option D is correct. Thus, option A is correct. Ref: KDT, Topic: General
anaesthetics

347. A. Propofol
Propofol is the most commonly used anesthetic agent for `day care surgery’.
Thus, option A is correct. Thus, option A is correct. Ref: KDT, Topic: General
anaesthetics

348. C. Analgesia
Ketamine is a powerful analgesic agent.
It increases blood pressure, intraocular pressure and intracranial tension.
It does not depress CVS and respiratory system.
Thus, option C is correct. Ref: KDT, Topic: Anaesthetic agents

349. D. Hypertensive
Ketamine is contra-indicated in hypertensive because it increases the blood pressure.
It is the induction agent of choice for:
Asthmatics
Shock
Children
Full stomach
It possesses very powerful analgesic action.
It can be used as a sole agent for minor procedures.
Thus, option D is correct. Ref: KDT, Topic: Anaesthetic agents

350. C. Ketamine
Ketamine produces dissociative anesthesia
Neurolept analgesia is produced by fentanyl droperidol
Thus, option C is correct. Ref: KDT, Topic: Anaesthetic agents

351. A. Halothane
Rarely, halothane can cause malignant hyperthermia, which is treated with
Dantrolene
Thus, option A is correct. Ref: KDT, Topic: Anaesthetic agents

352. C. Very good analgesic action


Newer inhalational anesthetic agents like halothane, isoflurane etc. lack analgesic
activity.
All other advantages listed in the question are present in halothane.
Thus, option C is correct. Ref: KDT, Topic: Anaesthetic agents

353. B. It is cheap and can be administered without anesthetic machine


Ether is the only complete anesthetic agent.
It is highly inflammable and explosive.
It has good analgesic and muscle relaxant action.
It can be delivered by open method.
It is a pungent smelling liquid.
Induction of anesthesia with ether is quite slow. All the four stages can be seen.
Thus, option B is correct. Ref: KDT, Topic: Anaesthetic agents

354. D. Nitrous oxide


Nitrous oxide is not a complete anesthetic (MAC 104%).
It is a good analgesic but poor muscle relaxant.
Thus, option D is correct. Ref: KDT, Topic: Anaesthetic agents

355. D. Nitrous oxide


Concentration effect, second gas effect and diffusion hypoxia are seen with
inhalational agents used in high concentrations (like N2O).
Thus, option D is correct. Ref: KDT, Topic: Anaesthetic agents

356. A. Ketamine
Ketamine is an intravenous inducing agent. It acts by blocking NMDA receptors.
Hallucinations, delirium and vivid dreams are important adverse effects that are seen
during recovery from anesthesia (emergence reaction).
It increases blood pressure, intraocular pressure and intracranial tension.
Thus, option A is correct. Ref: KDT, Topic: Anaesthetic agents
357. C. Potency
MAC is inversely related to potency of an inhalational agent.
N2O has maximum MAC (104%) and is thus the least potent agent.
Methoxyflurane is the most potent drug due to minimum value of MAC.
Thus, option C is correct. Ref: KDT, Topic: Anaesthetic agents

358. D. Succinylcholine
There are two clues to the correct answer in this scenario.
The patient underwent rapid sequence intubation (RSI). A depolarizing
neuromuscular blocking drug is commonly administered for RSI, because onset of
action is generally more rapid (within 60 seconds) than for most available non
depolarizing blockers.
The patient still exhibits residual muscle paralysis even after neostigmine, an anti-
cholinesterase. The persistence of paralysis indicates that Drug A is a depolarizing
blocker. Anticholinesterases do not reverse the action of depolarizing and may, in
fact, enhance them.
The only depolarizing blocker listed among the options is succinylcholine. In patients
with atypical pseudo cholinesterase, SCh may produce prolonged paralysis and
apnea.
Thus, option D is correct. Ref: Katzung

359. A. Succinylcholine
SCh is the shortest acting muscle relaxant due to its metabolism by pseudo
cholinesterase.
Some patients contain an atypical pseudo cholinesterase (which has abnormally low
activity) and are susceptible to develop apnea with the use of this drug.
Thus, option A is correct. Ref: KDT, Topic: Anaesthetic agents

360. A. Tubocurarine
Hypotension and bronchoconstriction (increase in airway resistance) are important
adverse effects caused by histamine. This is confirmed to be due to histamine
because of reversal with diphenhydramine.
Maximum histamine release is caused by d-tubocurarine.
Atracurium causes minimum histamine release, therefore is preferred agent in
asthmatic patients.
Thus, option A is correct. Ref: KDT, Topic: Anaesthetic agents

361. C. It may induce life threatening hyperkalemia


SCh is the shortest and fastest acting muscle relaxant.
SCh is contra-indicated in patients with nerve or muscle disorders due to the risk of
hyperkalemia
Nerve disorders: Hemiplegia, paraplegia, Guillain Barre syndrome etc.
Muscle diorders: Myopathy, myasthenia gravis, rhabdorivolysis, crush injury etc.
Thus, option C is correct. Ref: KDT, Topic: Anaesthetic agents

362. C. Tizanidine
Tizanidine and Brimonidine are cc, adrenergic agonists. Tizanidine is used as a
centrally acting muscle relaxant whereas Brimonidine is used topically for the
treatment of glaucoma.
Chlormezanone is a centrally acting muscle relaxant that acts by inhibiting the spinal
internuncial neurons.
Quinine is a directly acting peripheral muscle relaxant.
Thus, option C is correct. Ref: KDT, Topic: Anaesthetic agents

363. A. Reducing Ca2+ release from sarcoplasmic reticulum in the muscle fibre
Dantrolene is the drug of choice for the treatment of malignant hyperthermia and
neurolept malignant syndrome.
It acts as an antagonist of ryanodine receptors (present on smooth endoplasmic
reticulum). It inhibits the release of Cat’ from sarcoplasmic reticulum in the muscle
fibre.
Thus, option A is correct. Ref: KDT, Topic: Anaesthetic agents

364. C. Streptomycin
Aminoglycosides (like streptomycin and gentamicin) can accentuate the
neuromuscular blockade produced by competitive blockers (like pancuronium
Mechanism of neuromuscular blockade produced by aminoglycosides is the inhibition
of presynaptic release of ACh.
Thus, option C is correct. Ref: KDT, Topic: Anaesthetic agents

365. D. Rocuronium
Rocuronium is the fastest acting non-depolarizing muscle relaxant (NDMR). It can be
used for the rapid sequence endotracheal intubation in patents
having contra-indications to the use of SCh.
Mivacurium is the shortest acting NDMR.
SCh is the shortest and fastest acting skeletal muscle relaxant. It is a depolarizing NM
blocker.
Thus, option D is correct. Ref: KDT, Topic: Anaesthetic agents

366. B. Mivacurium
Long acting non-depolarizing (competitive) NM blocking agents require reversal with
neostigmine.
Mivacurium is the shortest acting NDMR. It does not require reversal due to its short
duration of action.
Mivacurium can be used in day care surgery.
Thus, option B is correct. Ref: KDT, Topic: Anaesthetic agents

367. A. It can cause rise in BP on rapid I. V. injection


Ancuronium possesses vagolytic activity and can cause hypertension and tachycardia
on rapid i.v. injection.
Pancuronium is a competitive NM blocker (Non-depolarizing NM blocker). Its actions
can be reversed by anticholinesterases like neostigmine.
Unlike d-TC, histamine release is not seen with Pancuronium.
Thus, option A is correct. Ref: KDT, Topic: Anaesthetic agents

368. D. All of the above


d-tubocurarine is a competitive NM blocker. It produces fall in BP due to:
Blockade of sympathetic ganglia.
Histamine release.
Reduced venous return as a result of paralysis of limb and respiratory muscles.
Thus, option D is correct. Ref: KDT, Topic: Anaesthetic agents

369. D. Cocaine
All LAs are vasodilators except cocaine. It possesses vasoconstrictor activity.
Cocaine also has good surface activity.
Thus, option D is correct. Ref: KDT, Topic: Anaesthetic agents

370. A. 20 mg
1% solution means 1g (1000 mg) of a drug is present in 100 ml of the solution.
2% means 2000 mg in 100 ml of solution.
Therefore 1 ml will contain 20 mg.
Thus, option A is correct.

371. D. Preferential binding to resting channels


LAs act from within the neuron and are more active when the neuron is rapidly firing
(i.e., Na+ channels are open).
Thus, option D is correct. Ref: KDT, Topic: Anaesthetic agents

372. D. Increase the duration of action of the local anesthetic


Adrenaline and felypressin are the vasoconstrictors that are added to LA solution.
By causing vasoconstriction, these drugs decrease the systemic absorption resulting
in less CNS adverse effects (decreased chances of seizures).
Prolong the stay of drug at the site of action resulting in the increase in duration of
action of LA.
Thus, option D is correct. Ref: KDT, Topic: Anaesthetic agents

373. D. Bicarbonate
LAs are weak bases. These require penetration inside the neuron for their action. For
entry in the neuron, LAs have to cross the new membrane.
Unionized drugs (lipid soluble) can easily cross the membrane, therefore addition of
NaHCO3, in the local anesthetic solution (weak base are un-ionized in the alkaline
medium) makes them rapid acting.
Adrenaline increases the duration of action by causing vasoconstriction.
Methylparapben is the preservative added in LA solution.
Thus, option D is correct. Ref: KDT, Topic: Anaesthetic agents

374. A. Seizures and coma


If LA reaches the blood stream, most prominent adverse effects are related to CNS and
CVS.
In CNS, stimulation (convulsions) followed by depression (coma) is seen. Initial
stimulation is due to inhibition of the inhibitory neurons.
Thus, option A is correct. Ref: KDT, Topic: Anaesthetic agents

375. C. Orthopedic manipulation on the upper limb


IVRA is indicated for procedures on upper limb or lower limb of less than one-hour
duration.
Thus, option C is correct. Ref: KDT, Topic: Anaesthetic agents

376. D. Posture of the patient


Posture of the patient determines the height of block, not the duration.
Thus, option D is correct. Ref: KDT, Topic: Anaesthetic agents
377. D. Sympathetic block is higher than the sensory block
Spinal anesthesia creates a zone of differential blockade in which sympathetic fibers
are blocked two segments higher and motor fibers are blocked two segments lower
than the level of sensory block.
Thus, option D is correct. Ref: KDT, Topic: Anaesthetic agents

378. D. All of the above


Factors affecting the height of block:
Volume of drug
Baricity (Ratio of specific gravity of an agent to that of CSF).
Position of the patient
Infra-abdominal pressure
Curvature of spine
Factors affecting duration of block:
Dose
Concentration
Drug (LA used)
Added vasoconstrictors
Thus, option D is correct. Ref: KDT, Topic: Anaesthetic agents

379. D. By surface application, it can anaesthetize unbroken skin


Lignocaine or prilocaine cannot anaesthetize intact skin.
Eutectic mixture is the combination of equal proportions of lignocaine and prilocaine
at 25 °C. This mixture has a lower melting point than any of the two ingredients. It
helps to make the preparation oily that can be applied on the intact skin.
Eutectic mixture can be used to anaesthetize intact skin.
Thus, option D is correct. Ref: KDT, Topic: Anaesthetic agents

380. A. Dibucaine
Longest acting, most potent and most toxic LA is dibucaine.
Chlorprocaine is the shortest acting LA.
Thus, option A is correct. Ref: KDT, Topic: Anaesthetic agents

381. C. Cocaine
All LAs cause hypotension except cocaine.
Cocaine increases blood pressure by inhibiting the reuptake of catecholamines.
Thus, option C is correct. Ref: KDT, Topic: Anaesthetic agents

382. A. The local anesthetic binds to its receptor mainly when the Na+
channel is in the resting state
All LAs are weak bases.
LAs act by blocking Na+ channels from inside the neuron (intraneuronal face).
These can cross the membrane only in unionized (lipid soluble) form. Sodium
bicarbonate is therefore added to make the LA rapid acting.
Once inside the neuron, LAs again gets ionized and bind to Na+ channels.
Binding to Na+ channels is more in repetitively firing neurons than in resting
neurons.
Thus, option A is correct. Ref: KDT, Topic: Anaesthetic agents

383. A. Fentanyl
Fentanyl can be used as a transdermal patch for prolonged treatment of cancer
associated pain.
Thus, option A is correct. Ref: KDT, Topic: Anaesthetic agents

384. A. Methadone
Methadone is a long acting opioid agonist that has equal potency to morphine.
It can be used orally for opioid replacement and opioid rotation therapy.
Due to longer half-life, it produces mild withdrawal symptoms.
Thus, option A is correct. Ref: KDT, Topic: Anaesthetic agents

385. B. Naltrexone
Naltrexone is a long acting, orally effective opioid antagonist that can be used as the
maintenance treatment of opioid addicts. It is also used to decrease the craving for
alcohol.
Thus, option A is correct. Ref: KDT, Topic:

386. C. Thiopentone
Thiopentone:
Gradual i.v. infusion of subanaesthetic does can be used to facilitate verbal
communication with psychiatric patients and for ‘narcoanalysis’ of criminals; acts by
knocking off guarding.
Thus, C is correct. Ref: KDT, Topic:

387. B. Thiocolchocoside
Mephenesin congeners
Mephenesin
Cariosoprodol
Chlorzoxazone
Chlormezanone
Methocarbamol
Benzodiazepines: Diazepam
GABA mimetic: Baclofen, Thiocolchicoside
Central a2 agonist Tizanidine
Thus, B is correct.

388. B. Vecuronium
Sugamadex:
Reversing agent developed for terminating the action of non-polarizing muscle
relaxants rocuronium and vecuronium
It is a modified Y-cyclodextrin with high affinity for rocuronium and vecuronium
It encapsulates one molecule of the blocker within its molecule forming an inactive
chelate which is excreted in urine with a t1/2 of -2 hour
As a plasma concentration of free rocuronium falls, it rapidly dissociates from the Nm
receptor and neuromuscular transmission is restored
The mechanism of reversal by Sugamadex is entirely different from that of the
currently used reversing agents neostigmine and edrophonium.
Thus, B is correct. Ref: KDT, Topic:

389. D. Doxacurium
Shortest acting non-depolarizing skeletal muscle relaxant is gantacurium >
Mivacurium
Longest acting non-polarizing skeletal muscle relaxant is Doxacurium
Metocurine – 110min
Pipecuronium – 30-90min
Pancuronium – 85-100min
Thus, D is correct. Ref: GG

390. C. Succinylcholine
Shortest acting depolarizing skeletal muscle relaxant is Succinylcholine
Shortest acting depolarizing skeletal muscle relaxant is Ganatacurium > Mivacurium
Thus, C is correct. Ref: GG

391. B. Amlodipine
Drug metabolized by plasma/ pseudocholinestrase
Succinylcholine
Mivacurium
Ester group of Las
Clevidipine
Remifentanil
Ciclesonide, Beclomethasone (lung esterases)
Esmolol by erythrocyte esterases
Thus, B is correct. Ref: GG

392. B. 1:2,50,000
Concentrations of adrenaline used alone with Local Anesthetic drug during Local
anesthesia is 1:50,000 to 1: 200,000
The concentrations has following advantages
Prolongs duration of action of Las
Enhances the intensity of nerve block
Reduces systemic toxicity of Las
Provides a more bloodless field for surgery
Thus, B is correct. Ref: KDT, Topic:

393. A. 2.5% lidocaine and 2.5% procaine


EMLA (Eutectic Mixture of Local Anesthetics)
Combination of elements has lower melting temperature than its component
elements
Lidocaine and prilocaine can combine to form such a mixture
This formulation, containing 2.5% lidocaine and % prilocaine
Anesthetic penetration of the keratized layer of skin, producing localized numbness
Commonly used in pediatrics to anesthetize the skin prior to venipuncture for
Intravenous catheter placement
Thus, A is correct. Ref: KAT

394. A. Prilocaine
Rate of liver metabolism of individual amide compounds
Prilocaine (fastest) > lidocaine > Mepivacaine ropivacaine = bupivacaine and
Levobupivacaine (slowest)
Clinical Implications:
Toxicity from amide-type local anesthetics is more likely to occur in patients with
hepatic disease.
The average elimination half-life of lidocaine may be increased from 1.6 hours in
normal patients to more than 6 hours in patients with severe liver disease.
Thus, A is correct. Ref: KAT

395. C. Lidocaine
Local anesthetics:
Smaller and more highly lipophilic local anesthetics have a faster rate of interaction
with the sodium channel receptor
Potency is also positively correlated with lipid solubility
Lidocaine, procaine and Mepivacaine are more water soluble than Tetracaine,
bupivacaine and ropivacaine.
Clinical implications:
Tetracaine, bupivacaine and ropivacaine are more potent and less water soluble
They have longer durations of local anesthetic action because of lipophilicity
They also bind more extensively to proteins and can be displayed from these bindings
sites by other protein-bound drugs.
Thus, C is correct. Ref: KAT

396. C. Benzocaine
Ester have single ‘I’ in the name
Example: Benzocaine, Cocaine, Chlorprocaine Amides have double “I” in the name
Example: Lidocaine, Mepivacaine, Bupivacaine
Thus, C is correct. Ref: KDT, Topic:

397. B. C type
Susceptibility of nerve fibers to LA:
A>B>C
A (gamma > delta > beta > alpha)
Thus, B is correct. Ref: Miller’s Anesthesia/P 1033

398. D. Ketamine
Analyzing the history, patient is a known hypertensive and is being planned for
cholecystectomy Among the given options.
Ketamine increases all pressure like ICP, intra-ocular pressure and blood pressure.
So, in this case patient has hypertension, so Ketamine should be avoided in this
patient as it increases blood pressure.
Thus, D is correct. Ref: KAT

399. A. Ketamine
Ketamine:
Ketamine increases all pressures
So there it is dangerous for
Hypertensives
Ischaemic heart disease (increases cardiac work)
Congestive heart failure
Raised intracranial pressure (Ketmine increases cerebral blood flow and O2
consumption).
Thus, A is correct. Ref: KDT, Topic:

400. D. All of the above


Propofol:
Causes pain on injection
Can be reduced with lidocaine and the use of larger arm and antecubital veins.
Thus, D is correct. Ref: GG

401. C. Enflurane
Extent of hepatic metabolism for the inhaled anesthetics
Halothane > enflurane > sevoflurane > isoflurane > desflurane > nitrous oxide.
Thus, C is correct. Ref: KAT

402. D. Nitrous oxide


Inhalational anesthetic
Gas:
Nitrous oxide
Volatile liquids
Ether
Halothane
Isoflurane
Desflurane
Sevoflurane
Thus, D is correct. Ref: KDT, Topic:

403. B. Ketamine
Ketamine:
Intravenous ketamine at sub anesthetic doses produces rapid relief of depression,
even in treatment-resistant patients, that may persist for 1 week or longer.
Thus, B is correct. Ref: KAT

404. B. Xenon
Xenon:
Has analgesic and anesthetic effects
Effects are mainly due to
Noncompetitive antagonism of the NMDA receptor
Agonism at the TREK channel (a member of the two-pore K+ channel family)
Thus, B is correct. Ref: GG

405. C. Detection of generic susceptibility to halothane induced malignant


hyperthermia
Caffeine-halothane contracture test:
The most reliable test to establish susceptibility to halothane induced malignant
hyperthermia is the in vitro caffeine-halothane contracture test using skeletal
muscle biopsy samples.
Thus, C is correct. Ref: KAT

406. D. Long acting anesthetic


Remifentanil:
Short acting opioid analgesic
Useful for short painful procedures
Administered by intravenous bolus dose
Metabolized by plasma esterases
Thus, D is correct. Ref: GG

407. D. Slow induction and recovery


Xenon:
Non-explosive rare gas
Extremely insoluble in blood and other tissues
So has rapid induction and recovery from anesthesia
Well tolerated in patients of advanced gas
Minimal cardiovascular side-effects
Thus, D is correct. Ref: GG

408. C. Good analgesic


Nitrous oxide:
Is a good analgesic
Poor Muscle relaxation
Havier than air
Low blood solubility, so rapid induction and recovery
Thus, C is correct. Ref: KDT, Topic:

409. D. Produces least analgesia among intravenous anesthetics


Ketamine:
Lipid-soluble phencyclidine derivative
Produces significant analgesia among intravenous anesthetics
Causes “dissociative anesthesia”
S form is more potent than the R isomer
Thus, D is correct. Ref: KAT

410. C. Propofol
Propofol has a significant antiemetic action, so among options Propofol has Least
postoperative vomiting.
Thus, C is correct. Ref: GG

411. C. Xenon
XENON:
Is the preferred inhalational agent in patients with liver dysfunction, as xenon is
not metabolized in the body
Xenon is an inert agent
Does not undergo metabolism
Hepatic or renal toxicity is not seen
ANAESTHETIC ADJUNCTS:
Halothane causes hepatitis
Thus, C is correct. Ref: GG

412. D. Desflurane
Many inhalational anesthetics exert a stabilizing effect on the post junctional
Membrane:
They potentiate the activity of competitive NMI blocking agents
The rank order of potentiation is desflurance > sevoflurane > isoflurane > Halothane
> Nitros oxide-barbiturate opioid or propofol.
Anesthesia
Clinical implication:
When NMI blockers are used for muscle relaxation as adjuncts to these anesthetics,
their doses should be reduced.
Thus, D is correct. Ref: GG

413. B. Nitrous oxide


N2O:
N2O exchanges with N2 in any air containing cavity in the body.
Due to differential blood: gas partition coefficients of N2O and N2, N2O will enter the
cavity faster than N2 escapes
So N2O increases the volume or pressure in this cavity
Examples:
A Pneumothorax
An obstructed middle ear
An air embolus
An obstructed loop of bowel
An intraocular air bubble
A pulmonary bulla
Intracranial air
AVOID N2O in the above conditions
Thus, B is correct. Ref: GG
Central Nervous System — Questions
414. Long term use of pethidine is avoided because a metabolite of pethidine
is associated with
A. Constipation
B. Seizures
C. Dependence
D. Respiratory depression

415. Which of the following opioid analgesic acts primarily through Opioid
receptors?
A. Pethidine
B. Methadone
C. Buprenorphine
D. Pentazocine

416. Use of morphine should be avoided in au of the following patients


EXCEPT?
A. Elderly male patients
B. Bronchial asthma patients
C. Ischemic heart disease patients
D. Biliary colic patients

417. The rationale for using ethanol in methanol poisoning is that is


A. Inhibits the metabolism of methanol and generation of toxic metabolite
B. Stimulates the metabolism of methanol and reduces its blood level
C. Antagonises the actions of methanol
D. Replenishes the folate stores depleted by methanol

418. Disulfiram (Antabuse) is used for the treatment of


A. Acute alcoholic intoxication
B. Both physically and psychologically dependent alcoholics
C. Alcoholics psychologically but not physically dependent on alcohol
D. Both (a) and (b) are correct

419. The intense craving experience by the people recovering from chronic
alcoholism can be treated by a drug which acts by being an
A. Antagonist of opioid receptors
B. Agonist of alpha adrenoceptor
C. Agonist of beta adrenoceptor
D. Agonist of serotonin receptors

420. The combination of alcohol and disulfiram results in nausea and


hypotension as a result of accumulation of
A. Methanol
B. Acetate
C. Acetaldehyde
D. NADH

421. A 45-year-old male, Sanjeev was brought to the emergency with severe
agitation and aggressive behavior. He was started a haloperidol and the
patient became responsive and cooperative. After 8 days of treatment, he
developed his grade fever, diarrhea, confusion and muscle rigidity. Which of
the following should be used for the treatment of this conditions?
A. Diazepam
B. Dantrolene
C. Benzhexol
D. High dose of haloperidol

422. A 46-year-old male, Prabash being treated for depression was admitted
to the emergency with severe confusion and hallucinations. His mouth was
dry and his face was flushed. On examination, his blood pressure was found
to be 84/62 mmHg and his heart rate is 108 beats per minute. ECG of the
patient reveals sinus tachycardia, prolongation of PR and QT interval and
widened QRS complex. Which of the following agents would be best correct
this patient’s cardiac abnormalities?
A. Propanolol
B. Sodium bicarbonate
C. Atropine
D. Flumazenil

414. b 415. d 416. c 417. a 418. c 419. a 420. c 421. b 422. b

423. After starting your patient on imipramine, his heart rate rises to 120/min
and he has blurred vision. These effects can be explained by the fact
imipramine
A. Is a ganglionic blocker
B. Potentiates epinephrine
C. Is a muscarinic antagonist
D. Is a potent a-adrenergic blocker

424. A hypertensive patient Sattu already receiving a drug ‘X’ to control his
BP was prescribed a tricyclic antidepressant. This resulted in the abolition of
the antihypertensive action of ‘X’. Which of the following drug can be ‘X’?
A. Enalapril
B. Diltiazem
C. Atenolol
D. Clonidine

425. A patient Ashwani has been brought to the hospital with non-stop talking,
singing, uncontrollable behavior and apparent loss of contact with reality.
You diagnose it to be a case of acute mania. Which of the following is the
most suitable drug for rapid control of his symptoms?
A. Haloperidol
B. Phenobarbitone
C. Lithium carbonate
D. Valproic Acid

426. A patient present in your psychiatry OPD with complaints of diminished


vision. Ophthalmological examination revealed corneal and lenticular
opacities. He had been prescribed some antipsychotic drug during his last
visit. Which of the following it can be?
A. Flupenthixol
B. Haloperidol
C. Thioridazine
D. Pimozide

427. A patient Manoj with severe pain thought to be of gastrointestinal origin


received 60 mg of meperidine subsequent to which he developed reaction
characterized by tachycardia, hypertension, hyperpyrexia and seizures. He
gave the history that he is also taking some medicine for his psychiatric
condition. Which of the following drug can be held responsible for this sort
of reaction?
A. Alprazolam
B. Phenelzine
C. Lithium
D. Imipramine

428. A patient Ravan having depressive disorder has taken 25 times the
normal dose of amitriptyline. Which of the following is not likely to be
observed in this patient?
A. Coma and shock
B. Pinpoint pupil
C. Hypotension
D. Hot dry skin

429. MAO inhibitors patient are contra-indicated in all the following conditions
EXCEPT
A. With tricyclic antidepressants
B. With indirectly acting sympathomimetic
C. Aspirin
D. Cheese

430. Which of following statements about extrapyramidal effects of


antipsychotic drugs is FALSE?
A. Caused by blockade of dopamine receptors
B. Less likely to be produced by clozapine than by chlorpromazine
C. Can be counteract to some degree by ant muscarinic drugs
D. Haloperidol does not cause extrapyramidal syndrome

431. All of the following are limitations of typical tricyclic antidepressants


EXCET
A. Low oral bioavailability
B. Narrow safety margin
C. Frequent side effects
D. Long latent period for response

432. An antidepressant drug which is known to have both high sedative and
anticholinergic activity is
A. Phenalgine
B. Trazodone
C. Fluoxetine
D. Amitriptyline

423. c 424. d 425. a 426. c 427. b 428. b 429. c 430. d 431. a

432. d

433. Which of the following drugs is preferred for long term treatment of
severe anxiety disorder with intermittent panic attacks?
A. Phenothiazine
B. Selective serotonin reuptake inhibitor
C. P blocker
D. Azapirone

434. All of the following statement» about buspirone are incorrect EXCEPT?
A. It interacts with benzodiazepine receptor as an inverse agonist
B. It has anxiolytic but no anticonvulsants or muscle relaxant property
C. It produces physical dependence and suppresses barbiturate withdrawal
syndrome
D. It is a rapidly acting anxiolytic: good for panic states

435. Propanolol is useful in the management of which of the following side


effects of a typical neuroleptic?
A. Parkinsonism
B. Akathisia
C. Tardive dyskinesia
D. Acute muscle dystonia

436. The secretion of which of the following hormones increases with


chlorpromazine therapy?
A. Corticotropin
B. Gonadotropin
C. Prolactin
D. Antidiuretic hormone

437. AU actions of chlorpromazine are based on its anti-dopaminergic


property EXCEPT
A. Hypotensive
B. Hyperprolactinemic
C. Antiemetic
D. Antipsychotic

438. An adverse effect of neuroleptic drugs is directly correlated positively to


the antipsychotic potency of the different compounds. Which of the
following is it?
A. Sedation
B. Lowering of seizure threshold
C. Postural hypotension
D. Extrapyramidal motor disturbances

439. Which of the following effects is unlikely to occur during treatment with
imipramine?
A. Sedation
B. Mydriasis
C. Elevation of seizure threshold
D. Urinary retention

440. Which of the following drugs has a high affinity for 5-HT/receptors in the
brain, does not cause extrapyramidal dysfunction or hepatotoxicity, and is
reported to increase the risk of significant QT prolongation?
A. Ziprasidone
B. Clozapine
C. Olanzapine
D. Chlorpromazine

441. Which of the following drugs is both effective and safe to use in a
pregnant patient suffering from bipolar disorder?
A. Olanzapine
B. Lithium
C. Carbamazepine
D. Valproic acid

442. Which of the following will you like to give to a pregnant patient to
decrease the risk of neural tube defects in the offspring & if your patient is
receiving antiepileptic drugs?
A. Vitamin E
B. Vitamin A
C. Folic acid
D. Pyridoxine

443. An antiepileptic drug ‘A’ can also be used for the treatment of post-
herpetic neuralgia and pain due to diabetic neuropathy. Which of the
following can be the agent ‘A’?
A. Carbamazepine
B. Primidone
C. Lamotrigine
D. Gabapentin

433. b 434. b 435. b 436. c 437. a 438. d 439. c 440. a 441. a

442. c 443. d

444. A young male, Farhan suffers from a seizure disorder which is


characterized by tonic rigidity of limbs followed in 20-30 sec by tremors
progressing to massive jerking of the body. This clonic phase lasts for 1-3
min. The anti-seizure drug of choice for this patient is
A. Clonazepam
B. Valproic acid
C. Fosphenytoin
D. Ethosuximide

445. Raju, a 10-year-old boy is having difficulty in learning at school. He has


short lapses of awareness with eyelid fluttering that occur every 5-10
minutes. EEG studies reveal brief 3 Hz spike and wave discharges appearing
synchronously in all the leads. Which of the following drugs would be
effective but has the disadvantage that it causes sedation and tolerance?
A. Diazepam
B. Clonazepam
C. Ethosuximide
D. Valproic acid

446. Status epilepticus is managed best with the use of which of the following
drugs?
A. Intramuscular phenobarbitone
B. Intravenous phenytoin sodium
C. Intravenous diazepam
D. Rectal diazepam

447. A patient, Rama was diagnosed to be having febrile commissions in the


pediatric emergency. which of the following can be used for the treatment of
this patient?
A. Rectal diazepam
B. Intravenous phenytoin
C. Intramuscular phenobarbitone
D. Oral sodium valproate

448. The most common adverse effect particularly seen in young children
because of the use of sodium valproate is
A. Anorexia
B. Loss bf hair
C. Hepatitis
D. Tremor

449. The drug used in absence seizures and having a narrow spectrum of
antiepileptic activity is
A. Lamotrigine
B. Primidone
C. Sodium valproate
D. Ethosuximide

450. High plasma drug concentration of phenytoin can cause which of the
following adverse effects?
A. Gum hyperplasia
B. Hirsutism
C. Ataxia
D. All of the above

451. Phenytoin pharmacokinetics is highlighted by which of the following


characteristics?
A. Capacity limited metabolism saturating at higher therapeutic concentration range
B. Nonsaturation kinetics of metabolism
C. High first pass metabolism
D. Extrahepatic metabolism

452. Which of the following statements about vigabatrin is TRUE?


A. Blocks neuronal reuptake of GABA
B. Drug of choice in absence seizures
C. Visual disturbances can occur
D. Lift threatening skin disorders may occur

453. A patient of parkinsonism, Mr. Ghai noticed that the therapeutic effect of
levodopa decreased when he was given another drug by his physician but no
interaction was seen when he switched over to levodopa-carbidopa
combination. The possible drug prescribed by Ms. physician can be
A. Metoclopramide
B. Isoniazid
C. Chlorpromazine
D. Vitamin B complex
444. b 445. b 446. c 447. a 448. c 449. d 450. c 451. a 452. c

453. d

454. A compound X decreases the functional activities of several CNS


neurotransmitters including dopamine, adrenaline and serotonin at high
doses it may cause Parkinsonism like extrapyramidal system dysfunction.
Which of the following can be X?
A. Reserpine
B. Diazepam
C. Ketamine
D. Baclofen

455. Which statement is CORRECT about Pramipexole?


A. Activates brain dopamine receptors
B. Commonly a first line therapy for Parkinson’s disease
C. Not an ergot derivative
D. All of the above

456. Which of the following antiparkinsonian drugs directly activates


dopaminergic D2 receptors in the striatum?
A. Benserazide
B. Entacapone
C. Pramipexole
D. Selegiline

457. Which of the following adverse effects of levodopa is not minimized even
after combining it with carbidopa?
A. On-off effect
B. Nausea and vomiting
C. Cardiac arrhythmia
D. Involuntary movements

458. Entacapone may be useful in patients being treated with levodopa


carbidopa combination because it.
A. Activates COMT
B. Inhibits dopamine uptake
C. Inhibits monoamine oxidase type B
D. Decreases formation of 3-OMD

459. A 40 years old patient with liver dysfunction is scheduled for a surgical
procedure. Lorazepam can be used for pre-anesthetic medication in this
patient without concern for excessive CNS depression because the drug is
A. Selective anxiolytic like buspirone
B. Forming several active metabolites
C. Reversible by administration of naloxone
D. Conjugated directly

460. A very potent and short acting benzodiazepine was given to a patient
Kallu for the purpose of causing hypnosis but the drug caused psychiatric
disturbances in Mm. Which of the following can be the hypnotic used?
A. Triazolam
B. Nitrazgpam
C. Temazepam
D. Flurazepam

461. Which of the following hypnotic drugs facilitates the inhibitory actions of
GABA but lacks anticonvulsant or muscle relaxing properties and has
minimal effect on sleep architecture?
A. Buspirone
B. Zaleplon
C. Phenobarbital
D. Diazepam

462. Flumazenil can reverse the respiratory depression caused by which of the
following?
A. Midazolam
B. Ketamine
C. Fentanyl
D. Propofol

463. Which of the following statements regarding barbiturates is accurate?


A. Alkalization of the urine will accelerate the elimination of phenobarbital
B. Barbiturates may increase the half-lives of drugs metallized by the liver
C. Benzodiazepines exhibit a steeper dose response relationship as compared to
barbiturates
D. Respiratory depression caused by barbiturate over dosage can be reversed by
flumazenil
454. a 455. d 456. c 457. d 458. d 459. d 460. a 461. b 462. a

463. a

464. In strychnine poisoning, convulsions occur because of the antagonist


effects at receptors for.
A. Aspartate
B. Glutamate
C. GABA
D. Glycine

465. An i.v., bolus dose of Thiopentone leads to loss of consciousness within


10-15 sec, the patient regains consciousness in just a few minutes. This is
because it is
A. Redistributed from brain Lo other body tissues
B. Exhaled rapidly
C. Rapidly metabolized by hepatic enzymes
D. Renally excreted

464. d 465. a
Central Nervous System — Explanations
414. B. Seizures
Pethidine is mainly metabolized by hydrolysis to meperidinic acid but a minor pathway
of metabolism involving methylation to nor-pethidine is also present.
In overdose of pethidine, this minor pathway assumes importance and there is
accumulation of nor-pethidine. This compound possesses excitatory properties and
can lead to tremors, mydriasis, delirium, myodonus and convulsions.
Thus, option B is correct. Ref: KDT, Topic: opioids

415. D. Pentazocine
Pentazocine can cause dysphoric reactions (hallucinations) by stimulating the lc
receptors.
Thus, option D is correct. Ref: KDT, Topic: opioids

416. C. Ischemic heart disease patients


Morphine is used for the treatment of myocardial infarction (ischemic heart disease)
and LVF.
Morphine is contra-indicated in extremes of age i.e., very young and elderly persons.
It is also contra-indicated in bronchial asthma because it can cause respiratory
depression and worsen the condition.
By increasing intrabiliary pressure, morphine can worsen the pain of biliary colic.
Thus, option C is correct. Ref: KDT, Topic: opioids

417. A. Inhibits the metabolism of methanol and generation of toxic


metabolite
Methanol produces toxic metabolites (formaldehyde and formic acid) that are
responsible for blindness. This metabolism takes place with the help of and aldehyde
dehydrogenase.
Alcohol Aldehyde
Methanol Formic Acid Formaldehyde
dehydrogenase dehydrogenase
Ethanol is also metabolized by the same enzymes to produce acetaldehyde and
acetic acid. It competes with methanol for above enzymes and inhibits the
generation of toxic formaldehyde and formic acid.
Thus, option A is correct. Ref: KDT, Topic: Alcohols

418. C. Alcoholics psychologically but not physically dependent on alcohol


If a patient is physically dependent on alcohol, disulfiram can precipitate severe
withdrawal symptoms. It is therefore, not indicated for physically alcoholics.
Thus, option C is correct. Ref: KDT, Topic: Alcohols

419. A. Antagonist of opioid receptors


Naltrexone is used to decrease the craving for alcohol after de-addiction.
Thus, option A is correct. Ref: KDT, Topic: Alcohols

420. C. Acetaldehyde
Disulfiram is an aldehyde dehydrogenase inhibitor that can be used for de-addiction of
chronic alcoholics.
Alcohol Aldehyde
Ethyl alcohol Acetaldehyde Acetic acid
dehydrogenase dehydrogenase
Due to inhibition of aldehyde dehydrogenase, there is accumulation of acetaldehyde
that leads to several distressing symptoms (which strengthens the resolution of a
person to quit alcohol).
Thus, option C is correct. Ref: KDT, Topic: Alcohols

421. B. Dantrolene
Diagnosis in this patient is neuroleptic malignant syndrome (NMS). It is an adverse
effect caused by typical antipsychotic drugs like haloperidol. It presents clinically with
four primary features: (1) hyperthermia, (2) extreme generalized rigidity, (3)
autonomic instability, and (4) altered mental status.
Dantrolene is drug of choice for this condition.
Thus, option B is correct. Ref: Katzung 14E/P 499 Topic: Skeletal muscle relaxants

422. B. Sodium bicarbonate


Tricyclic anti-depressants (TCAs) can have quinidine-like effect on the cardiac
conduction system, potentially causing QRS and QT prolongation and cardiac. These
effects are due to inhibition of fast sodium channels. NaHCO3 can correct QRS
prolongation, reverse hypotension, and treat ventricular
Dysrhythmias. Sodium bicarbonate is the single most effective agent in treating TCA-
associated cardiac abnormalities.
Thus, option B is correct. Ref: Katzung 14E/P 525; Topic: antidepressants

423. C. Is a muscarinic antagonist


These symptoms are of anticholinergic drugs. Tricyclic antidepressants have powerful
anticholinergic properties and can lead to these symptoms.
Thus, option C is correct. Ref: KDT, Topic: antidepressants

424. D. Clonidine
TCAs abolish the antihypertensive effect of Guanethidine and clonidine by inhibiting
their transport into the adrenergic neurons.
Thus, option D is correct. Ref: KDT, Topic: centrally acting skeletal muscle relaxants

425. A. Haloperidol
Antipsychotic drugs like olanzapine and haloperidol are agents of choice for rapid
control of symptoms in acute mania.
Lithium is the drug of choice for the treatment of bipolar disorder (MDP) and
prophylaxis of mania.
Thus, option A is correct. Ref: KDT, Topic: antipsychotics

426. C. Thioridazine
Retinal degeneration and corneal and lenticular opacities are the adverse effects seen
most commonly with the use of Thioridazine.
Thus, option C is correct. Ref: KDT, Topic: antipsychotics
427. B. Phenelzine
Meperidine (pethidine) is metabolized mainly to meperidinic acid (inactive) by MAO
inhibitors. Minor pathway is conversion to nor-meperidine (possesses excitatory
properties and shows cumulation)
MAO inhibitors like phenelzine inhibits the major pathway; therefore, minor pathway
assumes importance resulting in generation of nor-pethidine that can cause seizures.
Long term use of meperidine can result in accumulation of nor-meperidine and thus
seizures can occur.
Thus, option B is correct. Ref: KDT, Topic: antipsychotics

428. B. Pinpoint pupil


Amitriptyline is a TCA like imipramine. It causes mydriasis due to anticholinergic
activity. Other adverse effects are also similar to imipramine.
Thus, option B is correct. Ref: KDT, Topic: antidepressants

429. C. Aspirin
In first three conditions mentioned in the question, MAO inhibitors will lead to
hypertensive crisis but these drugs do not interact with aspirin.
Thus, option C is correct. Ref: KDT, Topic: antidepressants

430. D. Haloperidol does not cause extrapyramidal syndrome


Extrapyramidal adverse effects of antipsychotic drugs are due to blockade of
dopamine receptors. These are less often seen with atypical antipsychotics like e and
risperidone (and more often with classical drugs like haloperidol and chlorpromazine).
These effects can be treated with central anticholinergic drugs like benztropine and
trihexiphenidyl
Thus, option D is correct. Ref: KDT, Topic: antipsychotics

431. A. Low oral bioavailability


TCAs have very good oral absorption
These agents have narrow therapeutic index and thus low safety margin.
Sedation and anticholinergic adverse effects are seen frequently with the use of TCAs.
Antidepressant actions of both TCAs as well as SSRIs take several weeks to manifest.
Thus, option A is correct. Ref: KDT, Topic:

432. D. Amitriptyline
Tricyclic antidepressants (TCA) possess anticholinergic and sedative properties.
Amitriptyline and tryptamine possess highest sedative and ant muscarinic actions.
Thus, option D is correct. Ref: KDT, Topic: antipsychotics

433. B. Selective serotonin reuptake inhibitor


SSRIs are the drugs of choice for long-term Management of panic disorder whereas
benzodiazepines are DOC for acute panic attacks.
Thus, option B is correct. Ref: KDT, Topic: antipsychotics

434. B. It has anxiolytic but no anticonvulsants or muscle relaxant property


Buspirone is a 5HT1A agonist useful for the treatment of anxiety.
Unlike benzodiazepines, it is non-sedative anti-anxiety drug.
Due to delay in the onset of its action, it is used for the treatment of chronic anxiety
and is ineffective in acute panic attacks.
Buspirone lacks anticonvulsant and muscle relaxing property.
Thus, option B is correct. Ref: KDT, Topic: antipsychotics

435. B. Akathisia
Treatment of various extrapyramidal symptoms is as follows:

Thus, option B is correct. Ref: KDT, Topic: antipsychotics

436. C. Prolactin
Dopamine acts like prolactin releasing inhibitory hormone. D, blockers decrease the
action of dopamine and can result in hyperprolactinemia. Some drugs blocking D2
receptors are:
Antipsychotics like chlorpromazine
Metodopramide
Thus, option C is correct. Ref: KDT, Topic: antipsychotics

437. A. Hypotensive
Chlorpromazine is a typical antipsychotic (D, blocker) with anticholinergic and a-
blocking properties.
D, blockade is also responsible for extrapyramidal symptoms and hyperprolactinemia
(dopamine acts like prolactin release inhibitory hormone).
Antiemetic effect of chlorpromazine is due to blockade of D2 receptors in CTZ.
Anticholinergic effects manifest as dry mouth, blurring of vision and urinary
retention.
Hypotension and impaired ejaculation may be seen due to a-blocking activity of
chlorpromazine.
Thus, option A is correct. Ref: KDT, Topic: antipsychotics

438. D. Extrapyramidal motor disturbances


Antipsychotic potency of typical antipsychotic drugs is related to their 132 receptor
blocking action.
D2 blocking action is also responsible for extrapyramidal adverse effects like muscle
dystonia and Parkinsonism etc.
Thus, option D is correct. Ref: KDT, Topic: antipsychotics

439. C. Elevation of seizure threshold


Imipramine is a tricyclic antidepressant. It inhibits the reuptake of serotonin and nor-
adrenaline. In addition, it possesses anticholinergic and a-blocking are:
Increased ‘risk of seizures due to lowering of seizure threshold is an adverse effect of
this drug.
Thus, option C is correct. Ref: KDT, Topic: antidepressants

440. A. Ziprasidone
Clozapine, olanzapine and ziprasidone are atypical antipsychotic agents that act by
blocking SHT, receptors.
Chlorpromazine is a typical antipsychotic drug. It blocks D2 receptors in the brain and
can cause extrapyramidal symptoms.
Major adverse effect of clozapine is agranulocytosis (hematotoxic) and seizures.
Olanzapine causes weight gain.
Ziprasidone can cause QT prolongation leading to torsades de pointes.
Thus, option A is correct. Ref: KDT, Topic: antipsychotics

441. A. Olanzapine
All of these drugs can be used for the treatment of bipolar disorder but
carbamazepine, lithium and Valproic acid are teratogenic. Olanzapine is a safe in
pregnancy.
Thus, option A is correct. Ref: KDT, Topic: antipsychotics

442. C. Folic acid


Folic acid is administered to prevent the development of neural tube defects in the
baby.
Risk of neural tube defects is increased with the use of Valproic acid in pregnancy.
Thus, option C is correct. Ref: KDT, Topic: antiepileptics

443. D. Gabapentin
Gabapentin and progabide act by increasing the release of GABA.
Gabapentin is the drug of choice for post-herpetic neuralgia and diabetic neuropathy.
Thus, option D is correct. Ref: KDT, Topic: antiepileptics

444. B. Valproic acid


Diagnosis of this patient is generalized tonic clonic seizure (GTCS) or grand mal
epilepsy. Drug of choice for this condition is Valproic acid.
Thus, option B is correct. Ref: Harrison 16E/P 2367; Topic: antiepileptics

445. B. Clonazepam
Diagnosis of the patient is petit mal epilepsy (absence seizures). Drugs effective
against absence seizures are:
Ethosuximide
Valproate
Clonazepam
Lamotrigine
Clonazepam is a benzodiazepine that can cause sedation and tolerance.
Thus, option B is correct. Ref: KDT, Topic: antiepileptics

446. C. Intravenous diazepam


Benzodiazepines like diazepam and lorazepam are fast acting anti-seizure drugs and
can be used to abort the seizures in status epilepticus.
Lorazepam is the drug of choice for status epilepticus.
Thus, option C is correct. Ref: KDT, Topic: antiepileptics

447. A. Rectal diazepam


Diazepam is the drug of choice for febrile convulsions. It is administered intravenously.
If i.v. access is not possible, rectal diazepam can be used.
Thus, option A is correct. Ref: KDT, Topic: antiepileptics

448. C. Hepatitis
Sodium valproate is contra-indicated in children less than 3 years due to risk of
hepatitis.
Thus, option C is correct. Ref: KDT, Topic: antiepileptics

449. D. Ethosuximide
Ethosuximide is useful only in the treatment of absence seizures (petit mal epilepsy).
Valproic acid and lamotrigine are broad spectrum antiepileptic drugs.
Thus, option D is correct. Ref: KDT, Topic: antiepileptics

450. C. Ataxia
Cerebellar dysfunction (ataxia, nystagmus and vertigo) is seen at toxic plasma
concentrations whereas hirsutism and gum hyperplasia is seen at therapeutic
concentrations (but on prolonged use).
Thus, option C is correct. Ref: KDT, Topic: antiepileptics

451. A. Capacity limited metabolism saturating at higher therapeutic


concentration range
Phenytoin follows zero order kinetics at high plasma concentration due to saturation of
metabolic pathways.
Thus, option A is correct. Ref: KDT, Topic: antiepileptics

452. C. Visual disturbances can occur


Vigabatrin act by inhibiting the enzyme GABA transaminase (involved in breakdown
of GABA).
It is used for the treatment of partial and generalized seizures. It is also used to treat
infantile spasms.
Visual disturbances are very important adverse effect of this agent.
Thus, option C is correct. Ref: KDT, Topic: antiepileptics

453. D. Vitamin B complex


Pyridoxine is a component of vitamin B complex.
Pyridoxine is a cofactor for the enzyme, dopa decarboxylase and therefore,
administration of vitamin B complex can stimulate the activity of this enzyme.
Dopa decarboxylase converts levo-dopa to dopamine. Increased formation of
dopamine in the periphery is undesirable because it cannot cross blood brain barrier.
Therefore, stimulation of dopa decarboxylase decreases the therapeutic effect of 1-
dopa.
If the enzyme, dopa decarboxylase is already inhibited with carbidopa, there will be
no interaction with pyridoxine.
Thus, option D is correct. Ref: KDT, Topic: antiepileptics
454. A. Reserpine
Neurotransmitters (like dopamine, serotonin and adrenaline) are stored in the vesicles
after their synthesis. These stores of neurotransmitters are released on stimulation of
the neuron. Reserpine inhibits the vesicular transport of these neurotransmitters
resulting in depletion of dopamine, serotonin and adrenaline.
Severe depression (due to deficiency of serotonin) leading to suicidal tendencies and
extrapyramidal symptoms (due to deficiency of dopa-mine in the brain) are the
adverse effects of this drug.
Thus, option A is correct. Ref: KDT, Topic: antidepressants

455. D. All of the above


Pramipexole and Ropinirole are newer non-ergot dopamine agonists. Now, these are
first choice drugs for the treatment of Parkinsonism (preferred over Levo-dopa)
Thus, option D is correct. Ref: KDT, Topic: parkinsons disease

456. C. Pramipexole
Directly acting D, receptor agonists can be
Ergot derivatives e.g. bromocriptine and pergolide
Non-ergot compounds e.g. Pramipexole and Ropinirole
Thus, option C is correct. Ref: KDT, Topic: parkinsons disease

457. D. Involuntary movements


Carbidopa can decrease all the adverse effects of levodopa except
Involuntary movements
Behavioral effects
Thus, option D is correct. Ref: KDT, Topic: parkinsons disease

458. D. Decreases formation of 3-OMD


Levo-dopa is converted to dopamine by dopa decarboxylase (that cannot cross blood
brain barrier).
Carbidopa is an inhibitor of dopa decarboxylase in the periphery. It therefore
increases the amount of 1-dopa that reaches the brain (due to inhibition of peripheral
metabolism to dopamine).
Due to inhibition of 1-dopa metabolism (by dopa decarboxylase), an alternative
pathway of metabolism by catechol-o-methyl transferase (COMT) is activated.
COMT metabolizes 1-dopa to 3-o-methyl dopa (3-0MD) that can complete with-l-dopa
for entry in the brain.
Entacapone and tolcapone is useful in Parkinsonism by inhibiting COMT.
Thus, option D is correct. Ref: KDT, Topic: parkinsons disease

459. D. Conjugated directly


Lorazepam and oxazepam are the benzodiazepines which are excreted from the body
by glucuronide conjugation without phase 1 metabolism in the liver. So these can be
used safely in hepatic dysfunction.
Thus, option D is correct. Ref: Katzung 14E Topic : sedative hypnotics

460. A. Triazolam
Triazolam is a very potent BZD with ultra-rapid elimination. Some cases of paranoia
and psychiatric disturbances have been noted with this drug.
Thus, option A is correct. Ref: KDT, Topic: sedative hypnotics

461. B. Zaleplon
Zolpidem, zaleplon and zopiclone are agonists at BZD receptors.
These are hypnotic drugs that lack muscle relaxant and anticonvulsant actions.
These have negligible effect on REM sleep and do not affect sleep architecture.
Thus, option B is correct. Ref: KDT, Topic: sedative hypnotics

462. A. Midazolam
Flurnazenil is an antagonist of benzodiazepine receptors and is used to prevent
respiratory depression due to benzodiazepines (like midazolam).
Thus, option A is correct. Ref: KDT, Topic: sedative hypnotics

463. A. Alkalization of the urine will accelerate the elimination of


phenobarbital
Barbiturates are acidic drugs. These are ionized in the alkaline urine and thus cannot
be reabsorbed. Alkalization of urine can be used to accelerate the elimination of
barbiturates in poisoning.
Other differences between benzodiazepines and barbiturates include.

Thus, option A is correct. Ref: KDT, Topic: sedative hypnotics

464. D. Glycine
GABA is the principal inhibitory neurotransmitter in the brain and glycine is the
inhibitory amino acid in the spinal cord.
By antagonizing the glycine receptors, strychnine can result in convulsions and other
stimulatory symptoms.
Thus, option D is correct. Ref: KDT, Topic: sedative hypnotics

465. A. Redistributed from brain Lo other body tissues


Thiopentone and other highly lipid soluble agents first reach the highly perfused
organs like, brain. Therefore, onset of action of these agents is very However, action
also terminates quickly because of redistribution to less well perfused tissues like fat
and muscle.
Thus, option A is correct. Ref: KDT, Topic: sedative hypnotics
Cardiovascular System — Questions
466. True statement about trimetazidine is
A. It is an inhibitor of fatty acid oxidation
B. lt improves tissue perfusion by modifying rheological property of blood
C. It is a novel sodium channel blocker
D. Both (a) and (b) are correct

467. Propanolol is contraindicated in a patient of angina pectoris who is


already receiving
A. Nifedipine
B. Aspirin
C. Isosorbide mononitrate
D. Verapamil

468. All of the following statements about amlodipine are true except
A. It has a very long elimination half life
B. It has a very large volume of distribution
C. It undergoes very in the first pass metabolism and possess high and consistent
oral bioavailability
D. It acts by conversion to an active metabolite in the liver

469. Which of the following drugs is most likely to precipitate angina?


A. Amlodipine
B. Verapamil
C. Diltiazem
D. Nifedipine

470. Concomitant administration of the drug with organic Nitrates is


contraindicated due to marked potentiation of vasodilatory action leading to
profound hypotension. This drug is
A. Propanolol
B. Sildenafil
C. Hydrochlorothiazide
D. Fluoxetine

471. Organic nitrates can lead to the development of tolerance when used
develop tolerance is least likely to
A. Sustained release oral nitroglycerine
B. Oral Pentaerythritol tetranitrate
C. Transdermal nitroglycerine
D. Sublingual nitroglycerine

472. Nitroglycerine exerts beneficial effects in classical angina pectoris


primarily by
A. Reduction of cardiac preload
B. Redistribution of coronary blood flow
C. Increase in total coronary blood Row
D. Reduction of cardiac after load

473. Which of the following statements best explains the action of


nitroglycerine on coronary vessels
A. It migrates angina pectoris by increasing total coronary Flow
B. It mainly decreases the after load
C. It preferentially dilates auto regulatory arterioles without affecting the larger
arteries
D. It preferentially dilates conducting arteries without affecting resistance arterioles

474. The drug effective for treatment as well as prophylaxis of angina pectoris
is
A. Diltiazem
B. Pentaerythritol tetra nitrate
C. Isosorbide dinitrate
D. Dipyridamole

475. Which of the following is an active metabolite of another drug and is


available as a separate drug for the treatment of angina?
A. Nitroglycerine
B. Isosorbide dinitrate
C. lsosorbide Mononitrate
D. Propanolol
466. a 467. d 468. d 469. d 470. b 471. d 472. a 473. d 474. c

475. c

476. Verapamil is associated with all of the following except?


A. Hyperglycemia
B. Bradycardia
C. Constipation
D. Increased PR interval

477. You decide not to prescribe sildenafil in a patient because the patient
told you that he is taking an antianginal dug. Which of the following can it
be?
A. Organic nitrates
B. B adrenergic blockers
C. Calcium channel blocker
D. Angiotensin converting enzyme inhibitors

478. A 40-year old politician suffered from attacks of chest pain diagnosed as
angina pectoris. He had a tense personality, resting heart rate was 96 min
blood pressure 17qR04mm Hg but blood sugar level and lipid profile were
normal. Select the most suitable antihypertensive for the initial therapy in
his case
A. Atenolol
B. Hydrochlorothiazide
C. Nifedipine
D. Methyldopa

479. Devraj is working as a CEO in a airlines company. He has a traveling job.


He is a known diabetic controlled on oral hypoglycemic drugs. On his recent
visit to the doctor, his blood pressure was found to be 164/102 mm Hg. Most
suitable drug for this patient should be?
A. Propanolol
B. Hydrochlorothiazide
C. Clonidine
D. Enalapril

480. A patient, amir is admitted to the emergency with severe bradycardia


following drug overdose. The concerned drug was being taken for
hypertension. Which of the following cannot be the drug?
A. Clonidine
B. Reserpine
C. Propanolol
D. Hydralazine

481. A patient has been prescribed a drug A by a doctor for treating his
hypertension. This drug can cause tachycardia and marked fluid retention.
Which of the following can be the drug A?
A. Captopril
B. Guanethidine
C. Metoprolol
D. Minoxidil

482. ln which of the following patients would Enalapril be the best first line
agent for high blood pressure control?
A. A 62-year-old man with renal artery stenosis
B. A 56-year-old diabetic woman
C. A 32-year-old pregnant female
D. A 41-year-old woman with hyperkalemia

483. When treating hypertension chronically, orthostatic hypertension is


maximum with
A. Clonidine
B. Prazosin
C. Propanolol
D. Guanethidine

484. Which of these antihypertensive Do NOT have any central action?


A. Propanolol
B. Methyldopa
C. Prazosin
D. Clonidine

485. Which antihypertensive is a prodrug and is converted to its active form in


brain
A. Clonidine
B. Minoxidil
C. Methyldopa
D. Nitroprusside

486. All of the following antihypertensive drugs increase plasma renin activity
except
A. Hydralazine
B. Nifedipine
C. Captopril
D. Clonidine

487. Clonidine is used as an antihypertensive agent but if used as a fast


intravenous injection, it can increase the blood pressure due to
A. Stimulation of vasomotor center
B. Agonistic action on vascular a2 adrenergic receptors
C. Release of noradrenaline from adrenergic nerve endings
D. Cardiac stimulation
476. a 477. a 478. a 479. d 480. d 481. d 482. b 483. d 484. c

485. c 486. d 487. b

488. Mr. Rushil has severe hypertension and is to receive minoxidil. Minoxidil
is a powerful arteriolar vasodilator that does not act on autonomic receptor.
When used in severe hypertension its effects would probably include
A. Tachycardia and decreased cardiac output
B. Decreased mean arterial pressure and decreased cardiac contractility
C. Tachycardia and increased cardiac contractility
D. Decreased mean arterial pressure and increased salt and water excretion by the
kidney

489. Which of the following is true regarding Enalapril treatment in patients


of essential hypertension
A. Deceased concentration of renin in the blood
B. Decreased sodium and increases potassium in the urine
C. Decreased angiotensin II concentration in the blood
D. All of tile above

490. A patient Vipin has been taking digoxin for several years and is about to
receive atropine for other indication. A common side effect digoxin that can
be blocked by atropine is
A. Deceased appetite
B. Increased cardiac contractility
C. Tachycardia
D. Increased PR interval on the ECG

491. Which of the following is not given in acute severe digitalis toxicity?
A. Digibind
B. Lignocaine
C. Potassium
D. None of these

492. Mechanism of action of digitalis in atrial fibrillation is


A. By decreasing cardiac contractility
B. Increase in Refractoriness of AV nodal tissue
C. Na+ K+ ATPase inhabitation
D. By causing Bradycardia

493. Which of the following is NOT true about the use of B blockers in CHF
A. These should be started at very low dose and slowly titrated upwards
B. Carvedilol is most widely used B blocker
C. These can reduce mortality in CHF patients
D. These are drug of choice in acute decompensated heart failure

494. Which of these drugs NOT decrease angiotensin II activity


A. Enalapril
B. Nesiritide
C. Valsartan
D. Omapatrilat

495. All of the following statements about the use of spironolactone in CHF
are true except
A. It affords rapid symptomatic relief in CHF patients
B. It should be administered in low doses to prevent hyperkalemia
C. It affords prognostic benefit in severe heart failure over and above that afforded
by ACE inhibitors
D. It helps to overcome the Refractoriness to thiazides

496. Which of the following drugs is beneficial in a Refractory congestive


heart failure by increasing cardiac contractility and decreasing preload and
afterload?
A. Amrinone
B. Amiloride
C. Amiodarone
D. Carvedilol
497. All of the following drugs are useful for long-term treatment of
congestive heart failure except.
A. Digoxin
B. Ramipril
C. Spironolactone
D. Dobutamine

498. All of the following are the actions of B-adrenoceptor blockers in CHF
except
A. Decrease in mortality associated with CHF
B. Prevention of pathological remodeling of ventricular myocardium
C. Prevention of dangerous cardiac arrhythmias
D. Antagonism of vasoconstriction due to sympathetic over activity

488. c 489. c 490. d 491. c 492. b 493. d 494. b 495. a 496. a

497. d 498. d

499. A patient of atrial fibrillation is on digoxin therapy. Which of the following


responses do you expect?
A. Restoration of normal sinus rhythm
B. Conversion of atrial fibrillation to atrial flutter
C. Decrease in atrial fibrillation frequency but increase in ventricular rate
D. Increase in atrial fibrillation frequency, but decrease in ventricular rate

500. Digitalis is used in the treatment of acute CHP. It can also be used as a
long-term maintenance therapy if CHP is associated with
A. Hypertension
B. Atrial fibrillation
C. Hypertrophic obstructive cardiomyopathy
D. Mitral stenosis

501. Advantage of using digoxin in CHP is


A. It reverses the pathological changes of UIF
B. It prolongs the survival of CHF patient
C. It is used Io provide relief of symptoms
D. All of these

502. The most important channel of elimination of digoxin is


A. Tubular secretion
B. Hepatic metabolism
C. Excretion in bile
D. Glomerular filtration

503. Drugs that have been found to be useful in compensated heart failure
include all of the following except.
A. Na+ K+ ATPase inhibitors
B. Beta receptor agonists
C. Alpha blockers
D. Beta receptor antagonists

504. Which of the following is incorrect about acetazolamide


A. Acts on the luminal and basolateral enzymes
B. Should be avoided in sulfa drug allergy patients
C. Competitively inhibits Carbonic anhydrase in PT
D. All of the above

505. Which of the following osmotic diuretic has long half life
A. Glycerin
B. Mannitol
C. Isosorbide
D. Urea

506. Which of the following osmotic diuretic mat cause hyperglycemia


A. Isosorbide
B. Mannitol
C. Glycerin
D. Urea

507. All are adverse effects seen with Loop diuretics except
A. Decrease plasma levels of triglycerides
B. Hyperuricemia
C. Increase plasma levels of LDL cholesterol
D. Decrease plasma levels of HDL, Cholesterol

508. Regarding Ototoxicity by Furosemide the false statement is :


A. Ototoxicity occurs most frequently with rapid intravenous administration
B. The rate of furosemide infusions should not exceed 4 mg/min to avoid ototoxicity
C. Ototoxicity occurs least frequently with oral administration
D. Most ototoxic loop diuretic is ethacrynic acid

509. Which of the following drugs is contraindicated with Spironolactone


A. Atenolol
B. Verapamil
C. Chlorthiazide
D. Enalapril

510. Which of the following condition Thiazide diuretics are not used
A. Idiopathic hypercalciurea with nephrocalcinosis
B. Congestive Heart Failure
C. Hypertension
D. Hyperlipidemia
499. d 500. b 501. c 502. d 503. b 504. c 505. c 506. c 507. a

508. c 509. d 510. d

511. Which of the following is a Diuretic


A. Conivaptan
B. Rolophylline
C. Both A and B
D. None

512. Metabolic acidosis with hypokalemia is seen with


A. Torsemide
B. Acetazolamide
C. Chlorthalidone
D. Amiloride

513. The following drugs are used in treatment of Liddle syndrome except
A. Acetazolamide
B. Triamterene
C. Spironolactone
D. None of the above

514. Which of the following aldosterone antagonist has highest oral


bioavailability
A. Spironolactone
B. Potassium Canrenoate
C. Canrenone
D. Epleronone

515. Which of the following is an inhibitor of Renal Epithelial Na+ Channels


A. Triamterene
B. Spironlonolactone
C. Indapamide
D. Xipamide

516. Thiazide diuretics are known to cause hyperglycemia, which of the


following ins incorrect about it
A. Thiazide diuretics decrease glucose tolerance
B. Thiazide-induced diabetes mellitus is associated with the same cardiovascular
disease risk as incident diabetes
C. Hyperglycemia is due to reduced insulin secretion and alterations in glucose
metabolism
D. Hyperglycemia is reduced when K+ is given along with the diuretic

517. Following fluid and electrolyte imbalance are seen with thiazide diuretics
except
A. Hypochloremia
B. Hypermagnesemia
C. Metabolic alkalosis
D. Hypercalcemia

518. Which of the following diuretic is active even when GFR is less than 30-40
mL/min
A. Chlorothiazide
B. Indapamide
C. Chlorthalidone
D. Clopamide

519. Most potent diuretic among the following diuretics is


A. Chlorthalidone
B. Hydrochlorothiazide
C. Chlorothiazide
D. Indapamide

520. Which of the following diuretic acts by inhibiting Na+-Cl symport


A. Furosemide
B. Acetazolamide
C. Spironolactone
D. Chlorthalidone

521. Loop diuretics causes ‘post diuretic Na+ retention due to their short half-
life, this can be overcome by
A. Restricting dietary Na+ intake
B. More frequent administration of the loop diuretic
C. Both A and B
D. Adding Potassium sparing diuretic

522. Drug not used in SIADH is


A. Demeclocycline
B. Restriction of free water intake
C. Desmopressin
D. 3% Nacl
511. c 512. b 513. c 514. b 515. a 516. b 517. b 518. b 519. d

520. d 521. c 522. c

523. A 30-year-old male, Rajinder presents to OPD your office with fatigue,
muscle weakness and headache. His blood pressure is 179020 nun Hg and
his heart rate is 100lmin. Laboratory evaluation reveals hypokalemia,
metabolic alkalosis and decreased plasma renin activity. On CT scan a mass
was noted on left suprarenal gland. Patient was prescribed a drug for few
weeks and the symptoms subsided. Laboratory values and blood Pressure
returned to normal values. The likely drug given to this patients?
A. Clonidine
B. Spironolactone
C. Hydrochlorothiazide
D. Propanolol

524. Which of the following agents is useful for the oral treatment of both
pituitary as well as renal diabetes insipidus?
A. Hydrochlorothiazide
B. Vasopressin
C. Chlorpropamide
D. Carbamazepine

525. Desmopressin is a synthetic analog of ADH. It is preferred over arginine


vasopressin in the treatment of diabetes insipidus for all of the following
reasons EXCEPT
A. It is a more potent antidiuretic
B. It is longer acting
C. It has a little vasoconstrictor activity
D. It is a selective vasopressin V2 receptor agonist

526. Desmopressin can be used for all of the following conditions Except
A. Neurogenic diabetes insipidus
B. Bleeding due to hemophilia
C. Bed wetting in children
D. Nephrogenic diabetes insipidus

527. All of the following indication of V2 receptor EXCEPT


A. Esophageal varices
B. Bed wetting in children
C. von-Willebrand’s disease
D. Central diabetes insipidus

528. Vasopressin decreases the volume of urine primarily by causing


A. Decrease in glomerular titration rate
B. Increase in water permeability
C. Decrease in water permeability
D. Decrease in renal blood How

529. A 46-year-old male, Jorawar Singh presented to the emergency with


muscle weakness and cramping. He has been taking hydrochlorothiazide for
recently diagnosed hypertension. Which of the following is the most likely
cause of his symptom?
A. Hypokalemia
B. Hyponatremia
C. Hypocalcemia
D. Hypoglycemia
530. A 50-year-old has a history of frequent episodes of renal colic with high
calcium renal stones. The most useful diuretic in the treatment of recurrent
calcium stones is
A. Furosemide
B. Spironolactone
C. Acetazolamide
D. Hydrochlorothiazide

531. Epleronone is
A. Aldosterone antagonist
B. Can cause hyperkalemia in predisposed patients
C. A diuretic
D. All of these

532. Which of the following is NOT associated with thiazide diuretics?


A. Hyperuricemia
B. Hyponatremia
C. Hypokalemia
D. Hypercalciuria

533. AU of the following statements about amiloride are true EXCEPT


A. It decreases calcium loss in the urine
B. It is the drug of choice for the treatment of lithium induced diabetes insipidus
C. It antagonises the action of aldosterone
D. It is more potent than triamterene

523. b 524. a 525. d 526. d 527. a 528. b 529. a 530. d 531. d

532. d 533. c

534. Amiloride differs from spironolactone in that.


A. It acts from the luminal membrane side of the distal tubular cells
B. Its diuretic action is more in the presence of conditions with elevated aldosterone
levels
C. It has greater natriuretic action
D. It can cause hypokalemia on long term use

535. Amiloride can cause hyperkalemia due to its action on


A. Electrogenic K+ channels
B. H+ -K+-ATPase
C. Non Electrogenic Na’, C1-symporter
D. Electrogenic Na+ channels

536. Both loop diuretics and thiazides can cause hypokalemia by


A. Increasing the availability of Na+ in the distal tubular fluid to exchange with
interstitial K+
B. Inhibiting Na-K-2C1 cotransport in the ascending limb of loop of Henle
C. Inhibiting proximal tubular K reabsorption
D. Potentiating the action of aldosterone

537. Most potent loop diuretic is


A. Furosemide
B. Ethacrynic Acid
C. Torsemide
D. Bumetanide

538. Intravenous furosemide is used for rapid control of symptoms in acute


left ventricular failure. It provides quick relief of dyspnoea by
A. Causing venodilation
B. Causing rapid diuresis and reducing circulating blood volume
C. Producing bronchodilation
D. Stimulating left ventricular contractility

539. A 60-year-old male Suresh is hospitalized with muscle pain, fatigue and
dark urine. His past medical history is significant for stable angina. The
patient’s medications include atenolol, atorvastatin, and aspirin. His urine
dipstick test is positive for blood but urine microscopy did not reveal RBCs in
the urine. Serum creatinine kinase was significant raised in this person. The
addition of which of the following medications is most likely to have
precipitated this patient’s condition?
A. Griseofulvin
B. Rifampin
C. Erythromycin
D. Azithromycin

540. The only drug who’s over dosage is not characterized by both
hypotension and bradycardia is
A. Theophylline
B. Verapamil
C. Clonidine
D. Propanolol

541. Surinder Singh developed acute CHF and was put on digitalis therapy.
ECG of this patient revealed the presence of ventricular extrasystoles.
Which of the following drugs can be administered safely to this patient in
order to counteract this arrhythmia?
A. Amiodarone
B. Quinidine
C. Atropine
D. Lignocaine

542. In deciding on a treatment for a 60 years old patient, Golu, who has
chronic heart disease and rheumatoid arteritis, you wish to give him
procainamide. He is already taking digoxin, hydrochlorothiazide and
potassium supplementation. Which of the following is a relevant statement?
A. Hyperkalemia should be avoided to reduce the likelihood of procainamide toxicity
B. A possible drug interaction with digoxin suggest that digoxin blood levels should
be obtained before and after starting procainamide
C. Procainamide cannot be used if the patient has asthma because it has a beta
blocking effect
D. Procainamide is not active by the oral route
534. a 535. d 536. a 537. d 538. a 539. c 540. a 541. d 542. a

543. A drug effect that is produced by therapeutic doses of both Timolol and
amiodarone is blockade of
A. Beta-1 adrenoceptor
B. Cardiac K+ channels
C. Cardiac Na+ channels
D. Alpha-adrenoceptor

544. Which of the following drugs hyperpolarizes AV nodal tissue, is used in


PSVT and has short lasting adverse effects?
A. Verapamil
B. Adenosine
C. Digoxin
D. Propanolol

545. Which of the following statements is WRONG about amiodarone?


A. It is longest acting anti-arrhythmic drug.
B. It causes pulmonary fibrosis.
C. It contains iodine
D. It has positive ionotropic action.

546. Lignocaine is useful for the treatment of


A. Digitalis induced ventricular extrasystoles
B. Paroxysmal supraventricular tachycardia
C. Atrial fibrillation
D. All of the above

547. All of the following statements are true about quinidine except
A. It blocks myocardial Na+ channels primarily in the open state
B. It delays recovery of myocardial Na+ channels
C. It produces frequency dependent blockade of myocardial Na+ channels
D. It has no effect on myocardial K+ channels

548. Which of the following antiarrhythmic drugs can decrease the slope of
Phase 0 and prolong the action potential duration?
A. Quinidine
B. Propanolol
C. Lignocaine
D. Adenosine
549. All of the following drugs can be used for the chronic oral treatment of
arrhythmia except
A. Amiodarone
B. Quinidine
C. Esmolol
D. Verapamil

550. Urgent treatment of procainamide toxicity is


A. Calcium chelation
B. Sodium lactate
C. Nitroprusside
D. KCI

551. A patient on nitroprusside therapy developed cyanide toxicity. Sodium


nitrate was administered i.v. to combat this poisoning. Beneficial effect of
sodium nitrite in this case is dependent on
A. Conversion of hemoglobin to methemoglobin by sodium nitrate
B. Vasodilatation caused by sodium nitrite
C. Direct chelation of cyanide with sodium nitrite
D. Facilitation of cyanocobalamin formation by sodium nitrate

552. A patient of acute myocardial infarction being treated in intensive care


unit developed left ventricular failure with raised central venous pressure. It
was decided to use nitroglycerine. which route of administration would be
most suitable?
A. Sublingual
B. Slow intravenous infusion
C. Intravenous bolus injection
D. Oral

553. Which of the following drugs is capable of maintaining blood levels for 24
hours after a single administration but has useful antianginal effects lasting
only about 10 hours?
A. Nitroglycerine (transdermal)
B. Isosorbide mononitrate
C. Nitroglycerine (sublingual)
D. Amyl nitrite
543. b 544. b 545. d 547. d 548. a 549. c 550. b 551. a 552. b

553. a

554. A drug lacking vasodilatory properties that is effective in angina is


A. Isosorbide dinitrate
B. Verapamil
C. Nifedipine
D. Metoprolol
555. Which of the following diuretic acts by inhibiting Na+-Cl symport
A. Furosemide
B. Acetazolamide
C. Spironolactone
D. Chlorthalidone

556. Loop diuretics causes ‘postdiuretic Na+ retention due to their short half-
life, this can be overcome by
A. Restricting dietary Na+ intake
B. Moe frequent administration of the loop diuretic
C. Both A and B
D. Adding Potassium sparing diuretic

557. Though the drug controlled hypertension on but it reduced resting heart
rate to 50/min. which of the following B blockers can be used in this patient
as an effective substitute which DOES NOT cause bradycardia
A. Labetalol
B. Bisoprolol
C. Pindolol
D. Atenolol

558. Which of the following acts by inhibiting late Na channels in myocardium


A. Trimetazidine
B. Ivabradine
C. Sildenafil
D. Ranolazine

559. Which of the following antianginal drug may increase the risk of
movement disorders
A. Ivabradine
B. Ranolazine
C. Trimetazidine
D. Fasudil

560. Which of the following has nitrate-like properties and acts as an agonist
at ATP sensitive potassium channels
A. Nifedipine
B. Nicorandil
C. Nitroglycerine
D. Nimesulide

561. The main advantage of giving Glyceryl nitrate by sublingual route is


A. It avoids tolerance
B. Has highest lipid solubility
C. It bypasses first pass metabolism when given by sublingual route
D. All of the above
562. A patient was treated with PDE 5 inhibitor for erectile dysfunction. His
physician decides to start him with nitrates for angina. He gives a history
that he has taken PDE 5 inhibitor 24 hrs. back. In this scenario the patient
may be started with nitrates if he has taken following PDE 5 inhibitor
A. Sildenafil
B. Vardenafil
C. Tadalafil
D. Both A & B

563. GTN should not be administered in case of MI in the below conditions


except
A. Left ventricular infraction is suspected
B. Systolic BP is < 90 mm Hg
C. Heart rate is < 50 or < 100 beats/min
D. Patient has taken sildenafil in the past 24 hours

564. Molsidomine is a
A. Phosphodiesterase inhibitor
B. Potassium channel opener
C. Direct NO donor
D. Steroid synthesis inhibitor

565. Treatment of choice for digitalis induced ventricular arrhythmias is


A. Esmolol
B. Quinidine
C. Disopyramide
D. Lignocaine

554. d 555. d 556. c 557. c 558. d 559. c 560. b 561. c 562. d

563. a 564. a 565. d

566. GTN should not be administered in case of MI in the below conditions


except
A. Left ventricular infraction is suspected
B. Systolic BP is < 90 mm Hg
C. Heart rate is < 50 or < 100 beats/min
D. Patient has taken sildenafil in the past 24 hours

567. Following anti-angina drug has cardio protective Ischaemic


preconditioning
A. Ranolazine
B. Nicorandil
C. Trimetazidine
D. Dipyridamole

568. Nicorandil acts on


A. Voltage dependent K+ channel
B. Receptor operated K+ channel
C. Na+ activated K+ channel
D. ATP sensitive K+ channel

569. The following drugs are used in treatment of Liddle syndrome except
A. Acetazolamide
B. Triamterene
C. Spironolactone
D. None of the above

570. The following statements are true about pentoxyphylline except


A. It does not cause coronary steal phenomenon
B. It decreases serum theophylline levels when used together
C. Inhibits production of TNF alpha
D. Used to improve sperm motility

571. A patient was on an anti-angina drug. He had difficulty in driving his car
in the night due to disturbances in nocturnal vision with flashing lights. The
drug known to cause this is
A. Trimetazidine
B. Ranolazine
C. Ivabradine
D. Nicorandil

572. A patient is on propranolol for performance anxiety, which of the


following drugs should not be given to this patient
A. Nifedipine
B. Nitrates
C. Verapamil
D. ACE inhibitors

573. Which of the following is not used in atrial arrhythmias


A. Verapamil
B. Mexiletine
C. Esmolol
D. Quinidine

574. DOC for ventricular arrhythmias due to digitalis toxicity is


A. Quinidine
B. Amiodarone
C. Lignocaine
D. Verapamil

575. An asthmatic present to the emergency department with PSVT, what will
be the drug used for its management
A. Adenosine
B. Nifedipine
C. Digoxin
D. Verapamil

576. Arrhythmias Refractory to the treatment of Lignocaine can be treated by


A. Sotalol
B. Amiodarone
C. Diltiazem
D. Quinidine

577. The proposed therapeutic plasma amiodarone concentration range is


A. 0.5 to 2 ug/ m L
B. 0.25 to 0.5 ug/ mL
C. to 0.20 ug/ mL
D. to 4 ug/ Ml

578. The most common adverse effect of Quinidine is


A. Hepatitis
B. Thrombocytopenia
C. Bone marrow depression
D. Diarrhea

579. Which of the following is incorrect regarding Procainamide induced lupus


syndrome
A. The symptoms of procainamide-induced lupus resolve during treatment with N-
acetyl Procainamide
B. The lupus-like symptoms resolve on cessation of therapy
C. Therapy should not be stopped because of the presence of antinuclear antibodies
only without symptoms
D. It is more common in rapid acetylators than slow acetylators
566. a 567. b 568. d 569. c 570. b 571. c 572. c 573. b 574. c

575. d 576. b 577. a 578. d 579. d

580. Which of the following drugs can be cause QT prolongation


A. Lignocaine
B. Esmolol
C. Quinidine
D. Flecainide

581. Which of the following Beta blocker is both class II and class III anti-
arrhythmic drugs as per Vaughan-Williams classification
A. Propranolol
B. Esmolol
C. Sotalol
D. Atenolol
582. Which of the following is a class IC anti-arrhythmic drugs as per Vaughan-
Williams classification
A. Lidocaine
B. Procainamide
C. Mexiletine
D. Propafenone

583. Carperitide is a
A. Recombinant BNP
B. Recombinant ANP
C. Synthetic form of urodilatin
D. Synthetic calcitonin

584. Which of the following beta blocker is not used in Chronic CHF
A. Metoprolol
B. Bisoprolol
C. Carvedilol
D. Nadolol

585. Which of the following is incorrect about Carvedilol


A. B blocker of choice in patients with peripheral artery disease
B. Longer acting because of its accumulation in adipose tissues
C. Ratio of a1- to B-antagonist potency for carvedilol is approximately 1:10
D. Reduces mortality in patients with congestive failure

586. Which of the following is incorrect about DIGIBIND


A. It is immunogenic
B. Used for treating digoxin toxicity
C. Given by IV infusion
D. Digoxin-Digibind complex is rapidly excreted by kidney

587. Digoxin toxicity is increased by all except


A. Renal impairment
B. Hypercalcemia
C. Hyperkalemia
D. Hypomagnesemia

588. Digoxin undergoes relatively slow distribution to effector site(s), so in


order to achieve its antiarrhythmic effect, loading dose should be given with
in
A. 6 hrs
B. 8 hrs
C. 24 hrs
D. 12 hrs

589. Mechanism of action of digitalis in atrial fibrillation is


A. By decreasing cardiac contractility
B. Na+ K+ ATPase inhibition
C. By causing bradycardia
D. Increase in Refractoriness of AV nodal tissue

590. Amrinone inhibits


A. PDE 4
B. PDE 5
C. PDE 2
D. PDE 3

591. Dobutamine is preferred over dopamine in cardiogenic shock because of


its effect related to
A. Better cardiac stimulation
B. Less peripheral Vasoconstriction
C. Lower risk of cardiac arrhythmias
D. More CNS stimulation

592. Following adverse effects are seen with methyldopa except


A. Gynaecomastia
B. Dryness of Mouth
C. Tremors
D. Diminution in Psychic energy

593. According to JNC VIII Criteria, Initial antihypertensive treatment in


population aged 18 years or older with CKD and hypertension is
A. ARB
B. Calcium channel blocker
C. Thiazide Diuretics
D. Beta Blockers
580. c 581. c 582. d 583. b 584. d 585. b 586. a 587. c 588. c

589. d 590. d 591. b 592. c 593. a

594. According to JNC VIII Criteria, Initial antihypertensive treatment in


general black population, including those with diabetes is
A. ARB
B. Calcium channel blocker
C. ACE inhibitor
D. Beta Blockers

595. A 59-Year-old woman was being treated with lithium for bipolar affective
disorder on examination, her BP was 160/102 mmHg. What’s the most
appropriate anti-hypertensive drug for her?
A. Thiazide
B. Doxazosin
C. Amlodipine
D. Losartan

596. The drug of choice for management scleroderma hypertensive crisis is


A. Captopril
B. Enalapril
C. Lisinopril
D. Ramipril

597. Calcium channel blocker preferred in hypertensive emergency is


A. Nifedipine
B. Belodipien
C. Amlodipine
D. Clevidipine

598. Which of the following antihypertensive drug absorption is Decreased by


high fatty meal
A. Losartan
B. Amlodipine
C. Aliskiren
D. Chlorthalidone

599. Beta-blockers are used in following disease except


A. Migraine
B. Thyrotoxicosis
C. Glaucoma
D. Hypertension arising from cocaine use

600. Antihypertensive drug that cause positive coomb’s test is


A. Clonidine
B. Methyldopa
C. Hydralazine
D. Sodium-Nitroprusside

601. Which of the following is correct regarding Hydralazine induced lupus


syndrome
A. More common in Caucasians than in African Americans
B. More common in men than women
C. Antinuclear antibody is seen faster among rapid acetylators
D. All are correct

602. Nitroprusside can worsen arterial hypoxemia in patients with chronic


obstructive pulmonary disease because
A. Nitroprusside interferes with hypoxic pulmonary vasoconstriction
B. Promotes mismatching of ventilation with perfusion
C. Both are correct
D. Both are false

603. What is the rationale to administer a beta blocker with sodium


nitroprusside while treating acute aortic dissection
A. Reduction of blood pressure with nitroprusside alone can increase the rate of rise
in pressure in the aorta as a result of increase myocardial contractility
B. Beta blockers decrease the increased myocardial contractility and prevents
propagation of aortic dissection
C. Both are correct
D. Both are false

604. Thiazide diuretics as anti-hypertensive are to avoided in


A. Older patients
B. Pregnancy induced hypertension
C. Isolated systolic hypertension
D. Obese with volume overload

605. Beta blockers are suitable antihypertensive for following conditions


except
A. Coexisting anxiety
B. Tachycardia
C. Migraine patients
D. Reynaud’s disease

594. b 595. c 596. a 597. d 598. c 599. d 600. b 601. a 602. c

603. c 604. b 605. d

606. Ankle edema due to Nifedipine is due to


A. Fluid retention
B. Greater dilatation of precapillary Vessels than post capillary vessels
C. Water and sodium reabsorption at tissue
D. None of the above

607. MOST selective ET A antagonist is


A. Bosentan
B. Macitentan
C. Ambrisentan
D. Sitaxsentan

608. Antihypertensive drug contraindicated in patients with allergy to soy or


egg is
A. Nicardipine
B. Esmolol
C. Clevidipine
D. Enalaprilat
609. Recommended drug for hypertensive emergency with intracerebral
hemorrhage and systolic blood pressure> 140-160 mm Hg is
A. Nitroprusside
B. Methyldopa
C. Clonidine
D. Labetalol

610. Cough and angioedema in a patient receiving ACE inhibitors is due to


A. Brandykinin
B. Renin
C. Angiotensin II
D. All

611. Which of the following ACE inhibitor is NOT a prodrug


A. Fosinopril
B. Enalapril
C. Ramipril
D. Lisinopril

612. Angiotensin receptor blocker with additional PPAR-r-agonist activity is


A. Losartan
B. Candisartan
C. Telmesartan
D. Eprosartan

613. ACE inhibitor decrease renal micralbumuruia by following mechanism


A. Inhabitation of cytoskeletal rearrangements
B. Inhabitation of retraction of podocyte processes
C. Inhabitation of reduction in proteins of the slit diaphragm
D. All of the above

614. A metabolite of an ARB reducing COX-2 mRNA up regulation among the


following is
A. Telmisartan
B. Candesartan
C. Olmesartan
D. Losartan

615. ARB having highest affinity for AT1 receptor among the following is
A. Olmesertan
B. Irbesartan
C. Telmisartan
D. Losartan

616. A patient with gout needs to be started in an ARB, which one of them is
best to this patient
A. Telmisartan
B. Olmesertan
C. Losartan
D. Irbesartan

617. Enzymes activating prodrugs of ACE inhibitors of active form is


A. Amidases
B. Esterases
C. Angiotensin converting enzyme
D. Conjugases

618. Among the following ACE inhibitor with Longest half-life is


A. Enalapril
B. Lisinorpril
C. Ramipril
D. Perindopril

619. Which of the following medications would be prescribed most frequently


to patients suffering from chronic atrial fibrillation?
A. Warfarin
B. Bretylium
C. Lidocaine
D. Adenosine

606. b 607. c 608. c 609. d 610. a 611. d 612. c 613. d 614. d

615. a 616. c 617. b 618. d 619. a

620. A 64-year-old male had an inferior STEMI and was treated with aspirin,
beta-blockers, an angiotensin-converting enzyme inhibitor, and a statin. He
was standing at the nurse’s station when telemetry showed the following:
Wenckebach. Which of the following should you do?
A. Decrease the beta-blocker dose.
B. Refer for coronary angiography.
C. Refer for permanent pacemaker implantation.
D. Insert a temporary trans venous pacemaker.

621. All of the following can be a side effect of digitalis toxicity except:
A. Gingival hyperplasia
B. Confusion
C. Blurred vision
D. Nausea and vomiting

622. A 70-year-old woman had a prosthetic hip joint placed 6 years ago. She
has not been to the dentist in years and has decided that maybe she should
make a visit. She read on the internet that some people need to take
antibiotics before they go to the dentist. She is penicillin-allergic
(anaphylaxis). Based on her history so far, which of the following is correct
about whether prophylaxis is warranted?
A. Yes, she should take amoxicillin 2 grams 1 hour before her visit.
B. No, presence of prosthetic joints does not require antibiotic prophylaxis.
C. Yes, she should take clindamycin 600 mg 1 hour before her visit.
D. Yes, she should take cephalexin 2 grams 1 hour before her visit.
E. No, presence of prosthetic joints does not require antibiotic prophylaxis.

623. A 35-year-old woman is 16-weeks pregnant. Her BP is 170/110, and her


urinalysis shows she has 2+ proteinuria. Otherwise, she has no complaints
and is Referred by her obstetrician for treatment of her hypertension. Of the
following, which is the best agent to use?
A. Hydrochlorothiazide
B. Ramipril
C. Methyldopa
D. Nitroprusside drip

624. An ECG for a 43-year-old female demonstrates the following rhythm:


Torsades. Which of the following electrolyte abnormalities is most likely to
cause this rhythm?
A. Hypocalcemia
B. Hypernatremia
C. Hyperkalemia
D. Hypomagnesemia

625. You are called to admit a 32-year-old male who presented to the
emergency department with complaints of chest pain. He is admitted to a
monitored floor and, during the course of hospitalization, he is shown to
have experienced a myocardial infarction. A homocysteine level is found to
be very high. Which of the following is not routinely included in the
treatment of an acute MI in this young patient?
A. Beta-blockers
B. B vitamin supplements
C. Aspirin
D. ACE inhibitors

626. A 50-year-old man presents to the emergency department with a history


of crescendo-pattern chest pain and is admitted to the CCU. While in the
CCU, he develops more chest pain, and the nursing staff records a 12-lead
ECG that shows 15-mm ST-segment elevation in leads V3 and V5 with
additional lesser elevations in leads I, aVL, and V6. These changes revert to
normal almost immediately following the administration of sublingual
nitroglycerin. Based on your clinical and ECG findings, which of the following
medications would be contraindicated in this patient?
A. Propranolol
B. Isosorbide dinitrate
C. Diltiazem
D. Nifedipine
620. a 621. a 622. e 623. c 624. d 625. b 626. a

627. A patient Amit Kumar is suffering from atherosclerosis. Which of the


following is the most beneficial drug for prevention of stroke in this patient?
A. Digoxin
B. Warfarin
C. Low dose subcutaneous heparin
D. Aspirin

628. An old woman. Nanda suffered stroke for which she was given Alteplase.
She improved considerably. To prevent the recurrence of stroke, this patient
is most likely to be treated indefinitely with
A. Urokinase
B. Warfarin
C. Aspirin
D. Enoxaparin

629. Which of the following statements regarding ticlopidine is TRUE?


A. It blocks Gp1ib/Ma receptors on platelet membrane
B. It does not prolong bleeding time
C. It inhibits thromboxane A2 synthesis in platelets
D. It prevents ADP mediated platelet adenylyl cyclase inhibition

630. Drugs used in acute myocardial infarction are all EXCEPT


A. Antiplatelet drugs
B. Thrombolytics
C. Plasminogen activator inhibitors
D. Alteplase

631. With respect to ticlopidine, clopidogrel


A. ls more likely to cause formation of antiplatelet antibodies
B. Has a greater antiplatelet effect
C. Is more likely to cause severe bleeding
D. Is less likely to cause neutropenia

632. An old man, Om Prakash presented with anorexia, weakness and


Paranesthesia. On further investigation his hemoglobin came out to be 5. 8
g% and the peripheral smear showed the presence of macrocytic and
neutrophils having hyper segmented nuclei. His tendon Reflexes also were
sluggish. Endoscopy revealed atrophic gastritis. Deficiency of which of the
following factors can lead to such a clinical situation?
A. Folic acid
B. Riboflavin
C. Pyridoxine
D. Vitamin B

627. d 628. c 629. d 630. c 631. d 632. d


Cardiovascular System — Explanations
466. A. It is an inhibitor of fatty acid oxidation
Trimetazidine is PFOX[ partial fatty acid oxidation] inhibitor suppose to stop progession
of angina by inhibiting free radicals generation. It is used as 2nd line drug in angina.
Adverse effect of this drug include increased risk of movement disorders. Thus, option
A is correct. Ref: Katzung; Topic: angina

467. D. Verapamil
Both verapamil and propanolol decrease the conduction through AV node and their
concomitant use can result in heart block.
Thus, option D is correct. Ref: KDT, Topic: angina

468. D. It acts by conversion to an active metabolite in the liver


Amlodipine is the longest acting DHP. It undergoes little first pass metabolism and is
distributed widely. Metabolites of amlodipine are inactive.
Thus, option D is correct. Ref: KDT, Topic: calcium channel blockers

469. D. Nifedipine
Vasodilators and peripherally acting CCBs can cause tachycardia and thus may
precipitate acute attack of angina. Risk of tachycardia is more with short acting drugs
like Nifedipine than with long acting agents like amlodipine.
Thus, option D is correct. Ref: KDT, Topic: angina

470. B. Sildenafil
Nitrates release NO, that acts by increasing cGMP. Phosphodiesterase inhibitors like
sildenafil inhibits the breakdown of cGMP. Concomitant use of these two drugs may
result in profound hypotension.
Thus, option B is correct. Ref: KDT, Topic: angina

471. D. Sublingual nitroglycerine


Tolerance develops to nitrates, when these are present constantly in the blood.
Sublingual route leads to immediate action of nitrates and these act for a shad time.
Other preparations mentioned in the question are more likely to give consistent
plasma levels of nitrates leading to development of tolerance.
Thus, option D is correct. Ref: KDT, Topic: angina

472. A. Reduction of cardiac preload


Mechanism of action of nitrates in classical angina is reduction in preload (due to
venodilation) and in variant angina is coronary vasodilation.
Thus, option A is correct. Ref: KDT, Topic: angina

473. D. It preferentially dilates conducting arteries without affecting


resistance arterioles
Nitrates cause redistribution of blood flow without affecting total coronary flow. Auto
regulatory small vessels remain unaffected whereas large conducting vessels are
dilated. Nitrates are predominantly venodilator, therefore decrease mainly preload.
Thus, option D is correct. Ref: KDT, Topic: angina
474. C. Isosorbide dinitrate
It can be administered sublingually for the treatment of acute attack of angina
whereas by oral route it is used for prophylaxis.
Pentaerythrital tetranitrate and diltiazem are used only for prophylaxis of angina.
Dipyridamole actually worsens angina by causing coronary steal phenomenon.
Thus, option C is correct. Ref: KDT, Topic: angina

475. C. lsosorbide Mononitrate


It is a metabolite of Isosorbide dinitrate and is also available as a separate drug. It
undergoes least first pass metabolism.
Thus, option C is correct. Ref: KDT, Topic: angina

476. A. Hyperglycemia
CCBs are safe in diabetic patients whereas thiazides and 0-blockers should be avoided.
Thus, option A is correct. Ref: KDT, Topic: Calcium channel blockers

477. A. Organic nitrates


Nitrates act by increasing cGMP and sildenafil inhibits the breakdown of this
compound (by, inhibiting phosphodiesterase). Marked increase in cGMP level may
result in the profound hypotension and Reflex tachycardia.
Thus, option A is correct. Ref: KDT, Topic: angina

478. A. Atenolol
Tense personality and high resting heart rate (96/min.) makes B-blockers ideal
candidate drugs to be used in this case. As blood sugar and lipid profile is normal,
beta blockers can be used safely.
Thus, option A is correct. Ref: KDT, Topic: angina

479. D. Enalapril
Thiazides and B-blockers are contra-indicated in diabetic patients. As the patient has
frequent travelling job, he is likely to miss the doses. Therefore, clonidine should be
avoided to prevent rebound hypertension. ACE inhibitors are safe and effective agents
in diabetic patients with hypertension.
Thus, option D is correct. Ref: KDT, Topic: antihypertensives

480. D. Hydralazine
It is a vasodilator and can result in Reflex tachycardia. All other drugs listed in the
question (clonidine, propanolol, reserpine) decrease the activity of sympathetic system
and can result in bradycardia.
Thus, option D is correct. Ref: KDT, Topic: antihypertensives

481. D. Minoxidi
Vasodilators can lead to Reflex increase in the sympathetic activity. Activation of 0,
receptors in the heart can cause tachycardia and release of renin can result in
retention of salt and water.
Thus, option D is correct. Ref: KDT, Topic: hypertension

482. B. A 56-year-old diabetic woman


ACE inhibitors are contra-indicated in bilateral renal artery stenosis, pregnancy and in
the setting of hyperkalemia.
Thus, option B is correct. Ref: KDT, Topic: hypertension

483. D. Guanethidine
Postural hypotension cause by a-blockers is called first dose hypotension because
tolerance occurs to this adverse effect on chronic use. On the other adrenergic neuron
blockers continue to produce orthostatic hypertension even on chronic use.
Thus, option D is correct. Ref: KDT, Topic: hypertension

484. C. Prazosin
Methyldopa and clonidine act via decreasing the central sympathetic outflow. Beta
blockers act by several mechanisms, one of which is decrease in central sympathetic
outflow. Alpha blockers like Prazosin, do not act via central mechanism.
Thus, option C is correct. Ref: KDT, Topic: hypertension

485. C. Methyldopa
Although both methyldopa and minoxidil are prodrugs, conversion of methyldopa to a-
methyl norepinephrine takes place in the brain whereas minoxidil is activated to
minoxidil sulphate (by phase II reaction) in the periphery.
Thus, option C is correct. Ref: KDT, Topic: hypertension

486. D. Clonidine
Plasma renin activity is increased by reflex increase in sympathetic discharge.
Clonidine decreases central sympathetic outflow and thus will decrease the plasma
renin activity. Vasodilators and ACE inhibitors result in reflex increase in plasma renin
activity.
Thus, option D is correct. Ref: KDT, Topic: hypertension

487. B. Agonistic action on vascular a2 adrenergic receptors


Alpha 2 receptors are present both presynaptically as well as postsynaptically.
Clonidine acts as an a, agonist and decreases the central sympathetic outflow.
However, it can also act on vascular postsynaptic a, receptors, leading to
vasoconstriction and increase in blood pressure. Chances of this reaction is when
clonidine is administered by quick i.v. injection.
Thus, option B is correct. Ref: KDT, Topic: hypertension

488. C. Tachycardia and increased cardiac contractility


Minoxidil is a powerful vasodilator and leads to Reflex stimulation of sympathetic
system. Thus, it causes tachycardia, increased contractility and retention of and water
by the kidney.
Thus, option C is correct. Ref: KDT, Topic: hypertension

489. C. Decreased angiotensin II concentration in the blood


Enalapril inhibits ACE responsible for the conversion of angiotensin Ito angiotensin II.
There is a reflex increase in renin activity. Inhibition of aldosterone auction decreases
the reabsorption of Na* and excretion of K’ in the urine. It can result in hyperkalemia.
Thus, option C is correct. Ref: KDT, Topic: hypertension

490. D. Increased PR interval on the ECG


Digitalis possesses vagornimetic action and can cause bradycardia and decreased av
conduction. This latter effect is manifested in the ECG as increased PR interval.
Atropine can block this adverse effect.
Thus, option D is correct. Ref: KDT, Topic: CHF

491. C. Potassium
Hypokalemia precipitates digitalis toxicity but in acute severe digitalis toxicity, there is
already hyperkalemia.
Thus, option C is correct. Ref: KDT, Topic: CHF

492. B. Increase in Refractoriness of AV nodal tissue


Digitalis acts in CHF by inhibiting Na+ K+ -ATPase whereas in AF, it acts by decreasing
the conduction through AV node.
Thus, option B is correct. Ref: KDT, Topic: CHF

493. D. These are drug of choice in acute decompensated heart failure


Beta blockers are contra-indicated in acute decompensated heart failure. For more
details, Refer to text.
Thus, option D is correct. Ref: KDT, Topic: CHF

494. B. Nesiritide
ACE inhibitors and AT, antagonists decrease angiotensin II activity.
Omapatrilat is a vasopeptidase inhibitor that acts by inhibiting two enzymes, ACE
and NEP. As it inhibits ACE, it may also decrease angiotensin II activity.
Nesiritide is a recombinant BNP and has no effect on angiotensin H activity.
Thus, option B is correct. Ref: Katzung, Topic : CHF

495. A. It affords rapid symptomatic relief in CHF patients


Loop diuretics are used to provide rapid symptomatic relief whereas aldosterone
antagonists decrease the mortality by reversing myocardial remodeling.
Thus, option A is correct. Ref: KDT, Topic: CHF

496. A. Amrinone
It is a phosphodiesterase inhibitor and acts as an INODILATOR. It has positive
ionotropic effect (increase cardiac contractility) as well as vasodilator action decrease
preload and afterload).
Thus, option A is correct. Ref: KDT, Topic: CHF

497. D. Dobutamine
Dobutamine is indicated only for the acute treatment of decompensated CHEF. Cardiac
glycosides (digoxin) can be used for acute treatment as well as maintenance therapy
of CHF’. ACE inhibitors and spironolactone are indicated only for chronic CHF.
Thus, option D is correct. Ref: KDT, Topic: CHF

498. D. Antagonism of vasoconstriction due to sympathetic over activity


Beta blockers may actually increase vasoconstriction by antagonizing the vasodilator
action of B2, receptors.
Thus, option D is correct. Ref: KDT, Topic: CHF
499. D. Increase in atrial fibrillation frequency, but decrease in ventricular
rate
Digitalis controls the ventricular rate in AF patients. Frequency of atrial fibrillation may
actually increase. Ibutilide is approved for the conversion of AF to sinus rhythm.
Thus, option D is correct. Ref: KDT, Topic: CHF

500. B. Atrial fibrillation


Digitalis is used to maintain ventricular rate in this condition.
Thus, option B is correct. Ref: KDT, Topic: CHF

501. C. It is used Io provide relief of symptoms


Digitalis and diuretics are used as symptomatic therapy of CHF. These drugs do not
retard ventricular remodeling or prolong survival in CHF patients.
Thus, option C is correct. Ref: KDT, Topic: CHF

502. D. Glomerular filtration


Digoxin is primarily eliminated unchanged by glomerular filtration whereas digitoxin is
eliminated by hepatic metabolism.
Thus, option D is correct. Ref: KDT, Topic: CHF

503. B. Beta receptor agonists


These agents are indicated for the treatment of acute decompensated CHF. Digitalis
(Na+ - ATPase inhibitor) is indicated both for acute treatment as well as maintenance
in CHF as digoxin is the only orally effective inotropic drug available.
Thus, option B is correct. Ref: KDT, Topic: CHF

504. C. Competitively inhibits Carbonic anhydrase in PT


Acetazolamide:
Noncompetitively but reversibly inhibits Carbonic anhydrase in Proximal tubule
Proximal tubular epithelial cells are richly endowed with the zinc metalloenzyme
carbonic anhydrase, which is found in the luminal and basolateral membranes
Carbonic anhydrase inhibitors act on the luminal and basolateral enzymes
Carbonic anhydrase plays a key role in NaHCO3 reabsorption and acid secretion
Acetazolamide is a sulfonamide derivatives and to be avoided in sulfa drug allergy
patients
Thus, option C is correct. Ref: GG, The Pharmacological Basis of Therapeutics, Topic
Diuretics

505. C. Isosorbide

Drug Half Life (in Hrs)

Glycerine 0.5 to 0.75

Isosorbide 5-9.5

Mannitol 0.25-1.7

Urea 1.2

Thus, option C is correct. Ref: GG, The Pharmacological Basis of Therapeutics; Topic
Diuretics
506. C. Glycerin
Glycerin:
An osomotic diuretic
It can be metabolized and can cause hyperglycemia
Osmotic diuretics are contraindicated in patients who are anuric owing to severe renal
disease
Thus, option C is correct. Ref: GG, The Pharmacological Basis of Therapeutics; Topic
Diuretics

507. A. Decrease plasma levels of triglycerides


Loop diuretics:
Hyperuricemia
Hyperglycemia
Increase plasma levels of LDL, cholesterol and Triglycerides
Decreasing plasma levels of HDL cholesterol
Thus, option A is correct. Ref: GG, The Pharmacological Basis of Therapeutics; Topic
Diuretics

508. C. Ototoxicity occurs least frequently with oral administration


Ototoxicity occurs most frequently with rapid intravenous administration
The rate of furosemide infusions should not exceed 4 mg/min to avoid ototoxicity
Most ototoxic loop diuretic is ethacrynic acid
Thus, option C is correct.
Ref: GG, The Pharmacological Basis of Therapeutics; Topic Diuretics

509. D. Enalapril
Spironolactone and Enalapril both cause hyperkalemia, therefore they should not be
given together
Chlorthiazide cause Hypokalemia and Spironolactone cause hyperkalemia, so both
can be combines to counteract thiazide induced Hypokalemia
No contraindication with Atenolol and verapamil with Spironolactone
Other important drug interactions of Spironolactone:
Aspirin blocks spironolactone action by inhibiting tubular secretion of its active
metabolite canrenone
Increases plasma digoxin concentration
Thus, option D is correct. Ref: KDT, Topic Diuretics

510. D. Hyperlipidemia
Thiazide diuretics uses are:
Hypertension
EDEMA (cardiac, liver, renal)
Idiopathic hypercalciurea with nephrocalcinosis
Congestive Heart Failure
Diabetes Insipidus BOTH central and peripheral
Osteoporosis
Hyperlipidemia is an adverse effect of thiazide diuretic
Thus, option D is correct. Ref: KDT, Topic: diuretic

511. C. Both A and B


Conivaptan:
Vasopressin antagonist
DIURETIC
Used in SIADH
Rolophylline
Adenosine A1 receptor blocker
DIURECTIC
Tried for CHF
Thus, option C is correct. Ref: GG, Topic: Antidiuretics

512. B. Acetazolamide
Metabolic acidosis with Hypokalemia is seen with acetazolamide
Metabolic acidosis with Hyperkalemia is seen with Amiloride
Metabolic alkalosis with Hypokalemia is seen with Torsemide and Chlorthalidone
Thus, option B is correct. Ref: KDT, Topic: diuretic

513. C. Spironolactone
Liddle syndrome (Pseudoaldosteronism):
An autosomal dominant disorder
Mutations in the B or Y subunits, leading to increased basal amiloride-sensitive
sodium channel activity
Due to hyperactivity of the amiloride-sensitive, sodium channel of the principal cell of
the cortical collecting tubble.
It includes early-onset severe hypertension, hypokalemia, metabolic alkalosis & low
renin and aldosterone
Treatment of Liddle syndrome with amiloride or triamterene lowers blood pressure
and corrects the hypokalemia and acidosis
Spironolactone is not an effective treatment because the increased
Thus, option C is correct. Ref: GG, Topic: diuretic

514. B. Potassium Canrenoate


Aldosterone antagonists and oral bioavailability
Spironolactone 65%
Canrenone 80%
Potassium Canrenoate 100%
Epleronone 69%
Thus, option B is correct. Ref: GG The Pharmacological basis of Therapeutics Topic:
diuretic

515. A. Triamterene
MOA of triamterene:
Amiloride and triamterene block Renal Epithelial Na+ Channels in the luminal
membrane of principal cells in late distal tubules and collecting ducts by binding to a
site in the channel pore.
Thus, option A is correct. Ref: GG The Pharmacological basis of Therapeutics Topic:
diuretic

516. B. Thiazide-induced diabetes mellitus is associated with the same


cardiovascular disease risk as incident diabetes
Thiazide diuretics are known to cause hyperglycemia
Thiazide diuretics also decrease glucose tolerance and unmask latent diabetes
mellitus
The mechanism of impaired glucose tolerance appears to involve
Reduced insulin secretion and alternations in glucose metabolism
Hyperglycemia is reduced when K+ is given along with the diuretic
Thiazide-induced diabetes mellitus is not associated with the same cardiovascular
disease risk as incident diabetes
Thus, option B is correct. Ref: GG The Pharmacological Basis of Therapeutics Topic:
diuretic

517. B. Hypermagnesemia
Thiazide related to abnormalities of fluid and electrolyte balance extracellular volume
depletion
Hypotension
Hypokalemia
Hyponatremia
Hypochloremia
Metabolic Alkalosis
Hypomagnesemia
Hypercalcemia
Thus, option B is correct. Ref: GG The Pharmacological Basis of Therapeutics 13E/P
455 Topic: diuretic

518. B. Indapamide
Most of the thiazide diuretics are ineffective when the GFR is less than 30-40 mL/min
But the possible exceptions of metolazone and Indapamide
Thus, option B is correct. Ref: GG The Pharmacological Basis of Therapeutics Topic:
diuretic

519. D. Indapamide
Thiazide and thiazide like diuretics:

Drug Relative Potency

Chlorthalidone 1

Hydrochlorthiazide 1

Chlorothiazide 0.1

Bendroflumethiazide 10

Metolazone 10

Methyclothiazide 10

Indapamide 20

Thus, option D is correct. Ref: GG, The Pharmacological Basis of Therapeutics Topic:
diuretic

520. D. Chlorthalidone
Thiazides:
Site of action in the cortical diluting segment or the early DT
Inhibit Na+-Cl symport at the luminal membrane
They enter the proximal tubule via organic acid secretory pathway
From there they reach early DT, where they bind to specific receptors located on the
luminal membrane
Spironolactone:
a. Spironolactone is an aldosterone antagonist
b. It combines with Mineralocorticoid Receptor and inhibits the formation of AIPs
Loop diuretics:
a. Major site of action: inhibits Na+-K+-2Cl cotransport at the thick Ascending Loop of
Henle
Acetazolamide:
a. Inhibits carbonic anhydrase enzyme at Proximal convulted tubule
Thus, option D is correct. Ref: GG, The Pharmacological Basis of Therapeutics Topic:
diuretic

521. C. Both A and B


Loop diuretics have short half life
As the concentration of loop diuretic in the tubular lumen declines
Nephrons begin to avidly reabsorb Na+, which often nullifies the overall effect of the
loop diuretic on total-body Na+.
This phenomenon of “postdiuretic Na+ retention” can be overcome by
Restricting dietary Na+ intake or
By more frequent administration of the loop diuretic
Thus, option C is correct. Ref: GG, The Pharmacological Basis of Therapeutics 13E/P
453 Topic: diuretic

522. C. Desmopressin
Desmopressin is ADH and will exacerbates the symptoms of SIADH. Hypertonic saline
(3%NaC1) is used for severely symptomatic patients with hyponatremia.
Demeclocycline decreases the release of ADH.
Thus, option C is correct. Ref: CMDT-Topic: diuretic

523. B. Spironolactone
Mostly likely diagnosis in this patient is aldosterone secreting tumor (adenoma)
leading to primary hyperaldosteronism (Conn’s Syndrome). Aldosterone excess will
cause hypertension, hypokalemia, metabolic alkalosis and depressed renin.
Aldosterone antagonists such as spironolactone or Epleronone can be used as medical
therapy for Conn’s syndrome.
Thus, option B is correct. Topic: diuretic

524. A. Hydrochlorothiazide
Thiazides are useful in the treatment of central as well as nephrogenic DI.
Vasopressin, Chlorpropamide and carbamazepine are useful only in central DI.
Thus, option A is correct. Ref: KDT, Topic: Diuretics

525. D. It is a selective vasopressin V2 receptor agonist


Vasopressin can act on V1 and V2 receptors whereas desmopressin is a selective V2
receptor agonist.
Thus, option D is correct. Ref: KDT, Topic: Diuretics
526. D. Nephrogenic diabetes insipidus
Desmopressin is the agent of choice for central diabetes insipidus. It has no role in
nephrogenic DI (because ADH is already present in this condition).
Pituitary DI and that due to head trauma are examples of central DI.
Li induced DI is nephrogenic. Drug of choice for this condition is amiloride.
Thus, option D is correct. Ref: KDT, Topic: Diuretics

527. A. Esophageal varices


Vasoconstriction is caused by V1 receptors whereas anti-diuresis and release of vWF
from endothelium is mediated by V, receptors of vasopressin.
Thus, option A is correct. Ref: KDT, Topic: Diuretics

528. B. Increase in water permeability


In the absence of ADH, collecting ducts are impermeable to water. ADH increases the
exocytosis and decreases the endocytosis of water channels (aquaporin’s) in the
collecting duct cells. This results in increase in permeability of collecting ducts to
water.
Thus, option B is correct. Ref: KDT, Topic: Diuretics

529. A. Hypokalemia
A decrease in the intravascular fluid volume stimulates aldosterone secretion and
leads to increased excretion of potassium and hydrogen ions in the urine. This results
in hypokalemic metabolic alkalosis, which is a common side effect of most diuretics
other than the potassium-sparing class. Hypokalemia manifests with muscle weakness
and cramping.
Thus, option A is correct. Ref: KK Sharma 2E/P 231 Topic: Diuretics

530. D. Hydrochlorothiazide
Thiazides cause hypercalcemia by decreasing the renal excretion of Ca”. These are
useful in a patient having hypercalciurea. In such a patient, thiazides decrease the
excretion of Ca” in the kidney and thus reduces the chances of stone formation.
Thus, option D is correct. Ref: KDT, Topic: Diuretics

531. D. All of these


Epleronone is an aldosterone antagonist (like spironolactone). It is used as potassium
sparing diuretic and can cause hyperkalemia in susceptible individuals particularly in
the patients on ACE inhibitors.
Thus, option D is correct. Ref: KDT, Topic: Diuretics

532. D. Hypercalciuria
Thiazides cause hypercalcemia by decreasing its excretion (hypocalciurea) whereas
loop diuretics cause hypocalcemia by increasing its excretion. [remember LOOP
LOOSES CALCIUM].
Thus, option D is correct. Ref: KDT, Topic: Diuretics

533. C. It antagonises the action of aldosterone


Amiloride acts by blocking epithelial Na. channels. It has no effect on aldosterone
receptors.
It is more potent than triamterene.
It decreases Ca” loss in the urine
It is the drug of choice for lithium induced DI.
Thus, option C is correct. Ref: KDT, Topic: Diuretics

534. A. It acts from the luminal membrane side of the distal tubular cells
Amiloride is a K+ sparing diuretic and like other diuretics, act from luminal
membrane side. On the other hand, spironolactone does not require access to tubular
lumen for action.
Diuretic action of both amiloride and spironolactone is quite feeble.
Due to its aldosterone receptor blocking action, spironolactone is more effective in
the presence of conditions with elevated levels of aldosterone (like CHF).
Both of these diuretics can cause hyperkalemia when used with K+ supplements or
ACE inhibitors.
Thus, option A is correct. Ref: KDT, Topic: Diuretics

535. D. Electrogenic Na+ channels


Amiloride and triamterene are K’ sparing diuretics that act by inhibiting epithelial Na’
channels.
In the distal tubules and collecting ducts, three separate channels are present (one
for Na+, K+ and H+ each). Aldosterone acts on DCT and CD to cause reabsorption of
Na+. This generates a lumen negative potential difference across the membrane of
this part of the nephron. To maintain the electric neutrality, K+ and If+ are secreted in
the lumen.
When amiloride and triamterene inhibits epithelial Na+ channels, trans epithelial
potential difference is not generated and therefore K+ and H+ are not secreted in the
lumen. Thus due to more retention of K’, amiloride may result in hyperkalemia.
Thus, option D is correct. Ref: KDT, Topic: Diuretics

536. A. Increasing the availability of Na+ in the distal tubular fluid to


exchange with interstitial K+
Diuretics prevent the reabsorption of Na. in various parts of tubules. Due to decreased
reabsorption, more amount of Na+ reaches the distal portion of the nephron i.e. late
DT and collecting ducts. Here, Na+ is exchanged with K+ and H+. Increased distal
delivery of Na+ results in more exchange anti-thus hypokalemia.
Thus, option A is correct. Ref: KDT, Topic: Diuretics

537. D. Bumetanide
Bumetanide is most potent and Torsemide is longest acting loop diuretic.
Thus, option D is correct. Ref: KDT, Topic: Diuretics

538. A. Causing venodilation


Furosemide is a high ceiling diuretic.
Its major mode of benefit in acute pulmonary edema is vasodilation. Due to its
vasodilatory action, it shifts the fluid from pulmonary to systemic circulation. This
results in the rapid relief of symptoms.
Diuretic action develops later.
Thus, option A is correct. Ref: KDT, Topic: Diuretics

539. C. Erythromycin
The diagnosis in this condition is rhabdomyolysis suggested by myoglobinuria (red
colored urine without RBCs) and raised creatinine kinase levels. Statins can cause
serious side effects like myopathy and hepatitis. Most statins are metabolized by
cytochrome P-450 3A4, with the exception of pravastatin. Concomitant administration
of drugs that inhibit statin metabolism (e.g. macrolides) is associated with increased
incidence of statin induced myopathy and rhabdomyolysis. Acute renal failure is a
possible sequela of rhabdomyolysis.
Thus, option C is correct. Topic: Diuretics

540. A. Theophylline
Propanolol, verapamil and clonidine cause hypotension as well as bradycardia.
Theophylline results in Reflex tachycardia due to its vasodilatory action.
Thus, option A is correct. Ref: KDT, Topic: Miscellaneaous

541. D. Lignocaine
Lignocaine is the drug of choice for ventricular arrhythmias due to digitalis toxicity.
Thus, option D is correct. Ref: KDT, Topic: CHF

542. A. Hyperkalemia should be avoided to reduce the likelihood of


procainamide toxicity
Thus, option A is correct. Ref: KDT, Topic: CHF

543. A. Beta-1 adrenoceptor


Amiodarone has wide spectrum of antiarrhythmic actions. It acts by all the four
mechanisms i.e. blockade of Na’ Na+ channels, blockade of p receptors, blockade of
IC” channels and blockade of Ca2+ channels.
Timolol is a R blocker. Therefore, common action of these. two drugs are inhibition of
13-receptors.
Thus, option A is correct. Ref: KDT, Topic: beta blocker

544. B. Adenosine
Thus, option B is correct. Ref: KDT, Topic: Arrhythmia

545. D. It has positive ionotropic action


Amiodarone possesses myocardial depressant property; not positive ionotropic action.
Thus, option D is correct. Ref: KDT, Topic: Arrhythmia

546. A. Digitalis induced ventricular extrasystoles


Lignocaine is used only for the treatment of ventricular arrhythmias; it has no role in
atrial arrhythmia.
Thus, option A is correct. Ref: KDT, Topic: Arrhythmia

547. D. It has no effect on myocardial K+ channels


Quinidine is a class I antiarrhythmic agent. It blocks Na’ channels in the open state
and also delays the recovery of these channels. Blockade by these antiarrhythmic
agents is frequency dependent. Class Ia antiarrhythmic, in addition blocks myocardial
IC’ channels also.
Thus, option D is correct. Ref: KDT, Topic: Arrhythmia

548. A. Quinidine
Na+ channel blockers reduce the slope of phase 0 whereas r channel blockers prolong
the APD. Both of these properties are present in class Ia antiarrhythmic like quinidine
and procainamide.
Thus, option A is correct. Ref: KDT, Topic: Arrhythmia

549. C. Esmolol
It is a very short acting (3-blocker. It has to be administered i.v. for acute therapy of
arrhythmias. Amiodarone, verapamil and quinidine can be used for chronic oral
treatment of arrhythmias.
Thus, option C is correct. Ref: KDT, Topic: Arrhythmia

550. B. Sodium lactate


Presser agents (to reverse hypotension) and sodium lactate (to reverse arrhythmias)
are indicated for the treatment of procainamide toxicity.
Thus, option B is correct. Ref: CMDT, Topic: Arrhythmia

551. A. Conversion of hemoglobin to methemoglobin by sodium nitrate


Cyanide has very high affinity for the enzymes of respiratory chain particularly
cytochrome oxidase. Methemoglobin possesses even higher affinity for cyanides. It can
free the enzymes due to competitive inhibition for cyanide. Nitrates act by reducing
hemoglobin to methemoglobin, which removes cyanide from the enzymes.
Thus, option A is correct. Ref: KDT, Topic: Arrhythmia

552. B. Slow intravenous infusion


Nitrates can be used in acute LVF by slow i.v. infusion.
Thus, option B is correct. Ref: KDT, Topic: AMI

553. A. Nitroglycerine (transdermal)


Continuous presence of nitrates in blood will result in tolerance.
Thus, option A is correct. Ref: KDT, Topic: Angina

554. D. Metoprolol
Beta blockers act in angina by decreasing exercise induced myocardial work. These do
not cause vasodilation. Nitrates and CCBs act by causing vasodilation.
Thus, option D is correct. Ref: KDT, Topic: Angina

555. D. Chlorthalidone
Thiazides:
Site of action in the cortical diluting segment or the early DT
Inhibit Na+-Cl symport at the luminal membrane
They enter the proximal tubule via organic acid secretory pathway
From there they reach early DT, where they bind to specific receptors located on the
luminal membrane
Spironolactone:
Spironolactone is an aldosterone antagonist
It combines with Mineralocorticoid Receptor and inhibits the formation of AIPs
Loop diuretics:
Major site of action: inhibits Na+-K+-2Cl cotransport at the thick Ascending Loop of
Henle
Acetazolamide:
Inhibits carbonic anhydrase enzyme at Proximal convulted tubule
Thus, option D is correct. Ref: GG, The Pharmacological Basis of Therapeutics 13E/P
692 Topic : Diuretics

556. C. Both A and B


Loop diuretics have short half life
As the concentration of loop diuretic in the tubular lumen declines
Nephrons begin to avidly reabsorb Na+, which often nullifies the overall effect of the
loop diuretic on total-body Na+.
This phenomenon of “postdiuretic Na+ retention” can be overcome by
Restricting dietary Na+ intake or
By more frequent administration of the loop diuretic
Thus, option C is correct. Ref: GG, The Pharmacological Basis of Therapeutics, Topic :
Diuretics

557. C. Pindolol
Drugs possessing ISA are useful in this situation.
Thus, option C is correct. Ref: KDT, Topic: antihypertensives

558. D. Ranolazine
Ranolazine:
Acts by inhibiting late Na Channels myocardium
Through Na+/Ca2+ exchanger, Reduction in Ca2+ overload in the myocardium
during ischemia decreases contractility and has a cardio protective effect
Inhibits fatty acid oxidation and helps in utilization of glucose as fuel in Ischaemic
myocardium, thereby decreasing oxygen consumption
Thus, option D is correct. Ref: KDT, Topic: Angina

559. C. Trimetazidine
Trimetazidine:
May increase the risk of movement disorders such a Parkinson disease,
particularly in older patients with decreased kidney function
This serious effect has led to use restrictions by the EMA and the recommendation to
use trimetazidine only as second-line treatment of stable angina in patients
inadequately controlled by or intolerant to first-line antianginal therapies.
Thus, option C is correct. Ref: GG, Topic: Angina

560. B. Nicorandil
Nicorandil:
Nicorandil is a nitrate ester of nicotinamide developed as an antianginal agent
Nicorandil has nitrate-like (cGMO-dependent) properties and acts as an agonist at
ATPsensitive potassium channels.
Thus, option B is correct. Ref: GG, Topic: Angina

561. C. It bypasses first pass metabolism when given by sublingual route


Glyceryl nitrate:
Sublingual route is used when terminating an attack or aborting an imminent on is
the m=aim
Tablet mat be crushed under the teeth and spread over buccal mucosa
It acts within 1-2 min (peak blood level in 3-6 min) because of direct absorption into
systemic circulation (bypassing liver where almost 90% is metabolized).
Thus, option C is correct. Ref: KDT, Topic: Angina

562. D. Both A & B


Drug interaction of Phosphodiesterase 5 inhibitors about nitrates:
Combination of Phosphodiesterase 5 inhibitors like Sildenafil, Vardenafil, Tadalafil
with organic nitrate vasodilators can cause extreme hypotension
In the presence of a PDE 5 inhibitor, nitrates cause profound increases in cyclic GMP
and can produce dramatic reductions in blood pressure
For this reason, there is a warning that PDE 5 inhibitors should not be prescribed to
patients receiving any form of nitrate and dictates that patients should be questioned
about the use of PDE5 inhibitors within 24 hours before nitrates are administered
A period of longer than 24 hours may be needed following administration of a PDE5
inhibitor for safe use of nitrates, especially with tadalafil because of its prolonged t1/2
Treatment of hypotension due to combined administration of Sildenafil; and a nitrate
IV fluids and a-adrenergic receptor agonists are used
Thus, option D is correct. Ref: GG, Topic: Angina

563. A. Left ventricular infraction is suspected


In case of MI, GTN should not be administered if:
Systolic BP is < 90 mm Hg
Heart rate is < 50 or < 100 beats/min
Left ventricular infraction is suspected
Patient has taken sildenafil in the past 24 hours
Thus, option A is correct. Ref: KDT, Topic: AMI

564. A. Phosphodiesterase inhibitor


Molsidomine:
Direct NO donor
Second choice for the prevention of angina
Adverse effects: headache, dizziness, postural, hypotension, syncope
No documented advantage over GTN/ISDN/ISMN
Thus, option A is correct. Ref: GG; angina

565. D. Lignocaine
Digitalis induced ventricular arrhythmias:
Lidocaine or phenytoin, which have minimal effects on AV conduction, may be used
for the treatment of ventricular arrhythmias that threaten hemodynamic compromise
Electrical cardioversion carries an increased risk of inducing severe rhythm
disturbances in patients with particular caution
Inhabitation of the Na+, K+ -ATPase activity of skeletal muscle can cause
hyperkalemia
An effective antidote for life-threatening diagnose (ordogotoxin) toxicity is DIGIBIND
Thus, option D is correct. Ref: GG, Topic Arrythmia

566. A. Left ventricular infraction is suspected


In case of MI, GTN should not be administered if:
Systolic BP is < 90 mm Hg
Heart rate is < 50 or < 100 beats/min
Left ventricular infraction is suspected
Patient has taken sildenafil in the past 24 hours
Thus, option A is correct. Ref: KDT, Topic: AMI

567. B. Nicorandil
Ischaemic preconditioning:
Is a phenomenon in which brief periods of ischemia and reperfusion exert a cardio
protective effect on subsequent total vascular occlusion, and involves opening of
mitochondrial ATP sensitive K+ channels
Nicorandil is believed to exert cardio protective action by simulating ‘Ischaemic
preconditioning’ as a result of activation of mitochondrial KATP channels
Thus, option B is correct. Ref: KDT, Topic: Angina

568. D. ATP sensitive K+ channel


Nicorandil:
Drug activates ATP sensitive K+ channels (KATP)
Hyperpolarizing vascular smooth muscle
Acts as a NO donor and acts by increasing cGMP and relaxes blood vessels
By activation of mitochondrial KATP channels it has cardio protective action by
simulating ‘ischemic preconditioning’
Thus, option D is correct. Ref: KDT, Topic: Angina

569. C. Spironolactone
Liddle syndrome (Pseudoaldosteronism):
An autosomal dominant disorder
Mutations in the B or Y subunits, leading to increased basal amiloride-sensitive
sodium channel activity
Due to hyperactivity of the amiloride-sensitive sodium channel of the principal cell of
the cortical collecting tubule
It includes early-onset severe hypertension, hypokalemia, metabolic alkalosis
& low renin and aldosterone
Treatment of Liddle syndrome with amiloride or triamterene lowers blood
pressure and corrects the hypokalemia and acidosis
Spironolactone is not an effective treatment because the increased activity of the
amiloride-sensitive Na+ channel is not mediated by aldosterone.
Thus, option C is correct. Ref: GG; Topic : Diuretics

570. B. It decreases serum theophylline levels when used together


Pentoxyphylline
Methylxanthine derivative that is called a rheologic modifier
Analog of theobromine & inhibits PDE
Reduces blood viscosity and improves microcirculation by improving flexibility of
RBCs
Has rheological property
Does not cause coronary steal phenomenon
Inhibits production of TNF alpha
USES:
To reduce the peripheral claudication
Non-hemorrhagic stroke
Chronic cerebrovascular insufficiency
Trophic leg ulcers
Improve sperm motality
Diabetic neuropathy
Increases serum theophylline levels when used together
Thus, option B is correct. Topic : Phosphodiesterase inhibitors

571. C. Ivabradine
Ivabradine:
Acts by selectively inhibiting the current in the SA node, known as the ‘funny current’
(If)
The (If) is an inward Na+ /K+ current activated by hyperpolarization and ischemia
Inhabitation of these current by ivabradine decreases myocardial oxygen demand by
reducing HR
No negative ionotropic effect and reduction in BP
(If) in retina
USE:
In chronic stable angina in patients with sinus rhythm who are intolerant to B
blockers or when the latter are contraindicated
Can also be used in inappropriate sinus tachycardia
Thus, option C is correct. Topic: Angina

572. C. Verapamil
Verapamil and diltiazem should be avoided with beta blockers because both cause
myocardial depression leading to marked depression of myocardium
Also addictive sinus depression, conduction defects or asystole may occur.
Thus, option C is correct. Ref: KDT, Topic: Arrythmia

573. B. Mexiletine
Mexiletine:
Is an analogue of lidocaine
Has been modified to reduce first-pass hepatic metabolism and permit chronic oral
therapy
Mechanism of action similar to Lignocaine
It is approved for treating ventricular arrhythmias
Lidocaine:
It is relatively selective for partially depolarized cell and those with longer APD
(whose Na+ Channels remain inactivated for longer period)
While normal ventricular and conducting fibres are minimally affected,
depolarized/damaged fibres are significantly depressed.
Brevity of atrial AP and lack of lidocaine effect on channel recovery might explain its
inefficacy in atrial arrhythmias
Thus, option B is correct. Ref: KDT, Topic: Arrhythmia

574. C. Lignocaine
Ventricular arrhythmias due to digitalis toxicity
Lidocaine i.v. repeated as required is the drug of choice
It suppresses the excessive automaticity, but does not accentuate A-V block
Quinidine, Procainamide and Propafenone are contraindicated
Lignocaine is DOC but 2nd alternative option is Phenytoin for digitalis induced
ventricular arrhythmias
Thus, option C is correct. Ref: KDT, Topic: Arrhythmia

575. D. Verapamil
Adenosine is DOC for acute termination of PSVT
Esmolol and Verapamil are alternatives
But in this case, Adenosine may precipitate bronchospasm in asthma patient also
avoid beta blocker in asthmatics
Thus, option D is correct. Ref: KDT, Topic: Arrhythmia

576. B. Amiodarone
Arrhythmias Refractory to the treatment of Lignocaine can be treated by
Amiodarone because it is a drug with broad spectrum of action.
Thus, option B is correct. Ref: KDT, Topic: Arrhythmia

577. A. 0.5 to 2 ug/ m L


A factual question. The proposed therapeutic plasma amiodarone concentration range
is 0.5 to 2 ug/ m L.
Thus, option A is correct. Ref: GG, Topic: Arrhythmia

578. D. Diarrhea
Adverse effect of Quinidine
Diarrhea is the most common adverse effect during quinidine therapy, occurring in
30%-50% of patients
Immunological reactions can occur during Quinidine therapy
The most common is thrombocytopenia which can be severe but which resolves
rapidly with discontinuation of the drug.
Hepatitis, bone marrow depression and lupus syndrome
Thus, option D is correct. Ref: GG 13E/P 569; Topic: Arrhythmia

579. D. It is more common in rapid acetylators than slow acetylators


Procainamide induced lupus syndrome:
In individuals who are “slow acetylators,” the procainamide-induced lupus syndrome
develops more often and earlier during treatment than among rapid acetylators.
It is more common in slow acetylators than rapid acetylators
The symptoms of procainamide-induced lupus resolve during treatment with N-acetyl
Procainamide
Therapy should not be stopped because of the presence of antinuclear antibodies
only without symptoms.
The lupus-like symptoms resolve on cessation of therapy
Thus, option D is correct. Ref: GG; Topic: Arrhythmia

580. C. Quinidine
Quinindine belongs to Class IA antiarrhythmics
These drugs have their effect on potassium channels and they reduce potassium
repolarizing current by blocking potassium channels.
They cause QT prolongation (Torsades ‘de pointes)
These agents also prolong action potential duration
Class III antiarrhythmic drugs also block potassium channels
They can cause QT prolongation
Amiodarone. Sotalol
Thus, option C is correct. Ref: GG; Topic: Arrhythmia

581. C. Sotalol
Anti-arrhythmic drugs as per Vaughan-Williams classification:
II. Antiadrenergic agents (b blocker)
Propranolol, Esmolol, Sotalol (also class III)
III. Agents widening AP
Amiodarone, Dronedarone, Sotalol (prolong repolarization and ERP) Dofetilide,
Ibutilide
IV. Calcium channel blockers
Verapamil, Diltiazem
Sotalol:
Sotalol is a nonselective B adrenergic receptor antagonist that also prolongs
cardiac action potentials by inhibiting delayed rectifier and possibly other K+
currents
Thus, option C is correct. Ref: KDT, Topic: Arrhythmia

582. D. Propafenone
Anti-arrhythmic drugs as per Vaughan-Williams classification
CLASS I (Membrane stabilizing agents)
(Na+ channel blockers)
1A. Moderately decrease dv/dt of 0 phase
Quinidine, Procainamide, Disopyramide
IB. Little decrease in dv/dt of 0 phase
Lidocaine, Mexiletine
1C. Marked decrease in dv/dt of 0 phase
Propafenone, Flecainide
Thus, option D is correct. Ref: KDT, Topic: Arrhythmia

583. B. Recombinant ANP


Natriuretic Peptides:
Recombinant ANP (carperitide)
Recombinant BNP (nesiritide)
Synthetic form of urodilatin (ularitide)
Natriuretic peptides produce vasodilation and natriuretic and have been investigated
for the treatment of congestive heart failure.
Thus, option B is correct. Ref: KAT, Topic CHF

584. D. Nadolol
Nadolol is not approved for use in Chronic CHF
Thus, option D is correct. Ref: Essentials of Medical Pharmacology, KDT, Topic:

585. B. Longer acting because of its accumulation in adipose tissues


Carvedilol:
Alpha + beta blocker
Nonselective B blocker with a1-antagonist activity
Ratio of a1-to B-antagonist potency for carvedilol is approximately 1:10
Duration of action is longer
Because drug dissociates slowly from its receptor
So duration of action is longer than the short (t1/2(2.2 h)
Not due to accumulation in adipose tissue
has antioxidant property
USE:
Hypertension and symptomatic heart failure
Carvedilol reduces mortality in patients with congestive heart failure
It has vasodilating effect
So, B-blocker of choice in patients with peripheral artery disease
Thus, option B is correct. Ref: GG, Topic CHF

586. A. It is immunogenic
Digoxin antibody (DIGIBIND):
It is nonimmunogenic because it lacks the fragment
Given by IV infusion
Digoxin-Digibind complex is rapidly excreted by kidney
Thus, option A is correct. Ref: KDT, Topic: Topic CHF

587. C. Hyperkalemia
Digoxin toxicity is increased by
Hypokalemia
Hypercalcemia
Hypomagnesemia
Others:
Renal Impairment drugs
Quinidine
Amiodarone
Verapamil
Thiazide diuretics
Thus, option C is correct. Ref: KDT, Topic: CHF

588. C. 24 hrs
The antiarrhythmic effects of digoxin can be achieved with intravenous or oral therapy
Digoxin undergoes relatively slow distribution to effector site(s); therefore, even with
intravenous therapy, there is a lag od several hours between drug administration and
the development of measurable antiarrhythmic effects such as PR interval
prolongation or slowing of the ventricular rate in atrial fibrillation
To avoid intoxication, a loading dose of approximately 0.6 to 1 mg digoxin is
administered over 24 hours.
Thus, option C is correct. Ref: GG, Topic CHF

589. D. Increase in Refractoriness of AV nodal tissue


Mechanism of action of digitalis in atrial fibrillation is:
Increase in Refractoriness of AV nodal tissue
So that the impulses arising from atrial are not conducted to ventricles and prevents
ventricle being continuously stimulated
It is due to vagomimmetic action of digitalis
Thus, option D is correct. Ref: KDT, Topic: CHF
590. D. PDE 3
Amrinone
Is a PDE 3 inhibitor
It is an ionotropic and vasodilator so called as ionodilator
USE
Severe and Refractory CHF
ADR
Thrombocytopenia
Thus, option D is correct. Ref: KDT, Topic: CHF

591. B. Less peripheral Vasoconstriction


Dobutamine acts only on beta 1 receptors and increases force of cardiac contraction
without causing peripheral vasoconstriction
But dopamine has varied action as below
Dopamine infusion rate and its different action:
1-2 ug/kg/min D1 action renal vasodilatation
2-5 ug/kg/min beta action increased heart rate
5-6 ug/kg/min alpha action vasoconstriction
So dopamine increases force of cardiac contraction and causes peripheral
vasoconstriction at higher doses
Thus, option B is correct. Ref: GG; Topic: CHF

592. C. Tremors
Methyldopa ADVERSE EFFECTS:
Produces sedation
Diminution in psychic energy
Dryness of the Mouth
Parkinsonian signs
Hyperprolactinemia: causing gynecomastia and galactorrhoea
Thus, option C is correct. Ref: GG; Topic: CHF

593. A. ARB
Population aged 18 years or older with CKD and hypertension
Initial (or add-on) antihypertensive treatment should include an ACEI or ARB to
improve kidney outcomes
This applies to all CKD patients with hypertension regardless of race or diabetes
status.
Thus, option A is correct. Ref: 2014 Evidence-Based Guideline for the Management of
High Blood Pressure in Adults: Report from the Panel Members Appointed to the Eighth
Joint National Committee (JNC 8)

594. B. Calcium channel blocker


In general, black population, including those with diabetes:
Initial antihypertensive treatment should include
Thiazide-type diuretic
CCB
Thus, option B is correct. Ref: 2014 Evidence-Based Guideline for the Management of
High Blood Pressure in Adults: Report from the Panel Members Appointed to the Eighth
Joint National Committee (JNC 8)

595. C. Amlodipine
Thiazide cause Hyponatremia leading to lithium toxicity
Losartan cause hyperkalemia
Doxazosin is not the first line drug for anti-hypertensive therapy.
Thus, option C is correct. Ref: Amlodipine

596. A. Captopril
E. The drug of choice for management scleroderma hypertensive crisis is Captopril.
Thus, option A is correct. Ref: CMDT

597. D. Clevidipine
Clevidipine
Intravenous Clevidipine is an L-type calcium channel blocker with a 1-minute half-life.
It facilitates swift and tight control of severe hypertension
It acts on arterial resistance vessels and is devoid of venodilatory or cardio depressant
effects even intravenous nicardipine is the most potent and the longest acting of the
parenteral calcium channel blockers used in hypertensive emergency.
CLEVIDIPINE and NICARDIPINE are two calcium channel blockers used for
hypertensive emergencies
Thus, option D is correct. Ref: CMDT; Topic Hypertension

598. C. Aliskiren
MOA:
Potent competitive inhibitor of renin
Binds the active site of renin and blocks conversion of angiotensinogen to Angl
Pk:
Bioavailability of aliskiren is low but still used orally as it has high affinity and
potency
T1/2 is 20-45 hours
It is a substrate for P-glycoprotein (Pgp), so bioavailability is less
Fatty meals significantly decrease the absorption of Aliskiren
Less hepatic metabolism, so fecal elimination of unchanged drug is common
One fourth of absorbed drug gets excreted in kidney unchanged
Thus, option C is correct. Ref: GG, Topic : Hypertension

599. D. Hypertension arising from cocaine use


Beta-blockers should not be used to treat hypertension arising from cocaine use:
Hypertension arising from cocaine use:
Cocaine increases sympathetic outflow
Increase BP
Beta-blockers should not be administered until alpha-blockade has been established
Otherwise, blockade of vasodilatory beta-2 adrenergic receptors will allow unopposed
vasoconstrictor alpha-adrenergic receptor activation with worsening of hypertension.
Thus, option D is correct. Ref: CMDT, Topic : Hypertension

600. B. Methyldopa
Methyldopa:
At least 20% of patients who receive methyldopa for a year develop a positive
Coombs test is not necessarily an indication to stop treatment with methyldopa
But 1%-5% of these patients will develop a hemolytic anemia that requires prompt
discontinuation of the drug.
Treating hypertension in eclampsia
Thus, option B is correct. Ref: GG, Topic: Hypertension

601. A. More common in Caucasians than in African Americans


Hydralazine induced lupus syndrome:
Drug-induced lupus syndrome usually occurs after at least 6 months of continuous
treatment with Hydralazine
Its incidence is related to dose, gender, acetylator phenotype and race.
The syndrome is seen more commonly in Caucasians than in African Americans
The rate of conversion to a positive antinuclear antibody test is faster in slow
acetylators than in rapid acetylators.
Thus, option A is correct. Ref: GG, Topic : Hypertension

602. C. Both are correct


Can worsen arterial hypoxemia in patients with chronic obstructive pulmonary
disease because the drug interferes with hypoxic pulmonary vasoconstriction and
therefore promotes mismatching of ventilation with perfusion
Thus, option C is correct. Ref: GG, Topic : Hypertension

603. C. Both are correct


In the treatment of acute aortic dissection:
It is important to administer b-blocker with nitroprusside because reduction of blood
pressure with nitroprusside alone can increase the rate of rise in pressure in the aorta
as a result of increase myocardial contractility, thereby enhancing propagation of the
dissection.
Thus, option C is correct. Ref: GG 13E/P 520 Topic: Hypertension

604. B. Pregnancy induced hypertension


Thiazide diuretics are suitable antihypertensive for following conditions
Older patients
Isolated systolic hypertension
Obese with volume overload
Low cost therapy
Thiazide diuretics to be avoided in the following conditions
Pregnancy induced hypertension
Gout
Abnormal lipid profile
Thus, option B is correct. Ref: KDT, Topic: Hypertension

605. D. Reynaud’s disease


Beta blockers are suitable antihypertensive for following conditions
Coexisting anxiety or Tachycardia
Migraine patients
Low cost therapy
Beta blockers to be avoided in the following conditions
Asthma, COPD
Bradycardia,
PVD like Reynaud’s disease
Condition defects
Decompensated heart-failure
Relatively young patients
Abnormal lipid profile
Thus, option D is correct. Ref: KDT, Topic: Hypertension

606. B. Greater dilatation of precapillary Vessels than post capillary vessels


Ankle edema due to
Because of greater dilatation of precapillary than post capillary vessels at ankle
It is not due to fluid retention
Thus, option B is correct. Ref: KDT, TopicHypertension

607. C. Ambrisentan
Bosentan:
Is a nonselective ET receptor blocker
Macitentan:
Cdual endothelin receptor antagonist (ET A and ET B) Additional ET receptor
Ambrisentan
ET A antagonists with some ET A selectively sitaxsentan
Most selective ET A antagonist
Thus, option C is correct. Ref: KAT, Topic : Pulmonary Hypertension

608. C. Clevidipine
Clevidipine
Intravenous clevidipine is an L-type calcium channel blocker with a 1-minuyte half-
life, which facilities swift and tight control of severe hypertension
It acts arterial resistance vessels and is devoid of cenodilartoruy or card depressant
effects formulated as lipid emilson for intravenous infusion so contraindicated
in patients with allergy to soy or egg.
Thus, option C is correct. Ref: CMDT, Topic Hypertension

609. D. Labetalol
Recommended drug for hypertensive emergency with intracerebral hemorrhage and
systolic blood pressure> 140-160 mm Hg
Nicardipine
Clevidipine
Labetalol
Drugs to avoid during hypertensive emergency with Intracerebral hemorrhage and
systolic blood pressure > 140-160 mm Hg
Nitroprusside, Methyldopa, Clonidine, Nitroglycerin
Thus, option D is correct. Ref: CMDT, Topic Hypertension

610. A. Brandykinin
Cough and angioedema in patient receiving ACE inhibitors is due to Bradykinin.
Bradykinin is broken down by enzyme ACE. So inhabitation of ACE leads levels of
Brandyklenin leading to Cough and angioedema.
Thus, option A is correct. Ref: CMDT, Topic Hypertension

611. D. Lisinopril
Captopril and Lisinopril are active rest all other ACE inhibitor are prodrugs
Fosinopril ……. Fosinoprilat
Enalapril……. Enalaprilat
Ramipril……. Ramiprilat
Thus, option D is correct. Ref: CMDT, Topic Hypertension

612. C. Telmesartan
Two ARBs, telmisartan and irbesartan, act as selective PPAR modulators
Due to its partial PPARy agonist effect, telmisartan inhibits vascular ACE activity, AT-R1
expression and increases endothelial NO synthesis preventing oxidative stress and
endothelial dysfunction
One more addition is azilsartan.
Thus, option C is correct.

613. D. All of the above


In long stranding diabetes:
At the level of renal epithelial podocytes, activation of ATI receptors by Ang II leads to
activation of protein kinase signaling cascades,
Cytoskeletal rearrangements
Retraction of podocyte processes
Reduction in proteins of the slit disphragm
All these process results in increased permeability of the renal epithelium to proteins
(proteinuria)
ACE inhibitors reduce these effects of Angil by blocking ACE enzyme, and thereby
reducing Angiotensin II LEVELS.
Thus, option D is correct. Ref: GG, Topic Hypertension

614. D. Losartan
Losartan:
Competitive antagonist of the thromboxane A2 receptor and attenuates platelet
aggregation
A metabolite of losartan COX-2 Mrna upregulation and COX- dependent prostaglandin
generation
An inactive ester prodrug that is completely hydrolyzed to the active form
Candesartan:
An inactive ester prodrug that is completely hydrolyzed to the active form
Telmisartan:
Has partial agonist of peroxisome proliferator-activated receptor gamma
Olmesertan:
Inactive ester prodrug that is completely hydrolyzed to the active form
Clearance is by both renal elimination and biliary excretion
No dose adjustment is required in patients with mild-to-moderate renal or hepatic
impairment
Thus, option D is correct. Ref: Goodman & Gilman’s, The Pharmacological Basis of
Therapeutics, Topic Hypertension
615. A. Olmesertan
Angiotensin II receptor blockers
Bind the ATI receptor with high affinity
More than 10,000-fold selective for the ATI receptor over the AT2 receptor
The rank-order affinity of the ATI receptor for ARBs is candesartan= olmesartan >
irbesartan = eprosartan > telmisartan = valsartan = ERP 3174 (the active metabolite
of losartan) > Losartan
Although binding of ARBs to the ATI receptor is competitive, the inhibitory ARBs of
biological responses to Angil often is insurmountable (the maximal response to Angil
cannot be restored in the presence of the ARB regardless of the concentration of
Angil added to the experimental preparation)
Thus, option A is correct. Ref: GG; Topic Hypertension

616. C. Losartan
Angiotensin Receptor Blocker which lowers uric acid levels in Losartan
Losartan
First ARB to discovered
It has mild probenecid like uricosuric action
So, may be preferred in patient with gout and HTN
Losartan is a competitive antagonist of the thromboxane A2 receptor and attenuates
platelet aggregation
EXP 3179, a metabolic of losartan without angiotensin receptor effects, reduces COX-
2 mRNA upregulation and COX- dependent prostaglandin generation
Thus, option C is correct. Ref: KDT, Topic: Hypertension

617. B. Esterases
Captopril, Lisinopril and Enalaprilat are active molecules
All the following drugs like Benazepril, Enalapril, Fosinopril, moexipril, perindopril,
quinapril, Ramipril and trandolapril are prodrugs
They need to be converted to active form by the enzyme hepatic esterases present in
liver
Easter moiety is removed by esterases and replaced with hydrogen atom to form the
active molecule
Clinical implications
In hypertensive urgencies Enalapril can’t be given because it is a prodrug, but its
active moiety Enalaprilat is used.
So also caution while using in hepatic failure patients
Thus, option B is correct. Ref: GG, Topic Hypertension

618. D. Perindopril
Perindopril
Prodrug metabolized to Perindoprilat
Perindoprilat displays biphasic elkimation kinetics with half-lives of 3-10 hours (the
major component of elimination) and 30-120 hours
(owing to slow dissociation of perindoprilat from tissue ACE)
Enalapril:
Prodrug converted to Enalaprilat
Enalapril has a t1/2-1.3 hours
Enalaprilat has a plasma t1/2 of -11 hours due to tight binding to ACE
Lisinopril:
Not a prodrug t1/2 in plasma is -12 hours and does not accumulate in tissues
Ramipril:
Prodrug converted to Ramiprilat
Peak concentration of Ramiprilat in plasma are achieved in -3 hours
Ramiprilat displays triphasic elimination kinetics with half-lives of 2-4 hours, 9-8
hours and >50 hours
This triphasic elimination is due to extensive distribution of all tissues (initial t1/2).
Clearance of free ramiprilat from plasma (intermediate t1/2), and dissociation of
ramiprilat from tissue ACE (long terminal t1/2)
So ACE inhibitor with Longest half-life is perindopril.
Thus, option D is correct. Ref: GG; Topic Hypertension

619. A. Warfarin
Chronic AF is associated with a 3–7% annual risk of Ischaemic stroke from
thromboembolism. Guidelines 3 recommend administration of heparin prior to, or
concurrently during, immediate electrical or pharmacological cardioversion. If AF has
been present for longer than 48 h or the duration is unknown, warfarin should be
given for 3–4 weeks following successful cardioversion. Patients admitted for elective
cardioversion require adequate anticoagulation with warfarin 3–4 weeks before and
after the procedure (INR 2–3). Those patients who cannot be anticoagulated due to
contraindications prior to cardioversion should undergo transesophageal
echocardiographic examination to exclude the presence of thrombus.
In chronic AF, the risks and benefits of antithrombotic therapy (aspirin, warfarin) must
be considered in each individual patient. Chronic hypertension, age >65 yr, diabetes
mellitus, previous Ischaemic stroke, ventricular dysfunction and co-existent Ischaemic
or valvar heart disease are considered as high risk factors for thromboembolism in AF.
Thus, option A is correct. Ref: GG; Topic Arrythmia

620. A. Decrease the beta-blocker dose.


This patient with a recent myocardial infarction was found on telemetry to have an
episode of 2nd degree atrioventricular block: Mobitz type I (Wenckebach block). This
conduction abnormality is common in patients with an inferior MI and does not
constitute an indication for a pacemaker, particularly in an asymptomatic patient.
Decreasing the beta-blocker dose is a reasonable response. Remember that the
indication for pacing is symptomatic bradycardia.
Thus, option A is correct. Ref: GG; Topic Arrythmia

621. A. Gingival hyperplasia


Arrhythmias are obviously one of the side effects of toxicity. Blurred vision and “halo”
lights are common. GI complaints are very common. Gingival hyperplasia, however, is
something that is not associated with digoxin—you can see it with calcium channel
blockers like Nifedipine and classically with anti-seizures medications. Headaches are
also common.
Ref: GG; Thus, option A is correct. Topic: CHF

622. E. No, presence of prosthetic joints does not require antibiotic


prophylaxis.
There is some controversy about the use of antibiotic prophylaxis for prolonged
surgery or surgery involving infected areas—but some authorities do recommend
prophylaxis. But for routine dental procedures, no prophylaxis is indicated. Remember:
She has anaphylaxis to penicillin—you certainly would not use amoxicillin, and you
would be a little leery of using a cephalosporin too.
Ref: GG; Thus, option E is correct. Topic: infective endocarditis

623. C. Methyldopa
Methyldopa is the only drug listed that is safe in pregnancy. Beta-blockers and
labetalol are commonly used as well. HCTZ may lead to volume depletion in a
pregnant woman. Lisinopril, Fosinopril, and all ACE inhibitors are absolutely
contraindicated because they cause fetal abnormalities and death. Nitroprusside drip
would poison the fetus.
Thus, option C is correct. Ref: GG; Topic: Hypertension

624. D. Hypomagnesemia
Torsades de pointes are a ventricular dysrhythmia with polymorphic QRS complexes. It
can arise in many conditions, including electrolyte abnormalities, drug interactions,
brain injuries, and heart blocks. The most common electrolyte abnormalities
associated with torsades de pointes are hypokalemia and hypomagnesemia.
Correction of these abnormalities, external pacing control, and beta-blockers are
mainstays of therapy. Hyperkalemia, hypocalcemia, and hypernatremia can all affect
heart function, but torsades de pointes are not a common feature of these
disturbances.
Thus, option D is correct. Ref: GG; Topic Arrhythmia

625. B. B vitamin supplements


Homocystinemia can arise in several situations including nutritional deficiencies,
medication side effects, and as a genetically based condition. High levels of serum
homocysteine were thought to have been linked to early-onset cardiovascular disease.
Levels of homocysteine can be significantly improved with the use of B vitamin
supplements, particularly B 12, B 6, and folate. However, recent data has shown that
lowering elevated homocysteine levels does not improve mortality or morbidity in
patients with an acute MI.
Thus, option B is correct. Ref: GG; Topic : AMI

626. A. Propranolol
Based on his clinical findings of crescendo type angina and the quickly reversible ECG
findings of ST-segment elevation, he likely has variant angina (formerly known as
Prinzmetal’s). The use of the nonselective beta-blocker propranolol has been shown to
actually prolong the duration of vasospasm in patients with variant angina. Nifedipine,
sorbate dinitrate, and diltiazem would all be therapeutic and help prevent the spasms.
Lovastatin would not help prevent the spasms but would not be contraindicated.
Aspirin should be used with caution and is likely best avoided because aspirin inhibits
prostacyclin production.
Thus, option A is correct. Ref: GG; Topic Arrhythmia

627. D. Aspirin
Antiplatelet drugs are used for the prophylaxis of arterial thrombotic conditions like
stroke and MI.
Atrial fibrillation increases the risk of thromboembolism and can result in stroke.
Thus, option D is correct. Ref: KDT, Topic: Atherosclerosis

628. C. Aspirin
Antiplatelet drugs like aspirin or clopidogrel are used to prevent arterial thrombosis (in
diseases like MI and stroke).
Thus, option C is correct. Ref: KDT, Topic: AMI

629. D. It prevents ADP mediated platelet adenylyl cyclase inhibition


Abciximab, eptifibatide and tirofiban inhibits GpIlb/llIa receptors.
Ticlopidine and clopidogrel inhibits ADP receptors.
Aspirin inhibits TXA2 synthesis in platelets.
Bleeding time is prolonged by antiplatelet drugs whereas PT is prolonged by oral
anticoagulants and aPTT by heparin.
Thus, option D is correct. Ref: KDT, Topic: AMI

630. C. Plasminogen activator inhibitors


Tissue plasminogen activators are used in acute MI (not the inhibitors)
Thrombolytic agents like streptokinase, Urokinase, Alteplase and reteplase are used
to lyse the thrombus.
Antiplatelet drugs like aspirin are started to prevent the re-infarction.
Thus, option C is correct. Ref: KDT, Topic: AMI

631. D. Is less likely to cause neutropenia


Ticlopidine and clopidogrel are ADP antagonists and are used as antiplatelet drugs.
Antiplatelet action, chances of bleeding and formation of antibodies is similar with
these two agents.
Clopidogrel is better tolerated because it is less likely to cause severe neutropenia and
thrombocytopenia as compared to ticlopidine.
Thus, option D is correct. Ref: KDT, Topic: AMI

632. D. Vitamin B
Diagnosis of the patient is pernicious anemia. Factors favoring this diagnosis are:
Megaloblastic anemia.
Demyelination (decreased tendon Reflexes)
Atrophic gastritis
So, he would require B12 supplementation.
Thus, option D is correct. Ref: KDT, Topic: Miscellaneaous
Hematology — Questions
633. An old woman is required to receive 4 cycles of cancer chemotherapy.
After her first cycle, she developed chemotherapy-induced
thrombocytopenia. Then in the next cycle, it would be appropriate to give
this patient
A. Iron dextran
B. Darbopoietin alpha
C. Oprelvekin
D. Filgrastim (G-CSF)

634. Dr. Nitin decided to give oral iron therapy to a patient of iron deficiency
anemia. Which of the following adverse effects leads to poor compliance of
medicine by the patient?
A. Black stools
B. Staining of Teeth
C. Epigastric pain and bowel upset
D. Metallic taste

635. Which of the following is not used for thromboprophylaxis


A. Warfarin
B. Aspirin
C. Heparin
D. Antithrombin III

636. In which of the following clinical conditions does the use of


anticoagulants provide maximum benefit
A. Cerebrovascular accident
B. Prevention of recurrence of myocardial infarction
C. Retinal artery thrombosis
D. Prevention of venous thrombosis and pulmonary embolism

637. Orally acting direct thrombin inhibitor is


A. Melagatran
B. Bivalirudin
C. Ximelgatran
D. Argatroban

638. Treatment of choice in a patient acute pulmonary embolism with right


ventricular hyperkinesia and a compromised cardiac output but normal
blood pressure is
A. IV Filters
B. Thrombolytic agent
C. Warfarin
D. Low molecular weight heparin
639. If a fibrinolytic drug is used for the treatment of acute myocardial
infarction the adverse effect most likely to occur is
A. Hemorrhagic stroke
B. Acute renal failure
C. Encephalitis secondary to liver dysfunction
D. Development of antiplatelet anti-bodies

640. Which of the following medications would be prescribed most frequently


to patient suffering from chronic atrial fibrillation
A. Bretylium
B. Adenosine
C. Lidocaine
D. Warfarin

641. An old woman Nanda suffered stroke for which she was given Alteplase.
She improved considerably. To prevent the recurrence of stroke, this patient
is most likely to be treated indefinitely with
A. Enoxaparin
B. Aspirin
C. Urokinase
D. Warfarin

642. Hyperlipidemia is caused by


A. A-methyldopa
B. Nifedipine
C. Propranolol
D. ACE inhibitors

643. Blood and serum samples for blood grouping and cross matching will
provide non-reliable results when which of the following colloidal solutions is
given to the patient
A. Albumin
B. Dextran
C. Mannitol
D. Haemaccel

633. c 634. c 635. a 636. d 637. c 638. b 639. a 640. d 641. b

642. c 643. b

644. Statin-induced myopathy is not exacerbated by which of the following


A. Clofibrate
B. Nicotinic Acid
C. Erythromycin
D. Enalapril

645. Which of the following can be used to treat this condition?


A. Fondaparinux
B. Platelet infusion
C. Whole blood infusion
D. LMWH

646. Streptokinase was infused in a patient for the management of deep vein
thrombosis, following which the patient developed hematemesis. Which of
the given agents can be chosen to manage this episode of hematemesis?
A. Aprotinin
B. Noradrenaline
C. Vitamin K
D. Rutin

647. An old woman is required to receive 4 cycle of cancer chemotherapy.


After her first cycle, she developed chemotherapy induced
thrombocytopenia. Then in the next cycle, it would be appropriate to give
this patient
A. Darbopoietin alpha
B. Oprelvekin (IL-11)
C. Iron dextran
D. Filgrastim (G-CSF)

648. Warfarin is administered to a 56-year-old man following placement of a


prosthetic cardiac valve. The warfarin dosage is adjusted to maintain an INR
of 2.5. Subsequently,trimethoprim-sulfamethoxazole therapy is begun for a
recurring urinary tract infection. In addition to monitoring prothrombin time,
which of the following actions should the physician take to maintain
adequate anticoagulation?
A. Begin therapy with vitamin K
B. Increase the dosage of warfarin
C. Make no alterations in the dosage of warfarin
D. Decrease the dosage of warfarin
E. Stop the warfarin and change to low-dose aspirin

649. Which of the following is most likely to be used in a young child with
chronic renal insufficiency?
A. Cyanocobalamin
B. Desferrioxamine
C. Filgrastim (G-CSF)
D. Erythropoietin

650. Hydroxyurea in sickle cell disease acts by


A. decreases nitric oxide
B. increases neutrophil and monocytes
C. inhibits DNA synthesis by acting as a ribonucleotide reductase inhibitor
D. inhibits DNA synthesis by acting as a ribonucleotide synthase inhibitor

651. Hydroxyurea produce which effect on the serum uric acid levels?
A. Hyperuricemia
B. Hypouricemia
C. Variable effect
D. None

652. What are the treatment options for wilson’s disease?


A. Pencillamine, Zinc
B. Riboflavin, Zinc, Trientine
C. Penicillamine, Riboflavin, Zinc, Trientine
D. Potassium disulfide, Penicillamine, Zinc, Trientine

653. Patients that are sensitive to aspirin can take:


A. Sulfinpyrazone
B. Clopidogrel, Ticlopidine
C. Ticlopidine
D. Ticagrelor , Clopidogrel, Ticlopidine

654. This drug can potentiate the effect of prostacyclin to antagonize platelet
stickiness and therefore decreases platelet adhesion to thrombogenic
surfaces.
A. Sulfinpyrazone
B. Dipyridamole
C. ticlopidine
D. None

644. d 645. a 646. a 647. b 648. d 649. d 650. c 651. a 652. d

653. d 654. b
Hematology — Explanations
633. C. Oprelvekin
Oprelvekin is used to stimulate the bone marrow to produce platelets in order to
prevent low platelets that may be caused by chemotherapy. Platelets are blood cells
that allow the blood to clot, and prevent bleeding.
Oprelvekin may be given to decrease the need for platelet transfusions.
Oprelvekin is not indicated for myeloid malignancies (diseases that start in the bone
marrow such as leukemia).
Oprelvekin is a support medication.Oprevelkin is interleukin 11 analogue
Iron dextran, Darbopoetin are used in in anemia due to chronic disease whereas
Figrastim is used in treatment of neutropenia secondary to cancer chemotherapy
Thus, option C is correct. Ref: KDT, Topic Hematopoietic progenitor

634. C. Epigastric pain and bowel upset


Common side effects may include:
constipation, diarrhea;
nausea, vomiting, heartburn;
stomach pain, upset stomach;
black or dark-colored stools or urine;
temporary staining of the teeth;
headache; or.
unusual or unpleasant taste in your mouth.
But out of these the side effects leading to maximum number of defaulters is gastric
upset.
Thus, option C is correct. Ref: KDT Topic: iron deficiency anemia

635. A. Warfarin
The efficacies of thromboprophylaxis of UFH and LWMH were similar in critically ill
patients and both drugs act by stimulating antithrombin III. Aspirin is also used as
thromboprophylaxis agent in AMI. Warfarin has time lag on 5 days in onset of action so
cannot be used in acute thromboprophylaxis.
Thus, option A is correct. Ref: KDT Topic: Anticoagulants
Park J, Lee JM, Lee JS, Cho YJ. Pharmacological and Mechanical Thromboprophylaxis in
Critically Ill Patients: A Network Meta-Analysis of 12 Trials. J Korean Med Sci.
2016;31(11):1828-1837.

636. D. Prevention of venous thrombosis and pulmonary embolism


Anticoagulants are predominantly used for venous clot prophylaxis whereas
antiplatelet drugs are used in arterial cot prophylaxis. Option A, B and C are arterial
clot whereas option D is venous clot. Thus, option D is correct. Ref: KDT, Topic:
Antiplatelet and Anticoagulants - uses

637. C. Ximelgatran
Ximelagatran is an anticoagulant that has been investigated extensively as a
replacement for warfarin that would overcome the problematic dietary, drug
interaction, and monitoring issues associated with warfarin therapy. In ٢٠٠٦, its
manufacturer AstraZeneca announced that it would withdraw pending applications for
marketing approval after reports of hepatotoxicity (liver damage) during trials, and
discontinue its distribution in countries where the drug had been approved.
Melagatran is not a drug name. Bivalirudin and argatroban are parenteral direct
thrombin inhibitors. Thus, option C is correct. Ref: KDT, Topic: Classification of
Anticoagulants

638. B. Thrombolytic agent


Inhaled Nitric oxide, Epoprostenol- Prostaglandin I2 (PGI2), Endothelin Antagonists are
used in the treatment of this condition. Atrial flutter and fibrillation is also an integral
component in this situation which requires warfarin sometimes.
Thus, option B is correct. Ref: Matthews JC, McLaughlin V. Acute right ventricular
failure in the setting of acute pulmonary embolism or chronic pulmonary
hypertension: a detailed review of the pathophysiology, diagnosis, and management.
Curr Cardiol Rev. 2008;4(1):49-59.

639. A. Hemorrhagic stroke


Adverse Effects and Contraindications. Common adverse effects of all the
thrombolytic drugs is bleeding complications related to systemic fibrinogenolysis
and lysis of normal hemostatic plugs. The bleeding is often noted at a
catheterization site, although gastrointestinaland cerebral hemorrhages may
occur.
Thus, option A is correct. Ref: KDT Topic: Fibrinolytic side effects

640. D. Warfarin
Dofetilide, or Flecainide. Or Propafenone, or Amiodarone, or Sotalol are used along
warfarin to prevent embolism in chronic atrial fibrillation patient. The imminent
danger is not rate control but embolism.
Thus, option D is correct. Ref: GG, Topic: Atrial fibrillation

641. B. Aspirin
Alteplase is intravenous drug with short t1/2. For chronic arterial prophylaxis, low dose
aspirin is used. Warfarin is predominantly a venous clot prophylaxis drug.
Thus, option B is correct. Ref: KDT Topic: Anticoagulants and Antiplatelets

642. C. Propranolol
Adverse drug reactions (ADRs) associated with the use of beta blockers include:
nausea, diarrhea, bronchospasm, dyspnea, cold extremities, exacerbation of
Raynaud’s syndrome, bradycardia, hypotension, heart failure, heart block, fatigue,
dizziness, abnormal vision, decreased concentration, hallucinations, insomnia,
nightmares, clinical depression, sexual dysfunction, erectile dysfunction and/or
alteration of glucose andipid metabolism. Mixed α1/β-antagonist therapy is also
commonly associated with orthostatic hypotension. Carvedilol therapy is commonly
associated with edema.
Thus, option C is correct. Ref: KDT, Topic: Nonselective Beta Blockers Side effects and
contra-indications

643. B. Dextran
results with dextran, as far as the shock is concerned, and it doesn›t give us a
reliable blood grouping and cross matching results, and is less preferred.
Mannitol, Haemaccel and albumin do not interfere. Thus, option B is correct. Ref: KDT,
Topic: Plasma binder/ Substitutes

644. D. Enalapril
Statins have some important drug interactions. The first type of interaction involves
the enzymes responsible for the elimination of statins by the liver. Liver enzymes
(specifically, the cytochrome P-٤٥٠ liver enzymes) are responsible for eliminating all
statins from the body with the exception of pravastatin and rosuvastatin. Therefore,
drugs that block the action of these liver enzymes [Nicotinic acid and erythromycin]
increase the levels of simvastatin, lovastatin, fluvastatin, and atorvastatin (but not
pravastatin or rosuvastatin) in the blood and can lead to the development of
rhabdomyolysis. Clofibrate have exactly similar ADR profile as that of statins.
Thus, option D is correct. Ref: KDT, Topic: Hypolipidemics

645. A. Fondaparinux
The HIT Syndrome is treated by Argatroban in concomitant renal failure and by
lepirudin if there is concomitant liver failure. Maintenance treatment can be done with
Fondaparinux.
Thus, option A is correct. Ref KDT, Topic: Anticoagulants

646. A. Aprotinin
Aprotinin, EACA and tranexamic acid are plasminogen inhibitors thus help in clot
formation.
Thus, option A is correct. Topic: Hematemesis

647. B. Oprelvekin (IL-11)


Oprelvekin acts like megakaryocyte colony stimulating factor and can be used to treat
and prevent chemotherapy induced thrombocytopenia.
Filgrastim (G-F) and Sargramostim (GM-CSF) are used to prevent leucopenia.
Thus, option B is correct. Ref: Katzung.

648. D. Decrease the dosage of warfarin


Sulfonamides displace warfarin from its plasma protein binding sites, thus
potentiating its action. This increases the risk of bleeding. Hence the dose of warfarin
must be reduced. Ref KDT, Topic Anticoagualnts - warfarin

649. D. Erythropoietin
Chronic renal failure may result in anemia due to deficient production of
erythropoietin.
Thus, option D is correct. Ref: KDT, Topic: Renal Insufficiency

650. C. Inhibits DNA synthesis by acting as a ribonucleotide reductase


inhibitor
Hydroxyurea in sickle cell disease acts by inhibits DNA synthesis by acting as a
ribonucleotide reductase inhibitor. Ref KDT/8/e Topic Sickle Cell Anemia

651. A. Hyperuricemia
Hyperuricemia is produced by hydroxyurea as a side effect thus contraindicated in
gout patients. Ref KDT, Topic : Sickle Cell Anemia
652. D. Potassium disulfide, Penicillamine, Zinc, Trientine
Potassium disulfide, Penicillamine, Zinc, Trientine are the treatment options for
wilson’s disease. Ref : KDT, Topic : Chelating agents

653. D. Ticagrelor, Clopidogrel, Ticlopidine


Ticagrelor, Clopidogrel, Ticlopidine can be used in aspirin sensitive patient.
Ref Manchette AM, Drucker AG, Januzzi JL Jr. Acute coronary syndrome antiplatelet
alternatives in clopidogrel allergy. Pharmacotherapy. 2014 Aug;34(8):e152-6. doi:
10.1002/phar.1446. Epub 2014 Jun 5. PubMed PMID: 24898185.

654. B. Dipyridamole
PDE-3 inhibitor action of Dipyridamole decreases cAMP in the platelets which
antagonize platelet stickiness and therefore decreases platelet adhesion to
thrombogenic surfaces.
Ref KDT, Dipyridamole : Antiplatelet drugs
Gastrointestinal System — Questions
655. Selective 5-HT4 agonist useful in gastroesophageal reflux disease and
lacking arrhythmogenic property is
A. Tegaserod
B. Buspirone
C. Sumatriptan
D. Cisapride

656. Following are drugs used for IBS-constipation predominant except


A. Alosertron
B. Lubipristone
C. Linaclotide
D. Loxiglumide

657. Prucalopride is a
A. 5HT 3 antagonist
B. M3 Antagonist
C. D2 Antagonist
D. 5HT 4 Agonist

658. The following is a stool wetting agent


A. Lactulose
B. Bisacodyl
C. Bran
D. Docussates

659. Which of the following is incorrect about Racecadotril


A. It is a prodrug
B. Decreases intestinal hypersecretion, without affecting motility
C. Lowers mucosal cAMP
D. Prevents degradation of endogenous enkephalins which are mainly mu opioid
receptor agonists

660. Long term of aluminum hydroxide can cause


A. Vitamin B 12 deficiency
B. Hyperuricaemia
C. Hyperglycemia
D. Osteomalacia

661. Chronic treatment with omeprazole causes following ADRs except


A. Spontaneous bacterial peritonitis in patients with ascites
B. Increased risk of bone fracture
C. Vitamin B 12 Deficiency
D. Hypergastrinemia
662. FDA-approved dose of intravenous pantoprazole for gastroesophageal
reflux disease daily is
A. 80 mg
B. 160 mg
C. 120 mg
D. 40 mg

663. Which of the following is inhibited by Lansoprazole


A. H+ Na+ ATPase pump
B. Na+ K+ ATPase pump
C. H+K+ ATPase pump
D. All of the above

664. Following are differences between Ranitidine and Cimetidine except


A. Ranitidine and cimetidine have wide pharmacokinetic variability
B. Ranitidine is about 5 times more potent than cimetidine
C. Ranitidine has no antiandrogenic action
D. Ranitidine has lesser drug interactions compared to cimetidine

665. Which of the following H2 blocker has no interaction with hepatic CYPs
A. Nizatidine
B. Cimetidine
C. Ranitidine
D. All of the above

666. Prokinetic drug which is mainly metabolized by Flavin monooxygenases is


A. Cisapride
B. Domperidone
C. Itopride
D. Metoclopropamide

667. True statement about Prokinetic action of domperidone and


metacloprapimde is
A. Metacloprapimde prokinetic action is not attenuated by atropine
B. Domperidone prokinetic action is attenuated by atropine
C. Domperidone prokinetic action is not attenuated by atropine
D. None of the above
655. a 656. a 657. d 658. d 659. d 660. d 661. d 662. d 663. c

664. a 665. a 666. c 667. c

668. The prokinetic activity of metoclopramide is mainly because of


A. Central D2 antagonism
B. Peripheral D2 antagonism
C. 5-HT4 agonism
D. Both B and C
669. Apart from diarrhea, oral rehydration solution has been employed in
A. Severe vomiting
B. Bum cases 571.
C. Heat stroke
D. Both (b) and (c)

670. The new formula WHO-ORS differs from the older standard formula WHO-
ORS in the following respect
A. It has no basic salt
B. It has higher K+ concentration
C. It has lower Na+ and glucose concentration
D. Both (b) and (c) are correct

671. The concentration of sodium ions in the standard WHO oral rehydration
solution is
A. 90 m moles/L
B. 60 m moles/L
C. 40 m moles/L
D. 110 m moles/L

672. The success of oral rehydration therapy of diarrhea depends upon, which
of the following process in the intestinal mucosa
A. Sodium pump mediated Na+ absorption
B. Passive Na+ diffusion secondary to nutrient absorption
C. Bicarbonate coupled Na+ absorption
D. Glucose coupled Na+ absorption

673. Stimulant purgatives are contraindicated in the following


A. Subacute intestinal obstruction
B. Before abdominal radiography
C. Bed ridden patients
D. All of these

674. Which of the following laxatives lowers blood ammonia level in hepatic
encephalopathy?
A. Bisacodyl
B. Lactulose
C. Liquid paraffin
D. Magnesium sulfate

675. A patient on cisplatin therapy develops intractable vomiting on the third


day of treatment. Agent of choice for controlling this vomiting is
A. Metoclopramide
B. Ondansetron
C. Aprepitant
D. Prochlorperazine
676. Indicate the drug which does not improve lower esophageal sphincter
tone or prevent gastroesophageal reflux, but is used as the first line
treatment of gastroesophageal reflux disease
A. Sodium alginate + aluminum hydroxide gel
B. Mosapride
C. Omeprazole
D. Metoclopramide

677. Which of the following Prokinetic drugs has been implicated in causing
serious ventricular arrhythmias, particularly in patients concurrently
receiving erythromycin or ketoconazole?
A. Domperidone
B. Metoclopramides
C. Mosapride
D. Cisapride

678. Activation of the following type of receptors present on myenteric


neurons by metoclopramide is primarily responsible for the enhanced
acetylcholine release and improving gastric motility
A. Serotonergic 5-HT4
B. Serotonergic 5-HT3
C. Muscarinic M
D. Dopaminergic D2

679. The most effective antiemetic for controlling cisplatin induced vomiting
is
A. Prochlorperazine
B. Aprepitant
C. Metoclopramide
D. Ondansetron

668. d 669. d 670. c 671. a 672. d 673. a 674. b 675. c 676. c

677. d 678. a 679. d

680. Which antipeptic drug selectively blocks levodopa induced vomiting


without blocking its anti-Parkinsonian action?
A. Domperidone
B. Cisapride
C. Metoclopramide
D. Ondansetron

681. Which of the following Prokinetic drugs produces extrapyramidal side


effects?
A. Domperidone
B. Cisapride
C. Metoclopramide
D. All of the above
682. Metoclopramide has the following actions EXCEPT
A. Increase lower esophageal sphincter tone
B. Increase large intestinal peristalsis
C. Increase gastric peristalsis
D. Prokinetic action is blocked by atropine

683. In case of hill journey, antimotion sickness drugs are best administered
at(a) Twelve hours before commencing journey
A. Twelve hours before commencing journey
B. At the first feeling of motion sickness
C. Immediately after commencing journey
D. One hour before commencing journey

684. Antacid combinations of magnesium and aluminum salts are superior to


single component preparations because
A. They have rapid as well as sustained acid neutralizing action
B. They are less likely to affect gastric emptying
C. They are less likely to alter bowel movement
D. All of the above

685. Choose the antiulcer drug that inhibits gastric acid secretion, stimulates
gastric mucus anti bicarbortate secretion and has cytoprotective action on
gastric mucosa
A. Carbenoxolone sodium
B. Sucralfate
C. Misoprostol
D. Colloidal bismuth subcitrate

686. Drug of choice for the treatment of peptic ulcer caused due to chronic
use of NSAIDs is
A. Esomeprazole
B. Loxatidine
C. Misoprostol
D. Pirenzepine

687. The following is true of anti-H. pylori therapy EXCEPT


A. Colloidal bismuth directly inhibits H. pylori but has poor patient acceptability
B. Resistance to any single antimicrobial drug develops rapidly
C. Concurrent suppression of gastric acid enhances the efficacy of the regimen
D. It is indicated in all patients of peptic ulcer

688. Choose the CORRECT statement about colloidal bismuth subcitrate


A. It causes prolonged naturalization of gastric acid
B. It relieves peptic ulcer pain promptly
C. It has anti H. pylori activity
D. All of the above are correct
689. The following anti-ulcer drug DOES NOT act by reducing the secretion of
or neutralizing gastric acid
A. Sucralfate
B. Megaldrate
C. Misoprostol
D. Omeprazole

690. Ranitidine differs from cimetidine in the following respect,


A. It is less potent
B. It is shorter acting
C. It produces more CNS side effects
D. It does not have anti-androgenic action

680. a 681. c 682. b 683. d 684. d 685. c 686. a 687. d 688. c

689. a 690. d

691. Choose the CORRECT statement about H, receptor blockers


A. They prevent stress ulcers in the stomach
B. They cause faster healing of duodenal ulcers
C. They are the most efficacious drugs in inhibiting gastric acid secretion
D. They afford the most prompt relief of ulcer pain

692. A patient Sonia is diagnosed to be having iron deficiency anemia. The


agent that can be used to improve the absorption of iron is
A. Antacids
B. Ascorbic acid
C. Phosphates
D. Tetracyclines

693. A 40-year-old man has megaloblastic anemia and early signs of


neurological abnormality. The drug most probably required is
A. Vitamin B12
B. Iron sulphate
C. Erythropoietin
D. Folic acid

694. Which of the following is an indication for the use of folinic acid?
A. Prophylaxis of neural tube defects in the offspring of women receiving
anticonvulsant medications
B. Pernicious anemia
C. Counteracting toxicity of high dose methotrexate therapy
D. Anemia associated with renal failure

695. Which of the following metabolic reactions require vitamin B. but not
folate?
A. Conversion of serine to glycine
B. Conversion of homocysteine to methionine
C. Conversion of malonic acid to succinic acid
D. Thymidylate synthesis

696. The difference between iron sorbitol-citric acid and iron dextran is that
the former
A. Is not excreted in urine
B. Is not bound to transferrin in plasma
C. Cannot be injected i.v.
D. Produces fewer side effects

697. A 46-year-old male presents to OPD with diarrhea and abdominal pain.
On investigations, it was found to be non-infective and you proceed with
diphenoxylate therapy in this patient. Which of the following is the primary
target for the drug you prescribed to this patient?
A. Secretion
B. Motility
C. Inflammation
D. Digestion

698. Aryan, a 14-year-old boy presented with chronic diarrhea and weight
loss. History reveals that he has repeated attacks of respiratory tract
infections with pseudomonas aeruginosa. His younger brother died from a
severe respiratory infection at the age of 7. Which of the following agents is
most likely to improve this patient’s condition?
A. Octreotide
B. Loperamide
C. Metronidazole
D. Pancreatic lipase

699. Choose the correct statement about the use of opioid anti-motility drugs
in the management of diarrhea
A. They are used to control diarrhea irrespective of its etiology
B. They are used as adjuvant to antimicrobial therapy of diarrhea
C. They are the drug of choice in irritable bowel syndrome diarrhea
D. They should be used only as a short term measure after ensuring that enter
invasive organisms are not involved

700. The preferred drug for controlling an acute exacerbation of ulcerative


Colitis is
A. Mesalazine
B. Sulfasalazine
C. Prednisolone
D. Vancomycin

691. a 692. b 693. a 694. c 695. c 696. c 697. b 698. d 699. d

700. c
Gastrointestinal System — Explanations
655. A. Tegaserod
Both Cisapride and Tegaserod are selective 5HT4, agonists useful in the treatment of
GERD. Cisapride possesses cardiac K+ channel blocking activity and can lead to
torsades de pointes. Tegaserod is devoid of this adverse effect. However, tegaserod has
recently been withdrawn due to increased risk of MI and stroke.
Thus, A is correct. Ref: KDT, Topic: prokinetics

656. A. Alosertron
Lubipristone:
A bicyclic fatty acid metabolite analogue of prostaglandin E1 activates type-2-
chloride channels
Linaclotide:
Guanylate cyclase agonist
Loxiglumide:
Is a CCK1 receptor antagonist
Used in constipation dominant IBS.
Alosertron:
5HT3 Antagonist
Diarrhea-predominant irritable bowel syndrome in women only
Thus, A is correct. Ref: HARRISON 19E/P 1970 Topic: (IBS)

657. D. 5HT 4 Agonist


Prucalopride:
Is a specific 5HT4 receptor agonist
Facilitates cholinergic neurotransmission
Acts throughout the length of the intestine, increasing oral-cecal transit and colonic
transit without affecting gastric emptying
USE:
Approved or use in women with chronic constipation in whom laxatives fail to
provide adequate relief
Thus, D is correct. Ref: GG; Topic : prokinetics

658. D. Docussates
Docusate salts
Are anionic surfactants/stool wetting agent
Softens the stools by net water accumulation in the lumen
Lower the surface tension of the stool to allow mixing of aqueous and fatty
substances, softening the stool and permitting easier defecation
These agents also stimulate intestinal fluid and electrolyte secretion (possibly by
increasing mucosal cyclic AMP) and alter intestinal mucosal permeability
Docusate sodium and docusate calcium are available in several dosage forms:
Lactulose
Is an osmotic purgative
Bisacodyl
Is a stimulant laxative
Bran
Is a forming laxative
Thus, D is correct. Ref: GG; Topic : Prokinetics

659. D. Prevents degradation of endogenous enkephalins which are mainly mu


opioid receptor agonists
Racecadotril:
Is a prodrug rapidly converted to thiorphan
Thiorphan is an enkephalinase inhibitor
Enkephalinase inhabitation prevents degradation of endogenous enkephalins which
are mainly mu opioid receptor agonists
It decreases intestinal hypersecretion, without affecting motility by lowering mucosal
cAMP due to enhanced ENK action
GI motility appears to be regulated through it receptors
USE:
Short term treatment of acute secretory diarrheas
Non contraindicated in children
Thus, D is correct. Ref: KDT, Topic: diarrhea treatment

660. D. Osteomalacia
Aluminum hydroxide:
Binds phosphate in the intestine and prevents its absorption
Hypophosphatemia occurs on regular use this leads to oseomalacia
This adverse effect (hypophosphatemia) can be taken as an and used therapeutically
in hyperphosphatemia and phosphate stones.
Small amount of aluminum that is absorbed is excreted by kidney. This is impaired in
renal failure leading to aluminum toxicity, which may present with encephalopathy,
Osteoporosis.
Thus, D is correct. Ref: KDT, Topic: Peptic ulcer disease

661. D. Hypergastrinemia
Chronic treatment with omeprazole:
Decreases the absorption of vitamin B12
Increased risk of bone fracture
Increased susceptibility to certain infections (e.g., hospital-acquired pneumonia,
community-acquired Clostridium difficile, spontaneous bacterial peritonitis in
patients with ascites)
Hypergastrinemia
Thus, D is correct. Ref: GG, Topic : Peptic ulcer disease

662. D. 40 mg
Pantoprazole:
The FDA approved dose of intravenous pantoprazole
For gastroesophageal reflux disease is 40 mg daily for up to 10 days
160-240 mg given in divided doses
Used in Zollinger-Ellison syndrome
Thus, D is correct. Ref: GG, Topic: Peptic ulcer disease

663. C. H+K+ ATPase pump


Lansoprazole:
Is a proton pump inhibitor
Acts by inhibiting H+ K+ ATPase pump irreversibly
Thus, C is correct. Ref: KDT, Topic: Peptic ulcer disease

664. A. Ranitidine and cimetidine have wide pharmacokinetic variability


Differences between Ranitidine and Cimetidine:
Ranitidine is about 5 times more potent than cimetidine
Ranitidine has similar pharmacokinetic profile and t1/2 of 2-3 hr similar to cimetidine
But ranitidine has longer duration of action (24-hour acid suppression) due to its high
potency
Ranitidine has lesser drug interactions compared to cimetidine
Ranitidine doesn’t cross BBB
Thus, A is correct. Ref: KDT, Topic: Peptic ulcer disease

665. A. Nizatidine
Cimetidine inhibits CYPs
CYP1A2, CYP2C9 and CYP2D6
Ranitidine
Also interacts with hepatic CYPs, but with an affinity of only 10% of that of cimetidine
Ranitidine interferes only minimally with hepatic metabolism of other drugs
Famotidine and Nizatidine:
Are even safer in this regard, with no significant drug interactions mediated by
inhibiting hepatic CYPs.
Thus, A is correct. Ref: GG, Topic : Peptic ulcer disease

666. C. Itopride
Itopride:
Itopride is metabolized mainly by Flavin monooxygenases and not by CYP450
isoenzymes.
So Minimal drug interactions other options
Cisapride, Domperidone and Metoclopropamide metabolized mainly by CYP450
isoenzymes.
Thus, C is correct. Ref: KDT, Topic: Prokinetics

667. C. Domperidone prokinetic action is not attenuated by atropine


Domeperidone:
Its prokinetic action is not attenuated by atropine and is based only on D2 receptor
blockade in upper g.i.t.
Metacloprapimde:
The peripheral action can augment Ach release in the gut, but appears to be minor
Its prokinetic action is attenuated by atropine
Thus, C is correct. Ref: KDT, Topic: Prokinetics

668. D. Both B and C


Metoclopramide:
This is the primary mechanism
Metoclopramide acts in the g.i.t. to enhance Ach release from myenteric motor
neurons
This results from 5-HT4 receptor activation on primary afferent neurons (PAN) of
the ENS via excitatory interneurons
The gastric hurrying and LES tonic effects are mainly due to this action which is
synergized by bethanechol and attenuated by atropine
Peripheral D 2 antagonism:
Hastening gastric emptying
Enhancing LES tone by augmenting Ach release
This is the secondary mechanism
Thus, D is correct. Ref: KDT, Topic: Antiemetics

669. D. Both (b) and (c)


Non-diarrheal uses of ORS include:
Maintenance of hydration in postsurgical, post burn and post-trauma patients.
Heat stroke
During changeover from total parenteral nutrition to external nutrition.
Thus, is correct. Ref: KDT, Topic: diarrhea management

670. C. It has lower Na+ and glucose concentration


New formula ORS contain less Na+ and glucose than standard formula ORS.
Total osmolality is decreased to 245 mmol/L in New Formula WHO ORS.
Thus, C is correct. Ref: KDT, Topic: diarrhea management

671. A. 90 m moles/L
Concentration of Na+ is 90 mmol/L in standard WHO-ORS whereas it is 75 mmol/L in
New Formula ORS.
Thus, A is correct. Ref: KDT, Topic: diarrhea management

672. D. Glucose coupled Na+ absorption


Concentration of Na is 90 mmol/L in standard WHO-ORS whereas it is 75 mmol/L in
New formula ORS.
Thus, D is correct. Ref: KDT, Topic: diarrhea management

673. A. Subacute intestinal obstruction


Stimulant (or irritant) purgatives are contra-indicated in pregnancy and intestinal
obstruction (both subacute as well as chronic).
Thus, A is correct. Ref: KDT, Topic: constipation management

674. B. Lactulose
Lactulose is degraded to lactic acid that converts NH3 to NH4+. As ionic molecules
cannot cross biological membranes, it is not absorbed and is thus excreted.
Thus, B is correct. Ref: KDT, Topic: constipation management

675. C. Aprepitant
Cisplatin induced vomiting has two phases.
Early Phase: It occurs within first 24 hours. 5 HT3 antagonists like ondansetron are
the agents of choice for this condition.
Delayed Phase: Vomiting occurring after 24 hours is less responsive to ondansetron
and other drugs. It is best controlled by substance P antagonist like aprepitant.
Thus, C is correct. Ref: Katzung, 1028 Topic : Antiemetics
676. C. Omeprazole
Treatment of GERD can be accomplished by
Increasing GI motility with Prokinetic drugs (like metoclopramide and Mosapride) or
By decreasing gastric acid secretion with PPIs like omeprazole.
Thus, C is correct. Ref: KDT, Topic: Peptic ulcer disease

677. D. Cisapride
Cisapride is a 5HT4 agonist that can block cardiac K+ channels at high concentration.
When these are administered with microsomal enzyme inhibitors (like erythromycin or
ketoconazole), polymorphic ventricular tachycardia can result.
Mosapride and tegaserod are other 5HT4 agonists that are devoid of arrhythmogenic
action.
Thus, D is correct. Ref: KDT, Topic: antiemetics

678. A. Serotonergic 5-HT4


Gastric motility is increased by ACh. Release of this neurotransmitter is enhanced by
5HT4 receptor stimulation and 5 HT3 and D2 receptor antagonism.
Thus, A is correct. Ref: KDT, Topic: antiemetics

679. D. Ondansetron
5HT3 antagonists like ondansetron, Granisetron and topisetron are the agents of
choice for chemotherapy induced vomiting.
Thus, D is correct. Ref: KDT, Topic: antiemetics

680. A. Domperidone
Levo-dopa induced vomiting is due to stimulation of D2 receptors in CTZ whereas its
antiparkinsonian action is due to agonistic action on D2 receptors in the nigrostriatal
pathway.
Both metoclopramide and Domperidone inhibit D2 receptors in CTZ and thus
counteract vomiting induced by 1-dopa.
Metoclopramide also crosses BBB and thus abolishes the therapeutic action of 1-dopa
by inhibiting central D, receptors.
Domperidone cannot cross BBB, thus does not interfere with antiparkinsonian action
of 1-dopa.
Cisapride (5HT4 agonist) and ondansetron (513T3 antagonist) do not affect
dopaminergic pathway.
Thus, A is correct. Ref: KDT, Topic: antiemetics

681. C. Metoclopramide
Metoclopramide and Domperidone act by blocking D, receptors.
Metoclopramide can cross BBB whereas Domperidone cannot.
Therefore, metoclopramide can produce extra-pyramidal adverse effects while
Domperidone is devoid of it.
Cisapride acts as 5HT4 agonist. It can cause torsades de pointes.
Thus, C is correct. Ref: KDT, Topic: antiemetics

682. B. Increase large intestinal peristalsis


Metoclopramide is a Prokinetic and anti-emetic drug. It acts by blocking 1202 and 5
HT, receptors and stimulating 5-HT4 receptors. D2 receptors are normally inhibitory
to the release of ACh. Metoclopramide enhances ACh release by blocking these
inhibitory D2 receptors. Due to increased release of ACh, there is increase in gastric
motility and LES tone. Prokinetic action can thus be blocked by atrophic.
Metoclopramide has no effect on colonic motility.
Thus, B is correct. Ref: KDT, Topic: antiemetics

683. D. One hour before commencing journey


Hyoscine is administered half to one hour before journey for prevention of motion
sickness. It has no role once the vomiting starts.
Thus, D is correct. Ref: KDT, Topic: antiemetics

684. D. All of the above


Mg (OH)2 has a quick onset whereas Al (OH), acts for a long time.
Magnesium salts cause osmotic diarrhea whereas aluminum salts cause constipation.
Combination of these two agents minimizes the impact on bowel movements.
Thus, D is correct. Ref: KDT, Topic: Peptic ulcer disease

685. C. Misoprostol
Thus, C is correct. Ref: KDT, Topic: Peptic ulcer disease

686. A. Esomeprazole
Proton pump inhibitors are the drugs of choice for peptic ulcer disease due to any
etiology.
Misoprostol is the MOST SPECIFIC drug for the treatment of PUD due to chronic NSAID
use because it is a PGE, analog.
Thus, A is correct. Ref: KDT, Topic: Peptic ulcer disease

687. D. It is indicated in all patients of peptic ulcer


Triple drug therapy for H. pylon is indicated in those patients in whom infection is
detected by urea breath test. Since H. pylon becomes less virulent in the absence of
acid, combination with PPI is more effective.
Because resistance can develop to single agents, these are used in combination.
Colloidal bismuth subcitrate dislodges H. pylon but produces metallic taste and
blackening of tongue.
Thus, D is correct. Ref: KDT, Topic: Peptic ulcer disease

688. C. It has anti H. pylori activity


Colloidal bismuth subcitrate forms an acid resistant protective coating over the ulcer
base. It also dislodges H. Pylori from the surface of gastric mucosa.
Thus, C is correct. Ref: KDT, Topic: Peptic ulcer disease

689. A. Sucralfate
Megaldrate is an antacid. It acts by neutralizing the gastric acid.
Omeprazole and misoprostol decrease the secretion of gastric acid.
Sucralfate is an ulcer protective agent. It forms the protective coating over the ulcer
base.
Thus, A is correct. Ref: KDT, Topic: Peptic ulcer disease

690. D. It does not have anti-androgenic action


Cimetidine is rarely used now because:
It is the least potent It blocker
It is a short acting agent
It is a potent inhibitor of microsomal enzymes
It can cause gynaecomastia
It produces more CNS adverse effects
Thus, D is correct. Ref: KDT, Topic: Peptic ulcer disease

691. A. They prevent stress ulcers in the stomach


H2 receptor blockers like ranitidine are used to prevent stress ulcers.
Most effective agents for inhibiting acid secretion and treatment of gastric as well as
duodenal ulcers are PPIs
Antacids afford most prompt relief of ulcer pain.
Thus, A is correct. Ref: KDT, Topic: Peptic ulcer disease

692. B. Ascorbic acid

Substances Improving the absorption of iron Substances Improving the absorption of iron

1. Acid 1. Antacids

2. Reducing substances like ascorbic acid 2. Phosphates

3. Meat 3. Phylates

4. Tetracyclines

5. Food in the stomach

Thus, B is correct. Ref: KDT, Topic: anemia

693. A. Vitamin B12


Deficiency of vitamin B„ results in megaloblastic anemia and demyelination. It can
cause subacute combined degeneration of spinal cord and peripheral neuritis.
Thus, A is correct. Ref: KDT, Topic: anemia

694. C. Counteracting toxicity of high dose methotrexate therapy


Prophylaxis of neural tube defects require treatment with folic acid.
Methotrexate toxicity can be prevented by 5’-formyltetrahydrofolate (folinic acid).
Pernicious anemia requires the therapy with vitamin B12.
Anemia associated with chronic renal failure is treated with erythropoietin.
Thus, C is correct. Ref: KDT, Topic: anemia

695. C. Conversion of malonic acid to succinic acid


Conversion of malonic acid to succinic acid requires vitamin B12 but not folate. This
reaction is required for myelin formation and deficiency of B12 is responsible for
demyelination.
Thus, C is correct. Ref: KDT, Topic: anemia

696. C. Cannot be injected i.v.


Iron sorbitol citrate should not be used i.v. because it may rapidly saturate the
transferrin receptors and can result in high concentrations of free iron.
Thus, C is correct. Ref: KDT, Topic: anemia

697. B. Motility
Diphenoxylate is an opioid; it binds to mu receptors in the GIT and slows motility.
Thus, B is correct. Ref: KAT, General considerations

698. D. Pancreatic lipase


In young male, a history of recurrent respiratory infections with P. aeruginosa, chronic
diarrhea, weight loss and death of a sibling due to respiratory infection suggests a
diagnosis of cystic fibrosis (CF). Chronic diarrhea and weight loss in patients with CF
are typically caused by malabsorption secondary to pancreatic insufficiency and can
be corrected by pancreatic enzyme supplementation.
Thus, D is correct. Ref: KAT, Topic: diarrhea management

699. D. They should be used only as a short term measure after ensuring that
enter invasive organisms are not involved
Thus, D is correct. Ref: KDT, Topic: diarrhea management

700. C. Prednisolone
Corticosteroids are the mainstay of treatment of acute exacerbation of ulcerative
colitis / acute exacerbation of any autoimmune disorder.
Thus, C is correct. Ref: KDT, Topic: inflammatory bowel disease
Antimicrobial Drugs — Questions
701. Drugs used for giardiasis during the 1st trimester of pregnancy is
A. Metronidazole
B. Tinidazole
C. Clindamycin
D. Paromomycin

702. Metronidazole is used in the following conditions except


A. Amoebiasis
B. Histoplasmosis
C. Trichomoniasis
D. Giardiasis

703. Which of the following is not a luminal amoebicide


A. TetracyclineS
B. Paromomycin
C. Diloxanide furoate
D. Dihydroemetine

704. Which of the following is used in Severe babesiosis


A. Clindamycin and Quinine
B. Azithromycin and Atovaquone
C. Tetracycline
D. Cotrimoxazole

705. Drug of choice for the Chagas disease is


A. Metronidazole
B. Suramin
C. Melarsoprol
D. Benznidazole

706. Drug affecting calcium channels of flukes is


A. Albendazole
B. Praziquantel
C. Ivermectin
D. Piperazine

707. Antihelminthic Drug contraindicated in seizure disorder is


A. DEC
B. Piperazine
C. Albendazole
D. Pyrantel Pamoate
708. Following Anti-retroviral drugs are triple phosphyraled for its activation
except
A. Stavudine
B. Lamivudine
C. Tenofovir
D. Zidovudine

709. Phosphorylated Antiretroviral drugs with low affinity for DNA polymerase-
y are all except
A. Emtricitabine
B. Zidovudine
C. Lamivudine
D. Tenofovir

710. Which of the following is an integrase inhibitor


A. Tipranavir
B. Darunavir
C. Atazanavir
D. Dolutegravir

711. The following Antiretroviral drug is a cytidine analogue


A. Zidovudine
B. Lamivudine
C. Stavudine
D. Didanosine

712. Which of the following antibacterial drug is not used in the treatment of
malaria
A. Tetracycline
B. Doxycycline
C. Clindamycin
D. Clarithromycin

713. Which is the following antimalarial drug act by inhibiting bifunctional


plasmodial dihydrofolate reductase-thymidylate synthetase
A. Primaquine
B. Proguanil
C. Pyronaridine
D. Piperaquine
701. d 702. b 703. d 704. a 705. d 706. b 707. b 708. c 709. b

710. d 711. b 712. d 713. b

714. A drug which acts by preventing by polymerization of heme to hemozoin


in malarial parasite is
A. Primaquine
B. Artesunate
C. Lumefantrine
D. Chloroquine

715. Which of the following is incorrect about primaquine


A. It is poor erythrocytic schizontocide
B. More active against the pre-erythrocytic stage of P. falciparum than that of P. vivax
C. Not effective on gametocytes
D. Effective on primary as well as secondary hepatic phases of the malarial parasite

716. A patient is diagnosed with chloroquine-resistant falciparum malaria. His


treating physician advice nurse to administer quinine by IV infusion. The
preferred IV fluid for quinine infusion is
A. Normal saline
B. 5% dextrose
C. Ringer lactate
D. ½ Normal saline

717. A patient was on treatment for multibacillary leprosy from last 3 months.
Now the patient developed easy fatiguability, decreased interest in work
and pallor n examination. What drug is responsible for the patient’s
symptom
A. Rifampicin
B. Clofazimine
C. Minocycline
D. Dapsone

718. About pretomanid. True is


A. Inhibits M. tuberculosis mycolic acid synthesis
B. Inhibits M. tuberculosis protein synthesis
C. Generates reactive nitrogen species
D. All of the above

719. Which is the following inhibits mycobacterial ATP SYNTHESIS


A. Isoniazid
B. Rifampicin
C. Ethambutol
D. Bedaquilline

720. Which of the following inhibits mycobacterial cell membrane synthesis


and trans-translation
A. Isoniazid
B. Pyrazinamide
C. Bedaquilline
D. Ethambutol
721. Uveitis and arthralgias are seen in a patient taking rifabutin doses 450
mg daily in combination with the following antifungal agent
A. Fluconazole
B. Itraconazole
C. Posaconazole
D. Ketoconazole

722. Safest 1st line ATT in renal failure is


A. Rifampicin
B. Streptomycin
C. Isoniazid
D. Ethambutol

723. “Shoulder-hand” syndrome is caused by the following drug


A. Ethambutol
B. Streptomycin
C. Bacitracin
D. Isoniazid

724. Isoniazid overdose has been associated with the clinical trial of below
except
A. Seizures refractory to treatment with phenytoin and barbiturates
B. Hepatotoxicity
C. Metabolic acidosis with an anion gap that is resistant to treatment with sodium
bicarbonate
D. Coma

725. “Grey baby syndrome” following statements are true except


A. Neonates, especially if premature are at greater risk
B. Mechanisms are due to inadequate/Deficiency of glucuronyl transferase
C. If baby recovers there will be no long-term sequelae
D. Begins within 24 hrs after treatment with chloramphenicol

726. Onycholysis and pigmentation of the nails is seen with following


antimicrobial drug
A. Amoxicillin
B. Neomycin
C. Mupirocin
D. Tetracycline

714. d 715. c 716. b 717. d 718. d 719. d 720. b 721. a 722. a

723. d 724. b 725. d 726. d

727. The wrong statement about Tigecycline is


A. Belongs to glycylcycline Group
B. Acts by inhibiting protein translation in bacteria by binding to the30S ribosomal
subunit
C. Active against MRSA, VRE, Multi-drug resistant streptococci
D. Dose adjustment needed in renal failure

728. Minocycline sustained release microspheres for sub gingival;


administration is done for
A. Oral ulcers due to anaerobic infections
B. Treatment of Ludwig’s angina
C. To reduce pocket depth in patients with adult periodontitis in dentistry
D. None of the above

729. Following drugs has the highest oral bioavailability


A. Doxycycline
B. Tetracycline
C. Demeclocycline
D. Minocycline

730. Following drugs inhibit bacterial protein synthesis except


A. Clindamycin
B. Daptomycin
C. Spectinomycin
D. Clarithromycin

731. Patients should be cautioned not to consume alcohol when given


prescription for any of the following EXCEPT
A. Chlorpropamide
B. Cefoperazone
C. Cefixime
D. Metronidazole

732. This drug depolarizes cell membranes of aerobic gram positive bacteria.
It is effective against vancomycin resistant enterococcal infections. It may
cause myopathy especially in patients taking statins. It is
A. Teicoplanin
B. Linezolid
C. Streptogramin
D. Daptomycin

733. A patient needs anti-biotic treatment for artificial valve, culture-positive


infective enterococcal endocarditis. His medical history includes severe
anaphylactic reaction to penicillin G during the past year. The best approach
would be treatment with
A. Vancomycin
B. Aztreonam
C. Cefazolin plus gentamicin
D. Amoxicillin-clavulanic acid
734. Drugs that can be used to treat infections caused by Bacteroides fragilis
are all EXCEPT
A. Metronidazole
B. Amikacin
C. Vancomycin
D. Trovafloxacin

735. A 14-year-old boy present with headache, fever and cough for 2 days.
Sputum is scant and non-purulent and gram statins reveals many white cells
but no organisms. The treatment should be initiated with
A. Cefazolin
B. Amikacin
C. Erythromycin
D. Trovafloxacin

736. The drug that should be used for prophylaxis of close contacts of a
patient suffering from meningococcal meningitis is
A. Erythromycin
B. Dapsone
C. Rifampicin
D. Amikacin

737. For a 23 old pregnant female having severe sensitivity to amoxicillin,


drug used to treat gonorrhea in a single dose should be
A. Ceftriaxone
B. Spectinomycin
C. Ciprofloxacin
D. Tetracycline

727. d 728. c 729. d 730. b 731. c 732. d 733. a 734. b 735. c

736. c 737. d

738. Antimicrobials effective against anaerobic bacteria include the following


EXCEPT
A. Metronidazole
B. Clindamycin
C. Chloramphenicol
D. Tobramycin

739. Which of the following drugs i6 LEAST likely to require dosage reduction
in renal dysfunction?
A. Amikacin
B. Ciprofloxacin
C. Vancomycin
D. Clindamycin
740. Which of the following drugs is most likely to cause loss of equilibrium
and auditory damage?
A. Isoniazid
B. Ethambutol
C. Amikacin
D. Rifabutin

741. The combination of trimethoprim and sulfamethoxazole is effective


against which of the following opportunistic infections in the AIDS patient?
A. Disseminated Herpes simplex
B. Pneumocystis jiroveci
C. Cryptococcal meningitis
D. Tuberculosis

742. A contraindication to the use of ciprofloxacin is a history of


A. Gout
B. Deep vein thrombosis
C. Epilepsy
D. G-6 PD deficiency

743. Methanamine salts are used as urinary antiseptics. The reason they lack
systemic antibacterial action is that they are
A. Converted to formaldehyde at low urinary pH
B. Rapidly metabolized by liver drug metabolizing enzymes
C. Not absorbed into systemic circulation after oral use
D. Substrates for active tubular secretion

744. Maximum incidence of photo toxicity is associated with


A. Norfloxacin
B. Cotrimoxazole
C. Lomefloxacin
D. Sparfloxacin

745. Which fluoroquinolones is highly active against Mycobacterium leprase


and is being used in alternative multidrug therapy regimens
A. Norfloxacin
B. Ciprofloxacin
C. Ofloxacin
D. Lomefloxacin

746. Which of the following adverse effects is most likely to occur with
sulfonamides?
A. Neurologic effects including headache, dizziness, and lethargy
B. Skin reactions
C. Fanconi’s anemia
D. Hematuria
747. Which of the following statements about the fluoroquinolones is FALSE?
A. A fluoroquinolone is the drug of choice for treatment of an uncomplicated urinary
tract infection in a 7-year-old girl
B. Modification of fluoroquinolones dosage is required in patients if creatinine
clearance is less than 50 mL/min
C. Gonococcal resistance to fluoroquinolones may involve changes in DNA gyrase
D. Fluoroquinolones inhibit relaxation of positively supercoiled DNA

748. Which of the following statements about sulfonamides is FALSE?


A. Sulfonamide crystal Luria is most likely to occur at low urinary pH
B. Dysfunction of the basal ganglia may occur in the newborn sulfonamides are
administered late in pregnancy
C. Sulfonamides inhibit bacterial dihydrofolate reductase
D. Sulfonamides are antimetabolites

738. d 739. d 740. c 741. b 742. c 743. a 744. d 745. c 746. b

747. a 748. c

749. This inhibitor of bacterial protein synthesis has a narrow spectrum of


antibacterial activity. It has been used in the management of abdominal
abscess caused by Bacteroides fragilis, but antibiotic associated colitis has
occurred. Which of the following drugs i8 being described?
A. Clarithromycin
B. Ticarcillin
C. Minocycline
D. Clindamycin

750. Which of the following drugs is most effective against an organism


producing aminoglycoside inactivating enzymes?
A. Gentamicin
B. Streptomycin
C. Amikacin
D. Tobramycin

751. Bactericidal inhibitors of protein synthesis are


A. Tetracyclines
B. Lincosamides
C. Macrolides
D. Aminoglycosides

752. The following tetracycline has the potential to cause vestibular toxicity
A. Doxycycline
B. Demeclocycline
C. Minocycline
D. Tetracycline
753. Regarding the antibacterial action of gentamicin, which of the following
statements is most accurate?
A. Efficacy is directly proportionate to the time that the plasma level of the drug is
greater than the minimal inhibitory concentration
B. The antibacterial action of gentamicin is time dependent
C. Antibacterial activity is often reduced by the presence of an inhibitor of cell wall
synthesis
D. Gentamicin continues to exert antibacterial effects even after plasma levels
decrease below detectable range

754. Which of the following statements about the clinical uses of the
aminoglycosides is FALSE?
A. The spectrum of antimicrobial activity of aminoglycosides includes Bacteroides
fragilis
B. Aminoglycosides are often used in combination with cephalosporin in the
empirical treatment of life-threatening bacterial infections
C. Owing to their polar nature, aminoglycosides are not absorbed following oral
administration
D. Gentamicin is used with ampicillin for synergistic effects in the treatment of
enterococcal endocarditis

755. Concerning streptogramins, which one of the following statements is


FALSE?
A. They are active against methicillin-resistant staphylococci
B. They may cause a syndrome of arthralgia and myalgia
C. They are used in the management of infections caused by vancomycin-resistant
enterococci
D. They induce formation of hepatic drug metabolizing enzymes

756. Which one of the following statements about doxycycline is FALSE?


A. It is more active than tetracycline against H. pylori
B. It is excreted mainly in the feces
C. It is bacteriostatic
D. It is used in Lyme’s disease

749. d 750. c 751. d 752. c 753. d 754. a 755. d 756. a

757. A 36 years old woman recently treated for leukemia is admitted to the
hospital with malaise, chills and high fever. Gram stain of blood reveals the
presence of gram negative bacilli. The initial diagnosis is bacteremia and
parenteral antibiotics are indicated. The record of the patient reveals that
she had severe urticarial rash, hypotension and respiratory difficulty after
oral penicillin V about 6 months ago. The most appropriate drug should be
A. Ampicillin plus sulbactum
B. Imipenem plus cilastatin
C. Cefazolin
D. Aztreonam
758. This drug has activity against many strains of P. aeruginosa. However,
treatment. Tire drug should not be used in penicillin-allergic patients. Its
activity against gram-negative rods is enhanced if it is given in combination
with tazobactum. Which of the following drugs is being described?
A. Piperacillin
B. Aztreonam
C. Amoxicillin
D. Vancomycin

759. TRUE statement regarding vancomycin is


A. It is not susceptible to Penicillinases
B. It has the advantage of high oral bioavailability
C. It is bacteriostatic
D. Staphylococcal enterocolits occurs commonly with its use

760. The following is true of vancomycin EXCEPT


A. It can cause deafness as a dose related toxicity
B. It acts by inhibiting bacterial protein synthesis
C. It is an alternative to penicillin for enterococcal endocarditis
D. It is a bactericidal antibiotic active primarily against gram positive bacteria

761. Amoxicillin + clavulanic acid is active against the following organisms


EXCEPT
A. Penicillinase producing N. gonorrhea
B. Penicillinase producing staph. aureus
C. Methicillin resistant Staph. aureus
D. P-lactamase producing E. coli

762. Methicillin resistant staphylococci do not respond to β-lactam antibiotics


because
A. They have acquired penicillin binding protein which has low affinity for β-lactam
antibiotics
B. They elaborate an amidase which destroys methicillin and related
C. They produce a β-lactamase which destroys methicillin and related
D. They are less permeable to β-lactam antibiotics

763. Which of the following statements about imipenem is most accurate?


A. The drug has a narrow spectrum of antibacterial action
B. It is used in fixed dose combination with sulbactum
C. Imipenem is active against methicillin-resistant staphylococci
D. In renal dysfunction, dosage reductions are necessary to avoid

764. Which of the following statement about the bio disposition of penicillins
and cephalosporins is NOT accurate
A. Oral bioavailability is affected by lability to gastric acid
B. Renal tubular reabsorption of beta-lactams is inhibited by probenecid
C. Procaine penicillin G is used via intramuscular injection
D. Nafcillin and ceftriaxone are eliminated mainly via biliary secretion

765. Which of the following drugs is NOT excreted in bile


A. Erythromycin
B. Gentamicin
C. Rifampicin
D. Ampicillin

757. d 758. a 759. a 760. b 761. c 762. a 763. d 764. b 765. b

766. Which of the following is not an established antimicrobial drug synergism


at clinical level?
A. Amphotericin B and flucytosine in cryptococcal meningitis
B. Carbenicillin and gentamicin in pseudomonal infections
C. Trimethoprim and sulfamethoxazole in coliform infections
D. Penicillin and tetracycline in bacterial meningitis

767. The persistent suppression of bacterial growth that may occur after
limited exposure to some antimicrobial drug is called
A. Post antibiotic effect
B. Time dependent killing
C. Concentration dependent killing
D. Sequential blockade

768. All of the following antibiotics act by interfering with cell wall formation
EXCEPT
A. Ceftriaxone
B. Clindamycin
C. Cycloserine
D. Vancomycin

769. A 38-year-old woman with recurrent hematuria is scheduled for


cystoscopy. She is otherwise healthy, has never had surgery, takes no
medications, does not smoke cigarettes, does not drink alcohol, and does
not use IV drugs. Review of systems is negative. Physical exam is normal,
except for a sharp, short sound heard in mid-systole followed by a grade II/VI
murmur audible at the apex. What antibiotic prophylaxis for infective
endocarditis should be given to this patient?
A. One dose of oral amoxicillin prior to the procedure
B. Intravenous ceftriaxone
C. Intravenous ampicillin plus gentamicin preoperatively
D. None

770. A 70-year-old male, status post AVR replacement 2 years ago for aortic
stenosis, presents with widespread ecchymosis on his back and legs and
some bruising on the back of both hands. His last INR was 3 weeks ago and
was 3. He states he saw a doctor 6 days ago for a cough and was put on a
medication described as a “white tablet. “His chronic medications include:
warfarin 5 mg qd, albuterol inhaler 2 puffs 4 times a day, and nortriptyline
25 mg q has. Which of the following medications was he placed on?
A. Cefixime
B. Trimethoprim/sulfamethoxazole
C. Amoxicillin
D. Codeine
766. d 767. a 768. b 769. d 770. b
Antimicrobial Drugs — Explanations
701. D. Paromomycin
Paromomycin is an alternative drug for giardiasis, especially during the 1st trimester of
pregnancy when metronidazole and other drugs are contraindicated.
Thus, D is correct. Ref: KDT, Topic: antihelminthics

702. B. Histoplasmosis
Uses of metronidazole
Amoebiasis
Trichomoniasis
Giardiasis
Bacterial vaginosis
Pseudomembranous colitis
H. Pylori
Anaerobes: Bacteroides fragile and Clostridium dificile
Acute necrotizing ulcerative gingivitis
Guinea worm infestation
Thus, B is correct. Ref: KDT, Topic: antiamoebics

703. d. Dihydroemetine
Both intestinal (luminal) and extra-intestinal herbicides: Nitroimidazoles
(metronidazole, tinidazole secnidazole, ornidazole), emetine and dehydroemetine.
Luminal amoebicides only: Diloxanide furoate, paromomycin, iodoquinol,
quiniodochlor and tetracyclines
Tissues (extra-intestinal amoebicides only: Chloroquine
Thus, D is correct. Ref: KDT, Topic: antiamoebics

704. A. Clindamycin and Quinine


Clindamycin and Quinine:
Used in Severe babesiosis
Azithromycin and Atovaquone
Used in Mild-moderate babesiosis
Thus, A is correct. Ref: GG, Miscellaneaous infections

705. D. Benznidazole
Drug of choice for Chagas disease in Benznidazole Alternative treatment for Chagas is
Nifurtimox.
Thus, D is correct. Ref: GG, Miscellaneaous infections

706. B. Praziquantel
Mechanism of action of Praziquantel:
Selectively of action of praziquantel on tapeworms and flukes may be dependent on
the presence of a specific variant of Ca2+ channel sensitive to Praziquantel in these
worms
The tapeworms lose grip of the intestinal mucosa and are expelled
Flukes and schistosomes are also dislodged in tissues and veins.
Praziquantel is active against adult as well as juvenile and larval stages of tapeworms
Thus, B is correct. Ref: KDT, Topic: antihelminthics

707. B. Piperazine
Piperazine
Excitement saw at higher doses and convulsions at toxic doses
Safe in pregnancy
Contraindicated in
Renal insufficiency
Epilepsy
Thus, B is correct. Ref: KDT, Topic: antihelminthics

708. C. Tenofovir
NRTIs are nucleosides that must be triphosphorylated at the 5-hydroxyl to exert
activity. The sole exception, tenofovir, is a nucleotide monophosphate analogue that
requires two additional phosphates to acquire full activity.
Thus, C is correct. Ref: GG, Topic : anti HIV / antiviral drugs

709. B. Zidovudine
Phosphorylated emtricitabine, lamivudine, and tenofovir have low affinity for DNA
polymerase-y and are largely devoid of mitochondrial toxicity
Among NRTIs: the intercellular triphosphates for all NRTIs drugs have low affinity for
human DNA polymerase-a and-b, some are capable of inhibiting human DNA
polymerase-y
DNA polymerase-y is a mitochondrial enzyme
As a result, the important toxicities common to this class of drugs result in part from
the inhabitation of mitochondrial DNA synthesis
These toxicities include anaemia, granulocytopenia, myopathy, peripheral neuropathy
and pancreatitis
Thus, B is correct. Ref: GG, Topic : anti HIV / antiviral drugs

710. D. Dolutegravir
Integrase inhibitors:
Raltegravir--- RAL
Elvitegravir---EVG
Dolutegravir---DTG
Protease inhibitors:
Tipranavir
Darunavir
Atazanavir
Thus, D is correct. Ref: GG, Topic : anti HIV / antiviral drugs

711. B. Lamivudine
Cytidine analogue:
Lamivudine AND emtricitabine
Emtricitabine is a cytidine analogue chemically related to lamivudine and shares
many of that drug’s pharmacodynamic properties
Thymidine analogue:
Zidovudine and stavudine
So not to be combined as the compete for thymidine kinase for activation leading to
pharmacological antagonism
Adenosine analogue:
Tenofovir: the ONLY nucleotide approved for HIV treatment
Didanosine
Guanosine analogue:
Abacavir: The only approved antiretroviral that is active as a guanosine analogue
Thus, B is correct. Ref: GG, Topic : anti HIV / antiviral drugs

712. D. Clarithromycin
The antibacterial drug s used in the treatment of malaria
Tetracycline
Doxycycline
Clindamycin
Sulfonamides: sulfadoxine, Sulfamethipyrazine
Sulfone: Dapsone
Thus, D is correct. Ref: KDT, Topic: antimalarials

713. B. Proguanil
Proguanil:
The drug acts by inhibiting bifunctional plasmodial dihydrofolate reductase-
thymidylate synthetase that is crucial for parasite de novo purine and pyrimidine
synthesis.
A fixed combination of atovaquone with proguanil hydrochloride is available in the
U.S. for malaria chemoprophylaxis
Thus, B is correct. Ref: GG, Topic: antimalarials

714. D. Chloroquine
Chloroquine:
MOA:
Act by preventing polymerization of heme to hemozoin in the malarial parasite
Resulting in accumulation of heme WHICH is toxic for the parasite
Thus, D is correct. Ref: KDT, Topic: antimalarials

715. C. Not effective on gametocytes


Primaquine:
Has poor erythrocytic schizontocide
Has weak action on P. vivax
Blood forms of P. falciparum are totally insensitive
It is more active against the preerhthrocytic stage of P. falciparum than that of P. vivax
Has marked effect on primary as well as secondary hepatic phases of the malarial
parasite
It is highly active against gametocytes and hypnozoites
That’s why used for the redical cure of relapsing (vivax) malaria
Thus, C is correct. Ref: KDT, Topic: antimalarials

716. B. 5% dextrose
A patient was diagnosed with chloroquine-resistant falciparum malaria. His treating
physician advice nurse to administer quinine by IV infusion. The nurse will give
quinine infusion through 5% dextrose. Hypoglycaemia due to hyperinsulinemia
is the most important side effect which can be prevented by infusing
quinine in 5% dextrose.
Adverse effects of quinine infusion
Hypoglycemia is also common, mostly in the treatment of severe malaria, and can
be life-threatening if not treated promptly with intravenous glucose.
Hypotension is rarer but also serious and most often is associated with excessively
rapid intravenous infusions of quinine or Quinidine
Quinine diluted preferably in 5% (w/v) glucose solution to counteract hypoglycemia
In the absence of glucose, physiological saline may be used
This method of administration minimizes the danger of severe hypotension and
subsequent respiratory collapse.
Don’t give quinine as a bolus
OTHER ADVERSE EFFECTS:
‘Cinchonism’ syndrome
Large-scale dose or higher therapeutic doses are taken for a few days
Consists of ringing in ears, nausea, vomiting (due to both gastric irritation and CTZ
stimulation), headache, mental confusion, vertigo, difficulty in hearing
Visual defects (due to direct neurotoxicity as well as constriction of retinal and
auditory vessels).
Diarrhoea, flashing and marked perspiration
Completely STOPS since the drug is stopped
Seldom cause hemolysis and idiosyncratic reactions
Thus, B is correct. Ref: KDT, Topic: antimalarials

717. D. Dapsone
Dapsone:
Causes anaemia and can increase/decompensate pre-existing anaemia
If haemoglobin is less than 10 gm %, start iron, folic acid, ascorbic acid and advise
the person to take high protein diet along with MDT
Thus, D is correct.

718. D. All of the above


Pretomanid:
Bicyclic nitroimidazopyrans
Pretomanid has two mechanisms of action
Under aerobic conditions, it inhibits M. tuberculosis mycolic acid and protein
synthesis
Is a prodrug that requires activation by the bacteria and generates reactive nitrogen
species
USE:
Treatment of extensively drug-resistant and multidrug-resistant TB
Thus, D is correct. Ref: GG, Topic: Antitubercular drugs

719. D. Bedaquilline
Bedaquilline
Mycobacterial ATP SYNTHESIS
Isoniazid and Ethionamide:
Inhibit mycolic acid synthesis
Rifampicin
Inhibits RNA synthesis by targeting RNA polymerase
Ethambutol
Inhibits cell wall synthesis
Thus, D is correct. Ref: GG, Topic : Antitubercular drugs

720. B. Pyrazinamide
Bedaquilline
Mycobacterial ATP SYNTHESIS
Isoniazid and Ethionamide:
Inhibit mycolic acid synthesis
Ethambutol
Inhibits cell wall synthesis
Pyrazinamide
Inhibits cell membrane synthesis and trans-translation
Thus, B is correct. Ref: GG, Topic : Antitubercular drugs

721. A. Fluconazole
Uveitis and arthralgias are seen in a patient taking rifabutin doses 450 mg daily in
combination with fluconazole
Advise patient to discontinue the drug if visual symptoms (pain or blurred vision)
occur
Rifabutin causes an orange-tan discolouration of skin, urine, faeces, saliva, tears and
contact lenses, like
Really, thrombocytopenia, a flu-like syndrome, hemolysis, myositis, chest pain, and
hepatitis develop in patients treated with rifabutin
Unique side effects include polymyalgia, pseudojaundice and anterior uveitis
Uveitis and arthra; as are seen in patients taking rifabutin doses > 450 mg daily in
combination with clarithromycin also.
Thus, A is correct. Ref: GG, The Pharmacological Basis of Therapeutics, Topic:
Antitubercular drugs

722. A. Rifampicin
Safest first line ATT in renal failure is rifampicin because it is majorly eliminated by
OATP1 channel in the bile. Thus, A is correct. Ref: GG, The Pharmacological Basis of
Therapeutics, Topic: Antitubercular drugs Topic: Antitubercular drugs

723. D. Isoniazid
Shoulder-hand syndrome is caused by Isoniazid. It’s a type of neuropathy due to
pyridoxine deficiency. Other adverse effects of ISONIAZID include hepatotoxicity,
peripheral neuropathy, psychosis, amnesia, risk of lupus like syndrome, Thus, D is
correct. Ref: GG, The Pharmacological Basis of Therapeutics, Topic: Antitubercular
drugs Topic: Antitubercular drugs

724. B. Hepatotoxicity
Isoniazid overdose has been associated with the clinical Trial of
Seizures refractory to treatment with phenytoin and barbiturates
Metabolic acidosis with an anion gap that is resistant to treatment with sodium
bicarbonate
Coma
Hepatoxicity is caused by Isoniazid but it’s not in the trial evidence. It was noticed in
post marketing surveillance not in CONTROLLED CLINICAL TRIAL.
Thus, B is correct. Ref: GG, Topic: Antitubercular drugs
725. D. Begins within 24 hrs after treatment with chloramphenicol
“Grey baby syndrome”
Neonates, especially if premature
Begin 2-9 days after treatment
Within the first 24 hours:
Vomiting, refusal to suck
Irregular rapid respiration, abdominal distention
Periods of cyanosis, and passage of loose, green stools
Next 24 hours: Ashen-grey colour, flaccid hypothermic
If recovered: - no sequelae
Mechanisms:
Deficiency of glucuronyl transferase (a hepatic enzyme that metabolizes
chloramphenicol in first 3-4 weeks of life)
Inadequate renal extraction of unconjugated drug.
Toxic effects have not been observed in the newborns when as much as 1g of the
antibiotic has been given every 2 hours to the mother during labour.
Thus, D is correct. Ref: GG, Topic : Chloramphenicol

726. D. Tetracycline
Onycholysis and pigmentation of the nails may develop with or without accompanying
Photosensitivity with the use of tetracyclines.
Thus, D is correct. Ref: GG, Topic : Tetracyclines

727. D. Dose adjustment needed in renal failure


Tigecycline:
Belongs to glycylcycline Group
Is active against most bacteria that have developed resistance to the classical
tetracyclines
Acts by inhibiting protein translation in bacteria by binding to 30S ribosomal subunit
It blocking entry of amino-acyl tRNA molecules streptococci
Excreted in bile no dose adjustment in renal failure
Tigecycline lacks cross-resistance with tetracycline due to
Low affinity for efflux pumps
In bacteria, ribosomal protection protein against tetracycline is less active in
protecting the ribosomal binding site from Tigecycline.
Thus, is correct. Ref: KDT, Topic: Newer drugs

728. C. To reduce pocket depth in patients with adult periodontitis in dentistry


Minocycline sustained release microsphere for sub gingival administration is used in
dentistry as an adjunct to scaling and root planning procedures to reduce
pocket depth in patients with adult periodontitis.
Thus, C is correct. Ref: GG

729. D. Minocycline
Oral absorption of most Tetracyclines is incomplete
The percentage of an oral dose that is absorbed with an empty stomach is
Modest for Demeclocycline and tetracycline (60-80%)
High for doxycycline (95%) and minocycline (100%)
Tigecycline
Is not appreciably absorbed from the gastrointestinal tract
Only available for parenteral administration
Thus, D is correct. Ref: GG, Topic : Tetracyclines

730. B. Daptomycin
Bacteriostatic, protein-synthesis inhibitors are
Tetracyclines
Glycycines
Chloramphenicol
Macrolides
Ketolides
Lincosamides (Clindamycin)
Streptogramins
(Quinupristin/Dalfopristin)
Oxazolidinones (linezolid)
Aminocyclitols (Spectinomycin)
Lipopeptides (Daptomycin) is bactericidal
Acts by causing depolarization of bacterial cell membranes with K+ efflux a rapid cell
death.
Thus, B is correct. Ref: GG, Topic : MRSA drugs

731. C. Cefixime
Drugs showing disulfiram like reaction with alcohol are
Cefamandole, Cefoperazone, Cefotetan
Moxalactam
Chlorpropamide
Metronidazole
Griseofulvin
Procarbazine
Thus, C is correct. Ref: KDT, Topic: Beta lactams

732. D. Daptomycin
Daptomycin is a newer antibiotic that acts by causing depolarization of bacterial cell
membranes. It is effective in MRSA, VRSA and even Streptogramin resistant SA
infections as well as VRE infections. It can cause myopathy in patients taking statins.
Thus, D is correct. Ref: KDT, Topic: MRSA drugs

733. A. Vancomycin
Severe allergy to penicillin rule out the use of amoxicillin and cefazolin.
Vancomycin is highly effective against MRSA and enterococcal infections.
Drugs for VRSA and VRE (Vancomycin Resistant Enterococcus faecalis) include
linezolid and Daptomycin.
Thus, A is correct. Ref: KDT, Topic: glycopeptides

734. B. Amikacin
Drugs used to treat anaerobic organisms include:
Metronidazole
Moxifloxacin and Trovafloxacin
Vancomycin
Clindamycin
Chloramphenicol
Aminoglycosides are ineffective against anaerobic organisms
Thus, B is correct. Ref: KDT, Topic: aminoglycosides

735. C. Erythromycin
Diagnosis is atypical pneumonia and DOC is erythromycin.
Thus, C is correct. Ref: KDT, Topic: macrolides

736. C. Rifampicin
Rifampicin and ciprofloxacin are used for the prophylaxis of meningococcal meningitis.
DOC for mass prophylaxis of meningococcal meningitis is CEFTRIAXONE. In order or
preference ceftriaxone>ciprofloxacin >rifampicin.
Thus, C is correct. Ref: KDT, Topic: Antitubercular drugs

737. B. Spectinomycin
Ceftriaxone is contraindicated in patients having severe allergy to penicillin.
Tetracyclines are not effective against gonorrhea.
Although ciprofloxacin is effective as a single dose treatment of gonorrhea but it is
contraindicated in pregnancy.
Spectinomycin can be given as single dose treatment of PPNG.
Thus, B is correct. Ref: KAT, Topic Aminoglycosides

738. D. Tobramycin
Aminoglycosides require oxygen for transport in the bacterial cell. These are therefore
ineffective against anaerobic organisms.
Thus, D is correct. Ref: KDT, Topic: Aminoglycosides

739. D. Clindamycin
Clindamycin is excreted by biliary route and therefore can be used safely in renal
dysfunction.
Other drugs excreted in bile are:
Ampicillin
Nafcillin
Chloramphenicol
Novobiocin
Rifampicin
Doxycycline
Thus, D is correct. Ref: KDT, Topic: Aminoglycosides

740. C. Amikacin
Aminoglycosides can cause nephrotoxicity, ototoxicity and neuromuscular block.
Thus, C is correct. Ref: KDT, Topic: Aminoglycosides

741. B. Pneumocystis jiroveci


Cotrimoxazole is effective against Pneumocystis and toxoplasmosis.
Thus, B is correct. Ref: KDT, Topic: Sulfonamides

742. C. Epilepsy
Ciprofloxacin is contraindicated with NSAIDs because this combination results in
increased risk of seizures.
It is also contra-indicated with theophylline because it increases the risk of
theophylline toxicity by inhibiting its metabolism.
It is contra-indicated in pregnancy because it increases the risk of cartilage damage
in newborn.
Thus, C is correct. Ref: KDT, Topic: Fluoroquinolones

743. A. Converted to formaldehyde at low urinary pH


Methanamine mandelate is used as a urinary antiseptic. It acts by conversion to
formaldehyde at acidic urinary pH. Mandelic acid formed from this salt also serves the
function of urinary acidifying agent. These are not effective against proteus because
proteus results in the formation of ammonia which alkalinizes the urine.
Thus, A is correct. Ref: KDT, Topic: UTI treatment drugs Please see section on
alkalinizers and acidifiers of urine

744. D. Sparfloxacin
Maximum phototoxic fluoroquinolone is Sparfloxacin.
Pefloxacin and Lomefloxacin are also phototoxic but less than Sparfloxacin.
Thus, D is correct. Ref: KDT, Topic: fluoroquinolones

745. C. Ofloxacin
Single lesion single dose treatment of leprosy utilizes ROM therapy.
R: Rifampicin
0: Ofloxacin
M: Minocycline Now days, even single lesion leprosy is treated as paucibacillary
leprosy.
Thus, C is correct. Ref: KDT, Topic: fluoroquinolones

746. B. Skin reactions


Rash is the most common adverse effect seen with the use of sulfonamides.
Thus, B is correct. Ref: KDT, Topic: sulfonamides

747. A. A fluoroquinolone is the drug of choice for treatment of an


uncomplicated urinary tract infection in a 7-year-old girl
Fluoroquinolones are contra-indicated in children (due to risk of cartilage damage)
and pregnant female.
Most common mode of resistance to fluoroquinolones is mutation in DNA gyrase.
Dose of fluoroquinolones should be adjusted in renal failure (except moxifloxacin and
trovafloxacin).
These drugs act by inhibiting DNA gyrase.
Thus, A is correct. Ref: KDT, Topic: fluoroquinolones

748. C. Sulfonamides inhibit bacterial dihydrofolate reductase


Sulfonamides are antimetabolites that act by inhibiting the enzyme folate synthase.
Dihydrofolate reductase is inhibited by methotrexate, trimethoprim and
pyrimethamine.
These can cause kernicterus in new born if used in late pregnancy.
At acidic pH, these can get precipitated and result in crystalluria.
Thus, C is correct. Ref: KDT, Topic: sulfonamides
749. D. Clindamycin
Important points about clindamycin are:
It acts by inhibiting protein synthesis (via. inhibition of translocation).
It is effective against anaerobic organisms.
Pseudomembranous colitis and hepatotoxicity are important adverse effects.
It is secreted in bile, therefore can be used safely in patients with renal dysfunction.
Thus, D is correct. Ref: KDT, Topic: anti-anaerobe drugs

750. C. Amikacin
Amikacin is most resistant to aminoglycoside inactivating enzymes.
Aminoglycosides are not effective against anaerobes.
Thus, C is correct. Ref: KDT, Topic: aminoglycoside

751. D. Aminoglycosides
Most protein synthesis inhibiting antibiotics are bacteriostatic but aminoglycosides are
bactericidal.
Thus, D is correct. Ref: KDT, Topic: aminoglycoside

752. C. Minocycline
Tetracycline causing maximum vestibular toxicity : Minocycline
Most phototoxic tetracycline : Demeclocycline
Tetracycline causing diabetes insipidus: Demeclocycline
Tetracycline safe in renal failure : Doxycycline
Thus, C is correct. Ref: KDT, Topic: Tetracyclines

753. D. Gentamicin continues to exert antibacterial effects even after plasma


levels decrease below detectable range
Aminoglycosides show concentration dependent killing and prolonged post antibiotic
effect.
Thus, D is correct. Ref: KAT, Topic Aminoglycosides

Time dependent Concentration dependent Time dependent concentration


post antibiotic effect post antibiotic effect enhanced post antibiotic effect

Beta lactams Aminoglycoside Clarithromycin


Vancomycin Daptomycin Clindamycin
Fluroquinolone Erythromycin
Metronidazole Linezolid
Azithromycin Streptogramins
Ketolides Tetracyclines
Tigecycline

754. A. The spectrum of antimicrobial activity of aminoglycosides includes


Bacteroides fragilis
Important points about aminoglycosides are:
These are bactericidal inhibitors of protein synthesis.
Due to formation of ionic molecules in GIT, these are ineffective orally.
Combination of aminoglycosides with penicillins is synergistic and is used for
pseudomonas and enterococcal infections.
Aminoglycosides require O2, for transport in the bacterial cell. These are therefore,
not effective against anaerobic organisms like Bacteroides fragilis.
Thus, A is correct. Ref: KDT, Topic: Aminoglycosides

755. D. They induce formation of hepatic drug metabolizing enzymes


Streptogramins like quinpristin and dalfopristin possess microsomal enzyme inhibitory
property (NOT inducing).
These are effective against MRSA and VRE infections.
Thus, D is correct. Ref: KAT, Topic : MRSA drugs

756. A. It is more active than tetracycline against H. pylori


Doxycycline is a bacteriostatic agent that acts by inhibiting protein synthesis.
It is safe in renal failure as it is excreted mainly in feces.
Doxycycline is useful in Lyme’s disease.
Tetracycline (and not doxycycline) is useful in H. pylori therapy.
Thus, A is correct. Ref: KDT, Topic: Tetracyclines

757. D. Aztreonam
In patient with severe sensitivity to penicillin, all beta lactams except monobactams
are contraindicated. Both aztreonam and imipenem are effective for gram-negative
infections but because imipenem causes seizures as serious adverse effect, aztreonam
is preferred in such a patient.
Thus, D is correct. Ref: KDT, Topic: beta lactams

758. A. Piperacillin
Piperacillin can be combined with beta-lactamase inhibitor, tazobactum.
Vancomycin is NOT effective against pseudomonas.
All (3-lactams except Aztreonam are contra-indicated if severe allergic reaction
develops to any P-lactam antibiotic.
Thus, A is correct. Ref: KDT, Topic: beta lactams

759. A. It is not susceptible to Penicillinases


Vancomycin is a glycopeptide bactericidal antibiotic that is administered by
parenteral route.
It is Penicillinase resistant, thus can be used in MRSA infections.
It is also used for the treatment of pseudomembranous colitis.
Vancomycin is ineffective against pseudomonas.
Thus, A is correct. Ref: KDT, Topic: Glycopeptides

760. B. It acts by inhibiting bacterial protein synthesis


Vancomycin is a glycopeptide that acts by inhibiting bacterial cell wall synthesis. It is
a bactericidal drug (like other cell wall synthesis inhibitors.
It is the drug of choice for MRSA and enterococci resistant to penicillins.
Nephrotoxicity, ototoxicity and red man syndrome are prominent adverse effects of
vancomycin.
Thus, B is correct. Ref: KDT, Topic: glycopeptides

761. C. Methicillin resistant Staph. aureus


Staphylococcus aureus develops resistance to methicillin by acquiring altered
penicillin binding proteins that have low affinity. No B-lactam is effective against
MRSA.
Clavulanic acid is an inhibitor of 13-lactamase. It can restore the sensitivity of
penicillins against the organisms who have developed penicillin.
Thus, C is correct. Ref: KDT, Topic: beta lactams

762. A. They have acquired penicillin binding protein which has low affinity for
β-lactam antibiotics
Resistance to most penicillins is due to elaboration of beta-lactamase.
Methicillin is most resistant penicillin to β-lactamase.
Staphylococcus aureus develops resistance to methicillin’s by acquisition of altered
penicillin binding proteins.
Thus, A is correct. Ref: KDT, Topic: beta lactams

763. D. In renal dysfunction, dosage reductions are necessary to avoid


Imipenem is a broad spectrum 13-lactam antibiotic.
It is used in combination with cilastatin.
No beta lactam antibiotic is effective against MRSA.
In high concentration, imipenem-cilastatin combination can result in seizures.
Thus, D is correct. Ref: KDT, Topic: beta lactams

764. B. Renal tubular reabsorption of beta-lactams is inhibited by probenecid


Probenecid inhibits renal tubular secretion of penicillins (not reabsorption)
Beta lactams eliminated by biliary route are:
Ampicillin
Nafcillin
Ceftriaxone
Cefoperazone
Penicillin G has to be given by i.m. route because it is broken down by gastric acid
(decreases oral bioavailability).
Thus, B is correct. Ref: KDT, Topic: beta lactams

765. B. Gentamicin
Gentamicin is an aminoglycoside and is excreted via renal route.
Thus, B is correct. Ref: KDT, Topic: aminoglycosides

766. D. Penicillin and tetracycline in bacterial meningitis


Combination of a bacteriostatic and bactericidal drug in most cases is antagonistic.
Bactericidal drugs act on fast multiplying organisms whereas bacteriostatic drugs
inhibit the growth. Here, penicillins are bactericidal whereas Tetracyclines are
bacteriostatic.
Thus, D is correct. Ref: KDT, Topic: aminoglycosides

767. A. Post antibiotic effect


Time dependent killing kinetics is shown by aminoglycosides. Here, the killing
activity depends upon the length of time, plasma concentration is above MIC.
Concentration dependent killing is shown by B lactum drugs. Here, killing activity
depends upon the ratio of plasma concentration of MIC.
Post antibiotic effect is the suppression of bacterial growth after limited exposure to
antibiotic.
Thus, A is correct. Ref: KAT, General antimicrobial considerations

768. B. Clindamycin
Clindamycin acts by inhibiting the protein synthesis. Ceftriaxone is beta lactam act by
inhibiting the transpeptidases; Cycloserine a neuropsychiatric toxicity producing
Antitubercular drug act by inhibiting Alanine ligase /racemase; Vancomycin act by
inhibiting d-ala-d-ala bond / transglycosylase inhibition.
Thus, B is correct. Ref KDT, Topic: cell wall inhibitors / general antimicrobial
considerations

769. D. None
SBE prophylaxis for dental procedures:
Prosthetic cardiac valve
Previous history of endocarditis
Congenital heart disease:
Unrepaired cyanotic
Completely repaired with prosthetic material or device for ≤ 6 months’ post
procedure
Cardiac transplant recipients who develop cardiac valvulopathy
This woman likely has mitral valve prolapse with a murmur. Note that all of the valvar
abnormalities have been taken out for needing antibiotic prophylaxis. For adults, the
only reason you’ll need to give antibiotic prophylaxis is for those with prosthetic valves
or who have a history of endocarditis.
Summary of major changes in updated guidelines:
Limit SBE prophylaxis to only those conditions listed above.
Antibiotic prophylaxis is no longer recommended for any other form of congenital
heart disease.
Antibiotic prophylaxis for procedures on respiratory tract or infected skin, etc. only for
conditions listed above.
No antibiotic prophylaxis for GU or GI procedures (regardless of cardiac valve
abnormality)!
Thus, D is correct. Ref KDT Topic: General considerations

770. B. Trimethoprim/sulfamethoxazole
The combination of warfarin and TMP/SMX can be a very deadly one. It is the most
common warfarin interaction leading to hospitalization. Amoxicillin, codeine, and
Cefixime do not interact with warfarin. Azithromycin only rarely causes an increase in
INR in patients on warfarin.
Thus, B is correct. Ref KDT Topic: Sulfonamides
General Microbiology — Questions
1. Microorganism used as weapon in biological terrorism is?
a. Smallpox virus
b. Rabies virus
c. Influenza virus
d. Human parvovirus

2. Disease transmitted by blood transfusion are all except


a. Syphilis
b. AIDS
c. Malaria
d. Histoplasmosis

3. True about sterilization is:


a. Spores are destroyed by drying
b. Soiled dressing is sterilized by hot air oven
c. Surgical instruments are sterilized by boiling
d. Vaccines sterilized by filtration

4. The following statements regarding bacterial spores are correct except


a. Are formed under adverse environmental conditions
b. Are resistant to boiling
c. Contain multiple layers
d. Are formed primarily by the organisms of genus Streptococcus.

5. The chemical nature of capsule may be


a. Polysaccharide
b. Polypeptide
c. Both of the above
d. None of the above

6. Most important surface active disinfectants are


a. Anionic
b. Cationic
c. Non-ionic
d. Amphoteric compounds.

7. The purpose of staining cells on a microscope slide is to:


a. Kill them
b. Secure them to the slide
c. Enlarge the cells
d. Add contrast in order to see them better
e. See motility

8. A microbiologist inoculates Staphylococcus epidermidis and Escherichia coli


into a culture medium. Following incubation, only E. coli grows in culture.
What is the most likely explanation?
a. The microbiologist used too much inoculum
b. The culture is contaminated
c. The incubation temperature was incorrect
d. The culture medium must be selective
e. The culture medium must be differential

9. You are observing a cell through a microscope and note it has no apparent
nucleus. You conclude that it most likely:
a. Has a peptidoglycan cell wall
b. Has a cellulose cell wall
c. Moves by pseudopods
d. Is part of a multicellular animal
e. None of the above

10. Koch’s Postulates are criteria used to establish that:


a. Microbes are found on dust particles
b. A specific microbe is the cause of a specific disease
c. Life forms can only arise from preexisting life forms
d. A specific microbe should be classified in a specific kingdom
e. Microbes can be used to clean up toxic spills
1. a 2. d 3. d 4. d 5. c 6. b 7. d 8. d 9. a 10. b

11. All microorganisms are best defined as organisms that:


a. Cause human disease
b. Lack a nucleus
c. Are infectious particles
d. Are too small to be seen with the unaided eye
e. Can only be found growing in laboratories

12. Which is true for endotoxins?


a. Highly antigenic
b. Found only in Gram-negative bacteria
c. Usually binds to specific cellular receptors
d. Highly toxic and fatal in microgram quantities
e. Toxoids are used to immunize

13. Which subunit of the exotoxin is the binding portion of the molecule?
a. A
b. B
c. C
d. D
e. E

14. Most Gram-negative bacteria have a lipopolysaccharide (LPS) which is


found in the
a. plasma membrane
b. Cell wall
c. Inner membrane
d. Outer membrane
e. Periplasmic space

15. Probiotics exhibit all BUT the following characteristics:


a. Nontoxic and nonpathogenic to host
b. Have beneficial effect on host
c. Mimic antibiotics
d. Remain viable for a long time
e. Withstand acidic nature of host’s stomach

16. The following statement is true


a. Blood cultures should be collected when the patient spikes a fever
b. Blood cultures should be collected after the patient spikes a fever
c. Blood cultures should be collected before the patient spikes a fever
d. Timing of blood culture collection need not coincide with fever spikes

17. Phenol is bactericidal at a concentration of


a. 0.1%
b. 0.25 %
c. 0.5 %
d. 1 %

18. Which of the following is the most ideal disinfectant for preparation of
skin prior to insertion of a central venous catheter
a. Povidone iodine and alcohol
b. 0.4% w/v chlorhexidine
c. 2% w/v chlorhexidine
d. Chlorhexidine and cetrimide

19. Bowie Dick test is used to assess


a. The efficiency of a steam sterilizer
b. The performance of steam sterilizer
c. The efficiency of a hot air oven
d. The efficiency of a hot air oven

20. The spore of the following is used as biological indicator to monitor steam
sterilizer
a. Geobacillus stearothermophilus
b. Bacillus subtilis subsp niger
c. Bacillus cereus
d. Bacillus anthrax

21. The following phenomenon is responsible for antibiotic resistance in


bacteria due to slime production
a. Enzymatic degradation
b. Biofilm formation
c. Mutation resulting in altered target site for antibiotics

22. Universal/Standard precaution sho-uld be applied to all except


a. Serum
b. Semen
c. Synovial fluid
d. Sputum
e. None of the above

23. The first thing to do in case of accidental occupational exposure to blood


and body fluids
a. Wash with plenty of soap and running water
b. Put the affected area in your mouth
c. Squeeze blood from the area
d. Apply bleach or 1% Sodium hypochlorite on the affected area
11. d 12. b 13. b 14. d 15. d 16. d 17. d 18. c 19. b 20. a

21. d 22. e 23. a

24. How should endoscopes be disinfected


a. Manual cleaning, disinfecting, rinsing
b. Manual cleaning and autoclaving
c. 70% alcohol and rinsing
d. Sonication and rinsing

25. In a post operative care unit, five patients developed post operative
wound infection on the same wound. The best method to prevent cross
infection occurring in other patients in the same ward is to
a. Give antibiotics to all other patients in the ward
b. Fumigate the ward
c. Disinfect the ward with sodium hypochlorite
d. Practice proper hand washing

26. Endoscopes have been known to be contaminated with the following


organism due to poor quality of rinse water
a. Mycobacterium chelonae
b. Mycobacterium tuberculosis
c. Mycobacterium xenopi
d. Mycobacterium bovis

27. Peptidoglycan synthesis occurs in 4 phases. Cross linking of polypeptide


chains occurs in:
a. Phase 1
b. Phase 2
c. Phase 3
d. Phase 4

28. What is the structure that is found in gram negative but not in gram
positive bacteria
a. Cell wall
b. Cytoplasmic membrane
c. Flagella
d. Outer membrane

29. Exotoxins are


a. Lipopolysaccharide in Nature
b. Produced by Gram –ve bacilli
c. Highly antigenic
d. Very stable & resistant to chemicals

30. A soldier developed thigh abscess following gunshot injuries during an anti
insurgency combing operation. Gram stain of the foul smelling exudates
revealed plenty of pus cells and a mixed flora of Gram positive and Gram
negative rods. However aerobic culture was sterile. Which of the following
enzymes is most likely involved in the following reaction: 2O2- + 2H+ = H2O2
+ O2
a. Catalase
b. Oxygen permease
c. Peroxidase
d. Superoxide dismutase

31. On GASPAK two types of bacteria was isolated. Biochemical studies


revealed that these bacteria failed to produce superoxide dismutase,
catalase and peroxidase. All are incorrect except
a. Aminoglycoside antibiotics is the drug of choice
b. Less virulent than the ones containing all the three enzymes
c. The enzymes are expressed in the reduced state
d. They have enzymes only for the EMP pathway

32. All are true regarding disinfectants except


a. Glutaraldehyde is sporicidal
b. Hypochlorites are virucidal
c. Ethylene glycol is intermediate disinfectant
d. Phenol usually requires organic matter to act
33. Endoscopes have been known to be contaminated with the following
organism due to poor quality of rinse water
a. Mycobacterium chelonae
b. Mycobacterium tuberculosis
c. Mycobacterium xenopi
d. Mycobacterium bovis

34. Which of the following is used for biological monitoring of gas plasma
sterilizer
a. Bacillus subtilis subspecies globigi
b. Bacillus pumilus
c. Bacillus subtilis subspecies niger
d. Bacillus stearothermophilus
24. a 25. d 26. a 27. d 28. d 29. c 30. d 31. d 32. d 33. a

34. d

35. The best way to prevent contamination of the PCR process is


a. Use of biosafety cabinet
b. Internal sterilizing agent such as uracil N-glycosyls (UNG)
c. Use universal precautions
d. Clean the area with 1% sodium hypochlorite

36. The best molecular typing method is


a. PFGE
b. RFLP
c. Ribotyping
d. Spoligotyping

37. All except one are contributions of Robert Koch to the field of
microbiology.
a. Germ theory of disease.
b. Discovery of Vibrio cholerae.
c. Discovery of staining technique.
d. Discovery of staining of bacteria.
e. Discovery of solid media.

38. Paul Ehrlich is known for its contribution to microbiology in the field of
a. Discovery of prontosil as antibacterial.
b. Discovery of gene splicing
c. Theories on immunity.
d. Theories of antibody production.
e. Theories of DNA synthesis

39. Peptidoglycan synthesis occurs in 4 phases. Cross linking of polypeptide


chains occurs in:
a. Phase 1
b. Phase 2
c. Phase 3
d. Phase 4
e. End of phase 3

40. Sedimentation coefficient of bacterial ribosomes is:


a. 70S
b. 60S
c. 80S
d. 90S
e. 100S

41. Biocides are substances that


a. Kill only pathogenic microorganisms
b. Kill all living microorganisms excluding spores
c. Kills all living microorganisms including spores
d. Prevent growth of pathogenic bacteria
e. Prevent growth of nonpathogenic bacteria

42. Chemical agents that destroy the microorganisms by interfering with


membrane functions are:
a. Formic acid
b. Hydrogen peroxide
c. Formaldehyde
d. Phenol
e. Ethylene oxide

43. Not true about drug resistance mechanism?


a. Most common mechanism is production of neutralizing substances
b. If resistance is plasmid mediated, it is always transferred vertically
c. Alteration of target in pneumococcal resistance
d. Complete removal of target is cause of resistance to vancomycin

44. All of the following are required in PCR except


a. Deoxyribonucleotides
b. Thermostable enzyme/DNA polymerase
c. Dideoxy ribonucleotides
d. Magnesium/ssdna/Template DNA

45. Penicillin is extracted from:


a. Penicillium chrysogenum
b. Penicillium notatum
c. Aspergillus niger
d. Yeast
46. Which of the following statements are true regarding flagella? (PGI)
a. It is made up of smaller units called flagellin.
b. Bacteria with polar flagella move very fast
c. Antibodies to flagella are protective.
d. Basal body is anchored in cytoplasm.
e. All flagella essentially have same chemical composition.
35. b 36. a 37. a 38. c 39. d 40. a 41. c 42. d 43. b 44. c

45. a 46.
a,b,e

47. Lysogenic conversion is seen in the following (PGI)


a. C. diphtheriae
b. Salmonella
c. Staphylococcus
d. Vibrio
e. Clostridium botulinum.

48. Which of the following are examples of basic dyes used to stain bacteria:
(PGI)
a. Crystal violet
b. Methylene blue
c. Nigrosin
d. Safranin
e. Malachite green

49. Molecular technique used to study expression of gene is


a. Polymerase Chain Reaction
b. Real time PCR
c. Ligase chain reaction
d. Microarray using gene CHIP
e. Gene splicing

50. Following statements are true regarding real time PCR (PGI)
a. Uses Fluorescent probe for detection of amplified product
b. Post amplification manipulation is required.
c. Cycle is very long
d. Risk of release of amplified product into environment less
e. Quantification cannot be done

51. Mycin is: (PGI)


a. Bactericidal antimicrobial
b. Effective against Gram positive bacteria
c. Effective against Gram negative bacteria
d. Useful in treatment of pneumonia.
e. Active even during stationary phase of growth cycle

52. Which of the following is/are examples of live vaccines?


a. Japanese encephalitis.
b. Influenza virus
c. Oral typhoid vaccine.
d. Acellular pertusis vaccine
e. BCG vaccine

53. Following statements are true regarding external quality assurance (EQA):
(PGI)
a. Establish intra laboratory performance
b. Ensure reliability of future testing.
c. Ensure credibility of the laboratory
d. Retrospective periodic assessment
e. Establish interlaboratory comparability

54. Ortho-phthaldehyde has following advantages over glutaraldehyde: (PGI)


a. Nonirritant
b. Does not stain skin
c. Odourless
d. Poor material compatability
e. No advantage over glutaraldehyde

55. Which of the following statement is true?


a. Solid media are enrichment media
b. Nutrient broth is basal media
c. Agar adds nutrient to media
d. Chocolate agar is selective media

56. Which of the following is most resistant to antiseptics?


a. Spore
b. Prion
c. Cyst
d. Fungus

57. A chest physician performs bronchoscopy in the procedure room of the


outpatient department. To make the instrument safe for use in the next
patient waiting outside, the most appropriate method to disinfect the
endoscope is by:
a. 70% alcohol for 5 min
b. 2% glutaraldehyde for 20 min
c. 2% formaldehyde for 10 min
d. 1% sodium hypochlorite for 15 min

58. Heat labile instruments for use in surgical procedure can be best sterilized
by:
a. Absolute alcohol
b. 2% glutaraldehyde for 2 hrs
c. Chlorine releasing compounds
d. Ethylene oxide gas
47. 48. a, 49. d 50. a,d 51. 52. c 53. 54. a,c 55. b
a,b,e b,d,e a,b,c b,c,d,e

56. b 57. b 58. d

59. All are true regarding disinfectants except: (PGI)


a. Glutaraldehyde is sporicidal
b. Hypochlorites are virucidal
c. Ethylene oxide is intermediate disinfectant
d. Phenol usually requires organic matter to act

60. The operating temperature in an ethylene oxide sterilization during warm


cycle is:
a. 20-35°C
b. 49-63°C
c. 68-88°C
d. 92-110°C

61. The sterilization method for catgut suture:


a. Steam
b. Radiation
c. Boiling
d. Burning

62. Choose the correct ones for the decreasing order of resistance to
sterilization:
a. Prions, bacterial spores, bacteria
b. Bacterial spores, bacteria, Prions
c. Bacteria, Prions, bacterial spores
d. Prions, Bacteria, bacterial spores
e. Bacterial spores, prions, bacteria

63. Sterilizing agents include: (PGI)


a. Dry heat
b. Ethylene oxide
c. Ether
d. Alcohol
e. Chlorohexidine

64. Which of the following can be reliably used for hand washing: (PGI)
a. Chlorhexidine
b. Isopropyl alcohol
c. Lysol
d. Cresol
e. Glutaraldehyde

65. Sporicidal agents are: (PGI)


a. Glutaraldehyde
b. Ethylene oxide
c. Formaldehyde
d. Ether

66. Radiation can be used to sterilize all except:


a. Bone graft
b. Suture
c. Bronchoscope
d. Artificial tissue graft

67. Sputum disinfection is done by all except:


a. Autoclaving
b. Cresol
c. Boiling
d. Chlorhexidine

68. Which of the following is most resistant to the action of antiseptics and
disinfectants:
a. Spores
b. Coccidia
c. Prions
d. Mycobacteria

69. The indicator used in autoclave is:


a. Clostridium tetani
b. Geobacillus stearothermophilus
c. Bacillus pumilis
d. Bacillus subtilis var niger

70. Which is used in digestion and decontamination of sputum in smear


preparation:
a. NaOH
b. KOH
c. NaCI
d. KCl

71. Which of the following are used for sterilization of surgical instrument:
a. Ethylene oxide
b. Gamma radiation
c. Autoclaving
d. Glutaraldehyde
e. Hot air oven
f. All the above

72. The disposable plastic syringes are best sterilized by:


a. Formaldehyde
b. Ethylene oxide
c. Hexachloride
d. UV radiation

73. Autoclaving is done in :


a. Dry air at 121 °C and 15 lb pressure
b. Steam at 100°C for 30 minutes
c. Steam at 121 °C for 15 minutes, 15 lb pressure
d. Dry air at 160°C for 30 minutes

74. Culture media are sterilized by :


a. Autoclaving
b. Radiation
c. Hot air oven
d. Tyndallisation

75. Glassware and syringes are sterilized by:


a. Autoclave
b. Hot air oven
c. Ethylene oxide
d. Irradiation

59. 60. b 61. b 62. a 63. 64. 65. 66. c 67. d 68. c
c,d a,b a,b a,b,c

69. b 70. a 71. f 72. b 73. c 74. a 75. b

76. Asepsis means:


a. Absence of pathogenic microbes
b. Disinfection of the surface
c. Prevention of infection
d. Destroying all forms of microorganism

77. MacConkey's Agar medium is:


a. Enriched medium
b. Enrichment medium
c. Differential medium
d. Synthetic medium

78. Gamma radiations are used for sterilizing:


a. Plastic Syringes
b. Cystoscopes
c. Dressing aprons
d. Metal instruments

79. The process of destroying all microbes including spores is called:


a. Disinfection
b. Antisepsis
c. Disinfestation
d. Sterilization

80. Out of the following the true statement regarding sterilization is:
a. Dry heat is the best method of sterilization of liquid paraffin
b. All glass syringes are best sterilized by boiling at 100°C
c. Bacterial vaccines are best sterilized by ethylene oxide
d. Pasteurization of milk by flash method is done by heating at 63°C for 30 minutes

81. Blood agar is an example of


a. Enriched media
b. Enrichment media
c. Nutrient media
d. Selective media

82. Conditions required for autoclave is


a. 121 °C temperature for 15 min
b. 121 °C temperature for 45 min
c. 100°C temperature for 60 min
d. 100°C temperature for 90 min

83. Sterilization of culture media containing serum is by:


a. Autoclaving
b. Micropore filter
c. Gamma radiation
d. Inspissation

84. Sterilization of fiberoptic bronchoscope is done by?


a. Glutaraldehyde
b. Chlorine
c. Autoclave
d. Phenol

85. Glassware is sterilized by?


a. Boiling
b. Hot air oven
c. Heating
d. None
86. The disposable plastic syringes are commonly sterilized by:
a. Formaldehyde
b. Ethylene oxide
c. Hexachloride
d. UV radiation

87. Which is enrichment media:


a. Selenite F broth
b. Chocolate media
c. Meat extract media
d. Egg media

88. In nutrient agar concentration of agar is:


a. 1%
b. 2%
c. 3%
d. 4%

89. Blood agar is an example of: (PGI)


a. Enrichment media
b. Indicator media
c. Enriched media
d. Selective media

90. Robert Koch's assistant advised him to use agar instead of gelatin as
culture media for cultivation of bacteria as:
a. Agar has more nutrition
b. Gelatin melts at 27°C
c. Gelatin is not easily available
d. Agar is cheaper

91. Endoscope tube is sterilized by:


a. Glutaraldehyde
b. Formalin
c. Autoclaving
d. Boiling

76. a 77. c 78. a 79. d 80. a 81. a 82. a 83. d 84. a 85. b

86. b 87. a 88. b 89. 90. b 91. a


b,c

92. Browne’s tube is used for:


a. Steam sterilization
b. Radiation
c. Chemical sterilization
d. Filtration
93. Prions are best killed by:
a. Autoclaving at 121°C
b. 5% formalin
c. Autoclave + Sodium hydroxide
d. Sodium hypochlorite

94. Virus mediated transfer of host DNA from one cell to another is known as:
a. Transduction
b. Transformation
c. Transcription
d. Integration

95. Bacteria may acquire characteristics by all of the following except:


a. Taking up soluble DNA fragments across their cell wall from other species
b. Incorporating part of host DNA
c. Through bacteriophages
d. Through conjugation

96. True about Lambda phage is:


a. Cause mad cow disease
b. Lytic and lysogenic interconversion can’t occur
c. Lytic form incorporated within host DNA and multiply causing rupture of cell
membrane
d. Lysogenic form incorporates with host DNA and remains dormant

97. Intraspecies competition is the competition among:


a. Species
b. Individuals of a population
c. Individuals of a community
d. Populations and their regulatory factors

98. The following phenomenon is responsible for antibiotic resistance in


bacteria due to slime production:
a. Co-aggregation
b. Biofilm formation
c. Mutation evolving in altered target site for antibiotics
d. Mutation evolving a target bypass mechanism

99. Shine-Dalgarno sequence in bacterial mRNA is near:


a. AUG codon
b. UAA codon
c. UAG codon
d. UGA codon

100. Incorrect about plasmid is:


a. Involved in multidrug resistance transfer
b. Involved in Transformation
c. Imparts capsule formation
d. Imparts pili formation

101. True about bacteriophage is:


a. Can transmit toxin to bacteria
b. Bacteria which transmits DNA to another bacterium
c. Causes transformation of bacteria
d. Is a virus which invades bacteria

102. Which of the following contains both DNA and RNA?


a. Plasmids
b. Bacteria
c. Prions
d. Viroids

103. Horizontal transmission of ‘R’ factor is by:


a. Transduction
b. Transformation
c. Conjugation
d. Fusion

104. Polymerase chain reaction was discovered by


a. Saiki
b. Salk
c. Watson and Crick
d. Kary B Mullis

105. F plasmid of high frequency recombination is:


a. Extrachromosomal
b. Chromosomal
c. Mesosome
d. Ribosomes

92. a 93. c 94. a 95. b 96. d 97. b 98. b 99. a 100. b 101. d

102. 103. c 104. 105.


b d a

106. False regarding bacterial plasmids is?


a. Extrachromosomal
b. Eliminated by treating with radiation
c. Transmission of different characters
d. Can cause lysogenic conversion

107. Drug resistance transfer by bacteriophage involves:


a. Transduction
b. Conjugation
c. Transformation
d. Convocation

108. True about transferable resistance:


a. High degree of resistance
b. Involves resistance to multiple drugs
c. Plasmids play a role
d. All the above

109. Phage typing is useful as an epidemiological tool in all, except:


a. Salmonella
b. Staphylococcus aureus
c. V. cholerae
d. Shigella dysenteriae

110. Drug resistance is not transmitted by:


a. HFr
b. Transposons
c. Plasmids
d. Chromosomes

111. Following transmit drug resistance except:


a. RTF
b. Plasmids
c. HFr
d. Chromosome

112. All are true regarding Plasmid except :


a. Involved in multidrug resistance transfer
b. Involved in Transformation
c. Imparts capsule formation
d. Imparts pili formation

113. Conjugation does not involve:


a. Bacteriophages
b. HFr
c. Fr
d. Plasmids

114. Jumping gene is:


a. Transposon
b. Episome
c. Cosmid
d. Plasmid
115. Microbial killing below MIC is known as:
a. Adverse effect
b. Post antibiotic effect
c. Both
d. None

116. Mechanism of direct transfer of free DNA:


a. Transformation
b. Conjugation
c. Transduction
d. None

117. Transfer of some Extra chromosomal material from one bacterium to


other can occur via:
a. F factor
b. R factor
c. C factor
d. All of the above

118. Endotoxin from gram negative organism is:


a. Polysaccharide
b. Glycoprotein
c. Lipoprotein
d. Lipopolysaccharide

119. The term “viable not cultivable” (VNC) is used for: (PGI)
a. M. leprae
b. M. tuberculosis
c. Treponema pallidum
d. Salmonella
e. Staphylococcus

106. 107. 108. 109. 110. 111. c 112. 113. a 114. a 115. b
d a d d d b

116. 117. 118. 119.


a d d a,c
General Microbiology — Explanations
1. A. Smallpox virus
All others also belong to bioterrorism agents of different categories.
Among the choices, potential weapon for biological terrorism is Smallpox virus.

2. D. Histoplasmosis
Histoplasmosis spreads by inhalation of spores.
Other choices are transmitted by blood transfusion.

3. D. Vaccines are sterilized by filtration


Spores are best destroyed by autoclaving and Eto
Linens, media, reagents are best sterilized by autoclaving

4. D. Are formed primarily by the organisms of genus Streptococcus.


Only spore forming human pathogenic bacteria are Bacillus and Clostridia.
Streptococcus is a non spore forming bacterium.
All other choices are correct regarding bacterial endospores.

5. C. Both of the above


Bacillus anthracis possesses polypeptide capsule.
S. pneumoniae, H. influenzae, N. meningitides and Cryptococcus neoformans possess
polysaccharide capsule.

6. B. Cationic
Most important surface active agent is quaternary ammonium compound (QAC) –
cationic agent.
Anionic – soaps are not as effective as QAC.

7. D. add contrast in order to see them better


Main purpose of staining is to add contrast in order to see them better.
Fixation mainly kills and secures then to slides.

8. D. the culture medium must be selective


The medium may be a selective medium for E. coli.
So it inhibits Staphylococcus epidermidis

9. A. has a peptidoglycan cell wall


If no nucleus, it may be a prokaryote (Bacteria)
Bacterial cell wall is made up of Peptidoglycan.
Pseudopod – Protozoa

10. B. a specific microbe is the cause of a specific disease


Koch postulates establishes the relationship between the microbe and the specific
disease caused by that microbe.
Germ cell theory establishes that life forms can only arise from preexisting life forms.

11. D. Are too small to be seen with the unaided eye


The Definition of microorganism is too small organisms that we cannot seen with
unaided / naked eye.
They are visualized with the help of microscope.

12. B. Found only in Gram-negative bacteria


Endotoxins are the outer part of the lipopolysaccharide.
Lipopolysaccharide is present only in Gram negative bacterial cell wall.
All other choices are the features of Exotoxins.

13. B. B
Some Exotoxins are having A and B subunits.
B subunit for binding with cell surface and push the A in to the cell.
A is having enzymatic action.

14. D. Outer membrane


Outer membrane of gram negative bacteria has Lipopolysaccharide (LPS).
LPS Contains Lipid A, Core polysaccharide, and O side chain or somatic antigen.

15. D. Remain viable for a long time


Probiotics are live microorganisms intended to provide health benefits when
consumed, generally by improving or restoring the gut flora.
Probiotics are considered generally safe to consume, but may cause bacteria-host
interactions and unwanted side effects in rare cases.

16. D. Timing of blood culture collection need not coincide with fever spikes

17. D. 1%
Phenol (carbolic acid) is one amongst the oldest antibacterial agents. It acts by
inhibiting biological process of bacteria at concentration of 1% and is fungicidal in
action at concentrations of 1%–2%.

18. C. 2% w/v chlorhexidine


As per CDC guidelines, the best disinfectant for preparation of skin prior to insertion
of a central venous catheter is 2% Chlorhexidine

19. B. The performance of steam sterilizer


Bowie Dick Test is a diagnostic visual test performed to ensure the dynamic-air-
removal system (Vacuum system) of a steam sterilizer. It effective in removing any air
that may be trapped or leaked into the sterilization chamber

20. A. Geobacillus stearothermophilus


The biological indicator in autoclave is Geobacillus stearothermophilus.
Bacillus pumilus for radiation
Bacillus subtilis (currently B. atropheus) for Hot Air oven

21. B. Biofilm formation


Mechanisms of antibiotic resistance exhibited by bacterial biofilms.
Bacteria that attach to a surface and grow as a biofilm are protected from killing by
antibiotics.

22. E. None of the above


Initially the universal / standard work precautions were applicable for blood and body
fluids.
But currently sputum also included.

23. A. Wash with plenty of soap and running water


Wash needlesticks and cuts with soap and water.
Flush splashes to the nose, mouth, or skin with water.
Irrigate eyes with clean water, saline, or sterile irrigants.
Report the incident to your supervisor.
Immediately seek medical treatment.
Do not squeeze/ put affected area in mouth or apply irritants like hypochlorite.

24. A. Manual cleaning, disinfecting, rinsing


Manual cleaning with 2% glutaraldehyde.
Other choices are not suitable for endoscopes.

25. d. Practice proper hand washing


The best method to prevent post operative wound infection due to cross infection in
the same ward is implementation of proper hand hygiene.

26. A. Mycobacterium chelonae


Many studies revealed that M. chelonae is the common agent involved in
contamination of endoscopes which are rinsed in poor quality tap water.

27. D. Phase 4
Peptidoglycan, also known as murein, is a polymer consisting of sugars and amino
acids that forms a mesh-like layer outside the plasma membrane of most bacteria,
forming the cell wall.
The sugar component consists of alternating residues of β-(1,4) linked N-
acetylglucosamine (NAG) and N-acetylmuramic acid (NAM) .
Attached to the N-acetylmuramic acid is a peptide chain of three to five amino acids.
The peptide chain can be cross-linked to the peptide chain of another strand forming
the 3D mesh-like layer. Peptidoglycan serves a structural role in the bacterial cell wall,
giving structural strength, as well as counteracting the osmotic pressure of the
cytoplasm.
Peptidoglycan is also involved in binary fission during bacterial cell reproduction.

28. D. Outer membrane


Outer membrane, LPS, Periplasmic space, Porins are exclusively present in Gram
negative bacteria.

29. C. Highly antigenic


Exotoxins are produced by both Gram positive and Gram Negative
Highly antigenic
Proteins
Very specific
Toxoidable
Heat labile

30. D. Superoxide dismutase


Superoxide dismutase, originally known as erythrocuprein, catalyzes the reaction of
two molecules of superoxide (O2.-) to yield one molecule of hydrogen peroxide
(H2O2) and one molecule of oxygen (O2).
Diffusion of hydrogen peroxide, the product of SOD1, across the cytosol is limited.

31. D. They have enzymes only for the EMP pathway


An important route of glucose conversion to pyruvate such as glycolysis is Embden-
Myerhof pathway.
Glycolysis is the most important type of mechanism by which organisms obtain
energy from organic compounds in absence of molecular oxygen. As it occurs in the
absence of oxygen, therefore, it is also called anaerobic fermentation.

32. D. Phenol usually requires organic matter to act


Phenol acts in the presence of organic matter. It doesn’t mean that organic matter is
needed for the action of phenol.

33. A. Mycobacterium chelonae


Refer Q. No : 26

34. D. Bacillus stearothermophilus


Bacillus stearothermophilus > B. atropheus Sub spp niger
B. pumilus - radition

35. B. Internal sterilizing agent such as uracil N-glycosyls (UNG)


The most common strategy to prevent carry-over contamination is to partially or
completely replace dTTP with dUTP during PCR amplification, thereby producing DNA
containing uracil.
Prior to initiating PCR, the PCR mixture is treated with Uracil-DNA Glycosylase (UNG).
During the initial denaturation step temperature is elevated to 95°C, resulting in
cleavage of apyrimidinic sites and fragmentation of carry-over DNA.
As the template contains thymidine, it will not be affected by the UNG treatment. It is
a prerequisite that all PCRs are carried out with dUTP substituting dTTP.

36. A. PFGE
PFGE is gold standard genotyping tool
Other genotypic typing methods are:
Chromosomal DNA analysis.
Plasmid profile analysis.
RFLP (Restricted fragment length polymorphism).
Ribotyping (RFLP analysis of ribosomal DNA).

37. A. Germ theory of disease


The germ theory was proposed by Girolamo Fracastoro in 1546.

38. C. Theories on immunity.


Prontosil – Domagk
Jerne’s theory of antibody production using the concept of clonal selection.

39. D. Phase 4
Refer Q. No 27

40. A. 70S
Bacterial ribosome -70S
Made up of 30S + 50S sub units
Target for many antibiotics acting on protein synthesis

41. C. Kills all living microorganisms including spores


Cidal – to kill
Static agents – To prevent the growth
Biocide – to kill all the microbes including spores
Virucidal – to kill all the viruses.

42. D. Phenol
Phenolic agents act by interrupting the cell membrane functions.
Ideal concentration is 5%.
Ethylene oxide and hydrogen peroxide acts by DNA damage.
Aldehydes are strong alkylating agents.

43. B. If resistance is plasmid mediated, it is always transferred vertically.


Plasmid mediated gene transfer – Conjugation
Conjugation is horizontal gene transfer method.

44. C. Dideoxy ribonucleotides


The important things needed for PCR are
1. DNA template,
2. DNA polymerase enzyme,
3. Primers,
4. Nucleotides
5. Reaction buffer

45. A. Penicillium chrysogenum & B. Penicillium notatum


Penicillium chrysogenum or P. notatum (formerly) is a species of fungus in the genus
Penicillium.

Penicillium chrysogenum Penicillin

Penicillium Griseofulvum Griseofulvin


Acremonium spp Cephalothin

46. A. It is made up of smaller units called flagellin, B. Bacteria with polar


flagella move very fast & E. All flagella essentially have same chemical
composition.
Organ of locomotion.
Antibodies to flagella are non productive
Contains tail, hook and basal body

47. A. C. diphtheriae, B. Salmonella & E. Clostridium botulinum.


Lysogenic conversion seen in the following organisms
Cholera toxin
Salmonella O antigens
Clostridium botulinum toxin
Exotoxins A–C (erythrogenic or pyogenic) of Streptococcus pyogenes
Corynebacterium diphtheria toxin
Shiga toxin

48. A. Crystal violet, B. Methylene blue, D. Safranin & E. Malachite green


Basic Dyes: This dye have positive charge & bind to negatively charged
molecules(nucleic acid, -COOH -OH).
Since, surface of bacterial cells are negatively charged (due to Teichoic acid), basic
dyes are most commonly used in bacteriology.
Examples: Crystal Violet, Methylene Blue, Safranin , basic fuschin

49. D. Microarray using gene CHIP


A microarray is a laboratory tool used to detect the expression of thousands of genes
at the same time. DNA microarrays are microscope slides that are printed with
thousands of tiny spots in defined positions, with each spot containing a known DNA
sequence or gene.

50. A. Uses Fluorescent probe for detection of amplified product & D. Risk of
release of amplified product into environment less
Real-time PCR (RT-PCR) is also called quantitative PCR or qPCR.
The key feature in RT-PCR is that amplification of DNA is detected in real time as PCR
is in progress by the use of fluorescent reporter.
The fluorescent reporter signal strength is directly proportional to the number of
amplified DNA molecules.

51. A. Bactericidal antimicrobial, B. Effective against Gram positive bacteria &


C. Effective against Gram negative bacteria
They are poorly absorbed from the gastrointestinal tract and cannot be given orally.
They penetrate the spinal fluid poorly and not suitable for treating meningitis

52. C. Oral typhoid vaccine & E. BCG vaccine


JE vaccine - Killed vaccine – Nakayama strain & Atten Live vaccine strain -14-14-2
Influenza – live attenuated – Nasal spray, not in routine use.

53. B. Ensure reliability of future testing, C. Ensure credibility of the


laboratory, D. Retrospective periodic assessment & E. Establish
interlaboratory comparability.
External quality assessment (EQA) is the challenge of the effectiveness of a
laboratories quality management system.
EQA typically refers specifically to the challenge of quality systems in a medical
laboratory.
The term external refers to the fact that an organizer outside of the laboratories
organisation provides a statement of quality to the laboratory.

54. A. Nonirritant & C. Odourless


OPA has the advantage that it a solid compared to liquid glutaraldehyde, though both
are normally supplied in solution, and so a OPA has lower vapor pressure and much
less smell than the pungent glutaraldehyde.
In addition, OPA is more effective and so can be used at a lower concentration.

55. B. Nutrient broth is basal media


Enrichment media are liquid media.
Agar has no nutritive value.
Chocolate agar is an enriched medium.

56. B. Prion

57. B. 2% glutaraldehyde for 20 min


Ideal method for disinfecting heat labile equipment like endoscopes are 2%
glutaraldehyde for minimum 20 minutes contact period.
Sodium hypochlorite – Spills management.
Formaldehyde – Fumigation.
Ethyl alcohol – Stethoscope.

58. D. Ethylene oxide gas


Eto is sterilant. Any surgical equipment contact with blood or tissue is comes under
critical. It need to be sterilized.
In case of heat labile items, Eto is the ideal method.

59. C. Ethylene oxide is intermediate disinfectant & D. Phenol usually requires


organic matter to act
Eto is sterilant.
Phenol acts in the presence of organic matter. It doesn’t mean that organic matter is
needed for the action of phenol.
60. B. 49-63°C
The warm cycle of Eto is 30° - 65°C for 8 – 12 hours

61. B. Radiation
The ideal method for heat labile items like catgut, disposable plastics is ionizing
radiation.

62. A. Prions, bacterial spores, bacteria


Refer Q. no : 56

63. A. Dry heat & B. Ethylene oxide


Autoclave, Eto, are the other examples of sterilization methods.

64. A. Chlorhexidine & B. Isopropyl alcohol


These two agents are the reliable agents for hand washing.
Anionic agents like detergents and soaps are also useful agents in hand washing

65. A. Glutaraldehyde, B. Ethylene oxide & C. Formaldehyde


Eto is a very effective sporicidal agent.
Aldehydes in activated stage, they exhibits sporicidal activity.

66. C. Bronchoscope
Bronchoscopes are best disinfected by using 2% glutaraldehyde.

67. D. Chlorhexidine
Sputum is disinfected by using 5% phenol, Cresol, Autoclaving, Boiling and burning.

68. C. Prions
Refer Q. no : 56

69. B. Bacillus stearothermophilus


The biological indicator in Autoclave is Geobacillus stearothermophilus.
Bacillus pumilus for radiation
Bacillus subtilis (currently B. atropheus) for Hot Air oven

70. A. NaOH
Sodium hydroxide is used as a digestive as well as decontaminating agent in sputum
sample processing for smear preparation and culture. NALC -NaOH method is
currently employed in many laboratories.

71. F. All of the above


Hence surgical equipment is a broader term which includes scalpel blades, suture
materials and many other items.
All the choices are used for different type of surgical equipments

72. B. Ethylene oxide


Disposable plastic syringes are best sterilized by ionizing radiation.
Next ideal method is Eto

73. C. Steam at 121 °C for 15 minutes, 15 lb pressure


Steam at 100°C for 30 minutes – Koch – Arnold steamer
Dry Heat at 160°C for 120 minutes – Hot Air oven

74. A. Autoclaving
Hot air oven – Oils, glasswares, liquid paraffin, metals
Tyndallization other names are intermittent sterilization or fractional sterilization.

75. B. Hot air oven


Glass wares and Glass syringes are sterilized by Hot air oven.
Plastic syringes – Gamma radiation
Heart lung machine – Eto

76. A. Absence of pathogenic microbes


Destroying all forms of microorganisms and spores – Sterilization.
Inhibits multiplication of pathogens on inanimate – Disinfection.
Inhibits multiplication of pathogens in tissues - Antispesis.

77. C. Differential medium


MacConkey agar differentiates Lactose fermenters from Non lactose fermenters
It acts as selective medium, solid medium and Indicator medium

78. A. Plastic Syringes


Cystoscopes - 2% cidex
Dressing aprons – Autoclaving
Metal instruments – Hot air oven / Autoclave

79. D. Sterilization
Refer Q. no 76

80. A. Dry heat is the best method of sterilization of liquid paraffin


Glass syringes – Hot air oven
Bacterial vaccines – Seitz filter
Pasteurization by flash method – 72°C – 15 -20 seconds followed by rapid cooling to
13°C

81. A. Enriched Media


Enrichment media – Selenite F broth.
Nutrient media – Nutrient agar.
Selective media – TCBS agar, PLET medium etc.

82. A. 121 °C temperature for 15 min


Refer Q. no 73

83. D. Inspissation
Heating at 80–85ºC for 30 min for 3 consecutive days.
Used for protein rich media like LJ and Dorset’s egg medium & serum based Loeffler’s
serum slope.

84. A. Glutaraldehyde
Less toxic, irritant & corrosive.
Ideal for endoscopes and cystoscopes:
2% concentration for 20 min.

85. B. Hot air oven


Refer Q. no 75

86. B. Ethylene oxide


Refer Q. no 72

87. A. Selenite F broth


Enrichment Medium: Liquid medium (broth) that allows the growth of certain
organisms and inhibits other organisms.
Selenite F & Tetrathionate broth, Alkaline peptone water.

88. B. 2%
In semisolid agar, the agar concentration is 0.5%
In solid medium like nutrients agar, agar concentration is 1.5% to 2%

89. B. Indicator media & C. Enriched media


Blood agar is an example for Enriched, Solid, Differential and Indicator medium

90. B. Gelatin melts at 27°C


Agar is used as a solidifying agent in culture media.
It has no nutritive value.
The advantage over gelatine are, it is bacteriologically inert, and it melts at 98°C and
usually solidifies at 42°C.

91. A. Glutaraldehyde
Refer Q. no 57

92. A. Steam sterilization


Brownie's tube used as a sterilization control for Hot air oven and autoclave.
Radiation – Bacillus pumilus
Filtration – Serratia marcescens

93. C. Autoclave + Sodium hydroxide


Prions are destroyed by.
Autoclaving at 134°C for 1–1.5 hour.
Treatment with 1 N NaOH for 1 hour.

94. A. Transduction
Transduction is the transfer of cell DNA by means of a bacterial virus (bacteriophage,
phage).
During the growth of the virus within the cell, a piece of bacterial DNA is incorporated
into the virus particle and is carried into the recipient cell at the time of infection.

95. B. Incorporating part of host DNA


The transfer of genetic information from one cell to another can occur by three
methods: conjugation, transduction, and transformation.
Not by integrating with host DNA.

96. D. Lysogenic form incorporates with host DNA and remains dormant
During the temperate or lysogenic life cycle, the lambda phage DNA remains
integrated with the bacterial chromosome as prophage.

97. B. Individuals of a population


Intraspecific competition is an interaction in population ecology, whereby members
of the same species compete for limited resources.
This leads to a reduction in fitness for both individuals. By contrast,
interspecific competition occurs when members of different species compete
for a shared resource.

98. B. Biofilm formation


Refer Q. No 21

99. A. AUG codon


The Shine-Dalgarno (SD) Sequence is a ribosomal binding site in bacterial and
archaeal messenger RNA, generally located around 8 bases upstream of the start
codon AUG.
The RNA sequence helps recruit the ribosome to the messenger RNA (mRNA) to
initiate protein synthesis by aligning the ribosome with the start codon.

100. B. Involved in Transformation


Plasmid involved in conjugation.
Responsible for multiple drug resistance, Capsule and pili formation.

101. D. Is a virus which invades bacteria


It can transmit toxin gene not toxin.
It is a virus.
It involved in transformation.

102. B. Bacteria
Bacteria possess both DNA and RNA.
Virus possesses either DNA or RNA.
Prions are devoid of nucleic acids.

103. C. Conjugation
R plasmids are transferred by conjugation – a horizontal gene transfer method.

104. D. Kary B Mullis


PCR discovered by Kary B Mullis.
Waston and Crick – DNA double helix structure.
Salk – IPV (Killed polio vaccine).

105. A. Extrachromosomal
F plasmid is for fertility factor.
It is an extrachromosomal element.
Integration with bacterial DNA - Episome.

106. D. Can cause lysogenic conversion


Lysogenic conversion caused by bacteriophage in the process of transduction.
Plasmids can transmit multiple characters and responsible for toxin production, drug
resistance and virulence.
107. A. Transduction
Among the gene transfer processes, bacteriophage is involved only in transduction.

108. D. All the above


Other features are:
No use of dose adjustment / combination.
Horizontal transfer.
No change in virulence.

109. D. Shigella dysenteriae


Phage typing is an epidemiological tool for Salmonella, Vibrio and Staphylococcus.

110. D. Chromosomes

111. C. HFr
All other choices can transmit drug resistance

112. B. Transformation
Refer Q. no 100

113. A. Bacteriophages
Conjugation involves pili and plasmid.
Bacteriophages are involved in Transduction.

114. A. Transposon
A transposable element or transposon is a DNA sequence that can change its position
within a genome, sometimes creating or reversing mutations and altering the cell’s
genetic identity and genome size.

115. A. Post antibiotic effect


The post antibiotic effect (PAE) is defined as persistent suppression of bacterial
growth after a brief exposure (1 or 2 hours) of bacteria to an antibiotic even in the
absence of host defense mechanisms.
Factors that affect the duration of the post antibiotic effect include duration of
antibiotic exposure, bacterial species, culture medium and class of antibiotic.
It has been suggested that an alteration of DNA function is possibly responsible for
post antibiotic effect following the observation that most inhibitors of protein and
nucleic acid synthesis (aminoglycosides, fluoroquinolones, tetracyclines, clindamycin,
certain newer macrolides/ketolides, and rifampicin and rifabutin) induce long-term
PAE against susceptible bacteria.

116. A. Transformation
Transformation is the process by which a bacterial DNA molecule is taken up from the
external environment and incorporated into the genome of the recipient bacteria.

117. D. All of the above


F plasmid – Fertility factor.
C plasmid – Colicin production.
R Plasmid – Resistance factor.

118. D. Lipopolysaccharide
Lipopolysaccharides (LPS), also known as lipoglycans and endotoxins, are large
molecules consisting of a lipid and a polysaccharide composed of O-antigen, outer
core and inner core joined by a covalent bond; they are found in the outer membrane
of Gram-negative bacteria.

119. A. M. leprae & C. Treponema pallidum


Other VNC are Chlamydia, Rickettsia & Virus.
Immunology – Questions
120. Antigen binding site in immunoglobulin is:
a. Variable region
b. Hypervariable region
c. Constant region
d. Idiotope

121. Most common infections after splenectomy are by


a. Capsulated bacteria
b. Uncapsulated bacteria
c. Gram positive sepsis
d. Gram negative bacteria

122. Antigen antibody precipitation is maximally seen in which of the


following?
a. Excess of antibody
b. Excess of antigen
c. Equivalence of antibody and antigen
d. Antigen-hapten Interaction

123. Antibody that can cross placenta to protect foetus


a. IgA
b. IgG
c. IgD
d. IgM

124. Which of the following vaccines should not be given to an


Immunodeficient person (PGI)
a. Killed Influenza vaccine
b. Rubella vaccine
c. Pneumococcal vaccine
d. Oral polio vaccine

125. Transplacental infections can occur with following (PGI)


a. N. gonorrhoeae
b. T. pallidum
c. T. gondii
d. P. aeruginosa

126. Which of the following complement components is a chemoattractant?


a. C3a
b. C4a
c. C5a
d. C3b
127. CRP is:
a. An antibody
b. Derived from pneumococcus
c. Decreased in pneumococcal infection
d. Detected by precipitation reaction with carbohydrate antigen

128. Differentiation between T cells & B cells is by all of the following except
a. T cells from Sheep-RBC rosette
b. B cells form EAC rossette
c. B cells have CD3 receptors
d. B cells have surface immunoglobulins

129. Major basic protein is formed by:


a. Lymphocyte
b. Basophil
c. Neutrophil
d. Eosinophil

130. Which one of the following does not contain C3b


a. Classical pathway C5 convertase
b. Alternate pathway C5 convertase
c. Classical pathway C3 convertase
d. Alternate pathway C3 convertase

131. In anaphylaxis
a. Antigen: cell bound; Antibody: free
b. Antigen: free; Antibody: cell bound
c. Antigen: cell bound; Antibody: cell bound
d. Antigen: free; Antibody: free

132. Which of the following is a neutralization test


a. Rose waaler
b. Paul Bunnell test
c. Widal test
d. Schick test

133. Hyperacute graft rejection is mediated by


a. Preformed antibodies
b. Lymphokines
c. In situ antibody formation
d. All of the above
120. b 121. a 122. c 123. b 124. b, 125. b, 126. c 127. d 128. c
d c

129. d 130. c 131. b 132. d 133. a


134. T cell multiplication is stimulated by
a. Bovine serum
b. Phytohaemagglutinin
c. Macrolin
d. Leukotrienes

135. The complement system aids the host defense by


a. Mediating inflammation
b. Attracting phagocytic cells
c. Activating phagocytic cells
d. All of the above
e. None of the above

136. Which of the following is a generic term for a protein or glycoprotein


released by one cell population that acts as an intercellular mediator?
a. Cytokine
b. Monokine
c. Lymphokine
d. Interleukin

137. Which HLA type is closely linked with Rheumatoid arthritis?


a. HLA-A
b. HLA-B
c. HLA-DR
d. HLA-DP
e. HLA-DQ

138. Which of the following statements are true regarding immunodeficiency?


(PGI)
a. Repeated microbial infection
b. Increased susceptibility to malignancy
c. Secondary immunodeficiencies are less common than primary
d. It involves CMI primarily
e. Decrease response to skin tests.

139. Which of the following mediators of hypersensitivity are formed de-novo


after cell activation? (PGI)
a. Neutrophil chemotactic factor
b. Prostaglandin D2
c. Platelet activating factor
d. Eosinophil chemotactic factor
e. Histamine

140. General properties of complement include: (PGI)


a. It is present in normal sera
b. Increases on immunization
c. Gets activated by antigen-antibody complex
d. Destroyed at 56°C for 60 minutes.
e. Classical pathway complex is made up of 3 proteins.

141. Important properties of antigen-antibody complex include: (PGI)


a. Antigen-antibody reaction is firm
b. Binding takes place at surface
c. Only one part and not the whole molecule reacts.
d. No denaturation of antigen or antibody occurs during reaction.
e. Antigen-antibody reaction is not reversible.

142. Factors that influence the immune complex mediated damage in type III
hypersensitivity include: (PGI)
a. Shape of immune complex
b. Size of the complex formed.
c. Local vascular permeability.
d. Rapidity with which these complexes are formed
e. Deposition of complexes in the body.

143. Adenosine deaminase deficiency is seen in the following:


a. Common variable immunodeficiency
b. Severe combined immunodeficiency
c. Chronic granulomatous disease
d. Nezelof syndrome

144. IL-1 produces:


a. T-lymphocyte activation
b. Delayed wound healing
c. Increased pain perception
d. Decreased PMN release from bone marrow

145. Which of the following best denotes classical complement pathway


activation in immuno- inflammatory condition?
a. C2, C4 and C3 decreased
b. C2 and C4 normal, C3 is decreased
c. C3 normal and C2 C4 decreased
d. C2, C4, C3 all are elevated

146. Which is not a macrophage?


a. Monocyte
b. Microglia
c. Kupffer cells
d. Lymphocytes

147. In Chediak higashi syndrome, defect is:


a. Fusion of lysosome
b. T-cells
c. B-cells
d. Complement
134. b 135. d 136. a 137. c 138. 139. 140. 141. 142.
a,b,e b,c a,c,d a,b,c b,c,e

143. b 144. a 145. a 146. d

148. Hereditary angioneurotic edema is due to:


a. Deficiency of C1 esterase inhibitor
b. Deficiency of NADPH oxidase
c. Deficiency of MPO
d. Deficiency of properdin

149. The serum concentration of which of the following human IgG subclasses
is maximum ?
a. IgG1
b. IgG2
c. IgG3
d. IgG4

150. All of the following forces are involved in antigen antibody reaction
except:
a. Van der Waals forces
b. Electrostatic bond
c. Hydrogen bond
d. Covalent bond

151. Prozone phenomenon is due to:


a. Antigen excess
b. Antibody excess
c. False +ve reaction
d. False -ve reaction

152. The following methods of diagnosis utilize labeled antibodies except:


a. ELISA
b. Hemagglutination inhibition test
c. Radioimmunoassay
d. Immunofluorescence

153. C-reactive protein is:


a. An antibody as a result of pneumococcal infection
b. Derived from pneumococci
c. Detected by precipitation reaction
d. Increased in pneumococcal infection
154. Antibody diversity is due to: (PGI)
a. Gene rearrangement
b. Gene translocation
c. Antigenic variation
d. CD40 molecules
e. Mutation

155. The secretory component of IgA molecule is:


a. Formed by epithelial cell of lining mucosa
b. Formed by plasma cell
c. Formed by epithelial cell and plasma cell
d. Secreted by bone marrow

156. Which of the following immunoglobulins can cross placenta?


a. IgA
b. IgM
c. IgG
d. IgD

157. True about interferon is:


a. It is a synthetic antiviral agent
b. Inhibits viral replication in cells
c. Is specific for particular virus
d. None

158. True regarding clonal selection


a. Specific
b. Secondary response is rapid
c. Generation of diversity of antibody specificity
d. All

159. Antigen antibody precipitation is maximally seen in which of the


following:
a. Excess of antibody
b. Excess of antigen
c. Equivalence of antibody and antigen
d. Antigen - Hapten interaction

160. Which portion of MHC 1 complex forms the component of antigen


presenting part:
a. Between alpha-beta 2 microglobulin
b. Distal part of alpha chain
c. Proximal part of alpha chain
d. beta 2 microglobulin
161. Ova albumin antigen was injected into a rabbit. What antibody will it
produce initially?
a. IgG
b. IgM
c. IgE
d. IgD
147. 148. 149. 150. 151. 152. 153. c 154. 155. a 156. c
a a a d b b a,e

157. 158. 159. c 160. 161.


b d b b

162. Role of adjuvant in vaccine are all except:


a. Stimulation of Toll-like receptors
b. Activate B-lymphocyte only
c. Increase both adaptive and innate immune response
d. Activate both B and T lymphocyte
e. Ensure prolonged delivery of antigen

163. Examples of type I hypersensitivity is:


a. Lepromin test
b. Tuberculin
c. Casoni’s test
d. Arthus reaction

164. Which of the following is an example of type IV hypersensitivity?


a. Granulomatous reaction
b. Schwartzman reaction
c. Arthus reaction
d. Serum sickness

165. Skin test based on neutralization reaction is/are:


a. Casoni test
b. Lepromin test
c. Tuberculin test
d. Schick test

166. Disorders of phagocytosis are all, except:


a. Job’s syndrome
b. Chediak-Higashi syndrome
c. Myeloperoxidase deficiency
d. Wiskott-Aldrich syndrome
e. Tuftsin deficiency

167. Type IV hypersensitivity includes all, except:


a. Paul Bunnell test
b. Lepromin test
c. Tuberculin test
d. Granulomatous reactions
162. b 163. c 164. a 165. d 166. d 167. a
Immunology – Explanations
120. B. Hypervariable region
The variable regions of both L and H chains have three extremely variable
(hypervariable) amino acid sequences at the amino-terminal end that form the
antigen-binding site.
Only 5 to 10 amino acids in each hypervariable region form the antigen-binding site.
Antigen–antibody binding involves electrostatic and van der Waals’ forces and
hydrogen and hydrophobic bonds rather than covalent bonds.
The remarkable specificity of antibodies is due to these hypervariable regions.

121. A. Capsulated bacteria


B cells producing anticapsular antibodies are primarily found in the spleen.

122. C. Equivalence of antibody and antigen


In precipitation test, the antigen is in solution.
The antibody cross-links antigen molecules in variable proportions, and aggregates
(precipitates) form.
In the zone of equivalence, optimal proportions of antigen and antibody combine;
the maximal amount of precipitates forms, and the supernatant contains neither an
excess of antibody nor an excess of antigen .
In the zone of antibody excess, there is too much antibody for efficient lattice
formation, and precipitation is less than maximal.
In the zone of antigen excess, all antibody has combined, but precipitation is
reduced because many antigen–antibody complexes are too small to precipitate (i.e.,
they are “soluble”).

123. B. IgG
IgG is the only antibody to cross the placenta; only its Fc portion binds to receptors on
the surface of placental cells. It is therefore the most abundant immunoglobulin in
newborns.

124. B. Rubella vaccine.


D. Oral polio vaccine.
Live vaccines are contraindicated in immunodeficient people and pregnant women.

125. B. T. pallidum
C T. gondii
Vertical Transmission of Some Important Pathogens

Mode of Transmission Pathogen Type of Disease in Fetus or Neonate


Organism1

Transplacental Treponema pallidum B Congenital syphilis

Listeria monocytogenes B Neonatal sepsis and meningitis

Cytomegalovirus V Congenital abnormalities

Parvovirus B19 V Hydrops fetalis

Toxoplasma gondii P Toxoplasmosis


Within birth canal/at the Streptococcus agalactiae (group B B Neonatal sepsis and meningitis
time of birth streptococcus)
B Neonatal sepsis and meningitis
Escherichia coli
B Conjunctivitis or pneumonia
Chlamydia trachomatis
Conjunctivitis

Neisseria gonorrhoeae B Skin, CNS, or disseminated


infection (sepsis)
Herpes simplex type-2 V
Hepatitis B
Hepatitis B virus V
Asymptomatic infection
Human immunodeficiency virus3 V
Thrush
Candida albicans F

Breast milk Staphylococcus aureus B Oral or skin infections

Cytomegalovirus V Asymptomatic infection

Human T-cell leukemia virus V Asymptomatic infection

126. C. C5a
C5a and the C5,6,7 complex attract neutrophils.
They migrate especially well toward C5a.
C5a also enhances the adhesiveness of neutrophils to the endothelium.

127. D. Detected by precipitation reaction with carbohydrate antigen


CRP is an “acute-phase” protein that is elevated as much as 1000-fold in acute
inflammation.
CRP is not an antibody (which are γ-globulins) but rather a β-globulin. (Plasma
contains α-, β-, and γ-globulins.)
Note that CRP is a nonspecific indicator of inflammation and is elevated in response
to the presence of many organisms, not just S. pneumoniae.
Clinically, CRP in human serum is measured in the laboratory by its
reaction(agglutination or precipitation) with the carbohydrate of S. pneumoniae. The
medical importance of CRP is that an elevated CRP appears to be a better predictor of
heart attack risk than an elevated cholesterol level.

128. C. B cells have CD3 receptors


CD3 is a pan T-cell marker.

129. D. Eosinophil
Eosinophils are white blood cells with cytoplasmic granules that appear red when
stained with Wright stain.
The red color is caused by the negatively charged eosin dye binding to the positively
charged major basic protein in the granules.

130. C. Classical pathway C3 convertase


Classical pathway C3 convertase is C14b2a.

131. B. Antigen: free; Antibody: cell bound


An immediate hypersensitivity reaction occurs when an antigen (allergen) binds to
IgE on the surface of mast cells with the consequent release of several mediators.
The process begins when an antigen induces the formation of IgE antibody, which
binds firmly by its Fc portion to receptors on the surface of basophils and mast cells.
Reexposure to the same antigen results in cross-linking of the cell-bound IgE,
degranulation, and release of pharmacologically active mediators within minutes
(immediate phase).
Cyclic nucleotides and calcium play essential roles in release of the mediators.
Symptoms such as edema and erythema (“wheal and flare”) and itching appear
rapidly because these mediators (e.g., histamine) are preformed.

132. D. Schick test


Schick test is an in-vivo toxin neutralization test to assess susceptibility of a person to
diphtheria.

133. A. Preformed antibodies


Hyperacute Rejection by Preexisting Antibodies:
In rare instances, a transplant is rejected so quickly that the grafted tissue never
becomes vascularized.
These hyperacute reactions are caused by preexisting host serum antibodies specific
for antigens of the graft and have the greatest impact in highly vascularized grafts
(such as kidney and heart).
Preexisting recipient antibodies bind to HLA antigens on the endothelial cells of the
graft .
These antigen-antibody complexes activate the complement system and result in an
intense accumulation of neutrophils.
The ensuing inflammatory reaction causes endothelial damage and obstructing blood
clots within the capillaries, preventing vascularization of the graft .

134. B. Phytohaemagglutinin
T cell multiplication is stimulated by phytohaemagglutinin and concanavalin A.

135. D. All of the above


There are three main effects of complement:
(1) lysis of cells such as bacteria, allografts, and tumor cells;
(2) generation of mediators that participate in inflammation and attract neutrophils;
and
(3) opsonization (i.e., enhancement of phagocytosis).
Complement proteins are synthesized mainly by the liver.

136. A. Cytokine
Molecules that communicate among cells of the immune system are referred to as
cytokines.
In general, cytokines are soluble molecules, although some also exist in membrane-
bound forms.
The interaction of a cytokine with its receptor on a target cell can cause changes in
the expression of adhesion molecules and chemokine receptors on the target
membrane, thus allowing it to move from one location to another.
Cytokines can also signal an immune cell to increase or decrease the activity of
particular enzymes or to change its transcriptional program, thereby altering and
enhancing its effector functions.
Finally, they can instruct a cell when to survive and when to die.
137. C. HLA-DR
Rheumatoid arthritis—In this disease, autoantibodies are formed against IgG.
These autoantibodies are called rheumatoid factors and are of the IgM class.
Rheumatoid arthritis affects primarily women between the ages of 30 and 50 years.
People with HLA-DR4 genes are predisposed to rheumatoid arthritis.

138. A. Repeated microbial infection.


B. Increased susceptibility to malignancy.

E. Decreased response to skin tests.

139. B. Prostaglandin D2.


C. Platelet activating factor.
Principal mediators involved in type I hypersensitivity.

Mediator Effects

Primary

Histamine, heparin Increased vascular permeability; smooth muscle contraction

Serotonin (rodents) Increased vascular permeability; smooth muscle contraction

Eosinophil chemotactic factor (ECF- Eosinophil chemotaxis


A)

Neutrophil chemotactic factor (NCF- Neutrophil chemotaxis


A)

Proteases (tryptase, chymase) Bronchial mucus secretion; degradation of blood vessel basement
membrane; generation of complement split products

Secondary

Platelet-activating factor Platelet aggregation and degranulation; contraction of pulmonary


smooth muscles

Leukotrienes (slow reactive Increased vascular permeability; contraction of pulmonary smooth


substance of anaphylaxis, SRS-A) muscles

Prostaglandins Vasodilation; contraction of pulmonary smooth muscles; platelet


aggregation

Bradykinin Increased vascular permeability; smooth muscle contraction

Cytokines

IL-1 andTNF-α Systemic anaphylaxis; increased expression of adhesion molecules on


venular endothelial cells
IL-4 and IL-13
Increased IgE production
IL-3, IL-5, IL-6, IL-10,TGF-β, and GM-
CSF Various effects (see text)

140. A. It is present in normal sera


C. Gets activated by antigen-antibody complex
d. Destroyed at 56°C for 60 minutes.
The term ‘complement’ represents a group of proteins normally found in serum in
inactive form, but when activated they augment the immune responses.
They constitute about 5% of normal serum proteins and their level does not increase
following either infection or vaccination.
It gets denatured by hearing the serum at 56°C for 30 minutes. Such serum with lost
complement activity is called inactivated serum.
Classical pathway of complement is activated by antigen-antibody complex.

141. A. Antigen-antibody reaction is firm.


b. Binding takes place at surface.
C. Only one part and not the whole molecule reacts.
Ag-Ab reaction involves specific interaction of epitope of an antigen with the
corresponding paratope of its homologous antibody.
It is a non-covalent interaction and hence reversible yet firm.

142. B. Size of the complex formed.


C. Local vascular permeability.
E. Deposition of complexes in the body.
The reaction of antibody with antigen generates immune complexes.
Generally, these complexes facilitate the clearance of antigen by phagocytic cells and
red blood cells.
In some cases, however, the presence of large numbers and networks of immune
complexes can lead to tissue-damaging type III hypersensitivity reactions.
The magnitude of the reaction depends on the number and size of immune
complexes, their distribution within the body, and the ability of the phagocyte system
to clear the complexes and thus minimize the tissue damage.
The deposition of these complexes initiates a reaction that results in the recruitment
of complement components and neutrophils to the site, with resultant tissue injury.
Immune complexes bind to mast cells, neutrophils, and macrophages via Fc
receptors, triggering the release of vasoactive mediators and inflammatory cytokines,
which interact with the capillary epithelium and increase the permeability of the
blood vessel walls.
Immune complexes then move through the capillary walls and into the tissues where
they are deposited and set up a localized inflammatory response.

143. B. Severe combined immunodeficiency


Patients with a hereditary absence of adenosine deaminase (ADA) and purine
nucleoside phosphorylase (PNP) can have a severe deficiency of B cells and T cells,
causing SCID, although some have only mild dysfunction.
The absence of these enzymes results in an accumulation of deoxyadenosine
triphosphate (dATP), an inhibitor of ribonucleotide reductase, and a consequent
decrease in the deoxynucleoside triphosphate precursors of DNA.
This reduces the formation of Bcell and T-cell precursors in the bone marrow.
Bone marrow transplantation can be helpful.
Injections of ADA conjugated to polyethylene glycol reduce the number and severity
of infections.
Several patients with ADA deficiency have benefited from gene therapy.
A retroviral vector carrying a normal copy of the ADA gene was allowed to infect the
patient’s bone marrow cells.
The ADA gene functioned within some of these cells, and the patient’s immune status
improved.

144. A. T-lymphocyte activation


Cytokines of the interleukin 1 (IL-1) family are typically secreted very early in the
immune response by dendritic cells and monocytes or macrophages.
IL-1 secretion is stimulated by recognition of viral, parasitic, or bacterial antigens by
innate immune receptors.
IL-1 family members are generally proinflammatory, meaning that they induce an
increase in the capillary permeability at the site of cytokine secretion, along with an
amplification of the level of leukocyte migration into the infected tissues.
In addition, IL-1 has systemic (whole body) effects and signals the liver to produce
acute phase proteins such as the Type I interferons (IFNs α andβ), IL-6, and the
chemokine CXCL8.
These proteins further induce multiple protective effects, including the destruction of
viral RNA and the generation of a systemic fever response (which helps to eliminate
many temperature-sensitive bacterial strains).
IL-1 also activates both T and B cells at the induction of the adaptive immune
response.

145. A. C2, C4 and C3 decreased


In the classic pathway, antigen–antibody complexes activate C1 to form a protease,
which cleaves C2 and C4 to form a C4b,2a complex.
The latter is C3 convertase, which cleaves C3 molecules into two fragments, C3a and
C3b.

146. D. Lymphocytes
Monocytes that migrate into tissues in response to infection can differentiate into
specific tissue macrophages.
Like monocytes, macrophages can play several different roles.
Some macrophages are long-term residents in tissues and play an important role in
regulating their repair and regeneration.
Other macrophages participate in the innate immune response and undergo a
number of key changes when they are stimulated by encounters with pathogens or
tissue damage.
These are referred to as inflammatory macrophages and play a dual role in the
immune system as effective phagocytes that can contribute to the clearance of
pathogens from a tissue, as well as antigen-presenting cells that can activate T
lymphocytes.
Osteoclasts in the bone, microglial cells in the central nervous system, Kupffer cells in
liver and alveolar macrophages in the lung are tissue-specifi c examples of
macrophages with these properties.

147. A. Fusion of lysosome


Chédiak-Higashi Syndrome:
In this autosomal recessive disease, recurrent pyogenic infections, caused primarily
by Staphylococci and Streptococci, occur.
This is due to the failure of the lysosomes of neutrophils to fuse with phagosomes.
The degradative enzymes in the lysosomes are, therefore, not available to kill the
ingested organisms.
Large granular inclusions composed of abnormal lysosomes are seen.
In addition, the neutrophils do not function correctly during chemotaxis as a result of
faulty microtubules.
The mutant gene in this disease encodes a cytoplasmic protein involved in protein
transport.
Peroxide and superoxide formation is normal, as are B-cell and T-cell functions.
Treatment involves antimicrobial drugs.
There is no useful therapy for the phagocyte defect.

148. A. Deficiency of C1 esterase inhibitor


Inherited deficiency of C1 esterase inhibitor results in angioedema.
When the amount of inhibitor is reduced, an overproduction of esterase occurs.
This leads to an increase in anaphylatoxins, which cause capillary permeability and
edema.

149. A. IgG1
Order of serum concentration of subtypes of IgG is: IgG1>IgG2>IgG3>IgG4.

150. D. Covalent bond


Bonds formed between antigen and antibody include hydrogen bonds,hydrophobic
interactions, ionic bonds/electrostatic forces and vander waal forces. Covalent bond is
not formed.

151. B. Antibody excess


Prozone is the zone of excess antibodies so the valency of all antibodies is not
satisfied.

152. B. Hemagglutination inhibition test


Many viruses clump red blood cells from one species or another (active
hemagglutination).
This can be inhibited by antibody specifically directed against the virus
(hemagglutination inhibition) and can be used to measure the titer of such antibody.
Red blood cells also can absorb many antigens and, when mixed with matching
antibodies, will clump (this is known as passive hemagglutination, because the red
cells are passive carriers of the antigen).

153. C. Detected by precipitation reaction


CRP is an “acute-phase” protein that is elevated as much as 1000-fold in acute
inflammation.
CRP is not an antibody (which are γ-globulins) but rather a β-globulin. (Plasma
contains α-, β-, and γ-globulins.)
Note that CRP is a nonspecific indicator of inflammation and is elevated in response
to the presence of many organisms, not just S. pneumoniae.
Clinically, CRP in human serum is measured in the laboratory by its
reaction(agglutination or precipitation) with the carbohydrate of S. pneumoniae. The
medical importance of CRP is that an elevated CRP appears to be a better predictor of
heart attack risk than an elevated cholesterol level.

154. A. Gene rearrangement.


E. Mutation.
Antibody diversity depends on
(1) multiple gene segments,
(2) their rearrangement into different sequences,
(3) the combining of different L and H chains in the assembly of immunoglobulin
molecules, and
(4) mutations.
A fifth mechanism called junctional diversity applies primarily to the antibody heavy
chain. Junctional diversity occurs by the addition of new nucleotides at the splice
junctions between the V-D and D-J gene segments.

155. A. Formed by epithelial cell of lining mucosa


The secretory component is a polypeptide synthesized by epithelial cells that
provides for IgA passage to the mucosal surface.
It also protects IgA from being degraded in the intestinal tract. In serum, some IgA
exists as monomeric H2L2.

156. C IgG
IgG is the only antibody to cross the placenta; only its Fc portion binds to receptors on
the surface of placental cells. It is therefore the most abundant immunoglobulin in
newborns.

157. B. Inhibits viral replication in cells


Alpha and beta interferons are a group of proteins produced by human cells after
viral infection (or after exposure to other inducers).
They inhibit the growth of viruses by blocking the synthesis of viral proteins.
They do so by two main mechanisms: One is a ribonuclease that degrades mRNA,
and the other is a protein kinase that inhibits protein synthesis.
Their inhibitory action is not specific for any particular virus.
However, they are typically specific in regard to the host species in which they act
(i.e., interferons produced by human cells are active in human cells but are much less
effective in cells of other species).

158. D. All

159. C. Equivalence of antibody and antigen

160. B. Distal part of alpha chain


161. B. IgM
IgM is the antibody of primary immune response.

162. B. Activate B-lymphocyte only


Adjuvants enhance the immune response to an immunogen.
They are chemically unrelated to the immunogen and differ from a carrier protein
because the adjuvant is not covalently bound to the immunogen, whereas the carrier
protein is.
Adjuvants can act in a variety of ways; they can cause slow release of immunogen,
thereby prolonging the stimulus; enhance uptake of immunogen by antigen-
presenting cells; and induce costimulatory molecules (“second signals”).
Another important mechanism of action of some adjuvants is to stimulate Toll-like
receptors on the surface of macrophages, which results in cytokine production that
enhances the response of T cells and B cells to the immunogen (antigen).
Some human vaccines contain adjuvants such as aluminum hydroxide or lipids.

163. C. Casoni’s test


Casoni’s Intradermal Test: It is an immediate hypersensitivity (Type 1) skin test
introduced by Casoni in 1911, using fresh sterile hydatid fluid.
The antigen in hydatid fluid is collected from animal or human cysts and is sterilized
by Seitz or membrane filtration.
The fluid is injected (0.2 mL) intradermally in one arm and an equal volume of saline
as control is injected in the other arm.
In a positive reaction, a large wheal of about 5 cm in diameter with multiple
pseudopodia like projections appears within half an hour at the test side and fades in
about an hour.
A secondary reaction consisting of edema and induration appears after 8 hours. The
test is almost abandoned now due to non specificity and has been supplemented by
serological tests.

164. A. Granulomatous reaction


Granulomas are the hallmark of type 4 hypersensitivity reactions, they develop when
continuous activation of macrophages induces them to adhere closely to one another.
Under these conditions, macrophages assume an epithelioid shape and sometimes
fuse to form multinucleated giant cells .
These giant cells displace the normal tissue cells, forming palpable nodules, and
releasing high concentrations of lytic enzymes, which destroy surrounding tissue. The
granulomatous response can damage blood vessels and lead to extensive tissue
necrosis.

165. D. Schick test


It is an in-vivo toxin neutralization test to find out susceptibility of a person to develop
diphtheria.

166. D. Wiskott-Aldrich syndrome


Recurrent pyogenic infections, eczema, and bleeding caused by thrombocytopenia
characterize this syndrome.
These symptoms typically appear during the first year of life. It is an X-linked disease
and thus occurs only in male infants.
The most important defect is the inability to mount an IgM response to the capsular
polysaccharides of bacteria, such as pneumococci.
IgG levels and IgA levels are normal, but cell-mediated immunity is variable.
The defect appears to be in the ability of T cells to provide help to B cells.
The mutant gene encodes a protein involved in actin filament assembly. Bone marrow
transplantation may be helpful.

167. A. Paul Bunnell test


Paul bunnel test is a heterophile agglutination test for presumptive diagnosis of
infectious mononucleosis.
Bacteriology — Questions
168. A 20-year old man presented with hemorrhagic colitis. The stool sample
grew Escherichia coli in pure culture . The following serotype of E.coli is
likely to be the causative agent:
a. O 157: H7
b. O 159: H7
c. O 107: H7
d. O 55: H7

169. “L-form” phenomenon is most likely to occur in:


a. Mycobacterium tuberculosis
b. Pseudomonas
c. Proteus
d. Mycoplasma

170. All of the following cause a grey white membrane in the throat except
a. Streptococcal tonsillitis
b. Diphtheria
c. Ludwig’s angina
d. Adenovirus pharyngitis

171. A male patient presented with granulomatous penile ulcer. On Wright-


giemsa stain tiny organisms of 2 microns within macrophages seen. What is
the causative organism?
a. LGV
b. Calymmatobacterium granulomatis
c. Neisseria
d. Staph aureus

172. Not used in leptospirosis


a. Microscopic agglutination test
b. Dark field illumination
c. Macroscopic agglutination test
d. Weil felix reaction

173. H.pylori infection is associated with development of


a. Gastrointestinal stromal tumor
b. Gastric (MALT) lymphoma
c. Gastric neuroendocrine tumor
d. Gastric polyp

174. Weil’s disease is caused by


a. Trepenema pallidum
b. Leptospira interrogans
c. Borrelia burgdorferi
d. All of above

175. Bacterial vaginosis is due to


a. Gardnerella vaginalis
b. Trichomonas vaginalis
c. Candida albicans
d. E.coli

176. Malignant otitis extena is due to


a. Human papilloma virus
b. Staphylococcus aureus
c. Radiotherapy
d. Pseudomonas

177. About enterotoxigenic E.coli true is


a. Does not cause travellers’ diarrhea
b. Most commonly causes diarrhea in developing countries
c. Invasion of gut mucosa
d. Fomite borne

178. Which of the followings is false about C.perfringens?


a. Food poisoning strains produce heat resistant spores
b. Show positive naegler reaction
c. Most important toxin is hyaluronidase
d. M.C. cause of gas gangrene

179. Malignant pustule is caused by


a. Carbuncle
b. Ulcerating melanoma
c. Anthrax of skin
d. None

180. All are true about Haemophillus influenzae except


a. Requires factors X and V
b. Doesn’t affect infants below 2 months of age
c. Capsular polypeptide protein is responsible for virulence
d. M.C. invasive disease of H. influenzae is meningitis
168. 169. 170. c 171. 172. 173. 174. 175. a 176. d 177. b
a d b d b b

178. c 179. c 180. c

181. All are true regarding tetanus except


a. Transmission through contaminated wounds and injuries
b. More common in winters and dry weather
c. Reservoir in soil and intestines of humans and animals
d. No herd immunity or lifelong immunity

182. A 35-year-old labourer 3 years back presented with single indurated


penile ulcer not treated. Later he presented with skin rash with neurological
symptoms for which he got treated.Test to moniter response to treatment is
a. TPI
b. VDRL
c. FTA-ABS
d. Dark field microscopy

183. Characteristic of Bacillus cereus food poisoning is


a. Presence of Fever
b. Presence of Pain abdomen
c. Absence of Vomiting
d. Absence of Diarrhoea

184. Most common cause of antibiotic induced colitis:


a. Escherichia coli
b. Clostridium difficile
c. Salmonella
d. Campylobacter jejuni

185. The most common cause of non-gonococcal urethritis is


a. Trichomonas
b. Chlamydia
c. Ureaplasma
d. Mycoplasma

186. Causative agent of granuloma inguinale is


a. Calymmatobacterium granulomatis
b. Chlamydia trachomatis
c. Haemophilus ducreyi
d. Treponema

187. Buruli ulcer is caused by


a. Brucella
b. Streptococcus
c. Spirillum minus
d. M. ulcerans

188. Painful genital ulcers occur in (PGI)


a. Primary syphilis
b. Herpes genitalis
c. Granuloma inguinale
d. Chancroid
189. Pneumonia can be caused by following organisms
a. Chlamydia psittaci
b. Trichuris trichiura
c. Toxoplasma gondii
d. Echinococcus granulosus

190. All are zoonotic diseases except


a. Brucellosis
b. Leptospirosis
c. Anthrax
d. Diphtheria

191. Lyme disease is caused by


a. Treponema macdadei
b. Borrelia burgdorferi
c. Leptospira cutaneum
d. Treponama dammini

192. Listeria monocytogenes may cause all except


a. Myocardial necrosis
b. Spontaneous abortion & still birth
c. Multiple organ abscess
d. Meningitis

193. A disease seen in sewer workers is


a. byssinosis
b. anthrax
c. brucellosis
d. leptospirosis

194. Man is the terminal end for


a. Gonococcus
b. Treponema
c. Tetanus
d. Ancylostomiasis

195. Bartholinitis is caused by


a. Trichomonas
b. Candida
c. Staphylococcus
d. Gonococcus

196. Frei’s test is used to diagnose


a. Granuloma inguinale
b. LGV
c. Donovanosis
d. Infectious mononucleosis

197. A biological false positive reaction for syphilis is most likely to occur in
(PGI)
a. Enteric fever
b. Malaria
c. Leishmaniasis
d. Tropical eosinophilia

181. 182. 183. 184. 185. 186. 187. 188. 189. a 190. d
b b b b b a d b,d

191. 192. 193. 194. c 195. 196. 197.


b a d d b b,c,d

198. Disease which can be transmitted by milk


a. Q fever
b. Poliomyelitis
c. Whooping cough
d. Diphtheria

199. Bubo is seen in all except


a. LGV
b. Plague
c. Donovanosis
d. Filariasis

200. Scalded skin syndrome is caused by


a. Staphylococcus
b. Klebsiella
c. Tinea
d. Candida

201. Granulomatosis infantiseptica is caused by


a. Measles virus
b. Streptococcus
c. Gonococcus
d. Listeria

202. All of the following can give membrane in the pharynx except
a. Staphylococcus aureus
b. Corynebacterium
c. Candida
d. Vincent’s angina

203. Malignant pustule is associated with


a. Corynebacterium diphtheriae
b. Clostridium tetani
c. Bacillus anthracis
d. Streptococcus pyogenes

204. Eaton agent is


a. Corynebacterium
b. Pseudomonas
c. Mycoplasma
d. Gonococcus

205. Acute osteomyelitis in children is most often caused by


a. Staphylococcus aureus
b. Streptococcus pyogenes
c. Escherichia coli
d. Pseudomonas

206. The organism most commonly implicated in sub acute bacterial


endocerditis
a. Staphylococcus albus
b. Staphylococcus aureus
c. Streptococcus faecalis
d. Viridans streptococci

207. Indicators of water pollution are all except


a. Escherichia coli
b. Clostridium perfringens
c. Streptococcus faecalis
d. Salmonella typhimurium

208. Following diseases are spread by mite


a. Trench fever
b. Endemic typhus
c. Scrub typhus
d. Q fever

209. Fatal septicemia is caused by


a. Streptococcus pyogenes
b. Group B streptococcus
c. Viridans streptococci
d. Staphylococcus albus

210. Erythrasma is caused by


a. Staphylococcus aureus
b. Streptococcus pyogenes
c. C. diphtheriae
d. C. minutissimum
211. Porter’s disease is a synonym for
a. Relapsing fever
b. Weil’s disease
c. Bubonic plague
d. Cutaneous anthrax

212. Helicobacter pylori is not associated with:


a. Gastrointestinal lymphoma
b. Gastric cancer
c. Gastric leiomyoma
d. Peptic ulcer

213. A child with pyoderma becomes toxic and presents with respiratory
distress. His chest radiograph shows patchy areas of consolidation and
multiple bilateral thin walled air containing cysts. The most likely etiological
agent in this case is:
a. Mycobacterium tuberculosis
b. Staphylococcus aureus
c. Mycobacterium avium intracellulare
d. Pneumocystis carinii
198. a 199. c 200. a 201. d 202. a 203. c 204. c 205. a 206. d

207. d 208. c 209. b 210. d 211. d 212. c 213. b

214. A 73-year-old woman with a history of diabetes presents with left ear
pain and drainage of pus from the ear canal. She has swelling and
tenderness over the left mastoid bone. Which of the following
microorganisms is the most likely causative agent?
a. Hemophilus influenzae
b. Mucor spp.
c. Pseudomonas aeruginosa
d. Streptococcus pyogenes

215. Which of the following is not a horizontal gene transfer process?


a. Conjugation
b. Transduction
c. Transcription
d. Transformation

216. A patient with fever of 103 degrees F, bradycardia & leucopenia probably
has
a. Influenza
b. Typhoid
c. Malaria
d. Meliodosis
217. The most probable organism causing food poisoning in a child who has
eaten ice cream 16 - 18 hrs earlier is:
a. Salmonella typhimurium
b. C. botulinum
c. C. perfringens
d. Staphylococcus aureus

218. True about Mycobacterium tuberculosis is :


a. Strict aerobe
b. Gram –ve
c. Thin wall
d. Curved rod & cocci

219. All of the following method(s) can be used for monitoring efficacy of
treatment in syphilis except:
a. FTA-Abs
b. RPR card test
c. Kahn Test
d. VDRL

220. Swarming growth is given by:


a. Clostridium tetani
b. Vibrio cholerae
c. Staphylococcus aureus
d. Bacillus anthracis

221. Treponemes can be visualized by all of the following except


a. Fluorescent microscopy
b. Gram stain
c. Silver stain
d. Dark-field microscopy

222. All of the following bacteria are halophilic except


a. V. parahaemolyticus
b. V. alginolyticus
c. V. vulnificus
d. V. cholerae

223. Tumbling motility is seen with


a. Proteus vulgaris
b. Vibrio cholerae
c. Listeria monocytogenes
d. Tatumella ptyseos

224. All of the following diseases are transmitted by louse except


a. Trench fever
b. Endemic relapsing fever
c. Epidemic relapsing fever
d. Epidemic typhus

225. Recrudescent infection is seen with


a. Plasmodium vivax
b. Chlamydia trachomatis
c. Rickettsia prowazekii
d. Mycoplasma pneumoniae

226. Acquisition of penicillin resistance in Staphylococcus due to


a. Mutation
b. Conjugation
c. Transduction
d. Transformation

227. Most common cause of UTI in pregnancy


a. Pseudomonas aeruginosa
b. Proteus mirabilis
c. Staphylococcus
d. Escherichia coli

214. c 215. c 216. 217. 218. 219. 220. 221. b 222. d 223. c
b a a a a

224. 225. c 226. c 227.


b d

228. Each of the following statements concerning Clostridium perfringens is


correct except
a. It causes gas gangrene
b. It causes food poisoning
c. It produces an exotoxin that degrades lecithin and causes necrosis and hemolysis
d. It is a gram-negative rod that does not ferment lactose

229. Each of the following statements concerning spirochetes is correct


except
a. Species of Treponema are part of normal flora of mouth
b. Species of Borrelia cause a tick borne disease called relapsing fever
c. The species of Leptospira that cause leptospirosis grow primarily in humans and
are usually transmitted by human to human contact
d. Species of Treponema cause syphilis and yaws

230. Ticks are vectors for transmission of each of the following diseases
except
a. Rockey mountain spotted fever
b. Trench fever
c. Tularemia
d. Lyme disease

231. CAMP test is used for the identification of:


a. Group B streptococcus
b. Streptococcus pyogenes
c. Enterococcus faecalis
d. Viridans streptococci

232. A 30-year-old male patient has a large, spreading and exuberant ulcer
with bright red granulation tissue over the glans penis. There was no
lymphadenopathy. The most likely causative organism is:
a. Treponema pallidum
b. Herpes simplex virus type 1
d. Herpes simplex virus type 2
d. Calymmatobacterium granulomatis

233. All of the following are common causes of nosocomial infections, except:
a. Staphylococcus aureus
b. P. aeruginosa
c. Enterobacteriaceae
d. Mycobacterium

234. Mcfadyean reaction is seen in


a. Weil’s disease
b. Meningococcemia
c. Syphilis
d. Malignant pustule

235. Panton valentine toxin is produced by


a. Staphylococcus
b. Streptococcus
c. Gonococcus
d. Pneumococcus

236. Anton’s test is used for


a. Nocardia asteroides
b. L. monocytogenes
c. E. coli
d. K. pneumoniae

237. Oils & grease are sterilized by


a. hot air oven
b. autoclaving
c. irradiation
d. ethylene oxide
238. Which of the following organisms is the most common cause of
septicemia in patients with sickle cell disease?
a. Salmonella enteritidis
b. Streptococcus pneumoniae
c. Streptococcus agalactiae
d. Shigella sonnei

239. A 42-year-old wig manufacturer who was involved in inspecting herds for
raw material initially experienced only mild upper respiratory tract
symptoms, but 2 days later was brought to the emergency room with fever,
severe dyspnea, cyanosis, and tachycardia. Blood cultures were drawn and
after 16 hours of incubation, the smears revealed large, gram-positive rods
with subterminal spores. The organism was nonhemolytic on blood agar and
nonmotile. What is the most probable identity of this organism?
a. Clostridium tertium
b. Bacillus anthracis
c. Clostridium perfringens
d. Bacillus subtilis
228. d 229. c 230. b 231. a 232. d 233. d 234. d 235. a 236. b

237. a 238. b 239. b

240. A patient who recently returned from a camping trip is hospitalized with
high fever and prostration that recurs every 5 to 7 days. Loosely coiled
spiral-shaped organisms 10 to 20 microns in length are noted in a Wright-
stained smear of the patient’s blood. Which of the following is the most
probably etiologic agent?
a. Leptospira interrogans
b. Spirillum minus
c. Borrelia hermsii
d. Treponema pallidum

241. A 16-year-old girl was admitted to a hospital because of hypotension,


cyanosis, high fever (39. 9°C), and bilateral, non-fluctuant, inguinal, and
axillary lymphadenopathy. The patient died the same day. Blood cultures
obtained on admission were positive for bipolar-staining, short, gram-
negative bacilli. What is the most probable etiologic agent?
a. Yersinia pestis
b. Brucella abortus
c. Klebsiella rhinoscleromatis
d. Francisella tularensis

242. Which of the following is the arthropod vector of Indian Tick Typhus?
a. Dermacentor species
b. Ornithodoros species
c. Culex species
d. Ixodes species
243. The antistreptolysin O titre is raised in infections caused by
a. Streptococcus sanguis
b. Streptococcus pneumoniae
c. Streptococcus pyogenes
d. Streptococcus bovis
e. Streptococcus mutans

244. Staphylococcal food poisoning is caused by:


a. Ingestion of preformed exotoxin
b. Colonization of a mucosal surface followed by the production of exotoxin
c. Colonization of a wound or abscess followed by local exotoxin production
d. All of the above
e. None of the above

245. The infectious stage of chlamydiae is called


a. Elementary Body
b. Reticulate Body
c. Contagious Body
d. Oogonial Body

246. The spirochetes include the causative agents for


a. Syphilis
b. Lyme disease
c. Both Syphilis and Lyme disease
d. Neither Syphilis and Lyme disease

247. Which genus of bacterium contributes to plaque, caries, gingivitis, and


periodontal disease?
a. Streptococcus
b. Staphylococcus
c. Bacillus
d. Escherichia
e. Proteus

248. A bacterial culture with a starting density of 103 cells/ml is incubated in


liquid nutrient broth. If the bacteria have both a lag time & generation time
of 10 minutes, what will the cell density be at 30 minutes
a. 1.0 x 103
b. 2.0 x 103
c. 3.0 x 103
d. 4.0 x 103

249. In the management of leprosy, lepromin test is most useful for


a. Herd immunity
b. Prognosis
c. Treatment
d. Epidemiological investigations

250. A 60 year old man is diagnosed with carcinoma colon & is scheduled for
surgery. He develops persistent low grade fever and constitutional
symptoms. Physical examination is unremarkable other than a heart murmur
not previously present. Outpatient blood cultures yield Gram positive, non-
hemolytic cocci. The most likely cause for his clinical picture is
a. Spread of carcinoma
b. Anxiety with skin flora contamination of the blood cultures
c. Subacute endocarditis caused by Streptococcus bovis
d. Subacute endocarditis caused by group B streptococci
240. c 241. a 242. d 243. c 244. a 245. a 246. c 247. a

248. d 249. b 250. c

251. A diagnosis of diphtheria is confirmed by


a. Typical microscopic appearance of the organism stained with methylene blue
b. Isolation of typical organisms from materials such as blood, showing invasiveness
c. Detection of beta phage plaques in cultures of suspicious isolates
d. Demonstration of toxin production by a suspicious isolate

252. A 20 years old male had pain in abdomen and mild fever followed by
gastroenteritis. The stool examination showed presence of pus cells & RBC
on microscopy. The etiological agent responsible is most likely
a. EIEC
b. ETEC
c. EPEC
d. EAEC

253. Anti tubercular drug susceptibility can be done by all of the following
methods, except
a. Resistance ratio method
b. Disc diffusion method
c. Molecular method
d. Radiometric broth method

254. All are true about small colony variants (SCVs) of Staphylococcus aureus
except
a. Grow as small, nonpigmented and non haemolytic colony on blood agar
b. Defective electron transport chain
c. Fail to express several putative virulence factors
d. Sensitive to gentamicin

255. Cefoxitin disk (30μg) diffusion is a better indicator of the presence of


mecA mediated resistance than the oxacillin disk because
a. hyperproduction of beta-lactamases by borderline oxacillin-resistant strains of S.
aureus
b. being an unstable compound, oxacillin is difficult to use in the laboratory
c. cefoxitin is a cheaper compound
d. MIC cannot be detected by oxacillin

256. A 10-year-old male child develops fever with signs of toxemia. BACTEC
blood culture is negative at 48 hours. Throat examination revealed a white
patch in the posterior pharyngeal wall. A Gram stained smear of the throat
swab collected shows Gram positive rods. Optimum iron concentration of 0.
1 mg/L is essential for expression of its virulence factors. Which of the
following macromolecules is important in iron metabolism?
a. Ferric oxide
b. Lactoferrin
c. Siderophores
d. Transferrin

257. A 45 year old female presented to OPD with carbuncle in the back of the
neck. The pus culture showed CA-MRSA. Which is true
a. CA-MRSA is more virulent & less resistant
b. Scc I, II, III is related
c. Not associated with PVL
d. All are true

258. An outbreak of bacillary dysentery due to shigella species has occurred in


the neonatal ICU. The epidemiologist has requested the Microbiologist to
carry out typing of the organism to identify the source. Serotyping would be
applicable in outbreaks due to all the species of Shigella except
a. S. dysenteriae
b. S. flexneri
c. S. sonnei
d. S. boydi

259. What typing method would be applicable for the above species
a. Phage typing
b. Colicin typing
c. Biotyping
d. Molecular typing

260. All are true except


a. Slide coagulase test detects coagulase enzyme
b. Slide coagulase test must be confirmed with tube coagulase test
c. Rabbit plasma with EDTA is the recommended media
d. S. intermedius & S. schleiferi may produce coagulase enzyme
251. 252. 253. 254. 255. 256. c 257. 258. c 259. b 260. a
d a b d a a
261. Rapid test for detection of Methicillin resistance are all except
a. Cycling probe amplification
b. Fluorescence technique
c. Latex agglutination
d. Automated antimicrobial susceptibility test

262. Vancomycin intermediate S. aureus (VISA) is an emerging infection. The


MIC of vancomycin for such strains as per CLSI guidelines is
a. <4 µg/ml
b. 8-16 µg/ml
c. 16-32 µg/ml
d. ≥ 32 µg/ml

263. Which of the following virulence factors in S. epidermidis is responsible


for colonization and establishment of infection in the absence of foreign
bodies such as catheter or stent?
a. Capsular polysaccharide Adhesin PS/A
b. Polysaccharide intercellular adhesin
c. Fibrinogen binding protein
d. Fatty acid modifying enzymes

264. Cell wall plays an important role in adherence. Central role in promoting
initial adherence of group A β hemolytic Streptococci to pharyngeal
epithelial cells
a. Lipoteichoic acid
b. M protein
c. Polysaccharide capsule
d. Hyaluronic acid

265. All are true for Streptococcus except


a. Streptodornase cleaves DNA
b. Streptolysin O is active in reduced state
c. Streptokinase is produced from serotype A, C, K
d. Pyrogenic toxin A is plasmid mediated

266. All are true regarding Pneumococci except


a. Strain isolated from conjunctiva are non-capsulated, rest are all having
polysaccharide capsules
b. Empyema is the most common complication of Pneumococcal pneumonia
c. Austrian’s syndrome is an occurrence in Pneumococcal pneumonia
d. 80% isolates are intermediate resistance to penicillin

267. A patient of RHD developed infective endocarditis after dental


extraction. Most likely organism causing this
a. Viridans streptococci
b. Streptococcus pneumoniae
c. Streptococcus pyogenes
d. Staphylococcus aureus

268. HLAR strain is seen in


a. Staphylococci
b. Streptococci
c. Enterococci
d. Pneumococci

269. Expression of Van A gene in Enterococci leads to


a. Inducible high level resistance to Vancomycin & Teicoplanin
b. Inducible variable resistance to Vancomycin & Teicoplanin
c. Inducible low level resistance to Vancomycin & sensitive to Teicoplanin
d. Intrinsic low level resistance to Vancomycin & Teicoplanin

270. A 2 years old boy is hospitalized with Staphylococcus aureus pneumonia.


He gives history of similar bouts of bacterial infections in the past. He had
recovered uneventfully from measles 6 months ago. On examination, scant
tonsillar tissue is noted with no lymphadenopathy. Immunoelectrophoresis
reveals subnormal levels of gamma globulin. Nitroblue tetrazolium and
chemiluminescence assays indicate phagocytic killing. The most likely
disorder the child is suffering from is
a. Defect of the Btk gene
b. Defect of the SAP gene
c. Defect of the WAS gene
d. ICAM-1 deficiency
261. 262. 263. c 264. 265. c 266. 267. 268. c 269. a 270. a
d b b d a

271. A 12-year-old boy develops glomerulonephritis a week after he was


treated for a sore throat.The causative agent is identified by serotyping of
the
a. Capsule
b. M proteins
c. Outer membrane proteins
d. Pili

272. Which of the following is incorrect about Mycobacterium tuberculosis?


a. Type specificity is due to protein Ag
b. Tuberculin from human, bovine and murine bacilli is indistinguishable
c. Ab is found to be useful for diagnosis
d. Ab are irrelevant in immunity

273. Which of the following is used for epidemiological typing of M.


tuberculosis strain?
a. PCR based DNA amplification
b. Ligase chain reaction
c. TMA targeting rRNA
d. RFLP & IS fingerprinting

274. Which of the following tests is useful for diagnosing latent tuberculosis?
a. Mantoux test
b. PCR
c. IS 6110
d. gIFN release assay

275. Which of the following is not a characteristic feature of family


Enterobacteriaceae?
a. Reduce nitrate to nitrite
b. Catalase +ve and Oxidase –ve
c. Motile only by peritrichous flagella or nonmotile, capsulated or non capsulated
d. Ferment lactose, produce acid and gas

276. All are true about brucella except


a. B. abortus is capnophilic
b. Transmitted by aerosol can occur occasionally
c. Paesturisation destroys it
d. 2 ME is used to detect IgA

277. Which of the following is incorrect about about UTI?


a. Commonly caused by E. coli serotypes found in faeces
b. Cystitis caused by mostly the strains which carry K Ag
c. Urine is a good medium for coliforms
d. S. aureus is significant even in low count

278. False about T. pallidum


a. T. phagedenis usually used in diagnosis of Syphilis
b. Transfusion syphilis can be prevented by storing the blood in refrigerator for 4
days
c. Polysaccharide Ag used in specific T. pallidum test
d. Serial passage in rabbit reduces the virulence of the spirochete to human being

279. Which of the following statements is false regarding relapsing fever?


a. Louse borne R. F. tends to occur as epidemic
b. Relapses are more frequent in tickborne R. F
c. Infection is transmitted by bite in both
d. More severe clinical picture in louse borne R. F

280. The alteration in serotype from S. anatum to S. Newington is due to


a. Mutation
b. Transduction
c. Lysogeny
d. Plasmid mediated
281. Regarding the antigenic variation of S. Typhi, which statement is
incorrect?
a. Smooth to rough variation indicates loss of virulence
b. H-O variation is due to total loss of flagella
c. Flagellar phase 1 Ag is specific
d. V-W variation indicate agglutinable with O antisera
271. b 272. c 273. d 274. d 275. d 276. d 277. b 278. d 279. c

280. c 281. b

282. A lac+, glucose fermenting, Gram negative rod isolated from a previously
healthy child with bloody diarrhoea is most likely to be
a. Shigella sonnei
b. Pseudomonas aeruginosa
c. Escherichia coli
d. Salmonella enterica

283. Which of the following is false about H Ag?


a. Heat labile
b. Highly immunogenic
c. Form chalky granular clump in widal test
d. Increased titre seen in anamnestic reaction

284. All of the following Gram-negative bacteria are known to cause corneal
ulcer EXCEPT:
a. Pseudomonas aeruginosa
b. Moraxella lacunata
c. Bacillus brevis
d. Klebsiella pneumonia

285. An elderly man presented with fever and cough. Sputum examination
revealed gram negative organisms that were grown on Buffered charcoal
yeast extract agar. The organism involved is?
a. H. influenzae
b. Legionella pneumophila
c. Burkholderia cepacia
d. Brucella

286. The bacteria producing pseudohaemoptysis is;


a. Erwinia herbicola
b. Mycobacterium tuberculosis
c. Serratia marcescens
d. Ehrlichia sennetsu
e. Legionella pneumophilla.

287. Diene’s method is used for


a. Mycoplasma
b. Plague
c. Chlamydia
d. Diphtheria
e. Rickettsia

288. Which one of the following microorganisms uses antigenic variation as a


major means of evading host responses?
a. Streptococcus pneumoniae
b. Borrelia recurrentis
c. Mycobacterium tuberculosis
d. Listeria monocytogenes
e. Helicobacter pylori

289. Diazo reaction is useful for the diagnosis of


a. Typhoid fever between 5th to 14th days
b. Cholera
c. Typhoid fever in the 4th week
d. Meningococcal meningitis
e. Tuberculosis

290. Reverse CAMP test is positive for:


a. Clostridium tetani
b. Clostridium perfringens
c. Clostridium difficile
d. Clostridium novyi
e. Clostridium septicum

291. Most immunogenic in typhoid is:


a. O antigen
b. Vi antigen
c. H antigen
d. M antigen
e. Fimbrial antigen

292. PLET medium used for isolation of Bacillus anthracis consists of:
a. Polymyxin, lactose, EDTA and Thallous acetate.
b. Polymyxin, lysozyme, EDTA and Tetracycline.
c. Polymyxin, lactose, EDTA and Tetracycline.
d. Polymyxin, lysozyme, EDTA and Thallous acetate
e. Polymyxin, lactic acid, EDTA and Tetracycline

293. Panton-Valentine leucocidin is seen in infection with:


a. Streptococci
b. Staphylococcus
c. Gonococcus
d. Pneumococcus
e. Enterococcus
282. c 283. c 284. c 285. 286. c 287. 288. 289. a 290. b 291. c
b a b

292. 293.
d b

294. Mucoid colonies are produced most often by Pnueumococci type_____:


a. I
b. II
c. III
d. IV
e. V

295. A 73-year-old woman with a history of diabetes presents with left ear
pain and drainage of pus from the ear canal. She has swelling and
tenderness over the left mastoid bone. Which of the following
microorganisms is the most likely causative agent?
a. Hemophilus influenzae
b. Klebsiella pneumoniae
c. Mucor spp.
d. Pseudomonas aeruginosa

296. The methods of isolation of Chlamydia are: (PGI)


a. Yolk sac inoculation
b. Cell culture using McCoy cell lines
c. Cell culture using HeLa cells
d. Cell culture using Vero cell lines
e. Preston medium

297. Regarding Helicobacter pylori infections following statements are true:


(PGI)
a. More prevalent in developing countries.
b. Toxigenic strains can cause ulcers
c. All children in developing countries have immunity by 5 years of age.
d. About 50 % world’s population is affected
e. Apart from humans the only other animal it infects is cat

298. A VDRL reactive mother has delivered recently. Which of the following
tests can be used to determine the risk of transmission to newborn? (PGI)
a. TPHA on serum sample of mother
b. TPHA on serum sample of newborn
c. VDRL on the paired sample of the infant and mother
d. VDRL on serum sample of newborn.
e. RPR on serum sample of mother

299. Which of the following factors are true regarding XDR-TB? (PGI)
a. MDR-TB cannot get converted to XDR-TB
b. Diagnosis of XDR-TB is always laboratory based.
c. Symptoms of XDR-TB are same as drug susceptible tuberculosis.
d. XDR-TB strains are more common in HIV patients.
e. XDR-TB is Mycobacterium tuberculosis that is resistant to isoniazid, rifampicin,
pyrazinamide any fluoroquinolone and streptomycin

300. Following statements are true regarding V cholerae O139 Bengal (PGI)
a. First discovered in Bengal.
b. Clinical manifestations are similar 01 EL Tor strains
c. Epidemiologically indistinguishable from 01 El Tor strain
d. Produces 01 lipopolysaccharide.
e. Preexisting immunity to V. cholerae offers protection against the V. cholerae O139
strain

301. Conjugate vaccines are available against: (PGI)


a. H. influenzae
b. Streptococcus pneumoniae
c. N. meningitides (group B)
d. N. meningitides (group C)
e. Listeria monocytogenes

302. Staining techniques used to stain Corynebacterium diphtheriae are (PGI)


a. Zeihl-Neelsen stain
b. Ponder’s stain
c. Negative stain
d. Albert stain
e. Trichrome stain
294. c 295. d 296. 297. 298. b,c 299. 300. b,c 301.
a,b,c a,b,d b,c,d a,b,d

302.
b,d

303. Following statements are true regarding botulism (PGI)


a. Infant botulism is caused by ingestion of preformed toxin
b. Clostridium botulinum A, B, E and F cause human disease.
c. The gene for botulinum toxin is coded by a bacteriophage.
d. Clostridium baratti may also cause botulinum.
e. The paralysis is of acute, spastic, ascending type

304. Weil Felix reaction is not useful in which of the following: (PGI)
a. Endemic typhus
b. Trench fever
c. Epidemic typhus
d. Q fever
e. Brill-Zinsser disease

305. Cat Scratch disease is associated with: (PGI)


a. Positive Frei test
b. Pathognomonic histological feature
c. Regional lymphadenopathy
d. Infection with Bartonella quintana infection
e. Oculoglandular syndrome of Parinaud commonly

306. XDR-TB is Mycobacterium tuberculosis that is resistant to:


a. Isoniazid and rifampicin
b. Isoniazid, rifampicin and pyrazinamide
c. Isoniazid, rifampicin, pyrazinamide any fluoroquinolone and streptomycin
d. Isoniazid, rifampicin and any fluoroquinolone
e. Isoniazid, rifampicin, any fluoroquinolone and kanamycin

307. DOTS strategy of WHO is an acronym, which stands for:


a. Directly observed treatment strategy
b. Directly observed therapy strategy
c. Directly observed therapy short course
d. Directly observed treatment short course
e. Directly observed tuberculosis strategy

308. 7 month-old infant presents with failure to thrive, progressive muscular


weakness, and poor head control. On questioning, the mother states that
she typically feeds the baby soy-based formula sweetened with honey.
Which of the following organisms is most likely to be responsible for the
child’s presentation?
a. Clostridium botulinum
b. Clostridium difficile
c. Clostridium perfringens
d. Clostridium tetani

309. Gene in Vancomycin resistant enterococci shows following features: (PGI)


a. It imparts high-level resistance to vancomycin
b. It is not inducible
c. Resistance due to van A is not transferable.
d. It acts by synthesis of different peptidoglycan.
e. The isolates show low-level resistance to teicoplanin.

310. One of three blood culture bottles drawn from a patient with unexplained
fevers reveals gram-positive cocci growing in clusters. Which of the
following tests would be most useful in determining whether this organism
is a part of the normal skin flora?
a. Bacitracin resistance
b. Catalase
c. Coagulase
d. Novobiocin resistance

311. Bacterial genome has been completely recognized for which one of the
following agents:
a. H. pylori
b. Yersinia enterocolitis
c. Campylobacter jejuni
d. Streptococcus
303. 304. 305. 306. e 307. d 308. a 309. a, 310. c 311. a
b,c,d b,d,e b,c d

312. A diabetic patient developed cellulitis due to S. aureus, which was found
to be methicillin resistant on the antibiotic sensitivity testing. All the
following antibiotics will be appropriate except:
a. Vancomycin
b. Imipenem
c. Teicoplanin
d. Linezolid

313. Staphylococcus aureus differs from staphylococcus epidermidis by:


a. Is coagulase positive
b. Forms white colonies
c. A common cause of UTI
d. Causes endocarditis of prosthetic valve

314. Which of the following statements is most correct regarding resistance


to methicillin in MRSA:
a. Resistance is produced as a result of alteration in penicillin binding protein
b. Resistance is produced by production of β lactamase
c. Resistance is mediated by plasmids
d. Expression of resistance is enhanced by incubating at 38°C during susceptibility
testing

315. All of the following statement are true about Staphylococci except:
a. A majority of infection caused by coagulase (-) ve Staph, are due to
Staphylococcus epidermidis
b. β-Lactamase production is under plasmid control
c. Expression of methicillin resistance in Staphylococcus aureus increases when it is
incubated at 37°C on blood agar
d. Methicillin resistance in Staph, aureus is independent of β-Lactamase production

316. Which one of the following Gram positive organisms is the most common
cause of UTI among sexually active women:
a. Staphylococcus epidermidis
b. Staphylococcus aureus
c. Staphylococcus saprophyticus
d. Enterococcus

317. The following is a characteristic feature of Staphylococcal food poisoning


except:
a. Optimum temperature for toxin production is 37°C
b. Intradietic toxin are responsible for intestinal symptoms
c. Toxin can be destroyed by boiling for 30 minutes
d. Incubation period is 1-6 hours

318. A patient in an ICCU is on CVP line. His blood culture shows growth of
Gram-positive cocci which are catalase positive and coagulase negative. The
most likely etiological agent is:
a. Staphylococcus aureus
b. Staphylococcus epidermidis
c. Streptococcus pyogenes
d. Enterococcus faecalis

319. Staphylococcus aureus causes vomiting in 6-8 hours. The mechanism of


action is:
a. Stimulation of CAMP
b. Vagal stimulation
c. Stimulation of CGMP
d. Acts through ganglioside GM receptor

320. A cook prepares sandwiches for 10 people going for picnic. Eight out of
them develop severe gastroenteritis within 4-6 hrs of consumption of the
sandwiches. It is likely that on investigations the cook is found to be carrier
of:
a. Salmonella typhi
b. Vibrio cholerae
c. Entamoeba histolytica
d. Staphylococcus aureus

321. A child after consuming food in a party complains of diarrhea within 1-5
hours. The diagnosis is:
a. S. aureus
b. Streptococcus
c. Clostridium perfringens
d. Clostridium botulinum

312. b 313. a 314. a 315. c 316. c 317. c 318. b 319. b

320. d 321. a

322. Synergohymenotropic toxin of staphylococci consist of: (PGI)


a. α toxin
b. β toxin
c. g toxin
d. δ toxin
e. Panton valentine toxin

323. Which of the following organisms is implicated in the causation of


botryomycosis:
a. Staphylococcus aureus
b. Staphylococcus albus
c. Pseudomonas aeruginosa
d. Streptococcus pneumonia Streptococcus pyogenes

324. Staphylococcus in stool occurs in:


a. Staphylococcal food poisoning
b. Ischiorectal abscess
c. Toxic shock syndrome
d. May be a normal finding
e. Pseudomembranous colitis

325. Staphylococcus can cause: (PGI)


a. Ecthyma
b. Erythrasma
c. Furuncle
d. Impetigo contagiosa
e. Sycosis barbae

326. A 25-year-old IV drug abuser presents with fever for 3 weeks. ECHO
shows tricuspid vegetation. The most likely organism associated with
endocarditis in this case is:
a. Staphylococcus aureus
b. Candida albicans
c. Pseudomonas
d. Viridans streptococci

327. All of the following statements about Staphylococcus aureus are true
except
a. Most common source of infection is cross infection from infected people
b. About 30% of general population is healthy nasal carriers
c. Epidermolysin and TSS toxin are super antigens
d. Methicillin Resistance is chromosomally mediated

328. Tropical pyomyositis is caused by


a. Streptococcus
b. Staphylococcus aureus
c. Pseudomonas
d. Pneumococcus

329. The antibody marker in serum for staphylococcal endocarditis is:


a. ASLO
b. Anti-teichoic acid
c. Anti-lipopolysaccharide
d. Anti-M-protein

330. Methicillin resistance in S. aureus is due to:


a. p-lactamase
b. mecA gene
c. AMPC gene
d. Porin develop

331. Cutaneous manifestation of Staphylococci are all except:


a. Furuncle
b. Follicular impetigo
c. TSS
d. TEN

332. Acute haematogenous osteomyelitis is commonly caused by:


a. S. aureus
b. Streptococcus pneumoniae
c. E. coli
d. Pneumococcus

333. Staphylococcal toxic syndrome is due to: (PGI)


a. Enterotoxin A
b. Enterotoxin B
c. Enterotoxin C
d. Enterotoxin D
e. Enterotoxin F

334. Staphylococcus aureus remains in the skin for longer period because of:
a. Catalase
b. Coagulase
c. Hyaluronidase
d. None
322. 323. 324. 325. 326. 327. 328. 329. b 330. b 331. c
c,e a a c,d,e a a b

332. 333. 334.


a b,c,e d

335. Postoperative parotitis is caused by:


a. Staphylococcus aureus
b. Streptococcus
c. E. coli
d. Pneumococcus

336. Hot cold phenomenon is seen due to which toxin of staphylococci:


a. Alpha lysin
b. Beta lysin
c. Gamma lysin
d. Theta lysin

337. Staphylococcus differs from Streptococcus by:


a. Coagulase test
b. Catalase test
c. Phosphatase
d. Gram negative

338. A 30 years old female is on antibiotics with prolonged IV cannulation, has


spike of fever, the likely cause is:
a. Pseudomonas aeruginosa
b. Coagulase negative staphylococcus
c. Streptococcus agalactiae
d. E. coli

339. Catalase positive coagulase negative gamma-haemolytic bacteria


a. Streptococcus pyrogens
b. Staphylococcus aureus
c. Coagulase negative staphylococcus
d. Enterococci

340. A cook is habitual of nose picking while cooking, his clients are at risk for
food poisoning with
a. Clostridia difficile
b. Staphylococcus aureus
c. Vibrio cholerae
d. Bacillus cereus

341. All cause Fournier gangrene except:


a. Staphylococcus
b. Streptococcus
c. Clostridium
d. Bacteroides

342. Eight months after prosthetic valve replacement, most common


organism causing infective endocarditis is:
a. Staphylococcus epidermidis
b. Viridans streptococci
c. Staphylococcus aureus
d. HACEK

343. A person from village is complaining of development of pustules. Extract


from pus has shown gram positive cocci, showing hemolysis, catalase -ve,
identified as a group of streptococci. Following test is used:
a. Bacitracin sensitivity
b. Novobiocin sensitivity
c. Optochin sensitivity
d. Hemolysis

344. True statement about antistreptolysin ‘O’ titre is:


a. In normal people the titre is <200
b. In acute glomerulonephritis, the titre is low
c. ASO titre >200 indicate rheumatic fever
d. Streptozyme test is an active hemagglutination test

345. The commonest organism causing cellulitis is:


a. Streptococcus pyogenes
b. Streptococcus faecalis
c. Viridans streptococci
d. Microaerophilic streptococci

346. 45 years old Ramlal has intra-abdominal sepsis. The causative organism
was found to be vancomycin, gentamycin and ampicillin resistant. It grows
well in presence of 6.5% NaCl and arginine. Bile esculin hydrolysis is
positive. Which of the following is this organism?
a. Streptococcus agalactiae
b. Enterococcus fecalis
c. Streptococcus bovis
d. Streptococcus pneumoniae
335. 336. 337. 338. 339. c 340. 341. c 342. a 343. a 344. b
a b b b b

345. 346.
a b

347. An infant had high grade fever and respiratory distress at the time of
presentation to the emergency room. The sample collected for blood culture
was subsequently positive showing growth of alpha-hemolytic colonies. On
Gram staining these were gram-positive cocci. In the screening test for
identification, the suspected pathogen is likely to be susceptible to the
following agent:
a. Bacitracin
b. Novobiocin
c. Optochin
d. Oxacillin
348. Draughtsman colonies are seen in:
a. Staphylococcus
b. Salmonella
c. Pneumococcus
d. Corynebacterium

349. Which of the following causes dental carries?


a. Viridans streptococci
b. S. salivarius
c. S. mutans
d. S. agalactiae

350. Quellung reaction is due to


a. Capsular swelling
b. Mitochondrial swelling
c. RBC swelling
d. Ribosomal swelling

351. Most common cause of Post splenectomy infections:


a. Pneumococcus
b. Gonococcus
c. Staphylococcus
d. Corynebacterium

352. Differentiation of Neisseria gonorrhoeae and Neisseria meningitidis is by:


a. Glucose fermentation
b. Maltose fermentation
c. V. P. reaction
d. Indole test

353. Neisseria infections are associated with:


a. Deficiency of early complements
b. Deficiency of late complements
c. There is not such association
d. Any deficiency can be associated

354. Meningococci differ from gonococci in that they


a. Are intra-cellular
b. Possess a capsule
c. Cause fermentation of glucose
d. Are oxidase positive

355. Waterhouse-Friderichsen syndrome is seen in:


a. Pneumococcus
b. N. meningitidis
c. Pseudomonas
d. Yersinia

356. Gastrointestinal enteritis necroticans caused by:


a. C. difficile
b. C. perfringens
c. C. botulinum
d. C. jejuni
e. Pseudomonas

357. Which species of Clostridium does not cause gangrene?


a. C. perfringens
b. C. histolyticum
c. C. novyii
d. C. sporogenes

358. The most effective way of preventing tetanus is:


a. Surgical debridement and toilet
b. Hyperbaric oxygen
c. Antibiotics
d. Tetanus toxoid

359. A patient of acute lymphocytic leukemia with fever and neutropenia


develops diarrhea after administration of amoxicillin therapy, which of the
following organisms is most likely to be the causative agent?
a. Salmonella typhi
b. Clostridium difficile
c. Clostridium perfringens
d. Shigella flexneri
347. c 348. c 349. c 350. 351. 352. 353. 354. b 355. b 356. b
a a b b

357. 358. 359.


d d b

360. Sub terminal spores are found in: (PGI)


a. Clostridium sordelli
b. Clostridium sporogenes
c. Clostridium difficile
d. Clostridium tertium
e. Clostridium botulinum

361. Clostridium group causes all except:


a. Acute oesophagitis
b. Diarrhea
c. Local necrosis
d. Antibiotic associated changes
362. Among the toxins produced by C. botulinum, the non- neurotoxic one is:
a. A
b. B
c. C1
d. C2
e. D

363. Which type of Clostridium tetani has no flagella?


a. Type 1
b. Type 2
c. Type 4
d. Type 6

364. Oval bulging terminal spores seen in:


a. C. tertium
b. C. welchii
c. C. perfringens
d. C. histolyticum

365. Non-motile Clostridia is:


a. C. perfringens
b. C. novyi
c. C. botulism
d. CI. difficile

366. Most common organism responsible for gas gangrene is:


a. Clostridium perfringens
b. Clostridium difficile
c. Clostridium tetani
d. Clostridium botulinum

367. All occur in botulism except


a. Diplopia
b. Diarrhoea
c. Dysphagia
d. Dysarthria

368. Boxcar appearance is seen in:


a. Clostridium perfringens
b. Streptococci
c. Toxoplasma
d. Treponema pallidum

369. Tetanospasmin encoding genes are located on:


a. Chromosome
b. Plasmid
c. Both
d. Transposon

370. Nagler reaction is shown by:


a. C. Difficile
b. C. perfringens
c. C. Tetani
d. C. Butyricum

371. Mechanism of action of Tetanospasmin:


a. Inhibition of GABA release
b. Inhibition cAMP
c. Inactivation of ACh receptors
d. Inhibition of cGMP

372. Cause of Clostridium difficile associated diarrhea.


a. Trauma
b. Dairy products
c. Fried rice
d. Antibiotic use

373. Positive Schick’s test indicates that person is:


a. Immune to diphtheria
b. Hypersensitive to diphtheria
c. Susceptible to diphtheria
d. Carrier of diphtheria

374. Ehrlich phenomenon is seen in:


a. Mycobacterium TB
b. Proteus
c. Staphylococcus
d. Corynebacterium
e. Mycoplasma

375. Literally leather term is used for:


a. Anthrax
b. Proteus
c. Diphtheria
d. Neisseria

376. True about diphtheria is:


a. Cause cranial nerve palsies in 2nd and 3rd week
b. Treatment with erythromycin
c. It is gram-negative organism
d. Passive immunization is harmful and should not be tried
360. 361. 362. 363. 364. 365. 366. 367. b 368. a 369. b
a,b,e a d d a a a

370. 371. 372. 373. c 374. 375. c 376.


b a d d a

377. Metachromatic granules are stained by:


a. Ponder’s stain
b. Negative stain
c. Gram’s stain
d. Leishman’s stain

378. Toxigenicity of C. diphtheria is determined by:


a. Elek’s gel ppt test
b. Ascoli’s reaction
c. Nagler’s reaction
d. All

379. Investigation of choice of diphtheria carrier is:


a. Throat swab culture
b. Gram’s stain
c. Albert’s stain
d. Ziehl-Neelsen stain

380. Selective medium for Corynebacterium diptheriae is:


a. Loffler’s serum
b. McConkey’s medium
c. Cysteine-tellurite agar
d. LJ medium

381. C. diphtheriae is also called as:


a. Koch’s bacillus
b. Roux bacillus
c. Klebs-Löffler bacillus
d. Yersin bacillus

382. Erythrasma is caused by:


a. Corynebacterium minutissimum
b. C. diphtheriae
c. C. ulcerens
d. C. vaginale

383. Schick test is for:


a. Susceptibility to diphtheria
b. Susceptibility to tetanus
c. Susceptibility to scarier fever
d. Susceptibility to RF

384. Characteristic of Bacillus cereus food poisoning is:


a. Presence of fever
b. Presence of abdominal pain
c. Presence of vomiting
d. Absence of diarrhoea

385. Noninvasive diarrhea can be caused by the following?


a. Shigella
b. B. cereus
c. Salmonella
d. Y. enterocolitica

386. Malignant pustule is caused by:


a. B. anthracis
b. Leishmania
c. Basal cell carcinoma
d. Pyoderma

387. An abattoir worker presented with a malignant pustule on his hand that
progressed to form an ulcer. Smear was taken from the ulcer and sent to
laboratory for investigation. The diagnosis is:
a. Cutaneous anthrax
b. Carbuncle
c. Ulcerating melanoma
d. Infected rodent ulcer

388. Characteristic infection of Nocardia asteroides is:


a. Diarrhea
b. Secondary
c. dissemination to liver
d. Brain abscess

389. A clinical specimen was obtained from the wound of a patient diagnosed
as Nocardiosis. For the selective isolation of Nocardia sp. which one of the
following would be the best method:
a. Paraffin bait technique
b. Castaneda’s culture method
c. Craigie’s culture method
d. Hair bait technique

390. True about Madura foot? (PGI)


a. Can erode bones
b. Spread to lymph nodes
c. Most commonly occur in hand
d. Slow growing
e. Antibiotic has no role
377. 378. 379. 380. c 381. c 382. 383. 384. b 385. b 386. a
a a a a a

387. 388. 389. 390.


a d a a,d

391. A wool cutter is suffering from fever with a cervical lymph node
enlargement for last 15 days, most likely he is suffering from:
a. Anthrax
b. Mycetoma
c. Sporotrichosis
d. Coccidiomycoses

392. Medusa head appearance:


a. Bacillus anthracis
b. Bacillus subtilis
c. Bacillus cereus
d. Bacillus licheniformis

393. A discharging sinuses seen in:


a. Sporotrichosis
b. Cryptococcosis
c. Histoplasmosis
d. Mycetoma

394. Most common cause of actinomycetoma in India:


a. Nocardia braziliensis
b. Actinomadura madure
c. Piedra
d. Tinea cruris

395. Actinomycosis most common site:


a. Cervicofacial
b. Thorax
c. Abdomen
d. Brain

396. A 28 years old lady presented with, headache, kernigs sign positive,
culture showed gram positive bacilli, most probable organism is:
a. Listeria monocytogenes
b. H. influenzae
c. Meningococci
d. Streptococcus pneumoniae

397. Tumbling motility is shown as


a. Listeria monocytognes
b. Proteus Vulgaris
c. Borrelia
d. Clostridia

398. Basanti, 29 years aged female from Bihar presented with active TB. She
delivers baby. All of the following are indicated except:
a. Administer INH to the baby
b. Withhold breastfeeding
c. Give ATT to mother for 2 years
d. Ask mother to ensure proper disposal of sputum

399. Selective media for TB bacilli is: (PGI)


a. NNN media
b. Dorset media
c. LJ media
d. Nutrient agar

400. Leprosy affects all the following except:


a. Uterus
b. Ovaries
c. Eyes
d. Nerves

401. The following drug is not used for the treatment of type II lepra reaction:
a. Chloroquine
b. Thalidomide
c. Cyclosporine
d. Corticosteroids

402. Tuberculin test is:


a. Anaphylaxis mediated
b. Cell mediated
c. Antibody mediated
d. Immune complex mediated

403. H2S04 concentration to stain M. leprae is:


a. 1%
b. 5%
c. 10%
d. 20%

404. Typical mycobacterium is differentiated from atypical mycobacterium by:


a. Acid fast staining
b. Amidase test
c. Niacin test
d. Motility under microscope

405. BCG vaccine in HIV (+) newborn is:


a. Contraindicated
b. Double dilution
c. Half dilution
d. Dose double

406. Lepra bacilli are best cultivated in:


a. Guinea pig
b. Armadillos
c. Mouse foot pad
d. Nutrient agar
391. 392. 393. 394. 395. 396. 397. 398. b 399. 400. a
a a d b a a a b,c

401. c 402. 403. 404. 405. 406.


b b b a b

407. The best diagnostic procedure of M. tuberculosis:


a. PCR
b. Auramine rhodamine stain
c. Sputum culture
d. Sputum examination

408. Drug resistance in tuberculosis is due to:


a. Transformation
b. Transduction
c. Conjugation
d. Mutation

409. For experimental work, Lepra bacilli are best cultured in:
a. Armadillos
b. Mouse foot pad
c. Guinea pigs
d. Rabbit testes

410. Which of the following is not a pathogenic Mycobacterium?


a. M. kansasii
b. M. scrofulaceum
c. M. chelonei
d. M. smegmatis

411. Buruli ulcer is caused by:


a. Streptococcus
b. Spirillum minus
c. M. ulcerens
d. Brucella

412. Reactive tubercular arthritis:


a. Spina ventusa
b. Pott’s disease
c. Poncet’s disease
d. None

413. ENL is seen in:


a. Lepromatous leprosy
b. Tuberculoid leprosy
c. Indeterminate leprosy
d. Pure neuritic leprosy

414. With reference to infection with Escherichia coli the following are true
except:
a. Enteroaggregative E. coli is not associated with persistent diarrhea
b. Enterohemorrhagic E. coli cause hemolytic uremic syndrome
c. Entero-invasive E. coli produces a disease similar to salmonellosis
d. Entero toxigenic E. coli is a common cause of traveler’s diarrhea

415. All are true about EHEC except:


a. Sereny test is positive
b. Does not ferment sorbitol
c. Causes HUS
d. Elaborates Shiga like exotoxin

416. E. coli attached to a surface with the help of:


a. Fucose
b. Concanavalin
c. Phytohemagglutinin
d. Lectin

417. A 24 years old cook in a hostel suffered from enteric fever 2 years back.
The chronic carrier state in patient is diagnosed by:
a. VI agglutination test
b. Blood culture in brain heart infusion broth
c. Widal test
d. C. reactive protein

418. For typhoid endemic country like India, immunization of choice is:
a. TAB vaccine
b. Typhoral 21A oral vaccine
c. Monovalent vaccine
d. Any of these
419. True about salmonella gastroenteritis is/are: (PGI)
a. Mainly diagnosed by serological tests
b. Blood and mucous are present in stool
c. Caused by animal products
d. Symptoms appear between 4-48 hours
e. Features are mainly due to exotoxin

420. About Vi polysaccharide vaccine, true is:


a. Can be given in patients with yellow fever and hepatitis B
b. Has many contraindications
c. Has many serious systemic side effects
d. Has many serious local side effects
407. c 408. 409. 410. 411. c 412. c 413. 414. a 415. a 416. d
d a d a

417. 418. c 419. 420.


a c,d a

421. Enterobacteriaceae includes all except: (PGI)


a. Pseudomonas
b. Klebsiella
c. V. cholerae
d. Proteus

422. Maximum urease is produced by:


a. H. pylori
b. P. mirabilis
c. K. rhinoscleromatis
d. Ureaplasma

423. DT 104 strain belongs to which of the following bacteria:


a. Salmonella gallinarum
b. Salmonella typhi
c. Salmonella enteritidis
d. Salmonella paratyphi A

424. Which is true of Enterobacteriacae?


a. All are oxidase negative
b. Nitrate reduction negative
c. Glucose not fermented by all
d. Motility by bipolar flagellum

425. ’’Pea-soup stool” is characteristically seen in:


a. Cholera
b. Typhoid
c. Botulism diarrhea
d. Traveller’s diarrhea

426. Which subtype of E. Coli causes hemorrhagic colitis?


a. 0157:H7
b. 026:H1
c. Both of the above
d. None of the above

427. Incubation period of typhoid is:


a. 10-14 days
b. 2-6 days
c. 18-72 hours
d. 2-3 months

428. Microorganisms that enter freshly laid eggs are:


a. Salmonella
b. Brucella
c. Shigella
d. Vibrio cholerae

429. All of the following Salmonella are motile except:


a. S. typhi
b. S. enteritidis
c. S.gallinarum
d. S.chester

430. Non-lactose fermenter includes all the following except:


a. Shigella sonnei
b. Shigella dysenteriae
c. Shigella flexneri
d. Shigella boydii

431. Shigella are be divided into sub group on the basis of ability to ferment:
a. Lactose
b. Maltose
c. Fructose
d. Mannitol

432. A person returns to Delhi from Bangladesh after 2 days and has diarrhea.
Stool examination shows RBCs in stool. The likely organism causing is:
a. Enteropathogenic E. coli
b. Enterotoxigenic E. coli
c. Salmonella typhi
d. Shigella dysenteriae

433. E coli labile toxin acts by:


a. Increasing cAMP
b. Decreasing cAMP
c. Decreasing protein synthesis
d. Decreasing cGMP

434. Which toxin is mediated by c-AMP except:


a. V. cholera 01
b. Heat stable E. coli toxin
c. Heat labile E. coli toxin
d. V. cholerae

435. E. coli gives pink color with:


a. Chocolate agar
b. L J medium
c. MacConkey’s medium
d. Saline broth

436. Preformed toxin is important in food poisoning due to all except


a. S.aureus
b. Clostridium botulinum
c. ETEC
d. B. cereus
421. 422. 423. 424. 425. 426. 427. 428. a 429. c 430. a
a,c a b a b a a

431. 432. 433. 434. 435. c 436. c


d d a b

437. In E. coli true is


a. ETEC is invasive
b. EPEC acts via cAMP
c. P- pili present in uropathogenic type
d. ETEC cause HUS

438. A child with fever with RBCs and pus in stool, causative organism is
a. ETEC
b. EHEC
c. EPEC
d. EAEC

439. In donovanosis:
a. Pseudo lymphadenopathy
b. Penicillin is used for treatment
c. Painful ulcer
d. Suppurative lymphadenopathy
440. Widal test is based on:
a. Complement fixation
b. Agglutination
c. Both
d. None

441. Recommended transport medium for stool specimen suspected to contain


enteric pathogens is:
a. Amie’s medium
b. Buffered glycerol saline medium
c. MacConkey medium
d. Blood agar

442. All of the following are true about V. cholerae 0139 except:
a. Clinical manifestations are similar to O1 Eltor
b. First discovered in Chennai
c. Produces 01 lipopolysaccharide
d. Epidemiologically indistinguishable from O1 Eltor

443. In the small intestine, cholera toxin acts by:


a. ADP ribosylation of G regulatory protein
b. Inhibition of adenyl cyclase
c. Activation of GTPase
d. Active absorption of NaCl

444. The effect of cholera toxin is mediated via the stimulation of following
second messenger:
a. cAMP
b. GMP
c. Calcium-calmodulin
d. Acetylcholine

445. The best suited medium for Vibrio cholerae is:


a. Thayer martin
b. TCBS medium
c. Skirrow’s medium
d. Loffler’s medium

446. A 32 years old male, Kalu who recently visited a sea coast presented with
ulcer over the left leg. The probable cause is:
a. Pasteurella multocida
b. Micrococcus halophillus
c. Vibrio vulnificus
d. Neisseria gonorrhea

447. Strain of V. cholerae in Bengal


a. 0:037
b. 0:139
c. 0:17
d. 0:40
e. 0:149

448. Which of the following has shortest incubation period?


a. Plague
b. Cholera
c. Measles
d. Typhoid

449. True about cholera vibrio is:


a. Can tolerate wide range of alkaline pH
b. Nonmotile bacilli
c. Cannot be grown in media
d. NaCl stimulates growth

450. Transport medium for cholera:


a. LJ medium
b. Cary Blair medium
c. MYPA medium
d. Stewart medium

451. True about cholera: (PGI)


a. Gram negative rod
b. Associated with fever
c. Causes painful watery diarrhea
d. It is an achlorhydria which renders an individual susceptible to disease.

437. c 438. 439. 440. 441. 442. c 443. 444. a 445. b 446. c
b a b b a

447. 448. 449. 450. 451.


b b a b a,d

452. The following statements are true regarding Melioidosis except:


a. It is caused by Burkholderia mallei
b. The agent is a gram-negative aerobic bacteria
c. Bipolar staining of etiological agent is seen with methylene blue
d. The most common form of Meliodosis is pulmonary infection

453. A 50-year-old chronic alcoholic male agriculture worker presented with


high grade fever of one week with spells of chills and rigor. Examination of
the respiratory system reveals bilateral crepitation with scattered rhonchi.
Multiple subcutaneous nodules were found on extensor surface of left
forearm and left leg. Direct microscopy of the pus aspirate from the nodules
reveals plenty of gram(-)ve bacilli with bipolar staining. Culture reveals
distinct rough corrugated grey white colonies on blood agar. The organisms
were motile and oxidase-positive. The most likely diagnosis is:
a. Plague
b. Melioidosis
c. Bartonellosis
d. Actinomycosis

454. True about Y. pestis: (PGI)


a. Gram +ve
b. Gram -ve
c. Motile
d. Non-motile
e. It is coccobacilli

455. A farmer presents to the emergency department with painful inguinal


lymphadenopathy and history of fever and flu like symptoms. Clinical
examination reveals on ulcer in the leg. Which of the following stain should
be used to detect suspected bipolar stained organism.
a. Alberts stain
b. Ziehl Neelsen stain
c. Wayson’s stain
d. McFadyen’s stain

456. Blue pus found in infection:


a. Staph
b. Streptococcus
c. Pseudomonas
d. Mycoplasma

457. Glanders is caused by:


a. Protozoa
b. Virus
c. Fungi
d. Bacteria

458. Pyocyanin is formed by:


a. Yersinia
b. Pseudomonas
c. Burkholderia
d. Pasteurella

459. Pseudomonas toxin acts by:


a. increased cAMP
b. decreased cAMP
c. inhibition of Protein synthesis
d. increased cGMP
460. In India outbreak of plague occurred in all the following states except
a. Gujarat
b. Maharashtra
c. Kerala
d. MP

461. Ecthyma gangrenosum is caused by:


a. Pseudomonas
b. Streptococcus
c. Staphylococcus
d. H. influenza

462. False about H. influenzae:


a. It needs X and V factor for growth
b. Protein capsule plays an important role in pathogenicity
c. It is a rare cause of meningitis in the first 2 months of life.
d. Most common invasive manifestation is meningitis

463. The following are true for Bordetella pertussis except:


a. It is a strict human pathogen
b. It can be cultured from the patient during catarrhal stage
c. It leads to invasion of the respiratory mucosa
d. Infection can be prevented by a acellular vaccine
452. 453. 454. 455. c 456. c 457. 458. 459. c 460. c 461. a
a b b,d,e d b

462. 463.
b d

464. The usual incubation period of pertussis is:


a. 7-14 days
b. 3-5 days
c. 21-25 days
d. Less than 3 days

465. True regarding pertussis vaccine is:


a. 95% of vaccinated are protected
b. Erythromycin should be given to contacts
c. Neuroparalytic complication is seen in 1 in 15000
d. Leukocytosis is diagnostic

466. In which of the following organism the capsule does not act as a
virulence factor?
a. H. influenzae
b. Streptococcus pneumoniae
c. N. meningitidis
d. Bordetella pertussis

467. Acellular pertussis vaccine contains:


a. Pertactin, flagellar hemagglutinin, cytotoxin, endotoxin
b. Pertactin, flagellar hemagglutinin, fimbriae, endotoxin
c. Pertactin cytotoxin, fimbriae
d. Filamentous hemagglutinin, pertussis toxin

468. A veterinary doctor had pyrexia of unknown origin. His blood culture in
special laboratory media was positive for gram-negative short bacilli which
was oxidase positive. Which one of the following is the likely organism
grown in culture?
a. Pasteurella spp.
b. Francisella spp.
c. Bartonella spp.
d. Brucella spp.

469. A farmer presenting with fever off and on for the past 4 years was
diagnosed to be suffering from chronic Brucellosis. All of the following
serological tests would be helpful in the diagnosis at this state except:
a. Standard agglutination test
b. 2-mercapto-ethanoI test
c. Complement fixation test
d. Coombs’ test

470. Malta fever is caused by:


a. Borrelia burgdorferi
b. Brucella melitensis
c. Pseudomonas
d. Leigionella

471. A farmer rearing sheep, presented with complaints of fever and


weakness for the last one month. There is generalized lymphadenopathy.
There was also associated hepatomegaly Biopsy of liver showed non-
caseating granuloma. There are most likely due to infection with:
a. Yersinia pestis
b. Brucella canis
c. Francisella tularensis
d. Brucella melitensis

472. Regarding Brucellosis, all of these are true except:


a. Man to man transmission
b. A zoonosis
c. Blood cultures used in diagnosis
d. Transmitted via animal products

473. All are true about brucella except:


a. B. abortus is capnophilic
b. Transmitted by aerosol can occur occasionally
c. Pasteurization destroys it
d. 2 ME is used to detect IgA

474. Milk ring test is for:


a. Brucellosis
b. Bacteroides
c. Tuberculosis
d. Salmonellosis

475. Hemophilus ducreyi causes


a. Lymphogranuloma venereum
b. Chancroid
c. Syphilis
d. Psittacosis

476. Satellitism is seen in culture of


a. Hemophilus
b. Streptococcus
c. Klebsiella
d. Proteus
e. Salmonella
464. 465. 466. 467. 468. 469. 470. 471. d 472. a 473. d
a b d d d a b

474. 475. 476.


a b a

477. All of the following are virulence factors of pertussis, except:


a. Tracheal cytotoxin
b. Pertussis toxin
c. Capsule
d. Pertactin

478. All are Campylobacter culture media except:


a. Butzler’s
b. CVA medium
c. Regan-Lowe medium
d. Skirrow’s medium
e. Campylobacter blood agar

479. All are true statements about Campylobacter jejuni, except


a. Human is the only reservoir
b. Can cause GB syndrome
c. Poultry is the source of infection
d. Common cause of Campylobacteriosis

480. A 35 years old patient complaint of abdominal cramps along with profuse
diarrhea. Treating physician wants to process the stool specimen for
isolation of Campylobacter jejuni. Which of the following is method of choice
for culture of stool?
a. Culture on TCBS media incubated at 37 °C on aerobic condition
b. Culture on Skirrow’s medium incubated at 42°C under microaerophilic condition
c. Culture on Mac Conkeys medium incubated at 42°C under anaerobic condition
d. Culture on Wilson and Blair’s medium at 37°C under microaerophilic condition

481. Helicobacter pylori is not associated with:


a. Gastrointestinal lymphoma
b. Gastric cancer
c. Gastric leiomyoma
d. Peptic ulcer

482. Which of following is false regarding H. pylori?


a. With chronic infection urea breath test become negative
b. H. pylori infection remains lifelong if untreated
c. Endoscopy is diagnostic
d. Toxigenic strains usually cause ulcer

483. True about H. pylori:


a. Seen in 85 to 90% cases of gastric ulcer
b. Seen in 20 to 25% cases of duodenal ulcer
c. Transmitted from man to man, Feco-orally and by oro- gastric route
d. Common in adults of developing countries

484. Microaerophilic bacteria:


a. Campylobacter
b. Vibrio cholera
c. Pseudomonas
d. Salmonella

485. Which of the following organism is strongly urease positive?


a. H. pylori
b. Proteus
c. Bordetella pertussis
d. Brucella

486. All of the following are true about Helicobacter pylori except:
a. About 50% of world population affected
b. 85% of population is affected, in some developing countries
c. All children in developing countries have immunity by five years of age
d. Infection is common in low socio-economic status
487. True about Helicobacter pylori is:
a. Culture and gram staining of biopsy is the gold standard investigation
b. Controlled urea breath is negative with massive infection
c. Anti-urease antibody is produced only by invasive strains
d. Urease activity provides protective environment to the bacilli
477. c 478. c 479. 480. 481. c 482. 483. c 484. a 485. a 486. c
a b a

487.
d

488. The most sensitive test for H. pylori is:


a. Breath test
b. Rapid urease test
c. Culture of biopsy
d. Microscopy of biopsy

489. H. pylori causes all except:


a. Gastric ulcer
b. Duodenal ulcer
c. Gastric cancer
d. Colorectal cancer

490. Which of the following is a selective media for Campylobacter?


a. MacConkeys Medium
b. Skirrow’s medium
c. Buffered charcoal yeast agar
d. Chocolate agar

491. True about H. pylori includes all except:


a. Gram positive bacillus
b. Urease positive bacillus
c. Highly associated with duodenal ulcer
d. Urea breath test can be performed only in specialized labs

492. True about Campylobacter jejuni:


a. Exclusively found in jejunum only
b. Shows darting motility by peritrichous flagella
c. Can grow at temperature around 42°C
d. Common cause of diarrhea in India

493. Seven sheathed flagella is seen in:


a. V. cholerae
b. H. pylori
c. P. aeruginosa
d. Spirochetes
494. A 70 years old patient presents with high grade fever, dry cough and
abdominal pain. Sputum sample collected from patient shows gram negative
organisms that are able to grow only on charcoal yeast extract medium. The
most likely organism is:
a. H. influenzae
b. Legionella
c. Listeria monocytogenes
d. M. catarrhalis

495. A 60-year old man is diagnosed to be suffering from Legionnaires disease


after he returns home from attending a convention. He could have acquired
it:
a. From a person suffering from the infection while traveling in the aeroplane
b. From a chronic carrier in the convention center
c. From inhalation of the aerosol in the air-conditioned room at convention center
d. By sharing an infected towel with a fellow delegate at the convention

496. Anju, a 28 years female, has diarrhea, confusion, high grade fever with
bilateral pneumonitis. Organism causing this is:
a. Legionella
b. Neisseria meningitidis
c. Streptococcus pneumoniae
d. H. influenzae

497. All of the following are true regarding Legionella except:


a. Legionella can be grown on complex media
b. L. pneumophilia serogroup 1 is the most common sero- group isolated from
humans
c. Legionella are communicable from infected patient to others
d. L. pneumophila is not effectively killed by polymorphonuclear leukocyte

498. Pontiac fever is caused by:


a. Listeria Monocytogenes
b. S. aureus
c. CI. tetani
d. Legionella pneumophila
e. Borrelia recurrentis
488. 489. 490. 491. 492. c 493. 494. 495. c 496. a 497. c
a d b a b b

498.
d

499. An elderly patient presented with fever, chest pain and dry cough.
Sputum cultured on charcoal yeast medium, the most likely organism is:
a. H. influenzae
b. Moraxella catarrhalis
c. Legionella
d. Burkholderia cepacia

500. Aerosol spread leading to epidemics is seen in infection with:


a. Legionella
b. Hemophilus
c. Influenza virus
d. Mycoplasma

501. Buffered charcoal yeast agar is the selective medium for:


a. Listeria monocytogenes
b. Legionella pneumophila
c. Pseudomonas aeruginosa
d. T. pallidum

502. Legionnaire disease is caused by:


a. Motile gram positive
b. Motile gram negative
c. Non-motile gram positive
d. Non-motile gram negative

503. Lice are not the vectors of:


a. Relapsing fever
b. Q fever
c. Trench fever
d. Epidemic typhus

504. All are true about scrub typhus, except:


a. Causative organism is R.tsusugamushi
b. Vector is trombiculid mite
c. Adult female feeds on vertebrate hosts
d. Tetracycline is the drug of choice

505. A patient complained of chills and fever following a louse bite 2 week
before. He had rashes all over the body and was delirious at the time of
presentation to the hospital and subsequently went into coma. A provisional
diagnosis of vasculitis due to Rickettsial infection was made. Which one of
the following can be the causative agent:
a. Rickettsia typhi
b. Rickettsia rickettsiae
c. Rickettsia prowazekii
d. Rickettsia akari

506. Scrub typhus is transmitted by:


a. Reduviid bug
b. Trombiculid mite
c. Enteric pathogens
d. Cyclops

507. It is true regarding endemic typhus that:


a. Man is the only reservoir of infection
b. Flea is a vector of the disease
c. The rash developing into eschar is a characteristic presentation
d. Culture of the etiological agent in tissue culture is a diagnostic

508. A man presents with fever, chills 2 weeks after a louse bite. There was
maculopapular rash on the trunk which spread peripherally. The cause of
this infection can be:
a. Scrub typhus
b. Endemic typhus
c. Rickettsial pox
d. Epidemic typhus

509. Disease caused by Orientia is transmitted by:


a. Rat flea
b. Tick
c. Louse
d. Trombiculid mite
e. Gamaxid mite

510. Tick is vector for: (PGI)


a. Crimean Congo fever
b. Rocky mountain spotted fever
c. Epidemic typhus
d. Endemic typhus
e. Scrub typhus
499. c 500. 501. 502. 503. 504. c 505. c 506. b 507. b 508. d
a b b b

509. 510.
d a,b

511. In a patient with UTI; on smear, no bacteria are found on gram stain with
abundant pus cells, to demonstrate organism, which of the following is
useful:
a. McCoy cell line
b. Thayer martin medium
c. L. J. medium
d. Acid fast staining

512. Which of the following is not true regarding Chlamydia?


a. Has biphasic life
b. Elementary body is metabolically active
c. Reticulate body undergoes binary fission
d. Once it invades into cell it abates phagoloysosomal fusion

513. Which one of the following statements is true regarding Chlamydia


pneumoniae?
a. Fifteen serovars have been identified as human pathogen
b. Mode of transmission is by the air-borne bird excreta
c. The cytoplasmic inclusions presents in the sputum specimen are rich in glycogen
d. The group specific antigen is responsible for the production of complement fixing
antibodies

514. A 45 years female complains of lower abdominal pain and vaginal


discharge. On examination there is cervicitis along with a mucopurulent
cervical discharge. The gram smear of the discharge shows presence of
abundant pus cells but no bacteria. The best approach to isolate the
possible causative agent would be:
a. Culture on chocolate agar supplemented with Hemin
b. Culture on McCoy cells
c. Culture on a bilayer human blood agar
d. Culture on Vero cell lines

515. A man presents to STD clinic with urethritis and urethral discharge. Gram
stain shows numerous pus cells but no microorganism. Culture is negative
on routine laboratory media. The most likely agent is:
a. Chlamydia trachomatis
b. H. ducreyi
c. T. pallidum
d. N. gonorrhoeae

516. Chlamydia trachomatis serovars D-K cause:


a. Arteriosclerosis
b. Trachoma
c. Lymphogranuloma venereum
d. Urethritis

517. Chlamydia causes: (PGI)


a. Infertility
b. Pneumothorax
c. Pelvic inflammatory disease
d. Congenital malformation in fetus

518. Isolation of Chlamydia from tissue specimen can be done by:


a. ELISA (Enzyme Linked Immune Assay)
b. Yolk sac inoculation
c. Direct Immunofluorescence Antibody test (DFA)
d. Polymerase Chain Reaction (PCR)
519. A male patient with symptoms of urethritis. Examination reveals only pus
cells without any organism. Most likely cause is:
a. Chlamydia trachomatis
b. H. ducreyi
c. Treponema pallidum
d. M. tuberculosis

520. Most sensitive test for diagnosing chlamydia:


a. Gram’s staining
b. Nucleic acid amplification testing
c. ELISA
d. Fluorescent microscopy

521. The primary site of multiplication of Rickettsial organisms is in the:


a. Parenchymal cells of the liver
b. Endothelial cells of small vessels
c. Media of arteries
d. Adventitia of all blood vessels
511. 512. 513. 514. 515. 516. 517. 518. b 519. a 520. b
a b d b a d a,c

521.
b

522. Anthropozoonosis are all except:


(PGI)
a. Guinea worm infection
b. Rabies
c. Plague
d. Hydatid cyst

523. Chlamydia cause all of the following except:


a. Trachoma
b. Non-gonococcal urethritis
c. Pneumonia
d. Parotitis
e. Pancreatitis

524. Chlamydia trachomatis causes all except:


a. Ophthalmia neonatorum
b. Lymphogranuloma venereum
c. Q-fever
d. Trachoma

525. “Genital elephantiasis” is seen in


a. Rickettsia
b. Chancroid
c. Lymphogranuloma venereum
d. Syphilis

526. Neil Mooser’s reaction is given by:


a. Rickettsial infection
b. Chlamydial infection
c. Mycoplasma
d. Pneumococci infection

527. Microorganism that does not obey Koch’s postulates:


a. M. tuberculosis
b. Poliovirus
c. M. leprae
d. Streptococcus

528. Chlamydia trachomatis, the causative agent for trachoma:


a. Is a yeast
b. Is an intracellular organism
c. Forms extracellular bodies which are metabolically active
d. Is never demonstrable in conjunctival scrapings

529. Causative agent of Q fever:


a. R. Quintana
b. R. prowazekii
c. R. typhi
d. Coxiella burnetii

530. For all of the following reaction Weil Felix reaction is diagnostic except:
a. Endemic typhus
b. Scrub typhus
c. Epidemic typhus
d. Q fever

531. Endemic typhus is transmitted by:


a. Rat flea
b. Mite
c. Tick
d. None

532. Following grows in the cell free medium:


a. Rickettsiae
b. M. leprae
c. Bartonella
d. T. pallidum
533. Transovarian transmission is seen in:
a. Rickettsiae
b. Chlamydia
c. Both
d. None

534. Vascular endothelial infection is caused by:


a. Rickettsiae
b. Mycoplasma
c. Chlamydia
d. None

535. All of the following are true about Chlamydia except:


a. Gram positive
b. Causes trachoma
c. Causative organism of psittacosis
d. Are also called basophilic viruses

536. Obligatory intracellular organism is:


a. Mycoplasma
b. Chlamydia
c. Cryptococcus
d. H. pylori

537. Well Felix reaction for Scrub typhus shows positivity for:
a. OXK
b. OXK + OX19
c. OX-2
d. OX-19

538. Which of the following is used for Rickettsia:


a. Weil-Felix reaction
b. Rose-waller test
c. Paul-Bunnel test
d. VDRL

522. 523. 524. c 525. c 526. 527. c 528. 529. d 530. d 531. a
a,c e a b

532. c 533. 534. 535. 536. 537. 538.


a a a b a a

539. Inclusion body is seen in:


a. Rickettsiae
b. Chlamydia
c. Mycoplasma
d. H. pylori
540. False +ve VDRL is seen in:
a. Lepromatous leprosy
b. Infectious mononucleosis
c. HIV
d. Pregnancy
e. All of the above

541. Chancre redux’ is a clinical feature of:


a. Early relapsing syphilis
b. Late syphilis
c. Chancroid
d. Recurrent herpes simplex infection

542. Best indicator for monitoring of syphilis treatment:


a. VDRL
b. FTA-ABS
c. TPI
d. TPHA

543. A 25-year-old laborer presented 3 years back with penile ulcer which
remain untreated. Later he presented with neurological symptoms for which
he has taken appropriate treatment. Test to monitor response to treatment
is:
a. VDRL
b. TPI
c. FTA-Abs
d. ELISA

544. Non-venereal treponemes is/are: (PGI)


a. T. pertenue
b. T. carateum
c. T. pallidum
d. T. cuniculi

545. A bacterial disease with 3 ‘R’s i.e. rats, rice fields and rainfall is:
a. Leptospirosis
b. Plague
c. Melioidosis
d. Rodent bite fever

546. A sweeper involved with repair-work of sewers was admitted with fever,
jaundice and renal failure. The most appropriate test to diagnose infection
of this patients:
a. Weill-Felix test
b. Paul-Bunnell test
c. Microscopic agglutination test
d. Micro immunofluorescence test

547. The following statements are true regarding leptospirosis, except:


a. It is a zoonosis
b. Man is the dead end host
c. Man is an accidental host
d. Lice acts as reservoir of infection

548. 20-year-old boy has admitted with history of fever, icterus, conjunctival
suffusion and hematuria for 20 days. Which of the following serological test
can be of diagnostic utility:
a. Widal test
b. Microscopic agglutination test
c. Paul-Bunnell-test
d. Weil-Felix reaction

549. Not used for diagnosis of Leptospirosis:


a. Microscopic agglutination test
b. Dark field illumination
c. Macroscopic agglutination test
d. Weil Felix reaction

550. A 25-year-old farmer presented with history of high grade fever for 7
days and altered sensorium for 2 days. On examination, he was comatose
and had conjunctival hemorrhage. Urgent investigations showed a
hemoglobin of 11 gm/dl. Peripheral blood smear was negative for malarial
parasite. What is the most likely diagnosis?
a. Brucellosis
b. Weil’s disease
c. Acute viral hepatitis
d. Q fever

551. Lyme disease caused by:


a. Leptospira
b. Borrelia
c. Treponema
d. Bordetella
e. Arbovirus

552. Tick born relapsing fever is/are caused by: (PGI)


a. Borrelia recurrentis
b. Borrelia duttonii
c. Borrelia burgdorferi
d. Borrelia hermsii
e. None of above
539. 540. 541. 542. 543. 544. 545. 546. c 547. d 548. b
b e a a a a,b a

549. 550. 551. 552.


d b b b,d

553. Test used for diagnosis of congenital syphilis:


a. IgM FTA - Abs
b. VDRL
c. TPI
d. Kahn flocculation test

554. The following is not true of syphilis:


a. TPI is most specific
b. VDRL is not specific
c. VDRL is negative in secondary syphilis
d. IgM test is specific for congenital syphilis

555. The causative agent of Lyme’s disease:


a. Borrelia burgdorferi
b. Borrelia recurrentis
c. Leptospira icterohaemorrhagiae
d. Clostridium difficile

556. Organisms that has not been cultured successfully so far is:
a. Leptospira
b. Treponema pallidum
c. Bordetella
d. Staphylococcus

557. Yaws is caused by:


a. Treponema pallidum
b. Campylobacter jejuni
c. Salmonella
d. S. aureus

558. Genetic variations are important in:


a. Borrelia
b. S. aureus
c. Leptospira
d. Hemophilus

559. Which of the following disease is not caused by Treponema:


a. Yaws
b. Bejel
c. Relapsing fever
d. Syphilis
560. Syphilis was first identified by:
a. Frankel
b. Nicholas
c. Schaudinn and Hoffman
d. Ogston

561. Consider the following serological tests:


1. FTA-ABS
2. TPI
3. TPHA
4. VDRL

Which of these are specific for diagnosing syphilis?


a. 1 only
b. 2 and 4
c. 3 and 4
d. 1,2 and 3

562. Which of the following is the most specific test to diagnose syphilis:
a. VDRL test
b. Wassermann test
c. RPR
d. FTA-ABS
e. TPPA

563. Dark ground microscopy is used for:


a. TPI
b. FTA-ABS
c. Kahn’s test
d. VDRL

564. RPR is done for diagnosis of:


a. Malaria
b. Syphilis
c. Leishmaniasis
d. None

565. Antigenic mutation is seen in: (PGI)


a. Trypanosoma
b. Streptococcus
c. Borrelia
d. Leptospira

566. Incubation period of syphilis:


a. 1 hour - 5 hour
b. 24 hour - 48 hour
c. 1 day -10 days
d. 10 days - 90 days

567. Borrelia undergoes antigenic variation due to:


a. Plasmids
b. Transposons
c. Intrinsic mutation
d. All of the above

568. Leptospirosis is transmitted by:


a. Rat
b. Cat
c. Dog
d. Fish
553. 554. c 555. 556. 557. 558. 559. c 560. c 561. d 562. e
a a b a a

563. 564. 565. 566. 567. 568.


a b a,c d a a

569. Reservoir of Leptospira:


a. Cat
b. Dog
c. Rat
d. Monkey

570. Stain for Treponema:


a. Fontana’s
b. Acid-fast
c. Methenamine-silver
d. PAS

571. Confirmatory test for syphilis:


a. VDRL
b. FTA-ABS
c. RPR
d. None

572. True about B. recurrentis:


a. Causes leptospirosis
b. Water borne disease
c. Vector borne disease
d. Transmitted by tick

573. All of them are features of Ureaplasma urealyticum except:


a. Non-gonococcal urethritis
b. Salpingitis
c. Epididymitis
d. Bacterial vaginosis
e. All of the above

574. In reference to Mycoplasma, the following are true except:


a. They are inhibited by Penicillins
b. They can reproduce in cell free media
c. They have an affinity for mammalian cell membranes
d. They can pass through fitters of 450 nm pore size

575. Positive cold agglutination test is seen in infections with:


a. Mycoplasma
b. Chlamydia
c. Infectious mononucleosis
d. Varicella

576. Cell wall deficient organisms are:


a. Chlamydia
b. Mycoplasma
c. Streptococcus
d. Anaerobes

577. Diene’s method is used for:


a. Mycoplasma
b. Chlamydiae
c. Plague
d. Diphtheria

578. True about mycoplasma:


a. Obligate intracellular organism
b. Penicillin is effective treatment
c. Require cholesterol for growth
d. Have thick cell wall

579. HACEK group includes all of the following except:


a. Hemophilus aphrophilus
b. Acinetobacter baumanni
c. Eikenella corrodens
d. Cardiobacterium hominis

580. Ehrlichia chaffeensis is causative agents of:


a. HME
b. HGE
c. Glandular fever
d. None
569. c 570. 571. 572. c 573. 574. 575. 576. b 577. a 578. c
a b e a a

579. 580.
b a
Bacteriology — Explanations
168. A -O 157: H7
Certain enterohemorrhagic strains of E. coli (i.e., those with the O157:H7 serotype)
also cause bloody diarrhea by producing an exotoxin called Shiga toxin, so called
because it is very similar to that produced by Shigella species.
Shiga toxin acts by removing an adenine from the large (28S) ribosomal RNA, thereby
stopping protein synthesis.
Shiga toxin is encoded by temperate (lysogenic) bacteriophages. Shiga toxin is also
called verotoxin because it has a cytopathic effect on Vero (monkey) cells in culture.

169. D -Mycoplasma

170. C- Ludwig’s angina

171. B- Calymmatobacterium granulomatis


A clinical diagnosis of donovanosis is made by an experienced practitioner on the
basis of the lesion’s appearance and usually has a high positive predictive value.
Then diagnosis is confirmed by microscopic identification of Donovan bodies in tissue
smears.
Preparation of a good-quality smear is important.
If donovanosis is suspected on clinical grounds, the smear for Donovan bodies should
be taken before swab samples to be tested for other causes of genital ulceration so
that enough material can be collected from the ulcer.
A swab should be rolled firmly over an ulcer previously cleaned with a dry swab to
remove debris.
Smears can be examined in a clinical setting by direct microscopy with a rapid
Giemsa or Wright’s stain.

172. D-Weil felix reaction


Weil felix reaction is a heterophile agglutination reaction used for presumptive
diagnosis of rickettsial infections.

173. B-Gastric (MALT) lymphoma


Helicobacter pylori colonizes the stomachs of ~50% of the world’s human population
throughout their lifetimes. Colonization with this organism is the main risk factor for
peptic ulceration as well as for gastric adenocarcinoma and gastric MALT (mucosa
associated lymphoid tissue) lymphoma .

174. B -Leptospira interrogans


Weil’s disease or ictero-haemorrhagic fever is a severe form of leptospirosis caused by
L.interrogans.

175. A- Gardnerella vaginalis


Bacterial vaginosis is alteration in normal vaginal flora with decrease in Lactobacilli
and increase in Gardnerella vaginalis.

176. D -Pseudomonas
P. aeruginosa infections of the ears vary from mild swimmer’s ear to serious life-
threatening infections with neurologic sequelae.
Swimmer’s ear is common among children and results from infection of moist
macerated skin of the external ear canal.
Most cases resolve with treatment, but some patients develop chronic drainage.
Swimmer’s ear is managed with topical antibiotic agents (otic solutions).

177. B- Most commonly causes diarrhea in developing countries


In tropical or developing countries, ETEC is a major cause of endemic diarrhoea.

178. C- Most important toxin is hyaluronidase


Most important enzyme of C.perfringens is lecithinase/phospholipase C.

179. C- Anthrax of skin


Malignant pustule is a manifestation of cutaneous anthrax (Hide porter’s disease).

180. C- Capsular polypeptide protein is responsible for virulence


Capsular polysaccharide is a virulence factor and it is made of polyribosyl ribitol
phosphate (PRP) in serotype b.

181. B -More common in winters and dry weather


C. tetani is an anaerobic, gram-positive, spore-forming rod whose spores are highly
resilient and can survive readily in the environment throughout the world.
Spores resist boiling and many disinfectants.
In addition, C. tetani spores and bacilli survive in the intestinal systems of many
animals, and fecal carriage is common.
The spores or bacteria enter the body through abrasions, wounds, or (in the case of
neonates) the umbilical stump.
Once in a suitable anaerobic environment, the organisms grow, multiply, and release
tetanus toxin, an exotoxin that enters the nervous system and causes disease.
Very low concentrations of this highly potent toxin can result in tetanus (minimum
lethal human dose, 2.5 ng/kg).
There is no seasonal predilection.

182. B- VDRL
VDRL becomes negative with successful treatment, hence is used as a prognostic test.

183. B -Presence of Pain abdomen

184. B -Clostridium difficile


Clostridium difficile infection (CDI) is a unique colonic disease that is acquired almost
exclusively in association with antimicrobial use and the consequent disruption of the
normal colonic flora.
The most commonly diagnosed diarrheal illness acquired in the hospital, CDI results
from the ingestion of spores of C. difficile that vegetate, multiply, and secrete toxins,
causing diarrhoea and pseudomembranous colitis (PMC).

185. B- Chlamydia

186. A -Calymmatobacterium granulomatis


Donovanosis is a chronic, progressive bacterial infection that usually involves the
genital region.
The condition is generally regarded as a sexually transmitted infection of low
infectivity.
This infection has been known by many other names, the most common being
granuloma inguinale .
The causative organism has been reclassified as Klebsiella granulomatis comb nov on
the basis of phylogenetic analysis, although there is ongoing debate about this
decision.
Some authorities consider the original nomenclature ( Calymmatobacterium
granulomatis ), which is based on analysis of 16S rRNA gene sequences, to be more
appropriate.

187. D -M.ulcerans
M. ulcerans, another waterborne skin pathogen, is found mainly in the tropics,
especially in tropical areas of Africa.
It causes Buruli ulcer.
Infection follows skin trauma or insect bites that allow admission to contaminated
water.
The skin lesions are typically painless, clean ulcers that slough and can cause
osteomyelitis.
The toxin mycolactone accounts for the modest host inflammatory response and the
painless ulcerations.

188. B-Herpes genitalis, D- chancroid

189. A- Chlamydia psittaci


Psittacosis in humans may present as acute primary atypical pneumonia (which can
be fatal in up to 10% of untreated cases); as severe chronic pneumonia; or as a mild
illness or asymptomatic infection in persons exposed to infected birds.

190. D- diphtheria
Corynebacterium diphtheriae does not have any animal reservoir, it is a human
pathogen. Other options have animal reservoirs.

191. B- Borrelia burgdorferi


Lyme borreliosis is caused by a spirochete, Borrelia burgdorferi sensu lato , that is
transmitted by ticks of the Ixodes ricinus complex.

192. A -Myocardial necrosis


Listeriosis in pregnancy is a severe and important infection.
The usual presentation is a nonspecific acute or subacute febrile illness with
myalgias, arthralgias, backache, and headache.
Pregnant women with listeriosis are usually bacteremic.
This syndrome should prompt blood cultures, especially in the absence of another
reasonable explanation.
Involvement of the CNS is rare in the absence of other risk factors.
Preterm delivery is a common complication, and the diagnosis may be made only
postpartum.
As many as 70–90% of foetuses from infected women can become infected.
Prepartum treatment of bacteremic women enhances the chances of delivery of a
healthy infant.
Women usually do well after delivery: maternal deaths are very rare, even when the
diagnosis is made late in pregnancy or postpartum.
Overall mortality rates for fetuses infected in utero approach 50% in some series;
among live-born neonates treated with antibiotics, mortality rates are much lower
(~20%).
Granulomatosis infantiseptica is an overwhelming listerial fetal infection with miliary
microabscesses and granulomas, most often in the skin, liver, and spleen.
Less severe neonatal infection acquired in utero presents at birth.
“Late-onset” neonatal illness typically develops ~10–30 days postpartum.
Mothers of infants with late onset disease are not ill.
L. monocytogenes causes ~5–10% of all cases of community acquired bacterial
meningitis in adults in the West.
It does not cause myocardial necrosis.

193. D -leptospirosis
Leptospirosis is also known as Sewer worker’s disease.

194. C- Tetanus
Tetanus is not transmissible from person to person.

195. D- Gonococcus
The vaginal mucosa of healthy women is lined by stratified squamous epithelium and
is rarely infected by N. gonorrhoeae.
However, gonococcal vaginitis can occur in anestrogenic women (e.g., prepubertal
girls and postmenopausal women), in whom the vaginal stratified squamous
epithelium is often thinned down to the basilar layer, which can be infected by N.
gonorrhoeae.
The intense inflammation of the vagina makes the physical (speculum and bimanual)
examination extremely painful.
The vaginal mucosa is red and edematous, and an abundant purulent discharge is
present.
Infection in the urethra and in Skene’s and Bartholin’s glands often accompanies
gonococcal vaginitis.
Inflamed cervical erosion or abscesses in nabothian cysts may also occur.
Coexisting cervicitis may result in pus in the cervical os.

196. B- LGV

197. B- Malaria, C-Leishmaniasis, D-Tropical eosinophilia


Biological false-positive (BFP) reactions:
BFP reactions are defined as positive results in non treponemal tests, with negative
results in treponemal tests, in the absence of syphilis and not caused by technical
faults.
Cardiolipin antigen being non-specific may react with the sera of patients suffering
from unrelated diseases but not having syphilis.
Reagin antibodies are induced against the cardiolipin antigen present in T.pallidum
or similar lipid haptens released from damaged host tissues.
BFP reactions may occur in about 1 % of normal sera.
BFP antibody is usually of IgM type, while regain antibody in syphilis is mainly IgG.
BFP reactions may be seen in the conditions such as lepromatous leprosy, relapsing
fever, malaria, tropical pulmonary eosinophilia, viral hepatitis, infectious
mononucleosis, HIV, pregnancy and IV drug abusers.

198. A -Q fever
Q fever is the one rickettsial disease that is not transmitted to humans by the bite of
an arthropod.
The important reservoirs for human infection are cattle, sheep, and goats.
The agent, C. burnetii, which causes an inapparent infection in these reservoir hosts,
is found in high concentrations in the urine, feces, placental tissue, and amniotic fluid
of the animals.
It is transmitted to humans by inhalation of aerosols of these materials.
The disease occurs worldwide, chiefly in individuals whose occupations expose them
to livestock, such as shepherds, abattoir employees, and farm workers.
Cow’s milk is usually responsible for subclinical infections rather than disease in
humans.
Pasteurization of milk kills the organism.

199. C- Donovanosis

200. A -Staphylococci
Scalded-skin syndrome is characterized by fever, large bullae, and an erythematous
macular rash.
Large areas of skin slough, serous fluid exudes, and electrolyte imbalance can occur.
Hair and nails can be lost.
Recovery usually occurs within 7–10 days.
This syndrome occurs most often in young children.

201. D- Listeria
Granulomatosis infantiseptica is an overwhelming listerial fetal infection with miliary
microabscesses and granulomas, most often in the skin, liver, and spleen.
Less severe neonatal infection acquired in utero presents at birth.

202. A -Staphylococcus aureus

203. C -Bacillus anthracis

204. C -Mycoplasma
Eaton’s agent: It refers to the most pathogenic species i.e. Mycoplasma pneumoniae;
which was first isolated by Monroe Eaton (1944).

205. A -Staphylococcus aureus


Osteomyelitis and septic arthritis may arise either by hematogenous spread from a
distant infected focus or be introduced locally at a wound site.
S. aureus is a very common cause of these diseases, especially in children.

206. D -Viridans streptococci


Subacute bacterial endocarditis (SABE): Viridans streptococci are the most common
cause of SABE.
The commensal viridans streptococci (S. sanguis) in the oral cavity can enter blood to
cause transient bacteraemia while chewing, tooth brushing and dental procedures
that can account for the predilection of these organisms to cause endocarditis.

207. D -Salmonella typhimurium


Normal gut flora are indicators of faecal pollution of water, not gut pathogens.

208. C -Scrub typhus


Scrub typhus is transmitted by trombiculid mite.
209. B -Group B streptococcus
GBS is the most common cause of neonatal septicemia and neonatal meningitis in the
West.

210. D- C. minutissimum
Erythrasma is a cutaneous infection producing reddish brown, macular, scaly, pruritic
intertriginous patches.
The dermatologic presentation under the Wood’s lamp is of coral-red fluorescence. C.
minutissimum appears to be a common cause of erythrasma, although there is
evidence for a polymicrobial etiology in certain settings.
In addition,this fluorescent microbe has been associated with bacteremia in patients
with hematologic malignancy.
Erythrasma responds to topical erythromycin, clarithromycin, clindamycin, or fusidic
acid, although more severe infections may require oral macrolide therapy.

211. D- Cutaneous anthrax


Cutaneous anthrax is also known as Hide porter’s disease.

212. C -Gastric leiomyoma


Helicobacter pylori colonizes the stomachs of ~50% of the world’s human population
throughout their lifetimes.
Colonization with this organism is the main risk factor for peptic ulceration as well as
for gastric adenocarcinoma and gastric MALT (mucosa associated lymphoid tissue)
lymphoma .

213. B- Staphylococcus aureus

214. C- Pseudomonas aeruginosa


The most serious form of Pseudomonas infection involving the ear has been given
various names: two of these designations, malignant otitis externa and necrotizing
otitis externa, are now used for the same entity.
This disease was originally described in elderly diabetic patients, in whom the
majority of cases still occur.
However, it has also been described in patients with AIDS and in elderly patients
without underlying diabetes or immunocompromise.
The usual presenting symptoms are decreased hearing and ear pain, which may be
severe and lancinating.
The pinna is usually painful, and the external canal may be tender.
The ear canal almost always shows signs of inflammation, with granulation tissue and
exudate.
Tenderness anterior to the tragus may extend as far as the temporomandibular joint
and mastoid process.
A small minority of patients have systemic symptoms.
Patients in whom the diagnosis is made late may present with cranial nerve palsies or
even with cavernous venous sinus thrombosis.
The ESR is invariably elevated (≥100 mm/h).
The diagnosis is made on clinical grounds in severe cases; however, the “gold
standard” is a positive technetium-99 bone scan in a patient with otitis externa due
to P. aeruginosa.
In diabetic patients, a positive bone scan constitutes presumptive evidence for this
diagnosis and should prompt biopsy or empirical therapy.
215. C- Transcription
Three methods of horizontal gene transfer in bacteria are transformation, transduction
and conjugation.

216. B -Typhoid
The most prominent symptom in typhoid is prolonged fever (38.8°–40.5°C; 101.8–
104.9 degree Farenheit), which can continue for up to 4 weeks if untreated.
Early physical findings of enteric fever include rash (“rose spots”; 30%),
hepatosplenomegaly (3–6%), epistaxis, and relative bradycardia at the peak of high
fever (<50%).
In 15–25% of cases, leukopenia and neutropenia are detectable.
Leukocytosis is more common among children, during the first 10 days of illness, and
in cases complicated by intestinal perforation or secondary infection.

217. A -Salmonella typhimurium


BACTERIAL FOOD POISONING

Incubatioin Period, Symptoms Common Food Sources


Organism

1-6 h

Staphylococcus Nausea, vomiting, diarrhea Ham, poultry, potato or egg salad, may-onnaise,
aureus cream pastries
Nausea, vomiting, diarrhea
Bacillus cereus Fried rice

8-16 h

Clostridium Abdominal cramps, diarrhea Beef, poultry, legumes, gravies


perfringens (vomiting rare)
Meats, vegetables, dried beans, cereals
B. cereus Abdominal cramps, diarrhea
(vomiting rare)

> 16 h

Vibrio cholerae Watery diarrhea Shellfish, water

Enterotoxigenie Watery diarrhea Salads, cheese, meats, water


Escherichia coli
Bloody diarrhea Ground beef, roast beef, salami, raw milk, raw
Enterohemorrhagic E. vegeta- bles, apple juice
coli Inflammatory diarrhea
Beef, poultry, eggs, dairy products
Salmonella spp. Inflammatory diarrhea
Poultry, raw milk
Campylobacter jejuni Dysentery
Potato or egg salad, lettuce, raw vegetables
Shigella spp. Dysentery
Mollusks, crustaceans
Vibrio
parahaemolyticus

218. A- Strict aerobe


Mycobacteria are aerobic, acid-fast bacilli (rods).
They are neither gram-positive nor gram-negative (i.e., they are stained poorly by the
dyes used in Gram stain).
They are virtually the only bacteria that are acid-fast. (One exception is Nocardia
asteroides, the major cause of nocardiosis, which is also acid-fast.)
The term acid-fast refers to an organism’s ability to retain the carbol fuchsin stain
despite subsequent treatment with an ethanol–hydrochloric acid mixture.
The high lipid content (approximately 60%) of their cell wall makes mycobacteria
acid-fast.

219. A- FTA-Abs
Non treponemal tests are used to monitor response to treatment in syphilis.

220. A- Clostridium tetani


Swarming is shown by C.tetani, Proteus, Bacillus cereus.

221. B- Gram stain


Spirochaetes are too slender to be seen on gram stain.

222. D- V. cholerae
V. cholerae and V.mimicus are non-halophilic vibrios.

223. C- Listeria monocytogenes

224. B -Endemic relapsing fever


Endemic relapsing fever is transmitted by soft tick.

225. C -Rickettsia prowazekii


Recrudescent typhus is also known as Brill Zinsser’s disease.

226. C -Transduction
β lactamase or penicillinase enzymes cleave the β lactam rings, and there by
organism producing this enzyme develops resistance to β lactam antibiotics.
This resistance is plasmid coded, can be transferred between S. aureus strains by
transduction.
It is produced by >90% of strains of S. aureus.
This resistance can be overcome by addition of β lactamase inhibitors such as
clavulanic acid or sulbactam.

227. D- Escherichia coli


Escherichia coli is the most common cause of community acquired and nosocomial
UTI.

228. D- It is a gram negative rod that does not ferment lactose


C.perfringens is a gram positive, anaerobic, spore-forming rod.

229. C- The species of Leptospira that cause leptospirosis grow primarily in


humans and are usually transmitted by human to human contact.
Leptospira has many mammals as animal reservoirs, most important being rodents.It
is not a primary human pathogen.Leptospirosis is a zoonotic disease, not transmissible
from person to person.

230. B- Trench fever


Trench fever is transmitted by louse.

231. A- Group B streptococcus


CAMP test is used for identification of GBS and Listeria monocytogenes.

232. D- Calymmatobacterium granulomatis


Donovanosis:
The incubation period is usually 1–4 weeks but may extend to 1 year.
Skin lesions have been detected in infants 6 weeks to 6 months after birth.
The disease begins as one or more subcutaneous nodules that erode through the skin
to produce clean, granulomatous, sharply defined, usually painless lesions .
These lesions, which bleed readily on contact, slowly enlarge.
The genitalia are involved in 90% of cases, the inguinal region in 10%, and the anal
region in 5–10%.
Genital swelling, particularly of the labia, is a common feature and occasionally
progresses to pseudoelephantiasis.

233. D- Mycobacterium

234. D- Malignant pustule


McFadyean reaction is used to demonstrate capsular material of Bacillus anthracis.

235. A- Staphylococcus
P-V leukocidin is a pore-forming toxin that kills cells, especially white blood cells, by
damaging cell membranes.
The two subunits of the toxin assemble in the cell membrane to form a pore through
which cell contents leak out.
The gene encoding P-V leukocidin is located on a lysogenic phage.
The importance of P-V leukocidin as a virulence factor is indicated by the severe skin
and soft tissue infection caused by MRSA strains that produce this leukocidin.
A severe necrotizing pneumonia is also caused by strains of S. aureus that produce P-
V leukocidin.
Approximately 2% of clinical isolates of S. aureus produce P-V leukocidin.

236. B- L. monocytogenes
Anton’s test is a test for demonstrating the invasive property of L.monocytogenes.

237. A- hot air oven

238. B- Streptococcus pneumoniae

239. B -Bacillus anthracis


Considering the clinical features and morphology of the organism mentioned in the
question, it is a case of inhalational anthrax or wool sorter’s disease.

240. C- Borrelia hermsii

241. A- Yersinia pestis


Considering the clinical features and morphology of the organism mentioned in the
question, it is a case of septicemic plague.

242. D- Ixodes species

243. C- Streptococcus pyogenes


Streptolysin O is a hemolysin that is inactivated by oxidation (oxygen labile).
It causes β-hemolysis only when colonies grow under the surface of a blood agar
plate.
It is antigenic, and antibody to it (ASO) develops after group A streptococcal
infections.
The titer of ASO antibody can be important in the diagnosis of rheumatic fever.

244. A- Ingestion of preformed exotoxin


Food poisoning (gastroenteritis) is caused by ingestion of enterotoxin, which is
preformed in foods and hence has a short incubation period (1–8 hours).
In staphylococcal food poisoning, vomiting is typically more prominent than
diarrhoea.

245. A- Elementary Body


Chlamydiae have a replicative cycle different from that of all other bacteria.
The cycle begins when the extracellular, metabolically inert, “sporelike” elementary
body enters the cell and reorganizes into a larger, metabolically active reticulate
body .
The latter undergoes repeated cycles of binary fission to form daughter reticulate
bodies, which then develop into elementary bodies, which are released from the cell.
Within cells, the site of replication appears as an inclusion body, which can be
stained and visualized microscopically .
These inclusions are useful in the identification of these organisms in the clinical
laboratory.

246. C- Both Syphilis and Lyme disease

247. A- Streptococcus

248. D-4.0 × 103


At the end of first 10 minutes( lag phase), number remains the same.Bacterial number
multiplies in next 10 minutes and again doubles in subsequent ten minutes.

249. B- Prognosis
Lepromin test assess CMI, it is used as a prognostic test.

250. C- Subacute endocarditis caused by Streptococcus bovis


S. bovis, a nonenterococcal group D streptococcus, causes endocarditis, especially
in patients with carcinoma of the colon.
This association is so strong that patients with S. bovis, bacteremia, or endocarditis
should be investigated for the presence of colonic carcinoma.

251. D- Demonstration of toxin production by a suspicious isolate


Toxigenicity tests are confirmatory for diphtheria.

252. A- EIEC
EIEC shares many genetic and clinical features with Shigella; however, unlike
Shigella, EIEC produces disease only at a large inoculum (108–1010 CFU), with onset
generally following an incubation period of 1–3 days.
Initially, enterotoxins are believed to induce secretory small-bowel diarrhoea.
Subsequently, colonization and invasion of the colonic mucosa, followed by
replication therein and cell-to-cell spread, result in the development of inflammatory
colitis characterized by fever, abdominal pain, tenesmus, and scant stool containing
mucus, blood, and inflammatory cells.
Symptoms are usually self-limited (7–10 days).

253. B- Disc diffusion method


M.tuberculosis is very slow-growing(generation time being 20 hours),hence disc
diffusion is not used for AST.

254. D- Sensitive to gentamicin

255. A- hyperproduction of beta-lactamases by borderline oxacillin-resistant


strains of S. aureus

256. C- Siderophores

257. A- CA-MRSA is more virulent & less resistant


Types of MRSA

Community associated MRSA (CA-MRSA) Hospital associated MRSA (HA-MRSA)

These strains express mecA gene subtype IV, These strains express mecA gene subtype I, II, III.
V, VI.

They are usually more virulent and express They are multidrug resistant I but their virulence is relatively
several toxins such as Pan ton Valentine (PV) low).
toxin.

They cause invasive skin and soft tissue They cause perioperative wound infections in hospitals and
infections such as necrotizing fasciitis. nosocomial outbreaks (hospital staff are the major carries).

Note: CA-MRSA and HA- MRSA terminologies are becoming artificial nowadays; as
many CA MRSA strains have been isolated in hospitalsand vice-versa.

258. C- S. sonnei
S. sonnei (group D) : It is antigenically homogeneous and has only one serotype. It can
be typed by colicin typing into 26 colicin types.

259. B- Colicin typing

260. A- Slide coagulase test detects coagulase enzyme


Coagulase enzyme combines with a plasma protein called CRF (coagulase reacting
factor), and together they activate prothrombin, which in turn, converts fibrinogen to
fibrin.
This is the basis of tube coagulase test. This has to be differentiated from slide
coagulase test, which is mediated by clumping factor .
Coagulase can react with rabbit or human plasma; but does not clot with guinea pig
plasma as it lacks CRF.

261. D- Automated antimicrobial susceptibility test

262. B-8-16 µg/ml

263. C- Fibrinogen binding protein


Staphylococcus epidermidis:
It is the most common CoNS (75- 80%) isolated from clinical samples. It is present as
normal flora on the skin, oropharynx and vagina; however, its pathogenic role is
greatly enhanced in presence of prosthetic-devices.
Pathogenesis: S. epidermidis involves a two step process:
1. Initial adhesion to the prosthetic device: The surface adhesins of the organism bind
to host serum or tissue constituents; such as fibrinogen or fibronectin., coated on the
implanted prosthetic surfaces.
2. Colonization: S.epidermidis can produce the extracellular polysaccharide material
(glycocalyx or slime) that facilitates formation of a protective biofilm on the device
surfaces. Biofilm appears to act as a barrier, protecting bacteria from host defence
mechanisms as well as from antibiotics.

264. B- M protein

265. C- Streptokinase is produced from serotype A, C, K

266. D-80% isolates are intermediate resistance to penicillin


Except for strains that cause conjunctivitis, nearly every clinical isolate of S.
pneumoniae has a polysaccharide capsule, a structure that renders the bacteria
virulent by preventing phagocytosis.
Empyema is the most common complication of pneumococcal pneumonia, occurring
in ~2% of cases.
The name Austrian’s syndrome is given to the concurrence of pneumococcal
pneumonia, endocarditis, and meningitis.
At present, ~20% of pneumococcal isolates in the United States exhibit intermediate
resistance to penicillin [minimal inhibitory concentration (MIC) 0.1–1.0 μg/mL], and
15% are resistant (MIC ≥2.0 (µg/mL).
The rate of resistance is lower in countries that, by tradition, are conservative in their
antibiotic use (e.g., Holland and Germany) and higher in countries where usage is
more liberal (e.g., France).
In Hong Kong and Korea, resistance rates approach 80%.

267. A- Viridans streptococci


Subacute bacterial endocarditis (SABE): Viridans streptococci are the most common
cause of SABE.
The commensal viridans streptococci (S. sanguis) in the oral cavity can enter blood to
cause transient bacteraemia while chewing, tooth brushing and dental procedures
that can account for the predilection of these organisms to cause endocarditis.

268. C- Enterococci

269. A- Inducible high level resistance to Vancomycin & Teicoplanin


Vancomycin-resistant enterococci (VRE), first reported from clinical sources in the
late 1980s, have become common in many hospitals.
Three major vancomycin resistance phenotypes have been described: VanA, VanB,
and VanC. The VanA phenotype is associated with high level resistance to vancomycin
and to teicoplanin.
VanB and VanC strains are resistant to vancomycin but susceptible to teicoplanin,
although teicoplanin resistance may develop during treatment in VanB strains.

270. A- Defect of the Btk gene


X-Linked Hypogammaglobulinemia (Bruton’s Agammaglobulinemia):
Very low levels of all immunoglobulins (IgG, IgA, IgM, IgD, and IgE) and a virtual
absence of B cells are found in young boys; female carriers are immunologically
normal.
Pre-B cells are present, but they fail to differentiate into B cells.
This failure is caused by a mutation in the gene encoding tyrosine kinase, an
important signal transduction protein.
Cell-mediated immunity is relatively normal.
Clinically, recurrent pyogenic bacterial infections (e.g., otitis media, sinusitis, and
pneumonia caused by Streptococcus pneumoniae and Haemophilus influenzae)
occur in infants at about 6 months of age, when maternal antibody is no longer
present in sufficient amount to be protective.
Treatment with pooled gamma globulin reduces the number of infections.

271. B- M proteins

272. C- Ab is found to be useful for diagnosis

273. D- RFLP & IS fingerprinting


Genotypic methods are more commonly used for the typing of M. tuberculosis:
PCR-RFLP (Restriction fragment length polymorphism) is used to detect variations in
IS6110 gene.
Spoligoyping: This is based on detecting the polymorphisms in DR locus (direct
repeat) of M. tuberculosis genome. It is more useful in strains having no or few copies
of IS6110 gene.
Pulse filed gel electrophoresis (PFGE).
DNA sequencing.

274. D-γIFN release assay


There are two approaches to the diagnosis of latent infections.
One is the PPD skin test.
Because there are problems both in the interpretation of the PPD test and with the
person returning for the skin test to be read, a quantifiable laboratory-based test is
valuable.
This laboratory test is an interferon-γ release assay (IGRA), and there are two versions
available: Quantiferon-TB and T-spot.TB.
In this assay, blood cells from the patient are exposed to antigens from M.
tuberculosis, and the amount of interferon-γ released from the cells is measured.
The sensitivity and specificity of the IGRA are as good as the PPD skin test.
Because the antigens used in the test are specific for M. tuberculosis and are not
present in BCG, the test is not influenced by whether a person has been previously
immunized with the BCG vaccine.

275. D- Ferment lactose, produce acid and gas


Members of the family Enterobacteriaceae should have the following properties:
They are gram-negative bacilli.
Aerobes and facultative anaerobes.
Nonfastidious. can grow in ordinary media like nutrient agar.
Ferment glucose to produce acid with or without gas.
Reduce nitrate to nitrite.
They produce catalase (except Shigella dysenteriae type-l ).
They do no produce oxidase.
They are generally motile with peritrichous flagella, except some members which are
non motile, such as Shigella and Klebsiella.
Natural habitat: Most of them are commensals in human intestine, called coliform
bacilli, e.g. Escherichia, Klebsiella, Proteus, Morganella, Providencia and Citrobacter,
etc.The exceptions are Shigella, Salmonella which are enteric pathogens not
commensals.

276. D-2 ME is used to detect IgA


2-mercaploelhanol (2ME) agglutination test: Here, the serum is treated with
disulphide reducing agents such as 2-ME so that it destroys the agglutinability of lgM,
but does not alter IgG. SAT performed in 2ME treated serum, would detect only IgG.

277. B- Cystitis caused by mostly the strains which carry K Ag


Capsular or K antigen (K for Kapsel, German for capsule):
It is the polysaccharide capsular antigen present on the envelope or microcapsule o f
a few strains of E.coli.
When present, it encloses the O antigen and renders the strain inagglutinable by the
O antiserum.
It may also contributes to virulence by inhibiting phagocytosis.
It is expressed by only few strains o f E.coli e.g. those causing neonatal meningitis,
pyelonephritis and septicaemia.

278. D- Serial passage in rabbit reduces the virulence of the spirochete to


human being

279. C- Infection is transmitted by bite in both


Louse-Borne Relapsing Fever:
Body lice (Pediculus humanus var. corporis) become infected with B. recurrentis by
feeding on spirochetemic humans, the only reservoirs of infection.
In lice, B. recurrentis spirochetes are found almost exclusively in the hemolymph;
humans acquire infection when body lice are crushed and their infective fluids enter
breaks in the skin, typically abrasions caused by scratching of pruritic louse bites.
Spirochetes are not transmitted directly by the bite of a louse (anterior station
transmission) but may possibly be transmitted in a manner similar to epidemic
typhus by percutaneous inoculation of louse feces (posterior station transmission).
Lice have a life span of only a few weeks, feed at frequent intervals, and survive only
a few days off the human host.
Typically, body lice reside in seams of clothing and bedclothes, where they deposit
their eggs (nits).
Head lice have not been shown to be vectors of LBRF.
Tick-Borne Relapsing Fever:
Argasid ticks of the genus Ornithodoros transmit TBRF through their saliva and
excreta when they take blood meals.
Ticks typically become infected with TBRF borreliae as part of a zoonotic cycle when
they feed on spirochetemic rodents and lagomorphs; the exception is O. moubata, a
tick species that is thought to acquire B. duttoni only by feeding on infected humans.
Ticks transmit TBRF borreliae vertically from one stage to the next; in some species,
infection is transmitted transovarially over several generations.
Soft ticks are hardy and can survive for as long as 10 years with only an occasional
blood meal.
These ticks feed painlessly, relatively quickly (for 20–45 min), and usually at night
while hosts are sleeping.
Thus patients with TBRF are often unaware of tick exposures.

280. C- Lysogeny
Lysogenic conversion: infection with a bacteriophage to Salmonella may cause loss,
gain or change of an O antigen.
S. Anatum is converted into S. Newington by infection with one phage (gaining O15
antigen) and the latter into S. Minneapolis by another phage infection (gaining O34
antigen).

281. B- H-O variation is due to total loss of flagella

282. C- Escherichia coli

283. C- Form chalky granular clump in widal test

O antigen H antigen

Somatic antigen Flagellar antigen

It is a part of cell wall lipopolysaccharide (LPS) Made up of proteins


flagellin
It confers moti lity to the
bacteria

Heat stable Alcohol stable Heat labile. Alcohol labile

Formaldehyde labile Formaldehyde stable

In Widal test, O antigen of S.Typhi is used In Widal test, H antigens of


S.Typhi. S.Paratyphi A and B are
used

O Ag is less immunogenic HAg is more immunogenic

O antibody appears early, disappears early: indicates recent infection H antibody appears late,
disappears late: indicates
convalescent stage

When Oantigen reacts with O antibody forms compact, granular, chalky When H antigen reacts with H
clumps antibody forms large, loose
fluffy clumps.
Agglutination takes place slowly
Optimum temperature for agglutination is 55°C Agglutination takes place
rapidly.
Optimum temperature
for agglutination is 37°C

Serogrouping of salmonellae is based on the Oantigen Serogroups are differentiated


into serotypes based on H
antigen.

Flagellar antigens exist


O antigen is also called the Boivin antigen because it can be extracted from in two alter nati ve
the bacterial cell by treatment with trichloracetic acid – this property was phases Phase 1 and II.
first shown by Roivin Most of them are
biphasic except 5l Typhi
which is monophasic.
284. C- Bacillus brevis

285. B- Legionella pneumophila


Cell-mediated immunity is the primary mechanism of host defense against
Legionella, as it is against other intracellular pathogens.
Thus Legionnaires’ disease is more common and its manifestations are more severe
among patients with depressed cell-mediated immunity.
The disease also occurs with unusual frequency among patients with hairy cell
leukemia, which is characterized by monocyte deficiency and dysfunction.
The definitive method for diagnosis of Legionella infection is isolation of the organism
from respiratory secretions or other specimens.
Multiple selective BCYE media containing dyes are required for maximal sensitivity.
Colonies grow slowly, requiring 3–5 days to become grossly visible.

286. C- Serratia marcescens


The characteristic property of Serratia is production of a red non-diffusible pigment
called prodigiosin, which is formed optimally at 30’C .
S. marcescens is the medically most important species.
Human infection with other species is rare.
It is a saprophyte found in water, soil and food. It may grow in sputum after collection
and makes the sputum red (due to pigment production). This condition is known as
‘pseudohemoptysis’.

287. A- Mycoplasma
Colonies can be examined by:
a) Hand lens
b) Dienes’ staining: Plate is flooded with alcoholic solution of methylene blue and
azure, and examined under low power microscope.
Mycoplasmas retain colour for at least 2 days and appear intense royal blue, whereas
Ureaplasmas appear reddish to greenish blue.

288. B- Borrelia recurrentis


A unique process of DNA rearrangement within vmp genes located on linear plasmids
results in extensive variation in the expression of the surface antigens in relapsing
fever borreliae.
These vmp genes encode variable major proteins (VMPs) found on the spirochete’s
outer-membrane surface.
The antigenic variation generated by sequential expression of previously silent vmp
genes allows the borreliae to intermittently escape the immune response of the host
and results in the febrile spirochetemic relapses that are characteristic of infection
with these organisms.

289. A- Typhoid fever between 5th to 14th days

290. B- Clostridium perfringens


A CAMP positive Group B Streptococcus is streaked in the center of sheep blood agar,
and Clostridium perfringens is streaked perpendicular to it.
Following incubation at 37°C for 24-48 hours in anaerobic conditions, the “bow-tie”
zone of enhanced hemolysis pointing towards Streptococcus agalactiae is seen.
This is because of alpha toxin produced by Clostridium perfringens interacts with
CAMP factor and produce synergistic hemolysis.
291. C -H antigen
Please see explanation to Q 283.

292. D- Polymyxin, lysozyme, EDTA and Thallous acetate


PLET medium: It consists of polymyxin B, lysozyme, EDTA and thallous acetate added
in heart infusion agar. It has been devised to isolate B. anthracis from mixtures of
other spore bearing bacilli.

293. B- Staphylococcus

294. C-III

295. D- Pseudomonas aeruginosa


Please refer to explanation to Q.214.

296. A-Yolk sac inoculation


B-Cell culture using McCoy cell lines
C-Cell culture using HeLa cells
Chlamydiae cannot be cultivated in artificial media. They can grow only in
embryonated egg (yolk sac), animal (mice) and cell line.
Both egg and mice inoculation methods are no longer in use.
Mice inoculation was used in the past for isolation of C. psittaci and LGV serovars of C.
trachomatis. Others are not infective to mice.
Cell line culture is the traditional method of diagnosis in the past, was considered as
the gold standard method. Though highly specific, it is less sensitive (90% compared
with NAATs ), time consuming, technically demanding and labor intensive.
Choice of the cell line depends on the species-C. trachomatis recommended cell lines
are McCoy, HeLa 229, buffalo, green monkey and baby hamster kidney (BHK-21) cell
lines.
C. pneumoniae can be isolated from HEp2 or human fibroblast cell line.
C. psittaci although grow well in cell culture, isolation should not be attempted in the
routine laboratory because of the risk of laboratory infection.

297. A-More prevalent in developing countries.


B-Toxigenic strains can cause ulcers
D-About 50 % world’s population is affected
Helicobacter pylori, which persistently colonizes the stomachs of ~50% of the world’s
human population, is the main risk factor for peptic ulceration as well as for gastric
adenocarcinoma and gastric MALT (mucosa associated lymphoid tissue) lymphoma.
Humans are the only important reservoir of H. pylori.

298. B-TPHA on serum sample of newborn


C-VDRL on the paired sample of the infant and mother

299. B- Diagnosis of XDR-TB is always laboratory based.


C-Symptoms of XDR-TB are same as drug susceptible tuberculosis.
D-XDR-TB strains are more common in HIV patients.

300. B-Clinical manifestations are similar 01 EL Tor strains


C-Epidemiologically indistinguishable from 01 El Tor strain
139 (Bengal strain): It was isolated first from Chennai in 1992. Since it was not
agglutinated by any of the antisera available available at that time (O 1 to O 138), it
was designated as a new serogroup O 139 or the Bengal strain as it spread rapidly
along the coastal region of Bay of Bengal up to West Bengal, then to the adjacent
areas of Bangladesh.
O139 appears 10 be a derivative of O1 El Tor, but differs from the latter in having a
distinct LPS and being capsulated.
As a result, it is invasive and can cause bacteremia and extraintestinal manifestations
also.
There is no cross protection between O1 and O139.
O139 had caused large-scale outbreaks of clinical cholera and spread rapidly across
almost 11 Asian countries and became a threat to cause the next pandemic.
However, by 1994 the fear had come down and once again the O1 El Tor became
dominant and largely replaced 0139.
Currently, O 139 still causes a minority of cases in India and Bangladesh.

301. A-H. influenzae


B-Streptococcus pneumoniae
D-N. meningitides (group C)

302. B-Ponder’s stain


D-Albert stain
Metachromatic granules: They are present at ends or poles of !he bacilli ( also called
polar bodies or Babes-Ernst bodies or volulin granules).
They are storage granules of the organism, composed of polymetaphosphates.
Granules are stained strongly gram-positive compared to remaining part of the
bacilli.
The granules take up bluish purple metachromatic color when stained with Loeffler’s
methylene blue.
However, they are better stained with special
stains, such as Albert’s, Neisser’s and Ponder’s stain.
Granules are well developed on enriched media, such as blood agar or Loeffler’s
serum slope.
Volutin granules can also be possessed by other organisms such as-by C. xerosi and
Gardnerella vaginalis.

303. B-Clostridium botulinum A, B, E and F cause human disease.


C-The gene for botulinum toxin is coded by a bacteriophage.
D-Clostridium baratti may also cause botulinum.
Toxin types A, B, E, and (rarely) F cause disease in humans.
In intestinal botulism, toxin is produced in and absorbed from the intestine after the
germination of ingested spores.
Intestinal botulism in infants (infant botulism) is the most common form of botulism.
Botulinum toxin is also produced by other species such as C. butyricum, C. baratti
and C. argentinense.
BT types C and D are bacteriophage coded.
Symmetric descending paralysis is characteristic and can lead to respiratory failure
and death.
Cranial nerve involvement, which almost always marks the onset of symptoms,
usually produces diplopia, dysarthria, dysphonia, and/or dysphagia.

304. B-Trench fever


D-Q fever
E-Brill-Zinsser disease

305. B-Pathognomonic histological feature


C-Regional lymphadenopathy
CSD is typically a self-limited illness characterized by regional lymphadenopathy
lasting weeks to months.
The initial clinical manifestation—the primary inoculation lesion—consists of a 0.5- to
1-cm papule, vesicle, or nodule that appears at the site where B. henselae is
introduced and persists for ~1–3 weeks.
Adenopathy typically develops 2–3 weeks after the initial scratch or bite (range, 3–50
days).
Unilateral solitary or regional lymphadenopathy occurs in more than 90% of patients;
its location corresponds with the route of lymphatic drainage.
Lymph nodes are tender, firm, and mobile, and ~10% suppurate; overlying erythema
is occasionally present.
Nonspecific systemic symptoms, such as fever, anorexia, headache, myalgias,
malaise, and abdominal pain, are variably present at this stage.
Lymphadenopathy usually resolves within 3 months.
Early in the course of CSD, histologic examination of lymph nodes reveals follicular
hyperplasia and arteriolar proliferation.
Cortical granulomas, with occasionval multinucleated giant cells, neutrophilic
infiltrates, and coalescing microabscesses (stellate microabscesses), appear within
weeks; the granulomas are surrounded by histiocytes and peripheral lymphocytes.
Warthin-Starry stain may reveal typical clusters of pleomorphic gram-negative
organisms within areas of necrosis, blood vessel walls, or erythrocytes.

306. E- Isoniazid, rifampicin, any fluoroquinolone and kanamycin


Extensively drug-resistant Tuberculosis (XDR-TB) :
Definition: They are MDR -TB cases which are also resistant to:
Fluoroquinolones (ofloxacin /levofloxacin) and
At least one injectable aminoglycosides (kanamycin, amikacin or capreomycin).

307. D- Directly observed treatment short course

308. A- Clostridium botulinum


Ingestion of honey containing the organism is implicated in transmission of infant
botulism.
Affected infants develop weakness or paralysis and may need respiratory support but
usually recover spontaneously.

309. A- It imparts high-level resistance to vancomycin


D- It acts by synthesis of different peptidoglycan

310. C-Coagulase
There are two coagulase-negative staphylococci of medical importance: S.
epidermidis and S. saprophyticus.
S. epidermidis infections are almost always hospital-acquired, whereas S.
saprophyticus infections are almost always community-acquired.
S. epidermidis is part of the normal human flora on the skin and mucous membranes
but can enter the bloodstream (bacteremia) and cause metastatic infections,
especially at the site of implants.
It commonly infects intravenous catheters and prosthetic implants (e.g., prosthetic
heart valves [endocarditis], vascular grafts, and prosthetic joints [arthritis or
osteomyelitis]).
S. epidermidis is also a major cause of sepsis in neonates and of peritonitis in
patients with renal failure who are undergoing peritoneal dialysis through an
indwelling catheter.
It is the most common bacterium to cause cerebrospinal fluid shunt infections.

311. A- H. pylori
Complete bacterial genome has been identified for H.pylori and Haemophilus
influenzae.

312. B- Imipenem
313. A- Is coagulase positive
Staphylococci of Medical Importance

Species Coagulase Typical Important Typical Disease


Production Hemolysis Features1

S. aureus + β Protein A on Abscess, food poisoning, toxic shock syndrome


surface

S.epidermidis − None Sensitive to Infection of prosthetic heart valves and hips;


novobiocin common member of skin flora

5. − None Resistant to Urinary tract


saprophytics novobiocin

All staphylococci are catalase-positive.

314. A-Resistance is produced as a result of alteration in penicillin binding


protein
Approximately 20% of S. aureus strains are methicillin-resistant or nafcillin resistant
by virtue of altered penicillin-binding proteins.
These resistant strains of S. aureus are often abbreviated MRSA or NRSA, respectively.
Such organisms can produce sizable outbreaks of disease, especially in hospitals. The
drug of choice for these staphylococci is vancomycin, to which gentamicin is
sometimes added.
Daptomycin is also useful.
Trimethoprim-sulfamethoxazole or clindamycin can be used to treat non–life-
threatening infections caused by these organisms.
Note that MRSA strains are resistant to almost all β-lactam drugs, including both
penicillins and cephalosporins.
Ceftaroline fosamil is the first β-lactam drug useful for the treatment of MRSA
infections.

315. C- Expression of methicillin resistance in Staphylococcus aureus


increases when it is incubated at 37°C on blood agar
Detection of MRSA :
Antimicrobial susceptibility test: Disc diffusion test can be done by using cefoxitin or
oxacillin discs. Cefoxitin is die recommended disc to be used. If oxacillin disc is used,
then certain conditions are to be maintained such as- using media containing 2-4%
NaCl, incubation at 30 0C and full 24 hours incubation.
Oxacillin screening agar: Adding oxacillin 6 µg/ml and NaCl (2-4%) to the medium.
PCR detecting mecA gene

316. C- Staphylococcus saprophyticus


S. saprophyticus causes urinary tract infections, particularly in sexually active young
women.
Most women with this infection have had sexual intercourse within the previous 24
hours.
This organism is second to Escherichia coli as a cause of community-acquired urinary
tract infections in young women.

317. C- Toxin can be destroyed by boiling for 30 minutes


Food poisoning (gastroenteritis) is caused by ingestion of enterotoxin, which is
preformed in foods and hence has a short incubation period (1–8 hours).
In staphylococcal food poisoning, vomiting is typically more prominent than diarrhea.
Enterotoxin causes food poisoning characterized by prominent vomiting and watery,
nonbloody diarrhea.
It acts as a superantigen within the gastrointestinal tract to stimulate the release of
large amounts of IL-1 and IL-2 from macrophages and helper T cells, respectively.
The prominent vomiting appears to be caused by cytokines released from the
lymphoid cells, which stimulate the enteric nervous system to activate the vomiting
center in the brain.
Enterotoxin is fairly heat-resistant and is therefore usually not inactivated by brief
cooking.
It is resistant to stomach acid and to enzymes in the stomach and jejunum.
There are six immunologic types of enterotoxin, types A–F.

318. B- Staph epidermidis


Please refer to explanation to question 313.

319. B- Vagal stimulation


Please refer to explanation to question 317.

320. D- Staphylococcus aureus


Please refer to explanation to question 317.

321. A- S. aureus
Please refer to explanation to question 317.

322. C-γ toxin


E-Panton valentine toxin
Synergohymenotropic toxins: γ-hemolysin and PV toxin are called as
synergohymenotropic toxins.
Because they are not active individually, but in combination they are capable of
producing hemolytic and leukocidal activity.
There are six combinations possible by the interaction between three fragments of
gamma hemolysin with the two fragments of PV toxin.
PV toxin is expressed on MRSA (methicillin-resistant Staphylococcus aureus) strains,
which are associated with the community acquired infections.
323. A- Staphylococcus aureus

324. A- Staphylococcal food poisoning


325. C- Furuncle
D-Impetigo contagiosa
E-Sycosis barbae
Please refer to explanation to question 323.

326. A- Staphylococcus aureus


Please refer to explanation to question 323.

327. A- Most common source of infection is cross infection from infected


people
S. aureus is a part of the normal human flora; ~25–50% of healthy persons may be
persistently or transiently colonized.
The rate of colonization is higher among insulin-dependent diabetics, HIV-infected
patients, patients undergoing hemodialysis, and individuals with skin damage.
The anterior nares are a frequent site of human colonization, although the skin
(especially when damaged), vagina, axilla, perineum, and oropharynx may also be
colonized.
These colonization sites serve as a reservoir of strains for future infections, and
persons colonized with S. aureus are at greater risk of subsequent infection than are
noncolonized individuals.

328. B- Staphylococcus aureus


Please refer to explanation to question 323.

329. B- Anti-teichoic acid


Teichoic acids are polymers of ribitol phosphate.
They mediate adherence of the staphylococci to mucosal cells.
Lipoteichoic acids play a role in the induction of septic shock by inducing cytokines
such as interleukin-1 (IL-1) and tumor necrosis factor (TNF) from macrophages.

330. B- mecA gene


The isolation of MRSA was reported within 1 year of the introduction of methicillin.
The prevalence of MRSA has since increased steadily.
In many hospitals, 40–50% of S. aureus isolates are now resistant to methicillin.
Resistance to methicillin indicates resistance to all SPRPs as well as to all
cephalosporins.
Production of a novel penicillin-binding protein (PBP 2a or 2′) is responsible for
methicillin resistance.
This protein is synthesized by the mecA gene, which (as stated earlier) is part of a
large mobile genetic element—a pathogenicity or genomic island—called SCCmec.
It is hypothesized that this genetic material was acquired via horizontal transfer from
a related staphylococcal species, such as S. sciuri

331. C- TSS
Toxic shock syndrome is characterized by fever; hypotension; a diffuse, macular,
sunburn-like rash that goes on to desquamate; and involvement of three or more of
the following organs: liver, kidney, gastrointestinal tract, central nervous system,
muscle, or blood.

332. A- S.aureus
Please refer to explanation to question 323.
333. B-Enterotoxin B
C- Enterotoxin C
E- Enterotoxin F

334. D- None
Staphylococci may be introduced into tissue as a result of minor abrasions,
administration of medications such as insulin, or establishment of IV access with
catheters.
After their introduction into a tissue site, bacteria replicate and colonize the host
tissue surface.
A family of structurally related S. aureus surface proteins referred to as MSCRAMMs
(microbial surface components recognizing adhesive matrix molecules) plays an
important role in mediating adherence to these sites.
By adhering to exposed matrix molecules (e.g., fibrinogen, fibronectin), MSCRAMMs
such as clumping factor and collagen-binding protein enable the bacteria to colonize
different tissue surfaces; these proteins contribute to the pathogenesis of invasive
infections such as endocarditis and arthritis by facilitating the adherence of S. aureus
to surfaces with exposed fibrinogen or collagen.

335. A- Staphylococcus aureus

336. B- Beta lysin


Hemolysins of Staphylococcus aureus and their activity

Hemolysins Activities

α- It is inactivated at 70°C but again reactivated paradoxically at 100°C (This is beacuse at 60°C α-
hemolysins hemolysin combines with a heat labile inactivator which gets denatured at 100°C).

It possesses lethal, leukocidal, dermomercrotic, cytotoxic and neurotoxic activities.


It lyses rabbit RBCs, but is less active against sheep and human RBCs.

β-
hemolysins It is sphingomyelinase in nature:

Lyses sheep RBC, but not human or rabbit RBC.


Exhibits hot-cold phenomenon, i.e. hemolysis starts at 37°C but becomes evident only
after chilling.

γ- It has three protein fragments which act together along wtlh leukocidin to exhibit
hemolysins hemolytic activity.
It lyses rabbit sheep and human RBCs.

δ- It lias detergent like (surfactant) action.


hemolysins It lyses rabbit, sheep, horse and human RBCs.
It is also lethal, leukocidal and dermonecrotic.

337. B- Catalase test


Catalase test is the most important test to differentiate Staphylococcus(catalase
positive) and Streptococcus (catalase negative).

338. B- Coagulase negative staphylococcus


Please refer to explanation to question 310.

339. C- Coagulase negative staphylococcus

340. B- Staphylococcus aureus


S. aureus is among the most common causes of foodborne outbreaks of infection in
the West.
Staphylococcal food poisoning results from the inoculation of toxin-producing S.
aureus into food by colonized food handlers.
Anterior nares are the most common site of carriage of S,aureus, so nose picking is a
way to contaminate food.

341. C- Clostridium

342. A- Staphylococcus epidermidis


Prosthetic valve endocarditis (PVE) arising within 2 months of valve surgery is
generally nosocomial, the result of intraoperative contamination of the prosthesis or a
bacteremic postoperative complication.
This nosocomial origin is reflected in the primary microbial causes: S. aureus, CoNS,
facultative gram-negative bacilli, diphtheroids, and fungi.
The portals of entry and organisms causing cases beginning >12 months after
surgery are similar to those in community-acquired NVE.
PVE due to CoNS that presents 2–12 months after surgery often represents delayed-
onset nosocomial infection.
Regardless of the time of onset after surgery, at least 68–85% of CoNS strains that
cause PVE are resistant to methicillin.

343. A- Bacitracin sensitivity


Bacitracin test is used to differentiate Streptococcus pyogenes(Group A) from other
beta hemolytic Streptococci.

344. B- In acute glomerulonephritis, the titer is low

345. A- Streptococcus pyogenes


Streptococcus pyogenes is the most common cause of cellulitis.

346. B- Enterococcus fecalis


Enterococci give a positive bile aesculin hydrolysis lest (they grow in the presence of
40% bile and hydrolyse aesculin to aesculetin chat combines with ferric chloride to
produce black colored complex).
They can grow in presence of extremes of conditions such as- 6.5% NaCl, 40% bile,
pH 9.6, 45°C and 10°C.

347. C- Optochin
It is a typical case of pneumococcal pneumonia.
Optochin sensitivity: Pneumococci are sensitive to optochin disk (5 μg of ethyl
hydrocuprein) and produce wider zone of inhibition (14 mm or more).
Viridans streptococci are resistant to optochin.

348. C- Pneumococcus
After 18 hours of incubation, colonies on blood agar are small (0.5-1 mm), dome
shaped, glistening surrounded by green discoloration due to alpha hemolysis.
On further incubation, colonies become flat with raised edge and central depression
or umbonation due to autolysis of centre of the colonies by autolysin enzyme.
Colonies appear as concentric rings when viewed from above (draughtsman shaped
or carom coin appearance).

349. C- S. mutans
Dental caries: It is mainly caused by S mutans which breaks down dietary sucrose to
acid and dextran.
Acid damages the dentine, while adhesive dextran binds together with food debris,
mucus, epithelial cells and bacteria to produce dental plaques.

350. A- Capsular swelling


Quellung reaction: Capsular serotypes of Streptococcus pneumoniae can be detected
by adding antisera mixed with methylene blue.
Capsule becomes swollen, refractile and delineated.

351. A- Pneumococcus
The risk of serious pneumococcal infection is greatly increased in persons with
conditions that compromise IgG synthesis and/or the phagocytic function of PMNs
and macrophages.
Most patients hospitalized for pneumococcal pneumonia have one or more of these
conditions.

Conditions that Commonly Predispose to Pneumococcal Infection

Increased risk of exposure: Defective complement function:

Day-care centers Defective bacterial clearancea


Military training camps Congenital asplenia, hypospllenia
Prisons Splenectomy
Shelters for the homeless Sickle cell disease
Multifactorial conditions: Infancy and aging
Respiratory infection, inflammation:
Chronic disease
Influenza, other viral respiratory infections Prior hospitalization
Air pollution Alcoholism
Allergies Malnutrition
Cigarette smoking HIV infection
Chronic obstructive pulmonary disease Chronic lung disease
Other causes of chronic pulmonary Glucocorticoid treatment
inflammation or obstruction Cirrhosis of the liver
Renal insufficiency
Anatomical disruption of meninges (dural tear) Diabetes mellitus
Defective antibody formation: Anemia
Coronary artery disease Fatigue, stress,
Common variable hypogammaglobulinemia and/or exposure to cold
Selective IgG subclass deficiency
Multiple myeloma
Chronic lymphocytic leukemia
Lymphoma

Once a pneumococcal infection has been initiated, the absence of a spleen


predisposes to fulminant disease.
The liver can remove opsonized (antibody-coated) pneumococci from the circulation;
in the absence of antibody, however, only the slow passage of blood through the
splenic sinuses and prolonged contact with reticuloendothelial cells in the cords of
Billroth can result in bacterial clearance.
Patients without spleens tend to develop overwhelming pneumococcal disease that
rapidly progresses to death.

352. B -Maltose fermentation


Neisseriae of Medical Importance1

Species Portal of Polysaccharide Maltose β-Lactamase Available


Entry Capsule Fermentation Production Vaccine
N. meningitidis Respiratory + + None +
(meningococcus) tract

N. ganorrhoeae Genital tract - - Some -


(gonococcus)

1 All neisseriae are oxidase-positive

353. B- Deficiency of late complements


Complement is required for bactericidal activity and for efficient opsonophagocytosis.
Individuals deficient in any of the late complement components (C5–C9) cannot
assemble the membrane-attack complex (MAC) needed to kill Neisseria.

354. B- Possess a capsule


Please refer to explanation to question 352.

355. B- N. meningitidis
The most severe form of meningococcaemia is the life-threatening Waterhouse–
Friderichsen syndrome, which is characterized by high fever, shock, widespread
purpura, disseminated intravascular coagulation,
thrombocytopenia, and adrenal insufficiency.
Bacteremia can result in the seeding of many organs, especially the meninges.
The symptoms of meningococcal meningitis are those of a typical bacterial
meningitis, namely, fever, headache, stiff neck, and an increased level of PMNs in
spinal fluid.

356. B- C. perfringens
Necrotizing enteritis (enteritis necroticans or pigbel) is caused by beta toxin produced
by type C strains of C. perfringens after ingestion of a high-protein meal in conjunction
with trypsin inhibitors (e.g., in sweet potatoes) by a susceptible host who has limited
intestinal proteolytic activity.

357. D- C. sporogenes
Some 80% of cases are caused by C. perfringens, whereas C. novyi,C. septicum, and C.
histolyticum cause most of the remaining cases of gas gangrene.

358. D- Tetanus toxoid

359. B- Clostridium difficile


Clostridium difficile–associated disease (CDAD) is a unique colon infection that is
acquired almost exclusively in association with antimicrobial use and the consequent
disruption of the normal colonic flora.
The most commonly diagnosed diarrheal illness acquired in the hospital,CDAD results
from the ingestion of spores of C. difficile that vegetate, multiply, and secrete toxins,
causing diarrhoea and pseudomembranous colitis (PMC).
Risk factors for CDAD include older age, greater severity of underlying illness,
gastrointestinal surgery, use of electronic rectal thermometers, enteral tube feeding,
and antacid treatment.
Use of proton pump inhibitors may be a risk factor.

360. A-Clostridium sordelli


B- Clostridium sporogenes
E- Clostridium botulinum

361. A- Acute oesophagitis

362. D- C2
Of the eight distinct toxin types described (A, B, C1, C2, D, E, F, and G), all except C2
are neurotoxins; C2 is a cytotoxin of unknown clinical significance.

363. D- Type 6

364. A- C. tertium
C. tetani-produces spherical and terminal spore (drumstick appearance).
C. tertium-produces oval and terminal spore (tennis racket appearance).
C. bifermentans-produces central and oval spore.

365. A- C. perfringens
C. perfringens and C. tetani type 6 are non-motile Clostridia.

366. A- Clostridium perfringens


Some 80% of cases are caused by C. perfringens, whereas C. novyi,C. septicum, and C.
histolyticum cause most of the remaining cases of gas gangrene.

367. B- Diarrhea
Food-Borne Botulism:
After ingestion of food containing toxin, illness varies from a mild condition for which
no medical advice is sought to very severe disease that can result in death within 24
h.
The incubation period is usually 18–36 h but, depending on toxin dose, can range
from a few hours to several days.
Symmetric descending paralysis is characteristic and can lead to respiratory failure
and death.
Cranial nerve involvement, which almost always marks the onset of symptoms,
usually produces diplopia, dysarthria, dysphonia, and/or dysphagia.
Weakness progresses, often rapidly, from the head to involve the neck, arms, thorax,
and legs; occasionally, weakness is asymmetric.
Nausea, vomiting, and abdominal pain may precede or follow the onset of paralysis.
Dizziness, blurred vision, dry mouth, and very dry, occasionally sore throat are
common.
Patients are generally alert and oriented, but they may be drowsy, agitated, and
anxious.
Typically, they have no fever.
Ptosis is frequent; the pupillary reflexes may be depressed, and fixed or dilated pupils
are noted in half of patients.
The gag reflex may be suppressed, and deep tendon reflexes may be normal or
decreased. Sensory findings are usually absent.
Paralytic ileus, severe constipation, and urinary retention are common.

368. A. Clostridium perfringens


Smears of C. perfringens shows large gram-positive rods typically look like Box car
appearance.
369. B. Plasmid
Tetanospasmin is a neurotoxin which is plasmid mediated.
It prevents the release of inhibitory neurotransmitter GABA and glycine in pre
synaptic area which leads tospastic muscle contraction.

370. B. C. perfringens
Nagler reaction is a useful test for rapid detection of C. perfringens in clinical
specimens
This reaction demonstrates biological property of the enzyme lecithinase—to produce
opalescence in the serumand in the egg yolk media.
This reaction is specifically neutralized by the use of specific antitoxin.

371. A. Inhibition of GABA release


Refer Q. no 369.

372. D. Antibiotic use


Antibiotics suppress drug-sensitive members of the normal flora, allowing C.difficile
to multiply and produce exotoxins A and B.
Both exotoxin A and exotoxin B are glucosyltransferases
The main effect of exotoxin B in particular is to cause depolymerization of actin,
resulting in a loss of cytoskeletal integrity, apoptosis, and death of the enterocytes.
Clindamycin and 3rd gen cephalosprins are commonly involved drugs.

373. C. Susceptible to diphtheria


Susceptible individuals develops erythema and induration on test arm
followingintradermal inoculation of toxin.
Vaccination is needed only to these susceptible population.

374. D. Corynebacterium
Ehrlich phenomenon - the difference between the amount of diphtheria toxin that will
exactly neutralize one unit of antitoxin.

375. C. Diphtheria
Diphtheria – leather
Anthrax - Coal

376. A. Cause cranial nerve palsies in 2nd and 3rd week


Gram positive bacilli.
Passive immunization is life saving management in respiratory diphtheria.
Best treatment is anti toxin.

377. A. Ponder’s stain


Metachromatic granules are best stained by Albert, Ponder and Neisser stain.

378. A. Elek’s gel ppt test


It is an in vitro neutralizationreaction between toxin and antitoxin.
The toxin produced by the growth of the test strain diffuses into the agar and meets
the antitoxin at the optimalconcentration, and the line of precipitation can be seen.
In strains that are negative, no precipitin lines are seen.

379. A. Throat swab culture


Carriers and early stage of cases are best diagnosed by throat swab culture.

380. C. Cysteine-tellurite agar


Tellurite medium is very useful for isolation of C. diphtheriae from contacts, carriers,
and convalescents, in which clinicalspecimens contain a number of other bacteria.
The bacteria on tellurite medium produce characteristic black to grayish black
colored colonies after 48 hours of incubation

381. C. Klebs-Löffler bacillus


Koch bacillus – M.tuberculosis
Yersin& Roux – Diphtheria toxin

382. A. Corynebacterium minutissimum


Erythrasma is a bacterial infection that affects the skin.
It usually appears in the folds of the skin. It’s more commonly seen in warm or humid
climates, and is usually caused by the bacteria Corynebacterium minutissimum.

383. A. Susceptibility to diphtheria


The Schick test, invented between 1910 and 1911, is a test used to determine
whether or not a person is susceptible to diphtheria.
It was named after its inventor, Béla Schick.

384. B. Presence of abdominal pain


The symptoms consist of acute abdominal pain, diarrhea, and nausea; vomiting is
rare.
The disease generally lasts for 24 hours or more. The fecal samples contain small
numbers of B. cereus.
Diarrheal form of food poisoning is caused by B. cereus serotypes 2, 6, 8, 9, 10, or 12

385. B. B. cereus
It produces secreted and a pre formed toxin which are non invasive.

386. A. B. anthracis
The typical lesion of cutaneous anthrax is a painless ulcer with a black eschar
(crust,scab).
Local edema is striking. The lesion is called a malignantpustule.
Untreated cases progress to bacteremia and death

387. A. Cutaneous anthrax


Refer Q. no 386.

388. D. Brain abscess


N. asteroides typically causes either pneumonia, lung abscess with cavity
formation,lung nodules, or empyema. From the lung, the organism can spread to
various organs, notablythe brain, where it causes brain abscess.
Disease occurs most often inimmunocompromised individuals, especially those with
reduced cell-mediatedimmunity.

389. A. Paraffin bait technique


Paraffin baiting is a routine laboratory diagnostic procedure for the isolation of
Nocardia asteroids.
390. A. Can erode bones&d.Slow growing
Madura foot is a chronic infection of the skin and underlying tissues caused by both
bacteria (actinomycotic mycetoma or actinomycetomas) and fungi (eumycetomas or
mycotic mycetoma).
Mycetoma occurs most often in people who work in rural areas, usually in farmers,
hunter-gatherer populations, and field labourer.

391. A. Anthrax
Pulmonary (inhalation) anthrax, also known as “wool-sorter’s disease,” begins with
nonspecific respiratory tract symptoms resembling influenza, especially a drycough
and substernal pressure.
This rapidly progresses to hemorrhagic mediastinitis, bloody pleural effusions, septic
shock, and death

392. A. Bacillus anthracis


On nutrient agar after 24 hours of incubation, B. anthracis produces grayish and
granular colonies measuring2–3 mm in diameter.
Under low-power microscopy, the edges of the colony appear as long, interlacing
chains of bacilli, resembling locks of matted hair, which gives them a “medusahead”
appearance with an uneven surface and wavy margin

393. D. Mycetoma
Mycetoma is characterized by a triad of painless subcutaneous mass, multiple sinuses
and discharge containing grains.
It usually spreads to involve the skin, deep structures and bone resulting in
destruction, deformity and loss of function, which may be be fatal.
Mycetoma commonly involves the extremities, back and gluteal region.

394. B. Actinomadura madure


Most common cause in India is A. madurae followed by M.mycetomatis and Nocardia
brasiliensis.

395. A. Cervicofacial
The typical lesion of actinomycosis appears as a hard, nontender swelling
thatdevelops slowly and eventually drains pus through sinus tracts.
Hard, yellow granules (sulfur granules) composed of a mass of filaments are formed
in pus.
In about 50% of cases, the initial lesion involves the face and neck (Cervicofacial
type).

396. A. Listeria monocytogenes


Among the choices, the only Gram positive bacilli is Listeria monocytogenes.
It is associated with meningitis (Usually neonatal meningitis)

397. A. Listeria monocytognes


Listeria must then navigate to the cell’s periphery to spread the infection to other
cells.
Outside the body, Listeria has flagellar-driven motility, sometimes described as a
“tumbling motility”.

398. B. Withhold breastfeeding


The mother and infant should stay together
The infant should be breastfed in the normal way.
If the mother is sputum smear positive just before delivery: give the mother anti-
tuberculosis treatmentand advise her to continue breastfeeding; give the infant
preventive chemotherapy with isoniazid for 6 months.

399. C. LJ media & B. Dorset media


NNN medium – Leishmania
Nutrient agar – Basal medium
Egg Based medium for MTB- Lowenstein Jensen, Petragnani, Dorset, Blood Based
Media – Tarshis

400. A. Uterus
Although Leprosy rarely involves the female genital tract, the ovary is the most
commonly involved gynaecological site

401. C. Cyclosporine
Cyclosporine is used to prevent organ rejection in people who have received a liver,
kidney, or heart transplant.
It is usually taken along with other medications to allow your new organ to function
normally.
Cyclosporine belongs to a class of drugs known as immunosuppressants.
Drugs used for Lepra reaction II are thalidomide, glucocorticoid, clofazimine &
antipyretics

402. B. Cell mediated


Tuberculin test is based on type IV HSR
Applications are
It is used for evaluation of cases that have tuberculosis and for diagnosis of cases
suspected to be infected withM. tuberculosis.
It is used to diagnose active tuberculosis infection in infantsand young children.
It is also used to select population susceptible for BCG(bacille Calmette–Guerin)
vaccination.
It is a valuable test to measure prevalence of tuberculosisinfection in a community.

403. B. 5%
M.tuberculosis – 20 %, asper RNTCP -25%
Nocardia – 1% & 0.5% (fluids)
Bacterial spore – 0.25%

404. B. Amidase test


Atypical mycobacteria are positive for Amidase test and Aryl sulfatase test.

405. A. Contraindicated
BCG vaccine is contraindicated in people with impaired immunity, and WHO does not
recommend BCG vaccination for children with symptomatic HIV infection.

406. B. Armadillos
Nine banded Armadillo is highly susceptible to leprosy, due to low body temperature.
Next is foot pad of mice.
Other animal models are Korean chipmunk, Indian pangolin, Slender loris and
European hedgehog.
407. C. Sputum culture
Sputum culture is the gold standard method for the diagnosis for pulmonary
tuberculosis.
Sputum microscopy is a screening test with low sensitivity and high specificity.

408. D. Mutation
Mutation in the following genes leads to resistance to the respective drugs.
KatG, Inh A, ampC - Isoniazid
rpoB - Rifampicin
Pnc A - Pyrazinamide
emb A, B, C – Ethambutol
rpSL - Streptomycin

409. A. Armadillos
Refer Q. no 406

410. D. M. smegmatis
This organism is classified as saprophytic and therefore relatively safe.
Mycobacterium smegmatis doesn’t normally reside in any animals, and doesn’t cause
dangerous or even any infections.
There have only been a few threats, which have seem to come out in only extreme
cases, however there hasn’t been any documented virulence of Mycobacterium
smegmatis in over 15 years.

411. C. M. ulcerens
Buruli ulcer, caused by Mycobacterium ulcerans is a chronic debilitating disease that
affects mainly affects the skin and sometime bone

412. C. Poncet’s disease


Tubercular rheumatism (Poncet’s Disease) is a non-destructive parainfective
symmetric polyarthritis occurring in patients with active visceral or disseminated
tuberculosis, in which there is neither evidence of bacteriological involvement of joint
themselves nor any other known cause of polyarthritis detected.

413. A. Lepromatous leprosy


Lepra type II reactions are type III humoral hypersensitivity reactions with systemic
inflammatory response due to deposition of immune complexes.
This condition occurs in 20% of patients with lepromatousleprosy and in 10% of
patients with borderline leprosy.
Thisreaction occurs after a few years of therapy and relapse intermittently over
several years.
Appearance of crops of painful erythematous nodules on the skin and subcutaneous
tissue is the characteristic manifestation. Fever, malaise, arthralgia, neuralgia,
iridocyclitis, arthritis, and proteinuria are other symptoms

414. A. Enteroaggregative E. coli is not associated with persistent diarrhea


EAEC strains are so called because they show a typical “stacked brick” arrangement
on Hep-2 cells orglass due to their autoagglutination.
EAEC increases mucus secretion, which forms a layer overlying the epitheliumof the
small intestine.
These strains are associated with persistent,watery diarrhea with dehydration in
infants, especially in developing countries

415. A. Sereny test is positive


Sereny test is positive for EIEC.

416. D. Lectin
Escherichia coli is able to adhere to epithelial cells of the gastrointestinal tract
because lectins on the E. coli surface recognize oligosaccharide units on the surfaces
of target cells.
These lectins are located on slender hairlike appendages called fimbriae (pili).

417. A. VI agglutination test


Salmonella typhi carriers are detected by Stool and urine culture and detection of
antibodies to Vi antigen in serum.

418. C. Monovalent vaccine


Monovalent typhoid vaccines: Vi polysaccharide is a well-standardized antigen that is
effective in a single parenteral dose, is safer than whole-cell vaccine, and may be
used in children 2 years of age or older

419. C. Caused by animal products & D.Symptoms appear between 4-48 hours
It’s a zoonotic disease and incubation period is few hours to 72 hours.
Main diagnosis is culture.

420. A. Can be given in patients with yellow fever and hepatitis B


It is composed of Vi capsular polysaccharide antigen derived from S.Typhi strain Ty2.
Less side effects than parenteral TAB vaccine
Protection period is 2 years.

421. A. Pseudomonas & C. V. cholerae


V. cholerae belongs to family Vibrionaceae
Pseudomonas belongs to family Pseudomonadaceae

422. A. H. pylori
Most potent urease producer than any other organism is H. pylori

423. B. Salmonella typhi


Multidrug-resistant Salmonella enterica serotype Typhimurium definitive phage type
104 (DT 104) has emerged during the last decade as a global health problem
because of its association with animal and human disease.

424. A. All are oxidase negative


All Enterobacteriacae are oxidase negative and catalase positive
Ferments glucose
Usually motile by peritrichous flagella

425. B. Typhoid
The motile bacterium Salmonella typhi, which is known to infect onlyhumans, is
spread by ingestion of food, drink or other materialcontaminated by this organism.
About 2 weeks after infection with Salmonella typhi, most people suffering from
typhoid develop a yellow-green foul liquid stool that resembles pea soup in
appearance – pea soup stool.

426. A. 0157:H7
HUS and Haemorrhagic colitis caused by EHEC serotype O157 : H7

427. A. 10-14 days


The incubation period for typhoid fever is usually 8–14 days,

428. A. Salmonella
The most common sources of salmonellae are milk and milk products, meat, poultry,
and eggs.
Of great concern are eggs and egg products. Salmonellae can enter through the shell
if eggs are left on contaminated chicken feed or feces andgrow inside.

429. C. S.Gallinarum
Salmonella are motile with the presence of peritrichous flagella except Salmonella
Gallinarum and Salmonella Pullorumwhich are nonmotile.

430. A. Shigella sonnei


All shigella species are non lactose fermenters except S. sonnei which is late lactose
fermenter.

431. D. Mannitol
Shigella ferments mannitol, forming acid but no gas. Mannitol fermentation test is an
important biochemical test, which is used to classify shigellae into mannitol-
fermenting and -nonfermenting species. S. flexneri, S. boydii, and S. sonnei
aremannitol-fermenting species, while S. dysenteriae is mannitolnonfermenting
species.

432. D. Shigella dysenteriae


Shigella dysenteriae causes bacillary dysentery.
It is an acute gastrointestinal illness manifested by fever, vomiting,abdominal
cramps, and tenesmus.
Incubation period is usually short. It lasts from 12 hours to 7 days, usually 48 hours,
and is inversely related to the load of ingestedbacilli.
The condition manifests with a sudden onset of high-grade fever along with
abdominal cramp, tenesmus, urgency, andpassage of loose, scanty feces containing
frank blood and mucus.

433. A. Increasing cAMP


The Heat labile toxin consists of one A subunit and five identical B subunits.
The B subunits bind to the GM1 gangliosides and facilitates entry of subunit A into
the cell by endocytosis.
The A subunit has ADP (adenosine diphosphate)- ribosyl transferase activity, this
results in an increase in cyclic adenosine monophosphate (cAMP) levels leading to an
increased secretion of chloride and a decreased absorption of sodium and chloride.
This ends in watery diarrhea due tohypersecretion of fluid into the lumen of the gut.

434. B. Heat stable E. coli toxin


Heat stable E. coli toxin is mediated by cGMP

435. C. MacConkey’s medium


E. coli is lactose fermenter.
MacConkey agar has Neutral red as an indicator.
It produces pink color colonies due to lactose fermentation and acidic pH.

436. C. ETEC
Diarrhea caused by ETEC is endemicin the developing countries, among all age
groups of the population.
This is also responsible for causing traveler’s diarrhea in which individuals from
developed countries visitingendemic areas often suffer from ETEC diarrhea.
It causes diarrhea because of its ability to produce heat-labile enterotoxins (LT-I, LT-II).
LT-I, which is structurally similar to choleratoxin, produces cholera-like diarrhea in
patients.

437. C. P- pili present in uropathogenic type


ETEC is non invasive
EPEC–Attachment/Effacement lesions
ETEC causes travellers diarrhoea

438. B. EHEC

439. A. Pseudo lymphadenopathy

440. B. Agglutination
Widal test is tube agglutination test for the detection of O and H anibodies.
Diagnostic test for enteric fever

441. B. Buffered glycerol saline medium


Buffered Glycerol Saline Base with added glycerol is used in the collection and
transportation of faecal specimens.

442. C. Produces 01 lipopolysaccharide


V. Cholerae O139 possesses serogroup O139 LPS

443. A. ADP ribosylation of G regulatory protein


Cholera toxin causes ADP ribosylation of G protein which upregulates the activity of
adenylate cyclase results in the intracellular accumulation of cyclic AMP.
Increase in cyclic AMP leads to Accumulation of sodium chloride and water
resultingin watery diarrhea. Loss of fluid and electrolytes leads to dehydration and
shock.

444. A. cAMP
Refer Q. No 443

445. b. TCBS medium


TCBS medium (pH 8.6): This medium contains thiosulfate,citrate, bile salts, sucrose,
and bromothymol blue (indicator).
V. cholerae produces large, yellow convex colonies onthis medium

446. C. Vibrio vulnificus


V. vulnificus is also a marine organism (i.e., it is found in warm salt waters such asthe
Caribbean Sea).
It causes severe skin and soft tissue infections (cellulitis), especially in shellfish
handlers, who often sustain skin wounds. It can also cause a rapidly fatal septicemia
in immunocompromised people who have eaten rawshellfish containing the
organism.
Hemorrhagic bullae in the skin often occur in patients with sepsis caused by V.
vulnificus. Chronic liver disease (e.g., cirrhosis)predisposes to severe infections.
The recommended treatment is doxycycline

447. B. 0:139
In 1992, O139 serogroup was identified in Chennai and the strain was named as
Bengal strain.

448. B. Cholera
Incubation period for
Cholera – few hours to 5 days
Plague – 2- 6 days
Typhoid – 8 – 14 days
Measles – 10 – 12days

449. A. Can tolerate wide range of alkaline pH


It grows well in alkaline pH (8.4 -8.6)
Non halophile
Darting motility
Selective medium is TCBS medium

450. B. Cary Blair medium


V-R medium and Cary Blair medium are the transport media for V. cholerae.
MYPA medium – B. cereus
LJ medium – M.tuberculosis
Stuart medium – transport medium for Gonococci

451. A. Gram negative rod & D. It is achlorhydria which renders an individual


susceptible to disease.
The pathogenesis of cholera is dependent on colonization of the small intestine bythe
organism and secretion of enterotoxin.
For colonization to occur, large numbers of bacteria must be ingested because the
organism is particularly sensitive tostomach acid.
Persons with little or no stomach acid, such as those taking antacids orthose who
have had gastrectomy, are much more susceptible

452. A. It is caused by Burkholderia mallei


B. pseudomallei is also known as Whitmore’s bacillus, is the causative agentof
melioidosis in rodents.
B. pseudomallei was first described by Whitmore and Krishnaswamy
Selective medium is Ashdown medium

453. B. Melioidosis
Melioidosis is characterized by development of anodule at the site of inoculation of
the bacteria in the skin.
The bacteria can subsequently spread, causing secondary lymphangitis, regional
lymphangitis, fever, and myalgia.
Acute melioidosis may progress rapidly to acute septicemia with highmortality rate.
Acute blood stream infection is most commonlyseen in patients with HIV, diabetes,
renal failure, etc. The condition results in septic shock.

454. B. Gram -ve, D. Non-motile & E. It is coccobacilli


Y. pestis is a Gram-negative coccobacillus
It is a nonmotile, nonsporing, and non–acid fast.

455. C. Wayson’s stain


It is a case of bubonic plague which is caused by Y. pestis.
Wayson stain is used for the demonstration of the typical bipolar (safety pin)
morphology of Y. pestis.

456. C. Pseudomonas
Blue pus is the characteristic feature of Pseudomonas aeruginosa infections due to
pyocyanin pigment production.

457. D. Bacteria
In humans, B. mallei may cause acute or chronic infectionwith that localized to the
skin, subcutaneous tissue, or respiratory tract.
Human cases of glanders are usually rare. Commonly seen in horses.

458. B. Pseudomonas
P. aeruginosa produces different types of pigments
1.pyocyanin, 2.pyoverdin, 3.pyorubin, and 4.pyomelanin

459. C. inhibition of Protein synthesis


Pseudomonas toxin is Exotoxin A
It acts on EF2 and preventing synthesis of proteins ineukaryotic cells

460. C. Kerala
In India, outbreaks are reported Kolar, Surat, Beed-Latur belt in Maharashta, Rohru in
Himachal Pradesh, Dangud (Uttar kasha) in Uttaranchal and Surat in Gujarat.

461. A. Pseudomonas
Ecthyma gangrenosum is a cutaneous infection most commonly associated with
Pseudomonas bacteraemia.
Ecthyma gangrenosum usually occurs in patients who are critically ill and
immunocompromised.
The characteristic lesions of ecthyma gangrenosum are haemorrhagic (bloody)
pustules that evolve into necrotic (black) ulcers.

462. B. Protein capsule plays an important role in pathogenicity


H. influenzae possesses polysaccharide capsule which plays an important role in
pathogenesis.

463. D. Infection can be prevented by a acellular vaccine


Acellular vaccine will prevent the disease and not the infection.
It contains pertussis toxoid & FHA,pertactin or fimbriae.
It is safer than whole cell vaccine

464. A. 7-14 days


The incubation period of pertussis is commonly 7 to 10 days, with a range of 4–21
days.

465. B. Erythromycin should be given to contacts


Protection is 85%
Leukocytosis is a surrogate marker and not diagnostic
DOC for Chemoprophylaxis is Erythromycin.

466. D. Bordetella pertussis


The capsular antigen- K antigen is heat labile. These K antigens are of different types
and are used for differentiating B. pertussis isolates in epidemiological studies.
It is not acting as a virulent factor.

467. D. Filamentous hemagglutinin, pertussis toxin


Acellular vaccine contains pertussis toxoid and more than two other bacterial
components including FHA, pertactin or fimbriae

468. D. Brucella spp.


Brucella is zoonotic disease. No Man to Man transmission.
After an incubation period of 1 to 3 weeks, nonspecific symptoms such as fever,chills,
fatigue, malaise, anorexia, and weight loss occur. The onset can be acute orgradual.
The undulating (rising-and-falling) fever pattern that gives the disease itsname
occurs in a minority of patients. Enlarged lymph nodes, liver, and spleen
arefrequently found.
Pancytopenia occurs.
B. melitensis infections tend to be moresevere and prolonged, whereas those caused
by B. abortus are more self-limited.
Osteomyelitis is the most frequent complication. Secondary spread from person
toperson is rare.

469. A. Standard agglutination test


Standard tube agglutination test: This test remains the standard method and is the
most commonly used serological testfor diagnosis of brucellosis.
It detects the presence of antibodiesagainst LPS component of Brucella.
It cannot differentiate acute and chronic stages of the disease

470. B. Brucella melitensis


Malta fever, Undulant fever, Mediterranean fever are caused by Brucella melitensis.

471. D. Brucella melitensis


Refer Q. No 468

472. A. Man to man transmission


Refer Q. No 468

473. D. 2 ME is used to detect IgA


2-mercaptoethanol is added to the patient’s sera before testing it.
Addition ofmercapto-ethanol causes disruption of disulfide bond of IgM; hence only
IgG is detected.

474. A. Brucellosis
This is a frequently used serological test fordemonstration of antibodies in the milk of
an animal.
This isa screening test used to detect the presence of Brucella infection in infected
cattle.
In this test, a concentrated suspension of killed B. abortus or B. melitensis stained
with hematoxylin is usedas antigen.

475. B. Chancroid
H. ducreyi is an important causative agent of sexually transmitted disease called soft
sore or chancroid.

476. Hemophilus
A suspected isolate ofH. influenzae is streaked on a blood agar plate. Then S. aureusis
streaked across the same blood agar plate and incubatedat 37°C for 18–24 hours.
After incubation, the colonies ofH. influenzae nearer to the S. aureus are larger than
those awayfrom it. This phenomenon is known as satellitism.
This demonstrates that V factor is available in increased concentration near the
staphylococcal colony and in a lower concentration away from it.

477. C. Capsule
Thecapsular K antigen is heat labile. These K antigens are ofdifferent types and are
used for differentiating B. pertussis isolates in epidemiological studies.

478. C. Regan-Lowe medium


Regan-Lowe medium is a selective medium for Bordetella pertussis.

479. A. Human is the only reservoir


Campylobacter infections are zoonotic. Infected animals arethe main reservoir of the
infection.
The animal reservoirs arethe dogs, cats, and other pets that carry the organism in
theirgastrointestinal tract.
Infected animal food products are thesource of infection.

480. B. Culture on Skirrow’s medium incubated at 42°C under microaerophilic


condition
Skirrow’s medium is the selective medium for C. jejuni.
C. jejuni is microaerophilic and grows well at 420C
TCBS - Vibrio
Wilson Blair medium - Salmonella

481. C. Gastric leiomyoma


H. pylori associated with Gastric ulcer, Gastric lymphoma, adeno carcinoma.

482. A. With chronic infection urea breath test become negative


UBT is prognostic test. Till the presence of the infection / organism UBT is positive.

483. C. Transmitted from man to man, Feco-orally and by oro- gastric route
Humans are the primary reservoirs of infection.
Poor hygiene, overcrowding, and poverty facilitatetransmission.
The most likely route of H. pylori infection is eitherfecal-to-oral infection (from stool to
mouth) or oral-to-oral(stomach contents transmitted from mouth to mouth) contact.

484. A. Campylobacter
Example for Microaerophilc bacteria are
C. jejuni
H. pylori
M. bovis
L.interrogans

485. A. H. pylori
H. pylori is the most potent urease producing microbe.

486. C. All children in developing countries have immunity by five years of age
H. pylori infection induces the production of IgM, IgG, and IgAantibodies and also
cellular immunity, but they do not appearto confer any protection against the disease

487. D. Urease activity provides protective environment to the bacilli


H. pylori infection begins by colonization of gastric mucosa,which is facilitated by
many virulence factors.
H. pylori blocksthe production of acid by its acid-inhibitory protein andalso
neutralizes the gastric acidity by production of ammonia by splitting urea by the
enzyme urease.
The biologicalactivity of urease is enhanced further by heat shock proteins.
Finally,Helicobacter infection causes atrophic and even metaplasticchanges in the
stomach.

488. A. Breath test


Most sensitive invivo test for H. pylori is Urea breath test.
H. pylori is the most potent urease producing microbe.

489. D. Colorectal cancer


H. pylori resides in Gastric antrum.
It causes gastric ulcers, Duodenal ulcer and gastric adeno carcinoma.

490. B. Skirrow’s medium


Skirrow’s medium, Campy BAP medium, Blaser medium are the selective medium for
C. jejuni
BCYE medium – Legionella

491. A. Gram positive bacillus


H. pylori is a curved, spiral, or S-shaped Gram-negative bacillus measuring 3 m in
length and 0.5–0.9 m in breadth.
Cork screw motilitydue to presence of a unipolar tuft of lophotrichous flagella.

492. C. Can grow at temperature around 42°C


Darting motility due to Monotrichous flagella
MCC of diarrhea in USA.
Zoonotic disease
493. B. H.pylori
A flagellum is a lash-like appendage that protrudes from the cell body of certain
bacteria.
Campylobacter jejuni& H. pylori has seven protofilaments.

494. B. Legionella
Buffered charcoal yeast extract (BCYE) agarcontaining buffered charcoal, yeast
extract, and cysteine isa useful medium for growth of L. pneumophila.
L. pneumophila is a small, slender, pleomorphic, Gramnegative bacillus.

495. C. From inhalation of the aerosol in the air-conditioned room at


convention center
Person-to-person transmission does not occur. Animals also donot play any role in
transmission of the infection
Inhalation of aerosolized mist from contaminated watersource, which is
contaminated with either the bacteria orfree-living amoeba infected with the
bacteria.
Nosocomial infection is transmitted through aspiration,contaminated water, or
respiratory equipment.

496. A. Legionella
Incubation period varies from 2 to 10 days.
Pneumonia is the primary manifestation of Legionnaire’s disease with multi-lobular
consolidation, inflammation, and abscesses in the lung.
Fever, chills, dry or nonproductive cough, and pleuritic or non- pleuritic chest pain are
the common symptoms of the disease.
Hyponatremia and diarrhea
Extrapulmonary manifestations include myocarditis, pericarditis, and prostheticvalve
endocarditis.

497. C. Legionella are communicable from infected patient to others


Refer Q. No 495.

498. D. Legionella pneumophila


Pontiac fever is a mild, self-limiting condition. The conditionpresents as fever and
myalgia that resolve without treatment.

499. C. Legionella
Refer Q. no 494

500. A. Legionella
Refer Q. no 495

501. B. Legionella pneumophila


Refer Q. No 494

502. B. Motile gram negative


L. pneumophila is a small, slender, pleomorphic, Gramnegative bacillus.
Motile in nature

503. B. Q fever
No vector for human Q fever. It is zoonotic

504. C. Adult female feeds on vertebrate hosts


Four stages of developmentusually take place in the life cycle of the mite.
Of these stages,only larval stage (chigger) is infectious to humans and other
mammals because these stages require blood meal for further development

505. C. Rickettsia prowazekii


Louse is the vector for R. prowazekii.
Epidemic typhus is characterized by highfever, severe headache, and chills.
Appearance of a petechial ormacular rash on the fourth or fifth day – first starting on
thetrunk and then spreading over to the extremities but withoutaffecting the face,
palms, and sole – is the characteristic featureof the condition.
This rash is seen in nearly 40% of patients

506. B. Trombiculid mite


Genus Orientia contains Orientia tsutsugamushi, the causativeagent of scrub typhus.
It is transmitted by Trombiculid mite

507. B. Flea is a vector of the disease


Endemic typhus - Rats (Rattus rattus), mice,and cats are the natural reservoirs of
infection. Humans arethe accidental hosts.
Rat flea (Xenopsyllacheopis) or cat flea(Ctenocephalides felis) are the main vectors
responsible for thetransmission of disease.

508. D. Epidemic typhus


Refer Q. no 505.

509. D. Trombiculid mite


Refer Q. no 506

510. A. Crimean Congo fever & B. Rocky mountain spotted fever


Tick borne disease are Tick Typhus, Viral Encephalitis, Viral Fevers, Viral Hemorrhagic
fevers (e.g. KFD), Tularemia, Tick Paralysis, Human Babesiosis.

511. A. McCoy cell line


Chlamydia grows well in McCoy cell lines.
It is obligate intra cellular organism.

512. B. Elementary body is metabolically active


The reticulate body (RB) is a large, noninfectious form of Chlamydia.
It is metabolically active and replicating form of Chlamydia.

513. D. The group specific antigen is responsible for the production of


complement fixing antibodies
Genus-specific antigenis a heat-stable, complement-fixing, and genus-pecificantigen.
It is an LPS–protein complex resembling the LPS ofGram-negative bacilli. It is present
in EBs and RBs.
The antigencan be extracted by ether, chloroform, or methanol. The antigen is
identified by CFT
514. B. Culture on McCoy cells
Refer Q. No 296

515. A. Chlamydia trachomatis


Chlamydia is the MCC of NGU
H. ducreyi and Gonococci take Gram stain.
Urethral discharge is not a feature of syphilis

516. D. Urethritis
Serotype D – K causes adult inclusion conjunctivitis, neonatal conjunctivitis, infant
pneumonia, and urogenital infections.

517. A. Infertility & C. Pelvic inflammatory disease


Chlamydia trachomatis is the MCC of PID.
The clinical manifestations in symptomatic patients includeurethritis (nongonococcal
urethritis), epididymitis, proctitis, and conjunctivitis in males
In females, it causes a mucopurulent cervicitis, endometritis, and salpingitis.
Ascending infection can result in pelvic inflammatorydisease, chronic pelvic pain, and
perinephritis.

518. D. Yolk sac inoculation


C. trachomatis grows better in various tissue cultures, using nonreplicating
stationary-phase cells. The bacteria can grow in a few cell lines, such as HeLa-229,
McCoy, BHK-21, and buffalo green monkey kidney cells. McCoy and HeLa cells
arefrequently used for the isolation of the bacteria.
C. trachomatis can be grown by inoculation into embryonated eggs. Chlamydia spp.
grow in yolk sac of 6–8 days old chickembryo.

519. A. Chlamydia trachomatis


MCC of NGU is Chlamydia trachomatis
H. ducreyi is Gram negative bacilli.
MTB and T. pallidum not mainly involved in urethritis.

520. B. Nucleic acid amplification testing


Molecular diagnosis of C. trachomatis infection is made by DNA probes and PCR
(polymerase chain reaction). DNA probes are currently available, which detect the
presence ofa species-specific sequence of 16S rRNA.
PCR is used for diagnosis of infection with a reported sensitivity of 80–90% and
specificity of 99%. The methodis useful specifically for testing urine specimens.

521. B. Endothelial cells of small vessels


Rickettsiae are primary pathogens of arthropods, such as lice, fleas, ticks, and mites.
In these hosts, they are found in theirintestinal tract.
They are usually transmitted to humans by arthropod vectors, such as lice, mites,
ticks, etc.
They also infect humans in whom they arefound in the reticuloendothelial cells and
vascular endothelium

522. B. Guinea worm infection&c. Plague


Anthropozoonosis is defined as any disease that is transmitted from animals to
humans.
Examples are rabies, Brucellosis, Hydatid cyst…etc.

523. E. Pancreatitis
Chlamydia causes trachoma,adult inclusion conjunctivitis, neonatal
conjunctivitis,infant pneumonia, andurogenital infections.

524. C. Q-fever
Q fever caused by Coxiella burnetti.

525. C. Lymphogranuloma venereum


Esthiomene is a medical term referring to elephantiasis of the female genitals.
In the past the term has also referred to elephantiasis of the male genitalia.
Esthiomene is generally the visible result of lymphogranuloma venereum, lymphatic
infection by Chlamydia trachomatis.

526. A. Rickettsial infection


It helps to differentiate R. typhi which is positive and R. prowazekii which is negative
for the test.
It is otherwise called as tunica reaction.

527. C. M. leprae
M. leprae is non cultivable. It does not obey Koch’s second postulate.

528. B. Is an intracellular organism


Chlamydiae are obligate intracellular parasites of humans and animals with marked
affinity for the squamous epithelial cells
of the gastrointestinal and respiratory tracts.

529. D. Coxiella burnetii


C. burnetii is the causative agent of Q fever, a zoonotic disease transmitted from
animals to humans.

530. D. Q fever
Weil – Felix reaction is not useful in Q fever, Rickettsial pox and Trench fever.
They failed to produce heterophile antibodies.

531. A. Rat flea


Rat flea is the vector for endemic typhus
Mite is for R.pox and scrub typhus
Tick is for RMSF and Indian tick typhus.

532. C. Bartonella
Bartonella can be isolated from blood in enriched blood agar.
All other choices are non cultivable.

533. A. Rickettsiae
Transovarian transmission is infection which can be spread to offspring.
It is seen in tick born rickettsial diseases.

534. A. Rickettsiae
Refer Q. no : 521
535. A. Gram positive
Chlamydia can’t be stained by Gram stain.
It usually takes giemsa stain

536. B. Chlamydia
Refer Q. no 528

537. A. OXK
Scrub typhus is exclusively positive for OXK.

538. A. Weil-Felix reaction


VDRL – Syphilis
Rose – Waller – RA
Paul Bunnel - EBV

539. B. Chlamydia
Chlamydial Inclusion bodies are called as HP bodiesHalberstaedter–Prowazek bodies))

540. E. All of the above


False positivity in VDRL seen in other conditions like Autoimmune disorders, Malaria…

541. A. Early relapsing syphilis


Chancre redux is a condition defined as the recurrence of the primary sore at its
original site
Any lesion at the site of a healed primary has been labelled pseudochancre redux

542. A. VDRL
Prognostic test for syphilis arenon specific tests.
Example : VDRL and RPR

543. A. VDRL
Refer Q. no : 542

544. A. T. pertenue & B. T. carateum


Yaws – T. pertenue
Pinta – T. carateum
Bejel – T. endemicum

545. A. Leptospirosis
Leptospiras infect various animals, including rats and other rodents,
domesticlivestock, and household pets.

546. C. Microscopic agglutination test


MAT depends on the ability of the patient’s serumto agglutinate live leptospires
obtained by culture.
The MAT test uses a battery of live leptospires serovars commonlyprevalent in the
area endemic for the disease.
The MAT usesa battery of antigens taken from common leptospire serovars prepared
from the leptospirestrains cultured on the media

547. D. Lice acts as reservoir of infection


Rats and mammals are the reservoirs for leptospirosis.

548. B. Microscopic agglutination test


It’s a suspected case of leptospirosis.
The gold standard test to diagnose leptospirosis is Microscopic slide agglutination
test. (MSAT)

549. D. Weil Felix reaction


Weil Felix reaction is a heterophile antibody detection test for rickettsial infections.
All other choices are employed in the diagnosis of leptospirosis.

550. B. Weil’s disease


Aseptic meningitis is the most important clinical manifestation of leptospirosis.
Profound jaundice, renal dysfunction, pulmonary dysfunction, hepatic necrosis, and
hemorrhagic diastases areother severe manifestations

551. B. Borrelia
Lyme disease is caused by bacteria, Borrelia burgdorferi that are transmitted to
humans through a bite from an infected black-legged or deer tick.
Symptoms can occur anywhere from 3 to 30 days after the bite and can be wide-
ranging, depending on the stage of the infection.

552. B. Borrelia duttonii & D. Borrelia hermsii


Humans are the only hosts. B. hermsii and b. Borrelia duttonii .
They aretransmitted to humans by soft ticks (Ornithodoros).

553. A. IgM FTA - Abs


Congenital syphilis is diagnosed by the demonstration of T.pallidum by DFM of
umbilical cord, placenta, nasaldischarge, or skin lesion.
IgM FTA ABS or IgM ELISA – IgM cannot crosses placenta

554. C. VDRL is negative in secondary syphilis


All the serological tests including specific and non specific are 100% sensitive in
secondary syphilis.

555. A. Borrelia burgdorferi


Refer Q, no 551

556. B. Treponema pallidum


Non cultivable organisms are Chalmydia, Rickettsia, M. leprae and Pathogenic
T.pallidum.

557. A. Treponema pallidum


Yaws is non venereal syphilis caused by Treponema pallidum pertenue

558. A. Borrelia
They undergo antigenic variation.
It does so by changing its outer surface protein(OSP). These OSPs vary antigenically
within humans.

559. C. Relapsing fever


Relapsing fever caused by Borrelia recurrentis
Yaws – T. pertenue
Pinta – T. carateum
Bejel – T. endemicum

560. C. Schaudinn and Hoffman


Frankel – Pneumococci
Ogston – Staphylococus aureus
Nicholas – C. tetani

561. D. 1,2 and 3


VDRL and RPR are non specific tests
TPPA, TPHA, TPI & FTA -Abs are specific serological tests for syphilis by using
treponemal antigens.

562. E. TPPA
Currently the most specific test for syphilis is TPPA.
The Treponema pallidum particle agglutination assay is an indirect agglutination
assay used for detection and titration of antibodies against the causative agent of
syphilis, Treponema pallidum subspecies pallidum.

563. A. TPI
T. pallidum immobilization test detects the treponemal antibodies in patient’s serum,
whichimmobilize motile virulent T. pallidum.
The test is performed by incubating live T. pallidum strains with test serum in
thepresence of complement.
If the serum contains treponemal antibodies, the treponemes become immobilized,
which can be demonstrated under dark ground microscope

564. B. Syphilis
Rapid plasma reagin (RPR) test is a popular test usedfor diagnosis of syphilis by
demonstrating reaginic antibodies.
The test uses VDRL antigen containing finely divided carbon particles suspended in
choline chloride.
The latter destroys inhibitory factors in the serum, thus avoiding the need to heat the
serumbefore testing.
Use of this antigen produces a clearer and welldefined flocculation reaction easily
observed by the naked eye.

565. A. Trypanosoma & C. Borrelia


Antigenic mutation seen in Neisseria gonorrhea, Tryponasoma brucei and Borrelia
species

566. D. 10 days - 90 days


Average time between infection with syphilis and the start of the first symptom is 21
days, but can range from 10 to 90 days.
Syphilis blood test results will be negative during this time.

567. A. Plasmids
Antigenic variation is a unique property exhibited by Borreliain humans.
DNA rearrangement in linear plasmid present in Borrelia appears to be responsible:
568. A. Rat
The main reservoir of leptospirosis is rat.
The disease is otherwise called as rat fever

569. C. Rat
Refer Q. No 568

570. A. Fontana’s
Silver impregnation method for treponema
Fontana stain for fluid specimen
Leviditi stain for tissue specimen

571. B. FTA-ABS
It is specific serological test for syphilis.
Currently this method is not recommended by CDC

572. C. Vector borne disease


B. recurrentis caused relapsing fever
Vector for epidemic type is Louse and endemic type id tick.

573. All of the above


U. urealyticum is a sexually transmitted pathogen and causeschorioamnionitis,
prematurity, vaginitis, cervicitis, acute salpingitis, and pelvic inflammatory disease in
women.
Ureaplasma organisms are most frequently isolated from infants, particularly girls.
The bacteria also cause urethritis, proctitis, balanoposthitis, and Reiter’s syndrome in
men

574. A. They are inhibited by Penicillins


Mycoplasma are devoid of cell wall.
Penicillin acts on cell wall.
So penicillin is ineffective against Mycoplasma.

575. A. Mycoplasma
Cold agglutination test: In this test, human O group erythrocytes are used as antigen.
This is based on the principle that autoantibodies that agglutinate human O group
cells at low temperatures appear in most of the cases of atypicalpneumonia.

576. B. Mycoplasma
Mycoplasma are naturally devoid of cell wall.

577. A. Mycoplasma
Mycoplasma colonies are best stained by Diene method.

578. C. Require cholesterol for growth


Devoid of cell wall.
Grows in artificial culture medium – PPLO agar
Needs cholesterol for their growth.

579. B . Acinetobacter baumanni


Haemophilusaphrophilus and Haemophilus paraphrophilus,
Actinobacillusactinomycetemcomitans
Cardiobacterium hominis
Eikenella corrodens
Kingella kingae.

580. A. HME
Ehrlichia chaffeensis is a member of the Rickettsia family and causes human
monocytic ehrlichiosis (HME). This disease resembles Rocky Mountain spottedfever,
except that the typical rash usually does not occur.
High fever, severe headache, and myalgias are prominent symptoms. The organism is
endemic in dogs and is transmitted to humans by ticks, especially the dog tick,
Dermacentor, and theLone Star tick, Amblyomma.
Ticks of the genus Ixodes are also vectors. E. chaffeensis primarily infects
mononuclear leukocytes and forms characteristicmorulae in the cytoplasm.
Virology — Questions
581. The vector of Japanese B encephalitis virus is :
a. Anopheles mosquito
b. Culex mosquito
c. Aedes mosquito
d. All of these

582. Regarding Hepatitis E, it is true that:


a. Occurs with Hepatitis B
b. Is single stranded DNA virus
c. Occurs along with HIV
d. Increases mortality in pregnancy

583. Virus implicated in nasopharyngeal carcinoma is


a. Herpes
b. Adenovirus
c. Corona virus
d. EBV

584. Diagnostic of Rabies


a. Guarneri bodies
b. Negri bodies
c. Cowdry A body
d. Cowdry B body

585. Arbovirus – True is


a. Yellow fever is endemic in India
b. Dengue has only one serotype
c. KFD was first identified in West Bengal
d. Chikungunya is transmitted by Aedes

586. Which of the viral disease is tick borne


a. Kyasanur forest disease
b. Chikungunya
c. Yellow fever
d. Dengue

587. Live vaccine except


a. Hepatitis B
b. Sabin
c. 17D (yellow fever)
d. BCG

588. All the following hepatitis viruses are RNA viruses except
a. Hepatitis A virus
b. Hepatitis B virus
c. Hepatitis C virus
d. Hepatitis D virus

589. Commonest cause of acute Bronchiolitis in children is


a. Adeno virus
b. RSV
c. Klebseilla
d. Staphylococci

590. Genetic reassortment is found in


a. Hepadna virus
b. Parvo virus
c. Rota virus
d. Astro virus

591. Regarding varicella false is


a. Produces rash in flexor surface of fore arm
b. Single stage lesions are found at a time
c. Secondary infection rate is >90%
d. Vaccine protection rate is high

592. Chlorination does not affect


a. Salmonella
b. Shigella
c. HIV
d. Polio virus

593. The Tzank test is used for diagnosing (PGI)


a. Herpes simplex
b. Impetigo
c. carbuncle
d. Chickenpox

594. Demonstration of inclusion bodies is diagnostic in all of the following


except
a. Chicken pox
b. Rabies
c. Molluscum contagiosum
d. Trachoma

595. Subclinical infection is not seen with


a. Diphtheria
b. Polio
c. Measles
d. Staphylococcus aureus

581. 582. 583. 584. 585. 586. 587. 588. b 589. b 590. c
b d d b d a a

591. 592. 593. 594.


b d a,d a

596. Glandular fever like syndrome is produced by all except


a. Cytomegalovirus
b. Toxoplasma gondii
c. Infectious mononucleosis
d. Herpes simplex type II

597. Paul bunnel test is used for


a. Malta fever
b. Enteric fever
c. Infectious mononucleosis
d. Typhus fever

598. Pharyngitis is most often caused by


a. Viruses
b. Pneumococcus
c. Streptococcus
d. Corynebacterium

599. Tiger mosquitoes are


a. Ticks
b. Culex
c. Anopheles
d. Aedes

600. Pneumonia is caused by all except


a. Influenza
b. Varicella
c. CMV
d. Mumps

601. A 9- Month-old infant presents to the ‘diarrhea clinic’ unit with some
dehydration. The most likely organism causing diarrhea is:
a. Entamoeba histolytica
b. Rotavirus
c. Giardia lamblia
d. Shigella

602. Early diagnosis of acute hepatitis-B infection is made by:


a. Presence of HbeAg in serum
b. Presence of IgM Anti-Hbc in serum
c. Presence HbsAg in serum.
d. Presence of IgG anti-HBc in serum

603. An 8year old child recently had erythema infectiosum. Her 33 year old
mother subsequentlydeveloped arthralgia followed by painful arthritis with
swelling in the small joints of both hands.In addition to the apparent tropism
for joints, human parvovirus B 19 is highly tropic for whichcell type?
a. CD4 T lymphocytes
b. Renal tubules cells
c. Erythroid cells
d. Glial cells
e. Peyer’ s patches

604. A 20year old female presented to her physician with a low grade fever,
headache, and painful genital lesions. Culture detects herpes simplex virus.
Which of the following statements best describes infection with this
common human pathogen?
a. Infection with type I virus is most common
b. Initial infection usually occurs by intestinal absorption of virus
c. It can be reactivated by emotional disturbances or prolonged exposure to sunlight
d. It rarely recurs in host who has high antibody titer
e. The CNS and visceral organs are usually involved

605. Echoviruses are cytopathogenic human viruses that mainly infect which
of the following?
a. Bladder and urinary tract
b. Blood and lymphatic system
c. Central nervous system
d. Respiratory system

606. A 10 year old boy is taken to his pediatrician after experiencing fever,
malaise, and anorexia, followed by tender swelling of his parotid glands.
Mumps was diagnosed on clinical presentation.Which of the following
characterizes infection by this virus?
a. Is apt recur periodically in many affected persons
b. Is maintained by a large canine reservoir
c. Is preventable by immunization
d. Usually produces severe systemic manifestations
e. Belongs to Orthomyxovirus
595. c 596. 597. c 598. 599. 600. 601. 602. b 603. c 604. c
d a d d b

605. c 606. c

607. Two siblings, ages 2 and 4, experienced fever, rhinitis, and pharyngitis
that resulted inlaryngotrachiobronchitis. Both had harsh cough and
hoarseness. Which virus is could beresponsible for this condition?
a. Adenovirus
b. Group B coxsackievirus
c. Parainfluenza virus
d. Rhino virus
e. Rota virus

608. An immunocompromised person with a history of seizures had an MRI


that revealed atemporal lobe lesion. Brain biopsy results showed
multinucleated giant cells with intranuclearinclusions. The most probable
cause of the lesion is which of the following?
a. Coxsackievirus
b. Hepatitis C virus
c. Herpes simplex virus 1
d. Herpes simplex virus 2
e. Parvovirus

609. Parvovirus infection, the cause of a mild exanthem in children, can also
cause which of thefollowing?
a. Aplastic crisis
b. Acute respiratory disease
c. Gastroenteritis
d. Keratoconjunctivitis

610. Within the first year of life, an infant exhibited severe hearing loss,
ocular abnormalities, and apparent mental retardation. Cytomegalovirus
infection was detected by viral isolation and PCR.Which of the following
statements best characterizes CMV?
a. It can be transmitted across the placental barrier
b. While a common infection, CMV is almost always symptomatic
c. The CMV can be cultured from red blood cells of infected patients
d. Unlike other viral infections, CMV is not activated by immune-suppressive therapy
e. There is no specific therapy for CMV

611. All are associated with HHV-6 except


a. Fifth disease
b. Roseola Infantum
c. Double stranded DNA virus
d. Exanthem subitem

612. What is true about Human Papillomaviruses (HPV)?


a. Papillomavirus infection is commonly diagnosed by viral culture
b. HPV-6 and HPV-18 are mainly sexually transmitted
c. HPV-16 and HPV-11 are mostly associated with cervical cancers
d. Are associated with progressive multifocal leukoencephalopathy

613. All are true about Influenza A virus except


a. May cause antigenic shift and antigenic drift
b. May cause pandemics
c. Vaccination offers lifelong protection
d. Responds to neuraminidase inhibitors

614. All are true about Respiratory Syncytial Viruses except:


a. May cause bronchiolitis
b. Is an RNA virus
c. May be prevented by vaccination
d. Belongs to paramyxovirus

615. All of the following may be present in chronic HBV infection except
a. HBeAg
b. HBV-DNA
c. HBsAg
d. Anti-HBc IgM

616. All of the following is true except


a. HCV belongs to flavivirus
b. No vaccine available for Hepatitis E
c. HBV is a DNA virus
d. HDV is defective virus

617. Which of the following viruses isclosely associated with exanthema


subitum?
a. Parvovirus B19
b. Human herpesvirus 6
c. Rubella virus
d. Rubeola virus

607. c 608. c 609. 610. 611. 612. 613. c 614. c 615. d 616. b
a a a b

617.
b

618. An individual who has received all three doses of hepatitis B vaccine and
who has never had hepatitis B virus (HBV) infection would be expected to
have which of the following serologic marker(s)?
a. HBcAb
b. HBsAb
c. HBeAb
d. HBeAb and HBsAb

619. Acute infantile diarrhea is most commonly due to infection with which of
the following?
a. Enteroviruses
b. Rotaviruses
c. Noroviruses
d. Enteric adenoviruses

620. Which of the following viruses is spread by way of nerves?


a. Rabies
b. Varicella-zoster
c. Herpes simplex
d. All of the above
e. None of the above

621. A 25 years old 2nd gravida lady is in her 1st trimester pregnancy. Her son
aged 4 has developedchicken pox at home since 1 day. She does not give any
history of chicken pox and is varicella IgGnegative.What is advised for the
mother?
a. Varicella zoster immunoglobulin
b. Oral Acyclovir
c. Both VZ immunoglobulin and oral Acyclovir
d. Nothing, only reassurance

622. Post exposure prophylaxis for HIV should be started preferably within
a. 2 hours of exposure
b. 4 hours of exposure
c. 72 hours of exposure
d. 20 minutes of exposure

623. All of the following methods are used for the diagnosis of HIV infection in
a 2 months oldchild except
a. DNA-PCR
b. Viral culture
c. HIV ELISA
d. p24 antigen assay

624. Antenatal maternal HIV diagnosis is of importance to


a. Prevent vertical transmission
b. Terminate
c. Discharge
d. Isolate the patient

625. After vaccination for Hepatitis B the following viral marker should be
measured to ascertain whether the vaccinee is a responder or non
responder
a. HBsAg
b. Anti HBsAg
c. IgM Anti HBcAg
d. IgM Anti HBeAg
626. A mother is HBsAg positive at 32 weeks of pregnancy. What should be
given to the newborn toprevent neonatal infection
a. Hepatitis B vaccine + immunoglobulins
b. Immunoglobulins
c. Hepatitis B vaccine only
d. Immunoglobulin followed by vaccine 1 month later

627. Detection of Dengue within 5 days can be done by


a. NS 1 Antigen detection
b. MAC ELISA
c. ICT test to detect IgM
d. None of the above

628. The best technique to detect growth in cell line cultures is


a. CPE
b. Haemadsorption
c. Immunofluoresence
d. Interference

629. To culture influenza virus for vaccine development in chick embryo the
best place for inoculation is
a. CAM
b. Allantoic cavity
c. Amniotic cavity
d. Yolk sac

618. 619. 620. 621. 622. 623. c 624. 625. b 626. a 627. a
b b d a a a

628. c 629.
b

630. For Hepatitis A vaccination all are true except


a. In low-endemicity regions, vaccination is only for high-risk individuals
b. In intermediate-endemicity regions, vaccination is required
c. In high-endemicity regions, vaccination is not required
d. Anti-HAV antibody testing has no role in determining vaccination.

631. A young female presents to her physician with low grade fever, headache
and painful genital lesions. Culture detects HSV. Which of the following best
describes infection with this commonhuman pathogen?
a. Infection with type 1 virus is most common
b. Initial infection usually occurs by intestinal absorption of the virus
c. It can be reactivated by emotional disturbances or prolonged exposure to sunlight
d. It rarely recurs in a host who has a high antibody titre

632. A 9 years old male presents with fever and nonspecific symptoms
followed by distinctive rashon the cheeks (slapped cheek). Which of the
following viruses is the most likely cause of this disease and has been
associated with transient aplastic crisis in persons with sickle cell disease?
a. Herpes simplex
b. Parvovirus B19
c. Rubella
d. Rubeola

633. EB virus is associated with all except


a. Infectious mononucleosis/glandular fever
b. Progressive lymphoproliferative disease in transplant recipients, immunodeficient,
AIDS
c. B-cell lymphoma in transplant recipients & AIDS
d. Oropharyngeal Ca

634. All are true regarding CMV except


a. Replicates in epithelial cells of salivary glands, kidneys & respiratory epithelium
b. Infected persons carries virus for life and is secreted in all body secretions
c. MC agent to cause intrauterine infection of the fetus
d. IgG detection by ELISA is used for diagnosis

635. All are true regarding HHV6 and HHV7 except


a. Infects CD4 T- lymphocytes & Macrophages
b. Reservoir is the salivary glands
c. Exanthema subitum/Roseola infantum is associated
d. Kaposi’s sarcoma is associated

636. All are true regarding adeno virus except


a. Causes Pharyngoconjunctival fever with types 3,4,7,14
b. Causes infantile gastroenteritis with type 40 & 41
c. Causes acute hemorrhagic cystitis in infants & children with 11 & 21
d. Rounding & aggregation of cells (grape like clusters) with basophilic, I/N IB
Cowdry type A

637. All are true regarding molluscum contagiosum except


a. Painless, umbilicated, pearly white nodules on trunk & anogenital area
b. Inverted fir tree appearance
c. Molluscum bodies are diagnostic
d. Do not spread by direct contact

638. All are true about Papova virus except


a. Nonenveloped, icosahedral, dsDNA
b. Cervical Cancer is caused by Papilloma virus type 6, 11
c. JC polyoma virus causes progressive multifocal leuco-encephalopathy in patients
of Hodgkin’s,CLL, AIDS
d. BK polyoma virus persists in kidney for life
639. The marker to detect superinfection of Hepatitis D in Hepatitis B patient
is
a. HBsAg
b. Anti HBsAg
c. IgG anti HBc
d. IgM anti HBc
630. 631. c 632. 633. 634. 635. 636. 637. d 638. b 639. c
d b d d d d

640. Vaccine may be developed in all the following except


a. Chick embryo
b. Vero cell line
c. HeLa cell line
d. Diploid cell line

641. A 29-year-old female diagnosed with AIDS has been suffering from a
progressive blurring ofvision in her right eye. On funduscopic examination, a
small white opaque lesion is noted on the retina of her right eye. Which of
the following is the most appropriate therapy for this patient?
a. Acyclovir
b. Amantadine
c. Flucytosine
d. Ganciclovir

642. Following statement is true regarding antigenic drift in influenza virus


a. It occurs once in 10-20 years
b. It is responsible for pandemics
c. It is seen only in influenza type A virus
d. Results from genetic recombination of human with animal
e. It is due to immunity selection to the group of the viruses existing in the
community

643. Which of the following statement is true regarding arboviruses


a. Yellow fever is endemic in India
b. Dengue virus has only one serotype
c. Kyasanur forest disease (KFD) is transmitted by ticks
d. Mosquito of culex vishnui complex is the vector of dengue fever
e. Japanese encephalitis is transmitted by Aedes

644. Thirty year old man presented with nausea, fever and jaundice of 5 days
duration. Thebiochemical tests revealed a bilirubin of 6.7 mg/dl (conjugated
5.0 mg/dl) with SGOT/SGPT(AST/ALT) of 1230/900 IU/ml. The serological tests
showed the presence of HBsAg, IgM anti HBc and HBeAg. The most likely
diagnosis is
a. Chronic hepatitis B infection with high infectivity
b. Acute hepatitis B infection with high infectivity
c. Chronic hepatitis B infection with low infectivity
d. Acute hepatitis B infection with low infectivity
e. Hepatitis B vaccinated individual presenting with obstructive jaundice

645. Human to human transmission is not seen in


a. SARS
b. Japanese B encephalitis
c. H5N1
d. Poliomyelitis
e. HIV

646. An otherwise healthy 3-year-old child is brought to the pediatrician with


umbilicated, fleshcolored papules on his trunk. This condition is related to
infection with which of the followingviruses?
a. Cytomegalovirus
b. Herpesvirus 6
c. Parvovirus
d. Poxvirus

647. HIV affects all the following cells except


a. T helper cells
b. Macrophages
c. NK cells
d. B lymphocytes
e. Follicular dendritic cells

648. Emergence or resurgence is seen in which of the following organism(s)


(PGI)
a. Poliovirus
b. Measles virus
c. Nipah virus
d. West Nile virus
e. Hepatitis B virus

649. In March 2003, Avian influenza outbreak in Netherlands was caused by


a. H5N1
b. H7N7
c. H9N2
d. H3N1
e. H5N2

650. Natural reservoir of Hendra virus is


a. Horse
b. Man
c. Bat
d. Pig
e. Duck

640. c 641. 642. 643. c 644. 645. 646. 647. c 648. 649. b
d e b b d c, d

650. c

651. Which of the following is a primary cell culture:


a. Chick embryo cell culture
b. Hela
c. HEP-2
d. HL-8

652. Human fibroblast cell line is used for cultivation of:


a. Adenovirus
b. Polio
c. HIV
d. Measles

653. A neonate develops encephalitis without any skin lesions; most probable
causative organism is:
a. HSV-I
b. HSV-II
c. Meningococci
d. Streptococci

654. A 29-year-old person comes with focal seizures. MRI shows frontal and
temporal enhancement. What is the most probable diagnosis:
a. Meningococcal meningitis
b. Herpes simplex encephalitis
c. Japanese encephalitis
d. Enterovirus encephalitis

655. Infectivity of chickenpox lasts for:


a. Till the last scab falls off
b. 6 days after onset of rash
c. 3 days after onset of rash
d. Till the fever subsides

656. A 40-year-old man underwent kidney transplantation. Two months after


transplantation, he developed fever and feature suggestive of bilateral
diffuse interstitial pneumonia. Which of the following is most likely etiologic
agent?
a. Herpes simplex virus
b. Cytomegalovirus
c. Epstein-Barr virus
d. Varicella - zoster virus
657. A patient has undergone a renal transplantation 2 months back and now
presented with difficult breathing. X-ray showed bilateral diffuse interstitial
pneumonia. The probable etiologic agent would be:
a. CMV
b. Histoplasma
c. Candida
d. Pneumocystis carinii

658. Renal involvement is seen in which of the following infections? (PGI)


a. Cytomegalovirus
b. Polyoma virus
c. Human papilloma virus
d. HIV
e. HBV

659. In a patient with a vesicle on shin. Microscopy on Tzank smear showed


giant cells. Causative agents is:
a. Vaccinia virus
b. Varicella zoster
c. Tuberculous
d. Molluscum contagiosum

660. Hypoplasia of limb and scarring is caused by:


a. Varicella
b. Herpes simplex
c. Rubella
d. Toxoplasma

661. Virus causing hemorrhagic cystitis, diarrhea and conjunctivitis is :


a. RSV
b. Rhinovirus
c. Adenovirus
d. Toxoplasma

662. Vaccine preparation requires which virus as vector: (PGI)


a. Rhinovirus
b. Vaccinia
c. Adenovirus
d. Ebola
e. Hepatitis B

663. About parvovirus B19 all are true except


a. Spread by respiratory route
b. Has affinity for erythrocyte progenitor cells
c. Causes transient aplastic crisis
d. Transplacental transfer occurs in only 10% of cases
664. Most sensitive test for diagnosis of infectious mononucleosis:
a. Monospot test
b. Paul Bunnel test
c. Lymphocytosis in peripheral smear
d. Culture of the virus

651. a 652. a 653. b 654. b 655. b 656. b 657. a 658. 659. b


a,b,d

660. a 661. c 662. 663. d 664. a


b,c

665. Which disease has viral etiology?


a. Myeloma
b. Nasopharyngeal Ca
c. Sarcoidosis
d. Hemophilia

666. True about virus is:


a. HSV-I causes encephalitis
b. EBV affects T lymphocyte
c. CMV is always symptomatic
d. Herpes zoster is not reactivated

667. Which virus remains dormant but reactivates later ?


a. HIV
b. VZV
c. HBV
d. HPV

668. Which virus reactivates and involves the eyes:


a. Herpes- zoster
b. CMV
c. EB virus
d. Enterovirus – 70

669. All are true regarding cytomegalovirus except:


a. It is DNA virus.
b. Most commonly infected in the last trimester
c. Diagnosed by increased IgA in fetal blood
d. Most common cause of congenital viral infection

670. Erythema infectiosum is caused by:


a. Human parvovirus B-19
b. Papova virus
c. Human herpes virus type 8
d. Measles virus
671. Which one given below is a DNA virus?
a. Poliovirus
b. Adenovirus
c. Parvovirus
d. Hepatitis-A virus

672. Most common extra skin manifestation of varicella is involvement of:


a. CNS
b. Lungs
c. Kidneys
d. CVS

673. Oral hairy leukoplakia caused by:


a. Ebsteinbarr virus
b. CMV
c. HIV
d. HZV

674. Adenovirus:
a. Double stranded DNA
b. Enveloped
c. Complex symmetry
d. None

675. African Burkitt’s lymphoma is caused by:


a. Cytomegalovirus
b. EB virus
c. Herpes zoster
d. Infectious mononucleosis

676. All of the following are true about Herpes group of viruses except:
a. Ether-sensitive
b. May cause malignancy
c. HSV II involves below diaphragm
d. Burkitt’s lymphoma involves T-cells

677. Varicella are classified under:


a. Enterovirus
b. Retrovirus
c. Poxvirus
d. Herpes virus

678. Human papilloma virus contains:


a. ssRNA
b. dsDNA
c. ssRNA
d. ssDNA

679. EBV causes:


a. Nasopharyngeal Ca
b. CRF
c. P. versicolor
d. None

680. Congenital varicella infection causes all except:


a. Macrocephaly
b. Limb hypoplasia
c. Cortical atrophy
d. Cicatrix

681. Which pox won’t grow in egg, animal cells:


a. Cow pox
b. Vaccinia
c. Variola
d. Molluscum

665. 666. 667. 668. 669. c 670. 671. 672. a 673. a 674. a
b a b a a b

675. 676. 677. 678. 679. 680. 681.


b d d b a a d

682. HHV-8 causes:


a. Burkitt’s lymphoma
b. Nasopharyngeal carcinoma
c. Kaposi’s sarcoma
d. Hepatic carcinoma

683. Slapped cheek sign is seen in:


a. Parvovirus B19
b. JC virus
c. Rota virus
d. Mumps

684. Pharyngoconjunctival fever is caused by:


a. Adenovirus 3 and 7
b. Adenovirus 11, 21
c. Adenovirus 40, 41
d. Adenovirus 8,10

685. Brick-shaped virus:


a. Chicken pox
b. Small pox
c. CMV
d. EBV

686. All of the following statements are true regarding polio- virus except:
a. It is transmitted by Feco-oral route
b. Asymptomatic infections are common in children
c. There is a single serotype causing infection
d. Live attenuated vaccine produces herd immunity

687. All of the following clinical features are associated with enteroviruses
except:
a. Myocarditis
b. Pleurodynia
c. Herpangina
d. Hemorrhagic fever

688. All are false regarding polio virus except:


a. Most cases are symptomatic
b. Inactivated vaccine given I/M
c. Inactivated polio vaccines are given to child less than 3 years of age
d. Only one type exists

689. Enterovirus causes all except:


a. Hemorrhagic fever
b. Pleurodynia
c. Herpangina
d. Aseptic meningitis

690. H5N1 is:


a. Bird flu virus
b. Vaccine for HIV
c. Causative agent of Japanese encephalitis
d. An eradicated virus

691. Segmented RNA is found in:


a. Influenza virus
b. Rabies virus
c. Herpes virus
d. Molluscum contagiosum virus

692. Commonest complication of Mumps is:


a. Orchitis and Oophoritis
b. Encephalitis
c. Pneumonia
d. Myocarditis
693. Regarding mumps, which is true?
a. Causes SSPE
b. Mumps causes aseptic meningitis in children
c. Sublingual gland is involved commonly
d. All

694. Vaccination causing intussusception:


a. Rotavirus
b. Parvovirus
c. Inactivated polio d. BCG
d. Measles

695. Reassortment is typically seen in:


a. Herpes
b. Hepadna
c. Rotavirus
d. Astrovirus

696. Vaccines prepared by embryonated hen’s egg are:


a. Measles
b. Rabies
c. Rubella
d. Varicella

682. c 683. 684. 685. 686. c 687. 688. 689. a 690. a 691. a
a a b d b

692. 693. 694. 695. c 696.


a b a b

697. Class II exposure in animal bites includes the following:


a. Scratches without oozing of blood
b. Licks on a fresh wound
c. Scratch with oozing of blood on palm
d. Bites from wild animals

698. A 25-year-old girl has been admitted to the hospital with provisional
diagnosis of rabies. The most suitable clinical sample that can confirm the
antemortem diagnosis is:
a. Serum for antivirus IgG antibody
b. skin impression smear for immuno-fluorescence stain
c. CSF sample for viral culture
d. Giemsa stain on smear prepared from salivary secretions

699. A boy got unprovoked bite from a neighbor’s dog. The animal control
authority caught the dog and it was found to be healthy. What will be the
next step?
a. Test antibody level in the dog
b. Withhold immunization and observe the dog for 10 days for signs of rabies
c. Start post-exposure prophylaxis
d. Perform euthanasia for the dog

700. Diagnostic of Rabies:


a. Guarneri bodies
b. Negri bodies
c. Cowdry A body
d. Bollinger bodies

701. Soft tick transmits:


a. Relapsing fever
b. KFD
c. Tick typhus
d. Tularemia

702. Which is true about arboviral disease?


a. Yellow fever is endemic in India
b. Dengue viruses have only one serotype
c. KFD is transmitted by ticks
d. Japanese encephalitis is transmitted by Aedes

703. Mark true in following:


a. Hanta virus pulmonary syndrome is caused by inhalation of rodent urine and feces
b. Kyasanur forest disease is caused by bite of wild animal
c. Lyssa virus is transmitted by ticks
d. Chikungunya is caused by anopheles

704. Most specific for Dengue diagnosis


a. IgM Capture ELISA
b. Tissue culture
c. CFT
d. Electron microscopy

705. True about dengue fever. (PGI)


a. Caused by 5 serotypes
b. Effective vaccine is available
c. Presents with fever and joint pain
d. Virus belongs to Flavivirus genus
e. Contain segmented RNA

706. All of the following statements are true about congenital rubella except:
a. It is diagnosed when the infant has IgM antibodies at birth
b. It is diagnosed when IgG antibodies persist for more than 6 months
c. MC congenital defects are deafness, cardiac malformation and cataract
d. Infection after 16 weeks of gestation result in major congenital defects

707. Laboratory diagnosis of viral respiratory tract infections can be


established by all of the following tests except:
a. Detection of virus specific IgM antibodies in single serum specimen
b. Demonstration of viral antigens by indirect immunofluorescence assay in
nasopharyngeal washings
c. Isolation of viruses using centrifugation enhanced culture
d. Detection of viral hemagglutination inhibiting (HAI) antibodies in a single serum
specimen

697. 698. 699. 700. 701. 702. c 703. 704. a 705. 706. d
b b b b a a a,d

707.
d

708. A 11-month-old child presents with complaints of respiratory distress. On


examination there is bilateral crepitation and wheezing. Which of the
following is the most likely cause?
a. Pneumonia
b. Adenovirus
c. Respiratory syncytial virus
d. Rhinovirus

709. Choose the correct matches: (PGI)


a. Mumps-RA 27/3 strain
b. Rubella-Jeryl-Lynn strain
c. Measles-Edmonston-Zagreb strain
d. BCG-Danish 1331 strain

710. Organism (s) causing, bronchiolitis in infant: (PGI)


a. RSV
b. Rhinovirus
c. Parainfluenza
d. Influenza
e. H. influenzae

711. New infectious agents are: (PGI)


a. Nipah virus
b. Pneumocystis jerovecii
c. Corona virus
d. SARS
e. Prion

712. True about Crimean-Congo hemorrhagic fever EXCEPT:


a. Zoonosis
b. Develop petechial rashes
c. Transmitted by mites
d. Recently disease has been reported in Gujarat

713. Man is the only reservoir of:


a. Rabies
b. Influenza
c. Typhoid
d. Japanese B encephalitis

714. In India, human infections have been reported dengue virus type:
a. Types I and 1
b. Types 1 and 3
c. Types 2 and 4
d. Type 1 only
e. All 4 types

715. Pre-exposure cell culture vaccine used in Rabies


a. 3 doses
b. 4 doses
c. 5 doses
d. 6 doses

716. Diarrrhea in Rotavirus infection is due to:


a. Increased intestinal hypermotility
b. Decreased absorption by villi
c. Increased Secretion by villi
d. None of the above

717. Recommended vaccines for rabies:


a. DPT
b. MMR
c. BCG
d. HDCV

718. Break bone fever is caused by which virus:


a. Variola
b. Coxsackie
c. Dengue
d. Adenovirus

719. Commonest cause of Bronchiolitis is:


a. RSV
b. Adenovirus
c. Influenza
d. Herpesvirus
720. Which of the following is true regarding influenza:
a. It is caused by an enveloped DNA virus
b. Laboratory studies may show neutropenia early in the course of disease
c. Primary infectious pneumonia is less common than secondary bacteria pneumonia
d. Antiviral agents are given early prevents complications

721. Negri bodies are seen in infections due to:


a. Poliovirus
b. Rabies virus
c. Herpesvirus
d. Adenovirus

722. A vaccine for rabies was first developed by:


a. Louis Pasteur
b. Robert Koch
c. Edward Jenner
d. Landsteiner

708. c 709. 710. 711. 712. c 713. c 714. e 715. a 716. b


c,d a,c,d a,c,d

717. d 718. c 719. a 720. c 721. b 722. a

723. Which of the following viruses is composed of two distinct capsids


enclosing the double stranded RNA?
a. Adenovirus
b. Reovirus
c. Herpes virus
d. Myxovirus

724. Type of vaccine available commercially for rabies are all except:
a. Inactivated sheep brain vaccine
b. Genetically engineered glycoprotein vaccine
c. Duck embryo cultured vaccine
d. Human diploid cell vaccine

725. Rotaviruses are responsible for:


a. Acute nonbacterial gastroenteritis in adults
b. Infantile diarrhea
c. Teratogenic effects
d. Respiratory tract infection in immunocompromised individuals

726. Reverse transcriptase polymerase chain reaction can aid in diagnosis of


all of the following viral infection except:
a. Adenovirus
b. Astrovirus
c. Rotavirus
d. Poliovirus

727. All of the viruses cause pneumonia except:


a. Cytomegalovirus
b. Mumps
c. Measles
d. Retrovirus

728. Conjunctivitis is caused by all except:


a. CMV
b. Enterovirus 70
c. Coxsackie A 24
d. Adenovirus

729. All of them are cultivable virus except:


a. Rotavirus
b. Enterovirus
c. ECHO virus
d. Coxsackie virus

730. All of the following rabies vaccines are commercially available except:
a. Killed sheep brain vaccine
b. Human diploid cell vaccine
c. Vero continuous cell vaccine
d. Recombinant glycoprotein

731. Rabies virus is inactivated by:


a. Phenol
b. UV radiation
c. BPL (Beta propiolactone)
d. All

732. Antirabies vaccine is prepared by


a. Street virus
b. Fixed virus
c. Line virus
d. Wild virus

733. Swine flu is caused by:


a. H1N1
b. H5N1
b. H3N1
d. H2N3

734. Non-enveloped ssRNA virus is:


a. Picornavirus
b. Poxvirus
c. Retrovirus
d. Bunyavirus

735. Most virulent dengue fever strain is:


a. 1
b. 2
c. 3
d. 4

736. Antigenic shift:


a. Occurs every 2-3 years
b. Gradual change over time
c. Result from genetic recombination
d. Occurs in all influenza viruses

737. Herpangina is caused by:


a. Coxsackie A
b. Rhinoviruses
c. Myxovirus
d. Rabies virus

738. Hemorrhagic fever is caused by:


a. West-Nile fever
b. Sandfly fever
c. Ebola virus
d. All of the above
723. 724. 725. 726. 727. 728. 729. 730. d 731. d 732. b
b b b a b a a

733. 734. 735. 736. c 737. 738. c


a a b a

739. Suckling mice is used for isolation of:


a. Coxsackie Virus
b. Pox
c. Herpes
d. Adeno Virus

740. False about Polio:


a. Descending Paralysis
b. Bilateral symmetrical
c. Non-progressive
d. LMN type paralysis

741. Appearance of Cowdry type A inclusion bodies


a. Granular
b. Circumscribed
c. In polio
d. None

742. Classic tried of congenital rubella includes all except


a. Cataract
b. Deafness
c. Retinitis
d. CHD

743. Most common oncogenic RNA virus


a. Retrovirus
b. Picornavirus
c. Orthomyxovirus
d. Paramyxovirus

744. Which is enveloped virus?


a. Dengue Virus
b. Norwalk Virus
c. Hep A Virus
d. Adeno Virus

745. Which of the following is not prion associated disease:


a. Scrapie
b. Kuru
c. Creutzfeldt-Jakob disease
d. Alzheimer’s disease

746. True about Prion disease is all except:


a. Myoclonus is seen in 10% of the patients
b. Caused by infectious protein
c. Brain biopsy is diagnostic
d. Commonly manifests as dementia

747. Mad cow disease is due to:


a. Slow virus
b. Mycoplasma
c. Bacteria
d. Fungus

748. Fatal familial insomnia is associated with:


a. Prion disease
b. Degenerative disease
c. Neoplastic disease
d. Vascular disease
749. Creutzfeldt-Jakob disease is caused by: (PGI)
a. Prion
b. JC virus
c. Genetic factors
d. Nutritional deficiency

750. Which of the following infection agent lacks RNA:


a. Virus
b. Staphylococci
c. Prions
d. Cryptococcus

751. Which of the following hepatitis virus is cultivable?


a. Hepatitis A
b. Hepatitis B
c. Hepatitis D
d. Hepatitis E

752. A 30 years old patient presented with history of jaundice for 10 days. His
liver function tests showed bilirubin of 10 mg/dl, SGOT/SGPT - 1100/1450,
serum alkaline phosphatase-240 IU. He was positive for HbsAg. What should
be the confirmatory test to establish acute hepatitis B infection?
a. IgM Anti-HBc antibody
b. HbeAg
c. HBV DNA by PCR
d. Anti-HBc antibody

739. 740. 741. 742. c 743. 744. 745. 746. a 747. a 748. a
a b a a a d

749. 750. c 751. 752.


a,c a a

753. A 30 years old lady delivered a healthy baby at 37 weeks of gestation.


She was a known case of chronic hepatitis B infection. She was positive for
HBsAg but negative for HBeAg. Which of the following is the most
appropriate treatment for the baby?
a. Both active and passive immunization soon after birth
b. Passive immunization soon after birth and active immunization at 1 year of age
c. Only passive immunization soon after birth
d. Only active immunization soon after birth

754. A 30 years man presented with nausea, fever and jaundice of 5 days
duration. The biochemical tests revealed a bilirubin of 6.7 mg/dl (conjugated
5.0 mg/dl) with SGOT/SGPT (AST/ALT) of 1230/900 IU/ml. The serological tests
showed presence of HBs Ag ,IgM anti-HBc and HBeAg. The most likely
diagnosis:
a. Chronic hepatitis B infection with high infectivity
b. Acute hepatitis B infection with high infectivity
c. Chronic hepatitis B infection with low infectivity
d. Acute hepatitis B infection with low infectivity

755. A mother is HBsAg positive at 32 weeks of pregnancy. What should be


given to the newborn to prevent neonatal infection:
a. Hepatitis B vaccine + immunoglobulins
b. Immunoglobulins only
c. Hepatitis B vaccine only
d. Immunoglobulins followed by vaccine 1 month later

756. Hepatitis B vaccination is given to a patient. His serum will reveal:


a. HBsAg
b. Anti-HBs Ag
c. IgM Anti-HBc Ag and HBS Ag
d. IgM and IgG Anti-HBc Ag

757. Which of these is not a marker of active replicative phase of chronic


hepatitis B?
a. HBVDNA
b. HBV DNA polymerase
c. Anti-HBc
d. AST and ALT

758. About hepatitis B, false statement is:


a. Vertical transmission is more important than horizontal
b. Communicable period lasts for months
c. Virus can be found in blood 1 month before jaundice
d. Age of onset determines the prognosis

759. Chronic liver disease is caused by:


a. Hepatitis B
b. Hepatitis A
c. Hepatitis C
d. Hepatitis E

760. Hepatitis C virus is a:


a. Toga virus
b. Flavivirus
c. Filovirus
d. Retrovirus

761. A young pregnant woman presents with fulminant hepatic failure. The
most likely etiological agent is:
a. Hepatitis B virus
b. Hepatitis C virus
c. Hepatitis E virus
d. Hepatitis A virus

762. Which of the following acute viral hepatitis infections has the highest
risk of progression to chronicity?
a. Hepatitis C
b. Hepatitis B
c. Hepatitis A
d. Hepatitis E

763. Hepatitis A is transmitted by:


a. blood route
b. Inhalation
c. Feco-oral route
d. All
753. 754. 755. 756. 757. c 758. 759. c 760. b 761. c 762. a
a b a b a

763. c

764. Most common route of spread of hepatitis E is:


a. Sex
b. Feco-oral
c. Blood transfusion
d. IV injections

765. Serological markers of infectivity of hepatitis B:


a. HBsAg
b. HBeAg
c. IgM anti-HBc
d. HBcAg

766. Which is DNA virus?


a. Hepatitis A
b. Hepatitis B
c. Hepatitis C
d. Hepatitis D

767. Which of the following does not go into chronic hepatitis stage:
a. HBV
b. HCV
c. HDV
d. HEV

768. HBV present in India is:


a. Adw
b. Ayw
c. Adr
d. Ayr

769. Hepatitis D virus:


a. Transmitted by feco-oral route
b. Is a DNA virus
c. Has independent existence
d. Resembles some plant viruses

770. Which of the following is not matched correctly:


a. Hepatitis D Defective virus
b. Hepatitis C Parenteral transmission
c. Hepatitis B RNA virus
d. Hepatitis E Feco-oral transmission

771. Hepatitis B virus vaccine contain:


a. HBsAg
b. HBc
c. HBe
d. All of the above

772. Dependent hepatitis virus is:


a. Hep A
b. Hep D
c. Hep B
d. Hep C

773. Hep B is not secreted in:


a. Saliva
b. Semen
c. Stool
d. None

774. True about HCV include all except:


a. Highest rate of chronicity among all hepatitis viruses
b. Can be cultured
c. Diagnosed by detection of HCV RNA
d. Transmitted through transfusion of infected food

775. All of the following hepatitis virus can be transmitted through blood
except:
a. Hepatitis B
b. Hepatitis C
c. Hepatitis D
d. Hepatitis E
776. Regarding HBV and HDV,false is:
a. Both can infect simultaneously
b. HDV can cause more serious infection due to super infection
c. HDV cannot infect in absence of HBV
d. HDV is a DNA virus

777. HDV is
a. SS RNA virus
b. SS DNA virus
c. DS RNA virus
d. IDS DNA virus

778. First antibody to appear in hepatitis:


a. IgM anti HBe
b. IgM anti HBc
c. IgG anti HBe
d. IgM anti HBs

779. An HIV patient complains of visual disturbances. Fundal examination


shows bilateral retinal exudates and perivascular hemorrhages. Which of
the following viruses are most likely to be responsible for this retinitis:
a. Herpes simplex retinitis
b. Human herpes virus
c. Cytomegalovirus
d. EBV

780. HIV virus has:


a. Single stranded DNA
b. Single stranded RNA
c. Double stranded DNA
d. Double stranded RNA

764. 765. 766. 767. 768. 769. 770. c 771. a 772. b 773. c
b b b d b d

774. 775. 776. 777. 778. 779. c 780.


b d d a b b

781. Regarding HIV infection, not true is:


a. p24 is used for early diagnosis
b. Lysis of infected CD4 cells is seen
c. Dendritic cells do not support replication
d. Macrophage is a reservoir for the virus

782. A patient with HIV has diarrhea with AFB +ve organism in stool. The most
likely organism is:
a. Mycobacterium avium intracellulare
b. Mycobacterium tuberculosis
c. Mycobacterium leprae
d. Mycoplasmas

783. A person with AIDS related complex is most likely suffering from:
a. Opportunistic infection
b. Cancer related to AIDS
c. Generalized lymphadenopathy
d. Herpes zoster

784. A known HIV positive patient is admitted in an isolation ward after an


abdominal surgery following an accident. The resident doctor who changed
his dressing the next day found it to be soaked in blood. Which of the
following would be the right method of choice of discarding the dressing?
a. Pour 1% hypochlorite on the dressing material and send it for incineration in an
appropriate bag
b. Pour 5% hypochlorite on the dressing material and send it for incineration in an
appropriate bag
c. Put the dressing material directly in an appropriate bag and send for incineration
d. Pour 2% Lysol on the dressing material and send it for incineration in an
appropriate bag

785. Tissue of origin of Kaposi’s sarcoma is:


a. Lymphoid
b. Vascular
c. Neural
d. Muscular

786. All of the following methods are used for the diagnosis of HIV infection in
a 2-month-old child except:
a. DNAPCR
b. Viral culture
c. HIV ELISA
d. p24 antigen assay

787. HIV infects most commonly:


a. CD4 + helper cells
b. CD8 + cells
c. Macrophage
d. Neutrophil

788. HIV infection is associated with all except:


a. A glandular fever like illness
b. Generalized lymphadenopathy
c. Gonococcal septicemia
d. Sinus disease
e. Pre-senile dementia
789. Persistent diarrheas in AIDS is caused by A/E:
a. Microsporidia
b. Cryptosporidium parvam
c. Cryptococcus
d. Isospora belli
e. Giardia lamblia

790. HIV gene is/are:


a. Gp73
b. p24
c. Gpl20
d. Gp5
e. None

791. Receptor for HIV?


a. CD4
b. CD3
c. CD5
d. CD56

792. HIV virus was discovered in


a. 1976
b. 1983
c. 1996
d. 1988

793. In India, maximum cases of tuberculosis in AIDS patients are due to:
a. M. tuberculosis
b. M. avium intracellular
c. M. scrofulaceum
d. M. akari
781. c 782. 783. 784. c 785. 786. c 787. 788. d 789. c 790. e
a a b a

791. 792. 793.


a b a

794. Gag gene encodes for:


a. Reverse transcriptase
b. Core antigen
c. Envelope
d. Gene activation

795. The chances of acquiring HIV infection following needle prick is:
a. 0.3%
b. 3-0%
c. 0-30%
d. 0.003%

796. The common causative agent in diarrhea in AIDS patient is:


a. Toxoplasma
b. Cryptosporidium
c. Salmonellae
d. Mycobacteria

797. HIV is a which type of virus:


a. Picorna virus
b. Pox V
c. Retrovirus
d. Herpes

798. Following cells are infected by HIV:


a. CD8 + T lymphocytes
b. CD4 + T lymphocytes
c. B cells
d. Macrophages

799. All of them are true about AIDS except:


a. Seen in heterosexual only
b. Caused by retrovirus
c. Candidiasis is also common feature
d. Retrovirus is thermolabile

800. Reverse transcriptase is:


a. DNA polymerase
b. DNA dependent RNA polymerase
c. RNA dependent DNA polymerase
d. None

801. Screening test for AIDS:


a. ELISA
b. PCR
c. Western blot
d. CD-4 count

802. Which of the following is not a HIV gene?


a. Gag
b. Pol
c. Env
d. Rb
803. P24 antigen disappears from the blood after how many weeks infection
of HIV:
a. 2-4 weeks
b. 4-6 weeks
c. 6-8 weeks
d. 8-10 weeks

804. All of the following viral genes associated with HIV infection code for
structural proteins except
a. Gag gene
b. Env gene
c. Pol gene
d. Tat gene

805. A HIV mother delivers a baby. All are true except:


a. Risk of HIV in the baby is up to 90%
b. HIV infection cannot be diagnosed in the baby with available methods
c. AIDS can be transmitted from mother to child during delivery
d. Breastfeeding can transmit AIDS

806. Which bacterial infection is not common in HIV patients?


a. Cryptosporidiosis
b. Atypical mycobacterial infection
c. Aspergillosis
d. Candidiasis

807. In HIV patient hemiparesis, headache, vomiting occurs due to infection of


which organism:
a. Gonococcus
b. Streptococcus
c. Toxoplasmosis
d. None

808. Co-receptor for R5 variant of HIV virus:


a. Integrin
b. CCR 5
c. CXCR4
d. P53

809. Nef gene in HIV is for use:


a. Enhancing the expression of genes
b. Enhancing viral replication
c. Decreasing viral replication
d. Maturation
794. 795. 796. 797. c 798. 799. 800. c 801. a 802. d 803. c
b a b b a
804. 805. 806. c 807. c 808. 809.
d a b b
Virology — Explanations
581. B. Culex mosquito
Ades is vector for Dengue, Yellow fever and Chikungunya
Anopheles is vector for Plasmodium infections

582. D. Increases mortality in pregnancy


HEV is a major cause of hepatitis transmitted by the fecal–oral route. It is thought
tobe more common than HAV in many developing countries.
It is a common cause of waterborne epidemics of hepatitis in Asia, Africa, India, and
Mexico but isuncommon in the United States.
HEV is a nonenveloped, single-stranded RNA virusclassified as a member of the
hepevirus family.
Clinically the disease resembleshepatitis A, with the exception of a high mortality
rate in pregnant women.
Chronicliver disease does not occur, and there is no prolonged carrier state.

583. D. EBV
Epstein–Barr virus infection is associated with nasopharyngeal carcinoma endemic in
Asia and is common in malesof Chinese origin.
EBV DNA can be demonstrated in thenasopharyngeal carcinoma cells.
Epstein–Barr virus is also associated with Hodgkin’s lymphoma and most non-
Hodgkin’s lymphoma. EBV genome is demonstrated in Reed Sternberg cells in
Hodgkin’slymphoma

584. B. Negri bodies


The rabies viral genome encodes for production of five proteins associated with either
the ribonucleoprotein (RNP) complex or theviral envelope.
The L (transcriptase), N (nucleoprotein), and NS (transcriptase-associated) proteins
comprise the RNP complex,together with the viral RNA.
These proteins are aggregated together in the cytoplasm of virus-infected neurons
and constitute Negri bodies, the characteristic histopathological findingof rabies virus
infection.

585. D. Chikungunya is transmitted by Aedes


Refer Q. no 581
KFD first identified in Karnataka
Dengue has 4 major serotypes
Yellow fever not reported in India

586. A. Kyasanur forest disease


KFD virus was identified frommonkeysin the Kyasanur Forest, Shimoga district,
Karnataka.
Hard ticks are the vectors(Haemaphysalisspinigera)
Monkeys are the amplifier hosts

587. A. Hepatitis B
Hepatitis B vaccine is a recombinant vaccine.
HBsAg is used as vaccine candidate.
Cloning the S gene into the yeast genome.
588. B. Hepatitis Bvirus
All the hepatitis viruses are RNA viruses except Hepatitis B virus.
HBV possesses partially double stranded DNA

589. B. RSV
RSV is the MCC of bronchiolitis in infants.
Paramyxovirus -ssRNA
DOC is Ribavirin
No vaccine available

590. C. Rota virus


Genetic reassortment commonly seen with viruses possessing segmented RNA.
Example : Influenza virus, Rota virus, Bunya and Arena viruses.

591. B. Single stage lesions are found at a time


After an incubation period of 14 to 21 days, brief prodromal symptoms of fever and
malaise occur. A papulovesicular rash then appears in crops on the trunk andspreads
to the head and extremities.
At a point of time, the rash appears as papules,vesicles, pustules, and, finally, crusts.
Pleomorphic rash is characteristic.
Especially when vesicles are present

592. D. Polio virus


The chlorination of drinking water is useful for killing most of the common viruses
with exception of hepatitisA and polioviruses.
These two viruses are relatively resistant tochlorination.

593. A. Herpes simplex & D. Chickenpox


Tzanck smear preparation by Giemsa stain
Detects multinucleated giantcells with multifaceted nuclei.
It cannot differentiates between HSV-1, HSV-2, and VZV

594. A. Chicken pox


Inclusion bodies are not diagnostic for chicken pox. They cannot differentiate HSV1,2
and VZV.
Rabies – Negri bodies
MC – Henderson Peterson bodies
Trachoma – Halberstaedter-Prowazek bodies.

595. C. Measles
Clinical Cases are the only source of infection. There is no carrier stage and
subclinical infection.
Humans are the only reservoir of infection

596. D. Herpes simplex type II


HSV 2 causes Genital herpes, which is characterized by painful vesicular lesions of
the male andfemale genitals and anal area
Neonatal herpes originates chiefly from contact with vesicular lesions within the birth
canal. In some cases, although there are no visible lesions, HSV-2 is shedinto the birth
canal (asymptomatic shedding) and can infect the child during birth.
597. C. Infectious mononucleosis
Paul–Bunnell test was first described b Paul and Bunnell for demonstration of
heterophile antibodiesin patients with infectious mononucleosis in 1932.
This heterophile antibody, which is an IgM antibody, is not directed against EBV or
EBV-infected cells, but is produced due topolyclonal activation of B cells by EBV.
Such heterophilic antibodies are also found in serum sickness during drug reaction
and naturally occurring antibodies to the Forssmanantigen.

598. A. Viruses
Most sore throats are caused by colds, the flu, coxsackie virus or mono
(mononucleosis).
Bacteria that can cause pharyngitis in some cases: Strep throat is caused by group A
streptococcus.
Less commonly, bacterial diseases such as gonorrhea and chlamydia can cause sore
throat.

599. D. Aedes
The Asian tiger mosquito or forest day mosquito Aedes albopictus is characterized by
its black and white striped legs, and small black and white body.

600. D. Mumps
Clinical features of mumps includes Inapparent infection, Bilateral parotitis,
Epididymo-Orchitis, Aseptic meningitis and Pancreatitis.

601. B. Rotavirus
Rotavirus is a common cause of viral gastroenteritis, especially in young children.
It is characterized by nausea, vomiting, and watery, nonbloody diarrhea.
Gastroenteritis is most serious in young children, in whom dehydrationand electrolyte
imbalance are a major concern. Adults usually have minor symptoms.

602. B. Presence of IgM Anti-Hbc in serum


IgM Anti-Hbc is the earliest diagnostic marker to be elevated following infection.
HBs Ag is present in carriers also.

603. C. Erythroid cells


B19 virus infects primarily two types of cells: red blood cell precursors (erythroblasts)
in the bone marrow, which accounts for the aplastic anemia, and endothelial cells in
the blood vessels, which accounts, in part, for the rashassociated with erythema
infectiosum.

604. C. It can be reactivated by emotional disturbances or prolonged exposure


to sunlight
The factors involved in the reactivation of HSV are environmental triggers such as
emotional stress, fever, UV exposure, hormonal changes, dental surgery, and cranial
trauma.

605. C. Central nervous system


An ECHO (enteric cytopathic human orphan) virus is a type of RNA virus that belongs
to the species Enterovirus B, genus Enterovirus of the Picornaviridae family.
Echoviruses are found in the gastrointestinal tract (hence it being part of the
enterovirus genus) and exposure to the virus causes other opportunistic infections
and diseases, notably aseptic meningitis.

606. C. Is preventable by immunization


Prevented by vaccine prepared from Jeryl Lynn strain.
Belongs to Paramyxovirus
Local manifestation.
Human is the reservoir.

607. C. Parainfluenza virus


Parainfluenza virus cause croup (acute laryngotracheobronchitis), laryngitis,
bronchiolitis,and pneumonia in children and a disease resembling the common cold
in adults

608. C. Herpes simplex virus 1


Encephalitis caused by HSV-1 is characterized by a necrotic lesion in one temporal
lobe. Fever, headache, vomiting, seizures, and altered mental status aretypical
clinical features.
The onset may be acute or protracted over several days.
The disease occurs as a result of either a primary infection or a recurrence.

609. A. Aplastic crisis


Children with chronic anemia, such as sickle cell anemia, thalassemia, and
spherocytosis, can have transient but severe aplastic anemia (aplastic crisis)
wheninfected with B19 virus.
People with normal red blood cells do not have clinically apparent anemia, although
their red blood cell precursors are infected.

610. A. It can be transmitted across the placental barrier


CMV causes cytomegalic inclusion disease (especially congenital abnormalities)
inneonates.
It is the most common cause of congenital abnormalities in developed countries.
Mostly asymptomatic
Treatment : Gancyclovir

611. A. Fifth disease


Fifth disease is associated with Parvo virus B 19

612. B. HPV-6 and HPV-18 are mainly sexually transmitted


Papillomas of various organs are the predominant finding. These papillomas are
caused by specific HPV types.
For example, skin and plantar warts are caused primarily by HPV-1 through HPV-4,
whereas genital warts (condylomatafficacy) are caused primarily by HPV-6 and HPV-
11. HPV-6 and HPV-11 also cause respiratory tract papillomas, especially
laryngealpapillomas, in young children

613. C. Vaccination offers lifelong protection


Killed vaccine prepared in allantoic cavity of egg which contains HA antigen (15
µg/dose).
Two doses, given by IM route, fficacy 70–90%, lasts for 6–12 months or Single dose is
administered by IM or SC route. Protectivity is about 50–80%, Immunity lasts for 6–12
months.
614. C. May be prevented by vaccination
Refer Q. no 589

615. D. Anti-HBc IgM


Anti-HBc IgM and HBsAg present in Acute hepatitis.
Anti-HBc IgM and Anti HBe present in Window period.

616. B. No vaccine available for Hepatitis E


The one currently licensed hepatitis E vaccine (HEV 239 vaccine) is considered a
promising vaccine which has shown a high degree of efficacy against hepatitis E
disease in 16–65 year-old healthy subjects in China.

617. B. Human herpesvirus 6


HHV-6 targets T-cells by binding to CD46 receptor.
Sixth disease also called as exanthema subitum or roseola infantum.

618. B. HBsAb
Presence of HBsAb alone indicates vaccination

619. B. Rotaviruses
Refer Q. no : 601

620. D. All of the above


Neurotrophic viruses are HSV-1, HSV-2, VZV, JCV, Rabies, HIV, Enterovirus, JE
virus..etc

621. A. Varicella zoster immunoglobulin


Indication for Varicella zoster immunoglobulin(VZIG)
Should be given within 72 - 96 hrs of exposure.
Neonates born to varicella infected mothers - if the onset is between < 5 days before
delivery till 48 hrs afterdelivery.
VZIG not indicated if the mother has zoster.

622. A. 2 hours of exposure


Ideal time start PEP for HIV is 2 hours to 72 hours after exposure.

623. C. HIV ELISA


Antibody detection is useful after 18 months of age.
Presence of maternal antibody will interfere the results

624. A. Prevent vertical transmission.


Mother-to-child transmission of HIV is the spread of HIV from a woman with HIV to her
child during pregnancy, childbirth (also called labor and delivery), or breastfeeding
(through breast milk).
Pregnant women with HIV receive HIV medicines during pregnancy and childbirth to
prevent mother-to-child transmission of HIV. In some situations, a woman with HIV
may have a scheduled cesarean delivery (sometimes called a C-section) to prevent
mother-to-child transmission of HIV during delivery.
Women who take HIV medicines during pregnancy and childbirth and whose babies
receive HIV medicines for 4 to 6 weeks after birth have a low risk of transmitting HIV.
625. B. Anti HBsAg
Refer Q. No 618

626. A. Hepatitis B vaccine + immunoglobulins


HBIG + HBV vaccine is recommended for neonates born to HBV infected mother,
where a single injection of 0.5 ml of HBIG is given to the neonate immediately after
the birth, followed by full course of vaccine given at a different site.
First dose should be given within 12 hours of birth

627. A. NS 1 Antigen detection


The diagnosis can be made in the laboratory either by isolation of the virus in cell
culture (1-3 days) or by serologic tests (NS1 antigen -1-5 days)or demonstration of
the presence IgM antibody (5-10 days)
A PCR assaythat detects virus in the blood is also available

628. C. Immunofluoresence
Other methods are Cytopathic effect (CPE), Viral Interference, Viral antigens
demonstration by Direct IF assay, Immunoperoxidase,Electron microscopy & PCR

629. B. Allantoic cavity


Allantoic cavity is Used for vaccine preparation for Influenza, Yellow fever and Rabies

630. D. Anti-HAV antibody testing has no role in determining vaccination.


Two types of HAV vaccines available.
Killed vaccine - Formaldehyde inactivated vaccine (IM)
Live attenuated vaccine
Both vaccinesare producing long lasting immunity.

631. C. It can be reactivated by emotional disturbances or prolonged exposure


to sunlight
Refer Q. No 604

632. B. Parvovirus B19


Parvovirus B19 causes erythema infectiosum (slapped cheek syndrome, fifth disease),
aplastic anemia (especially in patients with sickle cell anemia), and fetal infections,
including hydrops fetalis.

633. D. Oropharyngeal Ca
Diseases associated with EBV are infectious mononucleosis, Burkitt’s lymphoma, other
B-cell lymphomas, nasopharyngeal carcinoma, oral hairyleukoplakia and Progressive
lymphoproliferative disease.

634. D. IgG detection by ELISA is used for diagnosis


CMV diagnosis includes Fluorescent antibody and histologic staining of inclusion
bodies in giant cells in urine and in tissue.
The inclusion bodies are intranuclear and have an oval owl’s eye shape.
A fourfold or greater rise in antibody titer is also diagnostic.
PCR-based assays for CMV DNA or RNA in tissue or body fluids, such as spinal fluid
and amniotic fluid, are also very useful.
PP65 antigen detection is also useful and specific.
635. D. Kaposi’s sarcoma is associated
Kaposi sarcoma – an AIDS defining illness is associated with HHV8

636. D. Rounding & aggregation of cells (grape like clusters) with basophilic,
I/N IB Cowdry type A
Adenoviruses produces grape like clusters as a Cytopathic effect.
It is intra nuclear Cowdry type B and not A.

637. D. Do not spread by direct contact


The lesion of molluscum contagiosum is a small (2–5 mm), flesh-colored papule on
the skin or mucous membrane that is painless, nonpruritic, and not inflamed.
The lesions have a characteristic cup-shaped (umbilicated) crater with a white core.
MCV is transmitted by close personal contact, including sexually. The disease is quite
common in children, in whom lesions often occur around the eyes and on the trunk.

638. B. Cervical Cancer is caused by Papilloma virus type 6, 11


Human cervical cancer – HPV type 16,18,31,33Papillom
as of various organs are the predominant finding. These papillomas are caused by
specific HPV types.
Skin and plantar warts are caused primarily by HPV-1 through HPV-4, whereas genital
warts (condylomataacuminata) are caused primarily by HPV-6 and HPV-11. HPV-6 and
HPV-11 also cause respiratory tract papillomas, especially laryngeal papillomas, in
young children.
Carcinoma of the uterine cervix, the penis, and the anus, as well as
premalignantlesions called intraepithelial neoplasia, are associated with infection by
HPV-16 and HPV-

639. C. IgG anti HBc


The diagnosis of HDV infection in the laboratory is made by detecting either delta
antigen or IgM antibody to delta antigen in the patient’s serum.
Super infection is followed by HBV infection. It is diagnosed by detection of IgG anti
HBcAg.

640. C. HeLa cell line


HeLa cell line is continues cell line obtained from Cervical Cancer cells.
Continues cell lines are not suitable for vaccine production

641. D. Ganciclovir
It’s a case of CMV retinitis.
Ganciclovir is moderately effective in the treatment of CMV retinitis and pneumonia
in patients with AIDS.
Valganciclovir, which can be taken orally, is also effective against CMV retinitis.
CMV strains resistant to ganciclovir and valganciclovir have emerged, mostly due to
mutations in the d gene that encodes the phosphokinase. Drug susceptibility testing
can be done.

642. E. It is due to immunity selection to the group of the viruses existing in


the community
Results in periodic epidemic and sporadic cases.
Occurs in all types
Mild due to point mutation.
Periodical - every 2–3 years

643. C. Kyasanur forest disease (KFD) is transmitted by ticks


Yellow fever not reported from India
Dengue has 4 common serotypes
Dengue and JE transmitted by Ades mosquito

644. B. Acute hepatitis B infection with high infectivity

HBsAg HBsAg Anti-HBS Anti-HBe Anti-HBc

IgM IgG

+ + − − − −
Late incubation period or early hepatitis

+ + − − + −
Acute hepatitis

+ +* − − − +
Late/chronic HBC infection

+ − − − − +
Simple carrier

+ + − − − +
Super carrier

− − + + − +
Past infection

− − + − − −
Immunity following vaccination

*When +, it indicates high infectivity while − indicates low infectivity.

645. B. Japanese B encephalitis.


Two types of JE transmission cycle are
Pigs to Culex to Pigs
Ardeid birds to Culex to Ardeid birds

646. D. Poxvirus
Refer Q no 637

647. C. NK cells
HIV affects T helper cells (Mainly), Macrophages, B lymphocytes and Follicular
dendritic cells

648. C. Nipah virus&d. West Nile virus


During the past 20 years there has been a dramatic resurgence or emergence of
epidemic arboviral diseases affecting both humans and domestic animals.
These epidemics have been caused primarily by viruses thought to be under control
such as dengue, Japanese encephalitis, yellow fever, and Venezuelan equine
encephalitis, or viruses that have expanded their geographic distribution such as
West Nile and Rift Valley fever.

649. B. H7N7
H7N7 – Netherlands
H9N2 -Hong Kong
H7N9 - China

650. C. Bat
Hendra virus is a new paramyxovirus first isolated from the cases of severe respiratory
disease in Hendra, Australia, in 1994; hence the nameHendra virus.
Infection is transmitted from infected horses tohumans.
Fruit bats are the natural reservoir.
No specific antiviral agent or vaccine is available.

651. A. Chick embryo cell culture


Human carcinoma of cervix cell line (HeLa)
Human epithelioma of larynx cell line (Hep-2)
Human carcinoma of nasopharynx cell line (KB)
Human synovial carcinoma cell line (McCoy)

652. A. Adenovirus
Human fetal lung fibroblast cell line used for the cultivation and preparation of
adenovirus vaccine.

653. B. HSV-II
Neonatal herpes originates chiefly from contact with vesicular lesions withinthe birth
canal.
In some cases, although there are no visible lesions, HSV-2 is shedinto the birth canal
(asymptomatic shedding) and can infect the child during birth.
Neonatal herpes varies from severe disease (e.g., disseminated lesions
orencephalitis) to milder local lesions (skin, eye, mouth) to asymptomatic infection.
Encephalitis is a common component of this category of infection, occurring in about
60 to 75% of infants with disseminated disease.
While the presence of a vesicular rash can greatly facilitate the diagnosis of HSV
infection, over 20% of neonates with disseminated HSV disease do not develop
cutaneous vesicles during the course of their illness.

654. B. Herpes simplex encephalitis


Encephalitis caused by HSV-1 is characterized by a necrotic lesion in onetemporal
lobe.
Fever, headache, vomiting, seizures, and altered mental status aretypical clinical
features. The onset may be acute or protracted over several days.
The disease occurs as a result of either a primary infection or a recurrence.
Magnetic resonance imaging often reveals the lesion. Examination of the spinal fluid
typically shows a moderate increase of lymphocytes, a moderate elevation in
theamount of protein, and a normal amount of glucose.
HSV-1 encephalitis has a highmortality rate and causes severe neurologic sequelae in
those who survive.

655. B. 6 days after onset of rash


Infectivity period of chicken pox is 2 days before the onset of rash to 5 -6 days after
thereafter, until thevesicles are crusted.

656. B. Cytomegalovirus
CMV is the MC transplant transmitted viral infection in transplantrecipients especially
solid organ transplant (Kidney) recipients.
It occurs usually less than 6 months following transplantation.
Presentation is Bilateral interstitial pneumonia.

657. A. CMV
Refer Q. No 656

658. A. Cytomegalovirus, B.Polyoma virus & D. HIV


Epstein-Barr virus, cytomegalovirus, adenovirus and polyomavirus (type BK) are
prominent members of this group causing specific diseases.
Other viruses lack direct histological evidence of viral replication in the kidney, such
as hepatitis C or human immunodeficiency virus (HIV).
They are associated with renal disorders in immunocompetent and/or
immunocompromised hosts, and glomerular or tubulo-interstitial injury can be
caused via indirect pathways, such as the deposition of immune complexes or
cryoglobulins.

659. B. Varicella zoster


Tzanck smear preparation by Giemsa stain
Detects multinucleated giantcells with multifaceted nuclei.
It cannot differentiates between HSV-1, HSV-2, and VZV
Poor Sensitivity

660. A. Varicella
Maximum risk seen in mother acquires a primary infection(Chicken pox) during
pregnancy.
Infection acquired in late first or early second trimester leads to severe congenital
malformation in fetus including cicatricial skin lesions, limb hypoplasia and
microcephaly.

661. C. Adenovirus
In the upper respiratory tract, adenoviruses cause such infections as pharyngitis,
pharyngoconjunctival fever, and acute respiratory disease, characterized by
fever,sore throat, coryza (runny nose), and conjunctivitis.
In the lower respiratory tract, they cause bronchitis and atypical pneumonia.
Hematuria and dysuria are prominentin hemorrhagic cystitis.
Gastroenteritis with nonbloody diarrhea occurs mainly inchildren younger than 2
years of age.
Most adenovirus infections resolvespontaneously. Approximately half of all adenovirus
infections are asymptomatic

662. B. Vaccinia & C. Adenovirus


Retrovirus
Lentivirus
Vaccinia virus
Adenovirus
Adeno-associated virus
Cytomegalovirus

663. D. Transplacental transfer occurs in only 10% of cases


B19 virus is transmitted primarily by the respiratory route;
transplacentaltransmission also occurs (30%).
Blood donated for transfusions also can transmit the virus.
Humans are the naturalreservoir; animals are not a source of human infection.

664. A. Monospot test


Comparing with Paul bunnel test, Most sensitive test is more sensitive and specific.
Differential absorption and Monospot tests are used for the confirmation.

665. B. Nasopharyngeal Ca
EBV infection is associated with several cancers, namely Burkitt’s lymphoma,some
forms of Hodgkin’s lymphoma, and nasopharyngeal carcinoma.
The word associated refers to the observation that EBV infection is the initiating
event thatcauses the cells to divide, but that event itself does not cause a
malignancy.

666. A. HSV-I causes encephalitis


EBV affects B cells
CMV asymptomatic and symptomatic
Reactivation of varicella is herpes zoster.

667. B. VZV
Varicella Zoster virus exhibits latency.
Zoster is due to reactivation of latent VZV
Presence of rashesin single dermatome
Ramsay Hunt syndrome - facial nerve paralysis with vesicle on tympanic
Membrane, tongue and external auditory meatus.

668. A. Herpes- zoster


Herpes Zoster Ophthalmicus (HZO) is an ocular disease which usually manifests as a
unilateral painful skin rash in a dermatomal distribution of the trigeminal nerve
shared by the eye and ocular adnexa.
HZO occurs typically in older adults but can present at any age and occurs after
reactivation of latent varicella-zoster virus (VZV) present within the sensory spinal or
cerebral ganglia.

669. C. Diagnosed by increased IgA in fetal blood


IgM antibodies are indicative of active infection.
PCR from amniotic fluid is diagnostic.

670. A. Human parvovirus B-19


Refer Q. No 632

671. B. Adenovirus
DNA viruses are Pox, Herpes, HBV, Adeno, Parvo and Papova viruses.

672. A. CNS
Most common extracutaneous complication is CNS involvement mainly cerebellar
ataxia, encephalitis andaseptic meningitis followed by varicella pneumonia (fatal)

673. A. Ebsteinbarr virus


Refer Q. No 633

674. A. Double stranded DNA


Adenovirus is space ship appearance
Non enveloped, dsDNA and Icosahedral symmetry.

675. B. EB virus
Refer Q. no 632

676. D. Burkitt’s lymphoma involves T-cells


EBV targets CD21/CR2 receptors of B cells.

677. D. Herpes virus


VZV classified under alpha group of Herpes viridiae.
Recent nomenclature is HHV3

678. B. dsDNA
All the DNA viruses are ds DNA viruses except Parvo virus - ssDNA

679. A. Nasopharyngeal Ca
Refer Q. no 632

680. A. Macrocephaly
VZV - Congenital malformation in fetus including cicatricial skin lesions, limb
hypoplasia and microcephaly.

681. D. Molluscum
Molluscum contagiosum is non cultivable virus.
It cannot be propagated in tissue culture, egg or in animals.

682. C. Kaposi’s sarcoma


Kaposi sarcoma – an AIDS defining illness is associated with HHV8

683. A. Parvovirus B19


Refer Q. no 632

684. A. Adenovirus 3 and 7


Hemorrhagic cystis – Adenovirus type 11 & 21
Infantile diarrhea- Adeno 40, 41
STD- Adenovirus type 37
Epidemic conjuntivitis – Adenovirus type 8, 19, 37 (shipyard eye)
Swimming pool conjunctivitis- Adenovirus type 3, 7, 14
Pharyngitis & Pneumonia- Adenovirus type 3 & 7

685. B. Small pox


Rabies –Bullet
Small Pox virus- Brick
Ebola virus- Filamentous
Tobaco mosaic virus– Rod
Space vehicle – Adenovirus
686. C. There is a single serotype causing infection
Polio virus - There are three serologic (antigenic) types based on different antigenic
determinants on the outer capsid proteins.
Because there is little cross-reaction, protection from disease requires the presence of
antibody against each of the three types.

687. D. Hemorrhagic fever


Enteroviruses transmitted by fecal-oral route.
They do not cause intestinalsymptoms, but are associated with systemic
manifestations like herpangina, pleurodynia, Myocarditis, Aseptic meningitis and
poliomyelitis.

688. B. Inactivated vaccine given I/M


Three serotypes
Most cases are asymptomatic / inapparent

689. A. Hemorrhagic fever


Refer Q. no 687

690. A. Bird flu virus


H1N1 - Swine flu
H5NI - Avian flu
H3N2 – Seasonal flu

691. A. Influenza virus


Segmented RNA is present in:
B-unyaviruses
O-rthomyxoviruses (Influenza virus)
R-eoviruses (Rotavirus)
A-denovirus

692. A. Orchitis and Oophoritis


Epididymo-orchitis is a manifestation of mumps, developing in 15–30% of cases in
postpubertal males, with bilateral involvement in 10–30% of those cases.
Orchitis, accompanied by fever, typically occurs during the first week of parotitis but
can develop up to 6 weeks after parotitis or in its absence.
The testis is painful and tender and can be enlarged to several times its normal size;
this condition usually resolves within 1 week.
Testicular atrophy develops in one-half of affected men.
Sterility after mumps is rare, although subfertility is estimated to occur in 13% of
cases of unilateral orchitis and in 30–87% of cases of bilateral orchitis.
Oophoritis occurs in ~5% of women with mumps and may be associated with lower
abdominal pain and vomiting but has only rarely been associated with sterility or
premature menopause.

693. B. Mumps causes aseptic meningitis in children


Documented CSF pleocytosis indicates that mumps virus invades the CNS in ~50% of
cases; however, symptomatic CNS disease, typically in the form of aseptic meningitis,
occurs in <10% of cases, with a male predominance.
CNS symptoms of aseptic meningitis (e.g., stiff neck, headache, and drowsiness)
appear ~5 days after parotitis and also occur often in the absence of parotid
involvement.

694. A. Rotavirus
A previously approved vaccine for Rotavirus (Rotashield) was withdrawn when a high
rate of intussusception occurred in vaccine recipients.

695. C. Rotavirus
Rotavirus has segmented RNA and therefore can undergo genetic reassortment.

696. B. Rabies
Egg-derived vaccines: Allantoic cavity of embryonated eggs is the best site for the
preparation of rabies vaccine.

697. B. Licks on a fresh wound


Risk categorization and recommended anti-rabies prophylaxis

Category of risk Type of exposure Recommended prophylaxis


(WHO**)

Touching, or feeding of animal No treatment needed if


Category I (No Licks on intact skin history is reliable
risk)

Category II Minor scratches or abrasions without bleeding or Wound management


(Minor risk) nibbling of uncovered skin Rabies vaccine
Observe the dog for 10
days

Category III Single or multiple transdermal bites with oozing Wound management
(Major risk) of blood Rabies immunoglobulin
Licks on broken skin (fresh wounds) or mucous Rabies vaccine
membrane Observe the dog for 10
Bite by wild animals/bat days*

*Vaccine may be discontinued if animal (dogs and cats) is healthy after 10 days of
bite. Other animals are humanly killed and tissue is examined for detection of rabies
antigen/Negri body in brain biopsies.
**In India post exposure prophylaxis is indicated following exposure to any animal bite
except rats.

698. B. skin impression smear for immuno-fluorescence stain


Direct immunofluorescence test (direct-IF) can be performed to detect rabies
nucleoprotein antigens in specimens by using specific monoclonal antibodies tagged
with fluorescent dye.
The best specimen is hair follicle of nape of neck (most sensitive).
Corneal impression smear can also be used. It is usually positive in late stage with a
sensitivity of 30%.

699. B. Withhold immunization and observe the dog for 10 days for signs of
rabies

700. B. Negri bodies


The most characteristic pathologic finding in rabies is the Negri body .
Negri bodies are eosinophilic cytoplasmic inclusions in brain neurons that are
composed of rabies virus proteins and viral RNA.
These inclusions occur in a minority of infected neurons, are commonly observed in
Purkinje cells of the cerebellum and in pyramidal neurons of the hippocampus, and
are less frequently seen in cortical and brainstem neurons.
Negri bodies are not observed in all cases of rabies.

701. A. Relapsing fever


Borrelia recurrentis, Borrelia hermsii, and several other borreliae cause relapsing
fever.
During infection, the antigens of these organisms undergo variation.
As antibodies develop against one antigen, variants emerge and produce relapses of
the illness.
This can be repeated 3 to 10 times.
B. recurrentis is transmitted from person to person by the human body louse.
Humans are the only hosts.
B. hermsii and many other Borrelia species are transmitted to humans by soft ticks
(Ornithodoros). Rodents and other small animals are the main reservoirs.
These species of Borrelia are passed transovarially in the ticks, a phenomenon that
plays an important role in maintaining the organism in nature.

702. C. KFD is transmitted by ticks


KFD is transmitted by hard tick Haemophysalis spinifera.

703. A. Hanta virus pulmonary syndrome is caused by inhalation of rodent


urine and feces
In 1993, an outbreak of a new disease, characterized by influenzalike symptoms
followed rapidly by acute respiratory failure, occurred in the western United States,
centered in New Mexico and Arizona.
This disease, now called hantavirus
pulmonary syndrome, is caused by a hantavirus (Sin Nombre virus) endemic in deer
mice (Peromyscus) and is acquired by inhalation of aerosols of the rodent’s urine and
feces.
It is not transmitted from person to person.
Very few people have antibody to the virus, indicating that asymptomatic infections
are not common.

704. A. IgM Capture ELISA


MAC-ELISA is the most recommended tool available currently for dengue infection. It
has a sensitivity and specificity of approximately 90% and 98% respectively.

705. A. Caused by 5 serotypes

d. Virus belongs to Flavivirus genus


Dengue virus is the most common arbovirus found in India. It has four serotypes (DEN-
1, to DEN-4). Recently, the fifth serotype (DEN-5) was discovered in 2013 from
Bangkok.

706. D. Infection after 16 weeks of gestation result in major congenital


defects
Infection during first trimester, specially first month(when organogenesis occurs) leads
to congenital malformations.

707. D. Detection of viral hemagglutination inhibiting (HAI) antibodies in a


single serum specimen
A four-fold increase in antibody titre in acute and convalescent sera is more
informative.

708. C. Respiratory syncytial virus


Respiratory syncytial virus (RSV) is the most important cause of pneumonia and
bronchiolitis in infants.
It is also an important cause of otitis media in children and of pneumonia in the
elderly and in patients with chronic cardiopulmonary diseases.

709. C. Measles - Edmonston - Zagreb strain & D. BCG-Danish 1331 strain


More than ten live attenuated mumps vaccine strains (Jeryl Lynn, Urabe, Hoshino,
Leningrad-3, L-Zagreb, Miyahara, Torii, NK M-46, S-12 and RIT 4385) have been used
throughout the world.
The Jeryl Lynn strain is used in many countries.
Most vaccines contain 25 mg of neomycin per dose.
Several manufacturers in Japan and Europe produce a mumps vaccine containing the
Urabe Am9 virus strain.
However, concerns about vaccine-associated meningitis prompted several countries
to stop using Urabe vaccine strain (WER 1992).
Other vaccines have more limited distribution. In most cases, the viruses are cultured
in chick embryo fibroblasts (such as for the Jeryl Lynn and Urabe strain-containing
vaccines); however, quail and human embryo fibroblasts are also used for some
vaccines.
Most live attenuated rubella vaccines used throughout the world contain the RA 27/3
virus strain (Plotkin, 1965).
Exceptions are vaccines produced in China (BRD2 virus strain) and Japan (Matsuba,
Takahashi, and TO- 336), produced on rabbit kidney cells, and the Matsuura strain,
produced on quail embryo fibroblasts.
The RA 27/3 strain is used most often because of consistent immunogenicity,
induction of resistance to reinfection, and low rate of side-effects (Plotkin et al.,
1973).
The live-attenuated virus produces viraemia and pharyngeal excretion, but both are
of low magnitude and are non-communicable (Plotkin & Orenstein, 1999).

710. A. RSV, C. Parainfluenza & D. Influenza

711. A. Nipah virus, C. Corona virus & D. SARS


Nipah virus is a paramyxovirus that causes encephalitis, primarily in the South Asian
countries of Bangladesh, Malaysia, and Singapore.
The natural reservoir appears to be fruit bats.
People who have contact with pigs are particularly at risk for encephalitis, and some
human-to-human transmission occurs.
In general, paramyxoviruses are transmitted by saliva or sputum and that is the likely
mode of transmission.
There is no treatment, or vaccine for Nipah virus infections.
Coronaviruses are an important cause of the common cold, probably second only to
rhinoviruses in frequency.
In 2002, a new disease, an atypical pneumonia called severe acute respiratory
syndrome (SARS), emerged.
In 2012, another severe pneumonia called Middle East respiratory syndrome
emerged.

712. C. Transmitted by mites


Crimean-congo HF :
This severe HF syndrome has a wide geographic distribution, potentially being found
wherever ticks of the genus Hyalomma occur.
The propensity of these ticks to feed on domestic livestock and certain wild mammals
means that veterinary serosurveys are the most effective mechanism for the
surveillance of virus circulation in a region.
Human infection is acquired via a tick bite or during the crushing of infected ticks.
Domestic animals do not become ill but do develop viremia; thus, there is danger of
infection at the time of slaughter and for a brief interval thereafter (through contact
with hides or carcasses).
Cases have followed sheep shearing.
An epidemic in South Africa was associated with slaughter of tick-infested ostriches.
Nosocomial epidemics are common and are usually related to extensive blood
exposure or needle sticks.

713. C. Typhoid
Salmonella typhi has no animal reservoir. Infection is acquired from cases or carriers.

714. E. All 4 types

715. A. 3 doses

716. B. Decreased absorption by villi


Rotaviruses infect and ultimately destroy mature enterocytes in the villous epithelium
of the proximal small intestine.
The loss of absorptive villous epithelium, coupled with the proliferation of secretory
crypt cells, results in secretory diarrhea.
Brush-border enzymes characteristic of differentiated cells are reduced, and this
change leads to the accumulation of unmetabolized disaccharides and consequent
osmotic diarrhea.
Studies in mice indicate that a nonstructural rotavirus protein, NSP4, functions as an
enterotoxin and contributes to secretory diarrhea by altering epithelial cell function
and permeability.
In addition, rotavirus may evoke fluid secretion through activation of the enteric
nervous system in the intestinal wall.
Recent data indicate that rotavirus antigenemia and viremia are common among
children with acute rotavirus infection, although the antigen and RNA levels in serum
are substantially lower than those in stool.

717. D. HDCV
In the West, the rabies vaccine contains inactivated virus grown in human diploid
cells. (Vaccine grown in monkey lung cells or chick embryo cells is also available.)
In other countries, the duck embryo vaccine or various nerve tissue vaccines are
available as well.
Duck embryo vaccine has low immunogenicity, and the nerve tissue vaccines can
cause an allergic encephalomyelitis as a result of a cross-reaction with human
myelin.
For these reasons, the human diploid cell vaccine (HDCV) is preferred.

718. C. Dengue
After an incubation period of 2–7 days, the typical patient experiences the sudden
onset of fever, headache, retroorbital pain, and back pain along with the severe
myalgia that gave rise to the colloquial designation “break-bone fever.”
There is often a macular rash on the first day as well as adenopathy, palatal vesicles,
and scleral injection.
The illness may last a week, with additional symptoms usually including anorexia,
nausea or vomiting, marked cutaneous hypersensitivity, and—near the time of
defervescence—a maculopapular rash beginning on the trunk and spreading to the
extremities and the face.
Epistaxis and scattered petechiae are often noted in uncomplicated dengue, and
preexisting gastrointestinal lesions may bleed during the acute illness.

719. A. RSV
Please refer to explanation to question 128.

720. C. Primary infectious pneumonia is less common than secondary bacteria


pneumonia
Primary influenza viral pneumonia is the least common but most severe of the
pneumonic complications.
Secondary bacterial pneumonia follows acute influenza.
Improvement of the patient’s condition over 2–3 days is followed by a reappearance
of fever along with clinical signs and symptoms of bacterial pneumonia, including
cough, production of purulent sputum, and physical and x-ray signs of consolidation.
The most common bacterial pathogens in this setting are Streptococcus pneumoniae,
Staphylococcus aureus, and Haemophilus influenzae—organisms that can colonize
the nasopharynx and that cause infection in the wake of changes in
bronchopulmonary defenses.

721. B. Rabies virus


Please refer to explanation to question 120.

722. A. Louis Pasteur


He had contributed for the vaccine development against several diseases, such as
anthrax, fowl cholera and rabies.

723. B. Reovirus
REO is an acronym for respiratory enteric orphan; when the virus was discovered, it
was isolated from the respiratory and enteric tracts and was not associated with any
disease.
Rotaviruses are the most important human pathogens in the reovirus family.
Rotavirus has a segmented, double-stranded RNA genome surrounded by a double-
layered icosahedral capsid without an envelope.
The rotavirus genome has 11 segments.

724. B. Genetically engineered glycoprotein vaccine


Semple vaccine: It is derived from infected sheep brain, inactivated with phenol. Beta
propiolactone (BPL) vaccine: It is a modified Semple vaccine which is inactivated with
beta propiolactone instead of phenol.
Infant mouse brain vaccines: It is derived from infected neural tissue of newborn
mice.
Human diploid cell (HDC)vaccine: This is derived from WI-38 (human embryonic lung
fibroblast cell line).
Recombinant viral vaccine: Vaccinia virus carrying the rabies surface glycoprotein
gene has been developed. It is given orally, has been successful for immunizing
animals, but still in experimental stage for human use.

725. B. Infantile diarrhea


Rotavirus is the most common cause of diarrhea in infants and children.

726. A. Adenovirus
Adenovirus is a DNA virus, hence cannot be diagnosed by Reverse Transcriptase PCR.

727. B. Mumps
Mumps virus infects the upper respiratory tract and then spreads through the blood
to infect the parotid glands, testes, ovaries, pancreas, and, in some cases, meninges.
Alternatively, the virus may ascend from the buccal mucosa up Stensen’s duct to the
parotid gland.
It only multiplies in the respiratory tract and does not cause any respiratory signs and
symptoms.

728. A. CMV
Enterovirus 70 and Coxsackie virus A24 cause acute hemorrhagic conjunctivitis.
Adenovirus causes epidemic keratoconjunctivitis.

729. A. Rotavirus
Isolation of rotavirus is difficult.
Rolling of tissue cultures may be attempted to enhance replication.

730. D. Recombinant glycoprotein


Please refer to explanation to question 724.

731. D. All

732. B. Fixed virus


Differences between street and fixed rabies viruses

Street viruses Fixed viruses

Freshly isolated Isolated after serial intracerebral passage in rabbits/cell lines

Produce Negri body Do not produce Negri body

Affect salivary glands Do not affect salivary glands

Pathogenic Not pathogenic (except-if no inactivated properly)

Incubation Period: 1-3 months (20-90 days) Incubation Period: 4-6 days
Produce disease Used for vaccine

733. A. H1N1
The pandemic A/ H1N1 virus of 2009–2010 was a quadruple reassortant among swine
influenza viruses that circulated in North America and Eurasia, an avian virus, and a
human influenza virus.
The influenza A/H1N1 virus responsible for the most severe pandemic of modern times
(1918–1919) appears to have represented an adaptation of an avian virus to efficient
infection of humans.

734. A. Picornavirus
Picornaviruses are small (20–30 nm) nonenveloped viruses composed of an
icosahedral nucleocapsid and a single-stranded RNA genome.
The genome RNA has positive polarity (i.e., on entering the cell, it functions as the
viral mRNA).
There is no polymerase within the virion.
Picornaviruses replicate in the cytoplasm of cells.
They are not inactivated by lipid solvents, such as ether, because they do not have an
envelope.

735. B. 2

736. C. Result from genetic recombination


Influenza viruses, especially influenza A virus, show changes in the antigenicity of
their hemagglutinin and neuraminidase proteins; this property contributes to their
capacity to cause devastating worldwide epidemics (pandemics). There are two types
of antigenic changes:
(1) antigenic shift, which is a major change based on the reassortment of segments of
the genome RNA; and
(2) antigenic drift, which is a minor change based on mutations in the genome RNA.
Note that in reassortment, entire segments of RNA are exchanged, each one of which
codes for a single protein (e.g., the hemagglutinin)

737. A. Coxsackie A
Herpangina is usually caused by coxsackievirus A and presents as acute-onset fever,
sore throat, odynophagia, and grayish-white papulovesicular lesions on an
erythematous base that ulcerate.
The lesions can persist for weeks; are present on the soft palate, anterior pillars of the
tonsils, and uvula; and are concentrated in the posterior portion of the mouth.
In contrast to herpes stomatitis, enteroviral herpangina is not associated with
gingivitis.
Acute lymphonodular pharyngitis associated with coxsackievirus A10 presents as
white or yellow nodules surrounded by erythema in the posterior oropharynx. The
lesions do not ulcerate.

738. C. Ebola virus


Ebola virus is named for the river in Zaire that was the site of an outbreak of
hemorrhagic fever in 1976.
The disease begins with fever, headache, vomiting, and diarrhea.
Later, bleeding into the gastrointestinal tract occurs, followed by shock and
disseminated intravascular coagulation.
The hemorrhages are caused by severe thrombocytopenia. The mortality rate
associated with this virus approaches 100%.
Most cases arise by secondary transmission from contact with the patient’s blood or
secretions (e.g., in hospital staff).
Reuse of needles and syringes is also implicated in the spread within hospitals.

739. A. Coxsackie Virus


The classification of Coxsackie viruses into group A or B is based on pathogenicity in
suckling mice.
Group A viruses cause widespread myositis and flaccid paralysis, which is rapidly
fatal, whereas group B viruses cause generalized, less severe lesions of the heart,
pancreas, and central nervous system and focal myositis.
At least 24 serotypes of Coxsackie virus A and 6 serotypes of Coxsackie virus B are
recognized.

740. B. Bilateral symmetrical


The least common presentation is that of paralytic disease.
After one or several days, signs of aseptic meningitis are followed by severe back,
neck, and muscle pain and by the rapid or gradual development of motor weakness.
In some cases the disease appears to be biphasic, with aseptic meningitis followed
first by apparent recovery but then (1–2 days later) by the return of fever and the
development of paralysis; this form is more common among children than among
adults.
Weakness is generally asymmetric, is proximal more than distal, and may involve the
legs (most commonly); the arms; or the abdominal, thoracic, or bulbar muscles.
Paralysis develops during the febrile phase of the illness and usually does not
progress after defervescence.

741. A. Granular

742. C. Retinitis
The significance of rubella virus is not as a cause of mild childhood disease but as a
teratogen.
When a nonimmune pregnant woman is infected during the first trimester, especially
the first month, significant congenital malformations can occur as a result of
maternal viremia and fetal infection.
The increased rate of abnormalities during the early weeks of pregnancy is attributed
to the very sensitive organ development that occurs at that time.
The malformations are widespread and involve primarily the heart (e.g., patent
ductus arteriosus), the eyes (e.g., cataracts), and the brain (e.g., deafness and mental
retardation).

743. A. Retrovirus
The acutely transforming oncogenic retroviruses possess viral oncogenes (V.onc).
The slow transforming oncogenic retroviruses possess additional regulatory gene
(e.g.,tax gene for HTLV-1 and tat gene for HIV).
Oncogenic retroviruses belong to exogenous type; (i.e. spread horizontally between
host cells).
Endogenous retroviruses spread vertically from parent host cells to offspring) and
they are mostly non-pathogens.

744. A. Dengue Virus


Remaining three options infect GIT and are non-enveloped.

745. D. Alzheimer’s disease


Important Features of Slow Viral Disease Caused by Prions

Disease Pathogenesis Important Features

Kuru Transmissible/infectious Caused by ingesting or handling brain tissue; occurred in


New Guinea tribes people

Creutzfeldt-Jakob disease 1.Transmissible / latrogenic transmission by corneal transplant, brain


infectious electrodes, and growth hormone

2. Hereditary/genetic Mutation in germ cells

3. Sporadic No relationship to any known cause; possible new


mutation in somatic cells; most common form

Variant Creutzfeldt-Jakob Transmissible/infectious Probably acquired by eating meat or nervous tissue


disease from animals with mad cow disease

Gerstmann-Straussler- Hereditary/genetic Mutation in germ cells


Scheinker syndrome

Fatal familial insomnia Hereditary/genetic Mutation in germ cells

Scrapie is a disease of sheep, characterized by tremors, ataxia, and itching, in which


the sheep scrape off their wool against fence posts.
It has an incubation period of many months.
Spongiform degeneration without inflammation is seen in the brain tissue of affected
animals.
It has been transmitted to mice and other animals via a brain extract that contained
no recognizable virus particles.
Studies of mice revealed that the infectivity is associated with a 27,000-molecular-
weight protein known as a prion.

746. A. Myoclonus is seen in 10% of the patients


Prions are protein-containing particles with no detectable nucleic acid that are highly
resistant to inactivation by heat, formaldehyde, and ultraviolet light at doses that will
inactivate viruses.
The human prion-mediated diseases (e.g., kuru and CJD) are called transmissible
spongiform encephalopathies (TSE).
The term spongiform refers to the spongy, Swiss cheese–like holes seen in the brain
parenchyma that are caused by the death of the neurons .
No virus particles are seen in the brain of people with these diseases.
The transmissibility of the agent of kuru and CJD (“prions”) was initially established
by inoculation of material from the brains of infected patients into the brains of
primates followed by serial transfer to the brains of other primates.

747. A. Slow virus


Bovine Spongiform Encephalopathy:
BSE is also known as mad cow disease.
The cattle become aggressive, ataxic, and eventually die.
Cattle acquire BSE by eating feed supplemented with organs (e.g., brains) obtained
from sheep infected with scrapie prions. (It is also possible that BSE arose in cattle by
a mutation in the gene encoding the prion protein.)
BSE is endemic in Great Britain. Supplementation of feed with sheep organs was
banned in Great Britain in 1988 and thousands of cattle were destroyed, two
measures that have led to a marked decline in the number of new cases of BSE. BSE
has been found in cattle in other European countries such as France, Germany, Italy,
and Spain, and there is significant concern in those countries that variant CJD may
emerge in humans.
Two cases of BSE in cattle in the United States have been reported.

748. A. Prion disease


Fatal familial insomnia is a very rare disease characterized by progressive insomnia,
dysautonomia (dysfunction of the autonomic nervous system) resulting in various
symptoms, dementia, and death.
A specific mutation in the prion protein is found in patients with this disease.

749. A. Prion & C. Genetic factors


CJD is a prion disease.
Although most cases of CJD are sporadic, about 10% are hereditary.
The hereditary (familial) form is inherited as an autosomal dominant trait.
In these patients, 12 different point mutations and several insertion mutations in the
prion protein gene have been found.

750. C. Prions
Prions are protein-containing particles with no detectable nucleic acid that are highly
resistant to inactivation by heat, formaldehyde, and ultraviolet light at doses that will
inactivate viruses.

751. A. Hepatitis A
It is the only hepatitis virus which is cultivable in-vitro.

752. A. IgM Anti-HBc antibody


Serologic Test Results in Four Stages of HBV Infection

Test Acute Disease Window Phase Complete Recovery Chronic Carrier State

HBsAg Positive Negative Negative Positive

HBsAb Negative Negative Positive Negative1

HBcAb Positive2 Positive Positive Positive

1 Chronic carriers have negative antibody tests, but HBsAb is being made by these
individuals. It is undetected in the tests because it is bound to the large amount of
HBsAg present in the plasma. They are not tolerant to HbsAg.
2 IgM is found in the acute stage; IgG is found in subsequent stages.
Note: People immunized with HBV vaccine have HBsAb but not HBcAb because the
immunogen in the vaccine is purified HBsAg.

753. A. Both active and passive immunization soon after birth


Both the vaccine and HBIG should also be given to a newborn whose mother is
HBsAg-positive.
This regimen is very effective in reducing the infection rate of newborns whose
mothers are chronic carriers.
The regimen of vaccine plus HBIG in those with needle-stick injuries and in neonates
is a good example of passive–active immunization, in which both immediate
protection and long-term protection are provided.

754. B. Acute hepatitis B infection with high infectivity


IgM anti HBc indicates acute infection.
HBeAg arises during the incubation period and is present during the prodrome and
early acute disease and in certain chronic carriers.
Its presence indicates a high likelihood of transmissibility, and, conversely, the
finding of HBeAb indicates a lower likelihood, but transmission can still occur.

755. A. Hepatitis B vaccine + immunoglobulins


Please refer to explanation to question 753.

756. B. Anti-HBs Ag
Please refer to explanation to question 752.

757. C. Anti-HBc
Please refer to explanation to question 752.

758. A. Vertical transmission is more important than horizontal


The three main modes of transmission are via blood, during sexual intercourse, and
perinatally from mother to newborn.
The observation that needle-stick injuries can transmit the virus indicates that only
very small amounts of blood are necessary.
HBV infection is especially prevalent in addicts who use intravenous drugs. Screening
of blood for the presence of HBsAg has greatly decreased the number of transfusion-
associated cases of hepatitis B.
However, because blood transfusion is a modern procedure, there must be another,
natural route of transmission.
HBV is found in semen and vaginal fluids, so it is likely that sexual transmission is
important.
Transmission from mother to child during birth is another important natural route.
Transplacental transmission, if it occurs, is rare.
There is no evidence that transmission of HBV occurs during breast feeding.

759. C. Hepatitis C
The rate of chronic carriage of HCV is much higher than the rate of chronic carriage of
HBV.

760. B. Flavivirus
HCV is a member of the flavivirus family.
It is an enveloped virion containing a genome of single-stranded, positive-polarity
RNA.
It has no virion polymerase.

761. C. Hepatitis E virus


Clinically the disease due to hepatitis E resembles hepatitis A, with the exception of a
high mortality rate in pregnant women.

762. A. Hepatitis C
The rate of chronic carriage of HCV is much higher than the rate of chronic carriage of
HBV.

763. C. Feco-oral route


HAV is transmitted by the fecal–oral route.
Humans are the reservoir for HAV.
Virus appears in the feces roughly 2 weeks before the appearance of symptoms, so
quarantine of patients is ineffective.
Children are the most frequently infected group, and outbreaks occur in special living
situations such as summer camps and boarding schools.
Common-source outbreaks arise from fecally contaminated water or food such as
oysters grown in polluted water and eaten raw.
Unlike HBV, HAV is rarely transmitted via the blood, because the level of viremia is
low and chronic infection does not occur.

764. B. Feco-oral
HEV is a major cause of hepatitis transmitted by the fecal–oral route.
It is thought to be more common than HAV in many developing countries.
It is a common cause of waterborne epidemics of hepatitis in Asia, Africa, India, and
Mexico but is uncommon in the West.

765. B. HBeAg
HBeAg arises during the incubation period and is present during the prodrome and
early acute disease and in certain chronic carriers.
Its presence indicates a high likelihood of transmissibility, and, conversely, the
finding of HBeAb indicates a lower likelihood, but transmission can still occur.

766. B. Hepatitis B
HBV is a member of the hepadnavirus family.
It is a 42-nm enveloped virion, with an icosahedral nucleocapsid core containing a
partially double-stranded circular DNA genome.

767. D. HEV
HAV and HEV do not cause chronicity.

768. B. Ayw

769. D. Resembles some plant viruses


It has been proposed that HDV may have originated from a class of plant pathogens
called viroids, which are much smaller than viruses.

770. C. Hepatitis B RNA virus


Please refer to explanation to question 766.

771. A. HBsAg
The vaccine (e.g., Recombivax) contains HBsAg produced in yeasts by recombinant
DNA techniques.
The vaccine is highly effective in preventing hepatitis B and has few side effects. The
seroconversion rate is approximately 95% in healthy adults.
It is indicated for people who are frequently exposed to blood or blood products, such
as certain health care personnel (e.g., medical students, surgeons, and dentists),
patients receiving multiple transfusions or dialysis, patients with frequent sexually
transmitted disease, and abusers of illicit intravenous drugs.

772. B. Hep D
HDV is unusual in that it is a defective virus (i.e., it cannot replicate by itself because
it does not have the genes for its envelope protein).
HDV can replicate only in cells also infected with HBV because HDV uses the surface
antigen of HBV (HBsAg) as its envelope protein.
HBV is therefore the helper virus for HDV.

773. C. Stool

774. B. Can be cultured


Amongst hepatitis viruses, only hepatitis A can be cultured.

775. D. Hepatitis E
Hepatitis E is transmitted via faeco-oral route.

776. D. HDV is a DNA virus


Please refer to explanation to question 766.

777. A. SS RNA virus


HDV is an enveloped virus with an RNA genome that is a single-stranded, negative-
polarity, covalently closed circle.
The RNA genome of HDV is very small and encodes only one protein, the internal core
protein called delta antigen.
HDV genome RNA has no sequence homology to HBV genome DNA.
HDV has no virion polymerase; the genome RNA is replicated and transcribed by the
host cell RNA polymerase.

778. B. IgM anti HBc


Anti-HBc IgM is the first antibody to be elevated following infection.
It appears within first 1- 2 weeks after the appearance of HBsAg and lasts for 3-6
months.
Its presence indicates acute hepatitis B infection.

779. C. Cytomegalovirus
CMV retinitis is an important cause of blindness in immunocompromised patients,
particularly patients with advanced AIDS .
Early lesions consist of small, opaque, white areas of granular retinal necrosis that
spread in a centrifugal manner and are later accompanied by hemorrhages, vessel
sheathing, and retinal edema.
CMV retinopathy must be distinguished from that due to other conditions, including
toxoplasmosis, candidiasis, and herpes simplex virus infection.

780. B. Single stranded RNA


The genome of HIV consists of two identical molecules of single-stranded, positive-
polarity RNA and is said to be diploid.
781. C. Dendritic cells do not support replication
Dendritic cells (DCs) may play an important role in the initiation of HIV infection by
virtue of the ability of HIV to bind to cell-surface C-type lectin receptors, particularly
DC-SIGN .
This allows efficient presentation of virus to CD4+ T cell targets that become
infected; complexes of infected CD4+ T cells and DCs provide an optimal
microenvironment for virus replication.
There has been considerable disagreement regarding the HIV infectibility and hence
the depletion as well as the dysfunction of DCs themselves.
The situation has recently been clarified by the recognition that DCs can be classified
into myeloid (mDC) and plasmacytoid (pDC) subsets, leading to an appreciation of
specific DC dysfunction in HIV disease.
pDCs are an important component of the innate immune system and secrete large
amounts of IFN-α in response to viral infections.
The numbers of circulating pDCs are decreased in HIV infection through mechanisms
that remain unclear.
It has recently been demonstrated that HIV gp120 interacts directly with pDCs and
interferes with TLR9 activation, resulting in a decreased ability of pDCs to secrete
antiviral and inflammatory factors that play a role in immune responses against
invading pathogens.

782. A. Mycobacterium avium intracellulare

783. A. Opportunistic infection


Symptomatic HIV infection (AIDS Related Complex, ARC):
After variable period of clinical latency, the CD4 T cell level starts falling. Eventually
patients develop constitutional symptoms such as
Unexplained diarrhea lasting for more than l month .
Weight loss more than 10% of body weight, fatigue malaise and night sweat .
Mild opportunistic infections such as oral thrush.

784. C. Put the dressing material directly in an appropriate bag and send for
incineration.
No pre treatment is needed for incineration, in case incineration is not done,
pretreatment with hypochlorite is required.

785. B. Vascular
The pathogenesis of KS is complex; fundamentally, it is an angioproliferative disease
that is not a true neoplastic sarcoma, at least not in its early stages.
It is a manifestation of excessive proliferation of spindle cells that are believed to be
of vascular origin and have features in common with endothelial and smooth-muscle
cells.

786. C. HIV ELISA


Antibodies to HIV typically appear 10 to 14 days after infection, and most patients
will have seroconverted by 3 to 4 weeks after infection.
The inability to detect antibodies prior to that time can result in “false-negative”
serologic tests (i.e., the person is infected, but antibodies are not detectable at the
time of the test).
This has important implications because HIV can be transmitted to others during this
period.
787. A. CD4 + helper cells
HIV infects helper T cells (CD4-positive cells) and kills them, resulting in suppression
of cell-mediated immunity.
This predisposes the host to various opportunistic infections and certain cancers such
as Kaposi’s sarcoma and lymphoma.
HIV infection also targets a subset of CD4-positive cells called Th17 cells. These cells
are an important mediator of mucosal immunity, especially in the gastrointestinal
tract.

788. D. Sinus disease

Clincal Categories of Hiv Infection

Category A: Consists of one or more of the conditions listed belwo in an adolescent or adult (>13 years) with
documented HIv infection. Conditions listed in categories B and C must not have occurred.

Asymptomatic HIV infection

Persistent generalized lymphadenopathy

Acute (primary) HIV infection with accompanying illness or history of acute HIV infection

Category B: Consists of symptomatic conditions in an HIV-infected adolescent or adult that are not incldued
among conditions listed in clinical category C and that meet at least one of the following criteria: (1) The
conditin sare attributed to HIV infection or are indicative of a defect in cell-mediated immunity; or (2) the
conditions are considered by physicians to have a clinical course or to require management that is complicated
by HIV infeciton. Examples include, but are not limited to the following:

Bacillary angiomatosis

Candidiasis, oropharyngeal (thrush)

Candidiasis, vulvovaginal; persistent, frequent, or poorly responsive to therapy

Cervical dysplasia (moderate or severe)/cervical carcinoma in situ

Constitutional symptoms, such as fever (38.50C) or diarrhea lasting > 1 month

Hairy leukoplakis, orla

Herpes zoster (shingles), involving at least two distince epidsoed or more than one dermatome

Idiopathic thrombocytopenic purpura

Listeriosis

Pelvic inflammatory disease, particularly if complicated by tuboovarian abscess

Peripheral neuropathy

Category C: Conditions listed in the AIDS surveillance case definition

Candidiasis of bronchi, trachea, or lungs

Candidiasis, esophageal

Cervical cancer, invasive3

Coccidioidomycosis, disseminated or extrapulmonary

Cryptococcosis, extrapulmonary

Crptosporidiosis, chronic intestinal (>1 months’ duration)

Cytomegalovirus disease (Other than liver, spleen, or nodes)

Cytomegalovirus retinitis (with loss of vision)

Encephalopathy, HIV-related
Herpes simplex: chronic ulcer(s) (>1 month’s duration); or bronchitis, pneumonia, or esophatitis

Histoplasmosis, disseminated or extrapulmonary

Isosporiasis, chronic intestinal (>1 month’s duration)

Kaposi’s sarcoma

Lymphoma, Burkitt’s (or equivalent term)

Lymphoma, primary, of brain

Mycobacterium avium complex or M. Kansasii, disseminated or extrapulmonary

Mycobacterium tuberculosis, any site (pulmonarya or extrapulmonary)

Mycobacterium, other species or unidentified species, dissminated or extrapulmonary

Pneumocystis jiroveci pneumonia

Pneumonia, recurrenta

Progressive multifocal leukoencephalopathy

Salmonella septicemia, recurrent

Toxoplasmosis of brain

Wasting syndrome due to HIV

789. C. Cryptococcus
Cryptococcus neoformans causes subacute to chronic meningitis in HIV-AIDS.

790. E. None

791. A. CD4
Please refer to explanation to question 787.

792. B. 1983

793. A. M. tuberculosis
Worldwide, approximately one-third of all AIDS related deaths are associated with TB,
and TB is the primary cause of death for 10–15% of patients with HIV infection.

794. B. Core antigen


Like other retroviruses, HIV-1 has genes that encode the structural proteins of the
virus: gag encodes the proteins that form the core of the virion (including p24
antigen); pol encodes the enzymes responsible for protease processing of viral
proteins, reverse transcription, and integration; and env encodes the envelope
glycoproteins.

795. A. 0.3%
Large, multi-institutional studies have indicated that the risk of HIV transmission
following skin puncture from a needle or a sharp object that was contaminated with
blood from a person with documented HIV infection is ~0.3% and after a mucous
membrane exposure it is 0.09% if the injured and/or exposed person is not treated
within 24 h with antiretroviral drugs.

796. B. Cryptosporidium
Cryptosporidial infection may present in a variety of ways, ranging from a self-limited
or intermittent diarrheal illness in patients in the early stages of HIV infection to a
severe, life-threatening diarrhea in severely immunodeficient individuals.
In patients with untreated HIV infection and CD4+ T cell counts of <300/μL, the
incidence of cryptosporidiosis is ~1% per year.
In 75% of cases the diarrhea is accompanied by crampy abdominal pain, and 25% of
patients have nausea and/or vomiting.
Cryptosporidia may also cause biliary tract disease in the HIV-infected patient,
leading to cholecystitis with or without accompanying cholangitis and pancreatitis
secondary to papillary stenosis.

797. C. Retrovirus
HIV is one of the two important human T-cell lymphotropic retroviruses (human Tcell
leukemia virus is the other).
HIV preferentially infects and kills helper (CD4) T lymphocytes, resulting in the loss of
cell-mediated immunity and a high probability that the host will develop
opportunistic infections.
Other cells (e.g., macrophages and monocytes) that have CD4 proteins on their
surfaces can be infected also.

798. B. CD4 + T lymphocytes


Please refer to explanation to question 207.

799. A. Seen in heterosexual only


HIV infection is predominantly a sexually transmitted disease (STD) worldwide.
By far the most common mode of infection, particularly in developing countries, is
heterosexual transmission, although in many Western countries a resurgence of
male-to-male sexual transmission has occurred.

800. C. RNA dependent DNA polymerase


Reverse transcriptase is the RNA-dependent DNA polymerase that is the source of the
family name retroviruses.
This enzyme transcribes the RNA genome into the proviral DNA.
Reverse transcriptase is a bifunctional enzyme; it also has ribonuclease H activity.
Ribonuclease H degrades RNA when it is in the form of an RNA–DNA hybrid molecule.
The degradation of the viral RNA genome is an essential step in the synthesis of the
double-stranded proviral DNA.

801. A. ELISA
The presumptive diagnosis of HIV infection is made by the detection of antibodies in
the patient’s serum to the p24 protein of HIV using the enzyme-linked
immunosorbent assay (ELISA) test.
Because there are some false-positive results with this test, the definitive diagnosis is
made by Western blot (also known as Immunoblot) analysis, in which the viral
proteins are displayed by acrylamide gel electrophoresis, transferred to nitrocellulose
paper (the blot), and reacted with the patient’s serum. If antibodies are present in the
patient’s serum, they will bind to the viral proteins (predominantly to the gp41 or p24
protein).
Enzymatically labeled antibody to human IgG is then added.
A color reaction reveals the presence of the HIV antibody in the infected patient’s
serum.
802. D. Rb
Rb is retinoblastoma gene.

803. C. 6-8 weeks


The p24 antigen becomes detectable after 1to 2weeks (mean 12 days) of infection
and lasts for 3-4 weeks thereafter.
Again, it is elevated during the late advanced stage of AIDS.

804. D. Tat gene


In addition to the three typical retroviral genes gag, pol, and env, which encode the
structural proteins, the genome RNA has six regulatory genes .
Two of these regulatory genes, tat and rev, are required for replication, and the other
four, nef, vif, vpr, and vpu, are not required for replication and are termed “accessory”
genes.

805. A. Risk of HIV in the baby is up to 90%


HIV infection can be transmitted from an infected mother to her fetus during
pregnancy, during delivery, or by breast-feeding.
This remains an important form of transmission of HIV infection in certain developing
countries, where the proportion of infected women to infected men is ~1:1.
Virologic analyses of aborted fetuses indicate that HIV can be transmitted to the fetus
during the first or second trimesters of pregnancy.
However, maternal transmission to the fetus occurs most commonly in the perinatal
period.
Two studies performed in Rwanda and the former Zaire indicated that the relative
proportions of mother-to-child transmissions were 23–30% before birth, 50–65%
during birth, and 12–20% via breast-feeding.

806. C. Aspergillosis
Most important risk factor for aspergillosis is neutropenia.
Neutrophil count remains normal until late stages of HIV infection, hence
aspergillosis is an uncommon infection in HIV-AIDS.

807. C. Toxoplasmosis
The most common clinical presentation of cerebral toxoplasmosis in patients with HIV
infection is fever, headache, and focal neurologic deficits.
Patients may present with seizure, hemiparesis, or aphasia as a manifestation of
these focal deficits or with a picture more influenced by the accompanying cerebral
edema and characterized by confusion, dementia, and lethargy, which can progress
to coma.
The diagnosis is usually suspected on the basis of MRI findings of multiple lesions in
multiple locations, although in some cases only a single lesion is seen. Pathologically,
these lesions generally exhibit inflammation and central necrosis and, as a result,
demonstrate ring enhancement on contrast MRI or, if MRI is unavailable or
contraindicated, on double-dose contrast CT.

808. B. CCR 5
The T cell–tropic strains of HIV bind to CXCR4, whereas the macrophage-tropic strains
bind to CCR5.
Mutations in the gene encoding CCR5 endow the individual with protection from
infection with HIV.
People who are homozygotes are completely resistant to infection, and heterozygotes
progress to disease more slowly.
Approximately 1% of people of Western European ancestry have homozygous
mutations in this gene, and about 10% to 15% are heterozygotes.
One of the best-characterized mutations is the delta-32 mutation, in which 32 base
pairs are deleted from the CCR5 gene.

809. B. Enhancing viral replication


Nef/ Negative effector (p27) :
Promotes downregulation of surface CD4 and MHC 1 expression.
Blocks apoptosis.
Enhance virion infectivity .
Alters state of cellular activation .
Progression to disease slowed significantly in absence of nef.
Mycology — Questions
810. Aspergilloma has:
a. Septate hyphae
b. Pseudohyphae
c. Metachromatic hyphae
d. No hyphae

811. Maltese cross seen on polarizing microscopy in


a. Cryptococcus neoformans
b. Penicillium marneffi
c. Blastomyces
d. Candida albicans

812. Organism which cannot be grown on artificial culture media


a. Rhinosporidium seeberi
b. Penicillium marneffi
c. Aspergillus flavus
d. Sporothrix schenckii

813. All can cause candidiasis EXCEPT


a. DM
b. HTN
c. Ca Breast
d. Pregnancy

814. The organism most commonly found in intertrigo


a. Candida
b. Trichophyton
c. Streptococcus
d. Staphylococcus

815. Following cause pyogenic meningitis except


a. N. meningitis
b. H. influenzae
c. S. pneumonia
d. C. neoformans

816. Which of the following is characterized by formation of septate brown


colored hyphae in vivo?
a. Candidiasis
b. Phaeohyphomycosis
c. Nocardiosis
d. Histoplasmosis.
817. Cryptococcus neoformans has been found most often in
a. water
b. soil containing pigeon feces
c. chicken feces
d. desert soil

818. Commonest fungal cause of corneal ulceration is


a. Candida
b. Aspergillus
c. Penicilium
d. Mucor

819. Valley fever or desert rheumatism is


a. histoplasmosis
b. blastomycosis
c. Coccidioidomycosis
d. aspergillosis

820. A patient, resident of Himachal Pradesh presented with a series of ulcers


in a row, on his right leg. The biopsy from the affected area was taken and
cultured on Sabauraud’s Dextrose agar. What would be the most likely
causative organism?
a. Sporothrix schenckii
b. Cladoporium spp.
c. Pseudoallescheria boydii
d. Nocadiabrasielensis

821. The following fungi are thermally dimorphic except:


a. Sporothrixschenkii
b. Cryptococcus neoformans
c. Blastomyces dermatitidis
d. Histoplasma capsulatum

822. Infection with Sporothrix schenkii is an occupational hazard for gardners.


The portal of entry for this organism is via which of the following routes?
a. Lymphatic system
b. Mouth
c. Mucous membranes
d. Respiratory tract
e. Skin
810. 811. 812. 813. 814. 815. 816. 817. b 818. b 819. c
a a a b a d b

820. 821. 822.


a b e
823. Inhalation of fungal spores can cause primary lung infections. Of the
following organisms, which one is most likely to be associated with this
mode of transmission?
a. C. albicans
b. C. tropicalis
c. S. schenkii
d. C. immitis

824. Tinea versicolor is caused by which of the following?


a. E. floccosum
b. E. werneckii
c. M. furfur
d. M. canis
e. T. beigelii

825. Which of the following are associated with chromoblastomycosis?


a. Tuberculate macroconidia
b. Spherules
c. Sclerotic bodies
d. Arthroconidia

826. A florist presents with a lymphocutaneous infection of the right hand and
forearm. What is the most probable fungal etiologic agent?
a. Cladosporium carrionii
b. Phialophora verrucosa
c. Sporothrix schenckii
d. Trichosporoncutaneum

827. Which of the following yeasts divides by binary fission


a. Penicillium marneffei
b. Candida albicans
c. Cryptococcus neoformans
d. Saccharomyces cerevisiae

828. Breakthrough zygomycetes infection can occur during pulmonary invasive


aspergillosis following treatment with
a. Voriconazole
b. Posaconazole
c. Amphotericin B
d. Caspofungin

829. Tissue should not be minced during processing for isolation of the
following
a. Mucor spp.
b. Aspergillus spp.
c. Penicillium spp.
d. Candida spp.

830. Id reaction is associated with


a. Mucor spp.
b. Aspergillus spp.
c. Dermatophytes
d. Yeast

831. A preterm, LBW infant delivered by LSCS after IVF was put on ventilator
in NICU for severe RDS. The respiratory distress subsided on day 7 and TPN
with lipid formulation was started from day 10. The neonate developed signs
of sepsis on day 15. The most likely causative agent is
a. Gram negative bacteria
b. Gram positive bacteria
c. Malassezia
d. Candida

832. All are true about fungus except


a. Cell wall of fungus contain chitin, mannan and glucans which are not seen in
bacteria
b. Echinocandins are cell wall inhibitors
c. Sporulation in fungus is for survival as well as reproduction unlike bacteria
d. Sporulation has no diagnostic value

833. All are thermally dimorphic fungi except


a. Candida albicans
b. Histoplasma capsulatum
c. Blastomyces dermatitidis
d. Coccidioidomyces immitis

834. All are asexual spores except


a. Chlamydoconidia
b. Blastospores
c. Arthrospores
d. Ascospores

823. 824. c 825. c 826. c 827. 828. 829. 830. c 831. c 832. d
d a a a

833. 834.
a d

835. A premature infant born at 24 weeks gestation after IVF had a birth
weight of 800 gms. O2 was instituted by head box due to recurrent apnea.
TPN was instituted with both amino acid and lipid formulation. After 10 days
the baby develops signs of sepsis but blood culture was negative. However
blood culture for fungus revealed a fungus with round oval cells and short
hyphae. The causative agent is
a. Malassezia furfur
b. Candida albicans
c. Histoplasma capsulatum
d. Trichosporon

836. Dermatophytic infections are characterized by


a. Aflatoxin induced hallucinations
b. Confined to keratinized tissues
c. Easily treatable with penicillin
d. Marked by alveolar irritation

837. All are true regarding Id reaction seen in dermatophytes except


a. Due to aggressive treatment with terbinafine
b. Common in children
c. Incidence rate 5%
d. Anthropophilic dermatophyte is more commonly associated than Zoophilic and
Geophilic dermatophytes

838. Hair perforation test is positive in


a. Trichophyton rubrum
b. Trichophyton mentagrophyte
c. Microsporum gypseum
d. Epidermophyton floccosum

839. All are true regarding Sporothrix schenkii except


a. Most common risk factor is thorn prick
b. It is a dimorphic fungus
c. Primary pulmonary sporotrichosis is the most common presentation
d. Spreads via lymphatic to produce a series of ulcers

840. A chronically ill young white male is unable to work due to his frequent
illnesses. He spends most of his time raising and training pigeons. He
develops a mild pulmonary infection and eventually presents to his primary
care physician with headache, mental status changes and fever. A clinical
diagnosis of meningitis is confirmed with LA test on CSF for capsular
polysaccharide Ag of the organism. Which would be the best screening
media for this organism for isolation
a. Niger seed agar
b. Sabouraud’s Dextrose agar
c. Mannitol salt agar
d. Sorbitol MacConkey agar

841. In Chromoblastomycosis, all are true except


a. Dematiceous fungi are the etiological agent
b. Hyaline fungi are the etiological agent
c. Sclerotic bodies are pathognomonic
d. Systemic invasion is very rare
842. All are true in eumycetoma except
a. Causative agents are hyaline and phaeoid fungi
b. Tumour mass is usually single and well defined
c. Opening of sinuses are flat and not flared up
d. Less extensive with osteolytic lesions

843. Histoplasma capsulatum, a dimorphic fungus is found in soil heavily


contaminated with bird droppings. Which of the following statements best
describes the presence of the organism in tissue biopsies
a. Arthrospores
b. Oval budding yeasts inside macrophages
c. Spherules filled with endospores
d. Yeasts with broad base bud

844. All are true about African Histoplasmosis except


a. Caused by Histoplasma capsulatum var duboisi
b. Yeast are larger and thick walled
c. Pronounced giant cell formation in infected tissue
d. Dissemination is rarely encountered
835. 836. 837. 838. 839. c 840. 841. 842. d 843. b 844. d
a b d b a b

845. A 10 years old female patient with thalassemia and diabetes is admitted
to ICU because of sudden swelling on the right side of the face and an
episode of bleeding from the right nostril. She had recently developed signs
of ketoacidosis and renal insufficiency. Her blood sugar level at the time of
admission is 700 mg/dL. The facial lesion becomes partially necrotic and
shows slight protrusion of the right eye and facial paralysis. The patient dies
on the second day. Histopathologic examination of the lesions reveals
occlusion of the small vessels and the presence of non-septate hyphae. This
is most probably caused by which of the following
a. Candidiasis
b. Erysipelis
c. Gas Gangrene
d. Mucormycosis

846. Aspergillus is best described by


a. Round black sporangia filled with endospores, sporangia unbranched, rising from
a runner called a stolon from aseptate hyphae
b. Thick walled spores with tubercle like projections all around, on septate hyphae
c. Widespread in environment, conidia may be inhaled microscopic appearance in
specimen reveals dichotomous branching and septate hyphae
d. Rosette like cluster of conidia, on conidiophores, arising out of very thin septate
hyphae

847. A ventilated patient in ICU develops invasive aspergillosis and is put on


voriconazole. What is the best way to diagnose invasive aspergillosis in this
case
a. Serum β galactomannan
b. Serum µESR
c. Serum Procalcitonin
d. Tracheal aspirate culture

848. Febrile neutropenia is seen in infection with


a. Candida
b. Aspergillus
c. Zygomycetes
d. Blastomyces

849. Which of the following is known to cause Ca-colon


a. Histoplasmin
b. Mycotoxin
c. Cryptococcus gatti
d. Fusarium

850. The nail clippings of a chronic infected nail subjected to 10% KOH showed
plenty of fungal hyphae and on culture yielded a fluffy white mycelial growth
in 10 days which on LPCB mount exhibits club shaped macroconidia in
clusters, few chlamydoconodia and no microconidia. The most probable
dermatophyte is
a. Trichophyton mentagrophyte
b. Trichophyton rubrum
c. Microsporum ferrugineum
d. Microsporum gypseum
e. Epidermophyton floccosum

851. An HIV infected person from Manipur, India with a CD4 count of 75
cells/ml presented with molluscum contagiosum like lesions on face and
upper trunk. Aspiration cytology revealed both intra and extracellular
septate yeasts and culture yielded a dimorphic fungus. The most likely
agent is
a. Sporothrix schenckii
b. Penicillium marneffei
c. Histoplasma capsulatum
d. Blastomyces dermatitidis
e. Coccidioides immitis

852. Correctly matched stain (PGI)


a. Mucicarmine – Cryptococcus
b. Giemsa – Candida
c. Methanamine silver – Histoplasma
d. Grams – Pneumocystis jiroveci
e. PAS – Sporothrix schenckii
853. Reynolds-Braude phenomenon is demonstrated by which of the following
yeasts (PGI)
a. Candida albicans
b. Candida tropicalis
c. Candida dubliniensis
d. Cryptococcus neoformans
e. Rhodotorula

845. d 846. c 847. a 848. a 849. b 850. e 851. b 852. 853.


a,c,e a,c

854. Aflatoxins are produced by the following fungus


a. Aspergillus flavus
b. Fusarium moniliforme
c. Penicillium puberulum
d. Penicillium verrucosum
e. Aspergillus parasiticus

855. Dermatophytosis is caused by:


a. Herpes simplex
b. Papilloma virus
c. Trichophyton
d. Candida

856. Tinea pedis is caused by which of the following:


a. E. floccosum
b. M. furfur
c. M. canis
d. E. werneckii

857. Regarding fungal cell wall all are true except:


a. Contains chitin
b. Prevent osmotic damage
c. Azoles act on them
d. Does not contain peptidoglycan

858. Trichophyton species which is zoophilic


a. T. tonsurans
b. T. violaceum
c. T. schoenleinii
d. T. mentagrophytes

859. The capsule of Cryptococcus neoformans in a CSF sample is best seen by:
a. Gram’s stain
b. Indian ink preparation
c. Giemsa stain
d. Methenamine-silver stain

860. Latex agglutination test of the antigen in CSF helps in the diagnosis of:
a. Cryptococcus
b. Candidiasis
c. Aspergillosis
d. Histoplasmosis

861. Cryptoccoccal meningitis is common in:


a. Renal transplant recipient
b. Agammaglobulinemia
c. Neutropenia
d. IgA deficiency

862. Pneumocystis carinii is a fungus because:


a. rRNA, mitochondrial protein gene sequence and presence thymidylate late
synthase
b. Cell wall contains glucans
c. Antifungals are effective against P. carinii
d. Commonest infection in AIDS

863. Which dye is most suitable for fungus demonstration in biopsy:


a. Alizarin red
b. Verihoff dye
c. Mason’s trichrome
d. PAS

864. In HIV infected individual Gram stain of lung aspirate shows yeast like
morphology. All of the following are the most likely diagnosis except:
a. Candida tropicalis
b. Cryptococcus neoformans
c. Penicillium marneffei
d. Aspergillus fumigatus

865. Pneumocystis carinii is:


a. Bacteria
b. Fungus
c. Virus
d. Parasite

866. Rhinosporiduim seeberi is a


a. Bacteria
b. Algae
c. Protozoa
d. Virus

867. Pneumocystis carinii is diagnosed by:


a. Silver nitrate staining
b. Leishman staining
c. Fontana staining
d. Acid fast staining

868. Blastomycosis is characterized by all except:


a. Yeast-like fungus
b. Commonly involves lung and skin
c. Dimorphic fungus
d. Common in South America

854. 855. c 856. 857. c 858. 859. 860. 861. a 862. a 863. d
a a d b a

864. 865. 866. c 867. 868.


d b a d

869. Which of the following feature is used for identification of Cryptococcus


neoformans?
a. Oxidase +ve
b. Dextran fermentation
c. Hydrolyze urea
d. Ability to grow at 42°C

870. Renauld Braude phenomenon is seen is:


a. Candida albicans
b. Candida psittaci
c. Histoplasma
d. Cryptococcus

871. Which of the following is only yeast:


a. Candida
b. Mucor
c. Rhizopus
d. Cryptococcus

872. In a patient, corneal scraping reveals narrow angled septate hyphae.


Which of the following is the likely etiologic agent:
a. Mucor
b. Aspergillus
c. Histoplasma
d. Candida

873. An early diabetic has left sided orbital cellulitis CT scan of paranasal
sinus shows evidence of left maxillary sinusitis. Gram stained smear of the
orbital exudate shows irregularly branching septate hyphae. The following is
most likely etiological agent:
a. Aspergillus
b. Rhizopus
c. Mucor
d. Candida

874. Common fungus causing corneal ulcer:


a. Aspergillus
b. Mucor
c. Fusarium
d. Sporothrix

875. A young woman complains of recurrent rhinitis, nasal discharge and


bilateral nasal blockage since one year. She also had history of allergy and
asthma. On examination, multiple polyps with mucosal thickening and
impacted secretions are seen in nasal cavities. Biopsy was taken and the
material on culture showed many hyphae with dichotomous branching
typically at 45 degree. Which of the following is most likely organism
responsible?
a. Rhizopus
b. Aspergillus
c. Mucor
d. Candida

876. Which of the following is an aseptate fungus


a. Aspergillus
b. Candida
c. Nocardia
d. Rhizopus

877. Aseptate hyphae is not seen in:


a. Rhizopus
b. Mucor
c. Aspergillus
d. None

878. A series of ulcers in lower extremities in sub-Himalayan area is often


caused by:
a. Trichophyton rubrum
b. Pseudallescheria boydii
c. Cladosporium species
d. Sporothrix schenckii

879. The following fungi are thermally dimorphic, except:


a. Sporothrix schenkii
b. Cryptococcus neoformans
c. Blastomyces dermatitidis
d. Histoplasma capsulatum
880. A gardener has multiple vesicles on hand and multiple eruptions along
the lymphatic. Most common fungus responsible is:
a. Sporothrix schenckii
b. Cladosporium
c. Histoplasma
d. Candida

881. Darling disease is caused by:


a. Histoplasma
b. Cryptococcus
c. Rhizopus
d. Blastomycosis

882. pH of Sabouraud’s dextrose agar is adjusted to:


a. 4-6
b. 1-2
c. 6-8
d. 8-10

869. c 870. 871. 872. 873. 874. 875. 876. d 877. c 878. d
a d b a a b

879. 880. 881. 882.


b a a a
Mycology — Explanations
810. A. Septate hyphae
Aspergillus has septate hyaline hyphae with dichotomous branching.
Pseudohyphae seen in candida albicans
Metachromatic hyphae seen in dematiaceous fungi

811. A. Cryptococcus neoformans


In polarizing microscopy, Cryptococcus neoformans has Maltese cross appearance.
Blastomyces in biopsy look like broad based yeast with refractile cell wall.
Candida albicans in tissues forms Pseudohyphae

812. A. Rhinisporidium seeberi


Now R. seeberi is considered as protozoa and it is not cultivable.
Other choices are the fungi grown in SDA.

813. B. HTN
Predisposing factor for candida infection is mainly due to CMI deficiency.
DM, HIV, Pregnancy, Malignancy, Steroid therapyand transplantation.

814. A. Candida
Intertrigo refers to a type of inflammatory rash (dermatitis) of the superficial skin that
occurs within a person’s body folds.
Areas of the body which are more likely to be affected by intertrigo include the
inframammary fold, intergluteal cleft, armpits, and spaces between the fingers or
toes. Skin affected by intertrigo is more prone to infection than intact skin.
The term “intertrigo” commonly refers to a secondary infection with bacteria (such as
Corynebacterium minutissimum), fungi (such as Candida albicans), or viruses. A
frequent manifestation is Candidial intertrigo.

815. D. C. neoformans
Bacterial meningitis are usually pyogenic meningitis except leptospiral and
tuberculous meningitis.
Fungal and viral meningitis are aseptic meningitis. Lymphocytes are predominant in
microscopic examination

816. B. Phaeohyphomycosis
Phaeohyphomycosis is a heterogeneous group of mycotic infections caused by
dematiaceous fungi whose morphologic characteristics in tissue include hyphae,
yeast-like cells, or a combination of these.
They are associated with pigmented hyphae

817. B. soil containing pigeon feces


C. neoformans occurs widely in nature and grows abundantly in soil containing
bird(especially pigeon) droppings. The birds are not infected.
Human infection resultsfrom inhalation of the organism. There is nohuman-to-human
transmission.
C. gattii is associated with eucalyptus trees, most often in the northwestern states of
theUnited States. It is also found in subtropical and tropical areas of many countries.
818. B. Aspergillus
Most common cause of fungal keratitis is Aspergillus fumigatus, followed by Candida
albicans and Fusarium.

819. C. Coccidioidomycosis
C. immitis is a dimorphic fungus that exists as a mold in soil and as a spherule
intissue .
Some infected persons have an influenza like illness with fever and cough. About
50% have changes in the lungs (infiltrates, adenopathy, or effusions) as seen on
chest X-ray, and 10% develop erythema nodosum or arthralgias.
This syndrome is called “valley fever” (in the San Joaquin Valley of California) or
“desert rheumatism” (in Arizona); it tends tosubside spontaneously

820. A. Sporothrix schenckii


Sporotrichosis is a chronic pyogenic granulomatous lesion ofthe skin and
subcutaneous tissue caused by Sporothrix schenckii.
S. schenckii is a dimorphic fungus found all over the world. The fungus is found in
soil, decaying woods, thorns, and on infectedanimals including rats, cats, dogs, and
horses.
It causes Rose Gardener’s disease – Ulcers along the lymphatic draining.

821. B. Cryptococcus neoformans


Dimorphic fungi exist as hyphal/mycelial forms in the soil and in the cultures at 22–
25°C. They occur as yeasts or other structuresin human tissue and in the culture at
37°C
Examples include Coccidioides immitis, Paracoccidioides brasiliensis,
Histoplasmacapsulatum, Blastomyces dermatitidis, Sporothrix schenckii and P.
marneffi.

822. E. Skin
Sporothrix schenckii is a dimorphic fungus. The mold form lives on plants, and
theyeast form occurs in human tissue.
When spores of the mold are introduced into theskin, typically by a thorn, it causes a
local pustule or ulcer with nodules along thedraining lymphatics.

823. D. C. immitis
C. immitis : inhalation of arthroconidia
Candidia is endogenous.
Sporothrix transmitted via skin mainly by thorn prick.

824. C. M. furfur
Tinea versicolor (pityriasis versicolor), a superficial skin infection of
cosmeticimportance only, is caused by Malassezia furfur.
The lesions are usually noticed ashypopigmented areas, especially on tanned skin in
the summer.
Diagnosis is usually made by observing this mixture in KOH preparations ofskin
scrapings. Culture is not usually done. The treatment of choice is topicalmiconazole,
but the lesions have a tendency to recur. Oral antifungal drugs, such asfluconazole or
itraconazole, can be used to treat recurrences.

825. C. sclerotic bodies


This is a slowly progressive granulomatous infection that is caused by several
soilfungi (Fonsecaea, Phialophora, Cladosporium, etc.) when introduced into the
skinthrough trauma.
In the clinical laboratory,dark brown, round fungal cells are seen in leukocytes or
giant cells called as sclerotic bodies or copper penny bodies. The disease istreated
with oral flucytosine or thiabendazole, plus local surgery.

826. C. Sporothrix schenckii


Refer Q. No 820.

827. A. Penicillium marneffei


Penicillium marneffei is a dimorphic fungus that causes tuberculosis-like disease
inAIDS patients, particularly in Southeast Asian countries such as Thailand.
It divides by binary fission. The treatment of choiceconsists of amphotericin B for 2
weeks followed by oral itraconazole for 10 weeks.

828. A. Voriconazole
Breakthrough invasive fungal diseases during voriconazole treatment are concerning,
as they are associated with high rates of mortality and pathogen distribution.
Voriconazle is not effective against Zygomycetes.

829. A. Mucor spp.


Zygomycetes are having aseptate hyphae. Mincing leads to death of the hyphae. So
culture yield will be very low.

830. C. dermatophytes
Hypersensitivity to dermatophyte antigens leads toappearance of secondary eruption
in sensitized patients because of presence of circulation of allergenicproducts.

831. C. Malassezia
Malassezia furfur is a lipophilic fungus that is found on skin.
It causesfungemia, primarily in premature infants on high-lipid intravenous
supplements.

832. D. Sporulation has no diagnostic value


Fungal spores are classified as Sexual and asexual spores and they are used as a
diagnostic tool for fungal infections

833. A. Candida albicans


Candida albicans is non thermally dimorphic fungus which is mold form in issues and
yeast form in culture.
All other choices are Thermally dimorphic fungi

834. D. Ascospores
Example for Asexual spores are, Blastospore, conidiospore, arthrospore and
Chlamydospore

835. A. Malassezia furfur


Refer Q. no 824

836. B. Confined to keratinized tissues


Dermatophytoses are caused by fungi (dermatophytes) that infect only
superficialkeratinized structures (skin, hair, and nails), not deeper tissues.
The most importantdermatophytes are classified in three genera: Trichophyton,
Epidermophyton, andMicrosporum.

837. D. Anthropophilic dermatophyte is more commonly associated than


Zoophilic and Geophilic dermatophytes
Refer Q. no : 836

838. B. Trichophyton mentagrophyte


This test is performed to differentiate T. rubrum from T. mentagrophytes.
T. mentagrophyte is positive and T. rubrum is negative for the test.

839. C. Primary pulmonary sporotrichosis is the most common presentation


This subcutaneous, nodular, fungal disease is generally not painful. When it spreads
via thelymphatics (lymphocutaneous sporotrichosis), it produces a chain of lesions on
the extremities, with the older (lower) lesions ulcerating and the newer (upper) ones
starting nodular.

840. A. Niger seed agar


A case of Cryptococcal meningitis.
Niger seed agar and bird seed agar is used to demonstrate melanin production and it
produces black or brown color colonies.

841. B. Hyaline fungi are the etiological agent


Refer Q No 825

842. D. Less extensive with osteolytic lesions.


Eumycotic mycetoma is a subcutaneous fungal disease characterized by (1)
swelling(tumefaction), (2) sinus tracts erupting through the skin (if not treated), and
(3) presence of“sulfur” granules (microcolonies) in the exudate.
It is caused by Pseudallescheria boydii and Madurella species, which are filamentous
truefungi found in soil or on vegetation; entry is by traumatic implantation.3. It
usually occurs in rural, third-world agricultural workers in the tropics

843. B. Oval budding yeasts inside macrophages


Inhaled spores are engulfed by macrophages and develop into yeast forms.
Intissues, H. capsulatum occurs as an oval budding yeast insidemacrophages. The
yeasts survive within the phagolysosome of themacrophage by producing alkaline
substances, such as bicarbonate and ammonia,which raise the pH and thereby
inactivate the degradative enzymes of thephagolysosome.

844. D. Dissemination is rarely encountered


H. capsulatum var. capsulatum - classical histoplasmosis
H. capsulatum var. duboisii - African histoplasmosis with frequent dissemination in
skin and bone.

845. D. mucormycosis
Patients with diabetic ketoacidosis, burns, bone marrow transplants, or leukemiaare
particularly susceptible. Diabetic patients are particularly susceptible torhinocerebral
mucormycosis, in which mold spores in the sinuses germinate to formhyphae that
invade blood vessels that supply the brain.
Rhizopusoryzae, causes about 60% of cases of mucormycosis.
In biopsy specimens, organisms are seen microscopically as nonseptate hyphaewith
broad, irregular walls and branches that form more or less at right angles

846. C. Widespread in environment, conidia may be inhaled microscopic


appearance in specimen reveals dichotomous branching and septate hyphae
Aspergillus species exist only as molds; they are not dimorphic. They have
septatehyphae that form V-shaped (dichotomous) branches
These molds are widely distributed in nature. They grow on decaying
vegetation,producing chains of conidia. Transmission is by airborne conidia.A.
fumigatus can colonize and later invade abraded skin, wounds, burns, the cornea,the
external ear, or paranasal sinuses. It is the most common cause of fungalsinusitis.
In immunocompromised persons, especially those with neutropenia, it caninvade the
lungs and other organs, producing hemoptysis and granulomas.

847. A. Serum β galactomannan


In persons with invasive aspergillosis, there may be high titers ofgalactomannan
antigen in serum.
Patients with ABPA have high levels of IgEspecific for Aspergillus antigens and
prominent eosinophilia.

848. A. Candida
Fever more > 38.5°C with absolute neutrophil count less than 500/µl.
Defect in CMI or neutropenia, disseminated Candidainfections are common.

849. B. Mycotoxin
Mycotoxins like aflatoxins, are coumarin derivatives produced byAspergillus flavus
that cause liver damage and tumors in animals and are suspectedof causing hepatic
carcinoma in humans.
Aflatoxins are ingested with spoiled grainsand peanuts and are metabolized by the
liver to the epoxide, a potent carcinogen.
Aflatoxin B1 induces a mutation in the p53 tumor suppressor gene, leading to a lossof
p53 protein and a consequent loss of growth control in the hepatocyte.

850. E. Epidermophyton floccosum


Club chaped macroconidia without microconidia suggestive of Epidermophyton.
Pencil shape Macroconidia – Trichophyton
Spindle shape macroconidia – Microsporum.

851. B. Penicillium marneffei


P. marneffei is the only dimorphic fungus in the genus Penicillium.
It has been reported as an important opportunistic pathogenin AIDS patients.
The fungus causes tuberculosis-like disease inpatients with AIDS in Southeast Asian
countries like Thailand.
Few case reports have also been documented from North Eastern part of India

852. A. Mucicarmine – Cryptococcus, C. Methanamine silver – Histoplasma & E.


PAS – Sporothrix schenckii.
Candida will not take Giemsa stain and Pneumocystis will not take Grams stain.
853. A. Candida albicans & C. Candida dubliniensis
Germ tube is a rapid method for identification of C. albicans and Candida
dubliniensis.
This test dependson the ability of C. albicans to produce germ tube within 2
hourswhen incubated in serum at 37°C.
This phenomenon iscalled Reynold–Braude phenomenon

854. A. Aspergillus flavus


A mycotoxin is a toxic secondary metabolite produced by organisms of the fungus
kingdom and is capable of causing disease and death in both humans and other
animals.
Aflatoxin produced by A. flavus

855. C. Trichophyton
Refer Q No 836

856. A. E. floccosum
Ringworm infection of the foot is called Tinea pedis or athletes foot.
It is caused by T. rubrum, T. mentagrophytesvar. interdigitale and E. floccosum

857. C. Azoles act on them


Azoles are acting by interfering the synthesis of ergosterol and leads to cell
membrane damage.
They act via 1-4 alpha demethylase

858. D. T. mentagrophytes
Other zoophilic dermatophytes areT. equinum T. verrucosum M. canis and M.
equinum

859. b. Indian ink preparation


Capsule is best demonstrated by negative staining – India ink or Nigrosin.
Methanamine silver nitrate is for fungal cell wall.

860. A. Cryptococcus
CrAg test – Cryptococcal Antigen detection by Latex agglutination method is very
simple, rapid and most sensitive and specific method for diagnosis of Cryptococcal
meningitis.

861. A. Renal transplant recipient


Disease caused byC. neoformans occurs mainly in patients with reduced cell-
mediated immunity, especially AIDS patients, renal transplant recipients and
Hodgkin’s lymphoma.

862. A. rRNA, mitochondrial protein gene sequence and presence thymidylate


late synthase
Pneumocystis is a unicellular fungus found in the respiratorytracts of many mammals
and humans.
The genus Pneumocystiswas initially mistaken for trypanosome, then later as a
protozoan.
Biochemical analysis of the nucleic acid composition ofPneumocystis rRNA and
mitochondrial DNA in 1980s established it as a fungus.
The cyst wall closely resembles that offungi. However, it does not haveergosterol in
its membrane asdo the fungi, but instead has cholesterol

863. D. PAS
Potassium hydroxide in a wet mount (KOH mount) of skin scrapings breaks down the
humancells, enhancing the visibility of the unaffected fungus.
A nigrosin or India ink wet mount of cerebrospinal fluid (CSF) highlights the capsule
of Cryptococcus neoformans but is very insensitive
A Giemsa or Wright’s stain of thick blood or bone marrow smear may detect the
intracellularHistoplasma capsulatum.
Calcofluor white stain “lights up” fungal elements in exudates, small skin scales, or
frozen sections under a fluorescent microscope, giving the fungus a fluorescent blue-
white appearanceon a black background
Tissue specimens –GMS and PAS staining

864. D. Aspergillus fumigatus


Refer Q No 810.

865. B. Fungus
Currently Pneumocystis classified as fungus by molecular method.

866. C. Protozoa
Currently R. seeberi is classified under protozoa.

867. A. Silver nitrate staining


GMS staining of BAL specimen
Demonstration of cup shape cyst is diagnostic for Pneumocystis

868. D. Common in South America


This fungus is endemic primarily in eastern North America, especially in the
regionbordering the Ohio, Mississippi, and St. Lawrence rivers, and the Great
Lakesregion. Less commonly, blastomycosis has also occurred in Central and
SouthAmerica, Africa, and the Middle East.
It grows in moist soil rich in organicmaterial, forming hyphae with small pear-shaped
conidia. Inhalation of the conidiacauses human infection

869. C. Hydrolyze urea


Urease test and inositol fermentation test are positive for C. neoformans

870. A. Candida albicans


Refer Q. No 853

871. D. Cryptococcus
Candida – yeast like (Pseudohyphae)
Rhizopus and Mucor are molds

872. B. Aspergillus
Refer Q. No 810

873. A. Aspergillus
Refer Q. No 810
874. A. Aspergillus
Refer Q. No 818

875. B. Aspergillus
Refer Q. No 810

876. D. Rhizopus
Aspergillus :Septate hyphae with acute angle branching.
Mucor and Rhizopus – Aseptate hyphae with right angle branching
Candida - pseudohyphae

877. C. Aspergillus
Aspergillus :Septate hyphae with acute angle branching.
Mucor and Rhizopus – Aseptate hyphae with right angle branching

878. D. Sporothrix schenckii


Refer Q. No 820

879. B. cryptococcus neoformans


Refer Q. No : 821

880. A. Sporothrix schenckii


Refer Q. No 820

881. A. Histoplasma
Pulmonary manifestations of histoplasma is called as Darling’s disease
It is dimorphic fungus. Non capsulated.
Also causes fungal flu and mimics like tuberculosis.

882. A. 4-6
Fungi are frequently cultured on Sabouraud’s agar, which facilitates theappearance
of the slow-growing fungi by inhibiting the growth of bacteria in thespecimen.
Inhibition of bacterial growth is due to the low pH (4-6) of the medium and tothe
chloramphenicol and cycloheximide that are frequently added.
The appearance ofthe mycelium and the nature of the asexual spores are frequently
sufficient to identifythe organism.
Parasitology — Questions
883. The most virulent Plasmodium species causing malaria is:
a. Plasmodium vivax
b. Plasmodium falciparum
c. Plasmodium ovale
d. Plasmodium malaria

884. Which egg does not float in a saturated solution of saline?


a. Ankylostoma eggs
b. Trichuris eggs
c. Unferitilized eggs of Ascaris
d. Fertilized eggs of Ascaris

885. All causes brain lesions except:


a. Giardiasis
b. Tuberculosis
c. Cysticercosis
d. Bacteroides

886. HIV patient with malabsoption, fever, chronic diarrohea, with acid fast
positive organism. What is the causative agent?
a. Giardia
b. Microsporidia
c. Isospora
d. E. histolytica

887. About Congenital toxoplasmosis, false is


a. Diagnosed by detection of IgM in cord blood
b. IgA is more sensitive than IgM for detection
c. Dye test is gold standard for IgG
d. Avidity testing must be done to differentiate between IgA & IgM

888. Malabsorption caused by A/E


a. Giardia
b. Ascaris
c. Strongyloides
d. Capillaria philipensis

889. Post kalazar dermal leishmaniasis is caused by


a. L.tropica
b. L.infantum
c. L.major
d. L.donovani
890. All the nematodes give birth to larvae except
a. Trichuris trichiura
b. Trichnella spiralis
c. Dracunculus medinensis
d. Wuchereria bancrofti

891. Amoebiasis is not transmitted through


a. Vertical Transmission
b. Faeco-oral
c. Cockroach
d. Oral

892. Which of the following is not a mosquito borne disease?


a. Filariasis
b. Trypanosomiasis
c. Dengue
d. Yellow fever

893. Sabin & Feldman dye test is used to diagnose infection with
a. Filariasis
b. Toxoplasmosis
c. Histoplasmosis
d. Ascariasis

894. Seat worm is


a. Enterobius
b. Drancunculus
c. Ankylostoma
d. Necator

895. Babesiosis is transmitted by


a. Dogs
b. Jackals
c. Hard ticks
d. Monkeys

896. Cranial calcification is caused by


a. Toxoplasmosis
b. Syphilis
c. Rubella
d. Tetanus

897. An AIDS patient presents to his primary care physician with a two week
history of watery, non-bloody diarrhea. The most likely diagnosis is which of
the following?
a. Acid-fast bacilli
b. Enterocytozoon
c. Cryptosporidium
d. Yeast

883. 884. c 885. 886. c 887. 888. 889. 890. a 891. a 892. b
b a d b d

893. 894. 895. c 896. 897. c


b a a

898. A 30 year old female stored her contact lenses in tap water. She noticed
deterioration in vision and visited an ophthalmologist, who diagnosed her
with keratitis. Culture would likely reveal which of the following?
a. Acanthamoeba
b. Babesia
c. Entamoeba coli
d. Naeglaria
e. Pneumocystis

899. The diagnostics characteristics of Plasmodium falciparum are best


diagnosed by which of the following statements?
a. An important diagnostic feature is the irregular appearance of the edges of the
infected red blood cell
b. A period of 72 hours is required for the development of the mature schizont,
which resembles a rosette with only 8 to 10 oval merozoites
c. Except in infections with very high parasitaemia, only ring forms of early
trophzoites and the gametocytes are seen in the peripheral blood.
d. Schuffner stippling is routinely seen in red blood cells that harbor parasites
e. The signet ring shaped trophozoites is irregular in shape with amoeboid
extensions of the cytoplasm

900. A divorced working mother takes her 4-year-old child to day care center.
She has noticed that the child’s frequent stools are non-bloody, foul smelling
and fatty. The child has no fever. One should suspect infection with which
parasite?
a. Amebiasis
b. Ascariasis
c. Balantidiasis
d. Giardiasis

901. Human infection with beef tapeworm usually is less serious than
infection with pork tape worm, because of which of the following
statements?
a. Acute intestinal obstruction is less common in beef tape worm infection
b. Beef tape worm eggs cause less irritation of the mucosa of the digestive tract
c. Larval invasion does not occur in beef tape worm infection
d. The adult beef tape worms are smaller in size
e. Toxic by-products are not given off by adult beef tape worm
902. Trypanosoma cruzi initially penetrates through the mucous membranes
and then multiplies in a lesion known as chagoma. In the chronic stage of
the disease, where are the main lesions often observed?
a. Digestive tract and heart
b. Heart and respiratory tract
c. Heart and liver
d. Liver and spleen
e. Spleen and pancreas

903. One of the most clinically significant infections in patients with AIDS is
Pneumocystis jirovecii pneumonia. The method of choice for detection of P.
jirovecii in respiratory specimens is which of the following?
a. Culture in rat lung cells
b. Direct fluorescent antibody microscopy
c. Indirect fluorescent antibody microscopy
d. Methenamine-silver stain
e. Toluidine blue stain

904. Which of the following protozoa is known to exist in the trophozoite


stage only?
a. Balantidium coli
b. Entamoeba histolytica
c. Giardia lamblia
d. Toxoplasma gondii
e. Trichomonas vaginalis

905. A medical technologist visited Goa and consumed raw fish daily for two
weeks. Six months after her return to her home, she had a routine physical
examination and was found to be anemic. Her vitamin B12 levels were below
normal. What is the most likely cause of her condition?
a. Cysticercosis
b. Infection with fish tape worm
c. Infection with Parvovirus B19
d. Infection with Yersinia
898. a 899. c 900. d 901. c 902. a 903. b 904. e 905. b

906. A renal transplant patient was admitted for graft rejection and
pneumonia. A routine evaluation of his stool showed rhabditiform larva.
Subsequent follow-up revealed similar worms in his sputum. What is the
most likely organism?
a. Ascaris
b. Hymenolepis
c. Loa Loa
d. Necator
e. Strongyloides
907. Which of the following infections requires a mosquito for transmission?
a. Babesiosis
b. Bancroftian filariasis
c. Dog tape worm
d. Guinea worm
e. Leishmaniasis

908. E. histolytica infection is best diagnosed by which of the following?(PGI)


a. Baermann technique
b. Dilution followed by egg count
c. ELISA
d. Examination of a cellophane tape swab
e. Sigmoidoscopy and aspiration of mucosal lesions

909. A protozoan with a characteristic jerky motility is most commonly


observed in which of the following?
a. Biopsied muscle
b. Blood
c. Duodenal contents
d. Sputum
e. Vaginal secretions

910. Crabs act as secondary intermediate hosts for


a. Paragonimus westermanii
b. Fasciola hepatica
c. Opisthorchis sinensis
d. Fasciolopsis buskii

911. Which of the following specimens is used to diagnose infections with


Naegleria fowleri?
a. Cerebrospinal fluid
b. Corneal scraping
c. Blood
d. Urine

912. A previously healthy, 24-year-old man is brought to an emergency room


because of seizures. A CAT scan reveals a solitary calcified lesion, 2 cm in
diameter, in the right cerebral hemisphere. What is the most probable
diagnosis?
a. Central nervous system coccidioidomycosis
b. Cerebral cysticercosis
c. Cerebral echinococcosis
d. Extraintestinal amebiasis

913. Which of the following, if ingested raw or poorly cooked, can be the
source of Taenia saginata infections?
a. Pork
b. Beef
c. Lamb
d. Fish

914. A woman presents in third trimester with high fever. Her PS is positive
for P.vivax.
a. There is a high chance of her baby developing malaria
b. The drug of choice is artesunate
c. She can develop complications
d. The baby may be LBW

915. All of the following can differentiate pathogenic from non-pathogenic


Entamoeba except
a. Hematophagous trophozoite
b. Zymodeme analysis
c. Nested PCR
d. DNA PCR

916. The diagnostic characteristics of Plasmodium falciparum are best


described by which of the following statements
a. An important diagnostic feature is the irregular appearance of the edges of the
infected RBC
b. Except in infections with very high parasitemia only ring forms of the early
trophozoites and the gametocytes are seen in the peripheral blood
c. Schuffner stippling is routinely seen in RBC that harbor parasites
d. The signet ring shaped trophozoites is irregular in shape with amoeboid
extensions of the cytoplasm’
906. e 907. b 908. 909. e 910. a 911. a 912. b 913. b 914. d
c,e

915. d 916. b

917. Babesiosis is most commonly seen in


a. AIDS patients
b. Foresters
c. Patients without a spleen
d. Transfusion recipients

918. An AIDS patient presents to OPD with a 2 months history of watery, non-
bloody diarrhea. The stool reveals acid fast oocysts of 6 µm size. All are
characteristic except
a. Autoinfection is seen
b. Macrolides is the treatment of choice
c. Pulmonary involvement may occur
d. Ag detection in stool and direct fluorescence testing are alternative methods of
detection
919. The best method to detect Strongyloides larvae is which of the following
a. Bearmann technique
b. Dilution followed by egg count
c. NIH swab
d. Sigmoidoscopy and aspiration of mucosal lesions

920. A teenager who works in a dog kennel after school has had a skin rash,
eosinophilia and an enlarged liver and spleen for 2 years. Which of the
following is the most likely cause of this infection?
a. Schistosomiasis
b. Toxoplasmosis
c. Trichinosis
d. Visceral Larva migrans

921. Onchocerciasis:
a. Is caused by a protozoan
b. Transmitted by the bite of fly
c. Is confined to the African continent
d. Ocular involvement results from lymphatic migration of the parasites

922. In Toxocariasis:
a. The infestation is acquired from both dogs and cats
b. Infestation occurs through ingestion of larvae
c. Intraocular calcification is a common feature of ocular toxocariasis
d. Steroid is contraindicated

923. A 30 year old patient presented with features of acute


meningoencephalitis in the casualty. His CSF on wet mount microscopy
revealed motile unicellular microorganisms. The most likely organism is
a. Naegleria fowleri
b. Acanthamoeba castellani
c. Entamoeba histolytica
d. Trypanosoma cruzi
e. Toxoplasma gondi

924. Winter bottoms sign in sleeping sickness refers to


a. Unilateral conjunctivitis
b. Posterior cervical lymphadenopathy
c. Narcolepsy
d. Transient erythema
e. Arthralgia

925. True about toxoplasmosis is all except


a. In adults toxoplasmosis is usually asymptomatic
b. IgG antibodies are diagnostic in congenital toxoplasmosis
c. Is a anthroponotic disease
d. Encephalitis is uncommon in immunocompetent individuals
e. The incidence of transplacental infection is highest in third trimester but the
infant is not severely affected

926. The pathogenicity of Entamoeba histolytica is indicated by (PGI)


a. Zymodeme pattern
b. Size
c. Nuclear pattern
d. ELISA test
e. Haematophagus trophozoite

927. Giardiasis is associated with (PGI)


a. Common variable immunodeficiency
b. C1 esterase deficiency
c. C8 deficiency
d. Anaemia
e. Autosomal recessive agammaglobulinemia
917. c 918. b 919. a 920. d 921. b 922. a 923. a 924. b 925. b

926. a, 927.
e a,e

928. About acute primary amoebic meningoencephalitis true is?


a. Meningitis caused by Acanthamoeba species is acute in nature
b. Diagnosed by trophozoite in CSF
c. Caused by Feco-oral transmission
d. More common in tropical climate

929. True about amoebic colitis is: (PGI)


a. Caused by E. histolytica
b. Cyst contains 8 nuclei
c. Flask-shaped ulcers are present
d. Cecum is most commonly affected
e. Is premalignant

930. An anxious mother brought her 4-year-old daughter to the pediatrician.


The girl was passing loose bulky stools for the past 20 days. This was often
associated with pain in abdomen. The pediatrician suggested the stool
examination, which showed the following organism. Identify the organism:
a. Entamoeba histolytica
b. Giardia lamblia
c. Cryptosporidium
d. E. coli

931. A patient present with diarrhea. Analysis of stool on wet mount shows
mobile protozoa without RBCs and pus cells. The diagnosis is:
a. Balantidium Coli
b. Giardiasis
c. Trichomonas hominis
d. Entamoeba histolytica

932. All parasites cause encephalitis except:


a. Entamoeba histolytica
b. T. gondii
c. Angiostrongylus cantonensis
d. T. cruzi

933. Which of the following infestations leads to malabsorption?


a. Giardia lamblia
b. Ascaris lumbricoides
c. Necator americana
d. Ancylostoma duodenale

934. Visceral leishmaniasis: (PGI)


a. Caused by L. tropica
b. Post-leishmaniasis dermatitis is common
c. Antimonial are useful drugs
d. Diagnosed by blood smear
e. Vector is Phlebotomus sergentii

935. Mucocutaneous leishmaniasis is caused by:


a. L. braziliensis
b. L. tropica
c. L. donovani
d. L. orientalis

936. Which of the following is true about Giardia:


a. CFT is diagnostic
b. Trophozoites and cysts are seen in man
c. Lives in lower intestine
d. Invades normal mucosa

937. All of the following statements about toxoplasmosis are true except:
a. Oocyst in freshly passed cat’s faces is not infective
b. May spread by organ transplantation
c. Maternal infection after 6 months has high risk of transmission
d. Arthralgia, sore throat and abdominal pain are the most common manifestation

938. Which of the following is true about P. falciparum?


a. James dots are seen
b. Accolé forms are seen
c. Relapse are frequent
d. Longest incubation period

928. b 929. 930. b 931. b 932. c 933. a 934. c,d


a,c,d

935. a 936. b 937. d 938. b

939. Toxoplasmosis in the fetus can be best confirmed by:


a. IgM antibodies against Toxoplasma in the mother
b. IgM antibodies against Toxoplasma in the fetus
c. IgG antibodies against Toxoplasma in the mother
d. IgG antibodies against Toxoplasma in the fetus

940. A 35-year-old male suffering from sudden onset of high-grade fever. On


malarial slide examination all stages of parasites are seen with schizonts of
20 microns size with 14-20 merozoites per cell and yellow brown pigment.
The diagnosis is
a. Plasmodium falciparum
b. Plasmodium vivax
c. Plasmodium malariae
d. Plasmodium ovale

941. True about toxoplasmosis: (PGI)


a. Due to ingestion of sporocyst with meat
b. Due to ingestion of oocyst from cat’s faeces
c. Spiramycin given in pregnancy
d. Due to bite of Anopheles mosquito
e. Mostly symptomatic

942. Which is the infective stage for mosquito in case of Plasmodium vivax:
a. Gametocyte
b. Sporozoite
c. Zygote
d. Merozoite

943. About Microsporidia, all of the following are false except: (PGI)
a. It is a fungus
b. It is a protozoa
c. It is a bacteria
d. It is trematode
e. It is associated with diarrhea in HIV patients

944. Causative agent of malaria:


a. Protozoa
b. Mosquito
c. Bacteria
d. Virus

945. Protozoa associated with megaesophagus:


a. Trypanosome
b. Ameba
c. Giardia
d. Gnathostoma

946. Toxoplasma in children causes:


a. Chorioretinitis
b. Conjunctivitis
c. Keratitis
d. Papillitis

947. True about Giardiasis:


a. Only cyst is infective
b. Reside in caecum
c. Only man to man transmission
d. Exists in one phase

948. A traveler present with conjugated hyper-bilirubinemia and on


investigations an egg was found in his biliary tract. Likely organism:
a. Clonorchis sinensis
b. Fasciola buski
c. Gnathostoma
d. Ascaris

949. Ingestion of raw fish leads to gallbladder cancer due to:


a. Clonorchis sinensis
b. Hymenolepis diminuta
c. Angiostrongylus
d. D. latum

950. Commonest parasite of CNS in India is:


a. Schistosomiasis
b. Cysticercosis
c. Trichinella spiralis
d. Hydatid cyst

951. Which of the following is the mostcommon location of intracranial


neurocysticerosis?
a. Brain parenchyma
b. Subarachnoid space
c. Spinal cord
d. Orbit

939. b 940. b 941. b,c 942. a 943. 944. a 945. a 946. a


b,e

947. a 948. a 949. a 950. b 951. a

952. A child from Bihar comes with fever. Blood examination shows sheathed
microfilaria with nuclei up to tail tip. The diagnosis is:
a. B. malayi
b. W. bancrofti
c. Loa loa
d. Onchocerca volvulus

953. Kalu, 30-year-old man, presented with subcutaneous itchy nodules over
left iliac crest. On examination, they are firm, nontender, and mobile skin
scrapings contain microfilaria and adult worms of:
a. Loa loa
b. Onchocerca volvulus
c. Brugia malayi
d. Mansonella perstans

954. Nematodes are differentiated from other worms by all except:


a. Absent fragmentation
b. Flat or fleshy leaf-like worms
c. Separate sexes
d. Cylindrical body
e. GIT is formed completely

955. Which of the following is not a neuroparasite?


a. Taenia solium
b. Acanthamoeba
c. Naegleria
d. Trichinella spiralis

956. Pigs are reservoir for: (PGI)


a. T. solium
b. T. saginata
c. Trichinella spiralis
d. Ancylostoma

957. Autoinfection is seen with:


a. Ankylostoma
b. Enterobius
c. Echinococcus
d. Ascariasis

958. All float in a saturated salt solution except:


a. Clonorchis sinensis
b. Fertilized eggs of ascaris
c. Larva of Strongyloides
d. Trichuris trichiura

959. Parasites causing lung infestation are: (PGI)


a. H. nana
b. Paragonimus westermanii
c. Taenia saginata
d. E. granulosus
e. E. multilocularis

960. Schistosomiasis is an example of:


a. Meta-zoonosis
b. Cyclo-zoonosis
c. Direct-zoonosis
d. Sporo-zoonoses

961. Cercariae are infective form of:


a. S. haematobium
b. P. westermanii
c. F. hepatica
d. T. solium

962. Wuchereria bancrofti, true is:


a. Unsheathed
b. Tail tip free from nuclei
c. Non-periodic
d. All

963. All cause malabsorption syndrome except:


a. Giardiasis
b. Ascaris
c. Strongyloides
d. Capillaria phillipinesis

964. Virus can be isolated from clinical samples by cultivation in the following
except:
a. Tissue culture
b. Embryonated eggs
c. Animals
d. Chemically defined media
965. Prokaryotes are characterized by:
a. Absence of nuclear membrane
b. Presence of microvilli on its surface
c. Presence of smooth endoplasmic reticulum
d. All of the above
952. c 953. b 954. b 955. d 956. a,c 957. b 958. a 959.
b,d,e

960. a 961. a 962. b 963. b 964. d 965. a


Parasitology — Explanations
883. B. Plasmodium falciparum
Plasmodium falciparum possesses a number of virulence factors and its pathogenesis
is different from other species. Hence the disease is more acute and severe in nature
with more complications than the benign malaria.

884. C. Unfertilized eggs of Ascaris


Fertilised eggs of Ascaris float in saturated salt solution.

885. A. Giardiasis
Giadia lamblia causes only gastrointestinal manifestations.

886. C. Isospora
Cryptosporidium, Cyclospora and Isospora are acid fast parasites that can cause
opportunistic infections (diarrhea) in HIV infected patients.

887. D. Avidity testing must be done to differentiate between IgA & IgM
Congenital toxoplasmosis can be diagnosed by detecting: Toxoplasma antigens in
amniotic fluid. Toxoplasma specific genes by PCR. IgM antibodies in fetal blood by
ELISA or IFA. IgA antibodies in fetal blood. Double-sandwich IgA-ELISA is more
sensitive than the IgM-ELISA. Isolation of the parasite by animal inoculation or tissue
culture.
IgG antibodies can cross placenta, so it cannot differentiate congenital infection from
maternal transfer. However, maternal IgG antibodies disappear after 6 months after
birth. So, its persistence beyond 6 months after birth suggests congenital
infection.Ultrasound of fetus at 20–24 weeks of gestation is useful for detecting the
lesions ofcongenital infection.

888. B. Ascaris
Ascaris lumbricoides lives in the lumen of the gut, does not attach to the wall, and
derives its sustenance from ingested food.

889. D. L. donovani
PKDL:
It is a nonulcerative lesion of skin, occurs in 2–50% of patients of VL following the
completion of treatment.
Mainly seen in India and East African
Countries.
It develops as hypopigmented macule
(most common feature) near mouth which later on spreads to face and then to arms
and trunk (extensor surfaces) and finally becomes nodules resembling leprosy.
Ocular lesions like conjunctivitis and uveitis are associated in some
patients.Sometimes, PKDL occurs in subclinical
patients without a history of VL.

890. A. Trichuris trichiura


Trichuris trichiura is oviparous, rest three options are viviparous.
891. A. Vertical Transmission
Mode of transmission of Entamoeba histolytica:
Feco-oral route (most common): By ingestion of contaminated food or water with
mature quadrinucleated cysts.
Sexual contact: Rare, either by anogenital or orogenital contact (especially in
developed countries among homo sexual males).
Vector: Very rarely, flies and cock roaches may mechanically transmit the cysts from
feces, and contaminate food and water.

892. B. Trypanosomiasis
T. cruzi passes its life cycle in two hosts—(1) humans and (2) vector reduviid bugs or
kissing bugs or triatomine bugs.
T. brucei passes its life cycle in two hosts.
1. The vertebrate host is man and other animals
2. Invertebrate host is the tsetse fly (genus
Glossina). Both male and female flies bite man and serve as vectors.
Glossina palpalis group serves as the vector for T. brucei gambiense whereas Glossina
morsitans group is the vector for T. brucei rhodesiense.
Reamaining three options are transmitted by mosquitoes.

893. B. Toxoplasmosis
Sabin-Feldman dye test:
This is the gold standard antibody detection method for toxoplasmosis, usually done in
the reference laboratories.
Other serological tests are evaluated taking this test as standard.

894. A. Enterobius
Enterobius vermicularis is also called as pin worm or threadworm or seat worm.

895. C. Hard ticks


Babesia: The nymph stage of the deer tick Ixodes scapularis is the primary vector
(definitive host) of the parasite. Occasionally it is transmitted by blood transfusion.
Humans act as intermediate host.
Mode of transmission: Man acquires infection by the bite of ticks where the sporozoites
enter through the site of bite and are discharged into circulation.

896. A. Toxoplasmosis
Manifestations of congenital toxoplasmosis include still birth, intracerebral
calcification, psychomotor disturbance, microcephaly and hydrocephaly.

897. C. Cryptosporidium
Cryptosporidium, Cyclospora and Isospora are acid fast parasites that can cause
opportunistic infections (diarrhea) in HIV infected patients.

898. A. Acanthamoeba

899. C. Except in infections with very high parasitaemia, only ring forms of
early trophzoites and the gametocytes are seen in the peripheral blood.
Late trophozoite, early and mature schizonts — are not ordinarily seen in peripheral
blood, except in very severe or pernicious malaria. The presence of P. falciparum
schizonts in peripheral smears indicates a grave prognosis.

900. D. Giardiasis
Often Giardia intestinalis infections are asymptomatic, but in some cases, Giardia may
lead to mucus diarrhea, fat malabsorption (steatorrhea), dull epigastric pain, and
flatulence. The stool contains excess mucus and fat but no blood.
¾ Children may develop chronic diarrhea, malabsorption of fat, vitamin A, protein,
sugars like xylose disaccharides, weight loss, and spruelike syndrome.

901. C. Larval invasion does not occur in beef tape worm infection

902. A. Digestive tract and heart


In chronic phase, T. cruzi produces inflammatory response, cellular destruction, and
fibrosis of muscles and nerves, that control tone of hollow organs like heart,
esophagus, colon, etc. Thus, it can lead to cardiac myopathy and megaesophagus
and megacolon (dilalation of esophagus and colon).

903. B. Direct fluorescent antibody microscopy


Diagnosis of P. jirovecii pneumonia is confirmed by histologlical identification of the
parasite in sputum or brochoalveolar lavage (BAL), or tracheal aspirate. Specimens are
stained by toludine blue, periodic acid-Schiff (PAS), Gomori methanamine silver
(GMS), or immunofluorescence stains, which show characteristic cysts.

904. E. Trichomonas vaginalis


Trichomonas differs from other flagellates, as they exist only in trophozoite stage.
Cystic stage is not seen.

905. B. Infection with fish tape worm


Infection by D. latum causes little damage in the small intestine. In some individuals,
megaloblastic anemia occurs as a result of vitamin B12 deficiency caused by
preferential uptake of the vitamin by the worm.

906. E. Strongyloides
In debilitated individuals and particularly in those with cellular immune defects,
extensive internal reinfection takes place, leading to an enormous number of adult
worms in the intestines and lungs and larvae in various tissues and organs. This is
known as hyperinfection.

907. B. Bancroftian filariasis


Humans are infected with W.bancrofti when the female mosquito (especially
Anopheles and Culex species) deposits infective larvae on the skin while biting.

908. C. Elisa
E Sigmoidoscopy and aspiration of mucosal lesions.
Elisa is used for diagnosis of extraintestinal amoebiasis.
Sigmoidoscopy is used for diagnosis of intestinal amoebiasis.

909. E. Vaginal secretions


Vaginal or urethral discharge is examined microscopically in saline wet mount
preparation for characteristic jerky and twitching motility and shape of T.vaginalis
trophozoites. In males, trophozoites may be found in urine or prostatic secretions.

910. A. Paragonimus westermanii


Paragonimus westermani, the lung fluke, causes paragonimiasis. Humans are infected
by eating raw or undercooked crab meat (or crayfish) containing the encysted larvae
(metacercariae).

911. A. Cerebrospinal fluid


The diagnosis of PAM is based on the finding of motile Naegleria trophozoites in wet
mounts of freshly-obtained CSF.

912. B. Cerebral cysticercosis


Neurocysticerosis (cysticercosis of brain) is the most common and most serious form of
cysticercosis. About 70% of adult-onset epilepsy is due to neurocysticercosis. Other
clinical features of neurocysticercosis are increased intracranial tension,
hydrocephalus, psychiatric disturbances, meningoencephalitis, transient paresis,
behavioural disorders aphasia, and visual disturbances. It is considered as the second
most common cause of intracranial space occupying lesion (ICSOL) after Tuberculosis
in India.

913. B. Beef
Man acquires infection by ingesting raw or undercooked beef containing cysticerci.

914. D. The baby may be LBW

915. D. DNA PCR


Cyst and trophozoites of E. histolytica are indis tinguishable from that of E. dispar or E.
moshkovskii except the presence of RBCs in trophozoites of E. histolytica (which might
not be there after dysentery episode is over). So, the report should always be sent as
“cyst or trophozoite of E. histolytica/ dispar/moshkovskii found in the stool
microscopy.”
Isoenzyme (zymodeme) analysis E. histolytica possesses several isoenzymes like malic
enzyme, hexokinase, isomerase, and phosphoglucomutase.
When these isoenzymes are subjected to electrop horesis, based on the electrop
horetic pattern (zymodeme pattern) and mobility of the isoenzymes, Entamoeba can
be speciated .
It can also be used to differentiate E. histolytica and E. dispar.
However, zymodeme analysis has a number of disadvantages such as: difficulty of
performing the test, timeconsuming and difficulty in preparing the antigens by
culture.
Molecular diagnosis: Nested multiplex polymerase chain reaction (PCR) is available
targeting small subunit ribosomal ribonucleic acid (rRNA) genes that can diff erentiate
E.histolytica, E.dispar and E.moshkovskii with a sensitivity of nearing 90% and
specificity of 90–100 % .
Realtime PCR can be used as alternate to the conventional PCR. It is more sensitive,
quantitates the parasite load and takes less time with less contamination rates.

916. B. Except in infections with very high parasitemia only ring forms of the
early trophozoites and the gametocytes are seen in the peripheral blood
Late trophozoite, early and mature schizonts – are not ordinarily seen in peripheral
blood, except in very severe or pernicious malaria. The presence of P. falciparum
schizonts in peripheral smears indicates a grave prognosis.

917. C. Patients without a spleen


Severe Babesia infections are seen in splenectomised people.

918. B. Macrolides is the treatment of choice


No chemotherapeutic agent effective against cryptosporidium has been identified,
although nitazomamide or parmomycin may be partially effective in few patients with
AIDS. Improvement in immune status with Antiretroviral therapy can lead to
amelioration of cryptosporidiosis. Other treatment methods include supportive therapy
with fluid, electrolytes, and nutrient replacement.

919. A. Bearmann technique


Stool may be concentrated by formol ether concentration method or Baermann’s
funnel gauze method and examined for larvae more efficiently. Baermann’s test
requires a special apparatus and relies on the principal that larva will actively migrate
out of the feces on a wire mesh covered with several layers of gauge.

920. D. Visceral Larva migrans


Visceral Larva Migrans:
This condition is caused by the migration of larvae of nonhuman species of
nematodes that infect by the oral route. The most common cause is the dog ascarid,
Toxocara canis and less often the cat ascarid, T. cati. Visceral larva migrans may also
be caused by Anisakis, which are large ascarid parasites of marine animals and also by
Gnathostoma spinigerum, Angiostrongylus cantonensis.
Human nematodes like A. lumbricoides and S. stercolaris may produce visceral larva
migrans, when they get lost in ectopic sites.
As children are more likely to swallow dirt, this condition is much more frequent in
them.
Fever, hepatomegaly, pneumonitis, hyperglobulinaemia, and pica are the common
findings.
Patients may develop neurological disturbances (neural larva migrans) and
endophthalmitis (ophthalmic larva migrans).
Marked leukocytosis occurs with persistently high eosinophilia.

921. B. transmitted by the bite of fly


Definitive host: Humans are the only definitive host.
Intermediate hosts: Day-biting female black flies of the genus Simulium (black
flies).

922. A. the infestation is acquired from both dogs and cats


Visceral Larva Migrans :This condition is caused by the migration of larvae of
nonhuman species of nematodes that infect by the oral route.
Etiology: The most common cause is the dog ascarid, Toxocara canis and less often the
cat ascarid, T. cati. Visceral larva migrans may also be caused by Anisakis, which are
large ascarid parasites of marine animals and also by Gnathostoma spinigerum,
Angiostrongylus cantonensis.
Human nematodes like A. lumbricoides and S. stercolaris may produce visceral larva
migrans, when they get lost in ectopic sites.
Pathogenesis: When the infective eggs present in the soil contaminated by dog and
cat feces are ingested, the larvae hatch in the small intestine, penetrate the gut wall,
and migrate to the liver. They may remain there or migrate to other organs such as
lungs, brain, or eyes. In humans they do not develop into adults, but induce
granulomatous lesions, which cause local damage.
Clinical Features: Clinical manifestations depend on the sites affected and the degree
and duration of infection. As children are more likely to swallow dirt, this condition is
much more frequent in them. Fever, hepatomegaly, pneumonitis, hyperglobulinaemia,
and pica are the common findings. Patients may develop neurological disturbances
(neural larva migrans) and endophthalmitis (ophthalmic larva migrans). Marked
leukocytosis occurs with persistently high eosinophilia. Diagnosis Serological tests,
such as passive hemagglutination, bentonite flocculation, microprecipitation, and
more specifically, ELISA have been developed for the diagnosis of toxocariasis (visceral
larva migrans).
Treatment: Diethylcarbamazine (DEC), 100 mg TDS for 3 weeks in an adult, kills the
larva and arrest the disease. Thiabendazole may be useful in treatment. Prednisolone
should be administered concurrently either topically or systemically. Prophylaxis:
Deworming of household pets helps in prevention by limiting the contamination of
soil.

923. A. Naegleria fowleri


Patients are mostly previously healthy young adults or children.
Infection comes from water containing the amoebae and usually follows swimming or
diving in ponds.
The amoebae invade the nasal mucosa and pass through the olfactory nerve branches
in the cribriform plate into the meninges, and brain to initiate an acute purulent
meningitis and encephalitis, called as primary amoebic meningo encephalitis
(PAM).
The incubation period varies from 2 days to 2 weeks.
In the incubation period, the patient experiences anosmia.
The disease advances rapidly, causing fever, headache, vomiting, stiff neck, ataxia,
seizure, and coma.
Cranial nerve palsies, especially of the third, fourth, and sixth nerves have also been
documented.
The disease almost always ends fatally within a week (average 5 days).

924. B. Posterior cervical lymphadenopathy


T. brucei gambiense causes African trypanosomiasis (West African sleeping sickness).
The illness is chronic and can persist for many years.
There is an initial period of parasitemia, following which parasite is localized
predominatly in the lymph nodes.
A painless chancre (trypanosomal chancre) appears on skin at the site of bite by
tsetse fly, followed by intermittent fever, chills, rash, anemia, weight loss, and
headache.
Systemic trypanosomiasis without central nervous system involvement is referred to
as stage I disease. In this stage, there is hepatosplenomegaly and
lymphadenopathy, particularly in the posterior cervical region (Winterbottom’s
sign).
925. B. IgG antibodies are diagnostic in congenital toxoplasmosis
Refer to explanation to question 887.

926. A. Zymodeme pattern


E. Haematophagus trophozoite
Based on electrophoretic mobility of 6 isoenzymes (acetylglucosaminidase, aldolase,
hexokinase, NADdiaphorase, peptidase and phosphoglucomutase), E. histolytica
strains can be classified into at least 22 zymodemes. Of these only 9 are invasive and
the rest are noninvasive commensals. The zymodemes show a geographical
distribution. Even in endemic areas, nonpathogenic zyodemes are far more common
than pathogenic ones, which account only about 10% of the total population. It has
been proposed that pathogenic and non-pathogenic strains though morphologically
identical may represent 2 distinct species—the pathogenic strains being E. histolytica,
and the nonpathogenic strains reclassified as E. dispar. Trophozoites of E. dispar
contain bacteria, but no RBCs.

927. A. Common variable immunodeficiency


E. Autosomal recessive agammaglobulinemia
Risk factors for giardiasis: Children are commonly affected. Other high-risk groups are
elderly debilitated persons and patients with cystic fibrosis, poor hygiene, and
immunodeficiency syndromes such as common variable hypoglobulinemia. How ever,
association with acquired immunodeficiency syndrome (AIDS) patient is not been
confirmed yet.

928. B. Diagnosed by trophozoite in CSF


The diagnosis of PAM is based on the finding of motile Naegleria trophozoites in wet
mounts of freshly-obtained CSF.

929. A. Caused by E. histolytica


C. Flask-shaped ulcers are present
d. Cecum is most commonly affected
Amoebic ulcer is the typical lesion seen in intestinal amoebiasis .The ulcers are
multiple and are confi ned to the colon, being most numerous in the caecum and
next in the sigmoidorectal region. The intervening mucous membrane between the
ulcers remains healthy.The typical amoebic ulcer is flask-shaped in cross section,
with mouth and neck being narrow and base large and rounded.

930. B. Giardia lamblia


Often Giardia intestinalis infections are asymptomatic, but in some cases, Giardia may
lead to mucus diarrhea, fat malabsorption (steatorrhea), dull epigastric pain, and
flatulence. The stool contains excess mucus and fat but no blood.
Children may develop chronic diarrhea, malabsorption of fat, vitamin A, protein,
sugars like xylose disaccharides, weight loss, and spruelike syndrome.
Cyst is the infective form of the parasite. The cyst is small and oval, measuring 12 μm
x 8 μm andmis surrounded by a hyaline cyst wall.
Its internal structure includes 2 pairs of nuclei grouped at one end. A young cyst
contains 1 pair of nuclei.
The axostyle lies diagnonally, forming a dividing line within cyst wall.
Remnants of the flagella and the sucking disc may be seen in the young cyst.
931. B. Giardiasis
Giardia intestinalis does not invade the intestinal wall deeper than mucosa or
submucosa, so no blood or neutrophils are present in stools.

932. C. Angiostrongylus cantonensis


Angiostrongylus cantonensis causes chronic meningitis.

933. A. Giardia lamblia


Refer to explanation to question 930.

934. C. Antimonial are useful drugs


d. Diagnosed by blood smear
About 3–10% cases of patients of visceral leishmaniasis in endemic areas develop
PKDL, about an year or 2 after recovery from the systemic illness.
Peripheral blood contains the amastigotes present inside circulating monocytes and
less often in neutrophils, but the numbers are so scanty that a direct blood smear may
not show them.
The standard treatment consists pentavalent antimonial compound, which is the drug
of choice in most of the endemic regions of the world, but there is resistance to
antimony in Bihar in India, where amphotericin-B-deoxycholate or miltefosine is
preferred.

935. A. L. braziliensis
L. brazilensis complex causes both mucocutaneous leishmaniasis and cutaneous
leishmaniasis.
L. braziliensis causes the most severe and destructive form of cutaneous lesion.
It involves the nose, mouth, and larynx.
The patient experiences a nodule at the site of sandfly bite with symptoms consistent
with oriental sore.
Subsequent mucocutaneous involvement leads to nodules inside the nose, perforation
of the nasal septum, and enlargement of the nose and lips (espundia).
If the larynx is involved, the voice changes as well.
Ulcerated lesions may lead to scarring and tissue destruction that can be disfiguring.
The disease occurs predominantly in Bolivia, Brazil, and Peru.

936. B. Trophozoites and cysts are seen in man


Giardia is the only protozoan parasite found in the lumen of the human small
intestine (duodenum and jejunum).
Trophozoites are pearshaped, bilaterally symmetrical with 2 nuclei, 4 pairs of fl
agella, and a ventral concave sucking disc. They exhibit motility resembling a ‘falling
leaf’.
Ellipsoid cysts contain 4 nuclei with remnants of flagella.
Infective form: Ellipsoid cysts.
Clinical features: Mostly asymptomatic but in some cases may cause diarrhea, dull
epigastric pain, and malabsorption. Stool contains excess mucus but no blood. It does
not invade deeper into the intestine.
Diagnosis: By microscopic demonstration of trophozoites or cysts in stool, enterotest,
and serodiagnosis by ELISA (ProSpec T/Giardia antigen assay).
Treatment: Metronidazole and tinidazole are the drugs of choice.
937. D. Arthralgia, sore throat and abdominal pain are the most common
manifestation
Cats shed millions of oocysts per day in feces for about 2 weeks during the primary
infection. The freshly passed oocyst is not infectious.
They undergo sporulation in the soil with formation of 2 sporocysts, each containing 4
sporozoites. The sporulated oocyst is infective.
Most human infections are asymptomatic.
Transplantation from infected donors is one of the modes of infection.
Congenital toxoplasmosis results when T. gondii is transmitted transplacentally from
mother to fetus. The risk of fetal infection rises with progress of gestation; from 25%,
when the mother acquires primary infection in first trimester to 65% in the third
trimester. Conversely, the severity of fetal damage is highest, when infection is
transmitted in early pregnancy.

938. B. Accolé forms are seen


Ring form of Plasmodium falciparum : The early ring form in the erythrocyte is very
delicate and tiny, measuring only a sixth(1/6) of the red cell diameter.
Rings are often seen attached along the margin of the red cell, the so-called form
appliqué or accole.
Binucleate rings (double chromatin) are common resembling stereo headphones in
appearance. Several rings may be seen within a single erythrocyte.
In course of time, the rings become larger, about a third of the size of the red cell and
may have 1 or 2 grains of pigment in its cytoplasm.

939. B. IgM antibodies against Toxoplasma in the fetus


Congenital toxoplasmosis can be diagnosed by detecting:
Toxoplasma antigens in amniotic fluid.
Toxoplasma specific genes by PCR.
IgM antibodies in fetal blood by ELISA or IFA.
IgA antibodies in fetal blood. Double-sandwich IgA-ELISA is more sensitive than the
IgM-ELISA.
Isolation of the parasite by animal inoculation or tissue culture.
IgG antibodies can cross placenta, so it cannot differentiate congenital infection from
maternal transfer. However, maternal IgG antibodies disappear after 6 months after
birth. So, its persistence beyond 6 months after birth suggests congenital infection.
Ultrasound of fetus at 20–24 weeks of gestation is useful for detecting the lesions of
congenital infection.

940. B. Plasmodium vivax


The appearance of malarial pigment varies, mostly it is brown black in color and
numerous (except in P. vivax it is yellowish brown in color and in P. falciparum, it is few
in number).
Number of merozoites per mature schizont varies:
P. vivax—12–24 number (average 16)
P. falciparum—18–24 number (average 20)
P. malariae – 6 – 12 number (average 8)
P. ovale – 8 – 12 number (average 8)
Forms seen in peripheral blood smear examination: Trophozoites (early and late),
gametocytes and schizonts.

941. B. Due to ingestion of oocyst from cat’s faeces


C. Spiramycin given in pregnancy
Humans acquire infection after:
Eating uncooked or undercooked infected meat, particularly lamb and pork containing
tissue cysts.
Ingestion of mature oocysts through food, water, or fingers contaminated with cat
faeces directly or indirectly.
Intrauterine infection from mother to foetus (congenital toxoplasmosis).
Blood transfusion or transplantation from infected donors.
Spipramycin is the DOC for toxoplasmosis in pregnancy.

942. A. Gametocyte
Gametocyte is the infective stage for mosquito and sporozoite is the infective stage for
man.

943. B. It is a protozoa
E. It is associated with diarrhoea in HIV patients
Microsporidia are classified under Phylum Microspora.
They are minute, intracellular, Gram-positive, spore-forming protozoa.
They can cause wide range of illness in patients with HIV and other
immunocompromised diseases.
In patients with AIDS Enterocytozoon bieneusi and Encephalitozoon intestinalis lead to
protracted and debilitating diarrhoea in 10–40% of cases.

944. A. Protozoa
Malaria is caused by Plasmodium, a protozoan parasite.

945. A. Trypanosome
In chronic phase, T. cruzi produces inflammatory response, cellular destruction, and
fibrosis of muscles and nerves, that control tone of hollow organs like heart,
oesophagus, colon, etc.
Thus, it can lead to cardiac myopathy and megaoesophagus and megacolon
(dilatation of oesophagus and colon).

946. A. Chorioretinitis
The manifestations of congenital toxoplasmosis include chorioretinitis, cerebral
calcifications, convulsions, strabismus, deafness, blindness, mental retardation,
microcephaly, and hydrocephalus.
A few children are born with manifestations of acute toxoplasmosis, which may
include fever, jaundice, petechial rashes, microphthalmia, cataract, glaucoma,
chorioretinitis, lymphadenopathy, hepatosplenomegaly, myocarditis, cerebral
calcifications, and chorioretinitis.

947. A. Only cyst is infective


Man acquires infection by ingestion of quadrinucleated cysts in contaminated water
and food.
948. A. Clonorchis sinensis
The adult worm may cause obstruction and blockage of the common bile duct leading
to cholangitis.
Patients in the early stage have fever, epigastric pain, diarrhoea, and tender
hepatomegaly.
This is followed by biliary colic, jaundice, and progressive liver enlargement.
Many infections are asymptomatic.
Chronic infection may result in calculus formation.
A few cases go on to biliary cirrhosis and portal hypertension.
Some patients with chronic clonorchiasis tend to become biliary carriers of typhoid
bacilli.
Chronic infection has also been linked with cholangiocarcinoma.

949. A. Clonorchis sinensis


Please refer to explanation to question 948.

950. B. Cysticercosis
Neurocysticerosis (cysticercosis of brain) is the most common and most serious form of
cysticercosis.
About 70% of adult-onset epilepsy is due to neurocysticercosis. Other clinical features
of neuro cysticercosis are increased intracranial tension, hydrocephalus, psychiatric
disturbances, meningoencephalitis, transient paresis, behavioural disorders aphasia,
and visual disturbances.
It is considered as the second most common cause of intracranial spaceoccupying
lesion (ICSOL) after Tuberculosis in India.
It is the most common parasitic infection of brain in India.

951. A. Brain parenchyma


Parenchymal > intrventricular > subarachnoid space > spinal > orbital.

952. C Loa loa


953. B. Onchocerca volvulus
The subcutaneous nodule or onchocercoma is a circumscribed, fi rm, non-tender
tumor, formed as a result of fibroblastic reaction around the worms.
Nodules vary in size from a few mm to about 10 cm.
They tend to occur over anatomical sites where the bones are superficial, such as the
scalp, scapulae, ribs, elbows, iliac crest, sacrum, and knees.
The nodules are painless and cause no trouble except for their unsightly appearance.

954. B. Flat or fleshy leaf-like worms


Nematodes are cylindrical, or filariform in shape, bilaterally symmetrical with a
secondary triradiate symmetry at the anterior end.
The adults vary greatly in size, from about a millimetre (Strongyloides stercoralis) to a
meter (Dracunculis medinensis) in length. Male is generally smaller than female and
its posterior end is curved or coiled ventrally.
Their body is covered with a tough outer cuticle, which may be smooth, striated,
bossed, or spiny.
The middle layer is hypodermis and the inner layer is the somatic muscular layer.
They move by sinuous flexion of the body.
The body cavity is a pseudocele, in which all the viscera are suspended.
The digestive system is complete, consisting of a anteriorly placed mouth leading to
the esophagus, which characteristically varies in shape and structure in different
groups. The intestine is lined with a single layer of columnar cells and leads to the
rectum, opening
through the anus. In the male, the rectum and the ejaculatory duct open into the
cloaca.
Nematodes have simple excretory and nervous systems.
The nematodes are diecious i.e. the sexes are separate.
The male reproductive system consists of a single delicate tubule differentiated into
testis, vas deferens, seminal vesicle, and ejaculatory duct, which opens into the
cloaca. It also includes copulatory structures such as spicules or bursa or both.
The female reproductive system consists of the ovary, oviduct, seminal receptacle,
uterus, and vagina.
Female nematodes may produce eggs (oviparous) or larvae (viviparous). Some lay
eggs containing larvae, which immediately hatch out (ovoviviparous).

955. D. Trichinella spiralis

956. A. T. solium
C. Trichinella spiralis

Parasites with source of infection

Pork Fish

Taenia solium Diphyllobothrium latum


Trichinella spiralis Clonorchis sinensis
Sarcocysts suihominis Metagonimus yokogawai
Heterophyes spp.
Beef Gnathostoma spp.

Taenia saginata
Sarcocystis hominis
Toxoplasma gondii

957. B. Enterobius
Parasites causing autoinfection:
Strongyloides stercoralis
H.nana
Enterobius vermicularis
Taenia solium

958. A. Clonorchis sinensis


Eggs of Clonorchis sinensis are broadly ovoid, 30 μm by 15 μm with a yellowish brown
(bile-stained) shell.
It has an operculum at one pole and a small hook-like spine at the other .
Eggs do not float in saturated solution of common salt.
The eggs passed in feces contain the ciliated miracidia.

959. B. Paragonimus westermanii,


d. E. granulosus,
E. E. multilocularis

960. A. Meta-zoonosis
It is a zoonosis that requires both a vertebrate and an invertebrate host for completion
of its life cycle.
961. A. S. hematobium
Humans are infected when the free-swimming, fork-tailed cercariae penetrate the
skin .They differentiate to larvae (schistosomula), enter the blood, and are carried via
the veins into the arterial circulation. Those that enter the superior mesenteric artery
pass into the portal circulation and reach the liver, where they mature into adult
flukes. S. mansoni and S. japonicum adults migrate against the portal flow to reside in
the mesenteric venules. S. haematobium adults reach the bladder veins through the
venous plexus between the rectum and the bladder.

962. B. Tail tip free from nuclei


Please refer to explanation to question 952.

963. B. Ascaris
Ascaris is intra-luminal worm, it does not attach to gut cells , hence does not cause
malabsorption.

964. D. Chemically defined media


Viruses are obligate intracellular infectious particles which do not grow in artificial
media.

965. A. Absence of nuclear membrane

Characteristic Prokaryotic Bacterial Eukaryotic Human


Cells Cells

DNA within a nuclear membrane No Yes

Mitotic division No Yes

DNA associated with histones No Yes

Chromosome number One More than one

Membrane-bound organelles, such as mitochondria and No Yes


lysosomes

Size of ribosome 70S 80S

Cell wall containing peptidoglycan Yes No


Questions
1. Which rule states that “A person is not
criminally responsible, if at the time of
committing the act, as a result of mental disease
or defect lacked substantial capacity either to
appreciate the wrongfulness of his conduct or to
conform his conduct to the requirements of the
law”?
A. McNaughten’s rule
B. Durham’s rule
C. Curren’s rule
D. American law Institute test

2. The enquiry into the circumstances of death is


called
A. Inquest report
B. Corpus delecti
C. Verdict
D. Open verdict

3. As per MCI rules, if any request is made for


medical records, the documents shall be issued
within the period of
A. 24 hours
B. 48 hours
C. 72 hours
D. 96 hours
4. One of the following may not amount to
professional misconduct
A. Dichotomy
B. Covering
C. Advertisement
D. Adultery

5. Fingerprints are discovered by


A. Sir William J Herschel
B. Sir Francis Galton
C. Sir Edward Henry
D. Sir Edmund Locard

6. The growth of the beard is


A. 3 mm/week
B. 0.4 mm/day
C. 0.1 mm/day
D. 0.2 mm/day

7. As per the THOA 1994, The burden of proof to


diagnose brainstem death rests on
A. Doctor in charge of the patient
B. Doctor in charge of the hospital
C. Independent specialist
D. Specialist with Neurosciences

8. Subpoena is a kind of
A. Designation
B. Decomposed body tissue
C. Court tribunal
D. Document

9. True about female pseudohermaphroditism


A. Nuclear sex is XY
B. Testicular feminization
C. Presence of testis
D. Male form of external genitalia

10. If a lady dies of unnatural and suspicious death


within 7 years after her marriage, the inquest
will be done by
A. Deputy Superintendent of police
B. Forensic medical expert
C. Coroner
D. Sub-divisional Magistrate

11. Lividity may be seen at the agonal period in


case of
A. Influenza
B. Cholera
C. Starvation
D. Malignant disease

12. A patient suffering from a contagious disease


should be treated
A. When he gives implied consent
B. When he gives expressed consent
C. When he gives written consent
D. Even if he does not consent

13. Angurugu syndrome is seen in case of


poisoning by
A. Copper
B. Manganese
C. Cobalt
D. Nickel

14. Loco Parentis is about


A. Inheritance
B. Consent
C. Paternity dispute
D. Maintenance to the child

1. 2. 3. 4. c 5. 6. 7. 8. 9. 10.
d a c a b d d d d

11. 12. 13. 14.


b d b b

15. Samples for virological examination are


preserved in
A. Liquid Nitrogen
B. Toluene
C. Thymol
D. Gycerol

16. After post-mortem, the body is handed over to


A. Coroner
B. Chief magistrate
C. The authority who has conducted inquest in that
particular case
D. Nearby Police station

17. Who can consider as an expert witness


A. Medical man
B. Firearm expert
C. Chemical examiner
D. All of the above

18. Forensic Medicine deals with


A. Medical aspects of Law
B. Legal aspects of Medicine
C. Doctor - Patient, Doctor & State relationship
D. Doctor - Doctor relationship

19. The type of inquest never followed in India is


A. Police inquest
B. Magistrate’s inquest
C. Coroner’s inquest
D. Medical Examiner’s system

20. The cause of death in hanging may be


A. Asphyxia
B. Cerebral anoxia
C. Fracture / dislocation of cervical vertebrae
D. Any of the above

21. Ewing postulates concerns with which of the


following:
A. Growth at the site following trauma
B. Growth after a neurological injury
C. Age related changes in the teeth
D. Old seminal stains
22. In India, magistrate inquest is done in the
following cases EXCEPT
A. Exhumation
B. Dowry deaths
C. Murder
D. Death of a person in police custody

23. Trial court for the offences committed by the


children is
A. High court
B. Sessions court
C. Magistrate court
D. Juvenile court

24. The high court hass power to stay the


execution of a pregnant woman according to
which section of criminal procedure code?
A. 416 Cr.P.C
B. 417 Cr.P.C
C. 418 Cr.P.C
D. 419 Cr.P.C

25. A doctor is
A. A common witness
B. An expert witness
C. An ordinary witness
D. Both common and expert witness

26. A police can arrest a person without warrant in


A. Cognizable Offence
B. Non cognizable Offence
C. Summons Case
D. Warrant Case

27. Maximum punishment for issuing false


certificate
A. 6 months
B. 1 year
C. 3 years
D. 7 years

28. The term ‘Professional Death Sentence’


denotes
A. Rigorous imprisonment
B. Death punishment
C. Severe warning to the RMP by the Medical Council
D. Permanent erasure of the name of the RMP from the
Medical Register

29. Medical qualifications awarded by institutions


outside India and recognized by MCI are
registered in
A. Second schedule of Indian Medical Council Act 1956
B. First schedule of Indian Medical Council Act 1956
C. Part II of third schedule of Indian Medical Council Act
1956
D. Part I of third schedule of Indian Medical Council Act
1956

15. 16. 17. 18. 19. 20. 21. 22. 23. 24.
d c d a d d a c d a

25. 26. 27. 28. 29.


d a d d a
30. Repeated advertisement in newspaper by a
medical practitioner is an example of
A. Criminal negligence
B. Privileged communication
C. Infamous conduct
D. Civil negligence

31. The sentence awarded by a court may be


enhanced by the
A. Magistrate court
B. High court
C. Higher court
D. All of the above

32. According to Criminal law amendment act


2013, the minimum age of a girl to give valid
consent for sexual intercourse is
A. 16 years
B. 18 years
C. 20 years
D. 15 years

33. Dying declaration can be received by


A. Medical officer
B. Lawyer
C. Police officer
D. Any of the above

34. Declaration of Oslo is related to


A. Torture
B. Human Experiment
C. Therapeutic abortion
D. Hunger strike

35. Criminal negligence is punishable under section


304A of IPC for a maximum imprisonment of
A. 1 year
B. 2 year
C. 3 year
D. 5 year

36. Finger printing finder involves recording prints


of 8 fingers. Which finger pair is excluded?
A. Ring finger
B. Thumb
C. Little finger
D. Middle finger

37. Disciplinary control over Registered Medical


Practitioners is under
A. Director of Medical and Health Service
B. Health secretary of State Government
C. State Medical Council
D. Indian Medical Association

38. After death all of following show rise in CSF


except
A. Lactic acid
B. Amino acid
C. Urea
D. Uric acid
39. Criminal negligence is punishable under
A. 337 IPC
B. 338 IPC
C. 304A IPC
D. All of the above

40. Law of Tort deals with


A. Traffic accidents
B. Civil wrong
C. Criminal wrong
D. Sexual assault

41. Prominent early signs of cyanide poisoning


include all EXCEPT
A. Cyanosis
B. Hyperpnoea
C. Cherry red blood
D. Convulsions

42. St. Anthony’s fire refers to poisoning by?


A. Ergot alkaloids
B. Aflatoxin
C. Calatropis Indica
D. Spanish fly

43. Which of the following is not a principle of


medical ethics
A. Consent
B. Non-maleficence
C. Beneficence
D. Justice

44. All the following are functions of State Medical


Council EXCEPT
A. Maintaining Medical register
B. Issuing Warning notice
C. Disciplinary control on Doctors
D. Recognition of Medical Qualifications

45. The modernized versions of the Hippocratic


Oath comprise the
A. Declaration of Sydney
B. Declaration of Geneva
C. Declaration of Venice
D. Declaration of Istanbul

46. In case of Hypothermia, shivering stops at


A. 35°C
B. 32°C
C. 30°C
D. 27°C
30. 31. 32. 33. 34. 35. 36. 37. 38. 39.
c c b d c b c c c d

40. 41. 42. 43. 44. 45. 46.


b a a a d b b

47. ‘Non voluntary Euthanasia’ means


A. At the will of the person
B. Person incapable of making their will
C. Against the will of the person
D. Against the will of the parents

48. A 19-year-old patient presented to the


emergency with altered sensorium. A non-
contrast CT revealed hydrocephalus, requiring
urgent neurosurgical intervention. The patient
had nobody accompanying him or any identity
proof. Under which of these sections of Indian
Penal code can the neurosurgeon perform the
procedure without obtaining an informed consent
from a guardian?
A. Section 87
B. Section 89
C. Section 90
D. Section 92

49. As per COPRA 1986, the maximum period to file


a case after the knowledge of damage is
A. 90 Days
B. 150 Days
C. 1 Year
D. 2 Years

50. A farmer ingested some unknown poisonous


seeds and had pain and vomiting. Soon he
developed paralysis of his lower limbs which
ascends till it affected the respiratory muscles
and he died within two days. Identify the likely
cause:
A. Strychnine nux-vomica
B. Conium masculatum
C. Semicarpus anacardium
D. Abrus precatorius
51. Hyperventilation before drowning is
A. Can cause delayed death
B. Can cause secondary drowning
C. Can cause immediate death
D. Does not alter the process of drowning

52. ‘Baby-farmer’ refers to


A. Child labourer
B. Child of a farmer
C. Farmer with small land
D. Children starved by their parents

53. Chilotic line is used to determine


A. Age
B. Sex
C. Race
D. Stature

54. Jamaica ginger (Jake) paralysis is due to


contamination of Ethanol with
A. Organochlorine
B. Organophosphorus
C. Carbamates
D. Lead

55. Stature is measured by one of the following


A. Trotter and Glesser’s Formula
B. Haase’s rule
C. Gustafson’s Method
D. Rule of 12
56. Best method of determine age up to 14 years is
A. Dentition
B. Ossification centres
C. Head circumference
D. Anthropometry

57. “Triage” is associated with


A. Poisoning
B. Firearm injury
C. Mass Casualties
D. Drowning

58. A lady is brought from village, unconscious,


about 12 hours after ingesting some kind of
unknown poison. Her heart rate is 103/min, blood
pressure in 90/50 mm Hg and respiratory rate is
19/min. Her breath smells like kerosene. All of
the following should be done in her management,
except:
A. Gastric lavage should be done
B. Atropine should be administered till signs of
recovery
C. Vasopressors should be administered intravenously
D. Immediate airway management

59. Fusion of xiphoid process occurs at


A. 20 years
B. 30 years
C. 40 years
D. 70 years

60. Second common pattern of finger printing is


A. Loops
B. Arches
C. Whorls
D. Composite
47. 48. 49. 50. 51. 52. 53. 54. 55. 56.
b d d b c d b b a a

57. 58. 59. 60.


c a c c

61. Species identification is done by


A. Precipitin test
B. Neutron activation analysis
C. Spectroscopy
D. Benzidine test

62. Atavism is resemblance of features to


A. Mother
B. Father
C. Neighbour
D. Grandfather

63. One of the following hospital is not exempted


from COPRA.
A. Government hospitals where services are rendered
free of charge
B. Private hospitals where services are rendered free of
charge to everybody
C. Hospitals with only a nominal registration fee
D. Hospitals where some patients are charged and
some are not charged
64. The father of Modern Forensic Medicine is
A. Bernard Spilsbury
B. Mathieu Orfila
C. Paracelsus
D. Paolo Zacchia

65. Stain used for detection of ‘ Y ’ chromosomes


is
A. Fluorsecent in situ hybridization
B. Quinacrine dihydrochloride
C. Feulgen reaction
D. Acriflavin Schiff reagent

66. Which presentation is most indicative of


possible child abuse?
A. Toddler with bruise and laceration on forehead, plus
older bruise over shin, mother says he ran into a
chair
B. 3 month old infant with bruise on right forearm and
left buttock; parent states infant rolled off changing
table
C. Mild scald burn with splash marks on trunk of 6
month infant; parent states she accidently spilled
coffee
D. Bite mark in shoulder of 9 months with distance
between canines about 2cm; father believes 2 year
old brother has bitten recently.

67. If father’s blood group is A and mother’s blood


group is B, the child’s blood group will be
A. A only
B. B only
C. AB only
D. A, AB, B or O

68. ‘Dry Submarine’ in custodial torture injuries


refer to -
A. Beating on soles of feet
B. Tying of a plastic bag over the head
C. Immersing head in contaminated water
D. Inserting heated metal skewer into anus

69. Edmund Locard is known for his


A. Formula for estimation of stature
B. Theory of exchange
C. System of personal identification using the body
measurement
D. Finger print study

70. In case of bodies recovered from water,


hypostasis is seen in all the parts EXCEPT
A. Head
B. Neck
C. Front of upper chest
D. Trunk

71. In acute starvation feeling of intense hunger


lasts
A. 12-24 hours
B. 24-36 hours
C. 30-48 hours
D. 48-60 hours
72. True statement is
A. Minimum imprisonment for forced male homosexual
sodomy is 7 years
B. Minimum imprisonment for forced heterosexual
sodomy is 10 years
C. Minimum imprisonment for bestiality is 7 years
D. All of the above

73. Lower end of the femur can be help to


determine the
A. Weight
B. Stature
C. Age
D. Height

74. Intraocular tension becomes nil after how many


hours after death
A. 1
B. 2
C. 3
D. 4
61. 62. 63. 64. 65. 66. 67. 68. 69. 70.
a d d a b b d b b d

71. 72. 73. 74.


c b c b

75. Ossification centre appearing at birth is


A. Upper end humerus
B. Scaphoid
C. Lower end of femur
D. Upper end of tibia

76. Which area of skull is prominent in female


A. Parietal
B. Occipital
C. Temporal
D. Supraorbital ridge

77. Closure of coronal sutures starts at age of


A. 10-15 years
B. 20-25 years
C. 30-35 years
D. 40-45 years

78. Oximes are contraindicated in?


A. Baygon
B. Tik-20
C. Malathion
D. Parathion

79. An arrested person who is accused of rape is


brought to you at CHC for examination who is not
willing to consent for the same. What should be
done in this case?
A. Sent him back
B. Refer to District hospital
C. Use reasonable amount of force and examine
D. Ask the Investigating officer to get a court order

80. Pre-auricular sulcus helps in detection of


A. Sex
B. Age
C. Height
D. Race

81. In maceration which of the following statement


is not true?
A. Foetus is flaccid
B. A sterile condition
C. Emits sweetish but disagreeable smell
D. Greenish discoloration

82. In blunt abdominal trauma, commonest site of


GI rupture is
A. Stomach
B. Duodenum
C. Jejunum
D. Transverse colon

83. Prinsloo-Gorden artefact in cases of neck


compression is seen
A. Beneath the esophagus
B. Beneath the Hyoid bone
C. Beneath the Ligature mark
D. Beneath the vertebra

84. Best specimen of bone for sex determination is


A. Pelvis
B. Femur
C. Mandible
D. Skull
85. A person fabricates a contusion with marking
nut juice with an intension of false allegation on
one of his enemies. He is punishable under IPC
section
A. 191
B. 192
C. 193
D. 197

86. The fingerprint pattern may be impaired


permanently in cases of
A. Scabies
B. Leprosy
C. Eczema
D. Scalds

87. Corporo-basal index is useful for determination


of
A. Stature
B. Sex
C. Race
D. Age

88. Identical twins will not have same


A. Finger print pattern
B. DNA fingerprint pattern
C. HLA system
D. Blood group

89. ‘Crural index’ helps to differentiate


A Age B Sex
C Race D Stature

90. The most reliable bones for the purpose of


medullary index are the following except
A. Radius
B. Sternum
C. Humerus
D. Tibia

91. Minimum air resulting in fatal air embolism is


A. 50 ml
B. 100 ml
C. 150 ml
D. 200 ml

92. Re-examination of a prosecution witness will be


done by
A. Public Prosecutor
B. Defence Lawyer
C. Presiding Judge
D. Accused person
75. 76. 77. 78. 79. 80. 81. 82. 83. 84.
d a c a c a d c a a

85. 86. 87. 88. 89. 90. 91. 92.


c b b a c b b a

93. The parameters used in Bertillon system are all


EXCEPT
A. Descriptive data
B. Body measurements
C. Body marks
D. Finger prints

94. The parameter used to enlarge the photo to


life size for superimposition in Ruxton’s case is
A. Button
B. Ear stud
C. Tiara
D. Spectacle

95. A young adult came with casualty with history


of some substance abuse. His pulse was 110
beats per minute, respiratory rate 30/min, blood
gas analysis revealed metabolic acidosis,
hypocalcemia and electrolyte imbalance. Blood
urea – 100 mg/dl, creatinine – 3.4 mg/dl. Urine
analysis showed calcium oxalate crystals. He
improved on gastric lavage, ethanol, sodium
bicarbonate, calcium gluconate and 4-methyl
pyrazole. What is the substance he consumed?
A. Ethylene glycol
B. Methyl alcohol
C. Formaldehyde
D. Paraldehyde

96. What is the wrong statement about abrasion?


A. Involves superficial layers of skin alone
B. Caused by contact with a rough surface
C. Pointed object cannot cause it
D. Direction of application of force can be found out

97. Damage to whole skin is Dupuytren’s


A. 2nd degree burns
B. 3rd degree burns
C. 4th degree burns
D. 5th degree burns

98. Number of Ossification centers at birth are


A. 806
B. 450
C. 406
D. 206

99. Professional secrecy is


A. Legal responsibility
B. Ethical responsibility
C. Social responsibility
D. Implied term of contract between doctor and patient

100. Height of a child at the age of 1 year is about


A. 50 cm
B. 60 cm
C. 68 cm
D. 100 cm

101. A person keeps a man under his power and


threatens him to assault or cause harm without
any intention to kill comes under which section of
IPC?
A. 319
B. 320
C. 324
D. 351
102. The commonest heart defect in Turner’s
syndrome is
A. Bicuspid aortic valve
B. Coarctation of aorta
C. Septal defects
D. Aortic dissection

103. Early tooth eruption is seen in


A. Hypopituitarism
B. Syphilis
C. Rickets
D. Down’s syndrome

104. A man fell from 35 feet height. Eye witnesses


say that he fell on his feet. Which of the
following injuries will confirm this fact?
A. Ring fracture + lumbar spine injury
B. Pond fracture + cervical spine injury
C. Comminuted fracture + cervical spine injury
D. Depressed fracture + cervical spine injury

105. The method of age estimation by height and


weight of teeth is
A. Mile’s method
B. Stack’s method
C. Boyde’s method
D. Lamendil’s method

106. Up to how long after death can fluorescent


dyes demonstrate Y chromosomes in the dental
pulp in badly decomposed bodies?
A. 3 months
B. 6 months
C. 9 months
D. 12 months
93. 94. 95. 96. 97. 98. 99. 100. 101. 102.
d c a c c b d c d a

103. 104. 105. 106.


b a b d

107. Minimum quantity of blood required to be


preserved for chemical analysis is
A. 10 ml
B. 2 ml
C. 100 ml
D. 50 ml

108. Sexual intercourse by a husband with his wife


during separation without wife’s consent is
punishable under Section
A. 376A of IPC
B. 376B of IPC
C. 376C of IPC
D. 376D of IPC

109. The weight required to constrict trachea in


case of hanging is
A. 2 kg
B. 4-5 kg
C. 15 kg
D. 20 kg
110. The depth may be more than the length of the
blade in case of stab wounds over the
A. Head
B. Chest
C. Abdomen
D. Limbs

111. An adult came to casualty with complaints of


rapid heart rate. On examination everything else
was normal except for episodic tachycardia and
occasional extra systole and amblyopia. Which of
the following is the cause of it?
A. Nicotine
B. Cannabis
C. Atropine
D. Cocaine

112. When an autopsy surgeon wants to send the


maggots for examination, it should be preserved
in
A. Sodium chloride
B. Rectified spirit
C. Acetic alcohol
D. Dimethyl sulphoxide

113. The ideal method to recognize latent tattoo in


living is
A. Ultra violet light
B. Skin biopsy
C. Regional lymph node biopsy
D. Laser
114. Certain obligations on the part of doctor who
under takes post-mortem examination are
following except
A. The examination should be meticulous and
complete
B. Routinely record all positive findings and
important negative ones
C. He must keep the police informed about the
findings
D. He must preserve viscera and send for toxicology
examination in case of poisoning

115. Disclosure of the identity of the victim of rape


is punishable under
A. Sec. 327(2) CrPC
B. Sec. 357(C) CrPC
C. Sec. 228(A) IPC
D. Sec. 166(B) IPC

116. First post-mortem was conducted in India is in


A. Kolkata
B. Mumbai
C. Chennai
D. Delhi

117. A 14-years-old rape victim was brought to the


hospital with 22 weeks of pregnancy. All of the
following are correct statements regarding the
case, except:
A. Vaginal swab need not be taken
B. The fetus can be aborted after her consent
C. Examination can be done by a male doctor with a
female attendant
D. Urine pregnancy test is not necessary

118. The best preservative for blood for culture is


A. EDTA
B. Heparin
C. Sodium flouride
D. Sodium polyanethole sulfonate

119. Least dispersion of pellets are seen in


A. Full choke
B. ¾ choke
C. ½ choke
D. ¼ choke

120. Regarding rugoscopy, all are true except


A. Palatoprints do not change during growth
B. Secondary rugae are 3-5mm
C. Palatal rugae are used as a method of
identification
D. Primary rugae are less than 3mm

107. 108. 109. 110. 111. 112. 113. 114. 115. 116.
a d c c a c a c c c

117. 118. 119. 120.


b d a d

121. Pyrogallol test helps to differentiate


A. Antemortem from postmortem burns
B. Air embolism from putrefactive gas
C. Joule from filigree burns
D. Postmortem from antemortem clots

122. Which of the following is NOT a feature of


post-mortem clot?
A. Yellow chicken fat appearance
B. Adherence to the wall
C. Red currant jelly appearance
D. Friable

123. The amount of soil to be collected at the time


of exhumation is about
A. 100 grams
B. 250 grams
C. 400 grams
D. 500 grams

124. Sexual arousal from enemas is known as


A. Agalmatophilia
B. Coprolagnia
C. Mysophilia
D. Klismaphilia

125. Atropine is not an antidote in:


A. Tik 20
B. Endrin
C. Baygon
D. Parathion

126. The mode of death in case of CO poisoning is


A. Anoxic Anoxia
B. Anaemic Anoxia
C. Stagnant Anoxia
D. Histotoxic Anoxia

127. Post-mortem caloricity may be seen in all of


the following causes of death except
A. Septicaemia
B. Barbiturates poisoning
C. Strychnine poisoning
D. Tetanus

128. Consumption of unripe Carambola can cause


poisoning by
A. Salicylic acid
B. Formic acid
C. Oxalic acid
D. Boric acid

129. True about suspended animation


A. Common phenomenon in yogic person
B. Can be voluntary
C. Person can be revived
D. All of the above

130. Postmortem lividity is unlikely to develop in a


case of
A. Drowning in a fast flowing river
B. Drowning in well
C. Drowning in chlorinated swimming pool.
D. Postmortem submersion
131. In a fetus nails reach the tip of the fingers at
A. 5th month
B. 6th month
C. 7th month
D. 8th month

132. Domain of ballistics concerned with forensic


pathologist is
A. Interior
B. Exterior
C. Intermediate
D. Terminal

133. Putrefaction is delayed in case of


A. Infants
B. Arsenic poisoning
C. Old age
D. Athlets

134. Putrefactive gases are all except


A. CO2
B. NO2
C. H2S
D. NH3

135. Pugilistic attitude is due to


A. Cadaveric spasm
B. Rigor mortis
C. Coagulation of protein
D. All of the above
136. One of the following can be given Only by oral
route
A. DMSA
B. DMPS
C. EDTA
D. BAL

137. Immediate sign of death is


A. Fall in body temperature
B. Changes in eye
C. Changes in skin
D. Cessation of circulation and respiration
121. 122. 123. 124. 125. 126. 127. 128. 129. 130.
b b d d b b b c d a

131. 132. 133. 134. 135. 136. 137.


d d b b c a d

138. ‘Zasko’ phenomenon’ after death is in relation


to
A. Mechanical stimulation
B. Chemical stimulation
C. Electrical stimulation
D. Thermal stimulation

139. Most common organ damaged in air blast


injury is
A. Ear drum
B. Lung
C. GIT
D. Bones
140. For transplantation skin can be removed from
dead up to
A. 6 hours
B. 24 hours
C. 48 hours
D. 72 hours

141. Following is not a stage of muscle changes


after death
A. Primary flaccidity
B. Secondary flaccidity
C. Tertiary flaccidity
D. Rigidity

142. The onset of rigor mortis is early in case of


A. Pneumonia
B. Poliomyelitis
C. Strychnine poisoning
D. Arsenic poisoning

143. Deep inspiration above the water level may


cause
A. Late floatation of the body
B. Late floatation of the body only in sea water
C. No effect on floatation of body
D. Early floatation of the body

144. Pseudocyesis is usually observed in


A. Childless wife nearing menopause
B. Males who are impotent to a particular woman
C. Unmarried woman in whom contraceptives has
failed
D. Young married woman with illicit sexual relation
with a colleague

145. ‘Casper’s dictum’ helps in the determination


of
A. Growth of foetus
B. Hair growth
C. Rate of cooling
D. Rate of putrefaction

146. Sub-dural haemorrhage is common in case of


A. Elderly people
B. Child abuse
C. Alcoholics
D. All of the above

147. Age of the fetus of weight 1000g, foot length


8cm, thick nails, opened eyelids, pupillary
membrane disappeared, testis in external
inguinal ring, crown rump length is 23cm is
A. 5 months
B. 6 months
C. 7 months
D. 8 months

148. In water, external putrefactive changes are


first seen in
A. Face
B. Chest
C. Upper limbs
D. Abdomen

149. Antemortem abrasions can be confused with


A. Ant bite marks
B. Eczema
C. Joule burn
D. Chemical burn

150. On 4th day the colour of a bruise is


A. Red
B. Green
C. Blue
D. Brown

151. Deep blue hypostasis is seen in case of death


by poisoning due to
A. Phosphorus
B. Nitrites
C. Aniline
D. Hydrogen sulphide

152. Which fracture may be seen in only young


adults due to a blunt force impact
A. Diastatic
B. Gutter
C. Pond
D. Perforating

153. ‘Lochia’ is the discharge seen


A. After menstruation
B. During pregnancy
C. After delivery
D. During labour

138. 139. 140. 141. 142. 143. 144. 145. 146. 147.
a a b c c c a d d c

148. 149. 150. 151. 152. 153.


a a d c a c

154. Age of a 15 year old female is best


determined by the radiograph of
A. Upper end of radius and ulna
B. Lower end of radius and ulna
C. Upper end of humerus
D. Lower end of femur

155. Professional negligence is failure on the part


of the doctor for
A. Wrong diagnosis
B. Wrong treatment
C. Absence of reasonable care
D. Refusal to accept a patient

156. Scab or crust of abrasion appears brownish


black in
A. 12-24 hours
B. 2-3 days
C. 4-7 days
D. After one week

157. All of the following documents are accepted in


the court without cross-examination EXCEPT
A. Chemical Examiner’s report
B. Dying Declaration
C. Fingerprint expert’s report
D. Medico-legal reports

158. One of the following is the first enzyme to be


released at the site of wound
A. Aminopeptidase
B. Acid phoshatase
C. ATP-ase
D. Alkaline phosphatase

159. Fracture at C6 - C7 vertebrae is called


A. Hangman’s fracture
B. Undertaker’s fracture
C. Jefferson’s fracture
D. Hinge fracture

160. All the following give rise to dimness of


cornea before death EXCEPT
A. Uraemia
B. Narcotic Poisoning
C. Wasting Disease
D. Carbon Monoxide poisoning

161. The callus is visible on X-ray at


A. 1 week
B. 2 weeks
C. 3 weeks
D. 6-8 weeks
162. The fertilisation of two ova released at two
different menstrual cycles refers to
A. Superfecundation
B. Superfoetation
C. Foetuspapyraceus
D. Fecundation ab extra

163. Which of the following is most characteristic


of concussion
A. Pupillary constriction
B. Nausea, vomiting
C. Transient amnesia and unconsciousness
D. Rapid, shallow breathing

164. In head injury bleeding from middle ear is


suggestive of fracture of
A. Anterior cranial fossa
B. Middle cranial fossa
C. Posterior cranial fossa
D. Vault of the skull

165. ‘Sudden death’ means, the maximum duration


between onset of illness and death is less than
A. 3 hours
B. 6 hours
C. 12 hours
D. 24 hours

166. The cause for ring fracture is


A. Fall from height on feet
B. Fall of heavy object on the head
C. Blow on the chin
D. All of the above

167. Retraction ball necrosis is associated with


A. Cerebral Concussion
B. Commotio Cerebri
C. Stunning
D. Diffuse axonal injury

168. Commotio cordis is seen in


A. Infants
B. Adolescents
C. Adults
D. Old age

169. Telefona refers to?


A. Legs & thighs are tied tightly with bamboo sticks
B. Forced standing
C. Beating on both ears
D. Rolling a wooden/iron rod up & down the thighs

154. 155. 156. 157. 158. 159. 160. 161. 162. 163.
a c c d c b d c b c

164. 165. 166. 167. 168. 169.


b d d d d c

170. In case of Decomposition of bone, the last


amino acid to disappear is
A. Proline
B. Hydroxy-Proline
C. Cysteine
D. Glycine

171. Use of Wadding in a smooth bore gun


produces all except
A. Optimum pressure
B. Sealing the air
C. Causes fatal injuries
D. Helps in lubrication

172. Which of the following tests is not used to


detect the presence of primer metal components
on the hands of an individual who fired the
firearm?
A. Atomic Absorption Spectroscopy
B. Dermal nitrate test
C. Harrison and Gilroy test
D. Neutron activation analysis

173. Femur is ----------- % of the individual’ height


A. 20
B. 22
C. 27
D. 35

174. Acid phosphatase activity of human semen in


Bodansky units per ml is
A. 240 - 260
B. 300 - 320
C. 340 - 360
D. 380 – 400

175. Pari passu means


A. Guilty mind
B. Necessary changes having been made
C. Equal in all respects
D. Beyond the power

176. Motile spermatozoa found on a wet mount of


vaginal secretions are indicative of intercourse
within the past
A. 6 hours
B. 12 hours
C. 24 hours
D. 48 hours

177. One of the following is generally not a


constituent of embalming fluid
A. Ethanol
B. Methanol
C. Formalin
D. Phenol

178. One of the following is not a feature of ‘Triad


of asphyxia’
A. Cyanosis
B. Increased fluidity of blood
C. Petechial hemorrhages
D. Visceral congestion & edema

179. Blackening and tattooing of skin and clothing


can be best demonstrated by
A. Infrared photography
B. Luminol spray
C. Comparison microscope
D. Ultraviolet light

180. Open verdict in coroner’s inquest means


A. Verdict made known to the accused
B. Judgement made public
C. Case is kept pending
D. Case decided on previous judgement

181. ”Knock out drops” to render a person helpless


in robbery is
A. Dhatura
B. Chloral Hydrate
C. Methaquolone
D. Barbiturates

182. Mental Health Act 1987 deals with such


individuals, which are known as:
A. Lunatics
B. Mad
C. Mentally ill
D. Insane

183. Subpubic angle in females is


A. <900
B. 900-1000
C. 1000-1100
D. >1100

184. All are components of black powder EXCEPT


A. Charcoal
B. Lead peroxide
C. Potassium nitrate
D. Sulphur

185. MTP act was last modified in the year


A. 2000
B. 2003
C. 2005
D. 2007

186. Which of the following bullets leaves a visible


mark, so that a person can see it.
A. Tracer Bullet
B. Explosive bullet
C. Incendiary Bullet
D. Gyrojet
170. 171. 172. 173. 174.c175. 176. 177. 178. 179.
d c b c c b a b a

180. 181. 182. 183. 184. 185. 186.


c b c b b b a

187. Smokeless gun powder is composed of


A. HCN
B. KMnO4
C. Sulphur
D. Nitrocellulose

188. Bullet scrapings around the wound is known


as
A. Smudging
B. Peppering
C. Fouling
D. Stippling

189. Judicial execution is carried out in


A. United Kingdom
B. Italy
C. USA
D. All of the above

190. In frost bite, skin becomes hard black in about


A. 7 Days
B. 3 days
C. 4-6 weeks
D. 2 weeks

191. After birth the umbilical cord falls in


A. 36-48 hours
B. 2-3 days
C. 5-6 days
D. 10-12 days

192. A doctor should not take fees for issuing the


following
A. Certificate issued to a government employee
B. Certificate issued to a poor patient
C. Certificate issued to a psychiatric patient
D. Death certificate

193. A quack injects penicillin after giving test


dose. But the patient dies of anaphylactic shock.
The quack can defend on the grounds of
A. Therapeutic misadventure
B. Medical maloccurence
C. Novus actus intervenience
D. None of the above

194. Lightening flash can cause injury by all of the


following, Except
A. Expanded and repelled air
B. Compressed air pushed in front of the current
C. Direct effect of electric current
D. Super-heated air

195. Yellow Poppy gives rise to


A. Epidemic Dropsy
B. Spastic Quadriplegia
C. Flashback Phenomenon
D. Bad Trip

196. Pink urine is seen in case of poisoning by


A. Nitric acid
B. Mercury
C. Carbolic acid
D. KMNO4

197. The type of the burn that is relatively painless


is
A. Epidermal
B. Dermo-epidermal
C. Deep
D. None of the above

198. District Consumer Court can pass maximum


compensation of
A. 5 Lakhs
B. 20 Lakhs
C. 50 Lakhs
D. 1 Crore

199. The causative electrode of electric injury can


be identified by
A. Acro reaction
B. Current pearl
C. Bone pearls
D. Wax drippings

200. Doctrine of ‘Res Ipsa Loquitur’ can be applied


in case of
A. Prescription of overdose
B. Complications in an operation
C. Failure to achieve total cure
D. None of the above

201. In both water and food are completely


withheld to a person, death usually occurs in
A. 5 to 7 days
B. 10 to 12 days
C. 1 to 2 days
D. 2 to 4 days

202. Gettler’s test is used in


A. Hanging
B. Strangulation
C. Drowning
D. Poisoning

203. Death in case of judicial hanging is because of


A. Occlusion of airway
B. Vasovagal shock
C. Coma
D. Fracture dislocation of upper cervical vertebra

187. 188. 189. 190. 191. 192. 193. 194. 195. 196.
d c c d c d d b a b

197. 198. 199. 200. 201. 202. 203.


c b a a b c d

204. The test to know the presence of air in the


stomach and intestine in autopsy on new born is
known as
A. Hydrostatic test
B. Raygat’s test
C. Breslau’s first life test
D. Breslau’s second life test

205. Rayalaseema phenomenon refers to


A. Back spatter
B. Insertion of bullet into stab wound
C. Surgical intervention of gunshot wound
D. Bullet retained in the body since a long time
206. The test in which weight of lungs is compared
with body weight is
A. Ploucquet’s Test
B. Fodere’s Test
C. Wredin’s Test
D. Static Test

207. The number of members in National Consumer


Disputes Redressal Forum is
A. 3
B. 5
C. 7
D. 9

208. Karyotyping of foetus may be done from all of


the following except
A. Monocyte
B. Lymphocyte
C. Fibroblast
D. Amniocyte

209. False Negative Hydrostatic test in a live born


foetus may be seen in
A. Emphysema
B. Putrefaction
C. Atelectasis
D. Artificial Respiration

210. What is meant by ‘Suppositions child’?


A. Child fictitiously claimed by a woman
B. Child born out of wedlock
C. Child is born after artificial insemination
homologous
D. Second born of a twin pregnancy

211. In prenatal diagnostic technique Act 1994


which one of the following is not a ground for
carrying out prenatal test?
A. History of two or more spontaneous abortion or
fetal loss
B. Pregnant woman above 35 years of age
C. History of exposure to potentially teratogenic
drugs
D. When fetal heart rate is 160 per min at fifth and
120 per min at ninth month

212. The definite signs of pregnancy include all of


the following EXCEPT
A. Hearing the fetal heart sounds
B. Palpating the fetal parts
C. Positive HCG test
D. X-ray diagnosis

213. The term ‘Affiliation cases’ refer to


A. Legitimacy
B. Posthumous child
C. An orphan child
D. Suits for adoption

214. False accusation by adopted children of being


mistreated by their adoptive mothers is known as
A. Battered baby syndrome
B. Shaken baby syndrome
C. Caffey’s syndrome
D. Cinderella syndrome

215. For diagnosing brain stem death, which of the


following tests must be done?
A. Apnoea test
B. Brain stem reflexes tests
C. Tests for irremediable structural brain damage
D. All of the above

216. Emmenagogues are the drugs which


A. Cause uterine contraction
B. Increase menstrual blood flow
C. Increase sweating
D. Cause vomiting

217. Sweating is not seen in


A. Heat stroke
B. Heat cramps
C. Heat fatigue
D. Heat syncope
204. 205. 206. 207. 208. 209. 210. 211. 212. 213.
d b a b a c a d c d

214. 215. 216. 217.


d d b a

218. One of following type of burn injury is not


seen in lightening
A. Spark burn
B. Filigree burn
C. Surface burn
D. Linear burn

219. Which is NOT a feature seen on the baby born


of precipitate labour?
A. Intracranial haemorrhages
B. Pond fracture
C. Prominent caput succedaneum
D. Torn umbilical cord

220. Berlin’s edema is associated with


A. Coup injury
B. Contrecoup injury
C. Cerebral concussion
D. Retinal injury

221. In which of the following shot gun the inner


diameter of the barrel is widest?
A. 10 bore
B. 12 bore
C. 14 bore
D. 16 bore

222. The percentage of fetal haemoglobin at birth


is
A. 94%
B. 80%
C. 7-8%
D. 0%
223. What is the most common cause of death in
secondary drowning?
A. Electrolyte imbalance
B. Aspiration pneumonia
C. Cardiac arrhythmia
D. Exhaustion

224. Mental ability to make valid will is


A. Curren’s rule is basis for this
B. Vicarious liability
C. McNaughton’s rule
D. Testamentary capacity

225. Phadebas test is used to detect


A. Milk
B. Colostrum
C. Saliva
D. Urine

226. Explosive bullet have detonator cap on


A. Proximal aspect
B. Distal aspect
C. Centre
D. Both proximal and distal

227. Which of the following is used for


narcoanalysis
A. Atropine
B. Phenobarbitone
C. Pethidine
D. Scopolamine

228. The term MERMER is related to


A. Brain fingerprinting
B. Narco analysis
C. DNA fingerprinting
D. Polygraph test

229. Simon’s hemorrhages may be seen in case of


A. Hanging
B. Strangulation
C. Suffocation
D. Drowning

230. Killing of a child by its own parents is called


as
A. Feticide
B. Neonaticide
C. Filicide
D. Infanticide

231. ABO antigen are found in all except


A. Saliva
B. Sweat
C. CSF
D. Gastric juice

232. What is the investigation of choice for blood


grouping in old blood stain on cloth
A. Benzidine test
B. Acid dilution test
C. Haemin crystal test
D. Precipitin test

233. Which of the following tests is used to detect


semen
A. Marsh test
B. Phenolphthalein test
C. Paraffin test
D. Barberio’s test

234. Cornpicker’s pupil is seen in


A. Cantharide
B. Scorpion
C. Datura
D. Snake bite

235. According to the transplantation of Human


organs act 1994, what is the punishable for a
doctor found guilty?
A. Less than 1 year
B. Less than 2 year
C. 2 to 5 years
D. More than 5 years
218. 219. 220. 221. 222. 223. 224. 225. 226. 227.
a c d a b a d c b d

228. 229. 230. 231. 232. 233. 234. 235.


a a c c b d c d

236. Gastric lavage is contraindicated in the


following
A. Barbiturate poisoning
B. Kerosene poisoning
C. Paracetamol poisoning
D. None of the above

237. Reversal of hot/cold temperature perception


is seen in
A. Ciguatera fish poisoning
B. Gymnothorax poisoning
C. Scombroid food poisoning
D. Sweet clover disease

238. There has been an outbreak of a new strain of


virus in your state and Government has made it
compulsory that every single person be
vaccinated at the earliest so as to check the
spread of the disease; but a colleague of yours
who is also supposed to be vaccinated, refuses to
give consent and says that he will sue you if you
vaccinate him without his consent. What is the
best option in this scenario?
A. Don’t vaccinate him as you will be penalized
B. Vaccinate him
C. Obtain a court order and vaccinate him
D. None of the above

239. Cause of death in crush syndrome is


A. Necrosis of muscles
B. Fat embolism
C. Septicaemia
D. Acute renal failure
240. Which of the following is known as Love drug?
A. Methylenedioxymethamphetamine
B. Methylenedioxyamphetamine
C. Methylenedioxyethamphetamine
D. Gama Hydroxy Butyric Acid

241. Almond odour is seen in which poisoning


A. Chloral hydrate
B. Cocaine
C. Cannabis
D. Cyanide

242. In acute arsenic poisoning, greatest amount is


found in
A. Liver
B. Muscle
C. Brain
D. Keratin tissue

243. Blast wind causes


A. Primary blast injury
B. Secondary blast injury
C. Tertiary blast injury
D. Quaternary blast injury

244. For embalming, chemicals are usually injected


into
A. Carotid artery
B. Jugular vein
C. Femoral artery
D. Femoral vein

245. Poisoning with all the following causes


constriction of the pupils except
A. Dhatura
B. Morphine
C. OP compounds
D. Pontine haemorrhage

246. Most common site of damage to stomach by


corrosives in food filled stomach is
A. Lesser curvature
B. Lesser curvature and pylorus
C. Greater curvature
D. Body

247. A child of 11 years is brought by her


neighbour with an alleged history of sexual
assault by her father. Mother is not available to
take consent. You should take consent from
A. The hospital superintendent
B. The police officer
C. The member of child welfare committee
D. The mother after her arrival

248. At the following blood carbon monoxide


concentration the clinic picture will simulate
alcoholic intoxication
A. 10-20%
B. 30-40%
C. 40-50%
D. 50-60%
236. 237. 238. 239. 240. 241. 242. 243. 244. 245.
b a c d b d a c a a

246. 247. 248.


b c c

249. One of the following substance produce


injuries which simulate contusion
A. Abrus precatorius
B. Capsicum annum
C. Semicarpus anacardium
D. Ricinus communis

250. In unexposed persons, the level of


coproporphyrin in urine is
A. Less than 150 microgram / litre
B. Less than 100 microgram / litre
C. More than 500 microgram / litre
D. None of the above

251. Ganja is obtained from


A. Fresh leaves
B. Dried leaves
C. Roots
D. Flowering tops

252. Which of the following facts regarding the


stature of the body is correct?
A. Immediately after death, the body may shorten by
about 2-3 cm
B. Immediately after death, the body may lengthen
by about 2-3 cm
C. Change in stature after death occurs only when
the deceased was engaged in struggle at the time
of death
D. There is absolutely no change in stature after
death

253. Aconite poisoning causes all except


A. Tingling and numbness
B. Hyper salivation
C. Hypothermia
D. Increased BP

254. Common salt is used as preservative in case


of poisoning by
A. Mineral acids
B. Alkalis
C. Aconite
D. Phenol

255. The characteristic features of poisonous


snakes are all except
A. Nocturnal habit
B. Belly scales are incomplete
C. Long fangs
D. Compressed tail

256. Speed ball refers to


A. Heroin + Cocaine
B. Heroin + Strychnine
C. Cocaine + Baking soda
D. Alcohol + Chloral hydrate

257. Widmark’s formula helps in the


measurements of blood level of
A. Barbiturates
B. Cocaine
C. Alcohol
D. Benzodiazepines

258. Stain used for tattooing are all except


A. Indian ink
B. Vermillion
C. Osmium dye
D. Prussian blue

259. Which of the following statements about


blood grouping is not correct?
A. It can be sued to resolve confusion of identity in
alleged exchange of babies in maternity unit.
B. It is the method to conclusively fix the paternity.
C. It can assist in matching fragmented human
remains in mass disasters.
D. It can help to show whether the blood stain on the
weapon belongs to the suspect or victim.

260. Scorpion sting may present with following


except?
A. Salivation and lacrimation
B. Arrhythmias
C. Hypothermia
D. Hypertension and Pulmonary edema

261. In starvation, the gall bladder may be


A. Distended
B. Atrophied
C. Normal
D. Show stones

262. Which of the following is the most reliable


method of estimating blood alcohol level
A. Breath alcohol analyser
B. Cavett’s test
C. Thin layer chromatography
D. Gas chromatography

263. Metal fume fever is common in chronic


poisoning with
A. Lead
B. Mercury
C. Thallium
D. Zinc
249. 250. 251. 252. 253. 254. 255. 256. 257. 258.
c a d b d d b a c c

259. 260. 261. 262. 263.


b c a d d

264. Callus is not formed in case of fracture of


A. Skull
B. Mandible
C. Clavicle
D. Phalanx

265. In chronic alcoholism which of the following is


seen
A. Delirium tremens
B. Wernike’s syndrome
C. Korsakoff’s psychosis
D. All of the above

266. Arsenophagists are


A. Person using arsenic as abortion risk
B. Criminal using arsenic for homicidal purpose
C. Persons using it as cattle poison
D. Person who can tolerate high doses of arsenic after
taking arsenic in low doses at frequent intervals

267. Specific antidote for nicotine is


A. Mecamylamine
B. Atropine
C. Dicobalamine
D. None

268. One of the following is Non-Poisonous


A. Elapidae
B. Viperidae
C. Crotalidae
D. Colubridae

269. Mercurialism is caused by


A. Inhalation of vapours of mercury
B. I.V. Injection
C. I.M. Injection
D. Oral ingestion

270. All are seen in Cadmium poisoning EXCEPT


A. Proteinuria
B. Painful bone lesions
C. Golden yellow staining of teeth
D. Cardiomyopathy

271. All of the following are phenanthrene group


EXCEPT
A. Morphine
B. Narcotine
C. Thebaine
D. Codeine

272. One of the following statement with regard to


Ethylene glycol is WRONG
A. Used as an anti freeze agent
B. There is no antidote
C. It give rise to tetany
D. It is a nephrotoxic poison

273. Exposure and abandonment of child under


twelve years by his parents may lead to their
prosecution under section
A. 315 IPC
B. 316 IPC
C. 317 IPC
D. 318 IPC
274. Mild poisoning by organophosphate occurs
when the ChE activity is
A. 50-75% of normal
B. 20-50% of normal
C. 10-20% of normal
D. <10% of normal

275. Following are the causes of death in patient


with burns except
A. Shock
B. Suffocation
C. Sepsis
D. Air embolism

276. One of the following statement is INCORRECT


with regard to Carbon Monoxide poisoning
A. It blocks Cytochrome - A3 Oxidase & P-450
B. Death is due to chemical anoxia
C. Blood gets clotted after death
D. It causes necrosis & cavitation of basal ganglia

277. One of the following is not present in Sewer


gas
A. CO
B. CO2
C. H2S
D. Methane

278. The ideal chelating agent in case of Glucose-


6PD deficiencies is
A. DMSA
B. DMPS
C. EDTA
D. BAL

279. Toxalbumin is present in the oil extracted


from
A. Castor
B. Croton
C. Abrus
D. Jetropa

280. 4th infralabial is largest in


A. Common cobra
B. Common krait
C. Russell’s viper
D. Saw scaled viper

264. 265. 266. 267. 268. 269. 270. 271.b272. 273.


a d d a d b d b c

274. 275. 276. 277. 278. 279. 280.


b d c a a b b

281. Mental retardation is defined when I.Q. is less


than
A. 90
B. 80
C. 70
D. 65

282. Match the following


1. Seiji Ogawa a. PCR
2. Kary Mullis b. DNA Fingerprinting
3. Lawrence Farwell c. Brain mapping
4. Alec Jeffreys d. Brain Fingerprinting
A. 1-c; 2-a; 3-d; 4-b
B. 1-d; 2-a; 3-c; 4-b
C. 1-c; 2-b; 3-d; 4-a
D. 1-b; 2-d; 3-a; 4-c

283. Which of the following is the earliest


secondary sexual character to develop in males?
A. Increase in the size of penis
B. Increase in the size of testis
C. Appearance of pubic hairs
D. Both A and B

284. According to PCPNDT ACT, what is the


maximum punishment for a doctor Found guilty?
A. Less than 1 year
B. Less than 2 year
C. 3-5 years
D. >5 years

285. In hanging, the Ligature Mark is above the


level of Thyroid Cartilage in
A. 80% of cases
B. 75% of cases
C. 70% of cases
D. 65% of cases

286. Ectopic bruise is common in the region of


A. Foot
B. Eye
C. Chest
D. Hand

287. Excessive sexual interest in females is termed


as
A. Satyriasis
B. Nymphomania
C. Hybristophilia
D. Formicophilia

288. The best method to avoid aspiration of fluids


during gastric lavage in a comatose patient is by:
A. Putting the head of the patient at a lower level
than his feet
B. Putting the patient in the left lateral position
C. Introduction of a cuffed endotracheal tube before
lavage
D. Continuous suction of the fluid from the trachea

289. The only test to detect both choline and


spermine in seminal stain is
A. Florence test
B. Barberio’s test
C. Acid phosphatase test
D. Thin layer chromatography

290. In case of embalming, best method of


injection is
A. Continuous injection and drainage
B. Continuous injection with disrupted drainage
C. Alternate injection and drainage
D. Discontinuous injection and drainage

291. Medical termination of pregnancy can be done


as per MTP Act, maximum up to a gestational age
of
A. 12 weeks
B. 16 weeks
C. 20 weeks
D. 24 weeks

292. Average blood alcohol required for alcohol


gaze nystagmus is
A. 30 mg%
B. 50 mg%
C. 80 mg%
D. 150 mg%

293. The preferred solution for stomach wash in


cyanide poisoning is
A. Potassium permanganate
B. Sodium thiosulphate
C. Calcium gluconate
D. Aluminium hydroxide

294. Cause of death in ‘Japanese Detergent Suicide


Technique’ is by inhalation of
A. CO
B. CO2
C. Methane
D. H2S
281. 282. 283. 284. 285. 286. 287. 288. 289. 290.
c a b c a b b c d d

291. 292. 293. 294.


c c b d

295. Paradoxical undressing is seen in case of


A. Hyperpyrexia
B. Hypothermia
C. Sexual perversion
D. None

296. Minimum age of a person who can be


punished under Indian railway act is
A. 5 years
B. 7 years
C. 12 years
D. No such limit

297. Live birth according to Indian law is when:


A. Any part of living child comes out
B. Child is completely out of mothers private parts
C. Child is completely born and cries
D. Child is completely born and shows signs of
respiration

298. Best defense for Adultery is


A. Accused is unmarried
B. Accused in Impotent
C. Woman only seduced
D. Age of accused is less than 12 years
299. In a female, consumption of same amount of
alcohol, the blood alcohol level in comparison to
male of same body weight will be
A. 10% less
B. 10% more
C. 25% more
D. Equal

300. After birth, the ductus venous closes by


A. 2 days
B. 3 days
C. 4 days
D. 10 days

295. b 296. d 297. a 298. b 299. c 300. c


Explanations
1. D. American law Institute test
McNaughton’s rule: To establish a defence on the
ground of insanity, it must be clearly shown that, at the
time of committing the act, the accused was labouring
under such defect of reason, from disease of mind, as
not to know the nature and quality of the act he was
doing, or if he did know it, that he did not know that
what he was doing was wrong.
Durham’s rule: An accused is not criminally
responsible if his unlawful act is the product of mental
Disease or mental Defect.
Currens rule: An accused is not criminally responsible,
if at the time of committing the act, the defendant, as a
result of mental disease or defect, lacks substantial
Capacity to Conform his Conduct to the requirements
of law.
American law institute (ALI) test: A person is not
criminally responsible, if at the time of committing the
act, as a result of mental disease or defect lacked
substantial capacity either to appreciate the
wrongfulness of his conduct or to conform his conduct
to the requirements of the law.
Irrestible impulse test (new Hampshire
doctrine): An accused is not criminally responsible,
even if knows the nature and quality of his act and
knows that it is wrong, if he is incapable of restraining
himself from committing the act, because the free
agency of his will has been destroyed by mental
disease.

2. A. Inquest report
Inquest means enquiry into the causes of death which
is apparently not due to natural causes.
Corpus delecti means “Body of offence” or “essence of
crime”; means, the facts (elements) of any criminal
offence e.g. murder.
Verdict is a decision on an issue of fact in a civil or
criminal case or an inquest.
Open verdict means an announcement of the
commission of crime without information regarding
the accused.

3. C. Inquest report
As per Director General of Health Services (DGHS),
Ministry of health and F.W., Govt. of India,
Medical Records of indoor patients is stored in
digitised form – indefinitely.
Medical Records in hard copy should be stored as
follows:
a. OPD Records – 3 years
b. In-patient’s Medical Records – 3 years
c. Medico-legal Registers and Case Sheets – 10
years or till the disposal of ongoing cases in any
of the courts related to these records
As per Indian Medical Council (Professional conduct,
etiquette and ethics/regulations, 2002)
Indoor patients records should be kept for – 3 years.
A doctor should provide the medical records within 72
hours when the patient or his/her authorised
representative makes a request for it.

4. C. Advertisement
Professional Misconduct is any conduct of the doctor,
which is considered as disgraceful or dishonourable
judged by medical practitioners of good repute and
competence.
Dichotomy or fee-splitting, i.e. receiving or giving
commission to professional colleague or a
manufacturer or trader in drugs or appliances, or to a
chemist, dentist etc.
Covering, i.e. having relations with unqualified
persons, which enable them to practice midwife, or
issuing certificates which enable such practices to
occur etc.
Adultery: abuse of professional position by committing
adultery or improper conduct with a patient or by
maintaining an improper association with a patient.
Advertisement: Can be given on occasions such as,
when starting a new clinic, when shifting the clinic
etc., but repeated advertisement in a newspaper by a
medical practitioner is an example of ethical
negligence

5. A. Sir William J Herschel


Fingerprints are discovered by Sir William J Herschel in
1877.
It is systematized by Sir Francis Galton in1892.
It is further improved by Sir Edward Henry.
The First finger print bureau was established at
Writer’s Building, at Kolkata, India.

6. B. 0.4 mm/day
Growth of Scalp hair: 3mm/day
Growth of Beard: 0.4mm/day, and
Growth of Nail: 0.1mm/day

7. D. Specialist with Neurosciences


Brain-stem death should be certified by a Board of
medical experts consisting of the following namely:
(i) The registered medical practitioner (RMP) in
charge of the hospital in which brain-stem death
has occurred;
(ii) An independent RMP, being a specialist, to be
nominated by RMP specified in cause (i)
(iii) A neurologist or a neurosurgeon to be
nominated by the RMP specified in clause (i)
(iv) The RMP treating the person whose brain-stem
death has occurred.
The burden of proof to diagnose brainstem death rests
on the specialist with Neurosciences.

8. D. Document
Sub = Under, Poena = Penalty.
Subpoena means summons.
It is a written document issued by the court and
served to the witness. under a penalty in all cases to
attend the court of law for giving evidence on a
particular day and time.
It is not a designation or decomposed body tissue.
Courts are established to maintain law and order in
the respective jurisdiction. On the contrary, tribunals
are a part of judicial set up that deals with direct
taxes, labour, cooperatives, claims for accidents, etc.

9. D. Male form of external genitalia


There are two types of pseudo-hermaphroditism:
a) Male pseudo-hermaphroditism
Nuclear sex is XY
There is presence of testis in the abdomen or inguinal
canal, but the external genitalia is of female form.
Characterized by female external genitalia, primary
amenorrhea, normal sized breasts, scanty or absent
axillary or pubic hair, due to testicular feminization.
5-a reductase deficiency occurs.
b) Female pseudo-hermaphroditism
Nuclear sex is XX
External genitalia are male form due to adrenal
hyperplasia.
21 hydroxylase deficiency is most common

10. D. Sub-divisional Magistrate


Sec. 304B IPC - Dowry death - Where the death of a
woman is caused by any burns or bodily injury or
occurs otherwise than under normal circumstances
within seven years of her marriage and it is shown
that soon before her death she was subjected to
cruelty or harassment by her husband or any relative
of her husband for, or in connection with, any demand
for dowry, such death shall be called “dowry death”,
and such husband or relative shall be deemed to have
caused her death.
And in case of dowry death, the magistrate has to hold
an Inquest and not by the police.
The Deputy Superintendent of police or Forensic
expert are not authorised to hold an inquest.
Coroner inquest is not held in India.

11. B. Cholera
Hypostatic congestion resembling post-mortem
hypostasis may be seen even before death in case of a
person slowly dying with circulatory failure, e.g.,
cholera, typhus, TB, uraemia, morphine and
barbiturate poisoning, CCF, deep coma and asphyxia
deaths.

12. D. Even if he does not consent


Protection of life and personal liberty- No person shall
be deprived of his life or personal liberty except
according to procedure established by law.
To protect the other people getting infections from the
contagious patient, the Article 21 of Constitution of
India allows treatment of the contagious patient
without consent

13. B. Manganese
Some manganese miners on an Australian island have
been reported to be affected with a peculiar
neurological disease characterised by upper motor
neuron, cerebellar signs, and oculomotor symptoms
known as Angurugu syndrome.
Cobalt is added in the beer to give more foam, and
excessive drinking of beer may cause
Cardiomyopathy, known as Beer Drinker’s Syndrome.
Nickel is known to give rise to lung cancer.

14. B. Consent
Doctrine of loco parentis: In an emergency situation
involving children, when parents are not available or
legal guardians are not available, consent from the
person-in-charge of that child can be taken.

15. D. Gycerol
For enzyme studies, blood is preserved in liquid
nitrogen in Thermos flask,
Toluene is the best preservative for urine. The other
preservatives for urine are hydrochloric acid, thymol
and Sodium fluoride (NaF is used in case of poisoning
by Alcohol, CO, Cocaine and Cyanide).
For virological examination, a piece of appropriate
tissue is collected under sterile conditions and the
sample is freezed or preserved in 80% glycerol in
buffered saline.

16. C. The authority who has conducted inquest in


that particular case
The doctor receives the dead body to perform post-
mortem examination from the investigating officer.
After post-mortem examination the doctor should
hand over the dead body to the investigating officer
only, that means the authority who has conducted
inquest in that particular case.

17. D. All of the above


Common / Ordinary Witness is a person who gives
evidence about the facts observed by himself.
Expert Witness is one who is qualified or experienced
in a scientific or technical subject and is capable of
giving opinion from the facts observed by himself or
others e.g. doctor, fingerprint expert, handwriting
expert, firearm expert etc.

18. A. Medical aspects of Law


Forensic medicine deals with application of medical
knowledge into the processes of Law, both Civil and
Criminal.
In short, it deals with Medical aspects of Law.
Medical jurisprudence deals with application of
knowledge of Law in the practice of Medicine.
In short, it deals with Legal aspects of Medicine.
Medical ethics deals with the moral principles, which
should guide members of the medical profession in
their dealings with each other, their patients and the
state.
That means it includes
Doctor – Doctor relationship
Doctor – Patient relationship
Doctor – State relationship
Medical etiquette deals with the conventional laws of
courtesy to be observed between members of the
medical profession (Doctor – Doctor Relationship).

19. D. Medical Examiner’s system


In India, there are only two types of Inquest:
1. Police Inquest and
2. Magistrate’s inquest
Previously there was Coroner’s Inquest which is
abolished.
Medical Examiner system is never held in India.

20. d. Any of the above


The cause of death in case of death due to hanging
may be due to:
Asphyxia
Venous congestion
Combined asphyxia and venous congestion
Cerebral ischemia
Reflex vagal inhibition
Fracture or dislocation of cervical vertebrae.
Out of all, the commonest cause of death is combined
asphyxia and venous congestion.

21. A. Growth at the site following trauma


A causal relationship between trauma and tumour is
extremely difficult to establish. However, one may
come across such cases and occasional claims be
litigated. The prerequisites to the assumption that any
given case of tumour was caused by trauma rests in
the fulfilment of Ewing’s postulates (James
Ewing, a US pathologist, 1866–1943):
Evidence of previous integrity of the injured
part,
The injury must have been of sufficient severity
to produce tissue disruption,
The tumour must have originated in the part of
the body that sustained the injury,
It must be of the histologic type that could
originate from the cells that have been
disrupted by the trauma and
There must be proof of reasonable time interval
between injury and appearance of the tumour.

22. C. Murder
In India, Magistrate’s inquest is done in cases of –
Death in police custody,
Death due to police firing,
Death while under police interrogation,
Death in prison,
Death of a mentally ill person in a psychiatric
hospital,
Dowry death,
Exhumation etc.
In case of murder, the inquest is done by Police.

23. D. Juvenile court


A juvenile offender is an offender who is less than 18
years of age (Both males and females).
In case of juvenile offender the trial will not take place
in a regular court, but in juvenile justice board.

24. A. 416 Cr.P.C


Sec. 416 CrPC - If a woman sentenced to death is
found to be pregnant, the High Court shall order the
execution of the sentence to be postponed, and may,
if it thinks fit, commute the sentence to imprisonment
for life.
Amendment: In section 416 of the principal Act, the
words “order the execution of the sentence to be
postponed, and may, if it thinks fit” shall be omitted.
That read as – “If a woman sentenced to death is
found to be pregnant, the High Court shall commute
the sentence to imprisonment for life”.

25. D. Both common and expert witness


Common witness is also known as ordinary witness.
A doctor is both common as well as expert witness. For
example, when a doctor is mentioning about the
dimensions and situation of a wound, he acts as
common witness and when he mentions about the
type of the wound, causative object etc., he acts as
expert witness.
The Indian Medical Council Act 1956 in section 15 (2)
(C) states that, no person other than a medical
practitioner enrolled on a state medical register shall
be entitled to give evidence at any inquest or in any
court of law as an expert under sec. 45 I.E.A. on any
matter relating to medicine.

26. A. Cognizable Offence


Cognizable offence is an offense in which a police
officer can arrest a person without warrant from the
Magistrate. Examples – Rape, murder, dowry death,
ragging etc.
Non-cognizable offence is an offense in which a police
officer cannot arrest a person without warrant from
the Magistrate.
Summons cases are offences for which imprisonment
is less than 2 years.
Warrant cases are offences for which imprisonment is
more than 2 years.

27. D. 7 years
Sec. 197 IPC - Issuing or signing false certificate.—
Whoever issues or signs any certificate required by law
to be given or signed, or relating to any fact of which
such certificate is by law admissible in evidence,
knowing or believing that such certificate is false in
any material point, shall be punished in the same
manner as if he gave false evidence.
Sec. 193 IPC - Punishment for false evidence.—
Whoever intentionally gives false evidence in any
stage of a judicial proceeding, or fabricates false
evidence for the purpose of being used in any stage of
a judicial proceeding, shall be punished with
imprisonment of either description for a term which
may extend to seven years, and shall also be liable to
fine, and whoever intentionally gives or fabricates
false evidence in any other case, shall be punished
with imprisonment of either description for a term
which may extend to three years, and shall also be
liable to fine.

28. D. Permanent erasure of the name of the RMP


from the Medical Register
The disciplinary action taken by medical council
against doctor may be warning or penal erasure
(removal of the name of the medical practitioner from
the medical register).
Penal erasure may be temporary or permanent.
In case of serious and gross medical negligence, the
name of the medical practitioner is removed
permanently, known as professional death sentence.
Rigorous imprisonment and Death penalty can be
passed only by the court of law.

29. A. Second schedule of Indian Medical Council


Act 1956
Recognition of Medical Qualifications by MCI is as
follows:
The First Schedule: Contains the recognized medical
qualifications granted by Universities in India.
The Second schedule: Contains the recognized
medical qualifications granted outside India.
Third schedule has two parts; – Part I: Contains
medical qualifications granted by Indian Universities
which are not included in First schedule – Part II:
Contains medical qualifications granted by Foreign
Universities which are not included in Second
schedule

30. C. Infamous conduct


A doctor can advertise only on certain occasions such
as, when he starts practice, when he shifts his clinic,
when he gains additional qualification to practice etc.
But repeated advertisement in a newspaper by a
medical practitioner amounts to Infamous conduct.
Professional negligence is the absence of reasonable
care and skill or negligence of a doctor in the
treatment of a patient, which may result in injury or
death.
In civil negligence a patient brings charges of
negligence against doctor for monetary
compensation (damages) for the damage
(harm) suffered by him.
Criminal negligence occurs when the doctor
shows gross lack of competency, gross
inattention, and criminal interference to the
patient’s safety, or gross negligence in the
selection and application of remedies.
Privileged communication is defined as “a
bonafide statement made by doctor upon any
subject matter to the appropriate authority to
protect the interests of the community or of the
State”.

31. C. Higher court


Whenever a court of law pass a sentence, the offender
has got the rights to appeal against the verdict of
lower court to higher court.
The higher court has got the power whether to retain
the same sentence or to enhance or to commute the
sentence passed by the lower court.

32. B. 18 years
As per Section 375 IPC, A man is said to commit “rape”
if he
a. penetrates his penis, to any extent, into the vagina,
mouth, urethra or anus of a woman or makes her to do
so with him or any other person; or
b. inserts, to any extent, any object or a part of the
body, not being the penis, into the vagina, the urethra
or anus of a woman or makes her to do so with him or
any other person; or
c. manipulates any part of the body of a woman so as
to cause penetration into the vagina,urethra, anus or
any part of body of such woman or makes her to do so
with him or any other person; or
d. applies his mouth to the vagina, anus, urethra of a
woman/makes her to do so with him or any other
person, under the circumstances falling under any of
the following 7 descriptions:
1. Against her will.
2. Without her consent.
3. With her consent, when her consent has been
obtained by putting her or any person in whom she is
interested in fear of death or hurt.
4. With her consent, when woman believes he is
another man to whom she is lawfully married.
5. With her consent, when she is unable to understand
the nature and consequences of that to which she
gives consent.
6. With or without her consent when she is under 18
years of age; or
7. When she is unable to communicate consent.
Exception 1 – A medical procedure or intervention
shall not constitute rape
Exception 2 – Sexual intercourse or sexual acts by a
man with his own wife, the wife not being under 15
years of age, is not rape
NOTE: The Supreme Court on October 11, 2017
criminalised sexual intercourse between a man and
his underaged wife, provided a complaint is filed
within a year.

33. D. Any of the above


Dying declaration is a written or oral statement of a
person, who is dying due to some unlawful act,
relating to the circumstances leading to his
impending death.
A magistrate should be called to record statement.
If the condition of the patient is serious, the doctor or
any other person can record the statement in the
presence of two witnesses.
34. C. Therapeutic abortion
Declaration of Tokyo is related to Torture and other
cruel and degrading treatment or punishment.
Declaration of Helsinki is related to Human
experimentation.
Declaration of Oslo is related to Therapeutic abortion.
The Declaration of Malta is related to Role of Doctors in
Hunger strike.

35. B. 2 year
Sec. 304A IPC - Causing death by negligence.—
Whoever causes the death of any person by doing any
rash or negligent act not amounting to culpable
homicide, shall be punished with imprisonment of
either description for a term which may extend to two
years, or with fine, or with both.

36. C. Little finger


Fingerprint reader (FINDER), is a computerized
automatic fingerprint reading system which can
record each fingerprint data in half a second.
Prints of 8 fingers are recorded except little finger.

37. C. State Medical Council


The doctors are governed by the Medical council of
India and state medical council.
If the doctor is guilty of professional misconduct
usually the state medical council takes disciplinary
action in the form of warning or penal erasure.
The Medical council of India can also take disciplinary
action, for example, if the MCI notices that a doctor is
given false declaration that he is working as a full time
teacher in a medical college, MCI takes action against
him.
Director of Medical and Health Service, Health
secretary of State Government and Indian Medical
Association cannot take action against professional
misconduct.

38. C. Urea
Post-mortem chemistry:
In Blood - Decrease in Glucose, Na and Cl.
In C.S.F. - Decrease in Glucose.
No change in Urea, Creatinine, Na, Ca and Mg.
In V. Humour - Decrease in Glucose, Na, and Pyruvic
acid.
All other parameters increase after death.

39. D. All of the above


Sec. 337 IPC - Causing hurt by act endangering life or
personal safety of others.—Whoever causes hurt to any
person by doing any act so rashly or negligently as to
endanger human life, or the personal safety of others,
shall be punished with imprisonment of either
description for a term which may extend to six
months, or with fine which may extend to five
hundred rupees, or with both.
Sec. 338 IPC - Causing grievous hurt by act
endangering life or personal safety of others.—
Whoever causes grievous hurt to any person by doing
any act so rashly or negligently as to endanger human
life, or the personal safety of others, shall be punished
with imprisonment of either description for a term
which may extend to two years, or with fine which
may extend to one thousand rupees, or with both.
Sec. 304A IPC - Causing death by negligence.—
Whoever causes the death of any person by doing any
rash or negligent act not amounting to culpable
homicide, shall be punished with imprisonment of
either description for a term which may extend to two
years, or with fine, or with both.

40. B. Civil wrong


A tort is a civil wrong that results when a defendant
engages in conduct that harms another person.
In tort claims the suing party asks for monetary
damages from the defendant to compensate the harm
he or she has suffered.
Medical malpractice cases in which patient demands
compensation are tort cases.

41. A. Cyanosis
Cyanide poisoning manifests with convulsions, dilated
pupils, initially bradycardia and hypertension followed
by tachycardia and hypotension, tachypnea followed
by bradypnea, pulmonary edema and brick red colour
of skin and mucus membrane.
Cyanosis is a late feature.

42. A. Ergot alkaloids


Ergotism is an intensely painful burning sensation in
the limbs and extremities caused by ergotamines from
a fungus (Claviceps purpurea) that can contaminate
rye and wheat.
Ergotism is the effect of long-term ergot poisoning,
traditionally due to the ingestion of the alkaloids
produced by the Claviceps purpurea fungus that
infects rye and other cereals, and more recently by the
action of a number of ergoline-based drugs. It is also
known as ergotoxicosis, ergot poisoning and Saint
Anthony’s fire.
The symptoms can be roughly divided into convulsive
symptoms and gangrenous symptoms.
Convulsive symptoms of ergotism: Convulsive
symptoms include painful seizures and spasms,
diarrhea, paresthesias, itching, mental effects
including mania or psychosis, headaches, nausea and
vomiting. Usually the gastrointestinal effects precede
central nervous system effects.
Gangrenous: The dry gangrene is a result of
vasoconstriction induced by the ergotamine-
ergocristine alkaloids of the fungus. It affects the more
poorly vascularized distal structures, such as the
fingers and toes. Symptoms include desquamation or
peeling, weak peripheral pulses, loss of peripheral
sensation, edema and ultimately the death and loss of
affected tissues. Vasoconstriction is treated with
vasodilators.

43. A. Consent
Principles of medical ethics are:
1. Respect for autonomy: Respect the decisions made
by other people concerning their own lives.
2. Beneficence: Action done for the benefit of others.
3. Non-maleficence: “Do no harm.”
4. Justice: Treat all people equally, fairly, and
impartially.

44. D. Recognition of Medical Qualifications


Functions of State medical council are:
1. Medical Register: It maintains Medical Register, in
which names of all doctors who are enrolled in the
state medical council are entered.
2. Warning notice: SMC can issue Warning Notice
containing certain practices which are regarded as
falling within the meaning of the term “Professional
Misconduct”.
3. Disciplinary action against doctors who is guilty of
professional misconduct.
Medical qualifications are recognized by the Medical
council of India.

45. B. Declaration of Geneva


Declaration of Sydney is related to the definition of
Death (Brain death).
Declaration of Geneva comprises the modernized
versions of the Hippocratic Oath.
Declaration of Venice is related to terminal illness.
Declaration of Istambul is related to Organ Trafficking
and Transplant Tourism.

46. B. 32°C
Exposure to cold produces hypothermia which is
defined as an oral or axillary temperature less than
35°C.
The ability of the hypothalamus to regulate
temperature is completely lost below 30°C.
III effects are manifested in 3 stages.
In the first stage, the patient feels old and
shivers and the body temperature falls.
In the second stage, shivering stops when the
temperature is at or below 32°C.
In the third stage, the temperature is lowered to
27°C or even less, which if maintained for 24
hours or longer is fatal.

47. b. Person incapable of making their will


Active Euthanasia: Inflicting painless death by an act
of commission e.g. injecting large doses of
barbiturates/KCl/curare etc.
Passive Euthanasia: Inflicting painless death by an act
of omission e.g. withdrawal of life saving measures/life
support.
Voluntary Euthanasia: With the consent of the dying
person.
Involuntary (Compulsory) Euthanasia: Without/against
the consent of the person.
Non voluntary Euthanasia: The person is incapable of
making their wishes known, e.g. irreversibly comatose
patient, severely defective patients.

48. D. Section 92
Sec. 87 IPC - Act not intended and not known to be
likely to cause death or grievous hurt, done by
consent.—Nothing which is not intended to cause
death, or grievous hurt, and which is not known by the
doer to be likely to cause death or grievous hurt, is an
offence by reason of any harm which it may cause, or
be intended by the doer to cause, to any person,
above eighteen years of age, who has given consent,
whether express or implied, to suffer that harm; or by
reason of any harm which it may be known by the
doer to be likely to cause to any such person who has
consented to take the risk of that harm.
Sec. 89 IPC - Act done in good faith for benefit of child
or insane person, by or by consent of guardian.—
Nothing which is done in good faith for the benefit of a
person under twelve years of age, or of unsound mind,
by or by consent, either express or implied, of the
guardian or other person having lawful charge of that
person, is an offence by reason of any harm which it
may cause, or be intended by the doer to cause or be
known by the doer to be likely to cause to that person
Sec. 90 IPC - Consent known to be given under fear or
misconception. A consent is not such a consent as it
intended by any section of this Code, if the consent is
given by a person under fear of injury, or under a
misconception of fact, and if the person doing the act
knows, or has reason to believe, that the consent was
given in consequence of such fear or misconception;
or Consent of insane person. If the consent is given by
a person who, from unsoundness of mind, or
intoxication, is unable to understand the nature and
consequence of that to which he gives his consent; or
Consent of child.—unless the contrary appears from
the context, if the consent is given by a person who is
under twelve years of age.
Sec. 92 IPC - Nothing is an offence by reason of any
harm which it may cause to a person for whose benefit
it is done in good faith, even without that person’s
consent, if the circumstances are such that it is
impossible for that person to signify consent, or if that
person is incapable of giving consent, and has no
guardian or other person in lawful charge of him from
whom it is possible to obtain consent in time for the
thing to be done with benefit.

49. D. 2 Years
As per Consumer protection Act 1986, a complaint is
to be filed within 2 years from the date on which a
cause of action has arisen.
An order is to be given within 90 days, unless the
goods in question are to be sent to an approved
testing laboratory.
Appeal to the higher forum/commission to be made
within 30 days.

50. B. Conium masculatum


Conium masculatum in also known as Hemlock,
contains mainly coniline which is a peripheral nerve
poison.
It causes burning in mouth and throat, vomiting,
diarrhoea, slow respiration, increased and later slow
pulse, mental confusion, tremors, ataxia, sometimes
blindness, progressive motor paralysis extending
upwards from the extremities, coma and death from
respiratory paralysis.

51. C. Can cause immediate death


Hyperventilation before drowning can cause rapid
death, as CO2 will be flushed out and there will be no
hypercapnia, which is necessary to stimulate
respiratory centre.

52. d. Children starved by their parents


Starvation may be homicidal. Children starved by their
parents/guardians is known as “Baby-farmer”.
Starvation may be accidental as in person is
caught/entombment occurs during earthquakes or
landslides. Accidental starvation may be seen in
famine and flood.
Suicidal starvation occurs when someone deliberately
withhold food as seen among prisoner who go on
hunger strike as a form of protest.

53. B. Sex
Chilotic line is an anthropometric line extending from
the posterior aspect of the ilio-pectineal eminence to
the closest point on the anterior auricular margin (the
pelvic segment) and then to the iliac crest (the sacral
portion).
The pelvic segment is predominant in females and the
sacral segment is predominant in males.
Thus, chilotic index may be used in forensic practice
to identify the sex of human skeletal remains,
although studies suggest its reliability is limited.

54. b. Organophosphorus
In 1930, thousands of Americans were poisoned by an
illicit extract of Jamaica ginger (“jake”) used to
circumvent the Prohibition laws.
A neurotoxic organophosphate compound,
triorthocresyl phosphate (TOCP), had been used as an
adulterant.
The earliest reports were of peripheral neuritis, but
later it was evident that an upper motor neuron
syndrome had supervened.
This TOCP poisoning apparently involved various cell
groups and tracts in the spinal cord; the lesions was
not peripheral at all.
The principal findings showed the spasticity and
abnormal reflexes of an upper motor neuron
syndrome.

55. A. Trotter and Glesser’s Formula


Stature from long bones can be estimated by
measuring the length of long bones and by using
regression formulae, such as:
Jit and Sing formula,
Trotter and Glesser’s formula,
Karl-Pearson’s formula,
McKern & Steele’s formula,
Dupertuis and Hadden’s formula etc.
Haase’s rule is used to estimate the gestational age of
a foetus.
Gustaffson’s method is used to estimate the age of a
person by secondary changes in the teeth.
Rule of 12 is for rigor mortis.

56. A. Dentition
The age of a child up to 12-14 years can better be
determined by the study of eruption of temporary and
permanent teeth.
The study of appearance and fusion of ossification
centres help to estimate the age up to 21 years.
Head circumference helps to estimate the age of a
foetus.
Anthropometry is used to identify an adult person by
the descriptive data, body moles/scars and body
measurements.

57. C. Mass Casualties


Triage is the sorting out and classification of casualties
of war or other disasters, to determine priority of need
and proper place of treatment.

58. A. Gastric lavage should be done


Signs and symptoms are of organophosphorus
poisoning including smell of kerosene.
Atropine is the vantidote for organophosphorus.
Airway management is an important step in all the
cases.
Gastric lavage is useful anytime within 3 hours after
ingestion of a poison. (But lavage can be done up to
12 hours after ingestion in poisoning by tricyclic
antidepressants, salicylates, antihistamines,
phenothiazines).

59. C. 40 years
The four pieces of the body of sternum start fusing
from below upwards.
The 4th piece fuses with 3rd piece at the age of 15
years, the 3rd piece fuses with 2nd piece at the age of
20 years and the 2nd piece fuses with the 1st piece at
the age of 25 years.
The xiphisternum fuses with body at the age of 40
years.
The manubrium fuses with body at an old age.

60. C. Whorls
The incidence of fingerprints are
Loops - 67%,
Arches - 6-7%,
Whorls - 25%, and
Composite - 1-2%.

61. A. Precipitin test


Precipitin test is species specific test which is used to
identify the species of any body fluid or tissue.
Neutron activation analysis is used to identify the
metals.
Spectroscopic examination is a confirmatory test for
blood.
Benzidine test is a presumptive test for blood.

62. d. Grandfather
Atavis=grandfather.
Normally a child resembles its parents, but in case of
atavism, a child doesn’t resemble its parents, but
resembles its grandparents.
It is due to inheritance of characteristics from remote
rather than from immediate ancestors, due to a chance
recombination of genes.

63. d. Hospitals where some patients are charged


and some are not charged
Exemption given to doctors from COPRA is
1. Doctors who are working in government hospitals,
where patients are not charged at all.
2. Doctors working in hospitals where services are
rendered free or charge to everybody.
3. Doctors working in hospitals where only a nominal
registration fee is collected, and all other services
are provided free of cost.
Doctors Not Exempted from COPRA is
1. Doctors who are working in hospitals where some
patients are charged and some are not charged.
2. Doctors working in private hospitals where the
patients are charged.

64. A. Bernard Spilsbury


Bernard Spilsbury (1877-1947) is the father of Modern
Forensic Medicine.
Mathieu Orfila (1787-1853) is the father of Modern
Toxicology.
Paracelsus (1493-1541) is the father of Toxicology and
Paolo Zacchia (1584-1659) is the father of Forensic
Medicine.

65. B. Quinacrine dihydrochloride


Staining for X chromosomes are:
1. Fluorescent Feulgen reaction using acriflavin Schiff
reagent; (Chromatin positivity: 50-70% in females
and 0-2% in males) and
2. FISH (Fluorescent in situ hybridization).
The ‘Y’ Chromosomes present in males are fluorescent
for the dye Quinacrine dihydrochloride.

66. B. 3 month old infant with bruise on right


forearm and left buttock; parent states infant
rolled off changing table
Baby may be able to kick himself over, from his
tummy to his back, as early as age 4 months. It may
take him until he’s about 5 or 6 months to flip from
back to front, though, because he needs stronger neck
and arm muscles for that manoeuvre.
The injuries mentioned under options A, B and C is
accordance with the history given.

67. D. A, AB, B or O
There are 4 phenotypes and 6 genotypes in blood
group inheritance.
If father is phenotype A (Genotype may be AA or AO)
and mother is phenotype B (Genotype may be BB or
BO), the offspring’s genotype can be AB or AO or OO
or OB, which will manifest the genotype as AB or A or
O or B.

68. b. Tying of a plastic bag over the head


Beating on the sole of the feet is known as Falanga or
Falaka or Bestinado.
Dry submarine means Head and face are covered with
a plastic bag.
In Wet submarine (Latina), Head is submerged in
polluted water.
Inserting heated metal skewer into the anus is known
as Black slave.

69. b. Theory of exchange


Jit & Sing, Trotter & Glesser, Karl-Pearson, McKern &
Steele and Dupertuis & Hadden are known for formula
for estimation of stature.
Sir Edmond Locard is known for Theory of exchange,
(i.e. whenever two objects come in contact with each
other always there is transference of material from
one object to the other) and also known for Poroscopy
(i.e. Locard’s method – Identification of a person by
the study of the mouths of the ducts of sweat glands
in the ridges of fingers).
Sir Alphonso Bertillon is known for System of personal
identification using the body measurement, body
marks and body description.
Sir William J Herschel, Sir Francis Galton and Sir
Edward Henry are known or dactylography.

70. D. Trunk
In case of bodies lying in water, the post-mortem
staining is usually found on the face, the upper part of
the chest, hands, lower arms, feet and legs, as the
body usually floats face down, buttocks up with limbs
hanging down in front of the body.

71. C. 30-48 hours


In acute starvation, there is a feeling of hunger and
the hunger pain (Hunger pangs) starts in about 12 to
24 hours, which lasts in about 30 to 48 hours but
thirst becomes intense.
After 4 to 5 days of starvation, general emaciation and
absorption of the subcutaneous fat begins to occur.

72. B. Minimum imprisonment for forced


heterosexual sodomy is 10 years
Sec.377 IPC. Unnatural offences.—Whoever voluntarily
has carnal intercourse against the order of nature with
any man, woman or animal, shall be punished with
1[imprisonment for life], or with impris¬onment of
either description for a term which may extend to ten
years, and shall also be liable to fine.
As per recent amendment in rape law, minimum
imprisonment for rape is 10 years and forced anal
intercourse (forced means without consent) amounts
to rape.

73. C. Age
Ossification centre for lower end of femur appears at
9th month of intra-uterine life and fuses with the shaft
at 18-19 years in males and 17-18 years in females
(one year earlier than males), which helps in
determination of age of a person.
Stature/Height of a person can be determined by the
entire femur.
However weight cannot be determined by the femur.

74. b. 2
Flaccidity of Eyeball: Sunken eyes due to decreased
intraocular tension.
Value of intra-ocular tension is
During life : 14-25 gm.
Soon after death : <12 gm.
Within half an hour : < 3gm.
End of 2 hours : NIL

75. d. Upper end of tibia


Ossification centre for upper end of humerus appears
at ½ to 1 year.
Ossification centre for scaphoid appears at 4-5 years.
Ossification centre for lower end of femur appears at
9th month of intra-uterine life.
The ossification centre for the upper end of tibia
appears at birth.
The other ossification centres appear at birth are, 4th
piece of the body of the sternum and cuboid.

76. A. Parietal
The glabella, supraorbital ridges, mastoid process,
occipital protuberance are prominent in males and not
prominent in females.
But the frontal and parietal eminences are more
prominent in females than that of males.

77. C. 30-35 years


Ectocranial sutures of sagittal starts fusing at 30-40
years, coronal starts fusing at 40-50 years and
lambdoid starts fusing at 50-60 years.
The endocranial sutures fuse 5-10 years earlier than
ectocranial sutures.
So, sagittal starts fusing at 20-25 years, coronal starts
fusing at 30-35 years and lambdoid at 40-45 years (in
this question we have to consider fusion of
endocranial sutures).

78. A. Baygon
Oximes are indicated only in Organophosphorus
compounds poisoning and contraindicated in
Carbamates.
The OP compounds are:

Alkyl phosphates Aryl phosphates

HETP Parathion (Follidol)

TEPP Methyl Parathion (Metacide)

OMPA Diazinon (Tik-20)

Malathion etc. Paraoxon etc.

Carbamate insecticides include carbaryl, aldicarb,


baygon, ficam and propoxur.
79. C. Use reasonable amount of force and
examine
Sec. 53 CrPC: Examination of accused by medical
practitioner at the request of police officer.
1. When a person is arrested on a charge of committing
an offence of such a nature and alleged to have been
committed under such circumstances that there are
reasonable grounds for believing that an examination
of his person will afford evidence as to the commission
of an offence, it shall be lawful for a registered medical
practitioner, acting at the request of a police officer not
below the rank of sub- inspector, and for any person
acting in good faith in his aid and under his direction,
to make such an examination of the person arrested as
is reasonably necessary in order to ascertain the facts
which may afford such evidence, and to use such force
as is reasonably for that purpose.
2 Whenever the person of a female is to be examined
under this section, the examination shall be made only
by, or under the supervision of, a female registered
medical practitioner.

80. A. Sex
The sex determination from the skeletal remains can
be done only after puberty, except in case of greater
sciatic notch where sex can be determined even in a
foetus.
In case of hip bone, sex can be determined from the
greater sciatic notch (narrow and deep in males and
wide and shallow in females), shape of the body of the
pubis (triangular in males and quadrangular in
females), and pre-auricular sulcus, which is not
prominent (sometimes not even visible), narrow &
shallow in males, and more frequent, broad and deep
in females
Best criteria in pelvis to determine sex is Sciatic notch
index.
Pre-Auricular Sulcus is a Tertiary sexual characteristic.

81. D. Greenish discoloration


Maceration is a process of aseptic autolysis.
The changes of maceration were well studied and
classified by Langley.
Amniotic membrane should be intact (exclusion of air)
with abundant liquor amnii.
The earliest sign is skin slippage which is seen in 12
hours after death of the fetus.
The typical features are seen in 3-4 days.
The skin is coppery brown in colour and has a
sweetish disagreeable odour.
Large blebs appear at 24 hours.
The body is soft, flaccid and flattens out when placed
on a level surface.
There will be hypermobility of the joints.
The skull bones are separated with collapsed vertebral
column.

82. C. Jejunum
In order of frequency, the structures most likely to be
damaged in blunt abdominal trauma are: liver, spleen,
kidney, intestine, abdominal wall, mesentery,
pancreas and diaphragm.
The jejunum is the commonest site of rupture,
followed by the ileum, duodenum, caecum and large
intestine.

83. A. Beneath the esophagus


Apparent “bruising” of the neck musculature does not
always indicate compression of or trauma to the neck.
In any autopsy, substantial haemorrhage may be seen
over the posterior surface of the oesophagus and the
anterior longitudinal ligament of the cervical spine.
vThis is a common post mortem artefact described
fully by Prinsloo and Gordon, sometimes known by
their name.
This can be reduced to certain extent by bloodless
field of dissection of the neck.

84. A. Pelvis
According to Krogman, percentage of accuracy in
determination of sex from bones is as follows:
Entire skeleton 100%
Pelvis + Skull 98%
Pelvis alone 95%
Skull alone 90%
Only long Bones 80%

85. C. 193
Sec.191 IPC. Giving false evidence.—Whoever, being
legally bound by an oath or by an express provision of
law to state the truth, or being bound by law to make
a declaration upon any subject, makes any statement
which is false, and which he either knows or be-lieves
to be false or does not believe to be true, is said to
give false evidence. Explanation 1.—A statement is
within the meaning of this sec¬tion, whether it is
made verbally or otherwise. Explanation 2.—A false
statement as to the belief of the person attesting is
within the meaning of this section, and a person may
be guilty of giving false evidence by stating that he
believes a thing which he does not believe, as well as
by stating that he knows a thing which he does not
know.
Sec.192 IPC. Fabricating false evidence.—Whoever
causes any circumstance to exist or 1[makes any false
entry in any book or record, or electronic record or
makes any document or electronic record containing a
false statement], intending that such circumstance,
false entry or false statement may appear in
evi¬dence in a judicial proceeding, or in a proceeding
taken by law before a public servant as such, or before
an arbitrator, and that such circumstance, false entry
or false statement, so ap¬pearing in evidence, may
cause any person who in such proceeding is to form
an opinion upon the evidence, to entertain an
errone¬ous opinion touching any point material to the
result of such proceeding, is said “to fabricate false
evidence”.
Sec.193 IPC. Punishment for false evidence.—Whoever
intentionally gives false evidence in any stage of a
judicial proceeding, or fabri¬cates false evidence for
the purpose of being used in any stage of a judicial
proceeding, shall be punished with imprisonment of
either description for a term which may extend to
seven years, and shall also be liable to fine, and
whoever intentionally gives or fabricates false
evidence in any other case, shall be punished with
imprisonment of either de¬scription for a term which
may extend to three years, and shall also be liable to
fine.
Sec.197 IPC. Issuing or signing false certificate.—
Whoever issues or signs any certificate required by law
to be given or signed, or relating to any fact of which
such certificate is by law admissi¬ble in evidence,
knowing or believing that such certificate is false in
any material point, shall be punished in the same
manner as if he gave false evidence.
86. b. Leprosy
Permanent impairment of fingerprints can occur in:
Leprosy, Charring, Radiation, Electrical injuries,
and Corrosives.
Ridge alteration take place in:
Eczema, acanthosis nigricans, scleroderma, dry
and atrophic skin.
Temporarily modification is seen in:
Coeliac disease.
Change in the distance between ridges but without
change in pattern is seen in:
Rickets and acromegaly.

87. B. Sex
Corporo-basal index : Length of Body of S1/Length of
Base of Sacrum x 100.
In Males, it is 45 and in females it is 40.5
It is the only index, which is more in Males.
Other indices to differentiate sex from bones are:
Sternal index,
Ischio-pubic index (Washburn index),
Sciatic notch index,
Sacral Index and
Medullary Index.

88. A. Finger print pattern


Identical twins have the same DNA fingerprint pattern,
HLA system and Blood group.
But the fingerprints pattern is different in identical
twins, that’s why it’s the best method of identification.

89. C. Race
The following indices help to differentiate race from
bones:
1. Cephalic Index: Maximum transverse breadth of skull
/ Maximum anteroposterior length of skull × 100
2. Brachial Index: Length of Radius/Length of Humerus
× 100
3. Crural Index: Length of Tibia/Length of Femur × 100
4. Humero-Femoral Index (Intra-membral index):
Length of Humerus/Length of Femur × 100
5. Inter-membral index : (Length of Humerus + Length
of Radius)/( Length of Femur + Length of Tibia) x 100

90. b. Sternum
Medullary Index is used to differentiate the sex.
Medullary index = Diameter of Whole bone/Diameter
of Medulla x 100.
Ideal bones for Medullary index are: Tibia, Humerus,
Radius, Ulna.
Medullary index is also used to differentiate human
hair from animal hair; i.e., Diameter of whole
hair/Diameter of Medulla x 100.

91. b. 100 ml
Minimum amount of blood required for fatality in case
of EDH is 100ml
Death may occur in cases of SDH, if haemorrhage is
about 100-150ml.
Minimum blood to give rise to Acute Cardiac
Tamponade: 200 ml
Amount of air needed to produce fatal air embolism:
100ml

92. A. Public Prosecutor


For a prosecution witness, the public prosecutor will
start with chief examination, followed by cross
examination by the defence lawyer.
Again, re-examination is conducted by the public
prosecutor. The object is to correct any mistake or to
clarify or add details to the statements the witness has
made in cross-examination.

93. D. Finger prints


Bertillon system or Anthropometry (Portrate Parle) is an
Anthropometric system based on the physical
measurement of specified parts of body, even if there
are no biological Specimens.
Three important criteria of this system is
1) Descriptive data: Includes full description of a
person such as complexion, colour of the hair, colour of
the iris, shape of the nose, lips, chin, etc.
2) Body marks: Such as moles, birth marks, scars, etc.
3) Body measurements: 11 in number, namely,
1. Standing height
2. Sitting height
3. Length of the head
4. Breadth of the head
5. Breadth of the face
6. Distance between two outstretched upper limbs
7. Length of left forearm and hand
8. Length of left middle finger
9. Length of left little finger
10. Length of left foot and
11. Length of right ear

94. C. Tiara
Superimposition is a technique in identifying a person
by superimposing the X-ray of skull found after death
on the photo of the face of the suspected person.
This was first applied by Glaister and Brash in 1935 in
Isabella Ruxton’s case.
The ‘TIARA’ was used to enlarge the photo to life size.
A negative result having more credibility because it
can definitely be stated that the skull and the
photograph are not those of the same person.

95. A. Ethylene glycol


Ethylene glycol; also called antifreeze. It is a clear,
colorless, odourless, non-volatile liquid with a bitter-
sweet taste.
MOA – Toxicity through metabolites, i.e. glycolic and
oxalic acids which inhibit oxidative phosphorylation.
Clinical features:
Tachycardia, tachypnoea and congestive heart failure
Hypocalcemia, electrolyte imbalance and metabolic
acidosis (due to glycolic acid)
Oxaluria, acute tubular necrosis, renal failure and
Calcium oxalate crystals (due to oxalic acid).
Treatment:
Gastric lavage, sodium bicarbonate (to combat
acidosis), calcium gluconate (to correct hypocalcemia)
Ethanol or 4 methyl pyrazole (fomepazole) as antidote
Hemodialysis in cases of severe poisoning.

96. C. Pointed object cannot cause it


An abrasion is destruction of the skin, which involves
only superficial layers of skin, i.e., only epidermis.
It is a two dimensional injury - only length and
breadth.
It bleeds less and does not form scar after healing.
A scratch can be produced by the tip of a knife or any
pointed object.
Direction can be made out by the heaped up
epithelium at the end.

97. C. 4th degree burns


Dupuytrens Classification of burns is as follows:
First degree Erythema
Second degree Vesication with blister
formation
Third degree Destruction of superficial layer
of dermis
Fourth degree Destruction of whole skin
(dermis)
Fifth degree Destruction of fascia and
muscles
Sixth degree Charring involving vessels,
nerve, Bones

98. b. 450
The number of Ossification Centers
at 8th week of intra-uterine life are 806 and
at birth are 450.
An adult will have 206 bones.

99. D. Implied term of contract between doctor and


patient
Professional secrecy is an implied term of contract
between the doctor and his patient.
The doctor is obliged to keep secret, all that he comes
to know concerning the patient in the course of his
professional work.
Its disclosure would be a failure of trust and
confidence.
The patient can sue the doctor for damages if the
disclosure is voluntary, has resulted in harm to the
patient and is not in the interest of the public.

100. C. 68 cm
The height of a child:
at full term is about 50 cm.
at 6 months is about 60cm.
at 1 year is about 68cm. and
at 4 years is between 90 and 100cm (Twice the birth
height).

101. d. 351
351 of IPC: Assault.—Whoever makes any gesture, or
any preparation intending or knowing it to be likely
that such gesture or preparation will cause any person
present to apprehend that he who makes that gesture
or preparation is about to use criminal force to that
person, is said to commit an assault. Explanation.—
Mere words do not amount to an assault. But the
words which a person uses may give to his gestures or
preparation such a meaning as may make those
gestures or preparations amount to an assault.
319 of IPC: Hurt.—Whoever causes bodily pain,
disease or infirmity to any person is said to cause hurt.
320 of IPC: Grievous hurt.
324 of IPC: Causing hurt by dangerous weapon.

102. A. Bicuspid aortic valve


Turner’s syndrome is characterized by 3 principal
features:
1. Short stature.
2. Congenital anomalies like web neck, shield chest,
wide-set nipples, cubitus valgus, bicuspid aortic
valve (Most common heart defect – 30%),
coarctation of aorta (5-10%), septal defect, aortic
dissection, osteoporosis, renal abnormalities, red
green colour blindness, pigmented navei, short 4th
metatarsal, dyslexia, high-arched palate, low-set
ears, spina bifida, low posterior hairline, etc.
3. Sexual infantilism: primary amenorrhoea, sterility,
grossly undeveloped breast, scanty pubic & axillary
hair, infantile vagina, infantile fallopian tubes and
ovaries containing fibrous tissue without ovarian
follicles(streak ovaries).

103. b. Syphilis
Early tooth eruption is seen in Syphilis, may be at
birth.
Delayed tooth eruption is seen in;
Hypothyroidism (Commonest cause),
Hypopituitarism,
Rickets,
Down’s syndrome,
Cleidocranial dysplasia,
Cysts and
Supernumerary teeth.

104. A. Ring fracture + lumbar spine injury


Ring Fracture is a type of fissured fracture that
encircles the base of the skull around the foramen
magnum, usually running 3–5 cm outside the foramen
magnum at the back and sides of the skull, passing
forward through the middle ears and roof of the nose.
Such types of fractures are usually noticed in the
following cases:
1. Fall from a height on to feet or buttocks,
when the force of fall is transmitted upwards
through the spinal column.
2. Vault of skull being driven against the spine
by falling of heavy load over the vertex or fall
from a height on the head or heavy blow over
the vertex.
3. Violent twisting of the head on the spine,
shearing the vault from the base.
4. A heavy blow directed underneath the
occiput or chin causing the fracture by
violently lifting the skull from the spine and
thereby breaking it away from its basal
attachment
Fractures of the thoracic and lumbar spine may result
from high-energy trauma, such as a:
Car or motorcycle crash
Fall from height
Sports accident
Violent act, such as a gunshot wound

105. B. Stack’s method


Mile’s method: This is used by studying only the root
transparency of a tooth after the age of 30 years.
Boyde’s method: Cross striations in the enamel and
dentine of deciduous teeth and permanent first molars
represent daily incremental lines. Studied by
histological sections. The age of an individual can be
calculated by counting the number of lines from
neonatal line onwards. Neonatal line is considered as
an indicator of birth. This method is applicable mainly
to infants. The age so calculated is accurate with an
error of around 20 days.
Stack’s method: Estimation of the age of infants from
the weight and height of the erupting teeth of child. It
is used on both deciduous and permanent during their
erupting phase.
Lamendil’s method: Estimation of age from using only
2 criteria namely, gingival recession and root
transparency.

106. D. 12 months
Sex chromatin cannot be made out in decomposed
bodies.
But by using fluorescent dyes, Y chromosomes can be
demonstrated up to 1 year after death in dental pulp
tissue.
The F-bodies are seen in 30-70% of cells in males.

107. A. 10 ml
The following must be preserved in all cases of
suspected poisoning
1. Stomach and its contents
2. Upper part of small intestine (About 30 cm) and its
contents
3. Liver - 200-300 gm
4. Kidney - ½ of each, as one kidney may be
dysfunctional
5. Blood - 30 ml (Minimum 10 ml)
6. Urine - 30 ml

108. D. 376D of IPC


376A IPC - Punishment for causing death or resulting
in persistent vegetative state of victim.
376B IPC - Whoever has sexual intercourse with his
own wife, who is living separately, whether under a
decree of separation or otherwise, without her
consent, shall be punished with imprisonment of
either description for a term which shall not be less
than two years but which may extend to seven years,
and shall also be liable to fine. Bailable and
Cognizable (on complaint of victim).
376C IPC - Sexual intercourse by person in authority.
376D IPC – Gang rape

109. C. 15 kg

Structure Constricting force required Causative effect

Jugular vein 2 Kg Cerebral apoplexy

Carotid artery 4-5 Kg Cerebral Anoxia

Trachea 15 Kg Asphyxia

Vertebral artery 20 Kg Cerebral anemia

110. C. Abdomen
The depth of a stab wound is usually equal to or less
than the length of the blade that was used in
producing it,
But on yielding surfaces like the anterior abdominal
wall, the depth of the wound may be greater than the
length of the blade, because the force of the thrust
may press the tissues underneath.

111. A. Nicotine
Nicotine is a alkaloid present in plant Nicotiana
tabacum. dried leaves of which are used as tobacco
for chewing, smoking and other recreational purposes.
It is considered a cardiac poison.
In the habitual smoker, nicotine causes amblyopia,
narrowing of the field of vision and some blurring, etc.
Habitual smoking is reported to lead to tobacco heart -
a condition characterised by irregularity, extra systole
and occasional attacks of pain suggesting angina
pectoris.

112. C. Acetic alcohol


Supersaturated sodium chloride or rectified spirit is
used as a preservative for body viscera.
Acetic alcohol is used as preservative for maggots.
Maggots are killed by dropping into boiling absolute
alcohol (so that it dies in extended position) and
preserved in Acetic alcohol.
Dimethyl sulphoxide (DMSO) is used as a preservative
for muscle for DNA.

113. A. Ultra violet light


Latent tattoos are invisible tattoo marks which can be
made visible by ultraviolet rays.
Infrared photography makes old tattoos readily visible
Regional lymph node biopsy: Previous application of
tattoo mark (Latent tattoo) can be determined in a
dead body where there will be presence of dye.
Laser: Is the best method to remove tattoo.

114. C. He must keep the police informed about


the findings
A medicolegal autopsy is to be carried out at the
behest of the appropriate legal authority. The
request/order may move from the police officer
(usually the station house officer or sub-inspector of
police) or from magistrate or from Coroner under
whose jurisdiction the incidence/ event leading to
death occurred.
A doctor should record all positive findings and
important negative ones.
No autopsy is complete unless all three major body
cavities are opened, i.e. cranial, thoracic and
abdominal cavities.
A doctor must preserve viscera and send for toxicology
examination in case of poisoning/suspected
poisoning/where there are no other findings/when
requested by the investigating officer.
No unauthorised person should be permitted to enter
the mortuary. No police official should be present
while the autopsy is being conducted and no need to
keep the police informed about the findings.

115. C. Sec. 228(A) IPC


Sec. 327(2) CrPC - The inquiry into and trial of rape or
shall be conducted in camera: Provided that the
presiding judge may, if he thinks fit, or on an
application made by either of the parties, allow any
particular person to have access to, or be or remain in,
the room or building used by the court.
Sec. 357(C) CrPC - All hospitals, public or private,
whether run by the Central Government, the State
Government, local bodies or any other person, shall
immediately, provide the first-aid or medical
treatment, free of cost, to the victims of any acid
attack or rape, and shall immediately inform the
police of such incident”
Sec. 228(A) IPC - Disclosure of identity of the victim of
certain offences etc.—Whoever prints or publishes the
name or any matter which may make known the
identity of victim of rape shall be punished with
imprisonment of either description for a term which
may extend to two years and shall also be liable to
fine.
Sec. 166(B) IPC - Whoever, being in charge of a
hospital, public or private, whether run by the centeral
government, the state government, local bodies, or
any other person, contravences the provisions of
section 357C of the code of criminal procedure, shall
be punished with imprisonment for a term which may
extend to one year or with fine or with both.

116. C. Chennai
First Medico legal autopsy in the world was done by
Bartolomeo Varignana, in Italy in 1302.
First book of Forensic Medicine was written by
Furtunato Fedele, an Italian in 1602.
First Post-mortem examination in India was done by
Dr. Buckley in Madras (Present Chennai) in 1663, a
case of suspected Arsenic poisoning.

117. B. The fetus can be aborted after her consent


Vaginal swab is not needed because the rape
incidence is not recent.
Option C is obvious.
Urine pregnancy test is not necessary.
Foetus cannot be terminated, because as per MTP Act
1971, pregnancy can be terminated only up to 20
weeks of pregnancy.
A woman can give consent for termination of
pregnancy only after 18 years of age, if termination is
indicated.

118. D. Sodium polyanethole sulfonate


EDTA or Heparin is used as preservative for blood for
DNA.
Sodium / Potassium fluoride, 10 mg/ml (prevents
glycolysis, inhibits enzymatic activity and bacterial
growth) + K oxalate is used as preservative for blood.
Sodium polyanethole sulfonate (SPS) is an
anticoagulant which is used for blood for culture.

119. A. Full choke


Choking: The degree of dispersion can be controlled to
some extent by using a constricting device near the
muzzle end, called choke.

Types of Choke Constriction % of shots found in 30” circle at


40yards

Full Choke 40/1000 of inch 65 – 70%

¾ Choke 30/1000 of an 65%


inch

½ Choke 20/1000 of an 60%


inch

¼ Choke 10/1000 of an 55%


inch

Improved Cylinder 3-5/ 1000 of an 50%


inch

True Cylinder (No 0/ 1000 30-40%


Choke)

120. D. Primary rugae are less than 3mm


Palatal rugae are used as a method of identification.
Discovered by Harrison Allen. Anterior palate (1/3rd) is
preferred over posterior part. Palatoprints do not
change during growth.
Thomas & Kotze have classified rugae into 3 types
Primary rugae : 5mm or more.
Secondary rugae : 3-5 mm.
Fragmentary rugae: Less than 3mm.

121. b. Air embolism from putrefactive gas


Alkaline pyrogallol test helps to differentiate air
embolism from putrefactive gases.
4 ml of alkaline pyrogallol is taken in a 10ml syringe
and air is aspirated from right side of the heart.
The needle is removed and replaced by a stopper.
If air (Oxygen) is present, the mixture turns brown on
shaking.
In a second syringe, some air is introduced by the
atmosphere and the test is repeated as a control.

122. B. Adherence to the wall


Differentiation between Ante-mortem and Post-
mortem clots:
The ante mortem clots are firm, rubbery and attached
to the vessel wall by lines of Zahn.
Post mortem clots are soft, friable, having appearance
of red currant jelly (red cells)/ yellow chicken fat
(serum) and not attached to the vessel wall by lines of
Zahn.
Post-mortem fibrinous clots in the heart are known as
Cardiac Polyps.

123. D. 500 grams


During exhumation, in case of suspected poisoning, 6
soil samples of 500 g each should be taken from each
side of the body or coffin, i.e., above the body, below
the body, both sides of the body, head end of the body
and foot end of the body.
In addition, at least two samples must be taken from
some distance (sample soil) - say around 25 to 30
yards from the grave.
In suspected mineral poisoning, hair, nails, and long
bone e.g., femur should be preserved for chemical
analysis.

124. D. Klismaphilia
Agalmatophilia: Sexual attraction to a statue, doll,
mannequin or other similar figurative object.
Coprolagnia: Sexual pleasure is obtained from the
thought, sight, or touching of excrement.
Mysophilia: Sexual pleasure associated with
something soiled or filthy, usually a kind of
undergarment after use.
Klismaphilia: is a paraphilia involving enjoyment of,
and sexual arousal from, enemas.

125. B. Endrin
Organochlorine (OC) compounds are Endrin, Dieldrin,
Aldrin, DDT, Gamaxane, BHC, Lindane, Endosulfan,
Toxaphene etc.
These interfere with the nerve impulse transmission.
There is no antidote available and hence only
symptomatic treatment is done.
Endrin is also called plant penicillin.
Parathion and TIK-20 are organophosphates, and
Baygon is a combination of organophosphate and
carbamate; for all of these atropine is the drug of
choice for poisoning.

126. B. Anaemic Anoxia


Gorden’s (Barcroft’s) modes of death are
1. Anoxic Anoxia - No oxygen available to the body,
e.g., High altitude, Hanging, Strangulation etc.
2. Anaemic Anoxia – anoxia resulting from a
decreased concentration of haemoglobin, e.g.,
Acute haemorrhage, Severe Anaemia, poisoning by
Carbon monoxide (Chemical anoxia), nitrates,
chlorates, coal tar derivatives etc.
3. Histotoxic /Cellular Anoxia - Poisoning of the
respiratory enzyme systems of the tissues, as in the
inhibition of cytochrome oxidase by cyanides;
because of the inability of tissue cells to use
oxygen, its tension in arterial and capillary blood is
usually greater than normal.
4. Stagnant Anoxia - a condition in which there is
inadequate blood flow in the capillaries, causing
low tissue oxygen tension and reduced oxygen
exchange. The condition is associated with shock,
cardiac standstill, and thrombosis.

127. B. Barbiturates poisoning


The temperature of the body remains raised for the first
two hours up to 1.50C to 20C after death, known as
post-mortem caloricity.
Conditions resulting in postmortem caloricity:
1. Disturbance of heat regulation
Sunstroke
Pontine haemorrhage
2. Increased production of heat due to muscular
activity
Tetanus
Strychnine
Violent asphyxial deaths (Struggle prior to death)
3. Excessive bacterial activity
Septicaemia
Cholera
Influenza etc.
4. Miscellaneous
Alcohol, Heroin, Cocaine, CO poisoning
High atmospheric temp., Drug reactions, Fat/Air
embolism

128. C. Oxalic acid


Oxalic acid is present in many plants.
Plants containing oxalic acid are – Cabbage, unripe
carambola, Jack-in-the-pulpit, unripe monster fruit,
rhubarb, spinach and wood sorrel.

129. D. All of the above


Suspended animation (Apparent death) is defined as
“a state of body in which the vital functions are at
such a low pitch that the body functions cannot be
determined by ordinary methods of clinical
examination”.
Suspended animation may last for few seconds to half
an hour or more. In this condition, actually, the
circulation, respiration etc. do not completely stop but
is being maintained in their minimum. Prompt
resuscitation can revive the state. Apparent death is of
two types:
1. Voluntary – Produced voluntarily. Yogis or
‘sadhus’ or ‘sanyasi’ may practice this method.
2. Involuntary – person lands in apparent death
spontaneously. Involuntary suspension of
animation lasting from a few seconds to half an-
hour or more may be found in:
A: After Anesthesia
B: in newborn Babies
C: Cholera/Cerebral concussion
D: Drowning.
E: Electrocution
Shock, Sunstroke, Syncopal attack, Insanity,
Hypothermia,
Narcotic poisoning, vagal inhibition etc.

130. A. Drowning in a fast flowing river


Postmortem lividity (hypostasis) is bluish purple or
purplish red discoloration, which appears under the
skin of the dependent parts of the body after death
due to capillo-venous distention.
P.M. Lividity is not seen in bodies found in running
water, as the position of the body is constantly
changing in running water.
Whereas it is seen bodies lying in well, swimming pool
or in case of post-mortem submersion.
In case of hanging, hypostasis is more marked in lower
limbs, external genitalia, lower part of arms and
forearms (glove and stocking pattern).
It is more marked in asphyxia and is less marked in
death from hemorrhage (> 65% of blood loss in adults
and > 45% of blood loss in infants), anemia, lobar
pneumonia (coagulation of blood) and wasting
disease.

131. D. 8th month


Age related changes in fetus:
4th month: Sex is recognized, lanugo is visible
5th month: Vernix caseosa appears on the body
6th month: Hair appears on the head
7th month: Pupillary membrane disappears
8th month : Nails reach the tip of the fingers.
Testicles:
6th month : Lies close to kidneys (on psoas)
7th month : Left near the external inguinal ring,
Right near the internal inguinal ring.
8th month : Left in the scrotum, Right near the
external inguinal ring.
9th month: Both in the scrotum.

132. D. Terminal
Forensic ballistics: It is the science dealing with the
investigation of firearms, ammunition and the
problems arising from their use.
Proximal/Interior/Internal Ballistics: Physiochemical
phenomenon from the time of detonation till the
projectile leaves the barrel.
Exterior/External Ballistics: Study of the projective
from the time it leaves the barrel till it reaches the
target.
Terminal/Wound Ballistics: Study of effect of the
projectile on the target (Study of wound)

133. B. Arsenic poisoning


Conditions which retard/delays putrefaction are:
Wasting disease - Anaemia, severe haemorrhage
(decreased blood) & Debility (atrophy of tissue)
Poisons: Arsenic, antimony and other heavy metals,
carbolic acid, zinc chloride, strychnine.

134. B. NO2
The chemical processes during putrefaction liberate
ammonia, carbon monoxide, carbon dioxide, hydrogen
sulphide, phosphorated hydrogen, methane and
mercaptans.

135. C. Coagulation of protein


Pugilistic attitude is also known as Boxing attitude or
fencing posture or defence attitude.
It is due to heat stiffening, i.e., coagulation of muscle
proteins, which causes contraction.
Seen in both in ante-mortem and post-mortem burns.
It occurs whether the person was alive or not at the
time of burns.
Heat stiffening is due to exposure of the body to
temperature above 650C.
It is seen in those dying due to bums and sudden
immersion in boiling liquid.
In heat stiffening, body assumes a posture of
generalized flexion. The body is flexed at all the joints
except over the back.

136. A. DMSA
Only Oral: DMSA (Succimer)
Only Parenteral: BAL (I.M.), EDTA (I.V.)
Both Oral + Parenteral: DMPS (I.V.), Penicillamine (i.V.),
Desferrioxamine (I.M.)

137. D. Cessation of circulation and respiration


Following death, signs of death appear in the following
order:
1. Immediate (somatic death):
a. Insensibility and loss of voluntary power
b. Cessation of respiration
c. Cessation of circulation.
2. Early (cellular death):
a. Pallor and loss of elasticity of skin
b. Changes in the eye
c. Primary flaccidity of muscles.
d. Cooling of the body
e. Post-mortem lividity
f. Rigor mortis
3. Late (decomposition and decay):
a. Putrefaction
b. Adipocere formation
c. Mummification

138. A. Mechanical stimulation


Zasko’s phenomenon - Striking the lower third of the
quadriceps femoris muscle about 10 cm. above the
patella with a reflex hammer causes an upward
movement of the patella because of contraction of the
whole muscle. This can be seen up to 1-2 hours after
death.
It is not for Chemical, Electrical or Thermal
stimulation.

139. A. Ear drum


In case of air blast, the most common injury is
tympanic membrane rupture. Damage to cochlea is
more frequent. Lung is the second common organ to
be affected (Blast Lung). Liver is least common to
injure.
The injuries are more serious in Immersion (under
water) blast than air blast. Tympanic membrane is
ruptured in case of complete submersion. Otherwise
most common organ to be injured is the
gastrointestinal tract. Injury to lung is less common.
Solid blast refers to wave energy that spreads through
a rigid structure. Most common injuries are skeletal
fractures. Fracture of legs and vertebrae is more
common. GIT damage is more common than Lungs.

140. B. 24 hours
For cadaveric transplantation, cornea can be removed
from a dead body within 6 hours, skin in 24 hours,
bone in 48 hours and blood vessels in 72 hours.
Kidneys, heart, lungs, pancreas, intestine and liver
must be obtained soon after circulation has stopped
as they deteriorate rapidly.

141. C. Tertiary flaccidity


Soon after death, the body goes for a stage of primary
flaccidity or primary relaxation. During the stage,
death is only somatic, and it lasts for one to three
hours. All the sphincters relax and there may be
incontinence of urine, semen and faeces.
After the first stage of primary flaccidity the body goes
for a stage of rigidity. This is stage of stiffening and
shortening of muscles fibers, known as rigor mortis.
Individual cell death takes place at this stage.
With the onset of putrefaction muscles become soft
and flaccid due to breaking down of actomyosin, i.e.
the stage of secondary relaxation or secondary
flaccidity.
There is nothing like tertiary flaccidity.

142. C. Strychnine poisoning


The onset of rigor is early and duration is short in:
death from diseases causing great exhaustion
and wasting, e.g. cholera, typhoid, tuberculosis,
cancer, etc.,
violent death as cut-throat, firearm,
electrocution, lightening, heat stroke
death due to strychnine poisoning.
Rigor mortis is frequently absent in Septicaemia.
The rigor mortis sets faster in children, elderly,
malnourished individuals.
Causes delaying the onset of rigor mortis are - severe
haemorrhage, pneumonia, apoplexy, deaths from
asphyxia, nervous diseases causing paralysis of
muscles.

143. C. No effect on floatation of body


Deep inspiration above the water level has no effect
on the floatation of the dead body, because floatation
is due to the putrefactive gases.
Floatation of body in India occurs in about 12-18 hours
in summer and 18-36 hours in winter.

144. A. Childless wife nearing menopause


Pseudocyesis is also known as False or Spurious or
Phantom or Hysterical or Imaginary pregnancy.
In this condition, the woman believes or imagines that
she is pregnant when she is not pregnant.
It is purely psychological.
It is generally believed that false pregnancy is caused
by changes in the endocrine system of the body,
leading to the secretion of hormones that cause
physical changes similar to those during pregnancy.
It may be seen in a childless married lady nearing
menopause.
The woman has subjective symptoms of pregnancy in
form of enlarged abdomen, breast changes with
amenorrhea.
woman have imaginary thoughts that fetus is growing
inside her womb.
X-ray or sonography can confirm the diagnosis.

145. D. Rate of putrefaction


According to Casper’s dictum, if the environmental
factors are same, a daad body decomposes in air twice
as rapidly as in water and eight times as rapidly as in
earth.
That means the ratio between putrefaction changes
between Air:Water:Earth = 1:2:8

146. D. All of the above


Subdural hemorrhage is commonly seen in:
1. Chronic alcoholics (Due to cortical thinning the veins
become taut)
2. Old age (Due to cerebral atrophy the veins become
taut)
3. Children (Infantile whiplash syndrome).

147. C. 7 months
Explantion:
At 7th month of intra uterine life of fetus:
Crown-heel length: 35 cm, Crown –rump length : 23
cm, Foot length : 8 cm, Weight: 900-1200 gram.
Skin is dusky red, thick and fibrous and covered with
vernix.
The eyelids are separated.
Pupillary membrane disappeared.
Scalp hairs are about 1 cm long.
Nails thick and up to fingertip.
Meconium in descending colon.
Left testes near the external inguinal ring and right
near the internal inguinal ring.

148. A. Face
In air (body placed openly), the order of putrefaction
is:
Abdomen – Chest - Face & Neck - Lower limbs -
Upper limbs.
In water, the order of putrefaction is:
Face & Neck – Chest - Upper limbs – Abdomen -
Lower limbs

149. A. Ant bite marks


Antemortem abrasions can be confused with erosion
of the skin produced by ants, excoriation of skin by
excreta, pressure sores, drying of the skin etc.
Eczema is a condition where patches of skin become
inflamed, itchy, red, cracked, and rough. Blisters may
sometimes occur.
In case of Joule burn there is an appearance of an
areola or pale halo around the electric mark. It occurs
at the periphery of electric lesion in form of blanched
area. This halo occurs due to arteriolar spasm from
direct effect of current on vessel wall musculature.
In case of chemical burns there will be distinctive
stains, eschars – moist and soft, slough away readily.

150. D. Brown
A bruise heals by destruction and removal of the
extravasated blood.
At first: Red
Few hours to 3 days: Blue (deoxyHb)
4th Day: Bluish-black to brown (haemo¬siderin)
5 to 6 days: Greenish (haematoidin)
7 to 12 days: Yellow (bilirubin)
2 weeks: Normal

151. C. Aniline
Color changes in post mortem lividity:

Normal Bluish-purple

Cherry Red CO (COHb)


Bright Red Cyanide (OxyHb)

Bright Pink Hypothermia, Refrigerated body

Dark Brown Phosphorus, Phosphate

Chocolate/Copper Nitrites, Aniline, Potassium chlorate, Nitrobenzene,


/Reddish brown Potassium bichromate, acetanilide, Bromates

Deep blue Aniline (in some cases)

H2S (SulphHb)
Bluish Green

Black Opium

Bronze Cl. Perfringens

152. A. Diastatic
Separation of the sutures by blunt force impact can
occur only in young persons (before cranial sutural
closure). Once the sutures are closed completely,
diastatic (sutural) fracture will not occur.
Gutter Fracture is the name used to indicate a furrow
in the outer table of the skull, ordinarily the result of a
glancing blow by a missile from a rifled firearm. These
are frequently accompanied with comminuted
depressed fractures of the inner table of the skull.
Pond or Indented fractures may be seen in infants
where the skull is elastic and usually is produced by
forcible compression of the skull by obstetric forceps
or impact against some protruding flat object.
Fissured fractures may be seen around the periphery
of the dent.
Perforating fractures are nothing but wound of entry
and wound of exit, usually seen in case of bullet injury.

153. C. After delivery


LOCHIA: It is a vaginal discharge seen after delivery,
which is as a result of healing process of placental
site.
Lochia Rubra: First 4-5 days, bright red in colour.
Lochia Serosa: Next 4 days, serous and paler in
colour.
Lochia Alba: After 9-10 days, thicker, scantier
and white or yellowish white.
It disappears in 2-3 weeks.

154. A. Upper end of radius and ulna


The ossification centers fuse about 1 year earlier in
females than males.
Upper ends of radius and ulna fuse at the age of 16-17
years in males (In females it fuses at 15-16 years).
In males, lower end of Ulna fuses at 17-18 years, lower
end of Radius fuses at 18-19 Years, upper end of femur
fuses at 17-18 years and upper end of humerus fuses
at 18-19 years.

155. C. Absence of reasonable care


Professional negligence (Malpractice) is the absence of
reasonable care and skill (Act of Commission and/or
Act of Omission), or negligence of a doctor in the
treatment of a patient, which may result in injury or
death.
Just because a doctor diagnose wrongly because of
error of judgement and give wrong medicine, it may
not amount to negligence.
It is up to the doctor whether to accept or reject a
patient, but he should not reject on the grounds of
caste, colour, religion, nationality etc.

156. C. 4-7 days


Age of abrasions:
Fresh: Bright red.
12 to 24 hours: Lymph and blood dries up leaving a
bright red scab.
2 to 3 days: Reddish-brown scab.
4 to 7 days: Dark brown to Brownish-black scab.
Epithelium grows and covers defect under the scab.
After 7 days: Scab dries, shrinks and falls out.

157. D. Medico-legal reports


Exceptions to oral evidence are:
1) Dying declaration.
2) Expert opinion expressed in treatise.
3) Evidence of a doctor recorded in a lower court.
4) Evidence given by a witness in a previous judicial
proceeding.
5) Evidence of Mint officer or any officer of India
Security Press.
6) Reports of certain Govt. Scientific experts, such as
Chemical
examiner, Chief Inspector of Explosives, Director of
Fingerprint
bureau, Director of Central Forensic Science Laboratory,
serologist etc.
7) Public records
8) Hospital Records: Routine entries such as DOA, DOD,
Pulse, Temp. etc.
9) When the certificate or document is acceptable to
the counsels (lawyer of the accused) without cross-
examination etc.
Medico-legal reports are accepted in a Court of law -
only after cross examination, unless the opposite
lawyer is willing to accept without cross examination

158. C. ATP-ase
Sequence of enzymes/biochemical markers rising after
injury:
Tissue cathepsin (5-10 min)
Seretonin (10 min)
Histamines (20-30 min)
Esterase and ATPase (within 1 hour)
Aminopeptidase (2 hours)
Acidphosphatase (4 hours)
Alkaline phosphatase (6 hours)

159. B. Undertaker’s fracture


Fracture of C2-C3 is known as Hangman’s fracture.
Undertaker’s fracture is caused due to the head falling
backwards forcibly after death, which tears open one
of the intervertebral disc usually around C-6 and C-7.
Fracure of C1 is known as Jefferson’s fracture.

160. D. Carbon Monoxide poisoning


After death cornea loses its glistening appearance and
becomes opaque.
Glistening of cornea remains even after death in case
of CO and Cyanide poisoning.
Dimness of cornea before death occurs in Cholera,
Uraemia, Narcotic poisoning and wasting disease.

161. C. 3 weeks
Healing of a fracture (Age of a Fracture):
12-24 Hours: Clotting of hematoma
7 Days: Formation of new vessels, Fibroblasts are seen
10 Days: Fibroblasts lay down reticulin and collagen
10-14 Days: Hematoma get absorbed
2 Weeks: Callus formation
3 Weeks: Callus visible on X-ray
1 Month: Obliteration of periosteal callus gap
2 Months: Callus is formed into hard bone

162. B. Superfoetation
SUPERFECUNDATION: Super fecundation is the
fertilization of two or more ova from the same cycle by
sperm from separate acts of sexual intercourse.
Heteropaternal superfecundation occurs when two
different males father fraternal twins. In common
usage, the term superfecundation is often used
instead of heteropaternal superfecundation.
SUPERFOETATION: Superfetation means the
fertilization of an ovum from a subsequent ovulation
in a woman who is already pregnant. Two fetuses are
born either at the same time showing different stages
of development, or two fully developed fetuses are
born varying from 1-3 months.
FETUS PAPYRASEOUS (Fetus compressus): In twin
pregnancy, one fetus may grow and develop more
than other fetus. The second under-developed fetus
may get compressed and flattened. Such fetus is
termed as fetus papyraseous
FECUNDATION AB EXTRA: Fertilisation of ova by the
semen deposited on vulva or inner aspect of the thigh.

163. C. Transient amnesia and unconsciousness


Cerebral concussion is a state of temporary
unconsciousness immediately following head injury,
and is always followed by amnesia, and tends to
spontaneous recovery.
Pre-traumatic (retrograde) and post-traumatic
(antegrade) amnesia may be present.
Usually, the duration of retrograde amnesia is quite
brief, while the duration of posttraumatic amnesia is
more variable (lasting seconds to minutes), depending
upon the injury. In severe concussion, the muscles are
flaccid, pupils are dilated and non-reacting, pulse is
weak and slow and respiration is shallow.

164. B. Middle cranial fossa


In case of fractures of the base of the skull,
Fracture of the anterior cranial fossa clinically
manifested with bleeding from nose/mouth or/and
CSF rhinorrhoea, and Raccoon eye.
Fracture of the middle cranial fossa clinically
manifested as bleeding from the ear and Battle’s sign
(Reddish discoloration on mastoid region).
Fracture of the posterior cranial fossa clinically
manifested as boggy swelling over upper nape of the
neck.

165. D. 12 hours
Sudden death: As per WHO, death is said to be sudden
or unexpected when a person not known to have been
suffering from a dangerous disease, injury or
poisoning is found dead or dies within 24 hours after
the onset of terminal illness.

166. D. All of the above


Ring fracture is the fracture of the base of the skull,
typically involving the temporal bone, occipital bone,
sphenoid bone, and/or ethmoid bone or sometimes
only occipital bone.
Usually the fracture is seen 3 to 5 cm. away from the
foramen magnum.
Causes of ring fracture are:
1. Fall from height on feet
2. Fall from height on buttocks
3. Fall of heavy object on the vertex
4. Heavy blow on the vertex
5. Heavy blow on the chin.

167. D. Diffuse axonal injury


Diffuse axonal injury (DAI) is a clinical condition in
which there is diffuse injury of the axons with
immediate loss of consciousness and coma of more
than 6 hours.
In mild DAI, there is coma for 6-24 hours.
In moderate DAI, there is coma for more than 24
hours, but there are no signs of brainstem
dysfunction.
In severe DAI, there is coma of more than 24
hours with brainstem signs.
It occurs in vehicle accidents in 90% of cases and due
to falls and assaults in 10% of the cases.
Histologically, after 12 hours, the cut ends of the
axons look club shaped (sausage) and finally appear
as round balls known as retraction balls.
Cerebral concussion is also known as Mild traumatic
brain injury (MTBI), or Stunning or Commotio Cerebri.

168. B. Adolescents
Commotio cordis typically involves young (8-18 years;
average age is 15), predominantly male athletes in
whom a sudden, blunt, non-penetrating and
innocuous-appearing trauma to the anterior chest
results in cardiac arrest and sudden death from
ventricular fibrillation.

169. C. Beating on both ears


Tying Legs & thighs tightly with bamboo sticks is
known as Chepuwa.
Forced standing is known as Planton.
Telefono refers to beating on both ears.
Rolling a wooden/iron rod up & down the thighs is
known as Ghotna.

170. D. Glycine
Decomposition of bone:
Bones decompose after death in 3-10 years.
Less than 7 amino acids in bone mean time since death
is more than 100 years.
First Amino acid to disappear is Proline and Hydroxy-
Proline.
Last Amino acid to disappear is Glycine.

171. C. Causes fatal injuries


Wads are present only in shotgun cartridges. The wads
Separate the projectiles from the propellant.
Help to obturate the bore effectively.
The air cushion wad which is impregnated with grease
helps to lubricate the barrel
Prevents heat from the gun powder from fusing or
distorting the pellets.
Acts as piston
Prevents expanding gases from escaping.
Allows optimum pressure to develop.
The top wad/ overshoot wad is meant to keep the shots
in place.
The wads does not cause fatal injuries.

172. B. Dermal nitrate test


The Gunshot residues on the hand if the person who
fired a gun are derived from both primer and gun
powder (propellant).
Dermal Nitrate or Paraffin test or Diphenylamine test is
used to detect Propellant (Nitrous compound
detection).
The primer contain metal compounds which can be
detected by
Harrison and Gilroy Test
(Neutron Activation Analysis)
FAAS (Flameless Automic Absorption Spectrometry)
TMDT (Trace Metal Detection Test)
SEM-EDX (Scanning Electron Microscope-Energy
Dispersion X-Ray) – Best Method.

173. C. 27
A useful rule of thumb is that, the humerus is 20%, the
tibia 22%, the femur 27% and the spine 35% of the
individual’s height in life.

174. C. 340 - 360


Acid phosphatase activity of human semen per ml is
340 - 360 Bodansky units or 2500 – 3500 Angstorm
units.
pH of seminal fluid is 7.35 – 7.5.
Seminal stains on clothes can be identified by blue-
white colour in the dark on passing Ultraviolet light.

175. C. Equal in all respects


Mens rea: Guilty mind
Actus rea: Guilty act
Mutatis Mutandis: The necessary changes having been
made
Pari Passu: Equal in all respects
Ultra Vires: Beyond the power /authority/scope

176. B. 12 hours
In living persons, motile sperms are usually seen up to
6 hours and rarely 12 hours after ejaculation into the
vagina.

177. A. Ethanol

Typical Embalming Fluid Use Proportion

Formalin (40%) Preservative 1.5 L

Methanol Preservative 500mL

Phenol Germicide 50mL

Thymol Fungicide 5g

Gylcerine Wetting agent 600 mL

Sodium borate Buffer 600 g

Sodium citrate Anticoagulant 900 g

Sodium chloride Controls pH 800 g

Eosin (1%) Cosmetic 30 mL

Soluble wintergreen Perfume 90 mL

Water Vehicle Upto 10 L

178. B. Increased fluidity of blood


TRIAD OF ASPHYXIA
1. Cyanosis due to diminished oxygen tension.
2. Patechial haemorrhages due to increased venous
pressure.
3. Visceral congestion & edema due to increased
capillary permeability.

179. A. Infrared photography


Gunshot residue (GSR) on skin and blood stained cloth
can be visualized by Infra-red rays.
In the crime scene, if the blood is washed, the
suspected area is sprayed with Luminol (3-amino
pthalylhydrazide), which would make entire area glow
bluish in the dark.
Comparison microscope is used to examine and
compare the secondary characteristics on a bullet.
Ultraviolet light is used to demonstrate seminal stain
and also to identify latent tattoo marks.

180. C. Case is kept pending


Under Coroner’s Act 1871, previously it was held at
Kolkata (Calcutta) and Mumbai (Bombay). However it
was abolished in Kolkata way back and was
discontinued in April 1978 in Kolkata and on 29 July
1999 in Mumbai.
A Coroner was an Officer of the rank of First Class
Magistrate, appointed by State Government.
The Coroner may be a doctor or a lawyer or both.
Under Coroner’s Act, the Coroner was empowered to
inquire all unnatural or suspicious deaths and death
occurring in jail.
After examining a body he then decides whether an
autopsy is required and if necessary he holds an inquiry
and forward the body to government doctor for
postmortem examination.
He had also power to order for exhumation.
Coroner’s court is only court of inquiry into the cause of
death.
The Coroner examines witnesses on oath and records
their evidence.
After completion of an inquiry, the Coroner finds a
verdict as to the cause of death. If the coroner finds a
verdict of foul play, he issues warrant to the accused
concerned and then handed over the case to the
Metropolitan Magistrate concerned.
When the accused was not found, the Coroner returns
an open verdict (No conclusion).
Open verdict means an announcement of the
commission of crime without information regarding the
accused.
It is held in U.K., some states in America and some
other countries.

181. B. Chloral Hydrate


The chloral hydrate is given in food or drink to render a
person suddenly helpless for the purpose of robbery or
rape.
It’s action is so rapid under such conditions that it has
been given the name “knockout drops”.

182. C. Mentally ill


Insanity (lunacy or mental unsoundness or mental
derangement or mental disorder) is defined as a
disease of mind or the personality in which there is
derangement or impairment of mental or emotional
processes.
The Indian Lunacy Act 1912 defined lunatic as an idiot
or a person of unsound mind. However, The Mental
Health Act 1987 has replaced the Indian Lunacy Act
1912. The Mental Health Act 1987 uses the term
“mentally ill person” instead of lunatic.
The Mental Health Act defines mentally ill person as “a
person who is in need of treatment by reason of any
mental disorder other than mental retardation”.
Insanity is an old term used loosely to denote any
mental disorder or mental illness. The Indian Penal
Code employs the term unsoundness of mind while
referring to insanity.
It is better to use term mentally ill person (as defined
under Mental Health Act 1987) rather than using term
such as insane or lunatic.

183. B. 90°-100°
In males subpubic angle is V-shaped, has sharp angle
of 700 to 750.
In females, the subpubic angle is U-shaped, rounded,
broader angle, 900 to 1000.

184. B. Lead peroxide


The black powder used as a propellant in firearms
consists of
1. K Nitrate (75%): Supplies Oxygen
2. Sulphur (10%): Increases density and makes it more
readily ignitable
3. Charcoal (15%): Acts as Fuel
It does not contain lead peroxide.

185. B. 2003
On 19th September 1964 Ministry of health appointed
Mr. Shantilal Shah, Health Minister of Maharastra as
Chairman of the committee. In Dec 1966, the
committee submitted its recommendations and the
Bill was introduced in 1969.
The Bill was passed in Rajya sabha and Lok sabha on
2nd August 1971.
The Act was enforced in India except the state of
Jammu and Kashmir on 1st April 1972.
The medical termination of pregnancy rules 1975
stands repealed because medical termination of
pregnancy rules 2003 has come into force. It lays
down the experience and training for the registered
medical practitioners.
1. Any medical practitioner having experience of 3
years in obstetrics and gynaecology or who has
completed 6 months housemanship in obstetrics and
gynaecology or worked in a hospital in a department of
obstetrics and gynaecology for at least one year can
perform the MTP.
2. Any registered medical practitioner who has assisted
25 cases out of which five cases he has done
independently in a hospital approved by the
government can perform the MTP. The registered
medical practitioners who have assisted in 25 cases
can do only first trimester terminations.
3. Any medical practitioner having diploma or degree
in obstetrics and gynaecology can perform MTP

186. A. Tracer Bullet


In tracer bullet the rear portion of the bullet core is
removed and the space is filled with a mixture of
barium nitrate and powdered magnesium with
strontium nitrate which burns during the plight of a
bullet shedding red sparks. It leaves a visible mark or
‘trace’ while in flight.
In explosive bullet the bullet tip is grilled and a tiny
canister containing Lead azide is inserted. On impact
the bullet explodes.
In Incendiary bullet the tip of the bullet contains white
Phosphorous, used to cause fire in the target (e.g., to
ignite fuel tank).
Gyrojets are cartridges or miniature rockets driven by
solid fuel, which produces considerable heat and
smokeless gas on burning.

187. D. Nitrocellulose
Smokeless powder is a type of propellant. It may be
1. Single based: Contains only Nitrocellulose.
2. Double based: Contains Nitrocellulose +
Nitroglycerine
3. Triple based: Contains Nitrocellulose + Nitroglycerine
+
Nitroguanidine
These produce much less flame and smoke and more
completely burn than black powder.

188. C. Fouling
Smudging or blackening in case of firearm wounds is
due to the deposition of smoke and soot.
Peppering or powder stippling or tattooing is due to
the partially burnt or unburnt particles of gunpowder.
When the bullet comes out through the barrel, the
metal fragments are scraped off, and these small
fragments of metal from the bullet or the barrel
impact the skin surface, around the wound of entry,
the term ‘fouling’ is used.

189. C. USA
Capital punishment in the United Kingdom was used
from ancient times until the second half of the 20th
century. The last executions in the United Kingdom
were by hanging, and took place in 1964, prior to
capital punishment being abolished for murder (in
1965 in Great Britain and in 1973 in Northern Ireland).
Although unused, the death penalty remained a
legally defined punishment for certain offences such
as treason until it was completely abolished in 1998.
The use of capital punishment in Italy has been
banned since 1889, with the exception of the period
1926-1947, encompassing the rule of Fascism in Italy
and the early restoration of democracy. Before the
unification of Italy in 1860, capital punishment was
performed in almost all pre-unitarian states, except for
Tuscany, where it was historically abolished in 1786. It
is currently out of use as a result of the adoption of the
current constitution, and defunct as of 1 January 1948.
Capital punishment is a legal penalty in the United
States, currently used by 31 states, the federal
government, and the military. The United States is the
only Western country currently applying the death
penalty, one of 54 countries worldwide applying it.

190. D. 2 weeks
Frost bite occurs after exposure to extreme cold (less
than -2.5°C) temperature or dry cold or from direct
contact with cold object (below 0°C).
In frostbite, skin becomes hard and black in 2 weeks.
It is commonly seen in mountaineering and polar
expeditions.
Tissue injury results from freezing and
vasoconstriction.
It usually affects the distal aspects of extremities or
exposed parts of face such as nose, ears, chin and
cheeks.

191. C. 5-6 days


Changes in Umbilical cord:
2 hours- Blood clots in cut end.
24 hours- Vessel begins to close.
12-24 hours-Cord attached to child shrinks and
dries.
36-48 hours-Inflammatory ring at the base.
2-3 days-Mummifies.
5-6 days-Falls off and leaves an ulcer.
10-12 days-Cicatrizes
192. D. Death certificate
There are certain specific requirements about death
certificates.
If the doctor in attendance is convinced that death is
due to any natural cause-
a) He must issue a death certificate
b) He cannot charge any fees for the same
c) He cannot delay it even if his own professional
charges are not paid and
d) The certificate must state the immediate cause of
death like coronary thrombosis instead of a vague term,
such as heart failure or cardiac arrest and the
underlying cause due to coronary artery disease.

193. D. None of the above


Any defence for medical negligence is only for
registered medical practitioner who is authorized to
practice modern medicine.
A quack is not supposed to practice modern medicine;
hence the question of defence does not arise.

194. B. Compressed air pushed in front of the


current
According to Spencer there are 4 factors that result in
injury in a lightning flash.
1. Direct effect of high voltage current.
2. Burning by super-heated Air.
3. Effect of expanded and repelled air around the flash.
4. Compressed air pushed before the current
produced by Lightning.
Even individuals quite remote from the flash may
sustain injuries due to the effect of compressed air.
195. A. Epidemic Dropsy
The plant of Argemone Mexicana is known as Yellow
Poppy. The oil contains two alkaloids, sanguinarine
and dihydrosanguinarine which causes epidemic
dropsy.
In Lathyrus sativus (Kesari Dhal, the active neurotoxic
principle is Beta (N)-oxalyl amino alanine-BOAA. The
continuous use of sativus produces Neurolathyrism,
characterised by progressive spastic
paraplegia/quadriplegia; sphincters sensation and
mental faculties are preserved.
LSD give rise to ‘Flashback phenomenon’ and ‘Bad
Trip’.

196. B. Mercury
Poisons imparting color to urine
Green: Phenol, Cresol
Orange: Rifampicin, Phenothiazines, Santonin
Yellow: Dinitrophenol, Arsine
Pink: Aniline, Eosin, Mercury
Purple: Porphyrins
Brown: Nitric Acid
Brown to black: Thymol, Naphthalene
Red: Phenolphthalein, Mephensin
Blue: Methylene blue

197. C. Deep
Wilson classified burns into 3 types. They are:
Epidermal - Dupuytren’s 1st and 2nd degree.
These burns are very painful and heals without scar
formation.
Dermo-epidermal - Dupuytren’s 3rd and 4th degree
Pain and shock are greater than in first degrees burns
(Most painful).
Deep - Dupuytren’s 5th and 6th degree (Relatively
painless)

198. B. 20 Lakhs
District consumer disputes redressal forum can pass a
maximum compensation of Rupees 20 lakhs.
The State consumer disputes redressal forum can pass
a maximum compensation of Rupees one crore.
The National consumer dispute redressal forum can
pass a compensation of more than Rupees one crore.

199. A. Acro reaction


The Acro reaction test is applied for the identification
of electric marks by demonstrating metal particles on
the skin surfaces by simple colour reactions.
Current Pearls - Small balls of molten metal, derived
from the electrode, may be carried deep into the
tissues. It helps to identify the wound of entry.
Bone Pearls or Wax Drippings - Heat generated by the
current may melt the calcium phosphate, which is
seen radiologically as typical round density foci,
known as, Bone Pearls.

200. A. Prescription of overdose


Res ipsa loquitor means, the thing speaks for itself.
This doctrine is applied both in civil and criminal
negligence.
The legal and procedural effect of the Court’s
acceptance of res ipsa loquitar in a negligence action
is to shift the burden of proof from the plaintiff
(patient) to one of disproof by the defendant (Doctor).
This doctrine cannot be applied against several
defendants (i.e. doctors).
To prove Doctrine of Res ipsa loquitar, the following
conditions should be satisfied:
1. That in absence of negligence, the injury would not
have occurred.
2. Patient has not contributed to negligence.
3. Doctor had exclusive control of the circumstances or
injury
Producing instruments or treatment.
E.g.: Amputation of wrong digit or limb, Leaving a swab
or instrument in
abdomen of patient after operation, Prescription of
overdose etc.

201. B. 10 to 12 days
Total deprivation of water and food results death in
about 10 to 12 days.
Newborns may survive for 7 to 10 days without food
and water.
If food alone is withdrawn, death may occur in 6 to 8
weeks or even more.
Death usually occurs when about 70 to 90% of body
fat, and 20% of body protein are lost.
Death usually occurs when 40% of original body
weight is reached.

202. C. Drowning
Gettler’s chloride test is used to differentiate fresh
water drowning from sea water drowning.
Normal value of chloride is 600mg% in both
chambers.
In Fresh water drowning, chlorides get reduced by
50% in left ventricle, where as in sea water drowning
it get increased by 30-40%.
Chloride estimation is not of any help after 12 hours.
It is not helpful in Putrefaction, Patent with Foramen
ovale, drowning in Brackish Water.
203. D. Fracture dislocation of upper cervical
vertebra
Judicial hanging causes fracture-dislocation of cervical
vertebrae at the level of C2-C3 or C3-C4.
Bilateral fractures of either the pedicles or laminae of
the arch of the second, third or fourth cervical
vertebrae occur (Hangman’s fracture).
Rarely dislocation of atlanto-occipital joint or odontoid
process of axis may occur.
With proper Judicial hanging, there is a rupture of the
brainstem between the pons and medulla resulting in
instantaneous and irreversible loss of consciousness
(Due to destruction of reticular formation) and in
irreversible apnoea (Due to destruction of the region
of respiratory center).
Asphyxial signs are not seen in properly performed
judicial hanging.

204. D. Breslau’s second life test


Hydrostatic test/Breslau’s first life
test/Raygat’s test: This test is conducted to know
whether the child has breathed or not.
First, the ligature is tied on the bronchi and lungs are
separated.
Each lung is individually placed in water, then is cut
into 12-20 pieces and placed in water to see whether
they float or sink.
A small piece of liver may serve as control.
If they float, they are each squeezed between thumb
and index finger to remove the tidal air.
If all the pieces float, it indicates that the respiration
has taken place.
If all the pieces sink, it means the child has not
breathed.
If some pieces sink and some pieces float, it indicates
partial or feeble respiration.
Stomach-bowel test (Breslau’s second life test):
This is a good corroborative test.
It is based on the principle that when respiration is
established, there is a probability that air is swallowed
and may pass into the stomach and small intestine.
It is performed by placing double ligatures at each end
of the stomach and at different parts of the small
intestine.
The various parts are tested for floatation, and if
positive results are shown, they are punctured under
water, then the contained may be seen to escape to
the surface.
If no escape, the test is negative.

205. B. Insertion of bullet into stab wound


In contact shot, the muzzle blast and the negative
pressure in the barrel following discharge may suck
blood, hair, fragments of tissue and cloth fibers,
several cm. back into the barrel called “back spatter”.
Rayalaseema phenomenon: A bullet stuffed into the
stab wound of the victim to mislead the police.
Kennedy’s phenomenon: Surgical intervention of
gunshot wounds, so that proper evaluation of wounds
becomes difficult.
Souvenir bullet: A bullet which is left in the body for
long time. It may get surrounded by fibrous tissue. It
may cause chronic lead poisoning.

206. A. Ploucquet's Test


Plocquet’s Test:
The ratio between the weight of the lung and body is,
1:70 before respiration and 1:35 after respiration.
Fodere’s/Static Test:
Before respiration the weight of the lungs is 30-40g,
After respiration it will be 60-66g.
Wredin’s Test:
Replacement of gelatinous tissue in the middle ear by
air after respiration.

207. B. 5
The number of members in District and State
consumer disputes redressal forum is 3 and the
number of members in National consumer dispute
redressal forum is 5.
One Judge and other eminent citizens and amongst
them, one should be a female.

208. A. Monocyte
Karyotyping is the process of pairing and ordering all
the chromosomes of an organism, thus providing a
genome-wide snapshot of an individual’s
chromosomes.
Karyotypes are prepared using standardized staining
procedures that reveal characteristic structural
features for each chromosome. Clinical cytogeneticists
analyze human karyotypes to detect gross genetic
changes—anomalies involving several megabases or
more of DNA.
Karyotypes can reveal changes in chromosome
number associated with aneuploid conditions, such as
trisomy 21 (Down syndrome).
Careful analysis of karyotypes can also reveal more
subtle structural changes, such as chromosomal
deletions, duplications, translocations, or inversions. In
fact, as medical genetics becomes increasingly
integrated with clinical medicine, karyotypes are
becoming a source of diagnostic information for
specific birth defects, genetic disorders, and even
cancers.
For karyotyping, the cells need to be arrested in the
metaphase stage for separating the chromosomes. As
the monocytes do not divide, they cannot be used for
karyotyping.

209. C. Atelectasis
Fallacies of hydrostatic test:
1. The expanded lungs may sink in cases of diseases
such as acute edema, pneumonia, congenital syphilis,
etc. or atelectasis.
2. The unexpanded lungs may float due to emphysema,
putrefaction or artificial respiration.

210. A. Child fictitiously claimed by a woman


Suppositious child is a fictitious child, never born to
the woman claiming that it is belonging to her.
A woman may claim for blackmailing, claiming
property, breech of promise, marriage etc.

211. D. When fetal heart rate is 160 per min at


fifth and 120 per min at ninth month
Pre-natal diagnostic techniques can be used or
conducted when the person qualified to do so is
satisfied that any of the following conditions are
fulfilled, namely:
1. Age of the pregnant woman is above thirty-five
years.
2. The pregnant woman has undergone of two or more
spontaneous abortions or foetal loss.
3. The pregnant woman had been exposed to
potentially teratogenic agents such as drugs, radiation,
infection or chemicals.
4. The pregnant woman has a family history of mental
retardation or physical deformities such as spasticity or
any other genetic disease.
5. Any other condition as may be specified by the
Central Supervisory Board.

212. C. Positive HCG test


Positive HCG test is only probable sign of pregnancy
because this test can be positive even in cases of
diseases like chorionic carcinoma & hydatidiform mole
and up to two weeks after pregnancy.
The definite/positive signs of pregnancy are
1. Foetal parts and movements:
Active foetal movements are felt by placing the hands
on the abdomen from 16-18 weeks and may be seen by
5th month. Foetal parts such as head and limbs can be
distinctly felt after 24 weeks of gestation.
2. Fetal heart sounds:
FHS are heard between 18 and 20 weeks by
stethoscope/foetoscope. The fetal heart rate is
160/minute at 5th month and 120/minute at 9th
month.
Fetal heart sounds are not felt:
When the fetus is dead
When there is excessive quantity of liquor amnii
When abdominal wall is very fat
When examination is made before 18 weeks of
pregnancy
3. X-Ray:
At 15-16 weeks, fetal parts can be detected with
certainty, but occasionally parts are detected as early
as 10 weeks. The shadows on X-ray are:
Crescentic or annular shadow of the skull
Beaded appearance of vertebrae
Step ladder pattern of ribs and
Irregular linear shadows of limbs
4. Ultrasonography:
Gestational sac is seen as white ring by 6th week.
Distinct echos from the embryo within the gestational
ring by 7th week.
Foetal heart beat can be made out by 10th week.

213. D. Suits for adoption


Legitimacy:
The fact that any person was born during the
continuance of a valid marriage between his mother
and any man, or within 280 days after its dissolution,
the mother remaining unmarried, shall be conclusive
proof that he is the legitimate son of that man, unless
it can be shown that the parties to the marriage had
no access to each other at any time when he could
have been begotten.
Posthumous child:
Child born after death of a father in a legally married
couple.
Orphan child:
It is a child whose parents are dead.
Affiliation cases (Suits for adoption):
A woman may allege a particular man to be the father
of her illegitimate child and file a case in the court for
fixing the paternity.
As per Sec 125 Cr.P.C., a first class Magistrate can
sanction a monthly allowance of any sum depending
upon the circumstances of the case for the
maintenance of such child.
214. D. Cinderella syndrome
Battered baby syndrome is also known as:
Caffey’s syndrome
Non accidental injuries of childhood (NAI)
Inflicted Traumatic Brain Injury (ITBI)
Inflicted Childhood Neurotrauma (ICN)
A battered child is a child who has received repetitive
physical injuries as a result of non-accidental violence,
produced by a parent or guardian.
In addition, there may be deprivation of love, care,
affection and nutrition.
Usually seen in a child less than 3 years, but may be
seen up to 5 years of age.
Incidence is slightly more in males.
Unwanted child.
Young parents (20-30years) with low socio-economic
status.
Inconsistent history and findings of physical
examination (Mismatch).
Multiple bruises of different colors in same individual
(Different age).
Shaken Baby Syndrome (Infantile Whiplash Syndrome)
is a form of child abuse. Usually occur in children <2
years, but may be seen up to the age of 5 years.
Cinderella Syndrome: False accusation by adopted
children of being mistreated or neglected by their
adoptive mothers is known as Cinderella syndrome.

215. D. All of the above


BEFORE CERTIFYING THE BRAIN-STEM DEATH ARE:
The cause of irreversible brain-stem damage (either
from a period of hypoxia, trauma, illness or toxic
insult) producing non-responsive coma, must be
clearly established.
Following reversible causes must be excluded:
Intoxication
Depressant drugs
Muscle relaxants
Primary hypothermia
Hypovolemic shock
Metabolic or endocrinal disturbances.
The patient must be examined by a team of doctors at
least twice, with a reasonable gap of time in between
(say about 6 hours or so).
None of the doctors who participate in the diagnosis of
brain-stem death should have any interest in the
transplantation of an organ being removed from the
cadaver.
The structural and functional damage of brain-stem
may be diagnosed depending upon the following
observations:
Dilated fixed pupils, not responding to sharp
changes in intensity of incident light.
Absence of motor responses within the cranial
nerve distribution on painful stimulation.
Absence of corneal reflexes.
Absence of vestibulo-ocular reflexes.
Absence of gag reflex or reflex response to
bronchial stimulation by a suction-catheter
passed down the trachea.
Absence of spontaneous breathing (Apnoea
test).

216. B. Increase menstrual blood flow


Ecbolics are the drugs which increase the uterine
contractions.
Ergot is the most commonly used ecbolic.
Others include Pituitary extract, Quinine, Cotton
root bark, Diachylon (lead oleate), Gossypium
etc.
Emmenagogues act by increasing the menstrual
blood, flow and causing expulsion.
Sanguinarin, estrogens, senecio, Oil of savin,
Borax, Apiol,are chief emmenagogues.
Diaphoretics increase sweating.
The excessive sweating can be produced by
application ofheat (e.g.: Blankets, Hot water
bottles and administration ofhot beverages will
cause increased perspiration).
The best diaphoretic is Pilocarpine Nitrate, 5
mgsubcutaneously. The other diaphoretics are
alcohols,salicylates and Antipyretics
Emetics are the drugs which induce vomiting.

217. A. Heat stroke


In heat stroke (Heat hyperpyrexia), body
temperature increases above the level of 41°C. There
is absence of sweating. The term ‘Thermic fever’ or
‘Sunstroke’ is used when there has been direct
exposure to the sun. There is failure of cutaneous
blood circulation.
Heat cramps (miner’s / stoker’s/fireman’s
cramps) is caused by rapid dehydration of body
through the loss of water and salt in the sweat. Muscle
cramp is due to loss of Na+ ion. There is no increase in
body temperature.
Heat prostration (heat exhaustion/heat
syncope/heat collapse) follows exposure to
excessive heat. There is no increase in body
temperature. It is due to peripheral vascular collapse.
Death is due to cardiac failure.

218. A. Spark burn


Lightning stroke:
It is a flash of lightning is due to an electrical discharge
from a cloud to the earth.
Types of Burns
1. Arborescent burns (Filigree/Lichtenberg’s
flower/Keranographic burns or Feathering):
2. Linear Burns:
Irregular, linear, first degree burns may follow skin
creases, especially if damp from sweating.
Often found in the moist creases and folds of skin.
Vary from 3 to 30cm. or more in length and 0.3 to 2.5
cm. in width.
3. Surface Burns:
These are true burns and occur beneath metallic
objects worn or carried by the person which are fused
by the flash.

219. C. Prominent caput succedaneum


In Precipitate Labour, all 3 stages of labour are
merged together and the mother may deliver the baby
without her knowledge suddenly.
The baby may fall into the lavatory pan and die of
drowning or suffocation or it may fall on the ground
and may die of head injury.
In case of skull fracture usually it is fissured and
limited to parietal bones or may be pond fracture.
If occurs it is usually seen in multiparae with roomy
pelvis and small foetus.
Caput succedaneum and cephalohematoma are not
seen.
The umbilical cord if torn is torn near the fetal end.

220. D. Retinal injury


Berlin’s edema (commotio retinae) a common
condition caused by blunt injury to the eye. It is
characterized by decreased vision in the injured eye a
few hours after the injury. Commotio retinae is usually
self limiting and there is no treatment as such. It
usually resolves in 3–4 weeks without any
complications and sequelae.
When an impact is imparted to a mobile head, the site
of maximum cortical damage is most likely to be
underneath or at least on the same side as the impact.
This is so called, ‘coup lesion’.
When a moving head is suddenly decelerated as in
case of a fall, though there might be a coup lesion at
the site of the impact, there is usually cortical damage
on the opposite side of the brain—‘contrecoup lesion’.

221. A. 10 bore
Bore of a Shot gun: It is the number of lead balls of
equal size and shape which is made from one pound
of lead, each ball precisely fitting the barrel.
For example, 12 bore shotgun means, 12 balls of
equal size and shape have been prepared from one
pound of lead, each ball is precisely fitting the the
interior of the barrel. Greater the bore lesser the
lumen of the barrel.

222. B. 80%
The percentage of fetal haemoglobin at
5th month : 94%,
Birth : 80%,
3 months : 7-8%,
6 months :0

223. A. Electrolyte imbalance


Secondary Drowning (Post-immersion syn¬drome or
Near Drowning or Delayed Drowning) is not drowning
in true sense but a complication or sequelae of
submersion.
The submersion victim is resuscitated and survives for
24 hours, but develop complications like hypoxemia
resulting in brain damage, electrolyte imbalance,
pulmonary edema, hemoglobinuria, sepsis, metabolic
acidosis, chemical pneumonitis, cerebral edema,
cardiac arrhythmias etc.
The most common cause of death is electrolyte
imbalance.

224. D. Testamentary capacity


Curren’s rule states that “an accused person will not
be criminally responsible, if at the time of committing
the act, he did not have the capacity to regulate his
conduct to the requirements of law, as a result of
mental disease or defect”.
Vicarious liability is a legal doctrine that assigns
liability for an injury to a person who did not cause the
injury but who has a particular legal relationship to
the person who did act negligently. It is also referred
to as imputed negligence.
Testamentary capacity (Testament = will) is the
mental ability of a person to make a valid will.
To make a valid Will, it should have following
components:
He should be major.
The person should be of sound mind.
The person making Will should understand the
nature of Will.
The person should have knowledge of his
property that has to be disposed of.
The person should recognize the individuals who
have moral claims to heir his property.
McNaughten’s rule: An accused person is not
criminally responsible, if it is proved that, at the time
of committing the crime that he was suffering from
such a defect of reason from disease of mind that he
did not know the nature and quality of act he was
doing or if he did know this that he did not know that
what he was doing was wrong.

225. C. Saliva
Phadebas test and/or Starch-Iodine Test/Alpha-
Amylase (or α-Amylase) test are used to detect Saliva
and can be confirmed by the Presence of buccal
squamous cell.
Presence of Colostrum: By the detection of Trypsin
inhibitor.
Presence of Milk: By the detection of Casein and
Lactose.
Chemical tests to detect urine from the stains depend
on the presence of Urea and creatinine.

226. B. Distal aspect


In Explosive bullet, the bullet tip is grilled and a tiny
canister containing Lead azide (a primer) is inserted.
On impact the bullet explodes.

227. D. Scopolamine
Narcoanalysis is a procedure of investigation of mental
content of a person done after application of a light
general anaesthetic drugs.
This investigative technique is based on the principle
that at a point very close to unconsciousness, the
subject would be mentally incapable of resistance to
questioning and incapable of inventing falsehood that
he has used to conceal his guilt.
Drugs commonly used are
1. Thiopentone sodium (Na Pentothal)
2. Scopolamine hydrobromide
3. Sodium seconol
4. Sodium amytal
5. Benzodiazepines

228. A. Brain fingerprinting


Brain fingerprinting is invented by Lawrence Farwell.
It is a technique that uses electroencephalography
(EEG) to determine whether specific information is
stored in a subject’s brain by measuring electrical
brainwaves and recording a brain response known as
a P300-MERMER (memory and encoding related
multifaceted electroencephalographic response) in
response to words, phrases, or pictures that are
presented on a computer screen.

229. A. Hanging
Red stripes of bleeding into the outer layers of
intervertebral discs of lumbar vertebrae may be seen
in bodies suspended for a long time, known as
Simon’s hemorrhages.
It may be seen not only in case of hanging, but
whenever dead body is suspended for a long time.

230. C. Filicide
Feticide: Killing of a fetus at any time prior to birth.
Neonaticide: Killing of a child within 24 hours of its
birth (As per medical journals).
Filicide: Killing of a child by its own parents.
Infanticide: Deliberate killing of a child below the age
of 1 year.

231. C. CSF
80% of the human population are secretors and 20%
are non-secretors.
The secretors secrete ABO antigen in all their body
fluids except in CSF.

232. B. Acid dilution test


Presumptive tests for blood are:
Benzidine test,
Phenolphthalein test,
Leucomalachite green test,
Orthotolidine test and
Luminol spray test
Confirmatory tests for blood are:
Microscopic examination,
Teichmann’s Haemin crystal test,
Takayama’s haemochromogen crystal test,
Spectroscopic examination,
Electrophoresis and
Chromatography.
For grouping in old blood stains,:
Acid dilution test or Latte’s crust method
Precipitin test is to identify the species of the blood
stain/tissue.

233. D. Barberio's test


Marsh test is used to detect Arsenic.
Phenolphthalein test is used to detect blood.
Paraffin test is used to detect gun powder residues
from the hand.
Barberio’s test, Florence’s test, Acid phosphatase Test,
Zinc test, Fluorescence test, Microscopy &
Precipitation reaction, ELISA, Prostate specific antigen
(P 30) are used to detect semen.

234. C. Datura
The Datura fruits are spherical and have sharp spines
(thorn- apple).
They contain 0.2 to 1.4% of hyoscine (scopolamine),
hyoscyamine and traces of atropine.
The pollen derived from Datura can cause unilateral
mydriasis, known as Cornpicker’s pupil.

235. D. More than 5 years


According to the transplantation of Human organs act
1994, the punishment for removal of human organ
without authority is
Imprisonment for a term which may extend to
10 years and with fine which may extend to
Rs.20,00,000.
Removal of the name of the medical practitioner
from the register of the Council for a period of 3
years for the first offence and permanently for
the subsequent offence.
Punishment for commercial dealings in human
organs:
Imprisonment for a term which shall not be less
than 5 years but
which may extend to 10 years and shall be liable to fine
which shall not be less than Rs.20,00,000 but may
extend to Rs.1,00,00,000.

236. B. Kerosene poisoning


Contraindications for gastric lavage are:
All corrosives except Carbolic acid
Mechanical poisons
Esophageal varices
Haemorrhagic diathesis
Comatose patients
Impaired gag reflex
Volatile poisons
Marked hypothermia
Significant electrolyte imbalance
Severe heart disease
Advanced pregnancy

237. A. Ciguatera fish poisoning


Ciguatera fish poisoning: Caused by eating reef fish,
which produces ciguatoxin which alters the Na+
permeability giving rise to reversal of hot/cold
temperature perception.
Gymnothorax poisoning: By eating Moray eels, giving
rise to prominent abdominal breathing & absent
thoracic breathing.
Scombroid food poisoning: Caused by eating spoiled
fish. The histidine is converted into histamine giving
rise to Allergic Reaction.
Sweet clover disease: The coumarin present in the
sweet clover is converted into Vit. K antagonist
dicoumarol giving rise to haemorrhagic disease.

238. C. Obtain a court order and vaccinate him


Article 21 in The Constitution of India provides
Protection of life and personal liberty - No person shall
be deprived of his life or personal liberty except
according to procedure established by law.
Accordingly, as there is a high risk of him spreading
disease to his patients, better to vaccinate him after
taking an order from the court of law.
239. D. Acute renal failure
Crush syndrome occurs if a large bulk of muscles is
crushed, as in case of vehicular run-over injury or by
falling masonry.
Crushing of muscles can liberate haeme and
myoglobin which can enter the circulation and get
excreted through the kidneys.
This can lead to Acute Tubular Necrosis.
Death may occur in a week’s time.

240. B. Methylenedioxyamphetamine
Ecstacy: MDMA (Methy lenedioxy methamphetamine)
Love drug: MDA (Methy lenedioxy amphetamine)
Eve: MDEA (Methy lenedioxy ethamphetamine)
Date rape drugs: Gama Hydroxy Butyric Acid (GHBA),
Flunitrazepam
(Rohypnol), Alcohol, ketamine

241. D. Cyanide
Diagnostic odour of various poisons

Odour Drug/ Poison

Garlicky Arsenic, P, OP, Thallium, Selenium, Tellurium

Bitter almond Cyanide, Prussic Acid

Burnt rope Cannabis

Fishy/musty ZnP, AlP (Garlicky also)

Rotten egg H2S, Disulfiram, Carbon disulphide, NAC


Shoe polish Nitrobenzene

Acetone (Apple like) Chloroform, Ethanol, Isopropanol

Acrid (Pear like) Chloral hydrate, Paraldehyde (C > P)

Wintergreen Methyl salicylates

Moth balls Camphor, Naphthalene

Hospital odour Carbolic acid

Carrot/Mousy Hemlock (Conium)

Sewer Gas smell H2S

Kerosene Organophosphates

242. A. Liver
Arsenic is rapidly cleared from blood.
100mg of arsenic in 24 hours urine indicates toxicity.
In the early stages, arsenic is found in greatest
quantity in liver, followed by Kidneys and Spleen.
When acute arsenic poisoning is suspected, an X-ray
of the abdomen may reveal ingested arsenic, which is
radiopaque.
Urinary arsenic should be measured in 24-hour
specimens collected after 48 hours of abstinence from
seafood ingestion. Normal levels of total urinary
arsenic excretion are less than 50 μg/dl, excretion of
100 μg or more per day is indicative of poisoning.
Urine becomes positive within 6 hours of poisoning
and may continue to be positive for about a couple of
weeks.
The greatest concentration of arsenic is found in hair
and nails. Its deposition in hair may begin in 15 days
after administration.
It is found in the Muscles for days; in the bones, and in
the keratin tissues – Hair, Nails and Skin for years.

243. C. Tertiary blast injury


Blast injuries are divided into the following 4
categories:
1. Primary: Caused solely by the direct effect of blast
overpressure on tissue.
2. Secondary: Caused by flying objects that strike
people.
3. Tertiary: Caused by high-energy explosions.
Displacement of air by the explosion creates a blast
wind that can throw victims against solid objects;
4. Quaternary: All other injuries not included in the first
three classes. These include flash burns, crush injuries,
and respiratory injuries

244. A. Carotid artery


Nearer the vessels to heart, the better the results,
specially for drainage.
Most commonly used artery is common carotid
(Usually right).
Axillary and femoral arteries are also used.
The corresponding veins are used for drainage.

245. A. Dhatura
Drugs causing miosis are:
Barbiturates, Benzodiazepines, Carbamates,
Caffeine, Carbolic acid, Nicotine, Opiates and
Organophosphates
Drugs causing mydriasis are:
Alcohol, Amphetamines, Antihistamines,
Cocaine, CO, Cyanide, Datura, and Ephedrine

246. B. Lesser curvature and pylorus


The pathway of acids and alkalis in food filled stomach
starts along the lesser curvature of the stomach and
leads to the pylorus, which explains the location of
greater damage in the food filled stomach.
Stomach without food tend to have significant injury
in the lower half to two-thirds and may have sparing of
the fundus.

247. C. The member of child welfare committee


As per POCSO, once it has come to knowledge that a
crime has been committed against a child, the child
should be taken immediately for a medical
examination.
The examination must be conducted by a registered
medical practitioner at a government hospital or a
hospital run by a local authority, within 24 hours from
the time of receiving information about an offence.
This can be conducted, irrespective of whether an FIR
has been filed or not.
In the case of a girl child, the examination must be
conducted by a woman doctor.
The examination must be conducted in the presence
of a parent or any other person that the child trusts.
In case such a person is not present, the head of the
hospital should appoint a lady who can stand in.

248. C. 40-50%
0-10% - No appreciable symptoms.
10-20% - Breathlessness on exertion, mild headache.
20-30% - Throbbing headache, disturbed judgment,
defective memory and rapid fatigue.
30–40% - Cherry red coloration, severe headache,
nausea & vomiting.
40-50% - Symptoms may resemble alcoholic
intoxication.
50-60% - Produces syncope or coma with intermittent
convulsions.
60-70% - Increasing depth of coma with incontinence
of urine and faeces
70-80% - profund coma with depressedor absent
reflexes, a weak thread pulse, shallow and irregular
respirations and death.
Above 80% - Rapid death from respiratory arrest

249. C. Semicarpus anacardium


The juice of semecarpus anacardium (Marking nut
juice) is used to produce artificial bruises because it
simulate a bruise.
Madar juice (derived from calotropis) and juice from
plumbago is also sometimes used to produce artificial
bruises.
Abrus precatorius and ricinus communos contain
toxalbumen which is haemolytic in nature.
Capsicum (chilli) may cause a condition known as
Hunan hand (Burning hand).

250. A. Less than 150 microgram / litre


Diagnosis of chronic lead poisoning:
1. Complete blood count and peripheral smear
2. FEP (Free Erythrocute Protoporphyrin) and ZnP (Zinc
Protoporphyrin) levels (> 50 µgm/ 100 ml)
3. Blood lead level: Permissible blood level is 35
µgm/100ml (Chelation therapy be considered when a
child has a blood lead test result greater than or equal
to 45 µgm/100ml).
4. Urinary lead (>150µgm /litre) and Urinary ALA and
Coproporphyrins.
5. Punctate basophilic RBCs : > 200 cells/Cu mm.

251. D. Flowering tops


Cannabis is used in the following forms:
1. Ganja (Marijuana, Marihuana, Mary Jane, Pot, Weed,
Grass, Reefer, PURPLE HAZE): Prepared from the flower
tops of female plant (15-25% of THC)
2. Bhang: it is prepared from the dried leaves and fruit
shoots (Ieaves*). It is mildest and contains 15% of THC.
3. Majun: it is a sweet prepared with bhang.
4. Charas or hashish (Most potent): it is the resin
exuding for the leaves and stems of the plant. It
contains 25-40% of active principle.
Hashish oil is most potent of all cannabis preparations
and contains up to 60% of THC.

252. B. Immediately after death, the body may


lengthen by about 2-3 cm
In a dead body, soon after death due to primary
relaxation of the muscles, the body length may be
more by 2–2.5 cm.
Later, when rigor mortis develops, it may be
shortened.
With the passage of rigor mortis and onset of
putrefaction, the length may change due to secondary
relaxation.
The height varies in the various hours of the day,
being maximum in the morning and less by 1.5 to 2
cm in the evening due to reduction of elasticity of
intervertebral discs.

253. D. Increased BP
Signs and symptoms of Aconite (Sweet Poison/Meetha
Zehar) are:
Tingling and numbness in the mouth and later all
over the body
Salivation, dysphagia,
Profuse sweating, subnormal temperature,
Hypotension,
Initial tachycardia followed by bradycardia.
Alternate dilatation and contraction of pupil, called
HIPPUS.

254. D. Alkalis

Contraindications for NaCl Contraindications for rectified spirit

Mineral acids Phenol

Alkalis Kerosene

Corrosive sublimate Formic acid

Aconite Acetic acid

Formaldehyde

Paraldehyde

Chloroform

Ether

Chloral hydrate

Alcohol

Phosphorus
255. B. Belly scales are incomplete

Trait Poisonous Non-Poisonous

Belly scales Cover the entire Does not cover the entire
breadth breadth

Head Small Large


scales

Teeth 2 long fangs Multiple small teeth

Tail Compressed Not much compressed

Habits Nocturnal Not so

256. A. Heroin + Cocaine


Speed ball: Heroin + Cocaine
Hot shot: Heroin + Strychnine
Crack: Cocaine + Baking soda
Micky Finn: Alcohol + Chloral hydrate.

257. C. Alcohol
According to Widmark’s formula,
a=cpr
a – Weight of alcohol in g.
c – Blood alcohol concentration in mg/kg
p – Body weight in Kg
r - constant and its value is 0.68 for males and 0.55 for
female
For Urine analysis the formula is a=3/4 prq, q is
alcohol concentration in urine in mg/kg.

258. C. Osmium dye


The different dyes used are
Black: Indian ink, Carbon (soot)
Red: Cinnabar, Vermillion (Mercuric Sulfide)
Green: Chromic Acid
Blue: Indian ink, China ink, Indigo, Cobalt, Prussian
blue, Ultramarine
Gold/Platinum Chloride is used for corneal tattoo – Jet
black stain.

259. B. It is the method to conclusively fix the


paternity
Blood grouping helps to solve paternity problems, but
does not conclusively fix paternity.
DNA Profiling (Fingerprinting) provides absolute
certainty and is a fool proof method to conclusively fix
paternity.
Comparing the blood grouping helps to certain extent
in case of exchange of babies in maternity hospitals,
to match the fragmentary remains of skeleton and to
compare the blood stains on the weapons.

260. C. Hypothermia
Signs and symptoms at the site of the sting may
include: Pain, which can be intense, Numbness and
tingling in the area around the sting and slight
swelling in the area around the sting.
Victims having envenomed by a scorpion suffer a
variety of pathologies, such as irritability,
hyperthermia, vomiting, profuse salivation,
lacrimation, tremor, and convulsion, High blood
pressure (hypertension), Accelerated heart rate
(tachycardia) or irregular heart beat (arrhythmia).

261. A. Distended
In case of starvation, gall bladder is distended with
bile as the bile is not used.
The heart is small from brown atrophy, and chambers
are empty.
Brain is the only organ which does not show reduction
in size and weight.
Loss of adipose tissue from omentum, mesentery and
peri-renal fat stores.
Gut is empty and filled with gas. The wall is
contracted and translucent from stomach to colon.
The walls of the intestine may appear like Tissue
paper (considered to be the sure sign).
Faecoliths are usually present and may ulcerate the
intestinal linings.
Bones may show features of demineralization. Stress
fractures are present.
All the organs shrink except Brain.

262. D. Gas chromatography


The various tests for estimation of Alcohol
Macro Method :- Hine and Kozelka method (Color/Spot
Test)
Micro Method: Cavett’s Test
Chemical methods: Based on principle of Reduction of
Potassiumdichromate
Breath analyser test is based on Henry’s Gas law; the
residual alcohol in mouth takes about 20 minutes to
disappear and within this period breath analyser test
may be false positive
Gas Chromatography is the most desirable method for
medico legal purpose.
263. D. Zinc
Metal Fume Fever (Monday fever, Foundry fever, Brass
chills, Brazier disease, Smelter shakes) is caused by
the inhalation of fumes while melting metals.
All the metals may give rise to metal fume fever on
exposure to fumes but it is most common in case of
zinc.
The symptoms are fever, chills and malaise.

264. A. Skull
In case of healing of skull Fracture healing occurs
without the formation of visible callus.
The edges of fissured fracture stick together within a
week.
The edges are slightly eroded and the inner surface of
the skull may show pitting or deposition of lime salts
in 14 days.
The edges become slightly smooth and bands of
osseous tissue run across the fissure in 3 to 5 weeks.
If the edges are not in apposition, they become quite
smooth in 3 months.
If there is much loss of bone, the gap is filled only with
fibrous tissue.

265. D. All of the above


Psychiatric manifestations of chronic alcoholism are:
1. Delirium Tremens: It occurs in chronic alcoholics due
to
Acute intake of large bout of alcohol
Sudden withdrawal of alcohol
Shock from injury, e.g., Fracture of a bone
Acute infection, e.g., Pneumonia, Influenza etc.
Expossure to cold Etc.
2. Korsakoff’s Psychosis:
Confabulation, Hallucinations, Disorientation, Multiple
neuritis, Amnesia, Lack of insight.
3. Marchiafava Bignami Disease
(Demyelinization of nerves-corpus callosum)
Ataxia, disarthria, hallucinations, epilepsy
4. Wernicke’s encephalopathy:
Global confusion, Ophthalmoplegia, Ataxia, Peripheral
Neuropathy

266. D. Person who can tolerate high doses of


arsenic after taking arsenic in low doses at
frequent intervals
Arsenophagists are people who take arsenic daily as
tonic or as an aphrodisiac and they acquire a tolerance
of up to 300mg. or more in one dose.

267. A. Mecamylamine
Mecamylamine is also known as Inversine, is a specific
antidote given orally in case of nicotine overdose.
In mild to moderate poisoning Atropine and
Hexamethonium can be given to counteract peripheral
autonomic disturbances and as respiratory stimulant.

268. D. Colubridae

Sl. Family Toxicity Examples


No.

1 Elapidae Neurotoxic Common Cobra, Common Krait

2 Viperidae Haemotoxic Russell’s viper & Saw-scaled viper

3 Crotalidae Haemotoxic Rattle snake, Pit viper


4 Hydrophidae Myotoxic Sea Snakes

5 Colubridae Non toxic Bird Snake, Boom slang, Checkered


keelback

269. B. I.V. Injection


I.V. injection of mercury results in a condition called,
Mercurialism which is characterised by
Thrombophlebitis
Granuloma
Pulmonary embolism

270. D. Cardiomyopathy
Poisoning by cadmium causes golden yellow staining
of teeth, proteinuria and painful bone lesions known
as “Ouch-Ouch Disease.
Cobalt is added in the beer to give more foam, and
excessive drinking of beer may cause
Cardiomyopathy, known as Beer Drinker’s Syndrome.

271. B. Narcotine
Opium contains two chemically different groups of
alkaloids, namely:
1. Phenanthrenes (Narcotic)
Morphine 10%
Codeine 0.5%
Thebaine 0.3% (can cause convulsions)
2. Isoquinolines (Non-narcotic)
Papaverine 10%
Narcotine 6%

272. B. There is no antidote


Ethylene glycol is mainly used as an anti-freeze agent.
It is metabolised to glycoaldehyde, glycolic acid and
oxalic acid.
Treatment includes gastric lavage, calcium gluconate,
ethyl alcohol, 4-Methyl pyrazole and hemodialysis.

273. C. 317 IPC


Sec. 315 IPC - Act done with intent to prevent child
being born alive or to cause it to die after birth.—
Whoever before the birth of any child does any act
with the intention of thereby preventing that child
from being born alive or causing it to die after its
birth, and does by such act prevent that child from
being born alive, or causes it to die after its birth,
shall, if such act be not caused in good faith for the
purpose of saving the life of the mother, be punished
with imprisonment of either description for a term
which may extend to ten years, or with fine, or with
both.
Sec. 316 IPC - Causing death of quick unborn child
by act amounting to culpable homicide.—Whoever
does any act under such circumstances, that if he
thereby caused death he would be guilty of culpable
homicide, and does by such act cause the death of a
quick unborn child, shall be punished with
imprisonment of either description for a term which
may extend to ten years, and shall also be liable to
fine. Illustration A, knowing that he is likely to cause
the death of a pregnant woman, does an act which, if
it caused the death of the woman, would amount to
culpable homicide. The woman is injured, but does
not die; but the death of an unborn quick child with
which she is pregnant is thereby caused. A is guilty of
the offence defined in this section.
Sec. 317 IPC - Exposure and abandonment of
child under twelve years, by parent or person
having care of it.—Whoever being the father or
mother of a child under the age of twelve years, or
having the care of such child, shall expose or leave
such child in any place with the intention of wholly
abandoning such child, shall be punished with
imprisonment of either description for a term which
may extend to seven years, or with fine, or with both.
Explanation.—This section is not intended to prevent
the trial of the offender for murder or culpable
homicide, as the case may be, if the child dies in
consequence of the exposure.
Sec. 318 IPC - Concealment of birth by secret
disposal of dead body.—Whoever, by secretly
burying or otherwise disposing of the death body of a
child whether such child die before or after or during
its birth, intentionally conceals or endeavours to
conceal the birth of such child, shall be punished with
imprisonment of either description for a term which
may extend to two years, or with fine, or with both.

274. B. 20-50% of normal


Normal level of cholinesterase is 4,000 to 11,000 I.U.
Signs and symptoms start only when the ChE level
falls below 50% of normal.
Mild Poisoning: 20-50% of normal
Moderate Poisoning:10-20% of normal
Severe Poisoning: <10% of normal

275. D. Air embolism


Cause of death in burns:
Immediate to within few hours
Primary shock- Neurogenic shock - due to pain
Asphyxia – inhalation of smoke, suffocation Within first
48 hours
Secondary shock – loss of fluid from burnt region 3-4
days
Toxaemia – absorption of various metabolite from
burnt region 4-5 days and later
Sepsis, Gastric ulceration, Oedema of glottis, acute
renal failure, Gangrene, Pulmonary embolism, ARDS,
tetanus Years after
Malignant transformation of burn scar (Marjolin’s
ulcer)

276. C. Blood gets clotted after death


CO blocks Cytochrome – A3 Oxidase and P450
Death is due to Anemic anoxia/chemical asphyxia.
Blood remains fluid without clotting.
There will be bilateral symmetrical necrosis and
cavitation of the lenticular nuclei and punctiform
haemorrhages in the white matter of brain.

277. A. CO
Sewer gas contain Hydrogen sulphide, carbon dioxide
and Methane.
Amyl nitrite and Sodium nitrites are given as
antidotes.

278. A. DMPS
DMSA (Dimercapto-succinic acid) also known as
SUCCIMER is similar to BAL in chelating properties.
It is superior to EDTA in the treatment of Lead
poisoning.
It can be given in Glucose-6-PD deficiencies.
It is less nephrotoxic.
It is used against Lead, Mercury and Arsenic poisoning.
Dose is 10 mg/kg orally every 8 hours for 5 days
followed by the same dose every 12 hours for 14 days.
A combination of DMSA and EDTA is said to be more
effective.

279. B. Croton
A toxalbumen or phytotoxin is a toxic protein, which
resembles a bacterial toxin in action and causes
agglutination of red cells with some haemolysis and is
antigenic in nature.
The toxalbumen is present in Abrus precatorius
(Abrin), Ricinus communis (Castor plant – Ricin) and
Croton tiglium (Crotin).
Oil cannot be extracted from abrus.
The castor oil doesn’t contain ricin, only the press cake
contain ricin.
But in case of croton tiglium, both croton oil and press
cake contain crotin.
Oil from jetropa is used as bio-diesel.

280. B. Common krait


The identifying features of snakes are:
1. Common cobra:
Hood with spectacle mark.
3rd supralabial touches both eye and nostril
2. Common krait:
4th infralabial is the largest
Dorsal central row of hexagonal scales.
3. Russel’s viper:
Triangular head with narrow neck
3 rows of diamond shaped marks on the back

281. C. 70

W.H.O. Classification of Mental Retardation


Mild mental retardation 50 –70

Moderate mental retardation 35 – 49

Severe mental retardation 20 – 34

Profound mental retardation < 20

Mental Retardation is presently known as


iIntellectual disability (ID), also called intellectual
development disorder (IDD) or general learning
disability.

282. A. 1-c; 2-a; 3-d; 4-b


Brain mapping (fMRI) is discovered by Seiji Ogawa.
The primary form of fMRI (Functional magnetic
resonance imaging) uses the blood-oxygen-level
dependent (BOLD) contrast. It is based on uptake of
oxygen in the brain cells in response to questionnaire.
Dr. Kary Banks Mullis invented Polymerase Chain
Reaction (PCR).
Brain fingerprinting is invented by Lawrence Farwell. It
is a technique that uses electroencephalography
(EEG) to determine whether specific information is
stored in a subject’s brain by measuring electrical
brainwaves and recording a brain response known as
a P300-MERMER (memory and encoding related
multifaceted electroencephalographic response) in
response to words, phrases, or pictures that are
presented on a computer screen.
Dr. Alec Jeffrey in 1985 developed DNA fingerprinting.

283. B. Increase in the size of testis


Tanner’s sexual maturity rating
Genitals (male)
Tanner I
Testicular volume less than 1.5 ml;
Small penis of 3 cm or less (prepubertal) (typically age
9 and younger)
Tanner II
Testicular volume between 1.6 and 6 ml; skin on
scrotum thins, reddens and enlarges;
Penis length unchanged (9–11)
Tanner III
Testicular volume between 6 and 12 ml; scrotum
enlarges further;
Penis begins to lengthen to about 6 cm (11–12.5)
Tanner IV
Testicular volume between 12 and 20 ml; scrotum
enlarges further and darkens;
Penis increases in length to 10 cm (12.5–14)
Tanner V
Testicular volume greater than 20 ml; adult scrotum;
Penis of 15 cm in length (14+)
Pubic hair (both male and female)
Tanner I
No pubic hair at all (prepubertal) (typically age 10 and
younger)
Tanner II
Small amount of long, downy hair with slight
pigmentation at the base of the penis and scrotum
(males) or on the labia majora (females) (10–11.5)
Tanner III
Hair becomes more coarse and curly, and begins to
extend laterally (11.5–13)
Tanner IV
Adult–like hair quality, extending across pubis but
sparing medial thighs (13–15)
Tanner V
Hair extends to medial surface of the thighs (15+)
BREASTS (FEMALE)
Tanner I
No glandular tissue; areola follows the skin contours of
the chest (prepubertal) (typically age 10 and younger)
Tanner II
Breast bud forms, with small area of surrounding
glandular tissue; areola begins to widen (10–11.5)
Tanner III
Breast begins to become more elevated, and extends
beyond the borders of the areola, which continues to
widen but remains in contour with surrounding breast
(11.5–13)
Tanner IV
Increased breast size and elevation; areola and papilla
form a secondary mound projecting from the contour
of the surrounding breast (13–15)
Tanner V
Breast reaches final adult size; areola returns to
contour of the surrounding breast, with a projecting
central papilla. (15+)

284. C. Less than 2 year


Punishment for a doctor or medical person who
misuses the techniques for sex selection as per
PCPNDT Act is:
First offence:
3 years imprisonment and/or fine up to
Rs.10,000 and
Suspension of Registration by the Medical
Council for 5 years.
Subsequent offence:
5 years imprisonment and/or fine
Rs.50,000 and
Permanent removal of Registration by the
Medical Council.

285. A. 80% of cases


In case of Hanging, the ligature mark is situated
Above the level of thyroid cartilage in: 80% of cases
At the level of thyroid cartilage in: 15% of cases
Below the level thyroid cartilage in: 5% cases

286. B. Eye
Ectopic bruise is also known as percolated bruise or
migratory contusion.
Blood will track along the fascial planes or between
muscle layers which form the least resistance and may
appear where the tissue layers become superficial.
E.g.:
Blunt trauma to the forehead may lead to
gravitating of blood around the eye and cause
black eye.
Black eye caused by the fracture of anterior
cranial fossa.
Bruise in the neck in case of fracture of jaw
bone.
Bruise in the thigh due to fracture of pelvis.
Fracture of femur may cause bruise over outer
aspect of lower thigh.
A blow on the upper thigh may cause a bruise
above the knee.
Impact over the calf may give rise to bruise over
the ankle.

287. B. Nymphomania
Satyriasis: Uncontrollable or excessive sexual desire in
a man.
Nymphomania: Excessive sexual desire and drive in
females.
Hybristophilia: Sexual arousal is obtained by being
with a partner known to have committed an outrage,
cheating, lying, known infidelities or crime, such as
rape, murder, or armed robbery.
Coprolalia: Sexual excitement is obtained by using
obscene language.

288. C. Introduction of a cuffed endotracheal tube


before lavage
Gastric lavage is done by putting the head of the
patient at a lower level than his feet so that the mouth
is at a lower level than larynx, so that any fluid which
may leak out through the sides of the tube will not
trickle down into the larynx and trachea.
Left lateral is the best position, so that the tip of the
gastric contents in the greater curvature of the body
can be sucked out.
Introduction of a cuffed endotracheal tube before
lavage seals the glottis, so that it prevents the
aspiration.

289. D. Thin layer chromatography


Florence test:
The stain is extracted by 1 % hydrochloric acid and a
drop is placed on a glass slide.
A drop of Florence solution (Postassium iodine, iodine
and water) is allowed to run under the cover slip.
If semen is present, dark-brown crystals of choline
iodide appear immediately.
They are rhombic crystals resembling haemin.
A negative test means the fluid is not semen.
Barberio’s test:
A saturated aqueous or alcoholic solution of picric acid
when added to spermatic fluid, produces yellow
needle shaped rhombic crystals of spermine picrate.
In Barberio’s test, the presence of spermine in semen
is detected.
The Acid phosphatase test:
Acid phosphatase activity of human semen per ml is
340 - 360 Bodansky units or 2500 - 3500 Angstorm
units.
It is the most commonly used presumptive test for the
detection of seminal fluid.
The prostatic secretion element of seminal fluid
contains a very much higher percentage of acid
phosphatase.
Thin layer chromatography detects both choline
and spermine in seminal stain.

290. D. Discontinuous injection and drainage


Method of injection for Embalming are:
Continuous Injection and drainage: The arterial
injection is given continuously, against vein tube that
is kept open throughout injection. The embalming
time is much shortened. Venous drainage and tissue
saturation is poor. This method is least satisfactory.
Continuous injection with disrupted drainage: The
injection is made continuously with vein tube closed.
The blood in the veins build up a resistance for the
arterial flow which helps in the better diffusion of the
fluid. Thick blood is discharged when the drain tube is
opened. This method is better that continuous
discharge and drainage.
Alternate injection and drainage: The arterial fluid is
injected for sometimes with the drain tube closed. The
injection is stopped when the superficial veins swell,
and the drain tube is opened. When the flow of blood
from the drain tube stops, it is closed and the injection
started. This process is repeated several times till the
embalming is complete.
Discontinuous injection and drainage: This consists of
repeated arterial injection of small quantities at two
hour intervals. The total quantity of injection fluid is in
excess of ordinary injection done at a tie. The injection
is continued for three ir four times. The venous drain
tube which is kept closed is opened a little before and
kept open a little after starting another dose of
injection. This is the best method.

291. C. 20 weeks
The purview of MTP ACT 1971 is up to 20 weeks of
pregnancy.
If the period of pregnancy is below 12 weeks, it can be
terminated on the opinion of a single doctor.
If the period of pregnancy is between 12 and 20
weeks, two doctors must agree that there is an
indication.

292. C. 80 mg%
When the jerking movement of the eyeball is in the
direction of the gaze and independent of the position
of the head, it is known as alcohol gaze nystagmus
and appears at blood levels of 40 to 100 mg%
(Average 80 mg%).
30 mg%: Maximum Blood Alcohol Level permissible
for a driver while driving as per Indian Motor Vehicle
Act.
50 mg%: Reaction time is impaired.
150 mg% or 0 .15%: Critical level of alcohol.
400mg% and above: Coma, and Death.
It is said that consumption of 150-250ml of absolute
alcohol in 1 hour is fatal

293. B. Calcium gluconate


Potassium permanganate in a dilution of 1:5000 is
most commonly used solution for gastric lavage as an
oxidizing agent.
Calcium gluconate is an antidote for oxalic acid
Aluminium hydroxide is given as an antidote for
mineral acids.
In case of ingestion of cyanide, stomach wash is
preferably given with 5% sodium thiosulphate
solution.
Antidotes for cyanide are,
Amyl nitrite is used by inhalation.
Sodium nitrite is given I.V.
Sodium thiosulphate in 50% solution I.V. -
converts cyanide to non-toxic thiocyanates.
4-dimethylaminophenol
Alpha-ketoglutaric acid
Dicobalt edetate.
Para-Amino Propiophenone (PAPP)
Hydroxocobalamine (Vitamin BI2 precursor) etc.

294. D. H2S
Dangerous Japanese ‘Detergent Suicide’ Technique
(Detergent or chemical suicide) is a suicide technique
that mixes household chemicals to produce a deadly
hydrogen sulphide gas in cars, closets or other
enclosed spaces.
Mixing of these detergents and chemicals liberate
Hydrogen sulphide (minly) and other poisonous gases.

295. B. Hypothermia
Paradoxical undressing:
It is a term for a phenomenon frequently seen in cases
of lethal hypothermia.
Shortly before death, the person will remove all their
clothes, as if they were burning up, when in fact they
are freezing.
Because of this, people who have frozen to death are
often found naked and are misidentified as victims of
a violent crime.
The reason for this paradoxical behaviour seems to be
the effect of a cold-induced paralysis of the nerves in
the vessel walls, which leads to a vasodilatation,
giving a feeling of warmth.

296. D. No such limit


As per Indian Railway’s Act, Section 168 - Provision
with respect to commission of offence by the children
of acts endangering safety of person travelling on
railway.-
(1) If a person under the age of twelve years is guilty of
any of the offences under sections 150 to 154, the
court convicting him may require the father or
guardian of such person to execute, within such time as
the court may fix, a bond for such amount and for such
period as the court may direct for the good conduct of
such person.
(2) The amount of the bond, if forfeited, shall be
recoverable by the court as if it were a fine imposed by
itself.
(3) If a father or guardian fails to execute a bond under
sub- section (1) within the time fixed by the court, he
shall be punishable with fine which may extend to fifty
rupees.

297. A. Any part of living child comes out


A child is live born according to English law when,
after complete extrusion from the mother, irrespective
of the attachment or severance of the cord, it exhibits
some signs of life like crying, movements of limbs,
twitching of eye lids, respirations and pulsation.
In India, if any part of a living child had been brought
forth, though the child may not have breathed or been
completely born =, it constitutes live birth. The
evidences in respect of live-birth or circumstantial and
from post mortem examination.

298. B. Accused in Impotent


Section 497 IPC. Adultery.—Whoever has sexual
intercourse with a person who is and whom he knows
or has reason to believe to be the wife of another man,
without the consent or connivance of that man, such
sexual intercourse not amounting to the offence of
rape, is guilty of the offence of adultery, and shall be
punished with imprisonment of either description for a
term which may extend to five years, or with fine, or
with both. In such case the wife shall not be punishable
as an abettor.
As an impotent cannot have sexual intercourse, it can
be a defense for adultery.

299. C. 25% more


A smaller volume of distribution for ethanol in women,
a faster and a more variable absorption from the gut, a
lower activity of gastric ADH enzyme, a faster hepatic
clearance, a higher concentration of acetaldehyde and
poor solubility in body fat are physiological factors due
to which females of the same body weight will have a
higher (25% higher) blood alcohol concentration for the
same amount of drink, as their aqueous compartment
is smaller.
300. C. 4 days
Contraction of the umbilical arteries start in about 10
hours and are completely closed by third day.
The umbilical vein and ductus venous are closed on
the 4th day.
The ductus arteriosus closed by 10th day, and
Foramen ovale closes by 2nd or 3rd month.

You might also like